Nothing Special   »   [go: up one dir, main page]

MCQS

Download as pdf or txt
Download as pdf or txt
You are on page 1of 603

BEST MCQ BANKING, ECONOMY AND FINANCIAL AWARENESS

SEPTEMBER 2023

Q. Which bank has launched its ‘Nation First Transit Card’, a RuPay prepaid instrument under the National
Common Mobility Card (NCMC) that can be used nationwide?
A) Punjab National Bank B) State Bank of India
C) Indian Overseas Bank D) Central Bank of India
Answer : B
✓ State Bank of India has launched its ‘Nation First Transit Card’, a RuPay prepaid instrument under the
National Common Mobility Card (NCMC) that can be used nationwide.
✓ About Nation First Transit Card:
✓ It aims to facilitate a seamless and convenient customer commuting experience and ensure easy digital
ticketing fare payments in metro, buses, water ferries, parking, etc., through a single card.
✓ In addition, individuals can also use this card for making retail and e-commerce payments.
✓ It is powered by RuPay and National Common Mobility Card (NCMC) technology.
✓ Key Facts about National Common Mobility Card (NCMC):
✓ NCMC feature enables the Bank’s customers to use their Debit Cards as travel cards for travelling on
Metro Rail and Buses where this facility is enabled.
✓ The idea of NCMC was floated by the Nandan Nilekani committee set up by the Reserve Bank of India
(RBI).
✓ NCMC is an initiative of the Ministry of Housing and Urban Affairs in India designed to promote
cashless transactions and offer a unified payment platform for commuters.
✓ It was launched on March 4, 2019.
✓ It offers a unified contactless transport solution through the RuPay platform, developed by the National
Payments Corporation of India (NPCI).
✓ What is a RuPay Card?
✓ It is an Indian domestic card scheme conceived and launched by the National Payments Corporation of
India (NPCI).
✓ RuPay is an affordable card and can be issued as credit cards, debit cards, and prepaid cards.
✓ It works to enable electronic payment at all Indian banks and financial institutions.
SBI IN NEWS 2023
▪ SBI Card has unveiled the 'SimplySAVE Merchant SBI Card,' specially designed to cater to the short-
term credit requirements of micro, small, and medium enterprises (MSMEs).
▪ SBI has introduced the 'Nation First Transit Card, providing a convenient means for digital fare payments
across diverse transportation modes through a single card.
▪ State Bank of India has achieved a major milestone in digital currency by enabling Unified Payments
Interface (UPI) interoperability with its Central Bank Digital Currency, the Digital Rupee.
▪ State Bank of India (SBI) has introduced an innovative Customer Service Points (CSP) functionality
enabling Aadhaar-based enrolment for social security schemes.
▪ State Bank of India (SBI) successfully raised Rs 10,000 crore through its third infrastructure bond
issuance. The bonds have a tenor of 15 years and coupon rate of the infrastructure bonds is set at 7.54
per cent.
▪ Swaminathan Janakiraman named as new deputy governor of RBI. He will replace Mahesh Kumar Jain.
▪ Janakiraman is currently the Managing Director in State Bank of India (SBI).
▪ Government of India has specified various denominations for electoral bonds, ranging from Rs. 1,000 to
Rs. 1 crore. SBI is the only bank authorised to sell these bonds.

Follow us: Official Site, Telegram, Facebook, Instagram, Instamojo 228


▪ An electoral bond will be valid for 15 days from the date of issue.
▪ State Bank of India will set up a new trustee company as its wholly-owned subsidiary for managing the
Corporate Debt Market Development Fund (CDMDF).
▪ State Bank of India (SBI) has inaugurated 34 Transaction Banking Hubs across 21 district centres in the
country.
▪ State Bank of India (SBI) appointed Kameswar Rao Kodavanti as its Chief Financial Officer (CFO).
▪ State Bank of India (SBI), Bengaluru Circle, recently launched 'Project Kuber' to meet the banking needs
of its customers.
▪ State Bank of India (SBI), the country’s biggest lender, has raised USD 750 million through bonds to
fund offshore business growth.
▪ State Bank of India (SBI) launched a coffee table book titled "The Banker to Every Indian" and the book
celebrates 75 years of Indian independence and 200 years of SBI's history.
▪ SBI to launch new current accounts and savings accounts in FY24 to attract deposits.
▪ The bank intends to launch two new variants of current accounts, one with a balance of ₹50,000 and
another with a balance of ₹50 lakh. It also plans to introduce a new savings account called “Parivar”
(family) account.
▪ State Bank of India (SBI) announced the completion of a $1 billion syndicated social loan facility.
▪ The facility of $1 billion was arranged through MLABs, MUFG bank and Taipei Fubon Commercial
Bank Co.
▪ State Bank of India (SBI) has partnered with the NGO That’s Eco Foundation to donate ₹48 lakh for the
plantation of 32,000 tree saplings at Garden City University in Bengaluru.
▪ State Bank of India (SBI) has raised Rs 3,717 crore through its third Basel III compliant Additional Tier
1 bond issuance in the current financial year at coupon rate of 8.25 per cent..
▪ SBI reintroduces 400 days ‘Amrit Kalash’ retail term deposit scheme and provides an interest rate of
7.6% for senior citizens.
▪ State Bank of India (SBI) has launched its coffee table book named ‘The Banker to Every Indian’.
▪ Commercial Indo Bank LLC (CIBL) is a joint venture in Russia between SBI (60 per cent) and Canara
Bank (40 per cent).
▪ SBI announces completion of $1 billion Syndicated Social Loan Facility.
▪ Warehousing Development Regulatory Authority (WDRA) signed a memorandum (MoU) of
understanding with State Bank of India to help farmers in getting low interest rate loans.
▪ According to the Indian Banks’ Association (IBA)’s report titled “EASE Reforms Index for the first
quarter of FY23″ the State Bank of India(SBI) was adjudged as the Best Bank in adopting reforms
prescribed for Public Sector Banks(PSB).
▪ SBI on 5 December 2022 announced that its personal banking advances excluding high value home loans
have crossed the Rs 5 lakh-crore mark as on 30 November 2022.
▪ State Bank of India (SBI), India’s largest lender, approved raising Rs 10,000 crore through infrastructure
bonds during the financial year 2023 (FY23) including a greenshoe option of Rs 5,000 crore.
▪ STATE BANK OF INDIA (SBI )
▪ Founded : 1 July 1955
▪ Headquarters : Mumbai, Maharashtra
▪ Chairman : Dinesh Kumar Khara
▪ Imperial Bank of India was previous name of SBI
▪ The presidency Banks of Bengal, Bombay and Madras with merged in 1921 to Imperial Bank of India.
▪ Tagline of State Bank of India
(1) Pure Banking Nothing Else
(2) With you all the way
(3) A bank of the common man
(4) The banker to every Indian

Follow us: Official Site, Telegram, Facebook, Instagram, Instamojo 229


(5) The Nation banks on us.
▪ It is one of the largest and oldest banks in India.

Q. Name the ‘Conversational Payments on UPI’ platform, recently announced by RBI?


A) Hello! UPI B) Namaste! UPI
C) Welcome! UPI D) Say! UPI
Answer : A
✓ Reserve Bank of India (RBI) Governor, Shri Shaktikanta Das has announced the launch of a suite of new
product offerings built by the National Payments Corporation of India (NPCI). These products are aimed
at creating an inclusive, resilient, and sustainable digital payments ecosystem. T
✓ These new products are – Credit Line on UPI, UPI LITE X and Tap & Pay, Hello! UPI – Conversational
Payments on UPI, BillPay Connect- Conversational Bill Payments.
✓ Credit Line on UPI- The RBI Governor launched Credit Line on UPI, to expand access to credit,
promote financial inclusion and innovation. This new offering enables pre-sanctioned credit lines from
banks via UPI and will revolutionize customer access to credit, fostering a more streamlined and digital
banking ecosystem
Q. For the assessment year [AY] 2023-24, CBDT has reduced the time taken to process income tax returns
to how many days?
A) 41 days B) 52 days
C) 18 days D) 10 days
Answer : D
✓ Central Board of Direct Taxes (CBDT), in a press release, informed that the average time it takes to
process income tax returns after the taxpayer has verified them has been reduced to 10 days for
assessment year [AY] 2023-24.
✓ This is very less as compared to 82 days for AY 2019-20 and 16 days for AY 2022-23.
Q. Name the organisation which is not involved for bringing the facility of offline Central Bank Digital
Currency payment application on feature phones?
A) IDEMIA B) Dell
C) HMD Global D) Airtel Payments Bank
Answer : B
✓ IDEMIA has announced that it has collaborated with Airtel Payments Bank and Nokia’s parent company
HMD Global to introduce an offline Central Bank Digital Currency payment application on feature
phones. IDEMIA is a French identity security service provider.
✓ These three organizations will work together to expand the reach of offline Central Bank Digital Currency
(CBDC) payments in India.
✓ This will be the first attempt to bring CBDC payments on feature phones using an application interface.

Q. Who has topped the Fortune India Rich List for 2023?
A) Gautam Adani
B) Mistry family of Shapoorji Pallonji Group
C) Mukesh Ambani
D) Poonawalla family

Follow us: Official Site, Telegram, Facebook, Instagram, Instamojo 230


Answer : C
✓ Mukesh Ambani top the Fortune India Rich List for 2023
✓ First: Mukesh Ambani, owner of Reliance Industries has topped the Fortune India Rich List for 2023
with a wealth of Rs. 8,19,225 Crore (USD 99.70 billion). Last year he was at second position.
✓ Second: Gautam Adani, the chairman of Adani Group of companies is at second position in 2023 with
a wealth of Rs. 5,23,538 Crore (USD 63.71 billion). Last year he was at first position.
✓ Third: The Mistry family of Shapoorji Pallonji Group holds the third position with a wealth of $34.6
billion.
✓ Fourth: Poonawalla family, owners of vaccine maker Serum Institute is ranked fourth with a wealth of
$32.9 billion.
✓ Fifth: Radhakishan Damani, claims the fifth spot on the India Rich List with a wealth of $23.4 billion.
✓ The annual Fortune India-Waterfield Advisors list ranks 157 dollar billionaires in the country with a
cumulative wealth of ₹69.30 lakh crore. Top 10 account for 41.65% of total wealth.
Q. Which Scheme that was announced in the Union Budget 2023 includes skilled people who are engaged
in various traditional and skilled professions?
A) PM VIKAS (Pradhan Mantri Vishwakarma Kaushal Samman)
B) Revamped credit guarantee scheme for MSMEs
C) PM Garib Kalyan Ann Yojana
D) Mahila Samman Bachat Patra
Answer : A
✓ Name of the scheme : PM Vishwakarma Kaushal Samman Yojana
✓ Who announced : Finance Minister Nirmala Sitharaman
✓ when was it announced During Budget 2023-24
✓ When was it launched Prime Minister Narendra Modi's birthday on September 17, 2023
✓ Objective Providing training and funds to people of Vishwakarma community
✓ Beneficiary : Castes under Vishwakarma community
✓ PM Vishwakarma Yojana:
✓ It is a Central Sector Scheme with a financial outlay of Rs.13,000 crore.
✓ Time period: Five years (FY 2023-24 to FY 2027-28).
✓ Aim:
✓ To strengthen and nurture the Guru-Shishya parampara or family-based practice of traditional skills by
artisans and craftspeople working with their hands and tools.
✓ The scheme also aims at improving the quality, as well as the reach of products and services of artisans
and craftspeople and to ensure that the Vishwakarmas are integrated with the domestic and global value
chains.
✓ Under this scheme, the artisans and craftspeople will be provided recognition through PM Vishwakarma
certificate and ID card, Credit Support upto Rs.1 lakh (First Tranche) and Rs.2 lakh (Second Tranche)
with a concessional interest rate of 5%.
✓ The Scheme will further provide Skill Upgradation, Toolkit Incentive, Incentive for Digital Transactions
and Marketing Support.
✓ Under the scheme, there will be two types of skilling programmes -- Basic and Advanced and a stipend
of Rs 500 per day will also be provided to beneficiaries while undergoing skills training.
✓ The Vishwakarma Yojana aims to support traditional workers and craftsmen, including goldsmiths,
ironsmiths, washermen, hairdressers, and masons

Follow us: Official Site, Telegram, Facebook, Instagram, Instamojo 231


Q. Reserve Bank of India (RBI) introduced the Incremental Cash Reserve Ratio (I-CRR) for which of these
reasons?
A) To encourage banks to increase their lending to the public.
B) To absorb surplus liquidity resulting from the return of Rs 2000 currency notes.
C) To reduce the reserve requirement for banks.
D) To promote digital payment methods
Answer : B
✓ I-CRR was introduced as a temporary measure to manage the excess liquidity in the banking system,
which occurred due to the return of Rs 2000 currency notes.
✓ This reserve required banks to maintain 10% of the increase in their net demand and time liabilities
between May and July 2023 to absorb the surplus liquidity.
✓ The RBI has decided to discontinue the I-CRR in a phased manner by releasing impounded funds in
installments, as mentioned in the text, to ensure that the system liquidity is not subject to sudden shocks
and that money markets function smoothly.

Q. Urban Infrastructure Development Fund (UIDF) is executed by which institution?


A) National Housing Bank B) Reserve Bank of India
C) NITI Aayog D) Securities Exchange Board of India
Answer : A
✓ The National Housing Bank (NHB), in charge of executing the Urban Infrastructure Development Fund
(UIDF), had issued guidelines in July and asked states to submit their proposals by September.
✓ The fund makes low-cost loans available to 459 tier-2 cities with a population between 1 lakh and
9,99,999 and 580 tier-3 towns with a population between 50,000 and 99,999 as per the 2011 Census.
Q. Who has re-appointed as the Managing Director and Chief Executive Officer (MD and CEO) of the
ICICI Bank?
A) K. V. Kamath B) Chanda Kochhar
C) Sandeep Bakhshi D) Sumant Kathpalia
Answer : C
✓ Reserve Bank of India has approved the re-appointment of Sandeep Bakshi as ICICI Bank MD & CEO
|for a period of three more years and he will remain at the post till 3 October 2026.
✓ He was appointed as the banks's MD and CEO since October 15, 2018.

ICICI BANK IN NEWS 2023


▪ RBI approves reappointment of Sandeep Bakshi as ICICI Bank MD & CEO for three more years till
October 2026.
▪ ICICI Bank gets RBI nod to make I-Process Services as wholly-owned subsidiary.
▪ ICICI Bank offers Ecosystem Banking for Indian Startups.
▪ ICICI Bank and BNP Paribas have signed an initial pact to cater to the banking requirements of European
corporates operating in India and Indian companies in the European Union.
▪ Zomato Partners with ICICI Bank to Launch Zomato UPI, Streamlining Payments for Users
▪ Industrial Credit and Investment Corporation of India (ICICI) BANK
▪ Founded : 1994
▪ HQ : Mumbai, Maharashtra

Follow us: Official Site, Telegram, Facebook, Instagram, Instamojo 232


▪ MD & CEO : Sandeep Bakhshi
▪ Tagline : Hum Hai Na Khayal Apka.
Q. Which Company Develops Self-Branded Laptop & Micro PC 'SMAASH'?
A) RailTel B) HMT Limited
C) ITI Limited D) BEML Limited
Answer : C
✓ ITI Limited develops self-branded Laptop & Micro PC ‘SMAASH’.
✓ ITI Limited has developed its own branded Laptop and Micro PC named ‘SMAASH’ that matches
international quality and performance.
✓ ITI Limited is a Public Sector Undertaking under Department of Telecommunication, Ministry of
Communications, Government of India.
✓ ITI Limited has designed ‘SMAASH’ in association with Intel Corporation with which an MOU has
been signed for design and manufacturing.
✓ SMAASH PC comes in variants like i3, i5, i7 and so on. One of the solutions that ITI Limited offers is
the SMAASH PCs with solar solutions. The SMAASH PCs are highly compatible with solar solutions
since they take DC input
Q. Which bank has recently launched the UPI ATM facility at over 6,000 ATMs across the country?
A) Indian Overseas Bank B) Bank of Baroda
C) Yes Bank D) IDFC First Bank
Answer : B
✓ Bank of Baroda (BoB) has enabled the UPI ATM facility at over 6,000 ATMs across the country.
✓ It is the first public sector bank to launch UPI ATMs, in coordination with the National Payments
Corporation of India (NPCI) and powered by NCR Corporation.
✓ Customers as well as customers of all participating issuer banks using any UPI-enabled Mobile app can
withdraw cash from Bank of Baroda UPI ATMs without using their debit card.
✓ Using the Interoperable Cardless Cash Withdrawal (ICCW) technology, which facilitates cardless cash
withdrawals through ATMs, the UPI ATM enables seamless QR-based cash withdrawals, doing away
with the need to carry a Card to withdraw cash, it

BOB IN NEWS 2023


▪ Bank of Baroda (BoB) has enabled the UPI ATM facility at over 6,000 ATMs across the country.
▪ Debadatta Chand has assumed charge as the managing director and chief executive officer of Bank of
Baroda (BoB).
▪ BoB introduces the BoB LITE Savings Account, offering customers the flexibility of a lifetime no
minimum balance requirement.
▪ BoB Set to Reduce NSE Shareholding for at least ₹661 Crore.
▪ Bank of Baroda, a public sector bank, has announced its partnership with National E-Governance
Services Limited (NeSL), a government-backed information utility appointed by the Insolvency and
Bankruptcy Board of India, to launch the Electronic Bank Guarantee (BG) system.
▪ Bank of Baroda, a prominent public lender, has recently launched the Interoperable Cardless Cash
Withdrawal (ICCW) facility for its customers. This innovative service allows customers to withdraw
money from ATMs using the United Payment Interface (UPI), eliminating the need for a physical card.
▪ Bank of Baroda (BOB)
▪ Founded : 1908
▪ Founder : Sayajirao Gaekwad III
▪ Headquarters : Vadodara, Gujarat

Follow us: Official Site, Telegram, Facebook, Instagram, Instamojo 233


▪ MD & CEO : Debadatta Chand
▪ Tagline : India’s International Bank
Q. Which institution has operationalized the Urban Infrastructure Development Fund (UIDF)?
A) Rs 10000 crore B) Rs 40000 crore
C) Rs 30000 crore D) Rs 20000 crore
Answer : A
✓ Officials from the Union Housing and Urban Affairs Ministry recently said the first tranche of loans to
fund ongoing projects in tier-2 and tier-3 cities - under the Urban Infrastructure Development Fund
(UIDF) will likely be disbursed soon.
✓ About Urban Infrastructure Development Fund (UIDF):
✓ UIDF is established through the use of priority sector lending shortfall.
✓ Purpose: The Fund will be used by public agencies to create urban infrastructure in tier-2 and tier-3 cities.
✓ The focus will be on basic services like sewerage and Solid Waste Management, water supply and
sanitation, construction and improvement of drains/storm water drains, etc., and impact-oriented
projects will be prioritised.
✓ It is managed by the National Housing Bank.
✓ The initial corpus for this Fund is ₹10,000 crore.
✓ It is established on the lines of the Rural Infrastructure Development Fund (RIDF).
✓ States will be encouraged to leverage resources from the grants of the 15th Finance Commission, as well
as existing schemes, to adopt appropriate user charges while accessing the UIDF.
✓ It currently covers 459 tier-2 cities and 580 tier-3 cities.
✓ UIDF Loans:
✓ The interest rate on UIDF loans has been kept at Bank Rate minus 1.5 per cent.
✓ The loan (Principal) will be repayable in five equal annual instalments within seven years from the date
of the draw, including a moratorium period of two years.
✓ Interest will be payable on a quarterly basis.
✓ What are tier-2 and tier-3 cities?
✓ The NHB, the nodal agency for the implementation of the UIDF, defines tier-2 cities as those with a
population of 50,000 to less than a lakh and tier-3 cities as those between one lakh and less than a million
as per the 2011 census.
✓ Key Facts about Rural Infrastructure Development Fund (RIDF):
✓ The RIDF was set up by the Government in 1995-96 to finance ongoing rural Infrastructure projects.
✓ The Fund is maintained by the National Bank for Agriculture and Rural Development (NABARD).
✓ Contribution: Domestic commercial banks contribute to the Fund to the extent of their shortfall in
stipulated priority sector lending to agriculture.
✓ Main Objective: To provide loans to State Governments and State-owned corporations to enable them
to complete ongoing rural infrastructure projects.
✓ Repayment period: Loan to be repaid in equal annual instalments within seven years from the date of
withdrawal, including a grace period of two years.

Q. What is the name given to India's first AI-powered anti-drone system?


A) Prakhar B) Chetak
C) Indrajal D) Gaganyaan
Answer : C
✓ India unveils the world's first AI-powered anti-drone system – Indrajal by Grene Robotics.
✓ Hyderabad-based robotics company Grene Robotics developed Indrajal, an innovative autonomous anti-
drone system powered by artificial intelligence (AI).

Follow us: Official Site, Telegram, Facebook, Instagram, Instamojo 234


✓ The Indrajal system is the first of its kind in India and is designed to protect critical facilities such as
nuclear installations and oil rigs, as well as large areas, potentially entire cities, from various drone
threats.
✓ It offers an impressive coverage range of up to 4,000 square km per unit, providing comprehensive
protection against a variety of drones.

Q. The Viability Gap Funding for development of BESS aims to generate 4,000 MWh of BESS projects by?
A) 2024-25 B) 2030-31
C) 2035-36 D) 2040-41
Answer : B
✓ The Union Cabinet has approved the Scheme for Viability Gap Funding (VGF) aimed at fostering the
development of Battery Energy Storage Systems (BESS).
✓ This scheme aspires to generate 4,000 MWh of BESS projects by 2030-31, backed by up to 40% capital
cost support in the form of VGF.
✓ the scheme seeks to provide clean, reliable, and cost-effective electricity to citizens.
✓ With an initial outlay of Rs. 9,400 crores, it aims to achieve a Levelized Cost of Storage (LCoS) ranging
from Rs. 5.50-6.60 per kWh
Q. India aims to become a global drone hub by which year?
A) 2035 B) 2025
C) 2030 D) 2027
Answer : C
✓ The Indian Air Force (IAF) has partnered with the Drone Federation of India (DFI) to co-host the ‘Bharat
Drone Shakti 2023’.
✓ The event is scheduled to be held on September 25 and 26, 2023 at the lAF’s airbase in Hindan,
Ghaziabad.
✓ The ‘Bharat Drone Shakti 2023’ will showcase the prowess of the Indian drone industry in using these
unmanned platforms.
✓ DFI shall host over 50+ live aerial demonstrations for survey drones, agriculture drones, fire suppression
drones, tactical surveillance drones, heavy-lift logistics drones, loitering munition systems, drone
swarms, and counter-drone solutions.
✓ Around 5,000 attendee are expected to witness the event including, 75 drone start-ups and corporates.
✓ This ‘Bharat Drone Shakti 2023’ event will help the country to achieve its commitment of becoming a
global drone hub by 2030.
TARGET YEAR IN NEWS 2023
▪ This ‘Bharat Drone Shakti 2023’ event will help the country to achieve its commitment of becoming a
global drone hub by 2030.
▪ Vaccine manufacturer Indian Immunologicals Limited (IIL) expects to commercially launch its dengue
fever vaccine by early 2026.
▪ Group Chief Economic Advisor at the State Bank of India (SBI), Soumya Kanti Ghoshon has reiterated
that India is likely to become the third-largest economy by 2027.
▪ The Ministry of Coal has set a target to gasify 100 million tonnes of coal by FY 2030.
▪ According to the report of Goldman Sachs Research, by the year 2075, India will become the second
largest economy in the world after China leaving behind not only Japan and Germany but also America.
▪ UNEP suggests measures to reduce 80% of world's plastic pollution by 2040.
▪ India aims to become one trillion dollar tourism economy by 2047.

Follow us: Official Site, Telegram, Facebook, Instagram, Instamojo 235


▪ The government announced a plan to add 250 GW of renewable energy capacity in the next five years
to achieve its target of 500 GW of clean energy by 2030.
▪ India has set targets for half of its installed electricity capacity to be from non-fossil fuel sources by 2030
and for the country to attain net-zero carbon emissions by 2070.
▪ 2023 foreign trade policy has the vision to take India’s goods and services exports to $2 trillion by 2030.
▪ India to get 9% of Electricity from Nuclear Sources by 2047.
▪ India sets target to become ‘Global Hub for Green Ship’ building by 2030
▪ Indian Railways to become Net Zero Carbon Emitter by 2030.
▪ India to produce 5 million tonnes of green hydrogen by 2030.
▪ India has set a target to replace the use of diesel by renewable energy in the agriculture sector by 2024.
▪ Indian Government has set a target to eliminate TB in India by 2025.
▪ India targets ₹25,000-cr worth defence exports by 2024.
▪ Reserve Bank of India’s 2023 monetary policy objective is to hold inflation within the mandated tolerance
band and guide it towards the medium-term target of 4% by 2024.
Q. What is the total outlay for the Scheme for Viability Gap Funding (VGF) for the development of Battery
Energy Storage Systems (BESS)?
A) Rs.7,400 crore B) Rs.3,400 crore
C) Rs.4,400 crore D) Rs.9,400 crore
Answer : D
✓ The Union Cabinet, chaired by the Hon’ble Prime Minister has approved the Scheme for Viability Gap
Funding (VGF) for the development of Battery Energy Storage Systems (BESS). BESS are devices that
enable energy from renewables, like solar and wind, to be stored and then released when the power is
needed most.
✓ Under the scheme, 4,000 MWh of BESS projects will be developed by 2030-31.
✓ Further, the project will provide financial support of up to 40% of the capital cost as budgetary support
in the form of Viability Gap Funding (VGF).
✓ Outlay of the scheme: The VGF for the development of the BESS Scheme has an initial outlay of Rs.9,400
crore. This includes a budgetary support of Rs.3,760 crore.
✓ To ensure that the benefits of the scheme reach the consumers, a minimum of 85% of the BESS project
capacity will be made available to Distribution Companies (Discoms).
Q. Paytm has launched Card Soundbox to allow merchants to accept card payments of up to ₹___
A) 5000 B) 3000
C) 7000 D) 4000
Answer : A
✓ Paytm unveils ‘card soundbox’ to empower merchants
✓ Fintech firm Paytm has unveiled ‘Card Soundbox’, an innovative device that will enable merchants to
accept both mobile and card payments across networks.
✓ The Paytm Card Soundbox is India’s first Soundbox that will also accept contactless card payments.
✓ The device is equipped with a built-in ‘tap and pay’ functionality, which will allow merchants to accept
card payments of up to ₹5,000.
✓ Soundbox is a device that provides instant audio alerts on amounts paid to merchants through an inbuilt
speaker.
✓ Paytm
✓ Founded : 2010
✓ Founder : Vijay Shekhar Sharma
✓ Headquarters : Noida, Uttar Pradesh

Follow us: Official Site, Telegram, Facebook, Instagram, Instamojo 236


✓ CEO : Vijay Shekhar Sharma
Q. IREDA has inked MoU with which banks to co-finance renewable energy projects?
A) DBS Bank and Kotak Mahindra Bank
B) Canara Bank and Bank of India
C) State Bank of India and Punjab National Bank
D) Union Bank of India and Bank of Baroda
Answer : D
✓ IREDA signs MoUs with Union Bank of India and Bank of Baroda to co-finance Renewable Energy
projects
✓ Indian Renewable Energy Development Agency Ltd. (IREDA) has signed Memorandums of
Understanding (MoUs) with Union Bank of India (UBI) and Bank of Baroda (BoB), on September 5,
2023.
✓ Under this, IREDA will collaborate with Union Bank of India (UBI) and Bank of Baroda (BoB), in co-
lending and loan syndication for a wide range of renewable energy projects, including both established
and emerging RE technologies.
✓ CMD of IREDA- Shri Pradip Kumar Das.
Q. As per SEBI’s latest decisions, which of these trade settlement time will be implemented by March 2024?
A) 1 day B) 2 hour
C) 1 hour D) 2 day
Answer : C
✓ Sebi plans to implement one-hour settlement of trades by March 2024
✓ The Securities and Exchange Board of India (Sebi) is planning to implement one-hour trade settlement
by March 2024 as announced by SEBI chairperson Madhabi Puri Buch.
✓ In January 2023, India moved to T+1 settlement (trade plus one day) against T+2 settlement earlier.
✓ Further SEBI plans to bring into effect real-time settlement by the second half of the financial year FY25.

Q. Who has been appointed as the new Chairperson of the technology industry body NASSCOM?
A) Sangeeta Gupta B) Debjani Ghosh
C) Kritika Murugesan D) Rajesh Nambiar
Answer : D
✓ MD Rajesh Nambiar appointed as the new chairperson of Nasscom
✓ Technology industry body, National Association of Software and Services Companies (Nasscom) has
appointed Rajesh Nambiar as its chairperson. He is the Chairman and Managing Director of Cognizant
India.
✓ Nasscom is the premier trade body and chamber of commerce of the tech industry in India and consists
of over 3,000 member companies
Q. The Government of India has approved the merger of Tata SIA Airlines (Vistara) with which of these
airlines company in India?
A) Go First B) Spice Jet
C) Jet Airways D) Air India
Answer : D

Follow us: Official Site, Telegram, Facebook, Instagram, Instamojo 237


✓ The Competition of Commission of India (CCI) has approved the merger of Tata SIA Airlines (Vistara)
into Air India, and the acquisition of certain shareholding by Singapore Airlines in Air India.
✓ Tata SIA Airlines (TSAL )which operates under the brand name “Vistara”, is a joint venture between
TSPL (an investment holding company) and Singapore Airlines Limited (SIA). TSPL and SIA holding
51% and 49% of the total shareholding, respectively.
✓ The proposed combination envisages
a) the merger of Tata SIA Airlines Limited (TSAL/Vistara) into Air India Limited (AIL/Air India), with
AIL being the surviving entity (Merged Entity) and
b) in consideration of the merger, the acquisition of shares in the Merged Entity by Singapore Airlines
Limited (SIA) and Tata Sons Private Limited (TSPL) and
c) acquisition of additional shares in the Merged Entity by SIA pursuant to a preferential allotment.
Q. Recently, Shri Shaktikanta Das has earned an “A+” grade in the Global Finance Central Banker Report
Cards 2023. Total how many governors have earned this A+ grade in 2023?
A) 2 B) 3
C) 4 D) 5
Answer : B
✓ Shri Shaktikanta Das, Governor of the Reserve Bank of India, has earned an “A+” grade in the Global
Finance Central Banker Report Cards 2023.
✓ The Bank Governors who earned an “A+” grade in the Global Finance Central Banker Report Cards
2023 are:
✓ Shri Shaktikanta Das, India (Reserve Bank of India)
✓ Thomas J. Jordan, Switzerland (Swiss National Bank)
✓ Nguyen Thi Hong, Vietnam (State Bank of Vietnam)
✓ The Central Banker Report Cards is published annually by Global Finance since 1994 and it grades the
central bank governors of 101 key countries. Grades are based on a scale from A to F for success in
inflation control, economic growth goals, currency stability and interest rate management.
✓ Reserve Bank Governor Shaktikanta Das has been honoured with the 'Governor of the Year 2023 Award'
by the international publication Central Banking.
✓ Shaktikanta Das has become the second governor to get this award after Raghuram Rajan, who was
awarded back in 2015.
✓ Shaktikanta Das is serving as the current & 25th governor of the Reserve Bank of India (RBI). He was
earlier a member of the Fifteenth Finance Commission and India's Sherpa to the G20.
✓ With the second term of three years, Das will head the RBI till December, 2024
Q. As per the data released by CBDT, total advance pricing agreements (APA) and _Bilateral Advance
Pricing Agreements (BAPAs) were inked 2022-23.
A) 95; 32 B) 98; 54
C) 91; 45 D) 97; 39
Answer : A
✓ CBDT signs 95 APAs and 32 BAPAs in 2022-23, highest ever since launch in 2012
✓ CBDT (Central Board of Direct Taxes) has signed 95 advance pricing agreements (APA) in 2022-23, the
highest ever in any financial year since its launch in 2012.
✓ CBDT also signed 32 Bilateral Advance Pricing Agreements (BAPAs) in FY 2022-23 being the maximum
number of BAPAs in any financial year till date.
✓ The Advance Pricing Agreement (APA) programme of CBDT is one of its foremost initiatives for
promoting an investor-friendly and non-adversarial tax regime in India.

Follow us: Official Site, Telegram, Facebook, Instagram, Instamojo 238


✓ Central Board of Direct Taxes (CBDT) has raised the tax exemption on leave encashment on retirement
of non-government salaried employees from ₹3 lakh to ₹25 lakh.
✓ Central Board of Direct Taxes (CBDT)
✓ Founded : 1860
✓ Headquarteres : New Delhi
✓ Chairman : Nitin Gupta
✓ Minister responsible : Ministry of Finance

Q. Which company has recently launched India’s first-ever UPI-ATM with NPCI?
A) IDFC FIRST Bank B) Hitachi Payment Services
C) Pine Labs D) Fiserv
Answer : B
✓ Hitachi Payment Services has announced the launch of India’s first-ever UPI-ATM named Hitachi
Money Spot UPI ATM in partnership with the National Payments Corporation of India (NPCI). It is a
White Label ATM (WLA)
✓ A UPI-ATM allows users to withdraw cash from multiple accounts using the United Payments Interface
(UPI) app. This ATM will also allow users the option for cardless cash withdrawals, eliminating the need
for physical cards.
✓ Hitachi Payment Services is a subsidiary of Japan-based Hitachi Ltd, on Tuesday. Hitachi Payment
Services is the only White Label ATM operator to offer a cash deposit facility
INDIA’S FIRST ATM IN NEWS
▪ India's first UPI ATM has been launched by Hitachi Payment Services, a subsidiary of Hitachi Limited.
▪ Hyderabad gets India’s first Gold ATM.
▪ India's first Aadhaar enabled ATM launched by DCB Bank.
▪ HSBC is the first bank that introduced ATM (Automated Teller Machine) in India.
▪ Hong Cong and Shanghai Bank Corporation introduced the ATM concept in India in 1987.
▪ The first ATM was set-up in Andheri-East, Mumbai.
▪ India’s first Grain ATM has been set up as a pilot project in Farrukhnagar, Gurugram.
▪ SBI launched the first floating ATM in Kerala in 2004.
▪ Country’s first post office savings bank ATM is located at the Head Post Office in Thyagaraya Nagar,
Chennai.
▪ National Payments Corporation of India (NPCI) is the apex authority for maintaining a nationwide link
of all the ATMs in India.
▪ TCPSL is Tata communications payment solutions limited was the first company to get a licence from
RBI to open White Label ATM in India in Mumbai.

Q. What is the primary purpose of the National Carbon Registry launched by the UNDP in August 2023?
A) To facilitate carbon credit trading between countries
B) To track emissions and invest in emission-reducing projects
C) To provide customizable software for carbon management
D) To promote digital public infrastructure for climate challenges
Answer : B
✓ National Carbon Registry is open-source software that allows countries to manage their carbon credit
trading data and processes.
✓ The software was launched by the UNDP in August 2023. It is the first of its kind to allow countries to
customize the code to their specific needs.

Follow us: Official Site, Telegram, Facebook, Instagram, Instamojo 239


✓ The registry is supported by the Digital4Climate Working Group, which includes the UNDP, World
Bank, UNFCCC, and EBRD.
✓ The group aims to build a digital public infrastructure to address climate challenges and carbon markets.
✓ The carbon registry system allows companies to track their emissions and then offset them by investing
in emission-reducing projects.
✓ For example, a company might invest in a project that captures methane from a landfill or replaces high-
emitting lightbulbs with low-emitting LED lights.
Q. Which of the following is planning to unveil a host of new measures, including 100 per cent cashless
claim settlement in health coverage?
A) LIC B) IRDAI
C) SIDBI D) RBI
Answer : B
✓ The Insurance Regulatory and Development Authority of India (IRDAI) is planning to unveil a host of
new measures, including 100 per cent cashless claim settlement in health coverage.
✓ About the Insurance Regulatory and Development Authority of India (IRDAI):
✓ It is a statutory body formed under an Act of Parliament, i.e., the Insurance Regulatory and Development
Authority Act, 1999 (IRDAI Act 1999), for overall supervision and development of the Insurance sector
in India.
✓ It is also responsible for registering and/or licensing insurance, reinsurance companies and intermediaries
according to the regulations.
✓ Entities regulated by IRDAI:
✓ Life Insurance Companies - Both public and private sector Companies
✓ General Insurance Companies - Both public and private sector Companies. Among them, there are some
standalone Health Insurance Companies which offer health Insurance policies.
✓ Re-Insurance Companies
✓ Agency Channel
✓ Intermediaries which include the following:
✓ Corporate Agents
✓ Brokers
✓ Third-Party Administrators
✓ Surveyors and Loss Assessors.
✓ Composition: It is a 10-member body- a chairman, five full-time members and four part-time members
appointed by the Government of India.
✓ IRDAI’s Head Office is in Hyderabad
✓ Insurance Regulatory and Development Authority (IRDAI)
✓ Founded : 1999
✓ Headquarteres : Hyderabad
✓ Chairman : Debasish Panda
Q. OTG Ring is India’s first contactless payment wearable ring. Which organisation has collaborated with
LivQuik to launch this technology?
A) NPCI B) Paytm
C) SBI D) IIT Indore
Answer : A
✓ NPCI launched India’s First Contactless Payment Wearable Ring “OTG Ring” in collaboration with
LivQuik.

Follow us: Official Site, Telegram, Facebook, Instagram, Instamojo 240


✓ NPCI has unveiled the “OTG Ring” which is the first contactless payment wearable ring made in India,
in collaboration with Indian startup LivQuik.
✓ “OTG Ring” can be enabled for National Common Mobility Card (NCMC) customers and used for
various open-loop transit programs.
✓ National Payments Corporation of India (NPCI).
✓ NPCI, an umbrella organization for operating retail payment and settlement systems in India, has been
initiated by the 'RBI' and the 'Indian Banks' Association' (IBA) under the 'Payment and Settlement
Systems Act, 2007'.
✓ It is a 'not-for-profit' company established under the provisions of Section 25 of the Companies Act 1956
(as amended in 2013), with the objective of providing physical and electronic payment infrastructure to
the entire banking system in India.
Q. Which of the following is not a valid tenor of Treasury bills?
A) 182 days B) 91 days
C) 365 days D) 364 days
Answer : C
✓ Recently, the yields on Treasury Bills (T-Bills) eased by up to seven basis points (bps) following an
improvement in liquidity in the banking system.
✓ About Treasury Bills:
✓ T-Bills are money market instruments.
✓ These are short-term debt instruments issued by the Government of India.
✓ Tenure: These are presently issued in three tenors, namely, 91 days, 182 days and 364 days.
✓ These are zero coupon securities and pay no interest. Instead, they are issued at a discount and redeemed
at the face value at maturity.
✓ When were treasury bills introduced?
✓ Treasury bills were first issued in India in 1917. They are issued via auctions conducted by the Reserve
Bank of India (RBI) at regular intervals.
✓ Who can buy it?
✓ Individuals, trusts, institutions and banks can purchase T-Bills. But they are usually held by financial
institutions.
✓ They have a very important role in the financial market beyond investment instruments.
✓ Banks give treasury bills to the RBI to get money under repo.
✓ Similarly, they can also keep it to fulfil their Statutory Liquid Ratio (SLR) requirements.
✓ How do T-bills work?
✓ Treasury bills are issued at a discount to the original value, and the buyer gets the original value upon
maturity.
✓ For example, a Rs 100 treasury bill can be availed of at Rs 95, but the buyer is paid Rs 100 on the maturity
date. The return on treasury bills depends on the liquidity position of the economy. When there is a
liquidity crisis, the returns are higher, and vice versa.
Q. Under the Global Biofuel Alliance, the G20 members are urged to take initiative of blending ethanol in
petrol upto what percent at global level?
A) 25% B) 10%
C) 20% D) 15%
Answer : C
✓ Global Biofuel Alliance is the third multilateral initiative created by the Narendra Modi-led government,
behind the International Solar Alliance and the Coalition for Disaster Resilient Infrastructure.
✓ India, Brazil and the United States are the founding members of the GBA.

Follow us: Official Site, Telegram, Facebook, Instagram, Instamojo 241


✓ The initiative urges countries at a global-level to take ethanol blending in petrol up to 20 percent.
Q. The Jal Jeevan mission has reported to provide tap water connections to how many rural households till
2023 since its launch?
A) 13 crore B) 11 crore
C) 09 crore D) 15 crore
Answer : A
✓ The Jal Jeevan Mission (JJM) has achieved the milestone of providing tap water connections to 13 Crore
rural households on September 05, 2023.
✓ Jal Jeevan Mission was launched on 15th August, 2019. It is a mission under Ministry of Jal Shakti.
✓ As on date, 6 States namely Goa, Telangana, Haryana, Gujarat, Punjab and Himachal Pradesh) and 3
UTs – Puducherry, D&D and D&NH and A&N Islands, have reported 100% coverage.
✓ About Jal Jeevan Mission
✓ It is a flagship programme of the Government of India which was announced from the ramparts of Red
Fort by the Prime Minister on August 15, 2019.
✓ The objective of the mission is to supply potable tap water in adequate quantity, prescribed quality and
on a regular and long-term basis to every rural household of the country by 2024.
✓ It is implemented by the Government of India in partnership with the States/Union Territories.
✓ This mission comes under the Ministry of Jal Shakti.
✓ Funding Pattern :
✓ The fund sharing pattern between the Center and the states under the mission is 90:10 for Himalayan and
North-Eastern states and 50:50 for other states.
✓ In the case of Union Territories, 100% contribution is made by the Centre

Q. Who is the present Chief Economic Advisor of India?


A) V Anantha Nageswaran B) Pramod Kumar Mishra
C) Sood Ajay D) T.V. Somanath
Answer : A
✓ Chief Economic Advisor V Anantha Nageswaran has said the economy is expected to grow at 6.5 per
cent in the current year i.e. FY2023-24.
Q. Which institution has been granted patent for the creation and manufacturing of a “reusable straw”?
A) KVIC B) BSI
C) NABARD D) e-NAM
Answer : B
✓ The Government of India’s patent office has granted a patent to the Botanical Survey of India (BSI) for
the creation and manufacturing of a “reusable straw.”
✓ This straw is developed from the endemic bamboo species known as Schizostachyum andamanicum,
which is native to the Andaman and Nicobar Islands. The bamboo’s economic potential has received a
significant boost with the patent issuance.
Q. The RBI has encouraged/called upon which set of entities, to establish Self-Regulatory Organizations
(SROs)?
A) Multi-state Cooperative Banks B) Fin-Tech Firms
C) Small Finance Banks D) Regional Rural Banks

Follow us: Official Site, Telegram, Facebook, Instagram, Instamojo 242


Answer : B
✓ The Reserve Bank of India (RBI) has recently called upon fintech entities to establish Self-Regulatory
Organizations (SROs).
✓ These SROs serve as non-governmental entities responsible for creating and enforcing industry-specific
codes of conduct on transparency, fair competition, and consumer protection.
✓ They would act as intermediaries between regulators and market participants.

Q. Which department launched ‘Recognition Scheme for ISPs Promoting Rural FTTH Broadband’?
A) Department of Telecommunications B) Department of Post
C) Department of Electronics D) Department of Expenditure
Answer : A
✓ The Department of Telecommunications (DoT) has introduced a scheme to acknowledge Internet
Service Providers (ISPs) for their outstanding contributions to expanding Fiber-to-the-Home (FTTH)
broadband connectivity in rural India.
✓ This initiative aligns with the government’s objective of promoting digital inclusion and fostering socio-
economic development in rural areas.
Q. Who has been appointed as the interim MD & CEO of Kotak Mahindra Bank in place of incumbent
Uday Kotak?
A) Nilesh Shah B) Jaimin Bhatt
C) Dipak Gupta D) Jaideep Hansraj
Answer : C
✓ Uday Kotak, the founder of Kotak Mahindra Bank has resigned as the Managing Director (MD) and
CEO with effect from September 1, 2023. His tenure as MD and CEO was to end on December 31, 2023.
✓ Dipak Gupta, the Joint Managing Director, will carry out the duties of the MD & CEO of Kotak
Mahindra Bank until December 31, 2023, subject to approval from the Reserve Bank of India (RBI).
✓ Kotak Mahindra Bank Limited
✓ Founded : 1985
✓ Founders : Uday Kotak
✓ Headquarters : Mumbai
Q. The Indian government has introduced the Unified Portal for Agricultural Statistics (UPAg), an online
platform developed by the Ministry which Ministry?
A) Ministry of Cooperation B) Ministry of Agriculture & Farmers Welfare
C) Ministry of Minority Affairs D) Ministry of Micro, Small and Medium Enterprises
Answer : B
✓ Recently, the Unified Portal for Agricultural Statistics developed by the Ministry of Agriculture and
Farmers’ Welfare (DA & FW) was launched by NITI Aayog members.
✓ About Unified Portal for Agricultural Statistics:
✓ This innovative platform is designed to streamline and enhance data management in the agricultural
domain
✓ It marks a significant step towards a more efficient and responsive agricultural policy framework.
✓ It is a crucial component of the Digital Public Infrastructure for Agriculture.
✓ It is an initiative by the Department of Agriculture and Farmers’ Welfare to streamline data management
in the agriculture sector.

Follow us: Official Site, Telegram, Facebook, Instagram, Instamojo 243


✓ It aims to provide real-time, standardised, and verified data on agricultural commodities, facilitating data-
driven decision-making for policymakers, researchers, and stakeholders.
Q. The Union Law Ministry has launched ‘One nation, One election’ committee. Who is the head of this
committee?
A) Ram Nath Kovind B) Amit Shah
C) Nitin Gadkari D) Rajnath Singh
Answer : A
✓ Government constitutes 8-member committee to examine ‘One nation, One election’: Head- Ram Nath
Kovind.
✓ Law Ministry has constituted an 8-member committee to examine ‘One nation, One election’. This
committee will be chaired by Ram Nath Kovind.
✓ The seven other members are- seven other members which includes Home Minister Amit Shah, Senior
Congress leader Adhir Ranjan Chowdhury, Ghulam Nabi Azad, NK Singh, Subhash C. Kashyap,
Harish Salve and Sanjay Kothari.
✓ The committee will examine and recommend specific amendments to the Constitution, the
Representation of the People Act and any other laws and rules which would require amendments for the
purpose of holding simultaneous elections.
Q. The Ministry of Education and, Ministry Skill Development & Entrepreneurship have partnered with
which company to launch 3-year partnership “Education to Entrepreneurship?
A) IBM B) Microsoft
C) Meta D) Alphabet
Answer : C
✓ Govt partners with Meta for ‘Education to Entrepreneurship’ to skill 500,000 entrepreneurs
✓ Ministry of Education, Ministry Skill Development & Entrepreneurship along with Meta have launched
a 3-year partnership “Education to Entrepreneurship: Empowering a generation of students, educators
and entrepreneurs”. It was launched by Union Minister for Education and Skill Development &
Entrepreneurship Shri Dharmendra Pradhan.
✓ Under this partnership, 5 lakh entrepreneurs will get access to digital marketing skills by Meta over the
next 3 years. In the initial stage, the budding and existing entrepreneurs will be trained in digital
marketing skills using Meta platforms in 7 regional languages.
✓ For this, 3 Letters of Intent (LoI) were exchanged between Meta and National Institute for
Entrepreneurship and Small Business Development (NIESBUD), AICTE and CBSE
Q. What is the purpose of the Malaviya Mission, recently launched by the Government for India?
A) Teachers Training Programme B) Health Education Programme
C) Satellite Launch Programme D) Drone Manufacturing Programme
Answer : A
✓ On the occasion of Teachers Day on September 05, 2023, the Union Minister for Education and Skill
Development & Entrepreneurship Shri Dharmendra Pradhan launched the Malaviya Mission – Teachers
Training Programme by the University Grants Commission in New Delhi.
✓ The minister also inaugurated the Portal of the Programme.
✓ The Malaviya Mission – Teacher Training Programme has been organised by the University Grants
Commission (UGC), in association with the Ministry of Education.

Follow us: Official Site, Telegram, Facebook, Instagram, Instamojo 244


✓ The mission aims to provide tailored training programmes for 15 lakh teachers in higher educational
institutions (HEIs) to ensure their continuous professional development and capacity building of faculty
members.
✓ Under this mission, 111 Malaviya Mission centres will be set up across India in a time-bound manner.

Q. Who has launches India's Own Blockchain-Backed Open-Source Project 'Falcon'?


A) RBI B) NPCI
C) SBI D) NTPC
Answer : B
✓ India has maintained a rather sceptic stance towards accepting cryptocurrencies as part of its financial
systems, but is keeping an open approach towards exploring blockchain technology.
✓ The National Payments Corporation of India (NPCI), on 29 August 2023, announced the launch of
Falcon.
✓ Aim : An open-source project that aims to simplify the management and use of blockchains based on
‘Hyperledger Fabric' and supported on Kubernetes clusters. Essentially, Falcon is expected to help
developers innovate on blockchain-based payment solutions...
✓ What is Hyperledger?
✓ Hyperledger Fabric was initiated by Digital Asset and IBM, as per Investopedia. Hyperledger fabric is an
enterprise-level permission blockchain network. The framework serves as a foundation for creating
blockchain-based products and solutions..
✓ What are Kubernetes Clusters?
✓ Developed by Google engineers Joe Beda, Brendan Burns, and Craig McLuckie in 2014, Kubernetes is
an open-source platform that assists the management of packaged software codes, also called
containerised applications.
Q. Which Company has launched First Fully Automated Investment Fund, 'The Ad Astra Fund'?
A) Savart B) Stratmark
C) Sreekavyam D) Shares Bazaar
Answer : A
✓ Savart, one of India’s largest investment advisors, is all set to launch its unique Portfolio Management
Service (PMS), The "Ad Astra Fund" in Hyderabad.
✓ Sankarsh Chanda, Founder & CEO of Savart briefed in detail about the launch event which is expected
to be graced by the presence of eminent investors from across the country. This unique fund transforms
the traditional human led investment research processes and increases the probability of achieving a
superior return on investment.
✓ The fund features end-to-end research automation, from investment opportunity discovery until exit i.e.,
the entire research and investment decision making is driven by their AI research system APART
(Advanced Process Automation & Research Technology), which comprises of three engines:

Q. Which company has launched of a new Mobile Application Called 'Share(dot)Market'?


A) Google Pay B) Amazon Pay
C) PhonePe D) Apple Pay
Answer : C
✓ PhonePe announced its foray into stock broking business with the launch of a new mobile application
called ‘Share(dot)Market’, under its subsidiary PhonePe Wealth Broking.

Follow us: Official Site, Telegram, Facebook, Instagram, Instamojo 245


✓ The announcement by the leading digital payment platform marks the completion of offerings in its
financial services bouquet, said Sameer Nigam, the CEO and Co-Founder of the company.
✓ According to PhonePe, the Share(dot)Market app elevates the discount broking by providing market
intelligence and quantitative research-based WealthBaskets, a scalable technology platform. It will be
available as a mobile app and a dedicated web platform, enabling retail investors to buy stocks, engage
in intra day trades, buy curated WealthBaskets and mutual funds
✓ Share(dot)Market app will provide a wide spectrum of investment products, including stocks (intraday
and delivery), mutual funds, exchange-traded funds (ETFs).
✓ WealthBaskets are curated collections of stocks/investment products by SEBI registered intermediaries
that align with specific themes, sectors, or market trends enabling active equity portfolio building with
great convenience, and are available at low costs.
Q. SEBI has reconstituted its alternative investment policy advisory committee with 25 members to advise
on issues impacting the development of the Alternative Investment Fund (AIF) space. Who will chair
this committee?
A) V K Paul B) KV Subramanian
C) Nidhi Khatri D) N R Narayana Murthy
Answer : D
✓ The Securities and Exchange Board of India (SEBI) has reconstituted its alternative investment policy
advisory committee with 25 members to advise on issues impacting the development of the Alternative
Investment Fund (AIF) space.
✓ The committee is chaired by Infosys co-founder N R Narayana Murthy and includes members from
SEBI, the Ministry of Finance, and industry associations.
✓ The panel, which was constituted by Sebi in March 2015, had 20 members when it was last rejigged by
the regulator in February 2022.
✓ Till now, the committee has submitted three reports on the AIF (Alternative Investment Fund) industry.
Q. A global caller identification platform, Truecaller has partnered with which payment banking platform
to provide a fast and secure online checkout experience for shoppers with its 1-Tap, OTP-Less
Verification solution?
A) Razorpay B) Paytm
C) PhonePe D) GooglePay
Answer : A
✓ A global caller identification platform, Truecaller has partnered with Razorpay to provide a fast and
secure online checkout experience for shoppers with its 1-Tap, OTP-Less Verification solution.
✓ Through this solution by Truecaller, Razorpay will be able to deliver a faster, seamless checkout
experience to the 200Mn+ customers that transact via Razorpay’s checkout annually.
✓ Customers would no longer need to manually fill up their contact details or wait for OTPs during the
checkout, as these details will get auto-populated by Truecaller with a single tap.
✓ Razorpay is an India-based, fintech company that provides payment gateway services to vendors,
merchants and ecommerce platforms.
✓ Razorpay
✓ Founded : 2014
✓ Founder : Shashank Kumar & Harshil Mathur
✓ Headquarters : Bangalore

Follow us: Official Site, Telegram, Facebook, Instagram, Instamojo 246


Q. Which ministry has signed an MoU with the National Safai Karamcharis Finance and Development
Corporation (NSKFDC) to enhance the socio-economic upliftment of Safai Karamcharis?
A) Ministry of Road Transport and Highways
B) Ministry of Corporate Affairs
C) Ministry of Tribal Affairs
D) Ministry of Social Justice and Empowerment
Answer : D
✓ Ministry of Social Justice signs MoU with NSKFDC
✓ A momentous collaboration has been cemented as the Ministry of Social Justice & Empowerment and
the National Safai Karamcharis Finance and Development Corporation (NSKFDC) inked a MoU for
the fiscal years 2023-24 and 2024-25.
✓ This strategic partnership aims to enhance the socio-economic upliftment of Safai Karamcharis, Manual
Scavengers, Waste Pickers and their dependants.
✓ NSKFDC is continuously working for the empowerment of these most vulnerable and deprived sections
of society through its various loan based and non-loan-based schemes like Short Term Skill Development
Training Programmes.
✓ NSKFDC also provides loans upto Rs 1 lacs under its Mahila Samridhi Yojana and Micro Credit Finance
Scheme. The overall rate of interest is 4% and 5%, respectively.
✓ The other loan schemes are General Term Loan and Green Business Schemes under which loans upto
Rs.15.00 lacs and upto Rs.30.00 lacs respectively are provided with rate of interest at 6% and 1% rebate
for women beneficiaries.
Q. Diversified group ITC will invest around Rs 1,500 crore to set up an integrated food manufacturing and
logistics facility in which state?
A) Uttar Pradesh B) Chhattisgarh
C) Madhya Pradesh D) Jharkhand
Answer : C
✓ Diversified group ITC will invest around Rs 1,500 crore to set up an integrated food manufacturing and
logistics facility and sustainable packaging products manufacturing facility at Sehore in Madhya Pradesh.
✓ The two projects spread over an area of nearly 57 acres will give a boost to agricultural and manufacturing
sectors in Madhya Pradesh.
✓ The two projects, when complete, would entail a total investment outlay of Rs 1,500 crores which will
support livelihoods across sustainable value chain.
✓ While the food plant will manufacture ITC’s products including atta brand Aashirvaad, Sunfeast biscuits,
and ‘YiPPee!’ noodles, the moulded fibre products facility will be a pioneer in sustainable packaging,
contributing to plastic substitution in areas such as packaging for electronic items, FMCG, and the food
and beverage sector.
Q. Under the GRIHA Rating Norms, a building securing 71 points on a 80 point scale, will be assigned
which of the following star ratings
A) Two star
B) Three star
C) Four star
D) Five star

Follow us: Official Site, Telegram, Facebook, Instagram, Instamojo 247


Answer : B
✓ The Indian Army’s new Thal Sena Bhawan (TSB) is coming up with GRIHA-IV (Green Rating for
Integrated Habitat Assessment) norms.
✓ About GRIHA norms:
✓ GRIHA is an acronym for Green Rating for Integrated Habitat Assessment. GRIHA is a Sanskrit word
meaning – ‘Abode’.
✓ It is a rating tool that helps people assess the performance of their building against certain nationally
acceptable benchmarks.
✓ It evaluates the environmental performance of a building holistically over its entire life cycle, thereby
providing a definitive standard for what constitutes a ‘green building’.
✓ The rating system, based on accepted energy and environmental principles, will seek to strike a balance
between the established practices and emerging concepts, both national and international.
✓ It is developed by TERI (The Energy and Resources Institute).
✓ This tool has been adopted by the Ministry of New and Renewable Energy.
✓ Criteria:
✓ It assesses a building out of 34 criteria and awards points on a scale of 100.
✓ In order to qualify for GRIHA certification, a project must achieve at least 50 points.
✓ Project scoring
✓ 50-60 points are certified as a 1-star GRIHA rated building,
✓ 61-70 is a 2-star GRIHA rated building,
✓ 71-80 is a 3-star GRIHA rating building,
✓ 81-90 is a 4-star GRIHA rated building
✓ 91-100 is a 5-star GRIHA rated building

Q. Which of the following statement about PM-Daksh Yojana is/are not correct?
A) During 2020-21 to 2022-23, a total of 107,156 beneficiaries
B) It was launched during 2020-21
C) The age criterion of the scheme is between 18-60 years
D) Nodal Ministry : Ministry of Social Justice & Empowerment
Answer : C
✓ During 2020-21 to 2022-23, a total of 107,156 beneficiaries have been trained under the PM-DAKSH
Yojana.
✓ PM-DAKSH Yojana:
✓ The Pradhan Mantri Dakshata Aur Kushalata Sampanna Hitgrahi (PM-DAKSH) Yojana is a Central
Sector Scheme.
✓ It was launched during 2020-21.
✓ The main objective of the Scheme is to enhance the competency level of the target groups to make them
employable both in self-employment and wage-employment for their socio-economic development.
✓ It is a National Action Plan for skilling marginalised persons covering SCs, OBCs, EBCs, DNTs,
Sanitation workers and waste pickers.
✓ The age criterion of the scheme is between 18-45 years, and the income criteria are no income limit for
SCs, SafaiKaramcharis, including waste pickers and DNT.
✓ The annual family income should be below Rs.3 lakh for OBCs, and the EBCs (Economically Backward
Classes) annual family income should be below Rs.1 lakh.
✓ Nodal Ministry: Ministry of Social Justice & Empowerment (MoSJ&E).

Follow us: Official Site, Telegram, Facebook, Instagram, Instamojo 248


Q. Which among the following ministries launched programme under "Gramodyog Vikas Yojana" to
benefit artisans?
A) Ministry of Human Resource Development
B) Ministry of Skill Development and Entrepreneurship
C) Ministry of Micro, Small and Medium Enterprises
D) Ministry of Environment, Forest and Climate Change
Answer : C
✓ Recently, the Chairman of the Khadi and Village Industries Commission (KVIC), Ministry of Micro,
Small and Medium Enterprises distributed tool-kits and machineries to artisans under 'Gramodyog Vikas
Yojna.
✓ About Gramodyog Vikas Yojana :
✓ It is one of the two components of the Khadi Gramodyog Vikas Yojana, which aims to promote and
develop the village industries through common facilities, technological modernisation, training, etc.
✓ It includes the activities carried out under different village industries.
✓ Target beneficiaries: Artisans, Unemployed Youth and Self Help Groups (SHGs)
✓ Eligibility criteria: Any person who is a Traditional Artisan, has Knowledge of Skill, and has experience
in Art and Craft
Q. Which organisation has developed open-source software that allows countries to effectively manage
national data and processes for trading carbon credits?
A) UNEP B) WEF
C) UNDP D) UNESCO
Answer : C
✓ Recently, the United Nations Development Programme (UNDP) has developed open-source software
that allows countries to effectively manage national data and processes for trading carbon credits.
✓ About National Carbon Registry:
✓ This software has recently been accredited as a digital public good (DPG).
✓ As a DPG, the registry uses open-source code, which allows countries to replicate and adapt the
information to fit their own needs and contexts.
✓ The registry’s modules, software and technical documentation can be reused and tailored by countries,
which could potentially reduce production costs and implementation timelines.
✓ The registry follows national and international best practices, which include UNDP, the World Bank,
the United Nations Framework Convention on Climate Change (UNFCCC) and the European Bank for
Reconstruction and Development (EBRD)
Q. Digital Infrastructure for Knowledge Sharing (DIKSHA) platform is associated with which Union
Ministry?
A) Ministry of Education B) Ministry of Civil Aviation
C) Ministry of Jal Shakti D) Ministry of Earth Sciences
Answer : A
✓ The National eGovernance Division (NeGD) under the Ministry of Electronics and Information
Technology (MeitY) is set to integrate Personalised Adaptive Learning (PAL) into its existing Digital
Infrastructure for Knowledge Sharing (DIKSHA) platform.
✓ About DIKSHA Platform:

Follow us: Official Site, Telegram, Facebook, Instagram, Instamojo 249


✓ It is a national platform for school education, an initiative of the National Council for Education
Research and Training (NCERT), Ministry of Education.
✓ It provides e-content for schools via an online portal and a mobile application.
✓ It was developed based on the core principles of open architecture, open access, open licensing, choice
and autonomy.
✓ It is built on open-source technology that is made in India and for India, incorporating internet-scale
technologies and allowing for a variety of use cases and solutions for teaching and learning.

Q. Which Union Ministry implements the Aatma Nirbhar Bharat Rozgar Yojana?
A) Ministry of Labour and Employment
B) Ministry of Finance
C) Ministry of Corporate Affairs
D) Ministry of Agriculture & Farmers Welfare
Answer : A
✓ The Central Government's innovative employment incentive scheme, the Aatmanirbhar Bharat Rozgar
Yojana (ABRY), has exceeded its initial employment generation goals, showcasing its success in
fostering job creation and recovery during the COVID-19 pandemic.
✓ About Aatmanirbhar Bharat Rozgar Yojana:
✓ It was launched in 2020.
✓ This was designed to stimulate the creation of new job opportunities by extending financial support to
employers of establishments registered with the Employees' Provident Fund Organization (EPFO).
✓ This scheme aimed to incentivise employment of unemployed individuals, including those who lost their
jobs due to the pandemic, by covering both the employee and employer contributions (24% of wages) for
establishments with up to 1000 employees.
✓ For establishments with over 1000 employees, only the employee's EPF contributions (12% of wages)
were covered in respect of new employees.
✓ As of July 31, 2023, the ABRY has already achieved an enrolment of over 7.58 million new employees,
surpassing its initial employment generation target.
✓ Significance: It substantially contributed to the job market's revival, underscoring its role in boosting
economic recovery during these challenging times.
✓ The EPFO is under the administrative control of the Ministry of Labour and Employment, Government
of India
Q. The government has exempted imports of LPG, liquified propane and liquified butane from ____ agri
cess with effect from September 1.
A) 15% B) 10%
C) 20% D) 13%
Answer : A
✓ The Government recently exempted imports of LPG, liquified propane and liquified butane from 15 per
cent Agriculture Infrastructure and Development Cess (AIDC).
✓ About Agriculture Infrastructure and Development Cess (AIDC):
✓ It was introduced in the Union Budget 2021-22.
✓ It is a tax that the government imposes on the commercial production of agricultural produce.
✓ The cess is imposed at a specific rate on the value of these goods.
✓ Purpose:
✓ The primary objective of AIDC is to provide financial support for the development of agricultural
infrastructure in India.

Follow us: Official Site, Telegram, Facebook, Instagram, Instamojo 250


✓ The funds collected through this cess will be utilised for the creation and maintenance of agricultural
infrastructure such as cold storage facilities, warehouses, and market yards.
✓ It aims to enhance the overall efficiency of the agricultural sector and improve farmers' income.
✓ What is a Cess?
✓ Cess is a form of tax charged/levied over and above the base tax liability of a taxpayer.
✓ A cess is usually imposed additionally when the state or the central government looks to raise funds for
specific purposes.

Q. Who is set to launch its innovative Adopt a Heritage 2.0 programme?


A) Geological Survey of India B) Archaeological Survey of India
C) British Standards Institution D) Indian Council of Agricultural Research
Answer : B
✓ The Archaeological Survey of India is set to launch its innovative Adopt a Heritage 2.0 programme.
✓ About Adopt a Heritage 2.0 Scheme:
✓ It seeks to invite corporate stakeholders to enhance amenities at monuments by utilising their CSR funds.
✓ It is a revamped version of the earlier scheme launched in 2017. It clearly defines the amenities sought
for different monuments as per the Ancient Monuments and Archaeological Sites and Remains Act
(AMASR Act) 1958.
✓ Key Facts about “Adopt a Heritage Scheme”:
✓ It is an initiative of the Ministry of Tourism in collaboration with the Ministry of Culture and the
Archaeological Survey of India.
✓ It was launched in September 2017 on World Tourism Day
✓ The corporate sector is expected to use corporate social responsibility (CSR) funds for the upkeep of the
site.
Q. Which of the following is the term used for storehouses where banknotes and rupee coins are stocked on
behalf of the Reserve Bank for distribution to bank branches?
A) currency depot B) coin depot
C) currency bank D) currency chest
Answer : D
✓ Two cases were recently registered after RBI detected fake notes from the SBI currency chest in Kochi,
Kerala.
✓ About Currency Chest:
✓ A currency chest is a place where the Reserve Bank of India (RBI) stocks the money meant for banks and
ATMs.
✓ The RBI has authorised select scheduled banks to establish currency chests.
✓ The monitoring authority over the chests lies with the RBI. They are inspected by representatives of RBI
from time to time.
✓ Whenever new currency notes are printed by RBI, they are first delivered to currency chests. These notes
are then delivered to other banks by the banks holding the currency chests.
✓ RBI takes back soiled notes and mutilated notes from the public through currency chests.
✓ Moreover, if the bank has an excess cash balance, the excess is moved to the currency chest. In a vice
versa situation, when the bank is low on cash balance, the currency chest provides it.
✓ Guidelines for banks to set up new currency chests:
✓ The area of the room or vault should be at least 1500 sq. ft.
✓ In case of hilly/ inaccessible places, the area of the strong room or vault should be at least 600 sq. ft.
✓ Apart from this, the new chests should have a processing capacity of 6.6 lakh pieces of banknotes per
day.

Follow us: Official Site, Telegram, Facebook, Instagram, Instamojo 251


✓ For those situated in hilly/ inaccessible places, a capacity of 2.1 lakh pieces of banknotes per day.
✓ The currency chests should have a Chest Balance Limit (CBL) of ₹1,000 crore, subject to ground realities
and reasonable restrictions, at the discretion of the Reserve Bank

Q. Which Country has granted its first golden visa to Sam Altman, the CEO of OpenAI?
A) Malaysia B) Indonesia
C) Japan D) Singapore
Answer : B
✓ Indonesia has granted its first golden visa to Sam Altman, the CEO of OpenAI, a pioneering firm in
artificial intelligence.
✓ The Indonesian immigration authority issued a 10-year visa to Altman, acknowledging his global renown
and potential contributions to the nation.
✓ Indonesia's Immigration Director General has said that Altman "has an international reputation and may
bring benefits to Indonesia", Bloomberg reported.
✓ He believes the visa will serve as an incentive for high-profile foreign investors

Q. Corning to Build $113m India's First Gorilla Glass Plant in Which State?
A) Karnataka B) Gujarat
C) Maharashtra D) Telangana
Answer : D
✓ US-based multinational technology company "Corning" has announced plans to set up a $113m Gorilla
Glass manufacturing facility in the Indian state of Telangana.
✓ Corning to build $113m Gorilla Glass plant in India was originally created and published by Investment
Monitor, a GlobalData owned brand.

Q. Union Minister Dharmendra Pradhan has tied up with whom to empower 'teachers and entrepreneurs'?
A) Meta B) Apple
C) Google D) Microsoft
Answer : A
✓ Union Minister for Education Dharmendra Pradhan launched a 3-year partnership of his Ministry with
Meta to provide Digital and marketing skills to students, educators, and entrepreneurs in New Delhi.
✓ Three Letters of Intent (LoI) were exchanged between Meta and NIESBUD, AICTE and CBSE.
Speaking at the event, Dharmendra Pradhan said that the initiative launched is in furtherance to Prime
Minister Shri Narendra Modi’s vision of making India a skilled capital of the world and empowering our
Amrit Peedhi.
✓ Under the partnership with NIESBUD, five lakh entrepreneurs will get access to digital marketing skills
by Meta over the next three years..

Q. Which Company Has Announced to Acquire 23,000 MWh of Energy for Nxtra Data Centers?
A) Bharti Airtel B) Jio
C) Flipkart D) Paytm
Answer : A
✓ Bharti Airtel has announced its commitment to purchase 23,000 MWh of renewable energy by Q4 FY
23–24 to power its data centre subsidiary, Nxtra.

Follow us: Official Site, Telegram, Facebook, Instagram, Instamojo 252


✓ Airtel will acquire stakes in renewable energy project companies Continuum Green India Pvt. Ltd. and
Vibrant Energy Holdings Pte. Ltd. to facilitate this endeavour
✓ This initiative is aligned with Airtel’s goal to reduce its carbon footprint and promote sustainability.
✓ Through an open access route, Airtel will acquire stakes in Continuum Green’s project company, which
will supply green power from solar and wind projects to six Nxtra Edge data centres located in Madhya
Pradesh.
Q. International Cricket Council has signed a sponsorship deal with which Indian bank?
A) State Bank of India B) Punjab National Bank
C) Yes Bank D) IndusInd Bank
Answer : D
✓ The International Cricket Council (ICC) has signed a multi-year sponsorship deal with IndusInd Bank.
This deal has been done for $ 20-24 million (about Rs 160-200 crores). In addition to this deal with
IndusInd Bank, Mastercard has stepped in for the top sponsorship for the upcoming ICC Cricket World
Cup 2023.
✓ IndusInd Bank Limited was established in April 1994 and is headquartered in Mumbai, Maharashtra
Q. Which Municipal Corporation has started a Cash for Waste scheme for common people at the recently
established Material Recovery Facility (MRF)?
A) Mumbai Municipal Corporation B) Noida Municipal Corporation
C) Darbhanga Municipal Corporation D) Patna Municipal Corporation
Answer : D
✓ Patna Municipal Corporation starts a Cash for Waste scheme.
✓ Working on the theme of waste to wealth, the Patna Municipal Corporation has started a Cash for Waste
scheme for common people at the recently established Material Recovery Facility (MRF).
✓ People are being paid for their dry waste under a prescribed price list for each item.
✓ The state-of-the-art facility has a capacity of processing 2 tonne of dry garbage and 1.5 tonne of wet waste
per day. It has been named Pink MRF as it is run by women workers.
✓ The MRF has been set up by Patna Municipal Corporation (PMC) with the help of the Ministry of
Environment, Forest and Climate Change, German institution GIZ, and other organizations.

Q. Which company has partnered with Dassault Systems as part of the Living Heart Project?
A) TCS
B) Infosys
C) Wipro
D) Reliance Industries
Answer : A
✓ Tata Consultancy Services Ltd. has partnered with Dassault Systems as part of the Living Heart Project.
✓ This initiative brings together a network of cardiovascular experts, medical professionals, and regulatory
bodies, including the US FDA, to develop and validate highly realistic digital simulations of the human
heart.
✓ The Living Heart Project aims to revolutionize medical solutions, including the utilization of heart
simulations as digital evidence for the approval of new cardiovascular devices.

Follow us: Official Site, Telegram, Facebook, Instagram, Instamojo 253


Q. The Reserve Bank of India has issued revised classification, valuation, and operation guidelines for the
investment portfolio of banks, which will be effective from 1 April 2024. Under the revised guidelines,
which types of banks are excluded from the application of RBI’s issues revised norms?
A) All Commercial Banks B) Cooperative Banks
C) NBFCs D) Regional Rural Banks
Answer : D
✓ The Reserve Bank of India has issued revised classification, valuation and operation guidelines for the
investment portfolio of commercial banks, which will be effective from April 1, 2024.
✓ Banks will have to categorize investments into three categories from the next financial year: available for
sale (AFS), held to maturity (HTM), and a new category called ‘fair value through profit and loss’, or
FVTPL.
✓ The existing held-for-trading (HFT) category will become a sub-category of the FVTPL.
✓ The revised directions include principle-based classification of the investment portfolio, tightening of
regulations around transfers to/from held to maturity (HTM) category and sales out of HTM, inclusion
of non-SLR (statutory liquidity ratio) securities in HTM subject to fulfillment of certain conditions and
symmetric recognition of gains and losses.
✓ The revised Directions shall apply to all commercial banks (excluding Regional Rural Banks) from the
financial year commencing on April 1, 2024.
Q. Which organization has partnered with the National Bank for Agriculture and Rural Development
(NABARD) to co-create data-driven innovations in agriculture and food systems and to support
smallholder farmers?
A) WEF B) ADB
C) UNDP D) UNEP
Answer : C
✓ The United Nations Development Programme (UNDP) and the National Bank for Agriculture and Rural
Development (NABARD) have signed a MoU to co-create data-driven innovations in agriculture and
food systems to support smallholder farmers.
✓ Under the MoU, the organizations will work to improve lives and livelihoods of smallholder farmers by
sharing open-source data for product development, transfer of technology and supporting the framing of
agrarian policies.
✓ The partnership aims at enhancing and disseminating collaborative digital public goods like DiCRA
(Data in Climate Resilient Agriculture).
✓ DiCRA provides open access to key geospatial datasets pertinent to climate-resilient agriculture.
✓ DiCRA, which is curated by UNDP and partner organizations to inform public about investments in
agriculture, already provides intelligence on climate resilience for 50 million hectares of farmland across
the country.
Q. Which bank has launched ‘ONDC in a Box’, a one-stop-shop proposition for corporates towards seamless
enablement on ONDC?
A) Citibank B) HSBC Bank India
C) Standard Chartered Bank D) MUFG Bank
Answer : B
✓ Hong Kong and Shanghai Banking Corporation (HSBC) India has launched ‘ONDC in a Box’, a one-
stop-shop proposition for corporates towards seamless enablement on ONDC.

Follow us: Official Site, Telegram, Facebook, Instagram, Instamojo 254


✓ With this launch, HSBC India became the first foreign bank in India to offer an ONDC-enabled
proposition.
✓ The Open Network for Digital Commerce (ONDC) is an initiative by the Department for Promotion of
Industry and Internal Trade, and the Ministry of Commerce and Industry to build an open, interoperable
network on which buyers and sellers can transact without needing to be present on the same platform.
Q. Which bank has unveiled its metaverse concept for Better World, a gamified adventure for metaverse use
cases?
A) MUFG Bank B) DBS Bank
C) HSBC Bank India D) Standard Chartered Bank
Answer : B
✓ DBS has unveiled its metaverse concept for DBS BetterWorld, a gamified adventure for metaverse use
cases.
✓ Through DBS BetterWorld, we aim to drive awareness about issues such as food waste and create
engagement opportunities.
✓ This intends to align with the DBS Foundation’s focus on championing social enterprises that bridge gaps
within food and agricultural value chains.
Q. GQG Partners has acquired how many equity shares of IDFC First Bank Ltd’s MD & CEO in a ₹479.50-
crore block deal transaction?
A) 2 crore B) 3 crore
C) 4 crore D) 5 crore
Answer : D
✓ GQG Partners has acquired about 5.1 crore equity shares of IDFC First Bank Ltd’s MD & CEO in a
₹479.50-crore block deal transaction.
✓ MD & CEO of IDFC Bank, Vaidyanathan’s shareholding in IDFC FIRST Bank will increase from 0.58%
as of June 30, 2023, to 1.04%.
✓ The net proceeds of the sale of ₹478.7 crore will be used to subscribe for new shares of the bank through
the exercise of options, to pay associated income tax, and to make contributions to specific pre-committed
social causes
✓ Out of the ₹478.7 crore sales proceeds, ₹229 crore would be paid to IDFC First Bank to subscribe to new
shares of the bank.
✓ In December 2018, Capital First merged with IDFC Bank. As part of the Amalgamation Scheme, which
IDFC Bank and Capital First jointly agreed upon, these Capital First CMD stock options were converted
to IDFC First Bank stock options
Q. Which insurance company has launched a new product “Life Income Laabh” with an aim to simplify
insurance and deliver maximum protection to its consumers?
A) Universal Sompo Insurance B) Cholamandalam MS Insurance
C) Bharti AXA Life Insurance D) Bajaj Allianz Insurance
Answer : C
✓ Bharti AXA Life Insurance has launched a new product “Bharti AXA Life Income Laabh” with an aim
to simplify insurance and deliver maximum protection to its consumers.
✓ The plan provides financial protection and a steady income stream to consumers.
✓ The newly launched plan provides customers with a steady stream of guaranteed income for 10 to 12
years, depending on the policy term they choose.

Follow us: Official Site, Telegram, Facebook, Instagram, Instamojo 255


✓ It starts with an initial income pay-out equivalent to 100 percent of the annual premium paid by the
policyholder. After 5 years, for those opting for a 10-year policy term, the income increases to 150
percent. Similarly, for a 12-year policy term, the income sees a boost of 200 percent after 6 years.
Q. Which of the following has partnered with the government’s computer emergency response unit CERT-
In to train 1,000 government officials?
A) Google Cloud B) Meta
C) Adobe D) Microsoft
Answer : A
✓ Google Cloud has partnered with the government’s computer emergency response unit CERT-In to train
1,000 government officials and provide 1 lakh cybersecurity certificate scholarships to learners.
✓ CERT-In is part of the Ministry of Electronics and Information Technology (MeitY), which deals with
cybersecurity threats, hacking, and other cyber-related issues.
✓ The ‘cyber force’ of government officials will be trained in cyber defence best practices, including the use
of generative AI, and conducting cybersecurity AI hackathons led by Google Cloud and Mandiant
experts.
Q. Which country has pledged $2 billion to the UN’s Green Climate Fund (GFC) to support vulnerable
countries in dealing with climate change?
A) United States B) United Kingdom
C) Japan D) Russia
Answer : B
✓ UK Prime Minister Rishi Sunak has pledged $2 billion to the UN’s Green Climate Fund (GFC) to support
vulnerable countries in dealing with climate change.
✓ The Green Climate Fund (GCF) is the biggest single funding commitment the UK has made to help the
world tackle climate change.
✓ It was established by 194 countries following the Copenhagen Accord at COP15.
✓ The GCF is the largest global fund dedicated to supporting developing countries to reduce global
emissions and support communities to adapt to the effects of climate change.
Q. Which card issuer company has launched new features of its super-premium card ‘AURUM’, targeted at
the premium section of society, such as high-net-worth individuals?
A) American Express
B) SBI Card
C) Mastercard
D) Visa
Answer : B
✓ India’s largest credit card issuer, SBI Card has launched new features of its super-premium card
‘AURUM’, targeted at the premium section of society, such as high-net-worth individuals.
✓ With the enhancements, AURUM cardholders will be able to avail benefits worth up to Rs 2 lakh
annually, based on their spending.
✓ The card offers unlimited international lounge access to cardholders, as well as four international lounge
visits for accompanying guests.

Follow us: Official Site, Telegram, Facebook, Instagram, Instamojo 256


Q. Which company has partnered with KredX in a strategic move aimed at revolutionizing B2B digital
payments, ensuring benefits for both enterprises and vendors?
A) Mastercard B) Visa
C) SBI Card D) American Express
Answer : A
✓ Mastercard has partnered with KredX, a supply chain finance platform, in a strategic move aimed at
revolutionizing B2B digital payments, ensuring benefits for both enterprises and vendors.
✓ This partnership involves Mastercard seamlessly integrating its commercial card services into the KredX
platform, effectively streamlining the intricacies commonly associated with B2B payments
Q. Which company has launched a new credit card Simply SAVE Merchant Card for MSMEs to cater to
their short-term credit requirements?
A) SBI Card B) Mastercard
C) American Express D) RuPay
Answer : A
✓ Credit card issuer SBI Card has launched a new credit card Simply SAVE Merchant SBI Card for MSMEs
to cater to their short-term credit requirements.
✓ Available on RuPay network, the card be linked to UPI for payments through various UPI-enabled third-
party apps.
✓ The launch of the SimplySAVE Merchant SBI Card is an attempt to address this larger segment by
extending interest-free short-term credit to MSMEs.
Q. Which of the following has signed a Memorandum of Understanding (MoU) with the Union Bank of
India and Bank of Baroda to co-finance renewable energy (RE) projects?
A) Solar Energy Corporation of India
B) Power Finance Corporation
C) Indian Renewable Energy Development Agency
D) THDC India Limited
Answer : C
✓ Indian Renewable Energy Development Agency (IREDA) has signed a Memorandum of Understanding
(MoU) with the Union Bank of India and Bank of Baroda to co-finance renewable energy (RE) projects.
✓ Through the agreement, IREDA will collaborate with the banks in co-lending and loan syndication of
emerging and established RE projects.
✓ This collaboration aims to extend our reach, particularly in tier-2 & tier-3 cities and rural areas, enabling
us to provide unique and innovative financial support to existing and new customers.

Q. Which Company Signed MoU With Uttar Pradesh Govt To Set Up Bus Plant At ₹200 Cr?
A) Tata Motors B) BharatBenz
C) Mahindra & Mahindra D) Ashok Leyland
Answer : D
✓ Ashok Leyland, one of the largest manufacturers of commercial vehicles, has decided to set up its electric
bus manufacturing unit in Uttar Pradesh. If all goes well, then the company may set up the unit in
Lucknow.

Follow us: Official Site, Telegram, Facebook, Instagram, Instamojo 257


✓ Ashok Leyland is setting up its electric vehicles manufacturing unit in the state. We gave them four to
five choices. It has narrowed down the choice to the land of Scooters India Limited in Lucknow. About
70-acre land of SIL will be given to the company for the purpose. An investment of ₹200 to ₹500 crore
will be made in the first phase. It will invest ₹1,000 crore later

Q. India has become the ____ Internationally Accepted OIML Certificates Issuing Authority.
A) 19th B) 16th
C) 13th D) 10th
Answer : C
✓ India has become the Internationally Accepted OIML (International Organization of Legal Metrology)
Certificates Issuing Authority, Consumer Affairs Ministry.
✓ Indiais the 13th country in the world that can issue the OIML approval certificates

Q. Which IIT signed a MoU with private lender ICICI Bank to support the startup ecosystem in the institute?
A) IIT Delhi B) IIT Kanpur
C) IIT Madras D) IIT-Guwahati
Answer : B
✓ The Indian Institute of Technology Kanpur (IITK) signed a memorandum of understanding (MoU) with
private lender ICICI Bank to support the startup ecosystem in the institute.
✓ The premier technological institution said that this partnership will empower startups and innovators at
its Startup Incubation and Innovation Centre (SIIC). SIIC is one of the oldest incubators in the country
having been engulfed on its journey in 2000.
Q. The Union Cabinet has recently approved the extension of which scheme?
A) Pradhan Mantri Ujjwala Yojana B) Har Ghar Jal Yojana
C) Jan Dhan Yojana D) PM Kisan Yojana
Answer : A
✓ The Union Cabinet chaired by Prime Minister Narendra Modi has approved the expansion of Pradhan
Mantri Ujjwala Yojana to release 75 lakh LPG connections in three years from the financial year 2023-
24 to 2025-26. With the provision of 75 lakh additional Ujjwala connections, the total number of
beneficiaries of this scheme will increase to 10.35 crore. This scheme was launched by Prime Minister
Narendra Modi on 1 May 2016.

Follow us: Official Site, Telegram, Facebook, Instagram, Instamojo 258


BEST MCQ BANKING, ECONOMY AND FINANCIAL AWARENESS
AUGUST 2023

Q. RBI has launched a centralized web portal "UDGAM" to help citizens search for their unclaimed
deposits. What is the full form of UDGAM?
A) Unclaimed Deposits Gateway to Account inforMation
B) Unclaimed Deposits Gateway to Allied inforMation
C) Unclear Deposits Gateway to Account inforMation
D) Unclaimed Deposits Gateway to Access inforMation
Answer : D
✓ The Reserve Bank of India (RBI) has launched a centralized web portal to help citizens search for their
unclaimed deposits known as UDGAM.
✓ UDGAM stands for Unclaimed Deposits Gateway to Access inforMation.
✓ The portal has been developed by the central bank for the public to help them search for their unclaimed
deposits across multiple banks at one place.
✓ How will the UDGAM portal work?
✓ The launch of the web portal will help users identify their unclaimed deposits/accounts and enable them
to either claim the deposit amount or make their deposit accounts operative at their respective banks.
✓ Reserve Bank Information Technology Pvt Ltd (ReBIT), Indian Financial Technology & Allied Services
(IFTAS) and participating banks have collaborated on developing the portal.
✓ Users would be able to access the details of their unclaimed deposits in respect of seven banks presently
available on the portal.
✓ These banks are the State Bank of India, Punjab National Bank, South Indian Bank, Central Bank of
India, Dhanlaxmi Bank, DBS Bank India, CitiBank N.A.
✓ The RBI said the search facility for the remaining banks on the portal would be made available in a
phased manner by October 15, 2023.
RBI IN NEWS 2023
▪ Reserve Bank of India (RBI) has launched a centralized web portal to help citizens search for their
unclaimed deposits known as UDGAM.
▪ UDGAM stands for Unclaimed Deposits Gateway to Access inforMation.
▪ Reserve Bank of India (RBI) has approved the re-appointment of Sandeep Bakshi as ICICI Bank's MD
and CEO from October 4, 2023, to October 3, 2026.
▪ Reserve Bank of India (RBI) has swiftly increased the offline payment transaction upper limit from ₹200
to ₹500, aiming to enhance user convenience.
▪ Reserve Bank of India (Classification, Valuation and Operation of Investment Portfolio of Commercial
Banks) Directions, 2023 has been issued.
▪ The revised Directions shall apply to all commercial banks (excluding Regional Rural Banks) from the
financial year commencing on April 1, 2024.
▪ Reserve Bank of India (RBI) has issued guidelines for banks, non-banking financial companies (NBFCs)
and other regulated entities to address the issues faced by the borrowers and towards promoting.
▪ The Regulated Entities (REs) shall release all the original movable / immovable property documents and
remove charges registered with any registry within a period of 30 days after full repayment/ settlement
of the loan account. In case where the delay is attributable to the RE, it shall compensate the borrower
at the rate of ₹5,000/- for each day of delay.
▪ ICICI Bank has successfully secured approval from the Reserve Bank of India (RBI) to establish I-Process
Services (India) Pvt Ltd (I-Process) as its wholly-owned subsidiary.

Follow us: Official Site, Telegram, Facebook, Instagram, Instamojo 259


▪ Bank for International Settlements (BIS) and the Reserve Bank of India (RBI) have recently unveiled the
winners of the G20 TechSprint 2023 challenge in a prestigious award ceremony held in Mumbai.
▪ Problem Statement #1: Combating Financial Crimes
▪ Winner: Team Secretarium Ltd. (UK)
▪ Solution: Transaction Monitoring and Privacy Protection.
▪ Reserve Bank of India (RBI) has announced a significant expansion of the Unified Payments Interface
(UPI) System, allowing users to transact with pre-sanctioned credit lines issued by banks.
▪ The UPI System was primarily used for transactions involving deposited amounts. However, as of April
6, 2023, the RBI has proposed an expansion of the UPI System to include pre-sanctioned credit lines.
▪ In July 2023, India witnessed a significant surge in bank credit outstanding to the real estate sector,
reaching an all-time high of Rs 28 trillion, as per data released by the Reserve Bank of India (RBI).
▪ This impressive growth was driven by both housing and commercial real estate segments, with annual
increases of approximately 37.4% and 38.1%, respectively.
▪ Reserve Bank of India (RBI) has greenlit the merger of Twin Cities Co-operative Urban Bank with Kranti
Co-operative Urban Bank.
▪ Reserve Bank of India (RBI) Introduces Reforms for Transparent Home Loan Equated Monthly
Installments (EMIs).
▪ Reserve Bank of India (RBI) Governor, Shaktikanta Das, launched a financial inclusion dashboard
named ‘Antardrishti‘.
▪ Swaminathan Janakiraman has been appointed as the new Deputy Governor of the Reserve Bank of
India (RBI) by the Government of India.
▪ Swaminathan Janakiraman will replace Mahesh Kumar Jain.
▪ Reserve Bank of India (RBI) instructed Mahalaxmi Cooperative Bank (Dharwad,
▪ Karnataka), to operate solely as a Non-Banking Financial Company (NBFC), effectively revoking its
banking
▪ licence granted on March 23, 1994.
▪ RBI floating rate bonds interest rate hiked to 8.05% from July 1, 2023.
▪ The minimum investment starts at Rs 1,000 with no limit on the maximum amount.
▪ RBI injects Rs 75,000 crore into the banking system as liquidity tightens due to tax outflows.
▪ RBI allows Card Network Portability from Oct 1, 2023.
▪ RBI Includes S.Korea’s ‘NongHyup Bank’ in the list of Scheduled Banks under Schedule II of RBI Act,
1934.
▪ As per the Reserve Bank of India's (RBI) report on Currency and Finance (RCF) for the year 2022-23,
India would require at least 2.5% of GDP annually until 2030 for green financing.
▪ RBI Mandates Complete Information for Money Transfers to Curb Money Laundering and Terrorism
Financing.
▪ The Reserve Bank of India (RBI) has levied penalties of Rs 44 lakhs on four cooperative banks for
violating various norms.
▪ RBI allows AU Small Finance Bank to deal with foreign exchange.
▪ RBI to launch centralised portal PRAVAAH for licencing, approval applications.
▪ “PRAVAAH” (Platform for Regulatory Application, Validation And AutHorisation).
▪ Reserve Bank of India (RBI) announced that Neeraj Nigam has been appointed as the new executive
director (ED).
▪ India and Tanzania have received approval from the Reserve Bank of India (RBI) to utilize their
respective national currencies in bilateral trade settlements.
▪ Governor of the RBI, Shaktikanta Das, initiated the establishment of a "Greenfield Data Centre" and an
"Enterprise Computing & Cybersecurity Training Institute" in Bhubaneswar.
▪ RBI’s Digital Payments Index Jumps to 377.46 in September from 349.30 in March.
▪ Reserve Bank of India (RBI) has approved the transfer of Rs 87,416 crore surplus to the government for
the financial year 2022-23.
▪ Reserve Bank of India (RBI) has decided to withdraw Rs 2,000 denomination banknotes from circulation
under the "Clean Note Policy".

Follow us: Official Site, Telegram, Facebook, Instagram, Instamojo 260


▪ The ₹2000 denomination banknote was introduced in November 2016 under Section 24(1) of RBI Act,
1934.
▪ A person can exchange Rs.2000 banknotes up to a limit of Rs.20,000 at a time.
▪ RBI has taken significant actions in the financial sector, canceling the certificate of registration of 7
NBFCs and accepting the surrender permits of 14 NBFCs.
▪ RBI has announced its participation in the Global Financial Innovation Network (GFIN) greenwashing
techsprint.
▪ HSBC Bank has been fined Rs 1.73 crore by the Reserve Bank of India (RBI) for violating the Credit
Information Companies Rules 2006 (CIC Rules).
▪ Reserve Bank of India (RBI) and the Bank for International Settlements (BIS) launched the G20
TechSprint 2023 competition for cross-border payments innovation..
▪ Policy Repo Rate : 6.50%
▪ Standing Deposit Facility Rate : 6.25%
▪ Marginal Standing Facility Rate : 6.75%
▪ Bank Rate : 6.75%
▪ Fixed Reverse Repo Rate : 3.35%
▪ CRR : 4.50%
▪ SLR : 18.00%.
▪ RBI and Central Bank of UAE signed a Memorandum of Understanding (MoU) to promote innovation
in financial products and services.
▪ Reserve Bank of India (RBI) has granted an ‘Infrastructure Finance Company (IFC)’ status to Indian
Renewable Energy Development Agency (IREDA) .
▪ Reserve Bank of India has imposed a penalty of Rs 3.06 crore on Amazon Pay (India) Private Limited
for non-compliance of certain directions related to Prepaid Payment Instruments and Know Your
Customer.
▪ RBI imposed restrictions on 5 co-operative banks.
▪ Eligible depositors of all five co-operative banks will receive deposit insurance claim amount of up to ₹5
lakh from the Deposit Insurance and Credit Guarantee Corporation.
▪ Reserve Bank of India on 6 January said that maiden Sovereign Green Bonds (SGrBs) would be issued
in two tranches of Rs 8,000 crore each .
▪ Sovereign green bonds worth Rs 8000 crore will be issued on January 25, 2023. In this, green bonds worth
Rs 4000 crore will be issued for 5 years and sovereign green bonds worth Rs 4000 crore will be issued for
10 years.
▪ Reserve Bank of India (RBI) has retained SBI, ICICI bank and HDFC bank in the list of Domestically
Systemically Important Banks (D-SIB) list for the year 2021.
▪ RBI started disclosing the name of the bank in the D-SIB list from 2015 and SBI was included in the list.
▪ ICICI Bank was included in 2016 and HDFC bank was included in 2017.
▪ Reserve Bank of India (RBI) has recently initiated the ‘100 Days 100 Pays’ campaign, aimed at tracing
and settling the top 100 unclaimed deposits of every bank in every district within a span of 100 days.
▪ RBI Annual Report 2022-23: General Government Deficit and Debt Moderate to 9.4% and 86.5% of
GDP, Respectively.
▪ RBI Imposes Rs 84.50 Lakh Penalty on Central Bank of India for Non-Compliance with Fraud Reporting
Norms.
▪ Reserve Bank of India (RBI) has granted approval for the voluntary Scheme of Amalgamation of Maratha
Sahakari Bank with The Cosmos Co-operative Bank.
▪ Reserve Bank of India (RBI) has instructed financial institutions and banks to adopt an Alternative
Reference Rate, primarily the Secured Overnight Financing Rate (SOFR), and end their reliance on the
scandal-shrouded London Interbank Offered Rate (LIBOR) and Mumbai Interbank Forward Outright
Rate (MIFOR) by July 1.
▪ Reserve Bank of India (RBI) imposed a monetary penalty of Rs 1.73 crore on the Hongkong and Shanghai
Banking Corporation Ltd (HSBC Bank) for violating the Credit Information Companies Rules, 2006.

Follow us: Official Site, Telegram, Facebook, Instagram, Instamojo 261


▪ Reserve Bank of India (RBI) has increased its gold reserves by 4.5% to 794.64 metric tonnes in the fiscal
year that ended on March 31, 2023.
▪ RESERVE BANK OF INDIA (RBI)
▪ Founded : 1 April 1935
▪ RBI Nationalised : 1 January 1949
▪ HQ : Mumbai, Maharashtra
▪ RBI set up : Hilton Young Commission
▪ 1st Governor : Sir Osborne Smith (Australia)
▪ 1st Indian Governor : CD Deshmukh
▪ 25th Governor : Shaktikanta Das
▪ RBI Deputy Governor (4)
1. Tavarna Rabi Sankar
2. Michael D Patra
3. M Rajeshwar Rao
4. Swaminathan Janakiraman
▪ FIVE SUBSIDIARIES OF RBI :
1. Deposit Insurance and Credit Guarantee Corporation of India (DICGC)
2. Bharatiya Reserve Bank Note Mudran Private Limited (BRBNMPL)
3. Reserve Bank Information Technology Private Limited (ReBIT)
4. Indian Financial Technology and Allied Services (IFTAS).
5. Reserve Bank Innovation Hub (RBIH)
▪ India has four currency note printing presses.
▪ Nashik in Maharashtra and Dewas in Madhya Pradesh owned by govt of India.
▪ Mysuru in Karnataka and Salboni in West Bengal owned by RBI.
▪ Coins are minted in four mints owned by the Government of India.
▪ Mumbai, Hyderabad, Calcutta and Noida
Q. The Reserve Bank of India (RBI) has approved the amalgamation of Akola Merchant Co-operative Bank
Ltd, located in Akola, Maharashtra, with The Jalgaon Peoples Co-operative Bank Ltd, situated in
Jalgaon, Maharashtra. This merger has been sanctioned by the RBI, exercising its powers as per Sub-
Section (4) of Section 44A in conjunction with ___of the Banking Regulation Act, 1949.
A) Section 48 B) Section 51
C) Section 56 D) Section 65
Answer : C
✓ The Reserve Bank of India (RBI) has approved the amalgamation of Akola Merchant Co-operative Bank
Ltd, located in Akola, Maharashtra, with The Jalgaon Peoples Co-operative Bank Ltd, situated in
Jalgaon, Maharashtra.
✓ This merger has been sanctioned by the RBI, exercising its powers as per Sub-Section (4) of Section 44A
in conjunction with Section 56 of the Banking Regulation Act, 1949.
Q. The Reserve Bank of India (RBI) has raised the upper limit of an offline payment transaction to ____
from the existing Rs 200 to promote the use of UPI-Lite wallet in areas where internet connectivity is
weak or unavailable.
A) Rs.500 B) Rs.1500
C) Rs.2000 D) Rs.1000
Answer : A
✓ The Reserve Bank of India (RBI) has raised the upper limit of an offline payment transaction to Rs 500
from the existing Rs 200 to promote the use of UPI-Lite wallet in areas where internet connectivity is

Follow us: Official Site, Telegram, Facebook, Instagram, Instamojo 262


weak or unavailable. The total limit for offline transactions on a payment instrument, however, remains
Rs 2,000. It also said that offline payments should only be made face-to-face.
✓ Moreover, these transactions may be offered without Additional Factor of Authentication (AFA).
✓ However, overall limit is retained at Rs 2000 to contain the risks associated with relaxation of two-factor
authentication.
✓ The information for the same was provided by RBI under Section 10 (2) read with Section 18 of the
Payment and Settlement Systems Act, 2007 (Act 51 of 2007)/PSS Act.

Q. GREAT scheme, seen recently in news, is associated with


A) Food Standards B) Technical textiles
C) Education sector reform D) Green revolution in eastern states
Answer : B
✓ Recently, the Union Ministry of Textiles released the guidelines of the GREAT scheme.
✓ About GREAT Scheme:
✓ Grant for Research and Entrepreneurship across Aspiring Innovators in Technical Textiles (GREAT)
Scheme focuses on supporting individuals and companies to translate prototypes to technologies &
products, including commercialisation.
✓ Aim: To provide much-needed impetus for the development of the technical textiles startup ecosystem in
India, especially in niche sub-segments such as bio-degradable and sustainable textiles, high-performance
and speciality fibres, and smart textiles.
✓ Funding:
✓ Under this scheme, a grant-in-aid of up to Rs 50 lakh for up to a period of 18 months will be provided.
✓ The textile ministry will additionally provide 10 per cent of the total grant-in-aid to incubators.
✓ Only a minimum of 10 per cent contribution has to be made by the incubatee.
Q. The Union Cabinet has approved the PM-eBus Seva initiative worth how many rupees to increase the
use of electric buses in 169 cities?
A) Rs 65,189 crore B) Rs 57,613 crore
C) Rs 48,781 crore D) Rs 37,723 crore
Answer : B
✓ The Cabinet, chaired by Prime Minister Narendra Modi, has approved the Rs 57,613 crore PM-eBus
Sewa initiative to augment the use of electric buses in 169 cities.
✓ About ‘PM-eBus Sewa’ Scheme
✓ Under PM-eBus Sewa Scheme, 10,000 e-buses will be deployed across cities in the country.
✓ Support for Operation:
✓ Under this scheme, city bus operations will be done on Public Private Partnership (PPP) model.
✓ This scheme will support bus operations for 10 years.
✓ States/Cities shall be responsible for running the bus services and making payments to the bus operators.
✓ The Central Government will support these bus operations by providing subsidy to the extent specified
in the proposed scheme.
✓ Funding:
✓ It has been allocated a total funding of Rs 57,613 crore.
✓ Out of this financial provision, the central government will contribute Rs 20,000 crore, while the
remaining portion will be covered by the state governments.
✓ Coverage: The scheme will cover cities with 3 lakhs and above population and priority will be given to
cities not having organised bus services.
✓ There are two segments of the scheme: Augmenting the city bus services and Green Urban Mobility
initiatives.

Follow us: Official Site, Telegram, Facebook, Instagram, Instamojo 263


✓ Segment A: Augmenting City Bus Services (169 cities)
✓ Segment B: Green Urban Mobility Initiatives (181 cities):
Q. With reference to UDGAM Portal, consider the following statements:
1.It is a platform designed to promote environmental conservation by encouraging the use of single-
use/biodegradable plastics and disposable products in everyday life.
2.It has been launched by the Reserve Bank of India (RBI).
Select the correct statements using the codes given below:
A) 1 only B) 2 only
C) Both 1 and 2 D) Neither 1 nor 2
Answer : B
✓ Statement 1 is incorrect. UDGAM Portal stands for Unclaimed Deposits – Gateway to Access
information. Its aim is to aid users to identify their unclaimed deposits/ accounts and enable them to
either claim the deposit amount or make their deposit accounts operative at their respective banks.
✓ Statement 2 is correct. The portal has been launched by the RBI.
✓ The Reserve Bank of India (RBI) recently launched a Centralised Web Portal, UDGAM (Unclaimed
Deposits – Gateway to Access inforMation), for the public to search for their unclaimed deposits across
multiple banks at one place.
✓ About UDGAM Portal
✓ It has been developed by RBI for use by members of public to facilitate and make it easier for them to
search their unclaimed deposits across multiple banks at one place.
✓ Reserve Bank Information Technology Pvt Ltd (ReBIT), Indian Financial Technology & Allied Services
(IFTAS), and participating banks have collaborated on developing the portal.
✓ The portal will enable users to either claim the deposit amount or make their deposit accounts operative
at their respective banks.
✓ Procedure:
✓ Customers can register on the 'Udgam' platform using their mobile number.
✓ Once registered, they can search for unclaimed deposits under their name and provide additional inputs
such as PAN, voter ID, driving licence and passport number.
✓ Customers can then retrieve their deposits by completing a KYC process with their branch.
✓ In case the deposit-holder has passed away, their nominee will be required to submit documents
✓ What are Unclaimed Deposits?
✓ According to RBI, "Unclaimed Deposits" refers to funds held in savings or current accounts that have
remained inactive for a duration of 10 years, or in the case of fixed deposits (FDs), have not been
withdrawn within 10 years from the maturity date.
Q. North East Special Infrastructure Development Scheme (NESIDS), a central sector scheme with 100%
central funding, receives a renewed approved outlay of Rs_______ for 2022-23 to 2025-26.23.
A) Rs. 7526.50 B) Rs. 7892.50
C) Rs. 8139.50 D) Rs. 8578.50
Answer : C
✓ Recently, the cabinet gave approval for the continuation of the North East Special Infrastructure
Development Scheme (NESIDS) with an approved outlay of Rs. 8139.50 crore
✓ About the North East Special Infrastructure Development Scheme (NESIDS):
✓ It was approved by the Government of India as a Central Sector Scheme in 2017.

Follow us: Official Site, Telegram, Facebook, Instagram, Instamojo 264


✓ Under the scheme guidelines of NESIDS, 100% of the central funding is provided to the State
Governments of North Eastern Region.
✓ The funding is provided for projects of physical infrastructure relating to water supply, power, and
connectivity, enhancing tourism, and Social infrastructure relating to primary and secondary sectors of
education and health.
✓ New guidelines:
✓ For the period from 2022-23 to 2025-26, Cabinet approved two components, , NESIDS-Road and
NESIDS-Other Than Road Infrastructure (OTRI).
✓ The scheme is a central sector scheme with 100% central funding.
✓ The decisions of the Government, including the merger of the erstwhile North East Road Sector
Development Scheme (NERSDS) into the NESIDS-Road component etc., necessitated the formulation
of fresh guidelines to administer and implement the restructured NESIDS during the balance period of
the 15th Finance Commission.
✓ The objective of schemes
✓ To supplement the efforts of the different Central Ministries & Departments for uncovered
development/welfare activities.
✓ These schemes help in gap-filling support to the eight North Eastern States as per their felt needs by taking
up projects
Q. What is the share provided by the Centre in implementation of projects under “North East Special
Infrastructure Development Scheme” (NESIDS)?
A) 75% B) 50%
C) 60% D) 100%
Answer : D
✓ The North East Special Infrastructure Development Scheme” (NESIDS) is a Central Sector Scheme on
launched in 2017, under which 100% centrally funding is provided to the State Governments of North
Eastern Region for the projects of physical infrastructure.
✓ Recently, the continuation of the NESIDS with an approved by the cabinet, for the period from 2022-23
till 2025-26 at an outlay of Rs.8139.50 crore.
Q. What is the outlay of the ‘Promotion of Research and Innovation in Pharma MedTech Sector’ scheme?
A) Rs 5000 crore B) Rs 3000 crore
C) Rs 2000 crore D) Rs 1000 crore
Answer : A
✓ The Union Cabinet recently approved the Promotion of Research & Innovation in Pharma-MedTech
sector (PRIP) scheme with an outlay of ₹5,000 crore for five years.
✓ The objective of the PRIP scheme is to transform the Indian Pharma MedTech sector from cost-based
competitiveness to innovation-based growth by strengthening the research infrastructure in the country.
✓ The aim of the scheme is to promote industry-academia linkage for R&D in priority areas to inculcate a
culture of quality research and nurture our pool of scientists.
✓ Rs 5,000 crore will be spent in five years (2024-28) under the PRIP scheme, and companies will be given
incentives for research.
Q. In August 2023, how many rupees of bad loans have been written off by banks in the last nine financial
years starting from 2014-15?
A) Rs 8.73 lakh crore B) Rs 10.39 lakh crore
C) Rs 12.41 lakh crore D) Rs 14.56 lakh crore

Follow us: Official Site, Telegram, Facebook, Instagram, Instamojo 265


Answer : D
✓ Banks write off Rs 14.56 lakh crore NPAs in last nine financial years.
✓ Banks have written off bad loans worth Rs 14.56 lakh crore in the last nine financial years starting 2014-
15 in Aug 2023.
✓ Out of the total Rs 14,56,226 crore, written-off loans of large industries and services stood at Rs 7,40,968
crore.
✓ the net of recovery in written-off loans during the financial year (net write-off) in public sector banks
(PSBs) was Rs 1.18 lakh crore in FY18, which declined to Rs 0.91 lakh crore in FY22 and to Rs 0.84
lakh crore (RBI provisional data) in FY23.
✓ Net write-off loans by private sector bank was Rs73,803 crore (RBI provisional data) in FY 2022-23.
✓ Net write-off as percentage of opening gross loans and advances in private sector banks was 1.25 percent
and 1.57 percent in FY 2017-18 and FY 2022-23 respectively, and it was 2 percent and 1.12 percent for
PSBs during the same period.
✓ During the year ended March 2023, banks wrote off non-performing assets (NPAs), or bad loans, worth
over Rs 2.09 lakh crore, as per Reserve Bank of India.

Q. 'Khanan Prahari' application, which was seen in the news, is associated with which Union Ministry?
A) Ministry of Coal B) Ministry of Corporate Affairs
C) Ministry of Commerce and Industry D) Ministry of Micro, Small and Medium Enterprises
Answer : A
✓ The mobile app Khanan Prahari is a significant step taken by the Ministry of Coal towards curbing illegal
coal mining activities.
✓ About Khanan Prahari App:
✓ It allows citizens to report incidents of illegal coal mining through geo-tagged photographs and textual
information.
✓ The corresponding web portal, called as Coal Mine Surveillance & Management System (CMSMS), has
been developed in association with the Bhaskaracharya Institute of Space Application & Geoinformatics,
Gandhinagar, and CMPDI, Ranchi.
✓ Objective: To encourage public participation through reporting about illegal coal mining

Q. What is the outlay of the BharatNet project?


A) Rs 1.10 lakh crore B) Rs 1.39 lakh crore
C) Rs 1.52 lakh crore D) Rs 1.75 lakh crore
Answer : B
✓ The Union Cabinet has approved an outlay of Rs 1.39 lakh crore for BharatNet, the government’s project
for last-mile connectivity across 6.4 lakh villages in the country
✓ About BharatNet Project
✓ BharatNet is the world's largest optical fiber-based rural broadband connectivity project.
✓ It is executed by Bharat Broadband Network Limited (BBNL), a special purpose organisation under the
Telecom Ministry.
✓ It is an ambitious rural internet access programme. An initiative by the Union government under its
Digital India programme
✓ Features & Benefits of BharatNet
✓ Using optical fibre, the programme is intended to bring broadband internet connectivity to each of the
more than 2.5 lakh gram panchayats across the country.
✓ The government intends to provide a minimum of 100 Mbps bandwidth at each Gram Panchayat through
BharatNet so that everyone, especially those in rural India, can access online services.

Follow us: Official Site, Telegram, Facebook, Instagram, Instamojo 266


✓ As part of BharatNet project, the Centre will also provide last mile connectivity through Wi-Fi and other
means and is setting up Wi-Fi hotspots in all gram panchayats
✓ Budget of BharatNet
✓ The total budget allocation for the BharatNet project is Rs. 61,000 crore.
✓ In the Union Budget 2022-23, the Government extended the project deadline to 2025.

Q. The global Technical Textile market size is expected to reach USD 272.3 billion by which year?
A) 2026 B) 2027
C) 2030 D) 2035
Answer : C
✓ According to a recent research report, the global Technical Textile market size is expected to reach USD
272.3 billion by 2030.
✓ About Technical Textiles.
✓ Technical Textiles are textile products that are manufactured primarily for their functionality and use
rather than aesthetic appeal.
✓ They are designed to have high levels of physical, mechanical, thermal, and/or chemical properties for
use in specific applications within industrial sectors such as earthworks, construction, civil engineering,
transport, defense, medical, and healthcare.
✓ Technical Textiles Mission in India.
✓ The Ministry of Textiles has launched NTTM to increase the penetration level of technical textiles in
India while leveraging the extraordinary growth rate of the sector.
✓ The aim of the mission is to position India as a global leader in Technical Textiles.
✓ The mission will comprise the following four components:
✓ Research, Innovation and Development;
✓ Promotion and Market Development;
✓ Export Promotion;
✓ Education, Training, Skill Development;
✓ Implementation: The NTTM has been approved with an implementation period of four years starting
from FY 2020-21 till FY 2023-24. The total outlay of the mission is INR 1480 Cr.
✓ Target : The Mission has a target to take the domestic market size of the technical textile sector to $ 40-
50 Bn by the year 2024 with an average growth rate of 15-20% per annum
Q. Which Small Finance bank has launched a 24x7of Video Banking Service, providing customers with real-
time video calls for banking services?
A) AU Small Finance Bank B) Ujjivan Small Finance Bank
C) Equitas Small Finance Bank D) Capital Small Finance Bank
Answer : A
✓ AU Small Finance Bank has launched 24×7 for 365 days of Video Banking, providing customers with
real-time video calls for banking services.
✓ AU Small Finance Bank is the first bank in India to launch this type of service.
✓ This facility is available to the bank’s customers irrespective of the type of their banking relationship
(credit card, loan, savings, digital savings, basic service savings, current account, etc).
✓ Even Pradhan Mantri Jan Dhan account holders of AU SFB can avail of this video banking service for
free.
✓ This 24X7 video calling feature deployed an advanced encryption layer for transmitting data between the
customer’s device to the bank’s servers.
✓ As of 2023, there are 12 Small Finance Banks in India.
✓ AU Small Finance Bank Limited

Follow us: Official Site, Telegram, Facebook, Instagram, Instamojo 267


✓ Founded :1996
✓ Founder : Sanjay Agarwal
✓ Headquarters : Jaipur, Rajasthan
✓ MD & CEO : Sanjay Agarwal
✓ Tagline : Chalo Aage Badhein
Q. Project AMBER ('Accelerated Mission for Better Employment and Retention') was launched by which
organization?
A) National Skill Development Corporation (NSDC)
B) Reserve Bank Information Technology Private Limited (ReBIT)
C) Reserve Bank Innovation Hub (RBIH)
D) Indian Financial Technology and Allied Services (IFTAS)
Answer : A
✓ The Ministry of Skill Development and Entrepreneurship (MSDE), in collaboration with Generation
India Foundation (GIF) and Amazon Web Services India Private Limited (AWS India), is providing
‘cloud’ skills training to 1,500 under project AMBER.
✓ About Project AMBER:
✓ The Accelerated Mission for Better Employment and Retention (AMBER) project is a joint initiative of
the National Skill Development Corporation (NSDC) – under the aegis of the Ministry of Skill
Development and Entrepreneurship (MSDE) - and GIF to create the necessary avenues.
✓ The initiative has been undertaken under the SANKALP programme of MSDE with a focus on women
to improve gender diversification in the tech industry and underprivileged groups.
✓ The Project AMBER was launched by National Skill Development Corporation (NSDC) and Generation
India Foundation (GIF) jointly. It aims to fill the gap between skilling and employment.
Q. Which of the following has recently unveiled an advanced multilingual multimodal AI translation and
transcription model named 'SeamlessM4T?
A) Google B) Microsoft
C) Apple D) Meta
Answer : D
✓ Meta, the technology company formerly known as Facebook, recently unveiled an advanced multilingual
multimodal AI translation and transcription model named 'SeamlessM4T.
✓ About SeamlessM4T:
✓ SeamlessM4T, which stands for Massively Multilingual and Multimodal Machine Translation, is an
advanced multilingual multimodal AI translation and transcription model.
✓ It was developed by Meta, the technology company formerly known as Facebook.
✓ SeamlessM4T is capable of performing various tasks including speech-to-text, speech-to-speech, text-to-
speech, and text-to-text translations.
Q. Which bank’s CEO has emerged as the highest-paid bank chief executive in the financial year 2022-23,
grossing over Rs 10.55 crore in overall pay?
A) ICICI Bank B) HDFC Bank
C) Yes Bank D) Axis Bank
Answer : B

Follow us: Official Site, Telegram, Facebook, Instagram, Instamojo 268


✓ HDFC Bank’s Jagdishan Highest Paid Bank CEO in FY23 with Rs 10.55cr Pay.
✓ HDFC Bank’s Sashidhar Jagdishan has emerged as the highest-paid bank chief executive in the financial
year 2022-23, grossing over Rs 10.55 crore in overall pay.
✓ Amitabh Chaudhry of Axis Bank drew Rs 9.75 crore payout and was the second highest-paid bank CEO.
✓ He was closely followed by Sandeep Bakhshi of bigger rival ICICI Bank, who drew Rs 9.60 crore for the
same year.
✓ Sashidhar Jagdishan, managing director (MD) and chief executive officer (CEO) of HDFC Bank, has
been chosen as the Business Standard Banker of the Year 2022.
HDFC BANK IN NEWS 2023
▪ HDFC Bank Launches India's First Co-Branded Hotel Credit Card named 'Marriott Bonvoy HDFC
Bank Credit Card' in partnership with Marriott International.
▪ HDFC Bank's Sashidhar Jagdishan is highest paid bank CEO in FY23 with Rs 10.55 crore pay.
▪ HDFC Bank overtakes SBI in CRISIL’s corporate banking ranking for 2023.
▪ Food delivery platform Swiggy has joined hands with HDFC Bank to launch a co-branded credit card.
▪ HDFC Bank, India’s largest private lender, has achieved a significant milestone by entering the $100
billion market-capitalization club as world’s 7th largest lender.
▪ HDFC emerges as fourth largest bank in the world after merger.
▪ According to Bloomberg, with a valuation of $172 billion, the merger will place HDFC in the fourth
position in equity market capitalisation behind JP Morgan Chase & Co., Industrial and Commercial
Bank of China Ltd. and Bank of America Corp.
▪ Deepak Parekh has announced his decision to step down as HDFC chairperson after serving as the
chairman for 46 years.
▪ HDFC Bank introduced a new credit card named ‘Millennia Credit card’ which offers cashback over e-
commerce platforms and multiple benefits.
▪ SEBI has granted approval for a change in control of HDFC Asset Management Company (HDFC
AMC) due to the amalgamation of HDFC Ltd and HDFC Bank Ltd.
▪ As part of the amalgamation, existing shareholders of HDFC Ltd will hold a 41% stake in HDFC Bank.
▪ HDFC Bank has launched a retail banking initiative called ‘Vishesh‘ aimed at customers in semi-urban
and rural areas.
▪ HDFC Bank partners Manipal Global to build up a talent pipeline.
▪ SEBI has granted approval for a change in control of HDFC Asset Management Company (HDFC
AMC) due to the amalgamation of HDFC Ltd and HDFC Bank Ltd.
▪ HDFC Bank has signed a “Master Inter Bank Credit Agreement” with Export-Import Bank of Korea for
a USD 300 million line of credit.
▪ HDFC Bank recommends re-appointment of Sashidhar Jagdishan MD, CEO for 3 yrs.
▪ HDFC Bank’s Sashidhar Jagdishan is ‘BS Banker of the Year 2022’.
▪ HDFC Bank, IRCTC launch India’s most rewarding co-branded travel credit card.
▪ HDFC Bank, Lulu Exchange ink deal to enhance cross-border payments between India-Gulf region.
▪ HDFC Bank collaborates with Microsoft as part of its digital transformation.
▪ Reserve Bank of India said SBI, ICICI Bank, HDFC Bank remain domestic systemically important banks
(D-SIBs). .
▪ India’s first Electronic Bank Guarantee issued by HDFC Bank.
▪ HDFC BANK
▪ Founded : 1994
▪ Founder : Hasmukhbhai Parekh
▪ HQ : Mumbai, Maharashtra
▪ MD & CEO : Sashidhar Jagdishan
▪ Tagline : We understand your world

Follow us: Official Site, Telegram, Facebook, Instagram, Instamojo 269


Q. Consider the following statements with respect to National Career service (NCS) Scheme:
1. National Career Service (NCS) is a one-stop solution that provides a wide array of employment and
career related services to the citizens of India
2. The scheme is being implemented by the Directorate General of Employment, Ministry of Labour &
Employment
Which of the following above statements is/are correct?
A) 1 only B) 2 only
C) Both 1 and 2 D) Neither 1 nor 2
Answer : C
✓ The Ministry of Labour & Employment recently said over one million vacancies in the private as well as
public sector are active on the National Career Service (NCS) portal.
✓ About National Career Service (NCS):
✓ NCS aims to bridge the gap between those who need jobs and those who want to hire them, between
people seeking career guidance and training and those who can provide the counselling and training.
✓ It was launched in the year 2015.
✓ It provides a host of career-related services such as dynamic job matching, career counselling, job
notifications, vocational guidance, and information on skill development courses, internships and alike.
Q. What is the current repo rate after the third bi-monthly monetary policy for FY24?
A) 4.5% B) 7.5%
C) 5.5% D) 6.5%
Answer : D
✓ Reserve Bank of India (RBI) Governor Shaktikanta Das announced the third bi-monthly monetary policy
for FY24.
✓ The three-day meeting of the six-member Monetary Policy Committee (MPC) of RBI was held from
August 8 to 10.
✓ The RBI kept the repo rate unchanged at 6.5%. Since May 2022, the central bank has raised the repo rate
by 250 basis points (bps).
✓ Interest Rates
✓ Repo rate unchanged at 6.5%
✓ Standing deposit facility (SDF) rate remains at 6.25%
✓ Marginal standing facility (MSF) rate and Bank Rate maintained at 6.75%
✓ CRR at 4.5%
✓ SLR at 18.00%
✓ Reverse Repo Rate is 3.35%
✓ Real GDP growth projection for FY224 retained at 6.5%
✓ The Reserve Bank is planning to artificial intelligence (AI) in UPI payments. Near Field Communication
(NFC) technology will be deployed in offline payments via in UPI-Lite
✓ RBI proposes to enhance transactions limits for small value digital payments to ₹500 from ₹200 in UPI
Lite
✓ To absorb surplus liquidity generated by various factors, including return of ₹2000 notes to the banking
system, RBI has asked banks to maintain an incremental Cash Reserve Ratio (ICRR) of 10 per cent on
the increase in their deposits between May 19 and July 28.
✓ RBI Governor Shaktikanta Das has announced that the cash reserve ratio (CRR) remains unchanged at
4.5 per cent.
✓ Next meeting of the RBI rate setting panel (MPC) will be held October 4-6.

Follow us: Official Site, Telegram, Facebook, Instagram, Instamojo 270


✓ The MPC has 6 members: three RBI officials and three externals. The three external members of MPC
are: Shashanka Bhide, Ashima Goyal and Jayanth R Varma. Besides governor Shaktikanta Das, the
other RBI officials in MPC are Rajiv Ranjan (executive director) and Michael Debabrata Patra (deputy
governor).

Q. What is the present Reverse Repo Rate as per the third Bi-monthly Monetary Policy Statement?
A) 3.15% B) 4.40%
C) 3.35% D) 4.90%
Answer : C
Q. RBI has kept the repo rate unchanged at what percent in the bi-monthly MPC meet on August 08, 2023?
A) 6.50% B) 6.25%
C) 6.10% D) 6.35%
Answer : A
Q. Who has inaugurated the AMC Repo Clearing Limited (ARCL) and Corporate Debt Market
Development Fund (CDMDF)?
A) Droupadi Murmu B) Arvind Kejriwal
C) Rajnath Singh D) Nirmala Sitharaman
Answer : D
✓ Recently, the Union Finance Minister launched the Corporate Debt Market Development Fund
(CDMDF) .
✓ About Corporate Debt Market Development Fund:
✓ It is a backstop facility for specified debt funds during market dislocations.
✓ The fund is intended to provide liquidity support in the event of a financial crisis.
✓ It will be in the form of an alternative investment fund (AIF), meant to instil confidence among the
participants in the Corporate Bond Market during times of stress
✓ The fund has Rs 33,000-crore backstop facility for Mutual Funds.
✓ Of the Rs 33,000 crore, Rs 30,000 crore will come from the government, while the balance Rs 3,000 crore
will be contributed by the Asset Management Companies.
✓ Contributions to the fund can be done by the specified debt-oriented mutual fund schemes and asset
management companies of mutual funds.
✓ This fund is guaranteed by the National Credit Guarantee Trust Company (NCGTC) and the backstop
facility will be managed by SBI Mutual Fund.

Q. Who will be responsible for regulating the Corporate Debt Market Development Fund (CDMDF)?
A) Reserve Bank of India (RBI)
B) Ministry of Finance, Government of India
C) Securities and Exchange Board of India (SEBI)
D) Association of Mutual Funds in India (AMFI)
Answer : C
✓ The Corporate Debt Market Development Fund (CDMDF) will be regulated by the Securities and
Exchange Board of India (SEBI).

Follow us: Official Site, Telegram, Facebook, Instagram, Instamojo 271


✓ The primary objective of CDMDF is to purchase investment-grade corporate debt securities during
stressed market conditions, thereby providing support and confidence to market participants.
✓ Any delay in contribution by AMCs of mutual funds to the CDMDF will attract a penalty of 15% per
annum on the respective AMC for the delayed period, with the interest credited to the CDMDF.
✓ The CDMDF will purchase corporate debt securities from mutual funds during stressed market
conditions, providing support to the market and confidence to participants
Q. Which bank has raised Rs 10,000 crore at a coupon rate of 7.54 percent through its third infrastructure
bond issuance?
A) Punjab National Bank B) State Bank of India
C) Kotak Mahindra Bank D) Bank of Baroda
Answer : B
✓ SBI raises 10,000 cr via 15-year Infrastructure bonds.
✓ State Bank of India (SBI) has raised Rs 10,000 crore at a coupon rate of 7.54 percent through its third
infrastructure bond issuance.
✓ The proceeds will be directed towards bolstering long-term resources for funding critical infrastructure
projects and supporting the affordable housing segment.
✓ The bank attracted bids of Rs 21,698 crores, a staggering 4.34 times the base issue size, indicating robust
demand for these financial instruments.
✓ The investors range from various sectors including provident funds, pension funds, insurance companies,
mutual funds, corporates, etc.
✓ Bidding for the 15-year tenor bonds took place on July 31
Q. Taiwanese electronics contract manufacturer Foxconn has signed a Letter of Intent (LOI) with which
state government for two projects with an estimated investment outlay of ₹5,000 crore?
A) Telangana B) Karnataka
C) Tamil Nadu D) Kerala
Answer : B
✓ Karnataka govt, Foxconn sign Letter of Intent with Rs 5000 cr investment.
✓ Taiwanese electronics contract manufacturer Foxconn has signed a Letter of Intent (LOI) with the
government of Karnataka for two projects with an estimated investment outlay of ₹5,000 crore, expected
to generate 13,000 jobs.
✓ These projects are in addition to the iPhone end assembly plant, which the company has planned to set
up at Devanahalli with an investment of about ₹14,000 crore.
✓ The intended projects include a phone enclosure project for which Foxconn Industrialist Internet (FII),
a Foxconn subsidiary will invest ₹3,000 crore and is estimated to create 12,000 jobs.
✓ Additionally, Foxconn will invest ₹2,000 crore for a semiconductor equipment manufacturing project.
✓ The project will be taken up in collaboration with Applied Materials and is expected to generate 1,000
jobs.
Q. Which bank has raised Rs 10,000 crore at a coupon rate of 7.54 percent through its third infrastructure
bond issuance?
A) Punjab National Bank B) State Bank of India
C) Kotak Mahindra Bank D) Bank of Baroda
Answer : B
✓ SBI raises 10,000 cr via 15-year Infrastructure bonds.

Follow us: Official Site, Telegram, Facebook, Instagram, Instamojo 272


✓ State Bank of India (SBI) has raised Rs 10,000 crore at a coupon rate of 7.54 percent through its third
infrastructure bond issuance.
✓ The proceeds will be directed towards bolstering long-term resources for funding critical infrastructure
projects and supporting the affordable housing segment.
✓ The bank attracted bids of Rs 21,698 crores, a staggering 4.34 times the base issue size, indicating robust
demand for these financial instruments.
✓ The investors range from various sectors including provident funds, pension funds, insurance companies,
mutual funds, corporates, etc.
✓ Bidding for the 15-year tenor bonds took place on July 31
Q. National Bank for Agriculture and Rural Development (NABARD) has sanctioned Rs 1,974.07 crore to
which state government under the Rural Infrastructure Development Fund (RIDF) for 2023-24?
A) Gujarat B) Madhya Pradesh
C) Rajasthan D) Chhattisgarh
Answer : C
✓ The National Bank for Agriculture and Rural Development (NABARD) has sanctioned Rs 1,974.07
crore to the Rajasthan government under the Rural Infrastructure Development Fund (RIDF) for 2023-
24.
✓ Rs 930.44 crore has been sanctioned for three rural drinking water supply projects in Ajmer, Jalore, and
Kota districts.
✓ Further, Rs 926.48 crore was sanctioned for the construction of 676 rural roads in the desert and tribal
areas of the state.
✓ What is Rural Infrastructure Development Fund (RIDF)?
✓ The RIDF was set up by the Government in 1995-96 for financing ongoing rural Infrastructure projects.
✓ The Fund is maintained by the National Bank for Agriculture and Rural Development (NABARD).
✓ Contribution: Domestic commercial banks contribute to the Fund to the extent of their shortfall in
stipulated priority sector lending to agriculture.
✓ Main Objective: To provide loans to State Governments and State-owned corporations to enable them
to complete ongoing rural infrastructure projects.
✓ Repayment period: Loan to be repaid in equal annual instalments within seven years from the date of
withdrawal, including a grace period of two years.
✓ About National Bank for Agriculture and Rural Development (NABARD)
✓ Founded : 12 July 1982
✓ Headquarters : Mumbai, Maharashtra
✓ Chairman : K V Shaji
✓ NABARD established on the recommendations of B. Sivaramman Committee.
✓ RBI had sold its stake and now the Government of India (GOI) has 99 percent in it.

Q. Which entity has plans to set up ‘Research Analyst Administration and Supervisory Body (RAASB)’?
A) RBI B) SEBI
C) NPCI D) SIDBI
Answer : B
✓ Capital Market Regulator SEBI plans to set up a Research Analyst Administration and Supervisory Body
(RAASB) to administer and supervise research analysts.
✓ This is along the lines of the Investment Adviser Administration and Supervisory Body set up for
administration and supervision of investment advisers.

Follow us: Official Site, Telegram, Facebook, Instagram, Instamojo 273


SEBI IN NEWS 2023
▪ SEBI Introduces Framework for Corporate Debt Market Development Fund (CDMDF).
▪ SEBI proposed that asset management companies set up surveillance and internal control systems for the
deterrence of possible market abuse and fraudulent transactions.
▪ SEBI plans to set up a Research Analyst Administration and Supervisory Body (RAASB) to administer
and supervise research analysts.
▪ SEBI has introduced a regulatory framework for online investment platforms that facilitate investments
in commission-free mutual fund (MF) schemes.
▪ SEBI sets up intermediary advisory panel to be led by S Ravindran.
▪ SEBI Introduced Risk disclosure Framework for Trading by Individual Traders in Equity F&O Segment.
▪ SEBI introduced the Legal Entity Identifier (LEI) system for issuers of non-convertible securities,
securitized debt instruments and security receipts.
▪ NDTV appoints former SEBI Chairman UK Sinha and business leader Dipali Goenka as Independent
Directors.
▪ Sebi introduces Application Supported by Blocked Amount (ASBA) -like facility for secondary market
trading in India.
▪ NSE gets the final SEBI approval to launch Social Stock Exchange.
▪ Markets regulator SEBI has launched an information database on municipal bonds.
▪ SEBI allowed Alternative Investment Funds(AIFs) to participate in the Credit Default
Swaps(CDS) market as protection buyers and sellers in a bid to facilitate the deepening of the domestic
corporate bond segment.
▪ Securities and Exchange Board of India (SEBI):
▪ It is the regulatory body for securities and commodity markets in India established in 1988.
▪ Its main objectives are to protect the interests of investors in securities and promote the development of
the securities market in India by regulating intermediaries, ensuring transparency and accountability of
listed companies, and promoting investor education.
▪ It is owned by the Ministry of Finance, Government of India, but functions as an autonomous body with
its own governing board.
▪ It has the power to issue guidelines, rules and regulations, investigate and punish fraudulent business
practices.
▪ It is the regulator of the Capital market and Commodities market in India.
▪ The first SEBI chairman was Dr S A Dave (1988-90)
▪ Madhabi Puri Buch is the current and 10th Chairperson of the SEBI.
▪ Headquarters : Mumbai
Q. Foxconn subsidiary inked ₹1,600 crore deal with which state to set up electronic components
manufacturing facility?
A) Maharashtra
B) Gujarat
C) Tamil Nadu
D) Karnataka
Answer : C
✓ Taiwanese firm Foxconn Technology Group, a key supplier to Apple Inc., signed a letter of Intent with
the Tamil Nadu government to invest ₹1,600 crore to establish a manufacturing facility in Kancheepuram
district.
✓ The letter of intent was signed in the presence of Chief Minister M.K. Stalin and Foxconn Chairman
Young Liu.
✓ The facility will potentially employ 6,000 people.

Follow us: Official Site, Telegram, Facebook, Instagram, Instamojo 274


Q. With reference to the recent announcement to allow the use of surety insurance bonds, consider the
following statements:
1.Surety bond is a legally binding contract.
2.These bonds are mainly aimed at infrastructure development.
Which of the statements given above is/are correct?
A) 1 only B) 2 only
C) Both 1 and 2 D) Neither 1 nor 2
Answer : C
✓ Recently, the National Highways Authority of India (NHAI) organised a brainstorming session with
stakeholders to expedite the adoption of Insurance Surety Bonds for its contracts.
✓ About Insurance Surety Bonds:
✓ These bonds can be defined in their simplest form as a written agreement to guarantee compliance,
payment, or performance of an act.
✓ These are instruments where insurance companies act as ‘Surety’ and provide the financial guarantee
that the contractor will fulfil its obligation as per the agreed terms.
✓ Surety is a unique type of insurance because it involves a three-party agreement.
✓ The three parties in a surety agreement are:
✓ Principal: The party that purchases the bond and undertakes an obligation to perform an act as promised.
✓ Surety: The insurance company or surety company that guarantees the obligation will be performed. If
the principal fails to perform the act as promised, the surety is contractually liable for losses sustained.
✓ Obligee: The party who requires and often receives the benefit of the surety bond. For most surety bonds,
the obligee is a local, state or federal government organisation.
Q. What is the total outlay of the expansion of ‘Digital India programme’?
A) Rs. 14,503 crores B) Rs. 14,903 crores
C) Rs. 15000 crores D) Rs. 15120 crores
Answer : B
✓ Union Cabinet, chaired by Prime Minister Narendra Modi, has approved the launch of an extended
version of the Digital India programme, aimed at further accelerating the country’s digital transformation
journey. The extended programme encompasses a wide range of initiatives spanning technology,
education, governance, and innovation with an additional outlay of Rs 14,903 crore.
✓ The ‘Digital India programme’ was launched on 1st July, 2015 to enable digital delivery of services to
citizens. The total outlay is ₹14,903 crores. The Union Cabinet recently approved the expansion of the
Digital India programme.
1. DIGILOCKER EXTENSION
In a bid to streamline documentation and enhance digital record-keeping, an extension of the DigiLocker
platform will be introduced, specifically tailored for Micro, Small, and Medium Enterprises (MSMEs)
and other organisations. This move is anticipated to improve operational efficiency and facilitate
seamless access to important documents.
1. FUTURESKILLS PRIME PROGRAMME
2. Acknowledging the rapidly evolving technological landscape, the FutureSkills Prime Programme is set
to re-skill and up-skill around 6.25 lakh IT professionals. The initiative aims to equip them with the
necessary expertise to excel in emerging technologies and contribute to the nation’s digital prowess.
3. ISEA PROGRAMME

Follow us: Official Site, Telegram, Facebook, Instagram, Instamojo 275


Under the Information Security & Education Awareness Phase (ISEA) Programme, 2.65 lakh individuals
will undergo comprehensive training in information security. This effort seeks to bolster the nation’s
cybersecurity framework and create a more secure digital environment for all citizens.
4. UMANG APP EXPANDS SERVICES
The Unified Mobile Application for New-age Governance (UMANG) app, which already offers over
1,700 services, is set to enhance its offerings with an additional 540 services. This expansion aims to
simplify access to government services and information through a single, user-friendly platform.
5. NATIONAL SUPERCOMPUTING MISSION GROWS
To further advance scientific research and computational capabilities, the National Supercomputing
Mission will see the addition of 9 new supercomputers. This expansion builds upon the existing 18
supercomputers already deployed across the country.
6. BHASHINI
The AI-enabled multi-language translation tool, Bhashini, currently available in 10 languages, will be
extended to cover all 22 Schedule 8 languages. This move is expected to break language barriers and
foster better communication across diverse linguistic communities
7. MODERNISATION OF NATIONAL KNOWLEDGE NETWORK
The National Knowledge Network, which interconnects 1,787 educational institutions, will undergo
modernisation to provide enhanced connectivity and collaboration opportunities for students and
researchers.
8. SUPPORTING STARTUPS
Under the extended version of the Digital India programme, 1,200 startups in tier 2 and 3 cities will
receive support, fostering innovation and entrepreneurship beyond metropolitan areas
9. CENTRE OF EXCELLENCE FOR AI APPLICATIONS
Three Centres of Excellence will be established, focusing on developing AI applications for health,
agriculture, and sustainable living in urban areas. These centres are poised to drive technological
advancements for societal benefit.
10. CYBERSECURITY AND CYBER AWARENESS INITIATIVES
A significant emphasis will be placed on cybersecurity and cyber awareness. Cyber awareness courses
will be made available for 12 crore college students, while initiatives will be taken to boost CERT-In
(Computer Emergency Response Team - India) and take this organisation to the next level. The
programme will also introduce new cybersecurity tools and integrate more than 200 sites with the
National Cyber Coordination Centre.

Follow us: Official Site, Telegram, Facebook, Instagram, Instamojo 276


Q. Who has been appointed as the new Managing Director (MD) of Life Insurance Corporation of India
(LIC)?
A) R. Doraiswamy B) Vibha Padalkar
C) Anup Bagchi D) Rakesh Jain
Answer : A
✓ Life Insurance Corporation of India (LIC), a public sector insurance firm, has announced the
appointment of Doraiswamy Ramachandran as the Managing Director, effective from September 1,
2023.
✓ Currently, Doraiswamy is serving LIC as the Executive Director at the head office in Mumbai. He will
be taking over from Mini Ipe, who is retiring as CEO and MD this month, after around 35 years of service
✓ Now, LIC has 4 MDs: Doraiswamy Ramachandran, Sat Pal Bhanoo; M Jagannath and Tablesh Pandey.

LIC IN NEWS 2023


▪ Doraiswamy Ramachandran named as Managing Director for LIC.
▪ SatPal Bhanu has been appointed as the Managing Director (MD) of Life Insurance Corporation of India
(LIC).
▪ Siddhartha Mohanty appointed as Chairman of LIC Until June 2024.
▪ LIC Acquires 6.7% Stake In Jio Financial Services Via RIL De-Merger Action.
▪ LIC Raises Stake in Tech Mahindra to 8.88% Through Open Market Transactions.
▪ Ratnakar Patnaik appointed as new Chief Investment Officer of LIC.
▪ Tablesh Pandey and M. Jagannath have been appointed as the managing directors of the Life Insurance
Company (LIC).
▪ Life Insurance Corporation of India (LIC) has launched LIC Jeevan Azad Plan.
▪ The minimum basic sum assured under LIC Jeevan Azad Plan is Rs 2 lakh and the maximum basic sum
assured is Rs 5 lakh. The policy can be taken for term of 15 to 20 years.

Follow us: Official Site, Telegram, Facebook, Instagram, Instamojo 277


▪ LIC to divest 60.72% share in IDBI Bank.
▪ LIC presently has four managing directors
▪ Life Insurance Corporation of India (LIC)
▪ Founded : 1956
▪ Headquarters : Mumbai
▪ Chairman : Siddhartha Mohanty
▪ It is India's largest life insurance company and also the country's largest investment company.
▪ Authorised Capital of LIC - Rs 25,000 Crore.
▪ At Rs 21,000 crore, LIC is India's largest IPO.
Q. The National Company Law Tribunal (NCLT) has approved the merger scheme of India’s homegrown
entertainment company Zee Entertainment Enterprises with which company?
A) Star Bharat B) TV18
C) Sony Pictures Networks D) India TV
Answer : C
✓ NCLT Approves $10 Billion Mega-Merger Between Zee and Sony.
✓ The National Company Law Tribunal (NCLT) has approved the merger scheme of India’s homegrown
entertainment company Zee Entertainment Enterprises with Culver Max Entertainment (erstwhile Sony
Pictures Networks India or SPNI).
✓ Objective – To create a $10-billion Indian media giant
Q. RBI has increased the per transaction limit for UPI Lite from existing Rs 200 to how much rupees to
promote digital transactions?
A) 500 B) 1000
C) 2000 D) 5000
Answer : A
✓ RBI has increased the offline transaction limit from 200 rupees to 500 rupees but the overall limit of small
value digital transaction limit is kept unchanged at 2,000 rupees.
✓ These initiatives will further deepen the reach of digital payments in the country.
✓ Central Bank has also proposed to use Near Field Communication (NFC) technology in UPI
transactions.
✓ What is UPI Lite?
✓ UPI Lite is an ‘on-device wallet’ feature that will allow users to make real-time small-value payments
without using a UPI PIN.
✓ How to use UPI Lite?
✓ First, you have to add funds to or load money into the wallet on the app, from your bank account. Then,
you can use those pre-loaded funds to pay from the wallet via UPI Lite.
✓ Do note that the total limit of UPI Lite balance for an on-device wallet will be Rs 2,000 at any point in
time.
✓ In many remote areas, the data network is patchy and RBI’s move of increasing the limit on UPI lite
transactions will help travellers in doing digital transactions.
✓ Further, RBI to introduce Near Field Communication (NFC) technology to facilitate offline payments.
To promote the use of UPI Lite, it is proposed to facilitate offline transactions using Near Field
Communication (NFC) technology.
✓ This feature will not only enable retail digital payments in situations where internet/telecom connectivity
is weak or not available, but it will also ensure speed, with minimal transaction declined.

Follow us: Official Site, Telegram, Facebook, Instagram, Instamojo 278


Q. Who has launched HSBC India’s partnerships aimed at innovation in Green Hydrogen?
A) Nirmala Sitharaman B) Piyush Goyal
C) Narendra Modi D) Rajnath Singh
Answer : A
✓ Finance Minister Nirmala Sitharaman has launched HSBC India’s partnerships aimed at innovation in
Green Hydrogen.
✓ That includes partnerships with IIT Bombay and Shakti Sustainable Energy Foundation to pursue
technological advancements to make green hydrogen more efficient, cost-effective, and scalable.
✓ These two partnerships, with total grant support of Rs 15 crore (USD 2 mn), will focus on innovation
projects that will help prioritize green hydrogen as a strategic alternate fuel, help in building a robust,
green hydrogen economy, and achieve the government’s vision of an energy-independent nation.
✓ Green hydrogen has a pivotal role to play as we counter climate change and work towards enabling a
low-carbon and self-reliant economy.
✓ Group Chairman of HSBC – Mark Tucker,
✓ CEO of HSBC India – Hitendra Dave

GREEN HYDROGEN IN NEWS 2023


▪ Cabinet has cleared India's National Green Hydrogen Mission, which aims to make the country a global
green hydrogen hub. The outlay of the mission is Rs 20,000 crore.

▪ NTPC Limited and Gujarat Gas Limited (GGL) have commissioned India’s first green hydrogen
blending project in the piped natural gas (PNG) network of NTPC Kawas, Gujarat.
▪ Himachal Pradesh government is currently formulating a Green Hydrogen Policy.
▪ The mission aims to achieve a production capacity of 5 million metric tons of hydrogen per year by 2030.
▪ India has announced the definition of Green Hydrogen. The Green Hydrogen Standard for India sets a
criterion of 2 kg CO2 equivalent per kg H2 as a 12-month average emission threshold.
▪ Tata Steel and ACME Group Join Forces For India’s Largest Green Hydrogen Project Odisha’s
Gopalpur Industrial Park.
▪ Ministry of New and Renewable Energy Sources has unveiled draft and roadmap for Green Hydrogen
Ecosystem.
▪ International Conference on Green Hydrogen (ICGH-2023) commenced in New Delhi, organized by the
Government of India. The conference aims to establish a Green Hydrogen ecosystem.
▪ European Investment Bank (EIB) has announced its support for India’s National Green Hydrogen
Mission by offering a loan facility of up to €1 billion.
▪ India’s top oil firm IOC will set up green hydrogen plants at all its refineries as it pivots a Rs 2-lakh crore
green transition plan to achieve net-zero emissions from its operations by 2046.
▪ Reliance Industries Limited (RIL) and Ashok Leyland unveiled India’s first Hydrogen Internal
Combustion Engine (H2-ICE) technology solution for heavy duty trucks.
▪ Indian PSU refiners to set up 137,000 tonnes per annum green hydrogen facility by 2030.
Q. Which bank has launched its ‘Infinity Savings Account’ to offer special features for its customers,
including no domestic transaction charges in exchange for monthly fees?
A) Axis Bank B) Yes Bank
C) HDFC Bank D) ICICI Bank
Answer : A

Follow us: Official Site, Telegram, Facebook, Instagram, Instamojo 279


✓ A private sector lender, Axis Bank has launched its ‘Infinity Savings Account’ to offer special features for
its customers, including no domestic transaction charges in exchange for monthly fees.
✓ The move aims to appeal to those who frequently adopt subscription-based models.
✓ This innovative savings account provides Axis Bank customers with exclusive privileges such as a waived
average monthly balance (AMB) requirement, complimentary debit cards, and a waiver of all domestic
charges against a small monthly recurring fee of Rs 150 or an annual fee of Rs 1,650.
✓ The Monthly plan is priced at Rs 150 (inclusive of GST) and requires a minimum subscription period of
six months. After the initial six months, the plan continues on a 30-day cycle, with Rs 150 deducted every
30 days.
✓ The Annual plan is priced at Rs 1,650 (inclusive of GST) and provides Infinity benefits for 360 days. The
plan automatically renews after this period
AXIS BANK IN NEWS 2023.
▪ Axis Bank Introduces ‘Infinity Savings Account’ with Zero Domestic Transaction Fees.
▪ Axis Bank partners with RBI Innovation Hub to launch Kisan Credit cards.
▪ Axis Bank Partners with Kiwi to Bolster ‘Credit on UPI’ on RuPay Credit Cards.
▪ Axis Bank has launched the ‘one-view’ feature on its mobile application which enables account
management across multiple bank accounts.
▪ Axis Bank has appointed NS Vishwanathan, former deputy governor of the Reserve Bank of India, as
non-executive chairman.
▪ Axis Bank launches Digital Onboarding platform ‘Sarathi’ for POS Terminals.
▪ Axis Bank and India Shelter Finance Corporation ltd announce strategic partnership under the co-lending
model.
▪ Axis Bank completes deal to buy Citibank’s India consumer business.
▪ Axis Bank Partners with OPEN to Launch a Fully Digital Current Account.
▪ AXIS BANK
▪ Founded : 1993
▪ HQ : Mumbai, Maharashtra
▪ Chairman : Rakesh Makhija
▪ MD & CEO : Amitabh Chaudhary
▪ Tagline : Badhti Ka naam Zindagi
Q. Which bank has launched a co-branded Corporate Credit Card in collaboration with spend management
platform Zaggle to empower businesses?
A) Yes Bank B) HDFC Bank
C) Axis Bank D) ICICI Bank
Answer : A
✓ Yes bank partnered with fintech Zaggle for next-gen corporate credit card.
✓ Yes Bank has launched co-branded Corporate Credit Card in collaboration with spend management
platform Zaggle, to empower businesses to reimagine their payment processes, streamline reconciliation,
and efficiently manage company expenditures.
✓ The new card ‘YES BANK Zaggle Corporate Credit Card’, provides businesses direct control over
company expenditures.

YES BANK IN NEWS 2023


▪ Yes bank partnered with fintech Zaggle for next-gen corporate credit card.
▪ YES Bank goes live with UPI interoperability on Central Bank Digital Currency (CBDC).
▪ Yes Bank Launches All-In-One ‘IRIS’ Mobile App.
▪ Reliance General Insurance becomes first insurer to accept Central Bank Digital Currency (CBDC) e-
Rupee (e₹) in tie-up with YES Bank

Follow us: Official Site, Telegram, Facebook, Instagram, Instamojo 280


▪ YES Bank announced the unveiling of its new logo, which forms part of its “refreshed brand identity.”
YES BANK launches new logo, to roll out campaign 'Life Ko Banao Rich'.
▪ YES Bank has collaborated with National E-Governance Services Limited (NeSL) to issue its first
electronic Bank Guarantee (e-BG).
▪ Cashfree Payments partners with YES Bank to offer Global Collections service for exporters.
▪ Yes Bank is an Indian bank headquartered in Mumbai, India and was founded by Rana Kapoor and
Ashok Kapoor in 2004.
▪ YES BANK
▪ Founded : 2004
▪ Founder : Rana Kapoor and Ashok Kapoor
▪ Headquarters : Mumbai, Maharashtra
▪ Chairman : Sunil Mehta
▪ MD & CEO : Prashant Kumar
▪ YES BANK launches new logo, to roll out campaign 'Life Ko Banao Rich'.
Q. Tata Power Renewable Energy Limited has tied up with which state for a 28.12 MW green energy
project?
A) Uttar Pradesh B) Bihar
C) Maharashtra D) Himachal Pradesh
Answer : C
✓ Tata Power Renewable Energy Limited (TPREL) has partnered with Sanyo Special Steel Manufacturing
India Pvt Ltd (SSMI) to set up a 28.12 MW green energy plant in Maharashtra.
✓ TPREL, an arm of Tata Power, has signed a Power Delivery Agreement (PDA) with Sanyo through TP
Alpha Limited.
✓ This plant will be set up at Achegaon in Solapur district of Maharashtra
Q. ADB and Fourth Partner Energy Collaborate for Solar Power Plant in Which State?
A) Kerala B) Karnataka
C) Tamil Nadu D) Andhra Pradesh
Answer : C
✓ The Asian Development Bank (ADB) and Fourth Partner Energy Private Limited (Fourth Partner)
signed a long-term loan of up to 1.2 billion Indian rupees (about $14.7 million) to construct and operate
a 25-megawatt solar photovoltaic-based power plant to increase clean and low-cost energy supply to
commercial and industrial customers.
✓ The power plant, located in Tirunelveli district in the state of Tamil Nadu in India, is expected to generate
about 50.7 gigawatt-hours of electricity annually and will directly sell power to commercial and industrial
users.
✓ India’s progressive open access renewable energy policy allows a group of consumers to purchase
electricity directly from an Independent Power Producer (IPP).

ADB IN NEWS 2023


▪ The Asian Development Bank (ADB) and Fourth Partner Energy Private Limited (Fourth Partner)
signed a long-term loan of up to 1.2 billion Indian rupees (about $14.7 million) .
▪ ADB Approves USD 40.5 Million Loan to Enhance Childhood Development and Maternal Mental
Health in Meghalaya.
▪ Government of India and Asian Development Bank (ADB) signed a $200 million loan as additional
financing for the ongoing Rajasthan.
▪ ADB approves $295mn loan to improve Transport Connectivity in Bihar.

Follow us: Official Site, Telegram, Facebook, Instagram, Instamojo 281


▪ ADB gives 130 million dollars to increase horticulture in Himachal Pradesh.
▪ Asian Development Bank (ADB) and the Government of India signed a $141.12 million loan agreement
for the development of an industrial corridor in Andhra Pradesh.
▪ Alok Kumar has been appointed as a member of the Asian Development Bank’s (ADB) High-level
Advisory Group on Digital Technology for Development.
▪ Pakistan is the largest recipient of ADB loans in Asia, Pakistan received loans of USD 5.58 billion.
▪ Asian Development Bank (ADB) has announced its plans to invest in non-convertible debentures valued
at Rs 150 crore in Tata Power Delhi Distribution Ltd (TPDDL).
▪ ADB approves $130 million loan to promote horticulture in Himachal Pradesh.
▪ Indian government and the Asian Development Bank (ADB) signed loan agreements aggregating to
$1.22 billion.
▪ ASIAN DEVELOPMENT BANK (ADB)
Founded : 19 December 1966
HQ : Mandaluyong, Metro Manila, Philippines
President : Masatsugu Asakawa (Japan)
Vice President : Ashok Lavasa (Indian?
▪ Director of ADB for India : Takeo Konishi
▪ Executive Director : Sameer Kumar
▪ Member Counrtries : 68 (Nuie).
▪ ADB has grown to encompass 68 members; of which 49 are from within Asia and the Pacific and 19
outside.
▪ Japan is the largest contributor in the subscribed capital of the country.
▪ Asian Development Outlook, reports published by ADB.
▪ It is a regional multilateral financial institution focused on countries in the Asia and Pacific region.
▪ It currently has 68 members – of which 49 are from the Asia and Pacific region and 19 from outside.
Q. Which IT company has bagged a €€1.5 billion ($1.64 billion) deal from Liberty Global to build and scale
the entertainment and connectivity platforms of the digital communications firm?
A) Wipro Limited B) Larsen & Toubro Infotech Limited
C) Infosys D) Tata Consultancy Services
Answer : C
✓ Indian IT services company, Infosys has bagged a €€1.5 billion ($1.64 billion) deal from Liberty Global
to build and scale the entertainment and connectivity platforms of the digital communications firm.
✓ The agreement has an initial tenure of five years which can be extended to eight years, which would take
the deal value to €€2.3 billion ($2.5 billion).
✓ Infosys’ deal follows, HCLTech, which last week a bagged a $2.1- billion contract from Verizon.
✓ Three months back, Tata Consultancy Services (TCS)-led consortium also received an order of Rs 15,000
crore ($1.8 billion) from BSNL, for the deployment of a 4G network across India.
✓ INFOSYS
✓ Founded : 1981
✓ Headquarters : Bangalore, Karnataka
✓ Chairman : Nandan Nilekani
✓ MD & CEO : Salil Parekh
Q. Asian Development Bank (ADB) has approved USD 40.5 million loan for childhood development in
Which State?
A) Meghalaya B) Nagaland
C) Manipur D) Arunachal Pradesh
Answer : A

Follow us: Official Site, Telegram, Facebook, Instagram, Instamojo 282


✓ The Asian Development Bank (ADB) has approved a USD 40.5 million loan for integrated early
childhood development and maternal mental health in the northeastern state of Meghalaya.
✓ The Meghalaya government is contributing USD 15.27 million to the project.
✓ The project is expected to strengthen home-based childcare (children 0-1.5 years) and center-based
childcare (1.5-6 years) through daycare (Anganwadi) centers in Meghalaya.
✓ Project Aim : To improve access to nurturing care, including a component of maternal mental health
care and group-based parenting programs to enable the inclusion of fathers in caregiving
Q. Ministry of Defence signed 19,000 crore contract with which company for five Fleet Support Ships for
Indian Navy?
A) HAL B) BDL
C) Infosys D) HSL
Answer : D
✓ Ministry of Defence signed a contract with Hindustan Shipyard Limited (HSL), Visakhapatnam for
acquisition of five Fleet Support Ships (FSS) for the Indian Navy at an overall cost of Rs 19,000 crore. It
would be a major boost towards achieving the goal of self-reliance in defence manufacturing as these
ships will be indigenously designed and constructed by HSL, Visakhapatnam
Q. Who has been appointed as the 1st woman Chairman and Managing Director (CMD) of Power Finance
Corporation (PFC) Limited?
A) Parminder Chopra B) Revathi Advaithi
C) Jayshree Ullal D) Padmasree Warrior
Answer : A
✓ Parminder Chopra was appointed as the next Chairman and Managing Director (CMD) of Power
Finance Corporation (PFC).
✓ Parminder Chopra is the first woman appointed as the Chairman and Managing Director (CMD) of a
Maharatna company.
✓ Joined PFC in 2005 and currently serves as Director (Finance) & CFO.
✓ Power Finance Corporation (PFC):
✓ Incorporated in 1986, Largest infrastructure finance company for power sector.
✓ Ministry of Power, under the administrative control of the Department of Public Enterprises.
✓ Awarded 'Maharatna' status, 11th PSU to receive this recognition.
Q. Who has been appointed as MD and CEO of South Indian Bank, with effect from 1st October 2023?
A) PR Seshadri B) B. Ramesh Babu
C) N. Kamakodi D) Abhijit Chakravorty
Answer : A
✓ PR Seshadri appoints as MD and CEO of South Indian Bank.
✓ The Reserve Bank of India has approved the appointment of PR Seshadri as Managing Director & CEO
of the South Indian Bank for a period of three years with effect from 1 October 2023.
✓ He has served, amongst other positions, as managing director & CEO of Karur Vysya Bank
✓ South Indian Bank
✓ Founded : 1929
✓ Headquarters : Thrissur, Kerala
✓ Chairman : Salim Gangadharan
✓ MD & CEO : Murali Ramakrishnan

Follow us: Official Site, Telegram, Facebook, Instagram, Instamojo 283


✓ Tagline : Experience Next Generation Banking.
Q. Based on the FY23 revised budget estimates, which state secured the top position as the best state in
overall fiscal health in a report conducted by a foreign brokerage firm?
A) Punjab B) Bihar
C) Tamil Nadu D) Maharashtra
Answer : D
✓ Maharashtra scored top position in fiscal health report.
✓ Based on the FY23 revised budget estimates Maharashtra secured the top position as the best state in
overall fiscal health in a report conducted by a foreign brokerage firm.
✓ Chhattisgarh, one of the poorest states in the country, is in 2nd position followed by Telangana and
Jharkhand, while the bottom 3 are Bengal, Punjab, and Kerala
Q. Which bank has partners with the RBI innovation hub to launch KCCs and MSME loans?
A) Yes Bank B) Axis Bank
C) ICICI Bank D) HDFC Bank
Answer : B
✓ Private lender Axis Bank has launched two lending products powered by the Public Tech Platform for
Frictionless Credit (PTPFC), introduced by the Reserve Bank Innovation Hub (RBIH).
✓ RBIH is a wholly-owned subsidiary of the Reserve Bank of India (RBI).
✓ Axis Bank will offer Kisan Credit Card (KCC) and unsecured micro, small and medium enterprises
(MSME) loans to customers powered by the platform.
✓ Both products will be offered in a completely digital manner and require no submission of documents by
customers.
✓ As a pilot, Kisan Credit Cards will be offered in Madhya Pradesh and will be available to customers for
up to Rs 1,60,000 to start with.
✓ MSME loans will be available across the country and will offer loans up to Rs 10 lakh to customers.
✓ As part of the pilot, Axis Bank will leverage the PTPFC to access customer data. These include PAN
validation, Aadhaar eKYC, account aggregator data, verification of land records and penny drop service
to validate bank accounts.
✓ Reserve Bank Innovation Hub (RBIH) recognized the problem, engaged with various banks to explore
possible solutions, and developed a blueprint to resolve the issue.
✓ Shri Shaktikanta Das, Governor of RBI has inaugurated the Reserve Bank Innovation Hub (RBIH) on
March 24, 2022 in Bengaluru, Karnataka
✓ RBI has set up the RBIH with an initial capital contribution of Rs. 100 crore.
✓ The objective of RBIH is to encourage and nurture financial innovation in a sustainable manner through
an institutional set-up. It has been setup as a Section 8 company under Companies Act, 2013.
✓ Reserve Bank Innovation Hub (RBIH) is one of the five wholly-owned subsidiaries of the RBI.
Q. Which entity has signed an a MoU with Government, Revenue Target for 2023-24 set at ₹ 4,350?
A) REC B) NTPC Limited
C) THDC India Limited D) IREDA
Answer : D
✓ Indian Renewable Energy Development Agency (IREDA), a Mini Ratna (Category – I) Government of
India enterprise under the administrative control of Ministry of New and Renewable Energy, has signed

Follow us: Official Site, Telegram, Facebook, Instagram, Instamojo 284


a performance-based Memorandum of Understanding (MoU) with the Ministry of New and Renewable
Energy, Government of India.
✓ The MoU, in alignment with guidelines issued by Department of Public Enterprises, Ministry of Finance,
outlines strategic targets that IREDA aims to achieve during the fiscal years 2023-24 and 2024-25.
✓ As per the MoU, the Government of India has set a Revenue from Operations target for IREDA, of ₹
4,350 crores for the financial year 2023-24 and ₹ 5,220 crores for 2024-25.
✓ Notably, the company had achieved a Revenue from Operations figure of ₹ 3,482 crores in the preceding
fiscal year, against a target of ₹ 3,361 crores
Q. Which bank has launched its state-of-the-art mobile banking app ‘iris’ to offer digital banking solutions
in India?
A) Axis Bank B) Yes Bank
C) ICICI Bank D) HDFC Bank
Answer : B
✓ Private lender YES Bank has launched its state-of-the-art mobile banking app ‘iris’ by YES Bank.
✓ The iris app has a sleek and intuitive user interface that aims to offer a simplified banking experience for
users of all age groups, and backgrounds.
Q. Which bank has recently launched the UPI Interoperable Digital Rupee mobile application as a part of
the Reserve Bank of India’s Central Bank Digital Currency (CBDC) pilot project?
A) Kotak Mahindra Bank B) Canara Bank
C) Indian Overseas Bank D) Indian Bank
Answer : B
✓ Canara Bank has launched the UPI Interoperable Digital Rupee mobile application as a part of the
Reserve Bank of India’s Central Bank Digital Currency (CBDC) pilot project.
✓ It is the first bank in both the public and commercial sectors to offer this feature via its CBDC mobile app
called the Canara Digital Rupee app.
✓ With the introduction of this feature, customers can scan merchant UPI QR codes using the Canara
Digital Rupee app and pay through digital currency.
✓ Moreover, the feature is aimed at empowering merchants to accept digital currency payments using their
existing UPI QR codes in addition to UPI-based payments without the requirement of a separate
onboarding process for CBDC.
✓ The Reserve Bank of India (RBI) has issued a concept note on Central Bank Digital Currency (CBDC)
on October 7, 2022.
✓ RBI has launched pilots of CBDC in both Wholesale and Retail segments. The pilot in wholesale
segment, known as the Digital Rupee -Wholesale (e₹-W), was launched on November 1, 2022, with use
case being limited to the settlement of secondary market transactions in government securities.
✓ The pilot in retail segment, known as digital Rupee-Retail (e₹-R), was launched on December 01, 2022,
within a closed user group (CUG) comprising participating customers and merchants.
✓ The first phase includes four banks, namely the State Bank of India, the ICICI Bank, the Yes Bank and
the IDFC First Bank. Subsequently, another four banks, viz., the Bank of Baroda, the Union Bank of
India, the HDFC Bank and the Kotak Mahindra Bank will participate in the retail pilot.
CANARA BANK IN NEWS 2023
▪ Canara Bank Pioneers UPI-Interoperable Digital Rupee Mobile App for CBDC Transactions.
▪ Public sector bank Canara Bankwill be signing a ₹3,000 crore IT transformation deal to revamp and
enhance its tech infrastructure.

Follow us: Official Site, Telegram, Facebook, Instagram, Instamojo 285


▪ Canara Bank Tops Public Sector Banks in Lending to State PSUs and Corporations for Fifth Consecutive
Year.
▪ Canara Bank, in partnership with the Reserve Bank Innovation Hub (RBIH), has introduced a new
customer-friendly service called “Digitalised Submission of Form 15G/15H,” which is aimed at
enhancing customer satisfaction.
▪ Canara Bank, in collaboration with the National Payments Corporation of India (NPCI), has announced
that its customers can now utilize their RuPay Credit Cards on the Unified Payments Interface (UPI)
platform.
▪ Canara Bank and NPCI Bharat BillPay Ltd (NBBL) have collaborated to launch a service that allows
Indians residing in Oman to make payments for their bills in India.
▪ Canara Bank said it plans to sell its stake in Russian Joint Venture(JV) Commercial Indo Bank LLC
(CIBL) to the other venture partner State Bank of India (SBI) for about Rs 114 crore.
▪ CIBL, incorporated in 2003, is a joint venture in Russia between SBI (60 per cent) and Canara Bank (40
per cent).
▪ K Satyanarayana Raju named as new MD and CEO of Canara Bank.
▪ He will be replacing L V Prabhakar.
▪ CANARA BANK
▪ It is a leading commercial bank of India.
▪ It was established in 1906, by Sri Ammembal Subba Rao Pai.
▪ Syndicate Bank was merged with Canara Bank on 1 April 2020. After this merger, Canara Bank has
become the fourth largest bank in the country.
▪ Headquarters Bangalore, Karnataka
▪ MD & CEO : K. Satyanarayana Raju
Q. Which institution has recently approved USD 25 million proposal submitted by the India’s Department
of Animal Husbandry & Dairying?
A) ADB B) IMF
C) World Bank D) G20 Pandemic Fund
Answer : D
✓ The G20 Pandemic Fund has approved a $25 million proposal submitted by India’s Department of
Animal Husbandry & Dairying, Ministry of Fisheries, Animal Husbandry & Dairying (DAHD),
Government of India on “Animal Health Security Strengthening in India for Pandemic Preparedness
and Response”.
✓ The G20 Pandemic Fund was established under Indonesia’s G20 Presidency, which finances critical
investments to strengthen pandemic prevention, preparedness, and response capacities at national,
regional, and global levels of low- and middle-income countries.
✓ Historical Background of G20 Summit
✓ The Group owes its origin to the Financial Crisis in 1997-98 of the Asian Tigers (Countries of East and
Southeast Asia), which caused its establishment in 1999. It worked first as a forum for the Central Bank
Governors and Finance Ministers of the major industrialised and developing economies to discuss global
economic and financial stability
✓ G20 Countries List
✓ G20 comprises 19 countries, namely Argentina, Australia, Brazil, Canada, China, France, Germany,
India, Indonesia, Italy, Japan, the Republic of Korea, Mexico, Russia, Saudi Arabia, South Africa,
Turkiye, the United Kingdom, the United States and the European Union.
✓ These members account for around 85% of the world GDP, 75% of the total international trade, and two-
thirds of the global population.
✓ In addition to these member countries, the G20 each year invites guest countries and international
organisations such as the United Nations, World Bank, IMF, OECD, ASEAN, etc., to participate in its
meetings

Follow us: Official Site, Telegram, Facebook, Instagram, Instamojo 286


✓ The G20 consists of two parallel tracks called the Finance Track and the Sherpa Track
✓ India's G20 Priorities
✓ Green Development, Climate Finance and Lifestyle for Environment (LiFE).
✓ Accelerated, Inclusive & Resilient Growth.
✓ Accelerating progress on SDGs.
✓ Technological Transformation and Digital Public Infrastructure.
✓ Multilateral Institutions for the 21st century.
✓ Women-led Development.
Q. The Central Vigilance Commission (CVC) has reconstituted the Advisory Board on Banking and
Financial Frauds (ABBFF) which conducts the first-level examination of bank frauds before
recommendations or references are made to investigative by agencies such as the CBI. The reconstituted
APBFF will be chaired by __.
A) Abhijit Sen B) M Rajeshwar Rao
C) Suresh N Patel D) Abid Hussain
Answer : C
✓ The Central Vigilance Commission (CVC) has reconstituted the Advisory Board on Banking and
Financial Frauds (ABBFF) that conducts the first-level examination of bank frauds before
recommendations or references are made to investigative by agencies such as the Central Bureau of
Investigation (CBI).
✓ The reconstituted APBFF will be chaired by Suresh N Patel, the former Central Vigilance Commissioner.
✓ Besides the Chairman, there are four Members and the tenure of the Chairman/ Members would be for
two years with effect from August 21, according to an order issued by the CVC.
✓ The ABBFF has been empowered to examine the role of officials/ wholetime directors (including ex-
officials/ ex-wholetime directors) in public sector banks, public sector insurance companies; and public
sector financial institutions in case of frauds amounting to ₹ 3 crore and above.
✓ The CVC has now stipulated that all PSBs, public sector insurance companies, and public sector financial
institutions should refer all matters of frauds involving ₹3 crore and above to the board for advice
Q. The International Cricket Council (ICC) has a brand tie-up with which company for the upcoming Men’s
Cricket World Cup 2023 to be held in India between 5 October and 19 November 2023?
A) Mastercard B) Visa Inc
C) Paytm D) VIVO
Answer : A
✓ The International Cricket Council (ICC) has a brand tie-up with Mastercard for the upcoming Men’s
Cricket World Cup 2023 to be held in India between 5 October and 19 November 2023.
✓ Under this tie-up, Mastercard will be a global partner for the World Cup.
✓ With this, Mastercard joins ICC’s other global partners MRF Tyres, Booking.com, Aramco and
Emirates.
✓ The ICC has three categories of sponsors — global partners, official partners and category partners.
Sponsors get multiple benefits like media integration, in-stadium branding, tickets, and hospitality.
✓ Incidentally, MasterCard was also the title sponsor for all bilateral matches played in India until March.
It was paying Rs 3.8 crore per match as the Indian cricket board’s title sponsor.
✓ MASTERCARD
✓ Founded : 1966
✓ Headquarteres : New York, U.S
✓ Executive Chairman : Ajay Banga
✓ President and CEO : Michael Miebach

Follow us: Official Site, Telegram, Facebook, Instagram, Instamojo 287


Q. Which bank has partnered with Marriott Bonvoy to launch the ‘Marriott Bonvoy Credit Card’?
A) ICICI Bank B) Yes Bank
C) HDFC Bank D) Axis Bank
Answer : C
✓ HDFC Bank launches India’s First Co-Branded Hotel Credit Card with Marriott.
✓ HDFC Bank has partnered with Marriott Bonvoy, the esteemed travel program by Marriott, to launch
‘Marriott Bonvoy HDFC Bank Credit Card’.
✓ This pioneering co-branded card will operate on the Diners Club platform, which is a part of the globally
recognized Discover Global Network.
✓ The newly launched credit card comes packed with travel benefits, designed to elevate cardholders’ travel
journeys.
Q. Which bank has partnered with commodities trader Maptrasco for the completion of the first “live”
transaction of electronic Bills of Lading (eBL) for shipments?
A) HSBC Bank B) DBS Bank
C) Citibank D) Standard Chartered Bank
Answer : B
✓ DBS Bank has partnered with commodities trader Maptrasco for the completion of the first “live”
transaction of electronic Bills of Lading (eBL) for shipments between Singapore and India.
✓ With this first shipment, businesses across Singapore and India can participate in the TradeTrust network
via their respective choice of TradeTrust-enabled platforms to enable the digital trade finance of live
shipments of goods between the two countries.
✓ The transaction was implemented under the TradeTrust framework, an initiative by the Infocomm Media
Development Authority (IMDA) and supported by Enterprise Singapore.
✓ Development Bank of Singapore (DBS) Bank
✓ Founded : 1968
✓ Headquarters : Singapore
✓ Chairman : Peter Seah
✓ CEO : Piyush Gupta

Q. Who has acquired the title sponsorship rights for the domestic matches of the Indian cricket team?
A) Master Card B) Star Sports
C) IDFC First Bank D) Paytm
Answer : C
✓ IDFC First Bank has bagged the title sponsorship rights for all the domestic matches of the Indian cricket
team.
✓ Under this, the bank will have to pay an amount of ₹ 4.2 crore for each international match.
✓ BCCI said that this contract will be for a long period of three years which will start with the India-
Australia ODI series
✓ IDFC First Bank
✓ Founded : October 2015
✓ Headquarters : Mumbai, Maharashtra
✓ Non-Exe. Chairman : Rajiv Lall
✓ MD & CEO : V. Vaidyanathan
✓ Tagline : Always You First.

Follow us: Official Site, Telegram, Facebook, Instagram, Instamojo 288


Q. Which company has successfully manufactured the first set of indigenous SCR Catalysts for limiting
NOx emissions from thermal power plants?
A) Bharat Heavy Electricals Limited (BHEL)
B) Bharat Petroleum Corporation Limited (BPCL)
C) Indian Oil Corporation Limited (IOCL)
D) Hindustan Petroleum Corporation Limited (HPCL)
Answer : A
✓ BHEL develops India’s first catalyst set to curb NOx emissions in power plants
✓ State-owned engineering firm Bharat Heavy Electricals Ltd (BHEL) has successfully manufactured the
first set of indigenous SCR Catalysts for limiting NOx emissions from thermal power plants.
✓ SCR Catalysts were earlier imported in line with the ‘Make in India’ initiative.
✓ The initial batch of indigenous SCR Catalyst was manufactured for 5×800 MW Yadadri Thermal Power
Station in Telangana.
✓ BHEL set up a manufacturing facility at its Solar Business Division unit to produce SCR catalysts for
reducing NOx emissions in thermal power plants.
✓ Burning coal converts its nitrogen content into nitrogen oxides (NOx), a major air pollutant that includes
substances such as NO, NO2, and N2O.
✓ Other orders for SCR units came from Maharashtra State Power Generation Company Limited
(MAHAGENCO), West Bengal Power Development Corporation Limited (WBPDCL), and NALCO
for various thermal power stations
✓ Bharat Heavy Electricals Limited (BHEL):
✓ It is a major Central Public Sector Enterprise in India.
✓ It has the distinction of being the largest manufacturer of power generation equipment under government
ownership.
✓ BHEL works as a part of the Government of India and comes under the purview of the Ministry of Heavy
Industry.
✓ Establishment - 1956
✓ Headquarters - New Delhi
Q. Which bank has launched a Customer Service Points (CSPs) functionality that allows customers to enroll
for social security schemes through Aadhaar Card?
A) Punjab National Bank
B) Indian Overseas Bank
C) State Bank of India
D) Kotak Mahindra Bank
Answer : C
✓ Public sector lender the State Bank of India has launched a Customer Service Points (CSPs) functionality
that allows customers to enroll for social security schemes through Aadhaar Card.
✓ Chairman Dinesh Khara introduced the feature which will be availed at the bank’s CSPs that aims to
make the process of enrolling in various social security schemes simpler.
✓ The customers visiting CSPs will need only their Aadhaar to enroll in social security schemes such as
Pradhan Mantri Jeevan Jyoti Bima Yojana (PMJJBY), Pradhan Mantri Suraksha Bima Yojana
(PMSBY) and Atal Pension Yojana (APY).
✓ They will no longer need to carry passbooks to the CSP outlet for enrolment.

Follow us: Official Site, Telegram, Facebook, Instagram, Instamojo 289


Q. The Centre has formed a nine-member committee to review the functioning of the Defence Research and
Development Organization (DRDO). Who will be chaired by this committee?
A) V K Paul B) KV Subramanian
C) K Vijay Raghavan D) Usha Thorat
Answer : C
✓ The Centre has formed a nine-member committee to review the functioning of the Defence Research and
Development Organisation (DRDO).
✓ The committee headed by Prof K Vijay Raghavan to review and redefine the role of the department and
submit a report within three months.
✓ Prof Vijay Raghavan is a former Principal Scientific Advisor to the Government of India and one of the
key architects of the National Research Foundation (NRF).
✓ The members also include a retired diplomat, an ISRO scientist and a serving officer.
✓ The committee would suggest measures to revamp DRDO to make it more result-oriented.

COMMITTEE IN NEWS 2023


▪ Union Minister Rajnath Singh, Ministry of Defence (MoD) has set up a 9-member committee of experts
to maintain the Defence Research and Development Organization (DRDO).
▪ The committee will be headed
▪ by Prof K Vijay Raghavan.
▪ Indian Banks’ Association (IBA) has formed a five-member working group for the Implementation of
recommendations from the Reserve Bank of India’s committee on the working of Asset Reconstruction
Companies (ARCs).
▪ The committee chaired by Sudarshan Sen
▪ Government of India has constituted an expert panel headed by former Law Secretary TK Vishwanathan
to propose reforms in the Arbitration and Conciliation Act and reduce the burden on the courts.
▪ National Tiger Conservation Authority (NTCA) has established an 11-member Cheetah Project Steering
Committee under the Ministry of Environment, Forest and Climate Change, and Rajesh Gopal, secretary
general of Global Tiger Forum, has been appointed as its chairman.
▪ Government of India has constituted a Peace Committee in Manipur under the Chairpersonship of
Governor, Manipur Anusuiya Uikey.
▪ IRDAI has recently constituted a five-member expert committee comprising medical experts and insurers
to advise on matters related to mental health and insurance; Headed By : Pratima Murthy.
▪ Razorpay forms 4-member advisory board to strengthen corporate governance; Head-former RBI deputy
governor NS Vishwanathan.
▪ Ministry of Finance has set up of a four-member committee forms panel to suggest changes to National
Pension Scheme (NPS); Headed By- Finance Secretary T.V. Somanathan.
▪ Ministry of Housing and Urban Affairs has set up a 14-member committee to look into stalled real estate
projects; Headed By – Amitabh Kant.
▪ Ministry of Electronics and Information Technology (MeitY) has constituted a nine-member task force
to make India a ‘product developer and manufacturing nation’.Chaired By - Bhuvnesh Kumar.
▪ International Financial Services Centres Authority (IFSCA) has formed an expert committee to create a
roadmap to encourage Indian startups/fintech domiciled abroad to relocate to the GIFT City in Gujarat.
Chaired By - G. Padmanabhan.
Q. Which state government has signed an MoU with TCPL Green Energy Solutions Private Ltd. (TGESPL)
to set up the country’s “first” hydrogen fuel project at an estimated cost of over Rs 350 crore?
A) Odisha B) Jharkhand
C) Madhya Pradesh D) Uttar Pradesh

Follow us: Official Site, Telegram, Facebook, Instagram, Instamojo 290


Answer : B
✓ The Jharkhand government has signed an MoU with TCPL Green Energy Solutions Private Ltd.
(TGESPL) to set up the country’s “first” hydrogen fuel project in Jamshedpur at an estimated cost of
over Rs 350 crore.
✓ The capacity of the proposed unit will be over 4,000 hydrogen IC engine/fuel agnostic engine and more
than 10,000 battery system.
✓ The facility is expected to start commercial production in March 2024, and likely provide employment
to about 1,000 people directly or indirectly.
Q. According to a research report by Standard Chartered Bank, the per capita income of India is likely to
grow around 70% by which year?
A) 2030 B) 2025
C) 2040 D) 2035
Answer : A
✓ According to a research report by Standard Chartered Bank, the per capita income of India is likely to
grow around 70% by 2030 and is expected to reach $4,000 from current levels of $2,450.
✓ The boost in income will help the country to become a middle-income economy with a GDP of $6 trillion
and half of it will be coming from household consumption.
✓ Since 2001, the per capita GDP grew from $460 to $1,413 in 2011 to $2,150 in 2021.
✓ The primary factor propelling significant economic growth will be external trade, which is anticipated to
nearly double to $2.1 trillion by 2030.
✓ This substantial increase would be a remarkable jump from the $1.2 trillion recorded in fiscal 2023 when
the GDP stood at $3.5 trillion.
✓ The report assumes an annual nominal GDP growth of 10% going forward.
✓ Telangana currently takes the top spot in the per capita income rankings with ₹2,75,443 (equivalent to
$3,360).
✓ Following closely are Karnataka with ₹2,65,623, Tamil Nadu with ₹2,41,131, Kerala with ₹2,30,601,
and Andhra Pradesh with ₹2,07,771.
✓ Standard Chartered plc is a British multinational bank with operations in consumer, corporate and
institutional banking, and treasury services.
✓ Headquarters
✓ London, England, UK

Q. India has signed a US$ 200 million loan with ADB to enhance urban services in which state?
A) Haryana B) Rajasthan
C) Maharashtra D) Gujarat
Answer : B
✓ Govt of India and the Asian Development Bank (ADB) have signed a $200 million loan as additional
financing in Rajasthan.
✓ The project will incorporate various innovative and climate-resilient solutions for expanding basic urban
services and incorporate nature-based solutions to rehabilitate heritage structures besides piloting public-
private partnerships in the state’s water and sanitation sector to deepen private sector engagement.
Q. A new report has revealed that by which year Metaverse users will exceed 600 million due to generative
artificial intelligence (AI)?
A) 2030 B) 2025
C) 2026 D) 2027

Follow us: Official Site, Telegram, Facebook, Instagram, Instamojo 291


Answer : C
✓ A new report has shown that Metaverse users will surpass 600 million by 2026 due to generative artificial
intelligence (AI).
✓ About Metaverse:
✓ It refers to a virtual or digital universe where people can interact with each other and digital objects in a
shared online space.
✓ Origin of the term: The term “metaverse” first appeared in author Neal Stephenson’s 1992 science-fiction
novel Snowcrash, which describes a future where millions of people use virtual avatars to participate in
a cyberspace realm.

Q. Which Ministry is implementing the scheme of "Rashtriya Vayoshri Yojana"?


A) Ministry of Tribal Affairs B) Ministry of Rural Development
C) Ministry of Housing and Urban Affairs D) Ministry of Social Justice and Empowerment
Answer : D
✓ The Department of Social Justice and Empowerment is implementing the scheme of “Rashtriya Vayoshri
Yojana(RVY)” with the objective to provide the senior citizens, suffering from any of the age related
disability/infirmity, with assisted living devices which can restore near normalcy in their bodily
functions, overcoming the disability/infirmity manifested such as low vision, hearing impairment, loss
of teeth and loco-motor disabilities.
✓ Funds are released to the Artificial Limbs Manufacturing Corporation(ALIMCO), a Public Sector
Undertaking under the administrative control of the Ministry of Social Justice and Empowerment, as a
sole implementing agency of the scheme.
✓ The scheme was launched on 01.04.2017 for providing physical aids and assisted-living devices to senior
citizens belonging to the Below Poverty Line(BPL) category. The scheme has been revised in the year
2020-21.
✓ After the revision, the assisted-living devices are being provided to the senior citizens belonging to the
BPL category and also to the senior citizens with monthly income not more than Rs.15000/-.
Q. How many loans have been sanctioned under Pradhan Mantri MUDRA Yojana (PMMY) in Financial
Year 2022-23?
A) 3.78core B) 5.91 core
C) 7.12 core D) 6.23 core
Answer : D
✓ Over 6.23 core loans sanctioned under Pradhan Mantri MUDRA Yojana.
✓ Over 6 core 23 lakh loans have been sanctioned under Pradhan Mantri MUDRA Yojana (PMMY) in
Financial Year 2022-23.
✓ The Union Minister for State for Finance, Dr. Bhagwat Kisanrao Karad, shared this information in a
written reply during a Lok Sabha session.
✓ The objective of PMMY is to provide access to institutional finance to new or existing micro units or
enterprises up to 10 lakh rupees.
✓ The complaints with regard to the implementation of PMMY are redressed in consultation with the
respective Banks
✓ The complaints received at the Centralised Public Grievance Redress and Monitoring System are also
being taken up with respective Banks for redressal within the prescribed timeline.
✓ Mudra loans under Pradhan Mantri Mudra Yojana (PMMY) can be availed of from a nearby branch
office of a bank.
✓ Pradhan Mantri MUDRA Yojana (PMMY) – MUDRA Scheme

Follow us: Official Site, Telegram, Facebook, Instagram, Instamojo 292


✓ Definition : Pradhan Mantri Mudra Yojana (PMMY) is a scheme to extend collateral free loans.
✓ Maximum limit of loan is Rs 10 lakh.
✓ Stands for Micro Units Development & Refinance Agency Ltd (MUDRA) Bank
✓ Founded : 8 April 2015
✓ Headquarters : Mumbai
✓ Chairman : S. Ramann
✓ it was converted as a wholly owned subsidiary of SIDBI and renamed as MUDRA (SIDBI) Bank.
✓ MUDRA Portal : Borrowers can also now file online application for MUDRA loans on Mudramitra
portal.
✓ Mudra Yojana offers three type of product namely Shishu, Kishor and Tarun
✓ Shishu : covering loans upto 50,000/- (Single page application form for this product)
✓ Kishor : covering loans above 50,000/- and upto 5 lakh
✓ Tarun : covering loans above 5 lakh and upto 10 lakh
Q. Who has been appointed as the new Chairman of the Central Board of Indirect Taxes and Customs
(CBIC)?
A) Rabindra Narayan B) Sahil Jindal
C) Preet Pal Thakur D) Sanjay Kumar Agarwal
Answer : D
✓ Union govt has appointed Sanjay Kumar Agarwal as the new Chairman of the Central Board of Indirect
Taxes and Customs (CBIC) after the superannuation of Vivek Johri.
✓ Central Board of Indirect Taxes and Customs (CBIC)
✓ Founded : 1944
✓ Headquarters : New Delhi
✓ Chairperson : Sanjay Kumar Agarwal
Q. Which insurance company has launched the ‘Direct Incentive Disbursement’ for its sales force in the
Delhi and Uttarakhand circles?
A) Max Life Insurance B) Postal Life Insurance
C) Bajaj Allianz Insurance D) Bharti AXA Insurance
Answer : B
✓ Recognising the pivotal role of its sales force, the Postal Life Insurance (PLI) introduced the pilot program
for “Direct Incentive Disbursement” in the Delhi and Uttarakhand Circles.
✓ About Direct Incentive Disbursement Program.
✓ This programme will recognize the role of PLI’s sales force which is the driving force behind the
department’s accomplishments.
✓ It will impact around two lakh sales force members across the nation, including Gramin Dak Sevaks,
Direct Agents, Field Officers, and Departmental Employees.
✓ Key benefits of pilot program.
✓ Swift and Secure Transactions: Sales force receives incentives directly in their Post Office Savings Bank
accounts.
✓ Convenience and Motivation: Sales force can conveniently manage their funds, and immediate rewards
drive optimal performance.
✓ Simplified Administration: Automated payouts reduce administrative costs, enabling a greater focus on
delivering client service.
✓ Key facts about Postal Life Insurance
✓ It is the oldest life insurer in this country which was introduced on 1st February 1884.

Follow us: Official Site, Telegram, Facebook, Instagram, Instamojo 293


✓ It started as a welfare scheme for the benefit of postal employees and was later extended to the employees
of the Telegraph Department in 1888.
✓ In 1894, PLI extended insurance cover to female employees of the erstwhile P & T Department at a time
when no other insurance company covered female lives.

Q. Who has been appointed as the new MD & CEO of SBI Life Insurance Company?
A) Amit Jhingran B) Debadatta Chand
C) J Packirisamy D) Mallikarjuna Rao
Answer : A
✓ Amit Jhingran appoints as MD & CEO of SBI Life Insurance.
✓ The Insurance Regulatory and Development Authority of India (IRDAI) has approved the appointment
of Amit Jhingran as the new MD & CEO of SBI Life Insurance Company.
✓ He had a successful career with the SBI spanning over 30 years.
✓ Currently, he serves as the Chief General Manager of the Hyderabad circle.
✓ SBI Life is an Indian life insurance company which was started as a joint venture between State Bank of
India (SBI) and French financial institution BNP Paribas Cardif. SBI has a 55.50% stake in the company
and BNP Paribas Cardif owns a 0.22% stake
✓ SBI Life Insurance Company
✓ Founded : 2001
✓ Headquarters : Mumbai
✓ MD & CEO : Amit Jhingran
Q. 5.33 crore subscribers have so far enrolled in Atal Pension Yojana (APY), the government told in Lok
Shaba.Atal Pension Yojana (APY) is administered by which organisation?
A) PFRDA B) RBI
C) LIC D) IRDAI
Answer : A
✓ 5.33 crore subscribers have so far enrolled in Atal Pension Yojana (APY), the government told in Lok
Shaba.
✓ It aims to deliver old age income security to all citizens of India, particularly to the workers in the
unorganised sector.
✓ Atal Pension Yojana (APY) was launched on 9 May 2015 and was operationalized from 1 June 2015.
✓ The minimum age of joining the APY is 18 years and the maximum age is 40 years. As per the scheme,
subscribers will receive pension benefits upon attaining the age of 60 years.
✓ The Scheme offers a flexible minimum guaranteed pension of Rs 1000, Rs 2000, Rs 3000, Rs 4000 or Rs
5000 per month based on the age of joining and pension amount chosen.
✓ Accordingly, the per month subscription amount presently varies from Rs 42 to Rs 1454 on the basis of
the chosen option.
✓ The same pension would be paid to the spouse of the subscriber after the demise of the subscriber and on
the demise of both the subscriber and spouse, the pension wealth as accumulated till age 60 of the
subscriber would be returned back to the nominee.

Q. Who launched the 3rd edition of the UPI Safety Awareness Campaign, "UPI Chalega" in August 2023?
A) NTPC B) NPCI
C) NITI Aayog D) Ministry of Finance
Answer : B

Follow us: Official Site, Telegram, Facebook, Instagram, Instamojo 294


✓ National Payments Corporation of India (NPCI) has launched the 3rd edition of the UPI Safety
Awareness Campaign, "UPI Chalega".
✓ The campaign aims to promote UPI as an easy, safe, and instant mode of payment. It also educates users
about various features such as UPI LITE, which enables swift low-value transactions;
✓ UPI AUTOPAY, which allows secure and convenient recurring payments using any UPI application;
and UPI Interoperability, which facilitates seamless money transfers between all UPI-enabled
applications.
✓ UPI has revolutionized digital payments, witnessing increased transaction value and volume. The
campaign was launched in 2020 under the Financial Literacy Advisory Committee (FLAC) guidance.
✓ UPI Chalega Campaign 1.0 and 2.0 played a crucial role in expanding UPI’s outreach, educating users
about its safety, and enhancing its ease of use for various transactions, making it the preferred payment
choice.
✓ UPI Chalega 3.0 Aim :
✓ To drive UPI adoption and safe usage through engaging initiatives. The campaign aims to boost UPI’s
brand awareness, amplify understanding of its features, and encourage wider adoption among new users.
Q. Which payments bank has launched an eco-friendly debit card for its customers with savings bank
accounts?
A) India Post Payments Bank B) Fino Payments Bank
C) Paytm Payments Bank D) Airtel Payments Bank
Answer : D
✓ Airtel Payments Bank Launched India’s 1st Eco-Friendly Debit Card.
✓ Airtel Payments Bank has launched an eco-friendly debit card for its customers with savings bank
accounts, underlining the company’s ‘commitment to sustainability and promoting eco-friendly practices
within the financial sector.’
✓ The company will manufacture these debit cards from r-PVC (recycled polyvinyl chloride) material,
which it claims to be eco-friendly.
✓ Moreover, the payments bank stated that every batch of 50,000 cards produced would reduce carbon
emissions by 350 kilograms, compared to conventional PVC cards in the market.
✓ Producing these eco-friendly debit cards will also diminish petroleum consumption during
manufacturing, cutting hydrocarbon usage by 43 percent. This manufacturing approach will conserve 6.6
million litres of water for every batch of r-PVC produced.
✓ Airtel Payments Bank is developing two cards under the classic variant; a personalized Classic card and
an Insta Classic card.
✓ The former can be ordered through the Airtel Thanks app, while the latter will be available at select
neighbourhood banking points by the end of the current quarter.
✓ The cards come with rewards including e-commerce benefits of up to Rs 10,000 and complimentary
dining experiences under the ‘One Dines Free’ programme across major cities of India.
✓ Airtel Payments Bank
✓ Founded : 2017
✓ Headquarters : New Delhi
✓ MD & CEO : Anubrata Biswas
✓ Stake : Bharti Airtel Limited(80.01%) & Kotak Mahindra Bank(19.9%)
✓ Subsidiaries : YTS Solutions Pvt. Ltd.
✓ Airtel Payments Bank became the first entity in India to receive a payments bank license from the Reserve
Bank of India (RBI).

Follow us: Official Site, Telegram, Facebook, Instagram, Instamojo 295


Q. Indraprastha Institute of Information Technology Delhi Innovation and Incubation Centre (IIITD-IC)
has signed an MoU with which bank to scale up Indian startups?
A) Yes Bank B) HDFC Bank
C) Axis Bank D) ICICI Bank
Answer : A
✓ Indraprastha Institute of Information Technology Delhi Innovation and Incubation Centre (IIITD-IC)
has signed a MoU with YES Bank.
✓ The partnership aims to enable IIITD-IC to strengthen its mentoring initiatives for startups, gain access
to experts across the industry, as well as identify newer investment and funding opportunities that can
benefit startups.
✓ In this collaboration, YES Bank aims to provide banking services to startups in the developmental phase
at IIITD-IC through the Bank’s specialized YES Head Startup programme.
✓ YES Bank has brought the bank to Startups, adopting a horizontal approach and bundling up the most
lucrative features possible.
Q. Who is the head of the Committee set up to ‘Onshoring the Indian Innovation to GIFT IFSC’?
A) G Padmanabhan B) Viral Acharya
C) Urjit Patel D) D Subbarao
Answer : A
✓ A ‘Committee of Experts to ‘Onshoring the Indian Innovation to GIFT IFSC’ constituted by
International Financial Services Centres Authority (IFSCA) submitted its report to Chairperson, IFSCA.
The committee was chaired by G. Padmanabhan, Former Executive Director, RBI. The committee
members comprised representatives from leading Venture Capital funds, Startups, Fintechs, Law firms,
Tax firms and other domain experts.

Q. Which company announced to release ‘Code Llama’ AI model to assist in writing computer code?
A) Google B) Apple
C) Microsoft D) Meta
Answer : D
✓ Meta Platforms said it would release an artificial intelligence (AI) model designed to assist in writing
computer code
✓ It is a large language model (LLM) that can use text prompts to generate and discuss code. It has the
potential to make workflows faster and more efficient for developers. Code Llama has the potential to be
used as a productivity and educational tool to help programmers write more robust software
Q. What is the name of the Invoice Incentive Scheme launched by the Government of India recently?
A) 'Mera Bill Mera Haque' B) 'Mera Bill Mera Adhikaar'
C) 'Mera Bill Nai Pahal' D) 'Mera Bill Nai Urja'
Answer : B
✓ Government of India is starting an 'Invoice Promotion Scheme' named 'Mera Bill Mera Adhikaar' to
encourage the initiative of customers seeking bills for all purchases.
✓ It is being taken forward in collaboration with all the state governments.
✓ This pilot scheme will be launched on September 1, 2023.
✓ About Mera Bill Mera Adhikar Scheme:

Follow us: Official Site, Telegram, Facebook, Instagram, Instamojo 296


✓ It is a Goods and Services Tax (GST) invoice incentive programme that offers cash incentives for
uploading invoices.
✓ It would initially be introduced in the states of Assam, Gujarat, and Haryana, as well as the UTs of
Puducherry, Daman & Diu, and Dadra & Nagar Haveli.
✓ Objective: To encourage customers to request a bill whenever they make purchases.
Q. Under the new regulations, the foreign portfolio investors (FPIs) is holding more than 50 percent of their
equity AUM in one Indian corporate group will have to reduce their holdings within how many certain
number of trading days.
A) 5 B) 10
C) 15 D) 20

Follow us: Official Site, Telegram, Facebook, Instagram, Instamojo 297


BEST MCQ BANKING, ECONOMY AND FINANCIAL AWARENESS
JULY 2023

Q. Which bank has overtaken State Bank of India (SBI) to earn top spot on the list of 2023 CRISIL’s
Greenwich market share Leaders in Large Corporate Banking?
A) Yes Bank B) ICICI Bank
C) HDFC Bank D) Indian Overseas Bank
Answer : C
✓ HDFC Bank overtakes SBI in CRISIL’s corporate banking ranking for 2023.
✓ India’s largest private lender HDFC Bank has overtaken State Bank of India (SBI) to earn top spot on the
list of 2023 CRISIL’s Greenwich market share Leaders in Large Corporate Banking.
✓ For middle market corporates, HDFC Bank again topped the list of 2023 Leaders among local banks,
and joined ICICI Bank as the year’s joint Greenwich Quality Leaders.
✓ Even among the PSUs, the trend toward consolidation among the largest providers persisted, with SBI
doing a better job than smaller banks at maintaining corporate relationships, it added.
✓ From 2021 to 2022, the share of Indian corporates working with one of the largest Indian private sector
banks for overall corporate banking services increased to 38 per cent from 33 per cent.
Q. AT-1 bonds are regulated by ?
A) RBI B) SEBI
C) SIDBI D) IRDAI
Answer : A
✓ The underwhelming subscription to State Bank of India’s additional tier-1 (AT-1) bond issue has
dampened market sentiment and is expected to make fund-raising harder for other PSU banks.
✓ About AT1 bonds:
✓ These are a type of unsecured, perpetual bonds that banks issue to improve their core capital base.
✓ The money raised through these bonds is kept aside as a shock absorber by the bank.
✓ They have a call option, which can be used by the banks to buy these bonds back from investors.
✓ These bonds were created in the wake of the 2008 financial crisis to absorb the losses.
✓ These bonds are also called contingent convertible bonds or CoCos.
✓ These bonds are also mandatory under Basel=III norms.
✓ The banks must maintain capital at a minimum ratio of 11.5 per cent of their risk-weighted loans. Of this,
9.5 per cent needs to be in Tier-1 capital. AT1 bonds fall under this type of capital.
✓ These bonds are long-term and do not carry any maturity date. Because of a higher risk, they offer a
higher yield.
✓ Regulation: In India AT-1 bonds are regulated by the Reserve Bank of India (RBI).
Q. Which institution released the ‘World Investment Report 2023’?
A) UNDP B) UNEP
C) WEF D) UNCTAD
Answer : D
✓ Recently, the United Nations Conference on Trade and Development (UNCTAD) published its World
Investment Report 2023.
✓ India and ASEAN were the most buoyant recipients of the Foreign direct investment (FDI), with
increases of 10 and 5%, respectively.

Follow us: Official Site, Telegram, Facebook, Instagram, Instamojo 298


✓ FDI inflows were higher in developing countries compared with those in developed economies.
✓ China, the second largest FDI host country in the world, saw a 5% increase.
✓ FDI in the Gulf region declined, but the number of project announcements increased by two thirds.
✓ Inflows in many smaller developing countries were stagnant, and FDI to the least developed countries
(LDCs) declined.
✓ Much of the growth in international investment in renewable energy has been concentrated in developed
countries.
✓ The investment gap across all sectors of the Sustainable Development Goals has increased to more than
$4 trillion per year from $2.5 trillion in 2015.
✓ The largest gaps are in energy, water and transport infrastructure.
✓ Key facts about the United Nations Conference on Trade and Development
✓ It is a permanent inter-governmental body established by the United Nations General Assembly in 1964.
✓ Members: 195 countries
✓ Headquarters: Geneva, Switzerland

Q. What is the maximum deposit limit in Senior Citizen Savings Scheme (SCSS)?
A) Rs 30 lakh B) Rs 10 lakh
C) Rs 15 lakh D) No Such Limit
Answer : A
✓ The collections under the revamped Senior Citizen Savings Scheme (SCSS) jumped 176% on year to Rs
55,000 crore in the first quarter of the current financial year.
✓ About Senior Citizen Savings Scheme (SCSS):
✓ SCSS was launched with the main aim of providing senior citizens in India a regular income after they
attain the age of 60 years old.
✓ Who is eligible?
✓ Indian citizens above the age of 60 years.
✓ Retirees in the age bracket of 55-60 years who have opted for Voluntary Retirement Scheme (VRS) or
Superannuation.
✓ Retired defence personnel above 50 years and below 60 years of age.
✓ Maturity: It has a maturity period of five years. But, a depositor can extend one's maturity period for
another three years.
✓ Number of accounts: Individuals are allowed to operate more than one account by themselves or open a
joint account with their spouse.
✓ Deposit Limits: Eligible investors can make a lump sum deposit
✓ Minimum Deposit– Rs. 1,000 (and in multiples thereof)
✓ Maximum Deposit– Rs. 30 Lakh.
✓ Interest Payment: Under SCSS, the interest amount is paid to the account holders quarterly.
✓ Premature withdrawal: After one year of opening the account, premature withdrawal is allowed.
✓ Deposits in SCSS qualify for deduction u/s 80-C of Income Tax Act.
✓ As per the proposal, the maximum investment limit in SCSS has been doubled from Rs 15 lakh to Rs 30
lakh.
Q. Department of Animal Husbandry & Dairying, Ministry of Fisheries, Animal Husbandry, and Dairying
(DAHD) has introduced the ‘Credit Guarantee Scheme. The DAHD has set up a credit guarantee fund
trust of _.
A) Rs 850 crore B) Rs 600 crore
C) Rs 500 crore D) Rs 750 crore
Answer : D

Follow us: Official Site, Telegram, Facebook, Instagram, Instamojo 299


✓ The Department of Animal Husbandry & Dairying, Ministry of Fisheries, Animal Husbandry, and
Dairying (DAHD) has introduced the ‘Credit Guarantee Scheme’ under the Animal Husbandry
Infrastructure Development Fund (AHIDF) to support the Livestock sector's Micro, Small & Medium
Enterprises (MSMEs).
✓ About Credit Guarantee Scheme:
✓ It aims to strengthen the credit delivery system and ensure smooth access to finance for entrepreneurs
engaged in the Livestock sector.
✓ Objective:
✓ The main objective is to encourage lenders to focus on the viability of projects and provide credit facilities
based on the primary security of the assets being financed.
✓ By providing access to financial assistance, it promotes investments in various areas of the livestock
sector, such as dairy and meat processing, animal feed plants, breed improvement technology, waste
management, and veterinary vaccine and drug manufacturing facilities.
✓ Funding
✓ The DAHD has set up a credit guarantee fund trust of Rs 750 crore, which will cover up to 25 per cent
of credit facilities extended to eligible MSMEs by lending institutions.
✓ The trust, formed in partnership with NAB Sanrakshan Trustee Company Private Ltd, a subsidiary of
NABARD, ensures credit guarantee for MSMEs under the AHIDF scheme.
✓ Key features include:
✓ Interest subvention of three per cent
✓ Loan of up to 90 per cent of the total project cost from any Scheduled Bank, National Cooperative
Development Corporation (NCDC).
Q. Which bank will set up a new trustee company as its wholly-owned subsidiary for managing the
Corporate Debt Market Development Fund (CDMDF)?
A) Kotak Mahindra Bank
B) State Bank of India
C) Punjab National Bank
D) Canara Bank
Answer : B
✓ State Bank of India will set up a new trustee company as its wholly-owned subsidiary for managing the
Corporate Debt Market Development Fund (CDMDF).
✓ CDMDF will act as a backstop facility for the purchase of investment-grade corporate debt securities to
instill confidence amongst the participants in the corporate bond market during times of stress and to
generally enhance secondary market liquidity by creating a permanent institutional framework for
activation in times of market stress.
✓ The setting up of the Fund, which will be classified as an Alternative Investment Fund (AIF) under the
SEBI (Alternative Investment Funds) Regulations, 2012, is subject to receipt of all regulatory approvals.
✓ SBI has a 62.53 per cent stake in SBI Funds Management Ltd..
✓ Close-ended scheme
✓ CDMDF will be launched as a close-ended scheme with an initial tenure of 15 years from the date of its
initial closing (date on which contribution from all AMCs and specified schemes is received by CDMDF).
✓ Based on the requirements of the corporate bond market, the tenure can be extended with prior approval
of the government in consultation with SEBI subsequent.
✓ To begin with, units of CDMDF shall be subscribed by Asset Management Companies (AMCs) of
Mutual Funds and “specified debt-oriented MF Schemes” (that is Open-ended Debt oriented Mutual
Fund schemes excluding Overnight funds and Gilt funds and including Conservative Hybrid funds.

Follow us: Official Site, Telegram, Facebook, Instagram, Instamojo 300


Q. According to the Goldman Sachs report, India will become the second-largest economy in the world by
which year?
A) 2050 B) 2040
C) 2075 D) 2055
Answer : C
✓ India to be world's second largest economy by 2075: Goldman Sachs Research.
✓ According to the report of Goldman Sachs Research, by the year 2075, India will become the second
largest economy in the world after China leaving behind not only Japan and Germany but also America.
✓ According to the world's important investment bank Goldman Sachs, India has left behind China in
terms of population, so its GDP is expected to expand.
✓ India's current economy: $3.2 trillion
✓ Presently India's economy ranks in the world: Fifth
✓ India's economy in the world by 2075: $52.5 trillion (2nd largest)
✓ Currently, the economy of the top countries (in trillions of dollars):
✓ America: 23.3
✓ China: 17.7
✓ Japan: 4.9
✓ Germany: 4.3
✓ India: 3.2
✓ UK: 3.1
✓ Economy of top countries in 2075 (in trillions of dollars):
✓ China: 57
✓ India: 52.5
✓ America: 51.5
✓ Europe: 30.3
✓ Japan: 7.5

Q. The 50th GST Council meeting in New Delhi concluded under whose leadership?
A) Rajnath Singh B) Smriti Irani
C) S. Jaishankar D) Nirmala Sitharaman
Answer : D
✓ The 50th GST Council meeting, headed by the Finance Minister, Nirmala Sitharaman, has been
concluded in New Delhi.
✓ Key Highlights :
✓ 28% GST rate has been levied on the full value of gaming, horse racing, and casino.
✓ Exempted GST on pharma products like cancer drug Dinutuximab, and Food for Special Medical
Purposes.
✓ GST on uncooked, unfried and extruded snack palettes have been brought down from 18% to 5%.
✓ The GST Council had decided to reduce the GST rate on fish soluble paste from 18% to 5%.
✓ Imitation zari thread brought down from 12% to 5%.
✓ The council announced that the GST on restaurants inside Cineplex will attract 5% GST against 18%
earlier.
✓ Before the change, movie tickets below Rs 100 were taxed at 12%, while those above the threshold
attracted 18% GST.
✓ Currently, the drug attracts a 12% integrated GST.

Follow us: Official Site, Telegram, Facebook, Instagram, Instamojo 301


Q. What percentage of GST has been fixed on 'Online Gaming' in the 50th meeting of the GST Council?
A) 05% B) 15%
C) 18% D) 28%
Answer : D
✓ The GST Council has approved the imposition of 28 per cent GST on the full value of online gaming,
casino and horse racing.
✓ The council approved a GST rate reduction on four items.
✓ These are: - Uncooked, unfried, and extruded snack palettes to a GST rate of 5% from 18%. - Fish soluble
paste to see 5% GST rate from earlier 18%
Q. Which bank will be signing a ₹3,000 crore IT transformation deal to revamp and enhance its tech
infrastructure?
A) Canara Bank B) Bank of Baroda
C) Indian Overseas Bank D) Kotak Mahindra Bank
Answer : A
✓ Public sector bank Canara Bank will be signing a ₹3,000 crore IT transformation deal to revamp and
enhance its tech infrastructure.
✓ The partnership will aim to modernize and enhance Canara Bank’s technological capabilities, enabling
it to provide efficient banking solutions to its customers.
✓ Over the last three years, Canara Bank has significantly on new technology initiatives. For the current
year alone, the spending has been ₹1,200 crore.
✓ The bank aims for 90 percent of its RAM (Retail, Agriculture, and MSME) transactions to go digital from
end to end by March 2024.
Q. The Ministry of Coal has set a target to gasify 100 million tonnes of coal by which year?
A) 2025 B) 2030
C) 2035 D) 2040
Answer : B
✓ The Ministry of Coal has set a target to gasify 100 million tonnes of coal by FY 2030.
✓ The ministry is considering a comprehensive scheme, with an outlay of ₹6,000 crore, to promote coal
and lignite gasification projects for both public sector undertakings (PSUs) and the private sector.
✓ The adoption of gasification technology in India will revolutionize the coal sector, reducing reliance on
imports of natural gas, methanol, ammonia, and other essential product.
✓ Currently, India imports around half of its natural gas, most of its methanol, and 13-15% of its total
ammonia requirement
Q. Which bank has become the first bank to officially roll out Mahila Samman Savings Certificate (MSSC)
scheme?
A) Kotak Mahindra Bank B) Indian Overseas Bank
C) Bank of India D) Canara Bank
Answer : C
✓ Bank of India, one of the largest public sector banks (PSBs) in the country, has become the first bank to
officially roll out Mahila Samman Savings Certificate (MSSC) scheme.

Follow us: Official Site, Telegram, Facebook, Instagram, Instamojo 302


✓ The scheme was announced by Union Finance Minister Nirmala Sitharaman during her Budget Speech
2023.
✓ About Mahila Samman Saving Certificate’ Scheme:
✓ The scheme offers deposit facility up to Rs 2 lakh in the name of women or girls for a tenor of 2 years.
✓ It offers fixed interest rate of 7.5 per cent.
✓ There are no tax benefits, but partial withdrawal is allowedin this scheme.
✓ This is a one-time scheme announced in Budget 2023 and will remain available for a two-year period i.e.
up to March 2025.
✓ Benefit: It will encourage more women to adopt formal financial saving instruments.
✓ Bank of India becomes first bank to offer Mahila Samman Savings Certificate.
✓ According to a statement by the bank, while the minimum investment is Rs 1,000, any sum in multiples
of Rs 100 can be deposited, up to a maximum limit of Rs 2 lakh.
✓ The account will mature after two years from the date of opening. Accounts under this scheme can be
opened until March 31, 2025

BOI IN NEWS 2023


▪ Bank of India becomes first bank to offer Mahila Samman Savings Certificate.
▪ Bank of India has inaugurated its IFSC Banking Unit (IBU) at GIFT SEZ area in Gandhinagar
▪ Rajneesh Karnatak has been appointed as the MD & CEO of the Bank of India (BOI) with effect from
April 29, 2023.
▪ He replaces Atanu Kumar Das.
▪ Bank of India has launched a ‘Centralized Pool Buy-Out and Co-Lending Cell’ to boost priority sector
lending in Mumbai, Maharashtra.
▪ Bank of India (BOI)
▪ Founded : 7 September 1906
▪ Founder : Ramnarain Ruia
▪ Headquarters : Mumbai
▪ MD & CEO : Rajneesh Karnatak
▪ Tagline : Relationship beyond Banking
Q. How much amount has been approved by the World Bank to accelerate the development of India's low-
carbon energy sector?
A) $1.5 billion B) $3.5 billion
C) $2.5 billion D) $1.8 billion
Answer : A
✓ World Bank’s board of executive directors has approved $1.5 billion in financing to accelerate India’s
development of low-carbon energy.
✓ The financing will help India promote low-carbon energy by scaling up renewable energy, developing
green hydrogen, and stimulating climate finance for low-carbon energy investments.
✓ Centre has set a target of achieving 500 GW of installed renewable energy capacity by 2030 and reaching
net zero by 2070.
✓ Further, recently the Indian government launched the much-anticipated incentive scheme for the
production of green hydrogen worth over ₹17,000 crore.
✓ The ‘First Low-Carbon Energy Programmatic Development Policy Operation’, the first in a series of two
envisaged operations, will support India in developing green hydrogen.
✓ The program will support the successful implementation of the National Green Hydrogen Mission that
aims to stimulate $100 billion in private sector investment by 2030,”

Follow us: Official Site, Telegram, Facebook, Instagram, Instamojo 303


Q. How much amount has been approved by the World Bank to accelerate the development of India's low-
carbon energy sector?
A) $1.5 billion B) $3.5 billion
C) $2.5 billion D) $1.8 billion
Answer : A
✓ World Bank’s board of executive directors has approved $1.5 billion in financing to accelerate India’s
development of low-carbon energy.
✓ The financing will help India promote low-carbon energy by scaling up renewable energy, developing
green hydrogen, and stimulating climate finance for low-carbon energy investments.
✓ Centre has set a target of achieving 500 GW of installed renewable energy capacity by 2030 and reaching
net zero by 2070.
✓ Further, recently the Indian government launched the much-anticipated incentive scheme for the
production of green hydrogen worth over ₹17,000 crore.
✓ The ‘First Low-Carbon Energy Programmatic Development Policy Operation’, the first in a series of two
envisaged operations, will support India in developing green hydrogen.
✓ The program will support the successful implementation of the National Green Hydrogen Mission that
aims to stimulate $100 billion in private sector investment by 2030,”
WORLD BANK IN NEWS 2023
▪ World Bank has approved $1.5 billion in financing to accelerate India’s development of low-carbon
energy.
▪ World Bank has approved a substantial funding package of 700 million dollars for Sri Lanka.
▪ World Bank has approved $391 million in financing to improve access to high-quality healthcare services
and to economic opportunities for people in Assam and Tripura.
▪ World Bank approves $150-million loan for Resilient Kerala programme.
▪ World Bank has granted its approval for a USD 255.5 million loan aimed at enhancing the quality of
technical education in government-run institutions across India.
▪ World Bank approves $150-million loan for Resilient Kerala programme.
▪ World Bank (WB) launched its first dedicated road safety project in South Asia with a USD 358 million
financing agreement signed in Dhaka with the government of Bangladesh.
▪ Indian origin Ajay Banga confirmed as the 14th President of World Bank.
▪ Ministry of Education and the World Bank held a unique workshop on School-to-Work Transition under
the STARS Program.
▪ The Cabinet approved the Strengthening Teaching-Learning and Results for States (STARS) Project in
October 2020.
▪ World Bank has approved a loan of US $ 363 million for the supply of clean drinking water from taps to
2 million rural households in Karnataka.
▪ World Bank’s Board of Executive Directors has approved USD 100 million loan under Odisha State
Capability and Resilient Growth Program.
▪ According to the World Bank’s (WB) annual report ‘State and Trends of Carbon Pricing 2023’ released
on May 23, 2023, revenues collected from carbon taxes and Emissions Trading Systems (ETS) globally
have reached a record high, about US$95 billion in 2022.
▪ World Bank’s Executive Board of Directors has approved USD 108 million loan for Assam, to help the
state improve disaster preparedness and enhance flood forecasting.
▪ India and the World Bank have signed aloan agreement for the construction of the Green National
Highway Corridors Project (GNHCP) in four States.
▪ These four states are-Himachal Pradesh, Rajasthan, Uttar Pradesh and Andhra Pradesh.
▪ World Bank has signed two complementary loans with India worth USD 500 million each to support
and enhance the country’s healthcare infrastructure.

Follow us: Official Site, Telegram, Facebook, Instagram, Instamojo 304


▪ India will be the first country in the world to receive $100 billion from remittances during 2022, the World
Bank.
▪ WORLD BANK
▪ Founded : 1944
▪ Headquarters : Washington Dc
▪ 14th President : Ajay Banga
▪ MD&CFO : Anshula Kant
▪ Chief Economist : Carman Reinhart
▪ Executive Director : Rajesh Khullar
▪ Member Countries :189
▪ World Bank Group : 5 (IBRD, IDA, IFC, MIGA and ICSID).
▪ All of these efforts support the Bank Group’s twin goals of ending extreme poverty by 2030 and boosting
shared prosperity of the poorest 40% of the population in all countries.
▪ World Bank was created at the 1944 Bretton Woods Conference, along with the International Monetary
Fund (IMF).
▪ India is a member of IBRD, IFC, IDA and MIGA. The country is not a member of the ICSID
Q. Chemical and Fertilizer Minister said Nine Nano Urea plants to be set up across the country by which
year?
A) 2030 B) 2025
C) 2028 D) 2026
Answer : B
✓ Recently, the Chemical and Fertilizer Minister said Nine Nano Urea plants to be set up across the country
by 2025.
✓ The ministry is aiming to manufacture 44 crore bottles of Nano Urea by the year 2025.
✓ To achieve the target nine plants will be set up across the country.
✓ About Nano Urea:
✓ It is a nanotechnology-based revolutionary Agri-input that provides nitrogen to plants.
✓ It is developed and patented by the Indian Farmers Fertiliser Cooperative Limited (IFFCO).
✓ IFFCO Nano Urea is the only nano fertilizer approved by the Government of India and included in the
Fertilizer Control Order (FCO).

Q. India has signed a US$ 295 million loan with ADB to upgrade the state highways of which state?
A) Maharashtra B) Assam
C) Bihar D) Rajasthan
Answer : C
✓ ADB, India sign USD 295 million loan pact for Bihar.
✓ Asian Development Bank (ADB) and the Indian government on have signed an agreement for a USD
295 million loan to upgrade around 265-kilometer state highways with climate and disaster-resilient
design and road safety elements in Bihar.
Q. According to the SBI Research’s Ecowrap Report 2023, India is expected to become the world’s 3rd
largest economy by which year, if it maintains its current rate of growth?
A) 2027 B) 2030
C) 2035 D) 2040
Answer : A

Follow us: Official Site, Telegram, Facebook, Instagram, Instamojo 305


✓ India is expected to become the world’s 3rd largest economy by 2027 if it maintains its current rate of
growth, according to the SBI Research’s Ecowrap Report 2023.
✓ Earlier, SBI Research had expected India to become the third-largest economy in the world by 2029.
✓ India is currently ranked third, a movement of seven places upward of what it was in 2014.
✓ Interestingly, the incremental increase by India between 2022-2027, is expected to be more than the
current size of Australia’s economy at USD 1.8 trillion.
✓ At this rate, India is likely to add USD 0.75 trillion in every two years, implying that India is all set to
touch USD 20 trillion by 2047. India’s global share in GDP will cross 4 percent by 2027.
✓ The share of India’s GDP is now at 3.5 percent, as against 2.6 percent in 2014, and is likely to reach 4
percent in 2027.
✓ Maharashtra and Uttar Pradesh will cross USD 500 billion mark by 2027.
Q. According to the RBI’s data, banks wrote off bad loans amounting to how many crores during the
financial year 2022-23?
A) Rs 1.09 lakh crore B) Rs 2.19 lakh crore
C) Rs 2.09 lakh crore D) Rs 1.89 lakh crore
Answer : C
✓ According to the RBI’s data during the financial year 2022-23, banks wrote off bad loans amounting to
over Rs 2.09 lakh crore, contributing to a total loan write-off of Rs 10.57 lakh crore by the banking sector
over the past five years.
✓ This huge loan write-off aided banks to bring down gross non-performing assets (GNPA) – or loans
defaulted by borrowers — to a 10-year low of 3.9 percent of advances in March 2023.
✓ Gross NPAs of banks had fallen from Rs 10.21 lakh crore in FY2018 to Rs 5.55 lakh crore by March
2023, mainly on the back of loan write-offs by banks.
✓ Banks have written off a whopping Rs 15,31,453 crore (US $ 187 billion) since FY2012-13, as per RBI
data.
✓ Banks recovered only Rs 109,186 crore from Rs 586,891 crore loans written off in the last three years,
revealing that they could only recover 18.60 percent of the write-offs during the three-year period
Q. What was the total amount of loans extended by Public Sector Banks (PSBs) to state-backed corporations
and public sector undertakings during the fiscal year 2022-23?
A) Rs 3.12 trillion B) Rs 4.12 trillion
C) Rs 5.12 trillion D) Rs 6.12 trillion
Answer : B
✓ Finance Minister Nirmala Sitharaman informed Parliament that during the fiscal year 2022-23, Public
Sector Banks (PSBs) extended loans amounting to Rs 4.12 trillion to state-backed corporations and public
sector undertakings.
✓ Canara Bank led the lending during 2022-23 to government-backed entities at ₹187,813 crore during the
recently concluded fiscal year.
✓ Followed by Punjab National Bank (₹70,142.5 crore), State Bank of India (₹66,523.2 crore), Bank of
India ( ₹25,147 crore), Bank of Baroda (₹15,706.8 crore), Union Bank of India (₹12,584.8 crore), Bank
of Maharashtra ( ₹10,822.7 crore), Indian Bank ( ₹9,021 crore), Indian Overseas Bank ( ₹7,490 crore),
Central Bank of India ( ₹3,949 crore), UCO Bank ( ₹2,939.4 crore), Punjab and Sind Bank ( ₹87.7 crore).
✓ Performance under PM SVANidhi scheme, which was started to support COVID-affected street vendors
to restart their businesses, was noted, wherein 50.57 lakh loans have been disbursed, amounting to ₹6,482
crore, to 38.5 lakh street vendors as on 20.7.2023.

Follow us: Official Site, Telegram, Facebook, Instagram, Instamojo 306


Q. Who has been appointed as the new Managing Director of Life Insurance Corporation?
A) Vivek Sinha B) Satpal Bhanu
C) Mudit Chauhan D) Kishan Kumar
Answer : B
✓ SatPal Bhanu has been appointed as the Managing Director (MD) of Life Insurance Corporation of India
(LIC).
✓ He succeeds Siddhartha Mohanty, who was appointed as the Chairman of LIC in April 2023.
✓ Now, LIC has 4 MDs: Sat Pal Bhanoo, Mini Ipe; M Jagannath and Tablesh Pandey.
Q. What is the name of new scheme announced under the station redevelopment drive of Ministry of
Railways?
A) Sansad Adarsh Gram Yojana B) Rashtriya Swasthya Bima Yojana
C) Amrit Bharat Station Scheme D) Pradhan Mantri Gram Sadak Yojana
Answer : C
✓ Southern Railway has identified 90 stations for development under the Amrit Bharat Station Scheme
(ABSS).

✓ About Amrit Bharat Station Scheme (ABSS).


✓ It envisages development of stations on a continuous basis with a long-term vision.
✓ The scheme will subsume all previous redevelopment projects where work is yet to begin.
✓ The scheme aims at preparation of Master Plans of the Railway stations and implementation of the
Master Plan in phases to enhance the facilities including and beyond the Minimum Essential Amenities.
✓ However, plans and consequent budgets will only be approved on the basis of factors such as footfall and
inputs from stakeholders.
✓ High level platforms(760-840 mill meter) shall be provided at all categories of stations.
✓ Provision for Roof Plaza to be created in future.
✓ Free Wi-Fi, space for 5G mobile towers.
Q. SAMARTH scheme implementing partners has been broadened with empanelment of 43 new
implementing partners and additional target of training around 75,000 beneficiaries has been allocated
to the training partners. The implementation period of the scheme is up to __.
A) March 2028 B) March 2024
C) March 2025 D) March 2027
Answer : B
✓ Recently, the panel of SAMARTH scheme implementing partners has been broadened with
empanelment of 43 new implementing partners and additional target of training around 75,000
beneficiaries has been allocated to the training partners.
✓ The Funding pattern has been revised with an increment of 5% in cost norms, which will give much
needed additional financial support to industries imparting skill under this Scheme.
✓ Key Facts about SAMARTH scheme
✓ Scheme for Capacity Building in Textiles Sector (SAMARTH) is a demand-driven and placement-
oriented umbrella skilling programme.
✓ It aims to incentivize and supplement the efforts of the industry in creating jobs in the organized textile
and related sectors, covering the entire value chain of textiles, excluding Spinning and Weaving.

Follow us: Official Site, Telegram, Facebook, Instagram, Instamojo 307


✓ In addition to the entry-level skilling, a special provision for upskilling/ re-skilling programme has also
been operationalized under the scheme towards improving the productivity of the existing workers in the
Apparel & Garmenting segments.
✓ The implementation period of the scheme is up to March 2024.
✓ Nodal Ministry: Ministry of Textiles.
Q. Global Multidimensional Poverty Index (MPI) report highlighted that a total of 415 million people
moved out of poverty in India. The Global Multidimensional Poverty Index was developed by which
organisation?
A) UNEP B) UNDP
C) WEF D) WTO
Answer : B
✓ Recently, the Global Multidimensional Poverty Index (MPI) report highlighted that a total of 415 million
people moved out of poverty in India within just 15 years from 2005/2006 to 2019/2021.
✓ Key Findings:
✓ It said that 25 countries, including India, successfully halved their global MPI values within 15 years,
showing that rapid progress is attainable.
✓ The report noted that deprivation in all indicators declined in India.
✓ The poorest States and groups, including children and people in disadvantaged caste groups, had the
fastest absolute progress.
✓ In India those people who are multidimensionally poor and deprived under the nutrition indicator have
declined.
✓ Children under the age of 18 account for half of MPI-poor people (566 million).
✓ The poverty rate among children is 27.7%, while among adults, it is 13.4%.
✓ Countries halved their MPI in periods as short as four to 12 years.
✓ Key facts about Global Multidimensional Poverty Index.
✓ It was developed by the United Nations Development Programme (UNDP) and the Oxford Poverty and
Human Development Initiative (OPHI).
✓ It was first launched in 2010.
✓ It constructs a deprivation profile of each household and person through 10 indicators spanning health,
education and standard of living and includes both incidence as well as intensity of poverty.
Q. eDAR Project is an initiative of the Ministry of Road Transport and Highways (MoRTH), Government
of India and is funded by
A) ADB B) IMF
C) World Bank D) AIIB
Answer : C
✓ All states and UTs have started using electronic detailed accident report (eDAR) application for recording
road crashes and over seven lakh accidents have been registered on this platform.
✓ About eDAR Project:
✓ The Integrated Road Accident Database (iRAD) / e-Detailed Accident Report (eDAR) Project is an
initiative of the Ministry of Road Transport and Highways (MoRTH), Government of India and is
funded by World Bank.
✓ Objective: The project aims to develop an Integrated Road Accident Database (iRAD), to enrich the
accident databases from every part of the country.
✓ The project would generate various types of insights by analysing the collected road accident data across
the country through implementation of data analytics technique

Follow us: Official Site, Telegram, Facebook, Instagram, Instamojo 308


Q. Which financial institution has recently approved USD 200 million additional financing for the Rajasthan
secondary towns development sector project?
A) World Bank B) ADB
C) AIIB D) IMF
Answer : B
✓ The Asian Development Bank (ADB) recently approved USD 200 million additional financing for the
Rajasthan secondary towns development sector project.
✓ About Asian Development Bank (ADB):
✓ It is a multilateral development bank established on 19th December 1966.
✓ Its primary mission is to "foster economic growth and cooperation" among countries in the Asia-Pacific
Region.
✓ Functions:
✓ ADB assists members and partners by providing loans, technical assistance, grants, and equity
investments to promote social and economic development.
✓ It also provides financing to certain private sector projects as well as public-private partnerships.
✓ The ADB was modelled closely on the World Bank and has a similar weighted voting system where votes
are distributed in proportion to members' capital subscriptions.
✓ As of 2022, ADB's five largest shareholders are Japan and the United States (each with 15.6% of total
shares), the People's Republic of China (6.4%), India (6.3%), and Australia (5.8%).
Q. The SWAMIH fund that has been created recently by the Government pertains to which sector?
A) Agriculture B) Banking
C) Housing D) MSME
Answer : C
✓ Recently, the Prime Minister of India congratulated new home owners in Bengaluru’s first project under
SWAMIH Fund.
✓ About SWAMIH Fund:
✓ The Special Window for Affordable and Mid-Income Housing (SWAMIH) Investment Fund I is a social
impact fund specifically formed for completing stressed and stalled residential projects.
✓ The Fund is sponsored by the Ministry of Finance, Government of India.
✓ It is managed by SBICAP Ventures Ltd., a State Bank Group company.
✓ It has one of the largest domestic real estate private equity teams focused only on funding and monitoring
the completion of stressed housing projects.
✓ It is a Category-II AIF (Alternate Investment Fund)debt fund registered with the Securities and Exchange
Board of India.
✓ Eligibility criteria for funding
✓ Real estate project must be registered under the Real Estate (Regulation and Development) Act (RERA)
2016.
✓ Project must be classified as a non-performing asset (NPA) or be under insolvency proceedings.
✓ The project should have been declared as a "stalled" or "delayed" project by a competent authority.
✓ The fund is available only for projects that fall under the affordable and mid-income housing categories.

Q. Which of the following does not forms a part of Foreign Exchange Reserves of RBI?
A) Foreign Currency Assets B) Special Drawing Rights
C) Reserve Tranche Position D) Silver
Answer : D

Follow us: Official Site, Telegram, Facebook, Instagram, Instamojo 309


✓ India’s Forex Reserves recently declined by $1.9 billion to $607.03 billion, its lowest level in nearly 3
months.
✓ About Foreign Exchange Reserves:
✓ What is it? Foreign Exchange Reserves (also called Forex Reserves) are reserve assets held by a central
bank in foreign currencies.
✓ These may include foreign currencies, bonds, treasury bills, and other government securities.
✓ Reserves are denominated and expressed in the US dollar, which is the international numeraire for the
purpose.
✓ RBI is the custodian of the Foreign exchange reserves in India.
✓ India’s foreign exchange reserves comprise of;
✓ Foreign currency assets (FCAs): These are maintained in currencies like the US dollar, euro, pound
sterling, Australian dollar and Japanese yen.
✓ Gold
✓ SDR (Special Drawing Rights): This is the reserve currency with IMF.
✓ RTP (Reserve Tranche Position): This is the reserve capital with IMF.
✓ The biggest contributor to India’s Forex reserves is foreign currency assets, followed by gold.
Q. Which Union Ministry launched the ‘ULLAS (Understanding Lifelong Learning for All in Society)
initiative’?
A) Ministry of Education B) Ministry of Finance
C) Ministry of Corporate Affairs D) Ministry of MSME
Answer : A
✓ Recently, the Union Minister of Education launched the logo, slogan-Jan Jan Sakshar and mobile
application of ULLAS in New Delhi.
✓ About ULLAS Initiative:
✓ The Understanding Lifelong Learning for All in Society (ULLAS) initiative is poised to revolutionise
education and literacy across the nation.
✓ It is done by fostering a learning ecosystem that reaches every individual, bridging the gaps in basic
literacy and critical life skills.
✓ It imparts basic education, digital and financial literacy and critical life skills to citizens aged 15 and above
who lost on the opportunity to go to school. It is being implemented through volunteerism.
✓ Slogan of the Initiative: ULLAS: Nav Bharat Saksharta Karyakram.
✓ For this purpose ULLAS app was launched which is user-friendly and interactive app available both on
android and ios.
✓ It will serve as a digital gateway for learners to engage in diverse learning resources through the DIKSHA
portal of NCERT.
✓ The ULLAS app can be used for registration of learners and volunteers either through self-registration or
by surveyors
Q. Which Union Ministry launched the ‘Unified Registration Portal For GOBARdhan’?
A) Ministry of Finance B) Ministry of Corporate Affairs
C) Ministry of MSME D) Ministry of Jal Shakti
Answer : D
✓ Over 1,200 biogas plants have registered on the Gobardhan portal since it was launched, according to an
official release.
✓ About Gobardhan portal:
✓ It serves as a centralized repository for assessing investment and participation in the Biogas/CBG
(Compressed Biogas) sector at a pan-India level.

Follow us: Official Site, Telegram, Facebook, Instagram, Instamojo 310


✓ Primary objective: To streamline the process of setting up CBG/Biogas plants in the country.
✓ It allows any government, cooperative or private entity operating or intending to set up a
Biogas/CBG/Bio-CNG plant in India to obtain a registration number by enrolling in the portal.
✓ This registration number will enable them to avail a multitude of benefits and support from the Ministries
and Departments of the Government of India.
✓ Key Facts about GOBARdhan Scheme:
✓ Galvanizing Organic Bio-Agro Resources Dhan is a crucial umbrella initiative of the Government of
India.
✓ The government launched the Gobardhan scheme in 2018 as a national priority project under the Swachh
Bharat Mission Grameen-Phase II program.
✓ Aim: To generate wealth and energy by converting cattle dung, agricultural residue, and other organic
waste into Biogas, CBG and bio-fertilizers.
✓ It adopts a whole-of-government approach and aims to convert waste into wealth, thereby promoting a
circular economy.
✓ Nodal Ministry: The Department of Drinking Water and Sanitation, Ministry of Jal Shakti.
Q. ‘Akhil Bhartiya Shiksha Samagam’ was organised to commemorate the anniversary of which
scheme/policy?
A) National Education Policy B) Pradhan Mantri Mudra Yojana
C) Pradhan Mantri Kaushal Vikas Yojana D) Pradhan Mantri Employment Generation Programme
Answer : A
✓ Prime Minister will inaugurate Akhil Bhartiya Shiksha Samagam 2023 and also release the first
instalment of funds under the PM SHRI Scheme at the event.
✓ About Pradhan Mantri Schools for Rising India (PM-SHRI) Scheme:
✓ It is a Centrally Sponsored scheme announced in 2022.
✓ Objective: Development of more than 14500 schools across the country by strengthening selected existing
schools being managed by Central Government/ State/ UT Government/ local bodies.
✓ The duration of the scheme is from 2022-23 to 2026-27,after which it shall be the responsibility of the
States/UTs to continue to maintain the benchmarks achieved by these schools.
✓ The selected schools will act as exemplar schools showcasing all components of the National Education
Policy 2020 (NEP) and offering mentorship to other schools in their vicinity.
✓ The PM SHRI Schools will be developed as Green Schools, incorporating environment-friendly aspects.
Q. Which regulatory body recently announced the creation of the Corporate Debt Market Development
Fund (CDMDF)?
A) RBI B) SEBI
C) SIDBI D) IRDAI
Answer : B
✓ Recently, the Union Finance Minister launched the Corporate Debt Market Development Fund
(CDMDF) .
✓ About Corporate Debt Market Development Fund:
✓ It is a backstop facility for specified debt funds during market dislocations.
✓ The fund is intended to provide liquidity support in the event of a financial crisis.
✓ It will be in the form of an alternative investment fund (AIF), meant to instil confidence among the
participants in the Corporate Bond Market during times of stress
✓ The fund has Rs 33,000-crore backstop facility for Mutual Funds.
✓ Of the Rs 33,000 crore, Rs 30,000 crore will come from the government, while the balance Rs 3,000 crore
will be contributed by the Asset Management Companies.

Follow us: Official Site, Telegram, Facebook, Instagram, Instamojo 311


✓ Contributions to the fund can be done by the specified debt-oriented mutual fund schemes and asset
management companies of mutual funds.
✓ This fund is guaranteed by the National Credit Guarantee Trust Company (NCGTC) and the backstop
facility will be managed by SBI Mutual Fund.
✓ SEBI announced the establishment of the Corporate Debt Market Development Fund (CDMDF).

Q. ‘Open Network for Digital Commerce’ (ONDC) is the initiative of which government body?
A) DPIIT B) SEBI
C) IRDAI D) RBI
Answer : A
✓ Recently, the Open Network for Digital Commerce (ONDC) introduced the ONDC Academy.
✓ About ONDC Academy:
✓ It is an initiative of Department for Promotion of Industry and Internal Trade (DPIIT) initiative to create
a facilitative model to help small retailers take advantage of digital commerce.
✓ It was launched by ONDC in collaboration with NSE Academy Ltd, a subsidiary of the National Stock
Exchange.
✓ It is not an application, platform, intermediary, or software but a set of specifications designed to foster
open, unbundled, and interoperable open networks.
✓ Benefits.
✓ The academy is a repository of educational and informative textual and video content programmes in
multiple Indian languages.
✓ It will provide a curated learning experience providing guidance and best practices for a successful e-
commerce journey.
✓ Anyone without any knowledge of e-commerce can learn how to make a seller app with a technology
service provider to aggregate all sellers from a nearby marketplace to make these products available
online.
✓ It would also enable certification issued by NSE Academy to individuals completing an assessment
developed by the institute.
✓ Key Facts about ONDC.
✓ It is a Section 8 company, under the initiative of DPIIT with a mission to democratize digital commerce.
✓ It develops and maintains the ONDC Protocol, an open technical standard similar to UPI, HTTP and
SMTP.
✓ It comprises of buyer-side apps where consumers can place orders, seller-side apps that on board
merchants and display their listings, and logistics platforms that handle deliveries.

Q. Logistics Data Bank (LDB) Project, which was seen in the news, is associated with which Ministry?
A) Ministry of Cooperation B) Ministry of Commerce and Industry
C) Ministry of Finance D) Ministry of Skill Development and Entrepreneurship
Answer : B
✓ Recently, the Department for Promotion of Industry and Internal Trade (DPIIT) held a Logistics Data
Bank Project (LDB) Meeting to review the measures taken by Port Authorities to improve port
performance.
✓ About Logistics Data Bank Project:
✓ This project was launched on 2016 at the Jawaharlal Nehru Port, Mumbai.
✓ Objectives: The project launched to make India’s logistics sector more efficient through the use of
Information Technology.
✓ Implementing Agency: It is being implemented through a Special Purpose Vehicle called Delhi Mumbai
Industrial Corridor Development Corporation Logistics Data Services Ltd. (DLDSL) - that is jointly

Follow us: Official Site, Telegram, Facebook, Instagram, Instamojo 312


(50:50) owned by the Delhi Mumbai Industrial Corridor (DMIC) Trust and Japanese IT services major
NEC Corporation.
✓ Every container is attached to a Radio Frequency Identification Tag (RFID) tag and then tracked through
RFID readers - aids importers and exporters in tracking their goods in transit.
✓ Nodal Ministry : Ministry of Commerce and Industry.

Q. Which institution issues the Sovereign Green Bonds (SGrBs) on behalf of the government?
A) RBI B) SEBI
C) IRDAI D) SIDBI
Answer : A
✓ Recently, the Chairman of Pension Fund Regulatory and Development Authority (PFRDA) said the
regulator is keen on pension funds investing in sovereign green bonds (SGBs).
✓ Why in news?
✓ The Government of India is expected to issue sovereign green bonds in the second half of the current
financial year as part of the overall market borrowing programme.
✓ About Sovereign Green Bonds:
✓ These are bonds issued by any sovereign entity, inter-governmental groups or alliances and corporates.
✓ Aim: The proceeds of the bonds are utilised for projects classified as environmentally sustainable.
✓ These are issued with longer maturity period.
✓ In India, the framework for the sovereign green bond was issued by the government on November 9,
2022.
✓ The first tranche of Sovereign Green Bonds (SGrBs) issued by the Reserve Bank of India (RBI) worth Rs
8,000 crore was subscribed fully. SGrBs will be issued through uniform price auction and 5 per cent of
the notified amount of sale will be reserved for retail investors. Coupon rates of the Bonds are in line with
the G-Sec
✓ Key Facts about the PFRDA
✓ It is a statutory regulatory body set up under PFRDA Act enacted in 2014.
✓ Objective: To promote old age income security by establishing, developing, and regulating pension funds
and to protect the interests of subscribers to schemes of pension funds and related matters.
✓ Composition: It consists of a Chairperson and not more than six members, of whom at least three shall
be Whole-Time Members, to be appointed by the Central Government.
✓ Nodal Ministry: Ministry of Finance
Q. Which of the following entity acts as the BBPCU for Bharat Bill Payment Systems (BBPS)?
A) RBI B) SBI
C) NPCI D) SBI and NPCI
Answer : C
✓ CEO of Bharat Bill Pay recently said Bharat Bill Pay has witnessed a threefold business growth in just
two years.
✓ About Bharat Bill Payment System (BBPS):
✓ BBPS is an integrated bill payment system or a platform which acts as a connect between various billers
and users.
✓ It is a one-stop ecosystem for payment of all bills providing an interoperable and accessible “Anytime
Anywhere” Bill payment service to all customers across India with certainty, reliability and safety of
transactions.
✓ It offers customers the convenience of payment by cataloguing various utility providers under one
platform.

Follow us: Official Site, Telegram, Facebook, Instagram, Instamojo 313


✓ It acts as a central reference for a customer who wants to make different payments — whether utility bills,
loan repayments, FasTag recharge, and so on.
✓ Bharat BillPay transactions can be initiated through multiple payment channels like Internet, Internet
Banking, Mobile, Mobile-Banking, Mobile Wallets Bank Branch, Agents and Business Correspondents
etc.
✓ It provides instant confirmation of payment via an SMS or receipt.
✓ Who are the stakeholders?
✓ BBPS was conceptualised by the Reserve Bank of India in 2013 and is a product of the National Payments
Council of India (NPCI).
✓ It was piloted in 2016 and went live a year later.
✓ By 2019, BBPS onboarded all recurring payments.
✓ Components: There are two key components in the BBPS system,
✓ Bharat Bill Payment Central Unit (BBPCU): The BBPCU is NPCI, which lays downs the operating
procedures and standards for BBPS.
✓ Bharat Bill Payment Operating Units (BBPOUs): BBPOUs adhere to the rules set by BBPCU. They are
the banking and non-banking entities that handle the payments load.
Q. Which of the following foreign borrowings are considered under External Commercial
Borrowings(ECBs) in India?
A) Buyer’s credit B) Supplier’s credit
C) Bank loans D) All the above
Answer : D
✓ Reserve Bank of India (RBI) recently said that agreements for external commercial borrowings (ECBs)
signed by Corporate India jumped to a massive $12 billion in the April-June quarter of 2023, three times
the level in the year-ago period.
✓ About External Commercial Borrowings (ECBs):
✓ ECBs refer to the borrowing of funds by Indian companies from foreign sources in the form of loans,
bonds, or other financial instruments.
✓ Purpose: It can be used to finance a variety of purposes, including the expansion of business, the
acquisition of assets, and the repayment of existing debt.
✓ Source of ECBs: ECB can be obtained from a variety of sources, including foreign banks, international
financial institutions, and foreign subsidiaries of Indian companies.
✓ ECB can be in the form of rupee-denominated loans, which are repaid in Indian rupees, or foreign
currency-denominated loans, which are repaid in a foreign currency.
✓ Regulation: ECB is subject to regulatory oversight by the RBI, which sets limits on the amount of ECB
that Indian companies can obtain and the purposes for which it can be used.
✓ Companies must also meet certain eligibility criteria in order to access ECB, such as minimum credit
ratings and debt-equity ratios.
✓ The ECBs are defined as money borrowed from foreign resources including the following: Commercial
bank loans, Buyers’ credit and suppliers’ credit, Securitised instruments such as Floating Rate Notes and
Fixed Rate Bonds , and Credit from official export credit agencies and commercial borrowings from the
private sector window of Multilateral Financial Institutions such as International Finance Corporation
(Washington), ADB, AFIC, CDC, etc.
Q. Which Union Ministry implements the ‘PM MITRA’ scheme?
A) Ministry of Textiles B) Ministry of Finance
C) Ministry of Home affairs D) Ministry of Education
Answer : A

Follow us: Official Site, Telegram, Facebook, Instagram, Instamojo 314


✓ Recently, the Prime Minister of India has lauded the foundation stone laying of 2 mega textile parks in
Maharashtra and Gujarat under the PM Mega Integrated Textile Regions and Apparel (PM MITRA)
scheme.
✓ About PM-MITRA:
✓ It will offer the opportunity to create an Integrated Textiles Value Chain right from spinning, weaving,
processing/dyeing and printing to garment manufacturing etc. at one location and will reduce the
logistics cost of the Industry.
✓ A Special Purpose Vehicleowned by the Centre and State Government will be set up for each park which
will oversee the implementation of the project.
✓ Funding.
✓ The Ministry of Textiles will provide financial support in the form of Development Capital Support up
to Rs. 500 crores per park to the Park SPV.
✓ A Competitive Incentive Support (CIS) of up to Rs 300 crore per park to the units in PM MITRA Park
shall also be provided to incentivise speedy implementation.
✓ State governments will provide a contiguous and encumbrance-free land parcel of at least 1000 acres of
land.
✓ Nodal Ministry : Ministry of Textiles.
✓ Government of India has planned to establish these parks in Tamil Nadu, Telangana, Gujarat,
Karnataka, Madhya Pradesh, Uttar Pradesh and Maharashtra.
Q. Recently, the Operational Guidelines of Production Linked Incentive Scheme 2.0 for IT Hardware has
been finalized. The incentives provided under the PLI 2.0 Scheme will be applicable for a period of how
many years?
A) 5 B) 6
C) 4 D) 3
Answer : B
✓ Recently, the Operational Guidelines of Production Linked Incentive Scheme 2.0 for IT Hardware has
been finalized.
✓ About Production Linked Incentive Scheme 2.0.
✓ The scheme proposes a financial incentive to boost domestic manufacturing and attract large investments
in the value chain.
✓ The target segments under PLI 2.0 Scheme shall include Laptops, Tablets, All-in-One PCs and Servers
and Ultra Small Form Factor.
✓ Implementation: Companies, both global and domestic, that meet the eligibility criteria specified in the
PLI 2.0 Scheme guidelines will receive support for manufacturing goods in India within the specified
target segment.
✓ Tenure: The incentives provided under the PLI 2.0 Scheme will be applicable for a period of 6 years
✓ Base year: For the calculation of net incremental sales of manufactured goods, the base year will be the
financial year 2022-23
Q. Who has launched a new campaign for banks under Agriculture Infrastructure Fund titled BHARAT
(Banks Heralding Accelerated Rural & Agriculture Transformation)?
A) Ministry of Cooperation
B) Ministry of Skill Development and Entrepreneurship
C) Ministry of Commerce and Industry
D) Ministry of Agriculture & Farmers Welfare
Answer : D

Follow us: Official Site, Telegram, Facebook, Instagram, Instamojo 315


✓ Recently, Ministry of Agriculture & Farmers Welfare launched a new campaign for banks under
Agriculture Infrastructure Fund titled BHARAT (Banks Heralding Accelerated Rural & Agriculture
Transformation).
✓ About BHARAT campaign:
✓ It is a one month-long Campaign with a target of Rs 7200 crore.
✓ It aims to get active involvement and support of members of commercial Banks in the public and private
sector, Regional Rural Banks, Small Finance Banks, NBFCs and select cooperative Banks in promoting
the Scheme of Agriculture Infrastructure Fund.
✓ What is Agriculture Infrastructure Fund?
✓ It is a Central Sector Scheme which was launched in 2020
✓ The scheme shall provide a medium - long term debt financing facility for investment in viable projects
for post-harvest management Infrastructure and community farming assets through interest subvention
and financial support.
✓ Duration: The duration of the Scheme shall be from FY2020 to FY2032 (10 years).
✓ All loans under this financing facility will have interest subvention of 3% per annum up to a limit of Rs.
2 crores. This subvention will be available for a maximum period of seven years.
✓ Further, credit guarantee coverage will be available for eligible borrowers from this financing facility
under Credit Guarantee Fund Trust for Micro and Small Enterprises (CGTMSE) scheme for a loan up
to Rs. 2 crores.
Q. Which company has launched its first international payment gateway for the Malaysian market with
Curlec?
A) Paytm B) PhonePe
C) Razorpay D) Pine Labs
Answer : C
✓ Razorpay has announced the launch of its first international payment gateway for the Malaysian market
with Curlec, now known as ‘Curlec by Razorpay’.
✓ In February 2022, Razorpay had acquired a majority stake in Kuala Lumpur-based fintech Curlec at a
valuation of $20 million with an aim to fuel its global expansion plans.
✓ The balance stake acquisition was to be completed in 2023.
✓ The company currently works with over 700 Malaysian businesses, including Tune Protect, CTOS,
Courts, Mary Kay and The National Kidney Foundation.
✓ “The new Curlec payment gateway will serve more than 5,000 businesses with a target of RM10 billion
($2 billion) in annualized gross transaction value (GTV) by 2025," Razorpay said in its statement.
✓ Razorpay
✓ Founded : 2014
✓ Founder : Shashank Kumar & Harshil Mathur
✓ Headquarters : Bangalore
Q. Which state has secured the first position in NITI Aayog's Export Preparedness Index 2022?
A) Gujarat B) Tamil Nadu
C) Uttar Pradesh D) Maharashtra
Answer : B
✓ NITI Aayog has recently released the Export Preparedness Index 2022.
✓ This time Tamil Nadu has achieved the top rank by leaving behind Maharashtra and Gujarat.
✓ The Export Preparedness Index evaluates a state's preparedness based on its export potential and
performance.

Follow us: Official Site, Telegram, Facebook, Instagram, Instamojo 316


✓ Tamil Nadu topped with a score of 80.89 while Maharashtra with a score of 78.20 and Karnataka (76.36)
came third
NITI AAYOG IN NEWS 2023
▪ NITI Aayog has recently released the Export Preparedness Index 2022.
▪ This time Tamil Nadu has achieved the top rank by leaving behind Maharashtra and Gujarat.
▪ NITI Aayog, United Nations join hands to accelerate sustainable development in India.
▪ PM Modi chairs the 8th meeting of the NITI Aayog Governing Council held at the New Convention
Center at Pragati Maidan, New Delhi.
▪ The meeting is focused on the theme of "Growth India @ 2047: Role of Team India".
▪ BVR Subrahmanyam was appointed as the new Chief Executive Officer of Niti Aayog.
▪ The government has monetised assets worth ₹26,000 crore during FY23 against the target of ₹1.6 lakh
crore for the current fiscal : NITI Aayog.
▪ NITI Aayog, the Indian government’s policy think-tank, recently released a report titled “Promoting
Millets in Diets: Best Practices across States/UTs of India.” .
▪ NITI Aayog ( National Institution for Transforming India)
▪ Founded : 1st January 2015
▪ Preceding : Planning Commission ( 15th March 1950)
▪ Headquarters : New Delhi
▪ Chairperson : Narendra Modi
▪ Vice Chairperson : Suman K Bery
▪ CEO : BVR Subrahmanyam
▪ NITI Aayog comes under the Ministry of Commerce and Industry

Q. According to RBI, India needs average annual ___ GDP growth to become developed by 2047.
A) 6.6% B) 7.6%
C) 8.6% D) 9.6%
Answer : B
✓ India could become a developed country by 2047 with an average annual real GDP growth of 7.6 percent
over the next 25 years, said an article published by the Reserve Bank in its July 2023 bulletin.
✓ The task, however, may not be easy, given the current level of capital stocks, infrastructure and skill sets
of the people, said the article titled 'India @ 100'.
✓ Addressing the nation on August 15, 2022 -- the 75th year of India's independence -- Prime Minister
Narendra Modi laid down a vision for the next 25 years to become a developed nation by 2047.
Q. Which company has launched the world's first Generative AI tool for insurance?
A) Tata Max B) Infosys
C) Simplify D) Meta
Answer : C
✓ AI automation company Simplify has launched InsuranceGPT, the world's first generative AI tool for
insurance.
✓ It is the world's first custom-built GPT tool powered by a no-code AI-powered platform.
ChatGPT IN NEWS 2023
▪ AI automation company Simplify has launched InsuranceGPT, the world's first generative AI tool for
insurance.
▪ Odisha TV, an Odia-based news station, has unveiled “Lisa,” India’s first regional AI news anchor.

Follow us: Official Site, Telegram, Facebook, Instagram, Instamojo 317


▪ Meta and Microsoft have jointly announced the expansion of their artificial intelligence partnership by
introducing their new large language model (LLM) called “Llama 2“.
▪ Elon Musk has launched his AI startup, xAI, aiming to create an alternative to ChatGPT.
▪ Inflection AI Company unveiled a modern talkbot named ‘Pi Chatbot’.
▪ ‘Gigachat’ Russia’s Sberbank launches Al to compete with ChatGPT.
▪ Google introduces AI chatbot ‘Bard’ to compete with Microsoft’s ChatGPT.
▪ Google to launch its AI Chatbot amid ChatGPT’s Growing Popularity.
▪ Lexi’: India’s first AI assistant powered by ChatGPT.
▪ Unique Identification Authority of India (UIDAI) has recently launched a chatbot to help people get an
answer to their queries related to the Aadhaar card. It is called “Aadhaar Mitra”.
▪ Aaj Tak, unveiled Sana, the first AI-generated news anchor in the country.
▪ ChatGPT.
▪ Cyber Security experts have sent out warnings about the potential use of ChatGPT to write phishing
emails, malicious code easily and at a much larger scale.
▪ About:
▪ ChatGPT (Generative Pre-trained Transformer) is a chatbot launched by OpenAI in November 2022. It
is built on top of OpenAI's GPT-3 family of large language models.
▪ It is a type of artificial intelligence that can understand and generate natural language text.
▪ It is trained on large amounts of text data and uses an algorithm called a transformer to learn how to
generate text that is similar to human conversation.
▪ OpenAI:
▪ OpenAI is a research institute and company that focuses on developing artificial intelligence technology
in a responsible and safe way.
▪ It was founded in 2015 by a group of entrepreneurs and researchers, including Elon Musk, Sam Altman,
and Greg Brockman.
Q. Which company has recently commissioned India’s first transnational power project?
A) Reliance Industries B) Adani Group
C) TATA Group D) Wipro Group
Answer : B
✓ Adani Group commissions India’s first transnational power project.
✓ Gautam Adani met Bangladesh PM Sheikh Hasina for full load commencement and handover of the
1,600 MW ultra-supercritical Godda power plant.
✓ Adani Power set up the 1,600 MW thermal power in Jharkhand’s Godda to supply power generated to
Bangladesh Power Development Board (BPDB) through a dedicated transmission line.
Q. Who has been appointed as the new MD & CEO of SBI Capital Markets Limited?
A) Ravi Tyagi
B) Rajay Kumar Sinha
C) Vijay Shekhar Sharma
D) Vinay Mohanty
Answer : B
✓ Rajay Kumar Sinha has been appointed as the Managing Director and Chief Executive Officer of SBI
Capital Markets Limited (SBICAPS).
✓ SBI Capital Markets Limited is a wholly owned subsidiary and investment banking company of State
Bank of India (SBI).
✓ Sinha has replaced Amitav Chatterjee

Follow us: Official Site, Telegram, Facebook, Instagram, Instamojo 318


Q. Reserve Bank of India has canceled the license of which co-operative bank?
A) Abhyudaya Co-operative Bank B) United India Co-operative Bank
C) Bharat Co-operative Bank D) Janta Sahakari Bank Limited
Answer : B
✓ The Reserve Bank of India (RBI) has canceled the license of United India Co-operative Bank, a bank in
Uttar Pradesh. RBI said that this bank has failed to comply with the relevant sections of the Banking
Regulation Act, 1949.
✓ Customers can withdraw up to Rs 5,00,000 from the Deposit and Credit Guarantee Corporation
(DIGCS) under depositor rules.
RBI CANCELED LICENSE IN NEWS 2023
▪ Reserve Bank of India (RBI) has canceled the license of United India Co-operative Bank, a bank in Uttar
Pradesh.
▪ RBI Cancels Certificate of Registration of 7 NBFCs and Surrender Permits of 14 NBFCs.
▪ RBI Cancels Registration of Four NBFCs, 11 Surrender Certificates.
▪ RBI Grants Non-Banking Institution License to Mahalaxmi Cooperative Bank in Dharwad, Karnataka
, Cancels Banking Permit.
▪ Equitas Holdings Ltd, a prominent non-banking financial company (NBFC) in India, has voluntarily
surrendered its NBFC licence to the Reserve Bank of India (RBI).
▪ RBI cancelled the registration of Pune-based Kudos Finance and Investments and Mumbai-based Credit
Gate for regulatory lapses in lending practices.
▪ Reserve Bank of India (RBI) due to insufficient capital and earning potential revoked the licence of
Madhya Pradesh’s Garha Co-operative Bank, Guna.
Q. Tata Motors-owned Jaguar Land Rover (JLR) has appointed whom as its Chief Executive Officer for a
3 Year term?
A) Adrian Mardell B) Ola Kallenius
C) Oliver Zipse D) Zak Brown
Answer : A
✓ Adrian Mardell (United Kingdom) has been appointed as the Chief Executive Officer of Jaguar Land
Rover (JLR) for a period of three years.
✓ The appointment was made by Tata Motors, which owns JLR.
✓ Prior to becoming CEO, Mardell served as interim CEO through November 16, 2022.
✓ With the appointment of Adrian Mardell as CEO, Richard Molyneux was announced as the new Chief
Financial Officer of JLR.
✓ About Tata Motors Limited
✓ It is an Indian multinational automotive manufacturing company.
✓ It operates as a part of the Tata Group conglomerate.
✓ Tata Motors produces a wide range of vehicles including passenger cars, trucks, vans, coaches and buses.
✓ Establishment - 1945
✓ Founder - Jehangir Ratanji Dadabhoy Tata
✓ Headquarters - Mumbai, Maharashtra
✓ Chairman - Natarajan Chandrasekaran

Q. Which country has recently applied to join the BRICS Bank with an amount of $1.5 billion?
A) Algeria B) Ethiopia
C) Nigeria D) Egypt

Follow us: Official Site, Telegram, Facebook, Instagram, Instamojo 319


Answer : A
✓ Algeria has applied to join the BRICS group and submitted a request to become a shareholder member
of the BRICS Bank with an amount of $1.5 billion.
✓ Algeria had sought to join the BRICS to open new economic opportunities.
✓ The North African country is rich in oil and gas resources and seeking to diversify its economy and
strengthen its partnership with countries such as China.
✓ China will also invest $36 billion in Algeria across sectors including manufacturing, new technology, the
knowledge economy, transport, and agriculture.
✓ The BRICS group of nations comprises Brazil, Russia, India, China, and South Africa. It accounts for
more than 40% of the world’s population and about 26% of the global economy.
✓ About BRICS
✓ The full form of BRICS is Brazil, Russia, India, China, and South Africa.
✓ Goldman Sachs economist Jim O'Neill coined the term BRIC (without South Africa) in 2001.
✓ He claimed that by 2050, the four BRIC economies would dominate the global economy by 2050.
✓ South Africa was included in the list in 2010.
✓ The presidency of the Forum is rotated annually among the members.
✓ BRICS accounts for about 40% of the world’s population.
✓ It accounts for 30% of the world's GDP (Gross Domestic Product).
✓ In the year 2014, during the 6th BRICS Summit in Fortaleza, Brazil, the BRICS leaders signed an
agreement to establish a New Development Bank (NDB).
Q. Finance Minister Nirmala Sitharaman has recently inaugurated a new Goods and Service Tax (GST)
Bhavan in which state?
A) Assam B) Odisha
C) Tripura D) Meghalaya
Answer : C
✓ Union Minister of Finance and Corporate Affairs Nirmala Sitharaman has inaugurated the state’s lone
Goods and Service Tax (GST) Bhavan at Agartala, Tripura.
✓ The GST Bhawan is a project of the CGST, CX and Customs Office, Agartala, Guwahati Zone, under
the Central Board for Indirect Taxes (CBIC) in Agartala. It is located near Netaji Chowmuhani on Mantri
Bari Road in Agartala.
✓ Union Finance and Corporate Affairs Minister Nirmala Sitharaman Tripura has received the GST
collection of 982.50 crore rupees in 2022-23.
✓ The inauguration of the project in Amrit Kaal during the Azadi Ka Amrit Mahotsav showcases the
strength of New India.
Q. The blue bird logo of social media giant Twitter has recently been changed. What is the new logo?
A) Y B) W
C) X D) Z
Answer : C
✓ Popular social media platform Twitter launched a new logo 'X' as part of a massive rebranding effort.
✓ The blue bird logo has been replaced by an 'X', which is now featured prominently on the website.
✓ Media reports have confirmed the unveiling of the new logo, which features a white 'X' on a black
background, symbolising the platform's updated identity.
✓ The new logo was announced jointly by Twitter founder Elon Musk and CEO Linda Yacarino, indicating
their involvement in the rebranding process.

Follow us: Official Site, Telegram, Facebook, Instagram, Instamojo 320


✓ This logo change represents the latest significant update since Elon Musk acquired Twitter last year for a
whopping $44 billion.
✓ 'X' logo:
✓ The 'X' logo introduced during Twitter's rebranding marks a shift towards turning the platform into an
all-encompassing app similar to China's WeChat.
✓ The change is aimed at enabling users to perform various tasks beyond social media including making
payments and booking hotels.
✓ Rebranding isn't just a visual change; It represents a major strategic effort to turn Twitter into a
comprehensive application with multiple functionalities.
✓ Martin Grasser, one of the original designers of the original blue bird logo.
✓ The X logo suggested by Merritt, was created by an engineer called Alex Tourville.
✓ About Twitter
✓ Twitter was founded in March 2006 by Jack Dorsey, Biz Stone, and Evan Williams as a social media
platform for short, real-time messages called "tweets."
✓ It launched publicly in July 2006 and quickly gained popularity.
✓ Linda Yaccarino named CEO of Twitter.

Q. With whom has 'India AI' signed an MoU to promote emerging technologies?
A) Spacex B) Google
C) Meta D) Microsoft
Answer : C
✓ India Artificial Intelligence ('India AI') has signed an MoU with 'META' to promote advancements in
emerging technologies in the country.
✓ It aims to establish collaboration between 'India AI' and META in the field of Artificial Intelligence and
emerging technologies.
✓ Also, Meta's open-source AI model will be tailored for the Indian AI ecosystem
✓ About Meta
✓ Founded - February 2004, Cambridge, Massachusetts, United States
✓ Founder - Mark Zuckerberg
✓ Headquarters - Menlo Park, California, United States

Q. Which fintech platform has recently launched a feature to pay income tax through its app?
A) PhonePe
B) GooglePay
C) Paytm
D) PayPal
Answer : A
✓ Digital payments and fintech platform, PhonePe has launched a feature to pay income tax through its
app.
✓ Individuals and businesses can pay self-assessment and advance tax through UPI or credit card via the
application, without logging into the income tax portal.
✓ PhonePe has partnered with digital B2B payments service provider PayMate for enabling the feature.
✓ Started in 2015, the Walmart subsidiary was recently separated from its e-commerce sibling Flipkart.
✓ PhonePe has about 50 crore registered users and processes 45 percent of transactions on Bharat Bill Pay
System (BBPS).
✓ The company became a fintech in 2017 and launched mutual funds and insurance products.

Follow us: Official Site, Telegram, Facebook, Instagram, Instamojo 321


Q. Food delivery platform Swiggy has partnered with which bank to launch co-branded credit cards?
A) State Bank of India B) Bandhan Bank
C) ICICI Bank D) HDFC Bank
Answer : D
✓ Food delivery platform Swiggy has joined hands with HDFC Bank to launch a co-branded credit card.
✓ The Swiggy-HDFC card will give customers 10 percent cashback on spending on the food delivery
platform.
✓ The co-branded credit card is hosted on the MasterCard payment network.
✓ Swiggy
✓ Founded : 2014
✓ Founders : Sriharsha Majety, Nandan Reddy & Rahul Jaimini
✓ Headquarters : Bangalore, Karnataka
✓ CEO : Sriharsha Majety
Q. The Union government has notified an interest rate of how much percent on deposits under the
Employees Provident Fund scheme for 2022-23?
A) 8.75% B) 6.89%
C) 7.65% D) 8.15%
Answer : D
✓ Employees' Provident Fund Organization (EPFO) has fixed the interest rate on Employees' Provident
Fund (EPF) deposits at 8.15% for the financial year 2022-23, which is higher than the previous year's
interest rate of 8.10%.
✓ About Employees Provident Fund Organization (EPFO)
✓ It is a statutory body under the Ministry of Labor and Employment, responsible for operating provident
funds, pension schemes and insurance schemes for the organised sector workforce in India.
✓ It manages a corpus of over Rs 17 lakh crore with over 6 crore clients.
✓ EPFO aims to provide social security benefits to workers in the organised sector and ensure their financial
stability and well-being.
✓ Since its inception in 1951, the EPFO has expanded its ambit to include pension and insurance benefits,
and in 2014, it launched the Universal Account Number (UAN) to ease EPF account management and
ensure transparency.
Q. RBI Permits Banks to Open Vostro Accounts from How Many Countries?
A) 22 Countries B) 24 Countries
C) 27 Countries D) 30 Countries
Answer : A
✓ Reserve Bank has permitted 20 banks operating in the country to open 92 Special Rupee Vostro Accounts
(SRVAs) of partner banks from 22 countries as part of efforts to promote bilateral trade in local
currencies..
✓ As on July 23, 20 banks in India have been permitted to open SRVAs of partner banks from 22 countries,
including Bangladesh, Belarus, Botswana, Fiji, Germany, Guyana and Israel.
Q. Which Company has acquired a 3.5% stake in RBL Bank for Rs 417 crore?
A) Tata Motors B) Mahindra & Mahindra
C) Honda Motor Company D) Hyundai Motor Company

Follow us: Official Site, Telegram, Facebook, Instagram, Instamojo 322


Answer : B
✓ Mahindra & Mahindra Limited has picked up 3.53% stake in RBL Bank as an investment at a cost of
₹417 crore, the company informed the exchanges.
✓ The stock later pared some gains to close 7.2% higher at ₹239.40 on the NSE.
✓ As per RBI norms, any entity looking to acquire over 5% stake in a bank needs to seek prior approval
from the regulator.
✓ RBL Bank, formerly known as Ratnakar Bank, is an Indian private sector bank.
✓ Founded : 1943
✓ Headquarters : Mumbai, Maharashtra
✓ MD & CEO : R. Subramania kumar
✓ Mahindra & Mahindra Limited (M&M)
✓ Founded : 1945
✓ Founders : J. C. Mahindra & K. C. Mahindra
✓ Headquarters : Mumbai, Maharashtra
✓ Chairman : Anand Mahindra
✓ MD & CEO : Dr. Anish Shah
Q. Who has been appointed as non-executive chairman of Axis Bank for next 3 years?
A) Sunil Mehta B) Shanti Lal Jain
C) NS Vishwanathan D) Srinivasan Varadarajan
Answer : C
✓ Reserve Bank of India has approved the appointment of NS Vishwanathan as non-executive (part-time)
chairman of Axis Bank for three years.
✓ His appointment to the post will come into effect on October 27, 2023. Axis Bank appointed him as an
independent director in May.
✓ Vishwanathan who joined the Reserve Bank of India in 1981, has a career of nearly four decades at the
central bank. He got retired in March 2020 as the deputy governor

Q. Who has released the 'National Multidimensional Poverty Index: A Progress Review 2023'?
A) NITI Aayog B) Ministry of Finance
C) Ministry of Agriculture D) Ministry of Home Affairs
Answer : A
✓ According to the ‘National Multidimensional Poverty Index: A Progress Review 2023, India has
registered a significant decline of 9.89 percentage points in the number of multidimensionally poor, from
24.85% in 2015-16 to 14.96% in 2019-2021.
✓ It claims that about 13.5 crore people came out of. multidimensional poverty during the period, assessed
by identifying.
✓ It said that rural areas witnessed the fastest decline in poverty from 32.59% to 19.28%, primarily due to
decrease in the number of multidimensionally poor in States such as Bihar, Uttar Pradesh, Madhya
Pradesh, Odisha, and Rajasthan. Delhi, Kerala, Goa and Tamil Nadu have the least number of people
facing multidimensional poverty
✓ Along with the Union Territories. Bihar, Jharkhand, Meghalaya, Uttar Pradesh and Madhya Pradesh
top the chart where the percentage of total population who are multidimensionally poor is high.
✓ Multidimensional poverty in urban areas, during the same period, saw a decrease from 8.65% to 5.27%.
✓ Uttar Pradesh registered the largest decline in number of poor with 3.43 crore people escaping
multidimensional poverty.
✓ Between 2015-16 and 2019-21, the MPI value has nearly halved from 0.117 to 0.066 and the intensity of
poverty has reduced from 47% to 44%.

Follow us: Official Site, Telegram, Facebook, Instagram, Instamojo 323


✓ Key facts about National Multidimensional Poverty Index:
✓ It is prepared based on the latest National Family Heath Survey of 2019-21 and is the second edition of
the National Multidimensional Poverty Index (MPI).
✓ Totally 12 parameters of health, education and standard of living are examined in the report.
✓ These include nutrition, child and adolescent mortality, maternal health, years of schooling, school
attendance, cooking fuel, sanitation, drinking water, electricity, housing, assets, and bank accounts.
✓ The report follows Alkire-Foster methodology developed by its technical partners -- the Oxford Poverty
and Human Development Initiative (OPHI) and United Nations Development Programme (UNDP).
Q. Who has stepped down as HDFC chairman after 46 years recently?
A) Deepak Parekh B) Keki Mistry
C) Renu Karnad D) Sashidhar Jagdishan
Answer : A
✓ Deepak Parekh has announced his decision to step down as HDFC chairperson after serving as the
chairman for 46 years.
✓ The board of the HDFC and HDFC Bank conduct a meeting on 30 June to approve the merger, Deepak
Parekh told reporters.
✓ He dubbed the merger as the biggest transaction in India’s corporate history and the proposed entity will
have combined assets of ₹18 lakh crore.
✓ The reverse merger of parent HDFC Ltd with HDFC Bank is effective from July 1.
✓ HDFC Bank became the fourth most valued lender in the world, and narrowed the gap by asset size with
state-owned SBI to be the second largest Indian bank.
Q. Which state has become India’s Top Electronics Exporter of 2023 with the export worth USD 5.37
billion?
A) Kerala B) Telangana
C) Karnataka D) Tamil Nadu
Answer : D
✓ Tamil Nadu has become India’s Top Electronics Exporter of 2023 with the export worth USD 5.37
billion while Uttar Pradesh was second.
✓ Companies like Tata Electronics and Pegatron starting production have helped Tamil Nadu overtake
three states – Uttar Pradesh, Karnataka and Maharashtra – to become India’s top exporter of electronic
goods in FY23.
✓ In 2020-21, of India’s total electronics goods export of $15.59 billion, Tamil Nadu’s share was 11.98
percent. However, this zoomed to 22.83 percent, out of a total value of $23.57 billion, according to
Central government data.
✓ Tamil Nadu is now home to over 15 leading electronics manufacturers, including Foxconn, Tata
Electronics, Salcomp and Pegatron.
Q. The boards of directors of Housing Development Finance Corp Ltd (HDFC) and HDFC Bank have
approved the merger between the two institutions creating the world’s ____ biggest bank by market value.
A) 2nd B) 3rd
C) 4th D) 5th
Answer : C
✓ HDFC merger to create world’s fourth-largest bank by market capitalization.

Follow us: Official Site, Telegram, Facebook, Instagram, Instamojo 324


✓ The boards of directors of Housing Development Finance Corp Ltd (HDFC) and HDFC Bank have
approved the merger between the two institutions creating the world’s fourth-biggest bank by market
value.
✓ The merged entity will be the world’s fourth largest bank by market cap in the world — behind JPMorgan
Chase, Industrial and Commercial Bank of China, and Bank of America.
✓ HDFC Bank announced in April last year that it will be acquiring its parent company Housing
Development Finance Corporation, the largest home financing lender in India, in a $40 billion all-stock
deal.
✓ The new entity now holds a market cap of approximately $172 billion, Rajan said, adding it will become
India’s second most-valued company by market cap after Reliance Industries.
Q. The Board of Control for Cricket in India (BCCI) has signed a three-year sponsorship deal with which of
the following?
A) Vivo B) TATA Group
C) Dream 11 D) Paytm
Answer : C
✓ The Board of Control for Cricket in India (BCCI) signed a three-year sponsorship deal with fantasy sports
gaming platform Dream 11.
✓ The fantasy gaming platform Dream11 has replaced the ed-tech platform Byju as the next lead sponsor
for Team India.
✓ The change in lead sponsor comes ahead of crucial Test series against West Indies which will be the first
task for Team India in the ICC World Test Championship 2023-25 cycle.
✓ Back in 2019, Byju’s replaced Oppo as the sponsor of the national team and signed a deal for three years
until 2022
✓ The learning app extended its contract until 2023 before being replaced by Dream 11, one of the most
popular fantasy platforms around the world.
✓ DREAM 11
✓ Dream11 is an Indian fantasy sports platform
✓ Founded : 2008
✓ Founders : Harsh Jain & Bhavit Sheth
✓ Headquarters : Mumbai, Maharashtra
✓ Owners : Sporta Technologies Pvt Ltd
✓ Board of Control for Cricket in India (BCCI)
✓ Founded : 1928
✓ Headquarters : Wankhede Stadium, Mumbai
✓ 1st President : R E Grant Govan
✓ 36th President : Roger Binny
✓ Vice president : Rajeev Shukla
✓ Secretary : Jay Shah
✓ Men's coach : Rahul Dravid
✓ Women's coach : Ramesh Powar
Q. Government has started a new relief scheme under the 'Mission Vatsalya' scheme for rape victims minor
girls. Mission Vastalya, A monthly grant of __ per child shall be provided for non-institutional care,
Kinship or foster care or after care .
A) Rs. 2000 B) Rs. 3000
C) Rs. 5000 D) Rs. 4000
Answer : D

Follow us: Official Site, Telegram, Facebook, Instagram, Instamojo 325


✓ 'Mission Vatsalya' scheme for rape victims minor girls.
✓ The Central Government has started a new relief scheme under the 'Mission Vatsalya' scheme in relation
to the abandonment of the families of minor girls who are victims of rape when they become pregnant.
✓ Under this, arrangements have been made to provide shelter, food and legal aid to those minors.
✓ According to Women and Child Development Minister Smriti Irani, the new scheme launched under the
aegis of 'Nirbhaya Yojana' aims to ensure institutional and financial support to pregnant minor victims
who have no means to take care of themselves.
✓ Main Initiatives of the Scheme:
✓ Under this, the administrative structure of 'Mission Vatsalya' has also been used in collaboration with
state governments and child care institutions to provide help to minor victims at the grassroots level.
✓ This additional assistance under the scheme will be available at the level of Child Care Institutions (CCIs)
for girls up to 18 years of age and care centers for young women up to 23 years of age.
✓ Along with legal aid, the victim will also be provided safe transportation to attend court hearings.
✓ To facilitate access to justice for minor victims, the Center has set up 415 POCSO (Protection of Children
from Sexual Offences) fast track courts in the country.
✓ Mission Vatsalya.
✓ There is an umbrella scheme for women and child protection services in the country.
✓ Scheme launched: in 2021 by the Ministry of Health and Family Welfare
✓ Implementation of the scheme: 1 April 2022
✓ Objective: To ensure a healthy and happy childhood for the children of the country.
✓ Through this, a National Human Milk Bank was established in the capital Delhi.
✓ Through Mission Vatsalya Yojana 2023, women are encouraged to breastfeed.
✓ A breastfeeding counseling center has also been formed through this scheme.
✓ Mission Vatsalya is brought for the better health of babies and women.
✓ Mission Vatsalya was allocated Rs 900 crore in the budget for the financial year 2021-2022.
✓ Efforts are being made to reduce the infant mortality rate through this scheme.
✓ A monthly grant of Rs. 4000/- per child shall be provided for Sponsorship or Foster Care or After Care.
Q. Which of the following has launched the eSARAS mobile App to support marketing for products made
by women of Self-help Groups (SHGs)?
A) Atal Pension Yojana
B) Smart Cities Mission
C) Pradhan Mantri Kaushal Vikas Yojana
D) Deendayal Antyodaya Yojana- National Rural Livelihoods Mission
Answer : D
✓ The Deendayal Antyodaya Yojana- National Rural Livelihoods Mission (DAY-NRLM) launched the
eSARAS mobile App to support marketing for products made by women of Self-help Groups (SHGs).
✓ eSARAS is an e-commerce mobile app that provides a platform for marketing handicrafts and hand-
looms made by SHG Didis.
✓ The eSARAS fulfillment centre will be managed by the Foundation for Development of Rural Value
Chains (FDRVC), a not-for-profit company constituted jointly by the Ministry of Rural Development
and Tata Trust.
✓ This fulfillment centre will be used for processing, packaging and shipping of products that customers
purchase through the eSARAS portal and eSARAS mobile app. It will handle the logistics required to
bring an online order to a customer’s doorstep.
✓ Deendayal Antyodaya Yojana - National Rural Livelihoods Mission (DAY-NRLM) is the flagship
program of the Ministry of Rural Development.

Follow us: Official Site, Telegram, Facebook, Instagram, Instamojo 326


Q. Which bank has launched a digital service under the ‘Project WAVE’ initiative in partnership with
National E-Governance Service Limited (NeSL)?
A) Canara Bank B) Bank of Baroda
C) Indian Bank D) Indian Overseas Bank
Answer : C
✓ Indian Bank rolls out digital services under ‘Project WAVE’ initiative
✓ Indian Bank, in partnership with National E-Governance Service Limited (NeSL), has rolled out e-BG
(electronic bank guarantee) services under Project WAVE to reduce the number of old paper-based
process.
✓ Besides, Indian Bank has also introduced a new facility under Project WAVE, to ensure fully end-to-end
digitally pre-approved business loans, for easy and smooth loan process.
✓ The facility is available to process a loan of ₹25 lakh.
✓ Indian Bank
✓ Founded : 1907
✓ Headquarters : Chennai, Tamil Nadu
✓ MD & CEO : Shanti Lal Jain

Q. Which of the following company is planning to launch a microblogging app ‘Threads’?


A) Meta B) Microsoft
C) Apple D) Google
Answer : A
✓ Facebook's Meta Launches 'Twitter Killer' Social Network Threads.
✓ Instagram maker Meta recently launched a new social media platform called Threads.
✓ Threads aims to capitalise on the perceived volatility of Twitter, which is currently owned by billionaire
Elon Musk.
✓ The app is now available in over 100 countries and can be downloaded from the Apple App Store and
Google Play Store.
✓ Similar to Twitter, threads allow users to share short text messages that can be liked, reposted, and replied
to by other users.
✓ Threads does not include direct messaging features, which differentiates it from Twitter.
✓ On threads, users can create posts with up to 500 characters and can also share links, photos and videos
of up to five minutes.
✓ About Meta
✓ Founded - February 2004, Cambridge, Massachusetts, United States
✓ Founder - Mark Zuckerberg
✓ Headquarters - Menlo Park, California, United States

Q. Who has been appointed as the new Executive Director by the Reserve Bank of India?
A) P. Vasudevan B) Urjit Patel
C) PK Sinha D) Rajeev Kumar
Answer : A
✓ The Reserve Bank of India (RBI) has appointed P. Vasudevan as Executive Director.

Follow us: Official Site, Telegram, Facebook, Instagram, Instamojo 327


Q. Who has appointed as FPSB of India Chief Executive Officer (CEO), effective 1 August 2023?
A) Krishna Mishra B) Bikram Nag
C) Emandi Sankara Rao D) Kristalina Georgieva
Answer : A
✓ Financial Planning Standards Board (FPSB) of India appointed Krishna Mishra as the Chief Executive
Officer (CEO) with effect from 1 August 2023.
✓ FPSB India is the Indian subsidiary of FPSB, a global standard-setting organization for the financial
planning profession.
✓ Krishna Mishra will lead the strategy and operations of FPSB India as CEO.
✓ His primary focus will be on advancing the financial planning profession in India.
✓ Financial Planning Standards Board (FPSB).
✓ It is a global non-profit organisation that sets and supports standards for the financial planning profession.
✓ Its primary mission is to benefit the global public by establishing, promoting and implementing
professional standards for financial planning.
✓ The FPSB develops and maintains the Certified Financial Planner (CFP) certification program, which is
recognized as the global standard for financial planning excellence.
✓ The FPSB aims to increase consumer confidence in the financial planning profession by establishing and
maintaining global standards.
✓ It aims to protect the public interest and promote professionalism, integrity and competence among
financial planners globally.
Q. RBI’s working group has recommended the inclusion of the rupee in the Special Drawing Rights (SDR)
basket and recalibration of the foreign portfolio investor (FPI) regime to accelerate the pace of
internationalization of the rupee. These recommendations have been given by an Inter-Departmental
Group (IDG) headed by_.
A) Deepak Mohanty B) Ajay Shankar
C) Janki Ballabh D) Radha Shyam Ratho
Answer : D
✓ RBI’s working group has recommended the inclusion of the rupee in the Special Drawing Rights (SDR)
basket and recalibration of the foreign portfolio investor (FPI) regime to accelerate the pace of
internationalization of the rupee.
✓ The recommendations by an Inter-Departmental Group (IDG), headed by RBI Executive Director
Radha Shyam Ratho.
✓ The group was constituted by RBI Deputy Governor T Rabi Sankar to review the position of the rupee
as an international currency and to frame a road map for the internationalization of the domestic
currency.
✓ Internationalization of the rupee is a process that involves increasing the use of the local currency in
cross-border transactions.
✓ The SDR is an international reserve asset created by the IMF (International Monetary Fund) to
supplement the official reserves of its member countries.
✓ The value of the SDR is based on a basket of five currencies—the U.S. dollar, the euro, the Chinese
renminbi, the Japanese yen, and the British pound sterling.
Q. Piramal Capital & Housing Finance Limited has launched five all-women branches as part of its
commitment to diversity and equal opportunities. What is the name of these branches?
A) Maitreyi B) Lady Brunchers
C) Coyote Sisters D) Pink Ladies

Follow us: Official Site, Telegram, Facebook, Instagram, Instamojo 328


Answer : A
✓ Piramal Finance opened the first all women branch named "Maitreyi" at Thripunithura near Kochi to
cater to the unique needs of women customers.
✓ Kerala was chosen as the location for the inaugural branch due to its demographic and market dynamics,
50% of customers in Kerala are women, as against the national average of 20%.
✓ Kerala also has a high percentage of self-employed individuals, highlighting the entrepreneurial spirit of
the state's population.
✓ Each Maitreyi branch has a dedicated team of 7-15 women employees to cater to the specific needs of
the customers.
✓ The branch offers a wide range of services including home loans, MSME loans with a focus on home
construction and property loans.
Q. Who has been appointed as MD & CEO of UK based Lloyds Banking Group?
A) Pramod Kumar Mishra B) Ajit Kumar Doval
C) Rajiv Gauba D) Sirisha Voruganti
Answer : D
✓ Sirisha Voruganti appointed MD & CEO of Lloyds Banking Group's Technology Center in India.
✓ Lloyds Banking Group, one of the leading UK-based financial services groups, has appointed Sirisha
Voruganti as Chief Executive Officer and Managing Director of its new Lloyds Technology Centre.
Q. Which Municipal Corporation has become the first urban body in the country to get the Extended
Producers Responsibility (EPR)?
A) Mumbai Municipal Corporation B) Indore Municipal Corporation
C) Pune Municipal Corporation D) Surat Municipal Corporation
Answer : B
✓ Indore Municipal Corporation (IMC) has become the first urban body in the country to get the Extended
Producers Responsibility (EPR) credit by recycling the banned single-use plastic items after seizing them.
✓ Notably, Indore was adjudged India’s cleanest city for the sixth time in a row last year in the central
government’s annual cleanliness survey.
✓ Single-use plastic is completely banned in the city. About eight tonnes of such plastic in the recent past
and stopped from getting circulated.
✓ On average, 1,162 tonnes of solid waste is generated every day in Indore, which includes about 164
tonnes of plastic waste.
✓ This waste is collected under different categories from every doorstep in the city by civic vehicles and
disposed of the same day in the plant being run under the PPP model.
Q. India's Make in India initiative suffered a major setback when Vedanta Ltd's deal with Foxconn to
manufacture semiconductors in India was cancelled. Foxconn company belongs to which country?
A) Taiwan B) Bhutan
C) Sri Lanka D) Maldives
Answer : A
✓ Foxconn pulls out of joint venture with Vedanta to make semiconductors.
✓ India's Make in India initiative suffered a major loss when Vedanta Limited's agreement with Taiwanese
company Foxconn to manufacture semiconductors in India was cancelled..
✓ It was established production for semiconductors in India.

Follow us: Official Site, Telegram, Facebook, Instagram, Instamojo 329


✓ According to a report, the company has expressed confidence about India's semiconductor development.
The company will fully support the "Make in India" campaign of the Government of India.
✓ According to Vedanta, it will work with local partners to meet the needs of its stakeholders.
✓ Foxconn and Vedanta last year signed an agreement to invest $19.5 billion to build a semiconductor and
display production plant in Gujarat.
✓ Vedanta Limited:
✓ Vedanta Limited is an Indian multinational mining company headquartered in New Delhi,
✓ Founder: Dwarka Prasad Agarwal
✓ Established: 25 June 1965
✓ CEO: Sunil Duggal
✓ Subsidiaries: Hindustan Zinc, Bharat Aluminum Company, More
✓ Parent organization: Vedanta Resources
Q. Which of the following has constituted a five-member working group for implementing the
recommendations of the Reserve Bank of India’s committee on the “working of the Asset Reconstruction
Companies (ARCs)”?
A) Indian Banks’ Association B) Finance Ministry
C) Reserve bank of India D) NITI Aayog
Answer : A
✓ The Indian Banks’ Association (IBA) has constituted a five-member working group for implementing the
recommendations of the Reserve Bank of India’s committee on the “working of the Asset Reconstruction
Companies (ARCs)”.
✓ The Group comprises Ajit Kumar (MD and CEO of Secondary Loan Market Association), Hari (CEO
of Association of ARCs in India), Mythili Balasubramanian (Executive Director of Edelweiss ARC),
Dharmendra Pali (senior officials from SBI) and Anupama Ranade (senior officials from ICICI Bank).
✓ In light of the committee’s recommendations aimed at enabling ARCs to fulfill the increasing demand of
the financial sector, the working group will undertake a thorough review of assignment agreements and
trust deed templates.
✓ RBI had set up a committee to undertake a comprehensive review of the working of ARCs under the
chairmanship of Sudarshan Sen, former ED, RBI, on April 28, 2021. The report of the committee was
released on November 2, 2021
Q. Who has been appointed as the new MD & CEO of SBI Cards & Payment Services (SBI Card)?
A) Abhijit Chakravorty B) Phanindra Subramanya
C) K. Satyanarayana Raju D) Sumant Kathpalia
Answer : A
✓ SBI appoints Abhijit Chakravorty as SBI Card’s CEO.
✓ The country’s largest pure-play credit card issuer, SBI Cards & Payment Services (SBI Card) has
appointed Abhijit Chakravorty as MD & CEO for two years.
✓ Chakravorty, who is presently a Deputy Managing Director at SBI, will assume charge of his new role
on August 12.
✓ Incumbent MD & CEO Rama Mohan Rao Amara has tendered his resignation owing to his transfer to
SBI. Amara will demit office on August 11.
✓ Amara was appointed as MD & CEO of SBI Card in January 2021.
✓ SBI Cards & Payment Services Ltd previously known as SBI Cards & Payment Services Private Limited,
is a credit card company and payment provider in India
✓ Founded : 1998
✓ Headquarters : Gurugram

Follow us: Official Site, Telegram, Facebook, Instagram, Instamojo 330


Q. Who has been reappoints as Chairman & Managing Director of 'ITC Ltd' effective from 22 July, 2024?
A) Sanjiv Puri B) Debasish Panda
C) Rajiv Kumar D) Madhabi Puri Buch
Answer : A
✓ ITC Ltd’s board has recommended the re-appointment of Sanjiv Puri as the company’s chairman &
managing director for a period of 5 years, effective 22 July, 2024.
✓ ITC Limited is an eminent Indian conglomerate, which holds a prominent position among the largest
companies in the country.
✓ Its operations encompass a diverse range of business sectors, such as tobacco manufacturing, food
processing, retailing, and financial services
✓ ITC Limited is an Indian conglomerate company headquartered in Kolkata.
✓ Founded : 1910
✓ Chairman & MD : Sanjiv Puri

Q. ‘SAMARTH scheme’, which was seen in the news, is associated with which Ministry?
A) Ministry of Finance B) Ministry of Home affairs
C) Ministry of defence D) Ministry of Textiles
Answer : D
✓ The panel of implementation partners of SAMARTH scheme has been expanded by inducting 43 new
implementation partners.
✓ The objective of this expansion is to enhance the reach and effectiveness of the scheme.
✓ Additionally, the scheme has set a new target of training around 75,000 beneficiaries, which will be
allocated to training partners.
✓ With the increase in cost norms by 5%, the funding pattern of the scheme has also been revised. This
amendment provides the much needed additional financial support to the industries involved in
imparting skills under Samarth scheme.
✓ About Samarth Scheme
✓ Scheme Name: Scheme for Capacity Building in Textile Sector (SAMARTH).
✓ Nature of the Scheme: Demand-driven and placement-oriented Umbrella Skill Program.
✓ Objective: To encourage and supplement the efforts of the industry in generating employment in the
organised textile and allied sectors, covering the entire value chain of textiles (excluding spinning and
weaving).
✓ Focus Areas: Entry level skilling and upskilling/reskilling programs to improve the productivity of
existing workforce in the apparel and garments sectors.
✓ Implementation Period: Till March 2024
✓ Nodal Ministry: Ministry of Textiles.
✓ This initiative is part of the steps taken by the government to make the textile sector worth 250 billion
USD by the end of this decade from the present 150-155 billion USD.

Q. Which company will acquire online marketplace OLX India’s auto sales business for Rs 537 crore?
A) Hyundai Motor Company B) Tata Motors
C) AutoTrader D) CarTrade Tech Ltd
Answer : D
✓ Mumbai-headquartered used car platform CarTrade Tech is lining up Rs 537 crore towards acquiring
Olx India’s auto sales business.
✓ It will acquire 100 percent stake of Sobek Auto India (Sobek), which operates Olx India’s auto business.

Follow us: Official Site, Telegram, Facebook, Instagram, Instamojo 331


✓ Sobek entered into an Intellectual Property License Agreement and a Transitional Support Agreement
for the use of certain brands and technology.
✓ The Mumbai-headquartered company is backed by investors including Warburg Pincus, Temasek,
JPMorgan, and March Capital, and operates as a multi-channel auto platform.
✓ In terms of financial turnover, Sobek’s revenue has grown from Rs 556.42 crore in FY2020 to Rs 1,110.40
crore in FY2022.
✓ OLX Group is a Dutch-domiciled online market place headquartered in Amsterdam
Q. Which bank has won the Best Performance in CASA at ICC Emerging Asia Banking Conclave & Awards
2022?
A) Karur Vyasya Bank B) Jammu & Kashmir Bank
C) Indian Overseas Bank D) Punjab & Sind Bank
Answer : B
✓ Jammu & Kashmir Bank has been felicitated for the Best Performance in CASA-India (1st Runner up in
Small Bank Category) at ICC Emerging Asia Banking Conclave & Awards 2022.
✓ The function was organized by the Indian Chamber of Commerce in Goa.
✓ It is a Indian Chamber of Commerce (ICC) is a non-governmental trade association and advocacy group
headquartered in Calcutta.
✓ The event was participated by banks from India, Nepal, Bhutan, Bangladesh, Sri Lanka & Myanmar.
Q. Which insurance company has won the Insurer Innovation Award 2023 for the APAC region at the
World’s Digital Insurance Awards?
A) Max Life Insurance B) Cholamandalam MS Insurance
C) Bharti AXA Insurance D) Bajaj Allianz Insurance
Answer : D
✓ Bajaj Allianz Life Insurance wins Insurer Innovation Awards 2023.
✓ One of the leading private life insurer in India, Bajaj Allianz Life Insurance has won the Insurer
Innovation Award 2023 for the APAC region at the World’s Digital Insurance Awards.
✓ Digital Insurer, acknowledges Bajaj Allianz Life’s groundbreaking WhatsApp Conversational Platform
empowering customers to digitally services their policies seamlessly.
✓ Bajaj Allianz Life emerged as the regional finalist winner for Asia Pacific region, amongst a sterling list
of Insurance majors like HSBC Life, AlA China, Cathay Life Insurance, MSIG, and Singlife
Q. Which bank has inaugurated IFSC banking unit at GIFT City, Gandhinagar?
A) Union Bank of India B) Punjab National Bank
C) Bandhan Bank D) Bank of India
Answer : D
✓ Bank of India has inaugurated its IFSC Banking Unit (IBU) at GIFT SEZ area in Gandhinagar.
✓ Bank of India expects to conduct $ 4 billion business from GIFT City in next 3 yrs.
✓ The first IBU was set up at GIFT City by Yes Bank in 2015.

Follow us: Official Site, Telegram, Facebook, Instagram, Instamojo 332


BEST MCQ BANKING, ECONOMY AND FINANCIAL AWARENESS
JUNE 2023

Q. Which institution raised ₹10,000 crore through listed bonds, which is the largest debt issuance by an
AIFI?
A) Reserve Bank Information Technology Private Limited (ReBIT)
B) National Bank for Financing Infrastructure and Development (NaBFID)
C) Indian Financial Technology and Allied Services (IFTAS)
D) Reserve Bank Innovation Hub (RBIH)
Answer : B
✓ The National Bank for Financing Infrastructure and Development (NaBFID) has raised ₹10,000 crore
via maiden issuance of listed bonds of 10-year duration at a coupon rate of 7.43 per cent.
✓ The development finance institution’s infrastructure bonds were oversubscribed by approximately 4.7
times, against the base issue of ₹5,000 crore.
✓ This is the largest debt issuance by an All India Financial Institution (AIFI).
✓ About NaBFID:
✓ It was set up in 2021, by an Act of the Parliament (The National Bank for Financing Infrastructure and
Development Act, 2021).
✓ It is a specialized Development Finance Institution in India.
✓ Objectives: Addressing the gaps in long-term non-recourse finance for infrastructure development,
strengthening the development of bonds and derivatives markets in India, and sustainably boosting the
country’s economy.
✓ It shall be regulated and supervised by RBI as an All-India Financial Institution (AIFI).
✓ The authorized share capital of the NaBFID is Rs one lakh crore. The initial paid-up capital of NaBFID
is Rs 20,000 crore.
✓ The headquarters of NaBFID is in Mumbai.
✓ What is a Development Finance Institution (DFI)?
✓ These are organizations owned by the government or public institutions to provide funds for
infrastructure and large-scale projects, where it often becomes unviable for large banks to lend.
✓ They provide two types of funds- Medium (1-5 years) and Large (< 5 years).
Q. What is the maximum limit that individuals are allowed to remit in a year under the Liberalised
Remittance Scheme?
A) USD 25,000 B) USD 3,00,000
C) USD 5,00,000 D) USD 2,50,000
Answer : D
✓ Twenty per cent tax on Liberalised Remittances Scheme (LRS) of the Reserve Bank of India is set to kick
off soon.
✓ The Union Budget 2023 introduced a Tax Collection at Source (TCS) for outward foreign remittance
under LRS (other than for Education and medical purpose) of 20% on the entire value.
✓ Tax liability on profit made: If any profit is made on foreign investments made under LRS, it is taxable
in India based on how long the investment was held.
✓ About Liberalised Remittances Scheme (LRS):

Follow us: Official Site, Telegram, Facebook, Instagram, Instamojo 333


✓ LRS allows Indian residents to freely remit up to USD $250,000 per financial year for current or capital
account transactions or a combination of both. Any remittance exceeding this limit requires prior
permission from the RBI.
✓ The scheme was introduced by RBI on February 4, 2004.
✓ Who can remit funds under LRS?
✓ Only individual Indian residents, including minors, are permitted to remit funds under LRS.
✓ Corporates, partnership firms, HUF, trusts, etc., are excluded from its ambit.
✓ Frequency of Remittances:
✓ There are no restrictions on the frequency of remittances under LRS.
✓ Once a remittance is made for an amount up to USD 2,50,000 during the financial year, a resident
individual would not be eligible to make any further remittances under this scheme.
Q. Who has been launched a Financial Inclusion Dashboard 'ANTARDRIHSTI'?
A) Urjit Patel B) Shaktikanta Das
C) Nirmala Sitharaman D) Droupadi Murmu
Answer : B
✓ Recently, the Reserve Bank of India Governor launched a financial inclusion dashboard named
'Antardrishti'.
✓ About Antardrishti dashboard:
✓ It provides the necessary knowledge to evaluate and track the development of financial inclusion by
recording relevant data.
✓ It will also make it possible to assess the degree of financial exclusion at a local level across the nation so
that such places may be addressed.
✓ What is Financial Inclusion?
✓ It aimed to provide access to affordable and appropriate financial products and services, such as savings
accounts, credit, insurance, and payment systems.
✓ The Reserve Bank has been promoting financial inclusion through various policy initiatives.
✓ To measure the extent of financial inclusion the central bank had constructed the Financial Inclusion (FI)
Index in 2021.
✓ It is based on three dimensions of financial inclusion -- 'Access', 'Usage' and 'Quality'.
✓ The FI-Index has been conceptualised as a comprehensive index incorporating details of banking,
investments, insurance, postal as well as the pension sector in consultation with the government and
respective sectoral regulators.
✓ The index captures information on various aspects of financial inclusion in a single value ranging between
0 and 100, where 0 represents complete financial exclusion and 100 indicates full financial inclusion.

Q. PM PRANAM, a proposed scheme, aims to achieve which of the following objectives?


A) To reduce the use of chemical fertiliser
B) Development of pilgrimage destinations
C) Vulnerable sections of the society
D) To provide electricity connections to all households
Answer : A
✓ Recently, the Cabinet Committee on Economic Affairs (CCEA) approved the PM Programme for
Restoration, Awareness, Generation, Nourishment and Amelioration of Mother Earth (PM-PRANAM)
scheme.
✓ About the PM-PRANAM scheme:
✓ It aims to reduce the use of chemical fertilisers by incentivising the state.

Follow us: Official Site, Telegram, Facebook, Instagram, Instamojo 334


✓ Under the scheme, the states which will adopt alternative fertilisers will be incentivised with the subsidy
that is saved by reducing the use of chemical fertilisers.
✓ The main objective of the scheme is to encourage the balanced use of fertilisers in conjunction with bio
fertilisers and organic fertilisers.
✓ Funding mechanism:
✓ It will not have a separate budget.
✓ It will be financed by the savings of existing fertiliser subsidies under schemes.
✓ Half of the subsidy savings will be passed on to the state that saves the money as a grant.
✓ Out of this grant, 70% can be used to create assets related to the technological adoption of alternate
fertilisers and alternate fertiliser production units at the village block, and district levels.
✓ The remaining 30% of grant money can be used to reward and encourage farmers, panchayats, and other
stakeholders involved in fertiliser reduction and awareness generation.
✓ It aims to bring down the subsidy burden on chemical fertilisers, which is estimated to reach Rs 2.25 lakh
crore in 2022-23 — 39% higher than 2021 figure of Rs 1.62 lakh crore.
✓ The total requirement of four fertilisers — Urea, DAP (Di-ammonium Phosphate), MOP (Muriate of
potash), NPKS (Nitrogen, Phosphorus and Potassium) — in the country increased by 21% to 640.27 lakh
metric tonnes (LMT) in 2021-22 from 528.86 lakh metric tonnes in 2017-18.
Q. Prime Minster Street Vendor’s AtmaNirbhar Nidhi (PM SVANidhi) Scheme has recently successfully
completed how many years?
A) 2 B) 3
C) 4 D) 5
Answer : B
✓ PM SVANidhi Scheme Celebrates Successful Completion of 3 Years.
✓ Union Minister Hardeep Singh Puri complimented the Prime Minster Street Vendor’s AtmaNirbhar
Nidhi (PM SVANidhi) Scheme on the occasion of its successful completion of 3 momentous years.
✓ The scheme was launched in June 2020.
✓ Objective – To empower street vendors by restoring self-employment, self-sustenance, and self-
confidence.
✓ Over the years, PM SVANidhi has emerged as one of the most beneficial and rapidly growing micro-
credit schemes in India, fostering financial inclusion, digital literacy, and offering dignity and stability to
street vendors.
✓ The scheme offers working capital loans in three installments to street vendors and has successfully
provided microcredit to over 3.6 million street vendors across the country.
✓ As of 30th June 2023, a sum of Rs. 4.85 million loan applications have been approved with over 4.64
million loans disbursed amounting to a cumulative sum of Rs. 5,795 crore.
✓ The Government of India has extended the PM Street Vendor’s AtmaNirbhar Nidhi (PM SVANidhi)
Scheme beyond March, 2022.
✓ The extension of the lending period will be till December 2024.
✓ Introduction of 3rd loan of upto ₹50,000 in addition to 1st & 2nd loans of ₹10,000 and ₹20,000
respectively.
✓ To extend ‘SVANidhi Se Samriddhi’ component for all beneficiaries of PM SVANidhi scheme across the
country.
✓ About PM Svanidhi Yojana .
✓ PM Svanidhi Yojana was launched on June 1, 2020.
✓ This scheme was started with the aim of providing financial support to small shopkeepers and hawkers.
✓ Under the PM Svanidhi Yojana, people who sell goods on the street tracks or do other small jobs can
take a loan of up to ₹ 10,000 from the bank.
✓ After repaying the loan taken in the first instance, the beneficiary can get a loan of up to ₹ 20,000 in the
second time and up to ₹ 50,000 in the third time.

Follow us: Official Site, Telegram, Facebook, Instagram, Instamojo 335


✓ The tenure of the Svanidhi scheme was earlier till March 2022, but the government has extended the
deadline of the Svanidhi scheme till December 2024.
Q. Which Union Ministry launched the ‘Report Fish Disease(RFD)’ App?
A) Ministry of Road Transport and Highways
B) Ministry of Agriculture & Farmers Welfare
C) Ministry of Women and Child Development
D) Ministry of Fisheries, Animal Husbandry and Dairying
Answer : D
✓ The Union Minister for Fisheries, Animal Husbandry and Dairying will launch the Report Fish Disease
(RFD) App at Krishi Bhawan, New Delhi.
✓ About Report Fish Disease App
✓ It will strengthen the farmer-based disease reporting system and for improving the reporting of aquatic
animal diseases in the country.
✓ The app has been developed by ICAR-NBFGR under National Surveillance Programme for Aquatic
Animal Diseases (NSPAAD).
✓ Funded under: Pradhan Mantri Matsya Sampada Yojana by the Department of Fisheries, Ministry of
Fisheries, Animal Husbandry and Dairying, Government of India.
✓ It will help the farmers in reporting the incidence of diseases in finfish, shrimps, and molluscs on their
farms with the field-level officers and fish health experts.
✓ Key Facts about Pradhan Mantri Matsya Sampada Yojana
✓ It is a flagship scheme for focused and sustainable development of the fisheries sector in the country with
an estimated investment of Rs. 20,050 crores for its implementation during a period of 5 years from FY
2020-21 to FY 2024-25 in all States/Union Territories.
✓ What are the aims of the scheme?
✓ It aims at enhancing fish production by an additional 70 lakh tonnes by 2024-25.
✓ Increasing fisheries export earnings to Rs.1,00,000 crore by 2024-25.
✓ Doubling of incomes of fishers and fish farmers, reducing post-harvest losses from 20-25%.
✓ Nodal Ministry: Ministry of Fisheries, Animal Husbandry and Dairying
Q. UTPRERAK’, a centre of excellence which was launched recently is related to
A) Energy efficiency B) Food engineering
C) Vaccine research D) Artificial Intelligence
Answer : A
✓ Recently, the Union Minister of State for Power inaugurated UTPRERAK, a dedicated Centre of
Excellence, to accelerate industry adoption of clean technologies in New Delhi.
✓ About UTPRERAK:
✓ The Unnat Takniki Pradarshan Kendra (UTPRERAK) is the Centre of Excellence to Accelerate
Adoption of Energy Efficient Technologies and seeks to play a catalytic role in improving the energy
efficiency of the Indian industry.
✓ It is also named as Advanced Industrial Technology Demonstration Centre (AITDC)
✓ It has been set up by the Bureau of Energy Efficiency (BEE), Ministry of Power
Q. Which country is the world's second largest producer of crude steel?
A) United States B) China
C) India D) Japan

Follow us: Official Site, Telegram, Facebook, Instagram, Instamojo 336


Answer : C
✓ India emerges as the world's second largest producer of crude steel.
✓ According to the Union Minister of Steel and Civil Aviation, Shri Jyotiraditya M. Scindia, India has
moved from the fourth largest producer of crude steel to the second largest producer
✓ From 2014-15 to 2022-23, India sees a 42% increase in crude steel production, from 88.98 MT to 126.26
MT.
✓ China remains the largest exporter of crude steel globally and India has emerged as the second largest
producer.
✓ India is a net exporter of steel, exporting 6.72 MT of finished steel in 2022-23, while importing 6.02 MT.
✓ In the financial year 2022-23, India is expected to produce 122.28 MT of finished steel, showing a growth
of 49% as compared to 81.86 MT in 2014-15.
✓ Steel CPSEs (SAIL, NMDC, MOIL, KIOCL, MSTC, and MECON) mobilised ₹90,273.88 crore of their
own resources for CAPEX from 2014-15 to 2022-23.
✓ Steel CPSEs paid a dividend of ₹21,204.18 crore to the Government of India during the same period.

Q. The World Competitiveness Index is an annual list compiled by which organisation?


A) International Monetary Fund (IMF)
B) Lowy Institute
C) Institute for Management Development (IMD)
D) World Economic Forum
Answer: C
✓ Recently, the International Institute for Management Development (IMD) published the World
Competitiveness Index.
✓ Denmark, Ireland, and Switzerland have been named the top three among 64 economies measured for
their global competitiveness.
✓ India fell three rungs to finish 40th but is still in a better position than it was between 2019-2021 when it
was placed 43rd three years in a row.
✓ India improved in government efficiency but fared slightly poorer than other countries in business
efficiency, infrastructure, and economic performance.
✓ Specifically, the top three measures that helped India in its score are exchange rate stability, compensation
levels, and improvements in pollution control.
✓ About the World Competitiveness Index:
✓ The IMD World Competitiveness Yearbook (WCY), was first published in 1989.
✓ It is a comprehensive annual report and worldwide reference point on the competitiveness of countries.
✓ It analyses and ranks countries according to how they manage their competencies to achieve long-term
value creation.
✓ It is based on 336 competitiveness criteria and four factors, namely Economic performance, Government
efficiency, Business efficiency, and Infrastructure.
Q. Which institution notified ‘Indian Standards Related to AYUSH Herbs and Products’?
A) Central Board of Indirect taxes and Customs (CBIC)
B) Bureau of Indian Standards (BIS)
C) Central Board of Direct Taxes (CBDT)
D) Federation of Indian Chambers of Commerce & Industry (FICCI)
Answer : B

Follow us: Official Site, Telegram, Facebook, Instagram, Instamojo 337


✓ BIS recently notified 31 Indian standards related to Ayush, which includes 30 herbs and 1 product
(stainless steel neti pot).
✓ These standards were recently published through a gazette notification.
✓ About Bureau of Indian Standards (BIS):
✓ BIS is the National Standards Body of India established under the BIS Act 2016.
✓ Objective: Harmonious development of the activities of standardisation and quality assurance of goods
and articles.
✓ It works under the Ministry of Consumer Affairs, Food & Public Distribution.
✓ BIS represents India in International Organization for Standardization (ISO)and International
Electrotechnical Commission (IEC).
✓ What is ISO?
✓ It is an independent, non-governmental international organisation with a membership of 167 national
standard bodies.
✓ What is IEC?
✓ The IEC is a global, not-for-profit membership organisation whose work underpins quality infrastructure
and international trade in electrical and electronic goods

Q. Which country is the largest producer of crude steel in the world?


A) Japan B) China
C) Russia D) United States
Answer : B
✓ Top 10 steel producing countries in the world by March 2023
1. China: 95.7 million metric tons (MT)
2.India: 11.4 MT
3.Japan: 7.5 MT
4.United States: 6.7 MT
5.Russia: 6.6 MT
6. South Korea: 5.8 MT
7. Türkiye: 3.3 MT
8. Germany: 2.7 MT
9. Brazil: 2.7 MT
10. Iran: 2.2 MT
Q. Which company has retained its first position as India's most valuable brand in 2023 with $26,381
million?
A) LIC B) TATA Group
C) Infosys D) Reliance Industries
Answer : B
✓ Tata Group retains 1st Position as India’s Most Valuable Brand: INDIA 100 2023 Brand Finance
Reporti.

Follow us: Official Site, Telegram, Facebook, Instagram, Instamojo 338


✓ The brand valuation firm Brand Finance released its INDIA 100 2023: The annual report on the most
valuable and strongest Indian brands- June 2023 report in which the Tata Group has retained its 1st
position as India’s most valuable brand at $26,381 Million in 2023.
✓ It has shown a growth at 10.3% as compared to 2022.
✓ This is the first time that an Indian brand has breached the brand value mark of $25 billion.
✓ Tata is followed by the Infosys at $13 billion and LIC at $9.8 billion at 2nd and 3rd position respectively.
✓ Both also retained their position as 2022.
✓ Mahindra Group is fastest growing brand amongst the top ten as it performed exceptionally well in FY23
with record profits crossing Rs 10,000 crore for the first time on revenues of Rs 1,21,269 crore.
✓ Taj is India’s strongest brand for the second consecutive year with Brand Strength Index (BSI) score of
89.4 out of 100 and a corresponding elite AAA brand strength rating.
TATA GROUP IN NEWS 2023
▪ Tata Group retains 1st Position as India’s Most Valuable Brand.
▪ Tata Sons Chairman Emeritus Ratan Tata has received the Order of Australia (AO), Australia's highest
civilian honour.
▪ Tata Power approves reappointment of Praveer Sinha as CEO and MD.
▪ K Krithivasan appointed CEO Designate of Tata Consultancy Services (TCS) after the current CEO and
MD Rajesh Gopinathan stepped down.
▪ Tata Sons chairman N Chandrasekaran claimed on 13 February that the Tata group is set to record the
highest growth in its history and that its compounded annual growth is expected to be around 20% in
2022-23.
▪ The Government of India has appointed Tata Sons Chairman N Chandrasekaran as the chairman of the
B20 to lead the business agenda during India’s G20 presidency.
▪ Tata Group replaces Vivo as the title sponsor of IPL 2022-23 editions.
▪ Tata Group bags Women's Premier League title sponsorship rights till 2027.

Q. Which Union Ministry launched ‘PM-Kisan Mobile App with Face Authentication Feature’?
A) Ministry of Agriculture and Family Welfare
B) Ministry of Corporate Affairs
C) Ministry of Minority Affairs
D) Ministry of Commerce and Industry
Answer : A
✓ Recently, the PM-Kisan Mobile App with Face Authentication Feature was launched by Union
Agriculture and Farmers’ Welfare Minister.
✓ PM KISAN App:
✓ The newly launched application has the feature of face Authentication.
✓ From this app, farmers can complete e-KYC remotely, sitting at home easily by scanning their face
without OTP or fingerprint.
✓ The app was designed and developed by the National Informatics Centre in collaboration with the
Ministry of Electronics and Information Technology.
✓ Key facts about PM Kisan Samman Nidhi (PM-KISAN)
✓ It is a Central Sector scheme with 100% funding from the Government of India.
✓ Objective: The scheme aims to supplement the financial needs of the farmers in procuring various inputs
to ensure proper crop health and appropriate yields commensurate with the anticipated farm income.
✓ Benefits and Eligibility Conditions:
✓ Under the scheme, an income support of 6,000/- per year in three equal instalments will be provided to
all land-holding farmer families.
✓ The definition of family for the scheme is husband, wife and minor children.

Follow us: Official Site, Telegram, Facebook, Instagram, Instamojo 339


Q. How many crore revival package for BSNL has been approved by the government which includes
allocation of 4G/5G spectrum?
A) Rs 89,047 crore B) Rs 93,059 crore
C) Rs 96,430 crore D) Rs 99,500 crore
Answer : A
✓ Union Cabinet chaired by the Prime Minister (PM) Narendra Modi approved the following proposals.
✓ Approved the third revival package for Bharat Sanchar Nigam Ltd (BSNL) with a total outlay of Rs
89,047.82 crore that includes allotment of 4G/5G spectrum through equity infusion.
✓ Revival Package:
✓ The total outlay of the revival package is Rs 89,047 crore.
✓ The package includes allocation of 4G and 5G spectrum for BSNL through equity infusion.
✓ The authorised capital of BSNL will be increased from Rs 1,50,000 crore to Rs 2,10,000 crore.

✓ Revival Package Objectives.


✓ BSNL will emerge as a stable telecom service provider focused on providing connectivity to remote parts
of India.
✓ The spectrum allocation will enable BSNL to provide pan India 4G and 5G services.
✓ BSNL will provide 4G coverage in rural and unserved villages under various connectivity projects.
✓ Fixed Wireless Access (FWA) services for high-speed internet will be offered by BSNL.
✓ BSNL will provide services and spectrum for the Captive Non-Public Network (CNPN).
✓ Previous Revival Package.
✓ The first revival package for BSNL and MTNL was approved in 2019, amounting to Rs 69,000 crore.
✓ The second revival package for BSNL and MTNL was approved in 2022 amounting to Rs 1.64 lakh
crore.
✓ About BSNL
✓ BSNL was incorporated on 15 September 2000.
✓ It is a 100% Government of India owned public sector undertaking.
✓ It is a technology oriented integrated telecom service provider company.
✓ It provides wire line services, 2G, 3G, 4G and GSM mobile services including Value Added Services
(VAS), Internet and Broadband services, Wi-Fi services, Data Center services etc.
✓ Headquarters -New Delhi, India
Q. Which organisation has and the Deendayal Antyodaya Yojana-National Urban Livelihoods Mission
(DAY-NULM) entered into a collaborative partnership?
A) UNDP B) UNEP
C) FAO D) WEF
Answer : A
✓ Recently, the United Nations Development Programme (UNDP) and the Deendayal Antyodaya Yojana-
National Urban Livelihoods Mission (DAY-NULM) entered into a collaborative partnership.
✓ About the UNDP and DAY-NULM Partnership:
✓ Aim: To empower women to make well-informed career choices in the field of entrepreneurship.
✓ It will provide support for women looking to start and expand their enterprises such as the care economy,
digital economy, electric mobility, waste management, food packaging and more.
✓ Focused on fostering entrepreneurship development and accelerating enterprise growth.
✓ Time-Period: It is a three-year project, extendable beyond 2025 which will cover eight cities in the initial
phase.

Follow us: Official Site, Telegram, Facebook, Instagram, Instamojo 340


✓ UNDP will offer national-level capacity-building support to DAY-NULM.
✓ This support will focus on knowledge generation and management, such as compiling compendiums of
best practices related to urban poverty, to enhance the implementation of national-level schemes.
✓ UNDP will also contribute to the initiative by developing community business mentors called Biz-Sakhis
in selected project locations.
✓ These mentors, who possess valuable business knowledge, can support new and existing enterprises, and
serve as a resource for DAY-NULM at a later stage.
✓ Key facts about the DAY-NULM
✓ It is a flagship mission under the Union Ministry of Housing and Urban Affairs.
✓ It has the aim to uplift the urban poor by enhancing sustainable livelihood opportunities through skill
development.
✓ Funding: It will be shared between the Centre and the States in the ratio of 75:25. For North Eastern and
Special Categories – the ratio will be 90:10.

Q. 'I am Adyar, Adyar is me' campaign was launched by which bank?


A) Bank of Baroda B) Canara Bank
C) Federal Bank D) Kotak Mahindra Bank
Answer : C
✓ Federal Bank launches 'I am Adyar, Adyar is me' campaign.
✓ The campaign 'I am Adyar, Adyar is me' was launched by the Federal Bank in Chennai
✓ The Adyar branch of Federal Bank has been converted into a living museum for the campaign.
✓ Vibrant paintings depicting the vibrant spirit of Adyar now adorn the walls of the branch.
✓ The campaign features a special exhibition showcasing 40 compelling stories of the individuals who made
Adyar special.
✓ The campaign aims to capture the essence of Adyar and promote its rich culture and heritage.
✓ It fosters a sense of pride and belonging among local residents.
✓ The 'I am Adyar, Adyar is me' campaign is an innovative initiative by Federal Bank to honour and
recognize the Adyar community.
✓ FEDERAL BANK
✓ Founded : 1931
✓ Headquarters : Aluva, Kerala
✓ MD & CEO: Shyam Srinivasan
✓ Tagline: Your Perfect Banking Partner

Q. The National Time Release Study (TRS) Report 2023, is released by which organization?
A) CBDT B) CBIC
C) BIS D) FSSAI
Answer : B
✓ Recently, the chairman of the Central Board of Indirect Taxes and Customs (CBIC) along with other
Members of the Board released the National Time Release Study (NTRS) 2023 report.
✓ About National Time Release Study (NTRS) 2023 report:
✓ It is a performance measurement tool.
✓ It aims to present a quantitative measure of the cargo release time at the Customs station.
✓ It also measures the domestic clearance in case of imports and the arrival of the cargo at the Customs
station to the eventual departure of the carrier in case of exports.
✓ The study included seaports, air cargo complexes (ACCs), inland container depot (ICDs) and integrated
check posts (ICPs) which handles the maximum per cent of bills of entry in the country.
✓ Highlights of the report

Follow us: Official Site, Telegram, Facebook, Instagram, Instamojo 341


✓ It reaffirms the ‘Path to promptness’e. comprising advance filing of import documents enabling pre-
arrival processing, risk-based facilitation of cargo and benefits of trusted client programme.
✓ Cargoes wherein all the three features under the Path to Promptness are combined, achieve the National
Trade Facilitation Action Plan (NTFAP) release time target across all port categories.
✓ It has placed a much greater focus on the measurement of export release time
Q. To realise Prime Minister Narendra Modi’s vision of ‘Sahakar se Samridhi, four important initiatives
have been taken to strengthen how many Urban Co-operative Banks (UCBs) in the country?
A) 1,514 B) 1,245
C) 1,976 D) 1,678
Answer : A
✓ To realise Prime Minister Narendra Modi’s vision of ‘Sahakar se Samridhi, four important initiatives
have been taken to strengthen 1,514 Urban Co-operative Banks (UCBs) in the country.
✓ Four initiative

1. In order to expand their business, Urban Cooperative Banks (UCBs) can now open new branches.
UCBs can now open new branches up to 10% (maximum 5 branches) of the number of branches in the
previous financial year without prior approval of RBI in their approved area of operation.
In order to avail this facility, UCBs have to get the policy approved by their board and comply with the
Financially Sound and Well Managed (FSWM) Norms.
2. UCBs can also do One Time Settlement at par with Commercial Banks
RBI has notified a framework governing this aspect for all regulated entities including Urban Co-
operative Banks.
Now co-operative banks through board-approved policies may provide a process for technical write-off
as well as settlement with borrowers.
This has brought cooperative banks at par with other commercial banks now.
3. Revised timelines for PSL targets given to UCBs
The Reserve Bank of India has decided to extend the timeline for UCBs to achieve Priority Sector
Lending (PSL) targets by two years i.e. up to March 31,2026
4. Designating a Nodal Officer in RBI
In order to meet the long pending demand of the cooperative sector for closer coordination and focused
interaction, RBI has recently notified a nodal officer as well.
✓ What are Urban Cooperative banks (UCB)?
✓ It refers to primary cooperative banks located in urban and semi-urban areas.
✓ These are registered as cooperative societies under the provisions of, either the State Cooperative
Societies Act of the State concerned under the provisions of the or the Multi State Cooperative Societies
Act, 2002.
✓ It is regulated by the Registrar of Cooperatives and also by the RBI.
✓ The Reserve Bank regulates and supervises the banking functions of UCBs under the provisions of
Banking Regulation Act, 1949
Q. What is the name of tariff levied on imported goods to offset subsidies given to producers of the goods in
exporting country?
A) Basic Customs Duty B) Special Additional Duty
C) Anti-dumping Duty D) Countervailing Duty
Answer : D

Follow us: Official Site, Telegram, Facebook, Instagram, Instamojo 342


✓ The finance ministry is considering a proposal to impose a countervailing duty on steel imports from
China.
✓ About Countervailing duty (CVD):
✓ It is a specific form of duty that the government imposes to protect domestic producers by countering the
negative impact of import subsidies.
✓ CVD is thus an import tax by the importing country on imported products.
✓ The World Trade Organization (WTO) permits the imposition of CVD by its member countries.
✓ Why is CVD imposed?
✓ Foreign governments sometimes provide subsidies to their producers to make their products cheaper and
boost their demand in other countries.
✓ To avoid flooding the market in the importing country with these goods, the government of the importing
country imposes CVD, charging a specific amount on the import of such goods.
✓ The duty nullifies and eliminates the price advantage enjoyed by an imported product.
✓ The duty raises the price of the imported product, bringing it closer to its true market price
Q. What amount has been approved by Asian Development Bank (ADB) to develop horticulture in
Himachal Pradesh?
A) $110 B) $130
C) $150 D) $200
Answer : B
✓ ADB gives 130 million dollars to increase horticulture in Himachal Pradesh.
✓ The Government of India signed a $130 million loan agreement with the Asian Development Bank
(ADB) to enhance the income of Himachal Pradesh farmers.
✓ This $130 million loan will be used to increase agricultural productivity, improve irrigation facilities and
promote horticulture agribusinesses in Himachal Pradesh.
✓ The implementation of this project will help at least 15,000 agricultural families in 7 districts of the state
– Bilaspur, Hamirpur, Kangra, Mandi, Sirmaur, Solan and Una – to increase their income and combat
the effects of climate change.
Q. As per an EXIM bank report, India has become a major exporter of Defence products of which region?
A) North America B) Asia
C) Antarctica D) Africa
Answer : D
✓ MD of EXIM Bank of India recently said that India's future Lines of Credit (LoCs) to Africa could focus
on defence exports to meet the requirements of the continent.
✓ According to a report by the India Exim Bank, India has recently become a major exporter of defence
products and will be able to meet Africa's needs in maritime, aerospace, and defence sectors. According
to the report, Mauritius, Mozambique, and Seychelles together accounted for 6.6% of India's arms
exports to Africa between 2017-2021. India was the 23rd largest exporter of defence goods from 2017-
2021.
✓ About Export-Import Bank of India (EXIM Bank):
✓ It is the premier export finance institution of the country.
✓ It was established by the Government of India, under the Export-Import Bank of India Act, 1981
✓ EXIM Bank wholly owned by the Government of India.
✓ EXIM Bank provides financial assistance to exporters and importers.
✓ What is a Line of Credit (LoC)?
✓ It is a preset borrowing limit that can be tapped into at any time.

Follow us: Official Site, Telegram, Facebook, Instagram, Instamojo 343


✓ All LOCs consist of a set amount of money that can be borrowedas needed, paid back, and borrowed
again.
✓ The borrower can take money out as needed until the limit is reached.
✓ As money is repaid, it can be borrowed again in the case of an open line of credit.

Q. What is the rank of Reliance Industries Limited in Forbes Global 2000 Top Companies list in 2023?
A) 16th B) 28th
C) 45th D) 53rd
Answer : C
✓ Reliance climbs eight spots to 45th rank on Forbes' Global 2000 list.
✓ Billionaire Mukesh Ambani’s Reliance Industries Ltd climbed eight spots to the 45th rank, the highest
for an Indian company on Forbes’ latest Global 2000 list of public companies worldwide.
✓ This achievement marks the highest position for an Indian company on the list.
✓ The Global 2000 ranks public companies worldwide based on sales, profits, assets, and market value.
✓ JPMorgan tops the list
✓ JPMorgan, America's largest bank with $3.7 trillion in assets, has secured the top spot on the Global 2000
list.
✓ This marks its first time at the top since 2011.
✓ The bank's strong performance during the regional banking crisis, coupled with increased deposits and
the opportunistic acquisition of the failed First Republic Bank, contributed to its ranking.
✓ Warren Buffett's Berkshire Hathaway, which topped the list in the previous year, fell to the 338th position
in the latest ranking.
✓ This decline is attributed to unrealized losses in its investment portfolio.
✓ Saudi Arabia's oil giant, Aramco, holds the second position on the list.
✓ Indian firms on the list
✓ Reliance Industries leads as the highest-ranked Indian company at the 45th spot.
✓ Other notable Indian firms include State Bank of India at 77th, HDFC Bank at 128th, ICICI Bank at
163rd, and Tata Consultancy Services (TCS) at 387th.
✓ A total of 55 Indian companies made it to the Global 2000 list, including Oil and Natural Gas
Corporation (ONGC), HDFC, Life Insurance Corporation (LIC), and Tata Steel.
✓ Gautam Adani's conglomerate firms
✓ Three companies from Gautam Adani's conglomerate are included in the list. Adani Enterprises secured
the 1062nd position, Adani Power is at 1488th, and Adani Ports & Special Economic Zones is at 1598th
Q. NTPC Secures Which Rank in Forbes 'The Global 2000'- List for 2023.
A) 389 B) 411
C) 433 D) 456
Answer : C
✓ NTPC climbs up 52 positions to 433rd rank in Forbes’ “The Global 2000” List.
✓ NTPC Limited, India's largest power generation company, has made significant progress in Forbes' "The
Global 2000" List for 2023, climbing 52 positions to reach the 433rd rank.
✓ This remarkable advancement showcases NTPC's growing influence and presence in the global market.
✓ It is a testament to the company's consistent expansion, strong financial performance, and unwavering
commitment to excellence.
✓ "The Global 2000" List compiled by Forbes acknowledges the world's largest companies based on four
key metrics: sales, profits, assets, and market value.
✓ NTPC's significant climb in the rankings from the 485th position in 2022 to the 433rd position in 2023
underscores its exceptional performance across these metrics.

Follow us: Official Site, Telegram, Facebook, Instagram, Instamojo 344


Q. VAIBHAV fellowships programme is implemented by which Department of Government of India?
A) Department of Science and Technology
B) Department for Promotion of Industry and Internal Trade
C) Department of Agricultural Research and Education
D) Department of Administrative Reforms and Public Grievances
Answer : A
✓ The Government of India has launched a new fellowship programme to connect the Indian Science,
Technology, Engineering, Mathematics and Medicine (STEMM)diaspora with Indian academic and
R&D institutions.
✓ About VAIBHAV fellowships Programme:
✓ The fellowship would be awarded to outstanding scientists/technologists of Indian origin
(NRI/OCI/PIO) who are engaged in research activities in their respective countries.
✓ The VAIBHAV Fellow would identify an Indian Institution for collaboration and may spend up to two
months in a year for a maximum of 3 years.
✓ Funding: The government offers the researchers an amount of INR 4,00,000 per month for the entire
period.
✓ Aim: Improving the research ecosystem of India’s Higher Educational Institutions by facilitating
academic and research collaborations between Indian Institutions and the best institutions in the world.
✓ This is done through the mobility of faculty/researchers from overseas institutions to India.
✓ Eligibility: Researchers from institutions featuring in the top 500 QS World University Rankings will be
eligible for the fellowship.
✓ The applicant must have obtained Ph.D./M.D/M.S degree from a recognized University.
✓ The 75 selected fellows would be invited to work in 18 identified knowledge verticals including quantum
technology, health, pharma, electronics, agriculture, energy, computer sciences, and material sciences
amongst others.
✓ It is implemented by the Department of Science and Technology (DST), Ministry of Science and
Technology.
Q. Who was given the 'Governor of the Year' award by Central Banking in London?
A) C. Rangarajan B) Urjit Ravindra Patel
C) Shaktikanta Das D) Bimal Jalan
Answer : C
✓ Reserve Bank of India Governor Shaktikanta Das was honored with the 'Governor of the Year' award by
Central Banking in London.
✓ Das is the second RBI governor to have received this award.
✓ In the year 2015, the then RBI Governor Raghuram Rajan received this award.
✓ The award is presented by Central Banking, a leading organisation that covers matters related to central
banks and financial regulators globally.
✓ The Central Banking Awards 2023 announced the winners at the end of March, the National Bank of
Ukraine received the title of Central Bank of the Year.
✓ Shaktikanta Das, representing the Reserve Bank of India, was honoured with the Governor of the Year
award at the event.
✓ Das is a retired Indian Administrative Service (IAS) officer of the Tamil Nadu cadre, who is currently
serving as the 25th governor of the RBI.
✓ With the second term of three years, Das will head the RBI till December, 2024.

Follow us: Official Site, Telegram, Facebook, Instagram, Instamojo 345


Q. Sale of which of the following is done through Open Market Sale Scheme (OMSS)?
A) Maize and Wheat B) Soybean and Cotton
C) Wheat and Rice D) Rice and Barley
Answer : C
✓ The Central Government recently discontinued the sale of rice and wheat from the central pool under
the Open Market Sale Scheme (OMSS) to State governments.
✓ About Open Market Sale Scheme (OMSS):
✓ Under OMSS, the Food Corporation of India (FCI) sells surplus stocks of wheat and rice at pre-
determined prices through e-auction in the open market.
✓ Objective: To enhance the supply of food grains, especially wheat, during the lean season and thereby
moderate the open market prices, specially in the deficit regions.
✓ The FCI conducts a weekly auction to conduct this scheme in the open market using the platform of
commodity exchange NCDEX(National Commodity and Derivatives Exchange Limited).
✓ The State Governments/ Union Territory Administrations are also allowed to participate in the e-
auction, if they require wheat and rice outside the Targeted Public Distribution Scheme (TPDS) and
Other Welfare Schemes (OWS).
✓ The reserve price is fixed by the government. In the tenders floated by the FCI, the bidders cannot quote
less than the reserve price.
✓ The present form of OMSS comprises 3 schemes as under:
✓ Sale of wheat to bulk consumers/private traders through e-auction.
✓ Sale of wheat to bulk consumers/private traders through e-auction by dedicated movement.
✓ Sale of Raw Rice Grade ‘A’ to bulk consumers/private traders through e-auction.
Q. The World Bank (WB) launched its first dedicated road safety project in South Asia, for which country?
A) Nepal B) Bhutan
C) Thailand D) Bangladesh
Answer : D
✓ World Bank launches its first road safety project in South Asia.
✓ World Bank (WB) launched its first dedicated road safety project in South Asia with a USD 358 million
financing agreement signed in Dhaka with the government of Bangladesh.
✓ The project seeks to improve road safety and reduce fatalities and injuries from road crashes in selected
cities, high-risk highways, and district roads.
✓ Implementation on National Highways.
✓ The project will focus on two national highways in Bangladesh: Gazipur-Elenga (N4) and Natore-
Nawabganj (N6).
✓ These highways will serve as the primary sites for implementing various road safety measures.
✓ The aim is to achieve a substantial reduction of over 30 percent in road traffic deaths on these two
highways.
Q. Which entity has published a circular that establishes a new type of intermediaries called execution-only
platforms (EOPs)?
A) RBI B) SEBI
C) SIDBI D) NPCI
Answer: B
✓ Recently, the Securities and Exchange Board of India (SEBI) issued a circular, creating a new category
of intermediaries called execution-only platforms (EOPs).

Follow us: Official Site, Telegram, Facebook, Instagram, Instamojo 346


✓ About Execution only platform (EOP)
✓ It is a digital or online platform which facilitates transactions such as subscription, redemption and switch
transactions in direct plans of schemes of mutual funds.
✓ Till now, there was no specific framework available for technology/digital platforms (including platforms
provided by Investment Advisers/Stock Brokers to non-clients) to provide execution-only services in
direct plans of mutual fund schemes.
✓ As per the new SEBI’s new guidelines, no entity would be allowed to operate as an EOP without
obtaining registration from SEBI or the Association of Mutual Funds in India (AMFI).
✓ Categories of EOP: The capital markets regulator has divided EOPs into two categories.
✓ Category 1 EOPs: These would need to be registered with AMFI, the mutual fund industry body.
✓ Under this category, the EOPs would act as an agent of asset management companies (AMCs) and
integrate their systems with AMCs and/or Registrar and Transfer Agents (RTAs) authorized by AMCs
to facilitate transactions in mutual funds.
✓ Category 2 EOPs: These would need to be registered as a stock broker with SEBI and can operate as an
agent of investors and operate only through the platforms provided by the stock exchanges.

Q. Which Country SBER Bank has Launched Indian Rupee Accounts for Individuals?
A) China B) Japan
C) Germany D) Russia
Answer : D
✓ Russia’s dominant lender, SBER Bank has launched Indian rupee accounts for individuals, expanding
the range of foreign currencies available as Moscow strives to reduce dependency on the U.S. dollar and
euro.
✓ As Sberbank has highlighted, the bank supports the trend of de-dollarisation and constantly increases the
number of foreign currencies available to clients.
✓ Sberbank has more than 100 million retail clients and already offers deposits in China’s yuan and UAE
dirhams.
Q. Which department has notified ‘India Infrastructure Project Development Fund Scheme (IIPDF
Scheme)’?
A) Department of Economic Affairs B) NITI Aayog
C) FSSAI D) FICCI
Answer : A
✓ Recently, to promote the Digital India initiative of the Government of India, the Infrastructure Finance
Secretariat has launched the IIPDF Portal for submitting applications for consideration under IIPDF
Scheme.
✓ About IIPDF Scheme:
✓ It is a Central Sector Scheme which will aid the development of quality PPP projects by providing
necessary funding support to the project sponsoring authorities, both in the Central and State
Governments.
✓ Funding:
✓ The corpus of the IIPDF shall comprise of initial budgetary outlay of Rs. 100 Crore by the Ministry of
Finance.
✓ Funding under IIPDF Scheme is in addition to the already operational Scheme for Financial Support to
PPPs in Infrastructure (VGF Scheme).
✓ The Department of Economic Affairs (DEA), Ministry of Finance has notified the ‘Scheme for Financial
Support for Project Development Expenses of PPP Projects’ – India Infrastructure Project Development
Fund Scheme (IIPDF Scheme).

Follow us: Official Site, Telegram, Facebook, Instagram, Instamojo 347


Q. Which has launched mission EVOLVE (Electric Vehicle Operations and Lending for Vibrant Ecosystem)
in association with NITI Aayog to finance MSMEs in the EV space?
A) SEBI B) RBI
C) SIDBI D) NABARD
Answer : C
✓ SIDBI launches EVOLVE mission with NITI Aayog.
✓ Small Industries Development Bank of India (SIDBI) launched the Electric Vehicle Operations and
Lending for Vibrant Ecosystem (EVOLVE) mission.
✓ The mission has been launched in collaboration with NITI Aayog, World Bank, Korean-World Bank
and Korean Economic Development Cooperation Fund (EDCF).
✓ Its main objective is to provide financial assistance to micro, small and medium-sized enterprises
(MSMEs) in the electric vehicle sector.
✓ The collaboration between SIDBI, NITI Aayog, World Bank, Korea-World Bank and Korean Economic
Development Cooperation Fund (EDCF) highlights the importance of public-private partnership.
✓ The EVOLVE mission contributes to the promotion of sustainable transportation solutions in India.
✓ By supporting MSMEs in the electric vehicle industry, the initiative aims to foster a vibrant ecosystem
for electric vehicles.

Q. Who are the beneficiaries of ‘Kanya Shiksha Pravesh Utsav’ campaign?


A) Adolescent Girls B) Nursing
C) Anganwadi D) Widow
Answer : A
✓ Recently, officials from the Union Ministry of Women and Child Development (WCD) have brought
back 1 lakh girls into the mainstream education system under the `Kanya Shiksha Pravesh Utsav’’.
✓ About Kanya Shiksha Pravesh Utsav:
✓ It was launched by the Ministry of Women and Child Development (MoWCD), in partnership with the
Ministry of Education and UNICEF
✓ Objective: Enhancing enrolment and retention of girls between 11-14 years of age in school.
✓ The initiative intends to build on the existing schemes and programmes like Schemes for Adolescent Girls
(SAG), BetiBachaoBetiPadhao (BBBP) and National Education Policy (NEP) to work on a
comprehensive system for out-of-school girls.
✓ Now Ministry of Women and Child Development has decided to club together three programmes under
a new scheme — Anganwadi services, poshan Abhiyan and scheme for adolescent girls — since all these
had the same nutritional targets.”
✓ In the new scheme, a new category is created in which adolescent girls between 14 and 18 years are
covered

Q. According to IEA, the growth in demand for oil in Which Country will overtake that in China by 2027?
A) Russia B) USA
C) India D) Canada
Answer : C
✓ The growth in demand for oil in India will overtake that in China by 2027, According to report by the
International Energy Agency (IEA).
✓ 'Oil 2023', the multilateral agency’s outlook for the next 5 years, said China’s demand will fall consistently
from 2024 after a massive rise this year (2023).
✓ The United States, China, and India are the world’s three largest oil consumers in that order.

Follow us: Official Site, Telegram, Facebook, Instagram, Instamojo 348


✓ IEA raised its forecast for world oil growth demand to 2.4 million barrels per day in 2023, after the
Chinese economy rebounded towards growth faster than expected. China will in 2023 account for nearly
60% of global growth in oil demand
✓ India consumed 222.30 million tonnes of petroleum products in 2022-23: 10.2% more year-on-year.
✓ Global crude output reached an estimated 82.3 million barrels per day till April 2023 due to record
production runs in Asia
Q. With reference to the Pradhan Mantri Matru Vandana Yojana’, consider the following statements:
1. It is a Centrally Sponsored Direct Benefit Transfer (DBT) scheme.
2. Under this scheme a maternity benefit of Rs.50000 is given to a woman.
Which of the statements given above is/are correct?
A) 1 Only B) 2 Only
C) Both 1 and 2 D) Neither 1 nor 2
Answer : A
✓ Recently, The Prime Minister of India has lauded the new initiative of celebrating ‘Pradhan Mantri Matru
Vandana Yojana’ as ‘God Bharai' ceremony in Dausa, Rajasthan.
✓ About PM Matru Vandana Yojana:
✓ It is a Centrally Sponsored Direct Benefit Transfer (DBT) scheme launched in 2017.
✓ Objectives
✓ Providing partial compensation for the wage loss in terms of cash incentive, so that the woman can take
adequate rest before and after delivery of the first child
✓ To improve health-seeking behaviour amongst Pregnant Women & Lactating Mothers (PW&LM).
✓ The scheme is to provide maternity benefits to women belonging to socially and economically
disadvantaged sections of society.
✓ The maternity benefit is to be provided to a woman for the first two living children provided the second
child is a girl
✓ Under this scheme monetary benefit of Rs. 5,000 is given to the expecting mothers from the time of the
initial stage of pregnancy to till the time the child is born.
✓ Additionally, Rs. 1,000 is provided under the Janani Suraksha Yojana post-institutional delivery.
✓ The Rs. 5,000 is provided in three installments.
✓ The first installment of Rs. 1,000 is paid at the time of registration of the pregnancy.
✓ The second installment of Rs. 2,000 is paid at the time of completing 6 months of pregnancy and receiving
at least one antenatal checkup.
✓ The third installment of Rs. 2,000 is received after the birth and registration of birth of such child and
after such child receives the first cycle of immunization for BCG, OPV, DPT and Hepatitis-B.
✓ Cases of miscarriage/stillbirths are to be treated as fresh cases for providing maternity benefits under the
scheme.
Q. How many crores dividend check has State Bank of India (SBI) given to FM Nirmala Sitharaman for the
financial year 2022-23?
A) Rs 4,500 Crores B) Rs 4,860 Crores
C) Rs 5,290 Crores D) Rs 5,740 Crores
Answer : D
✓ The country's largest public sector bank, State Bank of India (SBI), has given the government a cheque
of Rs 5,740 crore as its dividend income for the financial year 2022-23.

Follow us: Official Site, Telegram, Facebook, Instagram, Instamojo 349


✓ This is the highest dividend the State Bank of India has given in any financial year. Dinesh Kumar Khara,
chairman of the country's largest bank, handed over the dividend cheque to Finance Minister Nirmala
Sitharaman.
✓ According to the data available on the BSE website, the State Bank of India has paid a dividend of Rs
23.1130 per share to shareholders for the financial year 2022-23. The record date for this was 31 May
2023
Q. The Bank formed by BRICS countries is known as ___.
A) Inter-American Development Bank (IDB)
B) European Investment Bank (EIB)
C) Asian Development Bank (ADB)
D) New Development Bank (NDB)
Answer : D
✓ Honduras President recently requested the country’s admission to the BRICS-led New Development
Bank (NDB).
✓ About New Development Bank (NDB):
✓ NDB, formerly referred to as the BRICS Development Bank, is a multilateral development bank
established by the BRICS countries (Brazil, Russia, India, China and South Africa).
✓ Objective: Financing infrastructure and sustainable development projects in BRICS and other emerging
economies and developing countries.
✓ The idea of setting up NDB was first conceived in 2012 during BRICS Summit in New Delhi, India. The
Bank formally came into existence as a legal entity in 2015.
✓ Headquarters: Shanghai, China.
✓ The first regional office of the NDB was setup in Johannesburg, South Africa. The second regional office
was established in 2019 in São Paulo, Brazil, followed by Moscow, Russia.
✓ Capital: The Bank has an initial authorized capital of 100 billion dollars and an initial subscribed capital
of 50 billion dollars.
✓ Membership in NDB is open to any member of the United Nations.
✓ Governance Structure
✓ The Bank is governed by a Board of Governors made up of the finance ministers of the five BRICS
countries, and a Board of Directors.
✓ Voting power within the Board is based on each country’s shares in the bank.
✓ While new members can join the NDB, the five BRICS countries will retain a minimum of 55% of total
shares.
✓ The NDB’s management includes a presidency which rotates among BRICS members, and four vice
presidents who are selected from the remaining BRICS countries
Q. Which airline has placed a order with European manufacturer Airbus for 500 A320 Family aircraft?
A) SpiceJet B) Go First
C) Air India D) IndiGo
Answer : D
✓ IndiGo, India’s largest airline by market share, has placed a record order with European manufacturer
Airbus for 500 A320 Family aircraft.
✓ It is the biggest single purchase agreement in the history of commercial aviation.
✓ This IndiGo order book comprises a mix of A320NEO, A321NEO and A321XLR aircraft. The airline
will take the delivery of these aircraft between 2030 and 2035.

Follow us: Official Site, Telegram, Facebook, Instagram, Instamojo 350


✓ The agreement takes the total number of Airbus aircraft on order by IndiGo to 1,330, making it the
world’s biggest A320 Family customer.
✓ Earlier this year, Tata-owned Air India placed orders for 470 aircraft from Airbus and US manufacturer
Boeing.

Q. The aim of the 'SANKALP' Programme of Union government is ___.


A) eliminating the tuberculosis by 2025.
B) reducing the poverty level by 50% from the existing poverty level.
C) providing market-relevant training to 3.5 crore youth across the country.
D) providing affordable house to rural BPL failies.
Answer : C
✓ Recently, the Ministry of Skill Development & Entrepreneurship (MSDE) certifies 98 trainers trained
under the SANKALP programme.
✓ providing market-relevant training to 3.5 crore youth across the country.
✓ About SANKALP Programme:
✓ The Skills Acquisition and Knowledge Awareness for Livelihood Promotion (SANKALP) programme
was launched in 2018.
✓ Nodal Ministry: Ministry of Skill Development and Entrepreneurship (MSDE)
✓ It is a World Bank loan-assisted project and is aligned with the overall objectives of the National Skill
Development Mission (NSDM).
✓ The project focuses on transforming the overall skilling ecosystem of India, covering both central and
state-level agencies for improved outcomes.
✓ It also encourages innovative best practices at the local level, resulting in enhanced access, quality and
capacity in the skilling ecosystem and improved access to and completion of skills training for female
trainees and other disadvantaged groups.
✓ Strategy: Under SANKALP four key result areas have been identified viz –
✓ Institutional Strengthening (at National, State & District level)
✓ Quality Assurance Quality Assurance of skill development programs;
✓ Inclusion of marginalised population in skill development; and
✓ Expanding Skills through Public-Private Partnerships (PPPs)
Q. According to Hurun India, which is the most valuable listed private company in India?
A) HDFC Bank B) Infosys
C) Reliance Industries D) ITC
Answer : C
✓ Hurun India has recently released the list of India's listed most valuable private companies, in this list
Mukesh Ambani-led Reliance Industries is at the top position.
✓ According to Hurun India, the total market capitalization of Reliance Industries is ₹16,37,327 crore.
✓ Tata Consultancy Services (TCS) of Tata Group comes second in this list with a total market
capitalization of Rs 11.8 lakh crore.
✓ In this list, TCS is second and HDFC Bank is in third place with Rs 9.4 lakh crore.
✓ Reliance Industries Limited (RIL)
✓ Founded : 8 May 1973
✓ Founder : Dhirubhai Ambani
✓ Headquarters : Mumbai, Maharashtra,
✓ CMD : Mukesh Ambani
✓ Owner : Mukesh Ambani (50.49%)

Follow us: Official Site, Telegram, Facebook, Instagram, Instamojo 351


✓ Director : Nita Dalal Ambani
✓ Subsidiaries
✓ Jio Platforms (67.03%)
✓ Jio Payments Bank (70%).
✓ At Rs 19 lakh crore or $250 billion, Indian billionaire Mukesh Ambani-promoted Reliance Industries's
market capitalisation.
Q. Which entity has introduced a circular allowing wilful defaulters and companies involved in fraud to opt
for compromise settlements or technical write-offs?
A) RBI B) SEBI
C) IRDAI D) NPCI
Answer : A
✓ As per a recent circular by the Reserve Bank of India (RBI), Wilful defaulters and companies involved in
fraud can go for a compromise settlement or technical write-offs by banks and finance companies.
✓ About Wilful Defaulter:
✓ They are considered as entities that do not pay back money despite the ability to do so.
✓ The concept of ‘Wilful Defaulter’ was introduced when the RBI, as per its power under Sections 21 and
35A of the Banking Regulation Act, 1949, issued the Master Circular, which defined Wilful Defaulter
and detailed the measures to be adopted by the Banks and Financial Institutions (FIs) to adjudge the
instances of default, i.e., whether the same is a wilful default or not.
✓ According to the RBI, a wilful default is deemed to have occurred in any of the following four
circumstances:
✓ When there is a default in repayment obligations by the unit (company/individual) to the lender, even
when it has the capacity to honour the said obligations, there is a deliberate intention of not repaying the
loan.
✓ The funds are not utilised for the specific purpose for which finance was availed but have been diverted
for other purposes.
✓ When the funds have been siphoned off and not been utilised for the purpose for which it was availed.
Further, no assets are available which justify the usage of funds.
✓ When the asset bought by the lenders’ funds have been sold off without the knowledge of the bank/lender.
✓ At present, there is no specific law for legal action against Wilful defaulters. Reserve Bank of India has
framed rules defining wilful default, the process to be followed by banks for declaring borrower as “wilful
defaulter”. Banks initiate action against such accounts under laws like SARFAESI Act, Companies Act,
2013, Fugitive Economic Offenders Act etc.
Q. Who has recently been appointed as the Deputy Governor of RBI?
A) Venugopal Iyer B) Swaminathan Janakiraman
C) Urjit Patel D) Raghuram Rajan
Answer : B
✓ Swaminathan Janakiraman has been appointed as the new Deputy Governor of the Reserve Bank of
India (RBI) by the Government of India.
✓ Swaminathan Janakiraman will replace Mahesh Kumar Jain, whose term ends on June 22.
✓ Janakiraman is currently the Managing Director in State Bank of India (SBI).
✓ He has served as a nominee director of SBI on the boards of Yes Bank, Jio Payments Bank and NPCI.
✓ He has also representedSBI as a director in Bank of Bhutan, a joint venture with SBI.
✓ RBI Deputy Governor (4)
1. Tavarna Rabi Sankar
2. Michael D Patra

Follow us: Official Site, Telegram, Facebook, Instagram, Instamojo 352


3. M Rajeshwar Rao
4. Swaminathan Janakiraman.
Q. Insolvency regulator, IBBI, proposes to stipulate mandatory audit of Insolvency Resolution Process Costs
(IRPC) in resolution cases where the assets of the corporate debtor (CD) is in excess of_.
A) 125 crore B) 100 crore
C) 150 crore D) 200 crore
Answer : B
✓ Insolvency regulator, IBBI, proposes to stipulate mandatory audit of Insolvency Resolution Process Costs
(IRPC) in resolution cases where the assets of the corporate debtor (CD) is in excess of ₹ 100 crore.
✓ About Insolvency and Bankruptcy Board of India (IBBI):
✓ It was established on 1st October 2016 under the Insolvency and Bankruptcy Code (IBC), 2016.
✓ It is responsible for the implementation of the IBC. The IBC amends and consolidates the laws relating
to insolvency resolution of individuals, partnership firms and corporate persons in a time-bound manner.
Q. Which Indian company is included in the list of 100 most influential companies released by 'Time'
magazine?
A) Tata Group B) Reliance Industries
C) Bharti Airtel D) NPCI
Answer : D
✓ 'Time' magazine has recently released its annual list of the world's 100 most influential companies, which
includes top companies like OpenAI, SpaceX, Chess.com and Google DeepMind.
✓ India's National Payments Corporation of India (NPCI) along with e-commerce platform Meesho has
also been given a place in this list.
✓ More than 12% of the companies in this list are associated with the AI industry.
✓ This mainly includes companies like OpenAI, Nvidia, Google DeepMind, Hugging Face, Metaphysics.
Q. Which of the following is a capacity-building scheme that was launched in 2020 for civil servants aimed
at upgrading the post-recruitment training machanism of officers and employees at all levels?
A) Mission Rakshak B) Mission Sahayog
C) Mission Karmayogi D) Mission Swabhiman
Answer : C
✓ Prime Minister will inaugurate the first National Training Conclave at the International Exhibition and
Convention Centre Pragati Maidan, New Delhi.
✓ The conclave is part of the National Programme for Civil Services Capacity Building (NPCSCB) -
‘Mission Karmayogi’.
✓ Objective: To foster collaboration among civil services training institutes and strengthen the training
infrastructure for civil servants across the country.
✓ The Conclave is being hosted by the Capacity Building Commission.
✓ More than one thousand 500 representatives from various training institutes, including Central Training
Institutes, State Administrative Training Institutes, Regional and Zonal Training Institutes, and Research
institutes, will participate in the conclave.
✓ Civil Servants from the Central government departments, State governments, and local governments, as
well as experts from the private sector, will take part in the deliberations.
✓ The Conclave will have eight-panel discussions, each focusing on key concerns related to Civil services
training institutes such as faculty development, training impact assessment, and content digitisation.

Follow us: Official Site, Telegram, Facebook, Instagram, Instamojo 353


✓ What is Mission Karmayogi?
✓ Mission Karmyogi, or National Programme for Civil Services Capacity Building (NPCSCB), aims to
prepare Civil Servants for the future by making them more creative, constructive & innovative through
transparency and technology.
✓ This unique programme will help to lay the foundation for civil servants in the country.
✓ There will be more focus on 'on-site learning' in complementing "off-site learning".
Q. Asian Development Bank (ADB) has approved a $295 million loan to improve transport connectivity in
which state?
A) Bihar B) Jharkhand
C) Rajasthan D) Chhattisgarh
Answer : A
✓ ADB approves $295mn loan to improve Transport Connectivity in Bihar.
✓ The Asian Development Bank (ADB) approved a $295 million loan to improve transport connectivity in
Bihar.
✓ The loan amount will be utilized towards widening and upgradation of around 265 kilometers of state
highways, improving transport connectivity and safety in Bihar.
✓ The Central Government is contributing $156.6 million for the same.
✓ Since 2008, ADB has provided five loans totaling $1.63 billion to Bihar, which have upgraded about
1,696 kilometers of state highways and constructed a new bridge over the Ganga River.
Q. The Union Budget 2023-24 has allocated Rs. 2,516 crore for the computerization of which unit, over the
next 5 years?
A) State Cooperative Banks B) District central cooperative banks
C) Primary Agricultural Credit Societies D) Multi-state Cooperative Banks
Answer : C
✓ The Cooperation Ministry recently said that the Primary Agricultural Credit Societies (PACS) can also
be employed as drone entrepreneurs for spraying fertilisers and pesticides.
✓ The Cabinet Committee on Economic Affairs chaired by Prime Minister Narendra Modi approved
computerisation of Primary Agricultural Credit Societies (PACS) with a budgetary outlay of ₹2,516
crore.
✓ About Primary Agricultural Credit Societies (PACS):
✓ These are village-level cooperative credit societiesthat serve as the last link in a three-tier cooperative
credit structure.
✓ These are headed by the State Cooperative Banks (SCB) at the state level.
✓ Credit from the SCBs is transferred to the district central cooperative banks, or DCCBs, that operate at
the district level.
✓ These DCCBs work with PACS, which deals directly with farmers.
✓ Individual farmers are members of the PACS, and office-bearers are elected from within them. A village
can have multiple PACS.
✓ The main function of the PACS is to provide short and medium-term purpose loans to its members.
Q. The Union Budget 2023-24 has allocated Rs. 2,516 crore for the computerization of which unit, over the
next 5 years?
A) State Cooperative Banks B) District central cooperative banks
C) Primary Agricultural Credit Societies D) Multi-state Cooperative Banks
Answer : C

Follow us: Official Site, Telegram, Facebook, Instagram, Instamojo 354


✓ The Cooperation Ministry recently said that the Primary Agricultural Credit Societies (PACS) can also
be employed as drone entrepreneurs for spraying fertilisers and pesticides.
✓ The Cabinet Committee on Economic Affairs chaired by Prime Minister Narendra Modi approved
computerisation of Primary Agricultural Credit Societies (PACS) with a budgetary outlay of ₹2,516
crore.
✓ About Primary Agricultural Credit Societies (PACS):
✓ These are village-level cooperative credit societiesthat serve as the last link in a three-tier cooperative
credit structure.
✓ These are headed by the State Cooperative Banks (SCB) at the state level.
✓ Credit from the SCBs is transferred to the district central cooperative banks, or DCCBs, that operate at
the district level.
✓ These DCCBs work with PACS, which deals directly with farmers.
✓ Individual farmers are members of the PACS, and office-bearers are elected from within them. A village
can have multiple PACS.
✓ The main function of the PACS is to provide short and medium-term purpose loans to its members.
Q. Which IT Firm has bagged a $454-million deal for the digital transformation of Denmark's Danske Bank?
A) TATA B) Wipro
C) Infosys D) Cognizant
Answer : C
✓ IT major Infosys has bagged a digital transformation deal from Denmark-based Danske Bank estimated
to be around USD 454 million for five years.
✓ As part of the strategic collaboration with Danske Bank, Infosys will acquire Danske Bank's IT centre in
India, which employs 1400 digitally skilled professionals.

Q. FLDG arrangement, which was seen in the news, is related to which field?
A) Education B) Finance
C) Sports D) Agriculture
Answer : B
✓ The Reserve Bank of India (RBI) has recently granted its approval for First Loss Default Guarantee
(FLDG) framework.
✓ About First Loss Default Guarantee (FLDG):
✓ What is it? FLDG is a lending model between fintech firms and their partner banks and non-banking
finance companies where the initial hit on a default is taken by the fintech firm that originated the loan.
✓ Under these agreements, the fintech originates a loan and promises to compensate the partners up to a
pre-decided percentage in case customers fail to repay.
✓ The bank/NBFC partners lend through the fintech but from their books.
✓ FLDG helps expand the customer base of traditional lenders but relies on the fintech's underwriting
capabilities.
✓ It will also rationalise the existing prudential norms to implement resolution plans in respect of exposures
affected by natural calamities.

Q. Who is ranked 10 on Forbes list of top 50 global CMOs?


A) Sunalini Menon B) Sumit Virmani
C) Sunil A. D’Souza D) Harish Bhat
Answer : D

Follow us: Official Site, Telegram, Facebook, Instagram, Instamojo 355


✓ Forbes has ranked Tata Group’s brand custodian Harish Bhat as among the top ten most influential Chief
Marketing Officers (CMO) for the second consecutive year.
✓ Last year (2022), he was in the ninth position.
✓ William White, CMO of Walmart has topped the Forbes list of most influential CMOs 2023.
✓ While Ashimita Dubey, chief digital and marketing officer at L'Oreal, is ranked 21 and Infosys chief
marketing officer Sumit Virmani is at 41
Q. Which Company has decided to extend financial assistance of Rs. 3,045 crores to Bangalore Metro Rail
Corporation Limited (BMRCL)?
A) GAIL B) REC Limited
C) NTPC Limited D) CESC Limited
Answer : B
✓ REC Limited, a Maharatna Central Public Sector Enterprise under the Union Ministry of Power, has
decided to extend financial assistance of Rs. 3,045 crores to Bangalore Metro Rail Corporation Limited
(BMRCL), for establishment and development of Metro Lines under Phase-II project of Bangalore
Metro.
Q. Ministry of Power launched the ‘Mission on Advanced and High-Impact Research (MAHIR)’ along with
which Union Ministry?
A) Ministry of Finance B) Ministry of Home affairs
C) Ministry of Education D) Ministry of Renewable Energy
Answer : D
✓ Recently, the Ministry of Power and the Ministry of New and Renewable Energy have jointly launched
a National Mission on Advanced and High-Impact Research.
✓ About MAHIR:
✓ The mission aims to quickly identify emerging technologies in the power sector and develop them
indigenously, at scale, for deployment within and outside India.
✓ Funding:
✓ The Mission will be funded by pooling financial resources of the Ministry of Power, Ministry of New
and Renewable Energy and the Central Public Sector Enterprises under the two Ministries.
✓ Additional funding will be mobilized from the Government of India's budgetary resources.
✓ Duration: The mission is Planned for an initial period of five years from 2023-24 to 2027-28.
Q. World Bank approval for a __ million loan to enhance the quality of technical education in across India.
A) $200 Million B) $223.7 Million
C) $255.5 Million D) $280.2 Million
Answer : C
✓ World Bank has granted its approval for a USD 255.5 million loan aimed at enhancing the quality of
technical education in government-run institutions across India.
✓ This funding will support approximately 275 selected government-run technical institutions over the
course of the next five years, benefiting more than 350,000 students annually.
✓ The Multidisciplinary Education and Research Improvement in Technical Education Project will focus
on improving student skills and employability through various measures.

Follow us: Official Site, Telegram, Facebook, Instagram, Instamojo 356


Q. RBI expanded the scope of ‘Trade receivables discounting system (TReDS)’ by allowing which
institutions?
A) Private sector banks B) Insurance Companies
C) Public sector banks D) Co-operative Bank
Answer : B
✓ The Reserve Bank on Wednesday expanded the trade receivables discounting system (TReDS) by
permitting insurance companies to function as participants.
✓ About Trade Receivables Discounting System (TReDS) platform:
✓ It is an electronic platform for facilitating the financing/discounting of trade receivables of Micro, Small,
and Medium Enterprises (MSMEs) through multiple financiers.
✓ These receivables can be due from corporates and other buyers, including Government Departments and
Public Sector Undertakings (PSUs).
✓ Purpose: To allow MSME sellers to discount invoices raised against major corporations, which helps
them manage their working capital demands. The platform enables MSMEs to receive payments more
quickly.
✓ Participants:
✓ Sellers, buyers, and financiers are the participants on a TReDS platform.
✓ Only MSMEs can participate as sellers in TReDS.
✓ Corporates, Government Departments, PSUs, and any other entity can participate as buyers in TReDS.
✓ Banks, NBFC - Factors, and other financial institutions, as permitted by the RBI, can participate as
financiers in TReDS
Q. Which fintech company has launched “Turbo UPI” in partnership with Axis Bank and the National
Payments Council of India (NPCI)?
A) Paytm B) PhonePe
C) Google Pay D) Razorpay
Answer : D
✓ Fintech major Razorpay has launched “Turbo UPI” in partnership with Axis Bank and the National
Payments Council of India (NPCI).
✓ The new product will allow customers of online merchants to make UPI payments directly without
getting redirected to a third-party UPI app during checkout.
✓ Turbo UPI aims to reduce potential payment failures.
✓ Razorpay Turbo UPI can deliver a 5X faster payment experience and can help businesses achieve an
increase in the success rate of UPI payments by 10 percent.
Q. What is the name of the portal launched for monitoring the implementation of Centrally Sponsored
Schemes of Justice Ministry ?
A) Sacred Portal B) TAPAS Portal
C) Nyaya Vikas Portal D) Green Day Ahead Market (GDAM) portal
Answer : C
✓ The Nyaya Vikas Portal has been created for monitoring the implementation of the Centrally Sponsored
Scheme, Nyaya Vikas.
✓ About Nyaya Vikas Portal
✓ This portal helps in empowering stakeholders with seamless access to information about funding,
documentation, project monitoring and approval.
✓ Key features of the Nyaya Vikas Scheme.

Follow us: Official Site, Telegram, Facebook, Instagram, Instamojo 357


✓ The Department of Justice has been implementing the Centrally Sponsored Scheme (CSS) for the
Development of Infrastructure Facilities for Districts and Subordinate Judiciary since 1993-94.
✓ Under the Scheme, central assistance is provided to the State Government / UT Administrations for the
construction of court halls and residential units for Judicial Officers / Judges of District and Subordinate
Courts.
✓ The funds sharing pattern under the Scheme for Centre and State is 60:40 in respect of States other than
North Eastern and Himalayan States.
✓ The fund’s sharing pattern is 90:10 in respect of North Eastern and Himalayan States and 100% in respect
of Union Territories.
✓ The portal has been created for monitoring the implementation of this Scheme.

Q. Ashwani Kumar has been appointed as new MD & CEO of which bank?
A) Bank of Baroda B) Canara Bank
C) UCO Bank D) Indian Overseas Bank
Answer : C
✓ The Appointments Committee of the Cabinet (ACC) has approved the appointment of Ashwani Kumar,
Executive Director of Indian Bank as MD and CEO of UCO Bank.
✓ Kumar will replace Soma Sankara Prasad, who is due to superannuate on May 31. It may be recalled
that Centre had in December 2021 appointed Prasad as MD & CEO of UCO Bank.
✓ Ashwani Kumar will lead the public sector bank for period of three years from June 1
✓ About UCO Bank
✓ UCO Bank was established in 1943, as United Commercial Bank.
✓ Slogan of the bank is “Honours Your Trust“
✓ The bank also has two overseas branches in Singapore and Hong Kong.
Q. Which Card launched country's first mobile-based expense management card?
A) IndusInd Card B) Karbon Card
C) LIC Card D) Citibank Card
Answer : B
✓ Karbon Card has launched a mobile-based expense management card for businesses and individuals
wherein it offers its users uncapped cashback of upto 1%.
✓ In comparison to other prepaid instruments like UPI and debit cards, which do not offer any cashback,
Card ++ differentiates itself by providing interesting cashback offers, incentivizing people for their
spending. The unique feature adds value to the card, making it an option for users
Q. Government has decided to allow how many Primary Agricultural Credit Societies (PACS) to open
Pradhan Mantri Bhartiya Jan Aushadhi Kendras across the country?
A) 5000 B) 2000
C) 3000 D) 4000
Answer : B
✓ Recently, the Government of India has decided to allow 2000 Primary Agricultural Credit Societies
(PACS) to open Pradhan Mantri Bhartiya Jan Aushadhi Kendras across the country.
✓ About Pradhan Mantri Bhartiya Jan Aushadhi Kendras:
✓ These are set up under Pradhan Mantri Bhartiya Janaushadhi Pariyojana, which was launched by the
Department of Pharmaceuticals, Ministry of Chemicals and Fertilizers in November 2008.

Follow us: Official Site, Telegram, Facebook, Instagram, Instamojo 358


✓ Objective: To provide quality medicines at affordable prices for all, particularly the poor, to reduce out-
of-pocket expenses in healthcare.
✓ These Jan Aushadhi Kendras provide generic drugs, which are available at lesser prices but are equivalent
in quality and efficacy to expensive branded drugs.
✓ Jan Aushadhi stores also sell allied medical products commonly sold in chemist shops to improve the
viability of running the Jan Aushadhi store.
✓ Pharmaceutical & Medical Devices Bureau of India (PMBI) has been established under the Department
of Pharmaceuticals, Govt. of India, with the support of all the CPSUs for co-coordinating procurement,
supply, and marketing of generic drugs through the PMBKs.
✓ The incentive amount of Rs.5 lakh will be provided to set up Pradhan Mantri Bhartiya Jan Aushadhi
Kendra.
Q. Which Group has signed an MoU with the Gujarat government to set up a first-ever lithium-ion cell
manufacturing gigafactory in Gujarat?
A) Adani Group B) Mahindra Group
C) Aditya Birla Group D) Tata Group
Answer : D
✓ Tata Group signed an agreement with the Gujarat state government to set up a giga-factory for
manufacturing lithium-ion cells, with an estimated initial investment of around 130 billion rupees ($1.6
billion).
✓ A giga-factory of this kind will be operational in Gujarat for the production of the first-ever lithium-ion
cells in India. This plant will have an estimated initial investment of around Rs.13,000 crore and will
have a production capacity of 20 GWh

Q. Consider the following statements regarding prepaid payment instruments (PPIs).


1. Prepaid payment instruments (PPIs) are payment instruments that facilitate the buying of goods and
services, including the transfer of funds and remittances, against the value stored within the instrument.
2. PPIs are in the form of smart cards, mobile wallets and magnetic chips.
3. As per RBI regulations, NBFCs cannot issue PPIs.
4. The Deposit Insurance and Credit Guarantee Corporation (DICGC) cover is available for the
depositors under PPIs
How many of the above statements are correct?
A) Only one B) Only two
C) Only three D) All four
Answer : B
✓ Recently, a committee set up to review the Customer Service Standards in RBI Regulated Entities has
recommended that the central bank should examine the extension of Deposit Insurance and Credit
Guarantee Corporation (DICGC) cover to PPI.
✓ About Prepaid Payment Instruments:
✓ These are instruments that facilitate the purchase of goods and services, conduct of financial services and
enable remittance facilities, among others, against the money stored in them. PPIs can be issued as cards
or wallets.
✓ There are two types of PPIs – small PPIs and full-KYC (know your customer) PPIs.
✓ Further, small PPIs are categorized as – PPIs up to Rs 10,000 (with cash loading facility) and PPIs up to
Rs 10,000 (with no cash loading facility).

Follow us: Official Site, Telegram, Facebook, Instagram, Instamojo 359


✓ PPIs can be loaded/reloaded by cash, debit to a bank account, or credit and debit cards.
✓ The cash loading of PPIs is limited to Rs 50,000 per month subject to the overall limit of the PPI.
✓ Who can issue PPI instruments?
✓ PPIs can be issued by banks and non-banks after obtaining approval from the RBI.
✓ Some of the approved PPI issuing banks are; Airtel Payments Bank, Axis Bank, Bank of Baroda, Jio
Payments Bank, Kotak Mahindra Bank etc.
✓ Key Facts about Deposit Insurance and Credit Guarantee Corporation
✓ It is a statutory body established under the Deposit Insurance and Credit Guarantee Corporation Act, of
1961.
✓ It is a wholly-owned subsidiary of the Reserve Bank of India (RBI).
✓ It provides deposit insurance that works as a protection cover for bank deposit holders when the bank
fails to pay its depositors.
✓ The agency insures all kinds of deposit accounts of a bank, such as savings, current, recurring, and fixed
deposits up to a limit of Rs. 5 lakh per account holder per bank.
✓ In case an individual's deposit amount exceeds Rs.5 lakh in a single bank, only Rs.5 lakh, including the
principal and interest, will be paid by DICGC if the bank becomes bankrupt

Q. State Bank of India has launched 'Project Kuber' in Which Circle?


A) Hyderabad Circle B) Bengaluru Circle
C) Vijayawada Circle D) Chennai Circle
Answer : B
✓ State Bank of India (SBI), Bengaluru Circle, recently launched 'Project Kuber' to meet the banking needs
of its customers.
✓ Project Kuber' aims to facilitate, and cater to various banking requirements of customers in Bengaluru
Circle.
✓ Project Kuber' aims to facilitate, and cater to various banking requirements of customers in Bengaluru
Circle.
✓ The project includes the establishment of four Transaction Banking Hubs and one Corporate Salary
Package Hub.
✓ These hubs will focus on increasing SBI's market share in liability products.
✓ The hubs will serve as a one-stop destination for digital banking services such as Yono Business, e-
Payments, Cash Management Products, and more.
Q. With reference to the Peer-to-Peer (P2P) lending, consider the following statements:
1. It enables individuals to obtain loans directly from other individuals without the involvement of the
financial institution.
2. The minimum capital required to set up a P2P platform is Rs. 200 Crores. Which of the statements
given above is/are correct?
A)1 Only B) 2 Only
C) Both 1 and 2 D) Neither 1 nor 2
Answer : A
✓ After digital payments and digital lending, the Reserve Bank of India is looking closely at platforms that
facilitate direct, or peer-to-peer (P2P), lending between individuals.
✓ About Peer-to-Peer (P2P) lending:
✓ It is done through a website that connects borrowers and lenders directly.
✓ Those who want to lend money, open an account with a P2P platform as a lender. And those who require
a loan register themselves as a borrower.

Follow us: Official Site, Telegram, Facebook, Instagram, Instamojo 360


✓ It enables individuals to obtain loans directly from other individuals, cutting out the financial institution
as the middleman.
✓ In 2017, the Reserve Bank of India brought this service under its regulatory purview.
✓ Only an NBFC can register as a P2P lender with the permission of RBI. Every P2P lender should obtain
a certificate of registration from the RBI.
✓ The minimum capital requirement to set up a P2P platform is fixed at Rs. 2 Crores.
Q. According to a new report by Standard Chartered, India is set to remain UAE’s largest export destination
by which year?
A) 2025 B) 2026
C) 2030 D) 2028
Answer : C
✓ India set to remain UAE’s largest export destination by 2030
✓ India is set to remain the United Arab Emirates (UAE) largest export destination by 2030, according to
a new report by Standard Chartered.
✓ The report, titled “Future of Trade: New opportunities in high-growth corridors”, forecasts that the
UAE’s exports will reach around USD 543 billion by 2030, showcasing a robust annual growth rate of
5.5 percent.
✓ The report also highlights that trade corridors anchored in Asia, Africa, and the Middle East are expected
to outpace the global trade growth rate by almost four percentage points, propelling the combined trade
volume in these regions to an astounding USD14.4 trillion, accounting for 44 percent of global trade by
2030.
✓ India is set to remain the UAE’s largest export destination by 2030, while exports to Turkiye, Vietnam
and Singapore are among the fastest growing
Q. Which state government has recently handed over legal documents regarding Shanan Powerhouse to the
Union Power Ministry?
A) Uttarakhand
B) Jammu and Kashmir
C) Ladakh
D) Himachal Pradesh
Answer : D
✓ Himachal Pradesh Chief Minister recently handed over legal documents regarding Shanan Powerhouse
to the Union Power Ministry.
✓ About Shanan Powerhouse:
✓ Location: It is located in Joginder Nagar in the Mandi district of Himachal Pradesh.
✓ Commissioned in 1932, the powerhouse was constructed as per a 99-year lease executed between Raja
Jogendra Sen, the then king of Mandi, and Col BC Batty, Chief Engineer of the Punjab Government, in
1925.
✓ This powerhouse was constructed against the backdrop of dense deodar forests.
✓ It is one of the oldest powerhouses of the country, which used to feed the entire undivided Punjab, Lahore
and Delhi before Independence.
✓ After the reorganisation of states in 1966, the Shanan powerhouse was given to Punjab by the Centre as
the lease agreement, signed in 1925 was yet to expire.
✓ The project originally was of 48 MW capacity, but the Punjab government enhanced its capacity to 60
MW in 1982. Later, 50 MW more was added to make its capacity 110 MW.
✓ The main attraction of the hydro project is the four-stage haulage trolley service.

Follow us: Official Site, Telegram, Facebook, Instagram, Instamojo 361


Q. Which of the following entity has recently came out with draft guidelines for 'Bima Vahaks (BV)'?
A) SEBI B) SIDBI
C) IRDAI D) NABARD
Answer : C
✓ IRDAI issues draft Bima Vahaks guidelines.
✓ IRDAI recently came out with draft guidelines for 'Bima Vahaks (BV)'.
✓ It seeks to establish a dedicated distribution channel at the level of Gram Panchayats.
✓ The guidelines propose Corporate Bima Vahaks and Individual Bima Vahaks for the distribution channel.
✓ Corporate Bima Vahak would be legal personsregistered in accordance with the respective laws and
engaged by an insurer. Individual Bima Vahak could be any individual appointed by an insurer or
appointed by a Corporate Bima Vahak.
✓ The BVs, both corporate and individual, would be authorised to undertake activities like the collection
of proposal information, and KYC documents and coordinate claims-related services.
✓ As per the draft, every insurer will have to provide alternative mode for payment of premium by prospects
or policyholders.
✓ Objective of the guidelines.
✓ This initiative aims to improve accessibility and availability of insurance in every nook and corner of the
country.
✓ Insurance Regulatory Development Authority of India (IRDAI).
✓ IRDAI is the regulatory body responsible for overseeing and regulating the insurance industry in India.
✓ It was set up to promote and regulate the insurance sector in India.
✓ It was established on April 1, 2000, under the provisions of the Insurance Regulatory and Development
Authority Act, 1999.
✓ The primary role of the IRDAI is to regulate and promote the insurance industry in India
Q. Which bank has introduced a new credit card named ‘Millennia Credit card’ which offers cashback over
e-commerce platforms and multiple benefits?
A) ICICI Bank B) HDFC Bank
C) Yes Bank D) Axis Bank
Answer : B
✓ HDFC Bank Introduces Millennia Credit Card.
✓ HDFC Bank introduced a new credit card named ‘Millennia Credit card’ which offers cashback over e-
commerce platforms and multiple benefits.
✓ Millennia credit card is applicable to both self-employed and salaried Indian national.
✓ Age: The minimum age limit is 21 and maximum age is 40.
✓ Income: For Salaried persons the Gross Monthly Income has to be more than Rs 35,000 and for Self
Employed the income has to be Rs 6.0 Lakhs per annum.
✓ The card comes with contactless technology allowing customers to ‘tap and pay’ via POS machine.

Q. Which of the following is NOT an Indirect tax?


A) Sales Tax II B) Custom Duty
C) Gift Tax D) Excise Tax
Answer : C
✓ Recently, the Central Board of Direct Taxes (CBDT) has exempted buyers from gift tax when they
acquire equity shares in public-sector units (PSUs) through strategic disinvestment.
✓ About Gift Tax:

Follow us: Official Site, Telegram, Facebook, Instagram, Instamojo 362


✓ The Parliament of India introduced the Gift Tax Act in 1958, and gift tax is essentially the tax charged
on the receipt of gifts.
✓ The Income Tax Act states that gifts whose value exceeds Rs.50,000 are subject to gift tax in the hands
of the recipient.
✓ The gift tax is also applicable on certain transfers that are not considered a gift.
✓ The transfer of existing movable or immovable property in money or money's worth qualifies for gift tax.
✓ The gift is exempted from tax if it was given by a relative.
✓ The income tax rule Parent, Spouse, Siblings, Spouse's siblings, Lineal descendants Lineal descendants
of the spouse can be considered as a relative.
✓ Income tax, corporation tax, property tax, inheritance tax and gift tax are examples of direct tax
Q. Which bank has launched a new service called Interoperable Cardless Cash Withdrawal (ICCW), which
allows customers to withdraw cash using UPI (Unified Payments Interface) from the bank’s ATMs?
A) Bank of Baroda B) Kotak Mahindra Bank
C) Canara Bank D) Indian Overseas Bank
Answer : A
✓ Bank of Baroda has launched a new service called Interoperable Cardless Cash Withdrawal (ICCW),
which allows customers to withdraw cash using UPI (Unified Payments Interface) from the bank’s
ATMs.
✓ This makes Bank of Baroda the first public sector bank to introduce this service. With this, customers can
avail two transactions a day per account with a withdrawal limit of Rs 5,000 per transaction.
✓ The first public sector bank to launch this service, Bank of Baroda customers as well as customers of other
participating issuer banks that use BHIM UPI, bob World UPI or any other UPI application enabled for
ICCW on their mobile phone can withdraw cash from a Bank of Baroda ATM without using their debit
card.
✓ Bank of Baroda (BOB)
✓ Founded : 1908
✓ Founder : Sayajirao Gaekwad III
✓ Headquarters : Vadodara, Gujarat
✓ MD & CEO : Debadatta Chand
✓ Tagline : India’s International Bank
Q. Which Union Ministry launched the ‘Unified Registration Portal For GOBARdhan’?
A) Ministry of Jal Shakti
B) Ministry of Road Transport and Highways
C) Ministry of Corporate Affairs
D) Ministry of Agriculture & Farmers Welfare
Answer : A
✓ Recently, Union Minister for Jal Shakti has launched the Unified Registration Portal for GOBARdhan.
✓ About Unified Registration Portal:
✓ This portal will serve as a centralized repository for assessing investment and participation in the
Biogas/CBG (Compressed Biogas) sector at a pan India level.
✓ Primary objective: To streamline the process of setting up CBG/Biogas plants in the country.
✓ The Unified Registration Portal allows any government, cooperative or private entity operating or
intending to set up a Biogas/CBG/Bio-CNG plant in India to obtain a registration number by enrolling
in the portal.

Follow us: Official Site, Telegram, Facebook, Instagram, Instamojo 363


✓ This registration number will enable them to avail a multitude of benefits and support from the Ministries
and Departments of the Government of India.
✓ GOBARdhan Scheme:
✓ Galvanizing Organic Bio-Agro Resources Dhan, is a crucial umbrella initiative of the Government of
India.
✓ The government launched the Gobardhan scheme in 2018 as a national priority project under the Swachh
Bharat Mission Grameen-Phase II program.
✓ Aim:
✓ To generate wealth and energy by converting cattle dung, agricultural residue, and other organic waste
into Biogas, CBG and bio-fertilizers.
✓ It adopts a whole-of-government approach and aims to convert waste into wealth, thereby promoting a
circular economy.
✓ Nodal Ministry: The Department of Drinking Water and Sanitation, Ministry of Jal Shakti
Q. JP Morgan has Partners with how many Indian banks to launch a pilot program designed to test a
blockchain-based platform for settlement of interbank US dollar transactions?
A) 6 B) 5
C) 4 D) 7
Answer : A
✓ JPMorgan Partners with 6 Indian banks to trial blockchain-based USD settlement.
✓ JPMorgan Chase & Co, the largest bank by assets under management in the United States (US), has
joined forces with six leading Indian banks to launch a pilot program designed to test a blockchain-based
platform for the settlement of interbank US Dollar transactions.
✓ The 6 Indian banks to be joining in the pilot program to test the new blockchain-based platforms
includeHDFC Bank, ICICI Bank, Axis Bank, Yes Bank and IndusInd Bank and JPMorgan’s own
banking subsidiary at Gujarat International Finance Tec-City (GIFT City ).
✓ Under the pilot, the six Indian banks will open on-chain Nostro accounts with JPMorgan’s branch in
Gift City.
✓ Nostro Account –A nostro account, derived from the Latin word for “ours,” is an account held by a bank
in a foreign currency at another bank. These accounts are commonly utilized to facilitate foreign
exchange and trade transactions.
✓ A vostro account is an account held by a foreign bank in the local currency at another bank. Vostro
accounts, derived from the Latin word for “yours”.
Q. Recently, RBI has given its nod to which bank to form a special committee of directors in order to oversee
the operations?
A) Capital Small Finance Bank B) Utkarsh Small Finance Bank
C) Ujjivan Small Finance Bank D) Fincare Small Finance Bank
Answer : C
✓ The Reserve Bank of India (RBI)-appointed director, recently resigned from the board of Ujjivan Small
Finance Bank (SFB).
✓ About Small Finance Banks (SFBs):
✓ SFBs are specialized banks that are licensed by RBIto provide financial services and products to low-
income individuals and underserved communities, including microfinance and micro-enterprise services,
as well as other basic banking services.
✓ Aim:
✓ To provide financial inclusion to these segments of the population who are often excluded from the
traditional banking system.

Follow us: Official Site, Telegram, Facebook, Instagram, Instamojo 364


✓ SFBs help them to have access to financial products such as small loans, savings, insurance, and other
basic banking services.
✓ SFBs are registered as public limited companies under the Companies Act, 2013 and governed by
Banking Regulations Act, 1949; RBI Act, 1934 and other relevant Statutes and Directives from time to
time.
✓ The guidelines for SFBS were introduced in 2014 by RBI. RBI Guidelines on SFBs in India are:
✓ SFBs are granted the scheduled bank status after being operational and are deemed suitable under section
42 of the RBI Act,1934.
✓ SFBs are required to primarily focus on providing access to financial services to the unbanked and
underbanked segments of the population.
✓ They are required to maintain a minimum Capital to Risk-Weighted Assets Ratio (CRAR) of 15%.
✓ They are required to extend 75% of their Adjusted Net Bank Credit to Priority Sector Lending.
✓ SFBs are required to open at least 25% of their total branches in unbanked rural areas.
✓ The minimum paid-up voting equity capital for small finance banks shall be Rs.200 crore.
✓ SFBs are required to maintain at least 50% of their loan portfolio as microfinance and advances of up to
Rs. 25,00,000.

Q. Which organisation released the Global Economic Prospects Report ?


A) IMF B) ADB
C) AIIB D) World Bank
Answer : D
✓ World Bank (WB) in its Global Economic Prospects Report-June 2023 has lowered India’s growth
outlook to 6.3% for 2023, a 0.3 percentage point downward revision from 6.6% in January 2023.
✓ However, India will remain the fastest-growing economy (in terms of both aggregate and per capita GDP)
of the largest EMDEs (Emerging Market and Developing Economies).
✓ For 2024, WB expects GDP growth rate at 6.4%. While for 2025 forecast, the growth rate is expected at
6.5%.
✓ Global growth is projected to decelerate from 3.1% in 2022 to 2.1% in 2023. It will be 2.4% in 2024 and
3% in 2025.
Q. Union government proposes to sell up to what percentage stake in state-owned coal India through an
offer for sale (OFS).
A) 4% B) 3%
C) 5% D) 7%
Answer : B
✓ The Union government proposes to sell up to 3% stake in state-owned coal India through an offer for sale
(OFS).
✓ OFS is a simpler method of share sale through the exchange platform for listed companies.
✓ The OFS method was brought in by the Securities and Exchange Board of India (SEBI) in 2012 as a
simpler one to aid promoters of listed forms to dilute their stake and comply with the minimum public
shareholding norms by June 2013.
✓ Only promoters or shareholders holding more than 10 per cent of the share capital in a company can
come up with such an issue.
✓ The mechanism is available to 200 top companies in terms of market capitalisation.
✓ In an OFS, a minimum of 25 per cent of the shares offered, are reserved for mutual funds (MFs) and
insurance companies.

Follow us: Official Site, Telegram, Facebook, Instagram, Instamojo 365


Q. According to ICRA report on Indian banking sector, bond issuance by banks in FY2024 to decline from
a record high of Rs 1.1 lakh crore in FY23 to how many crore rupees?
A) Rs 90,000 crore B) Rs 70,000 crore
C) Rs 80,000 crore D) Rs 60,000 crore
Answer : A
✓ Bond issuances by banks to fall below Rs 90,000cr in FY24: ICRA Report.
✓ According to the ICRA report on Indian Banking Sector, Bond issuances by banks are projected to
decrease to Rs 90,000 crore in FY24 from a record high of Rs 1.1 lakh crore in FY23.
✓ This is attributed to improved liquidity conditions and increased inflows from foreign investors.
✓ Banks issue bonds as a means to secure funding and strengthen their capital base.
✓ Reason behind the expected decrease in bond issuances:
✓ Stronger foreign institutional investor (FII)/foreign portfolio investment (FPI) inflows, derecognition of
the Rs 2,000 note, and a potential dividend from the Reserve Bank of India (RBI) are anticipated to
alleviate liquidity conditions temporarily.
✓ Regarding Tier-I or Additional Tier I (ATI) bonds, issuances are projected to be lower than the levels
observed between FY2022 and FY2023, estimated at Rs 30,000-33,400 crore due to lower scheduled call
options .
✓ Public sector banks (PSBs) will remain larger issuers of tier-I bonds in the current financial year.
✓ ATI bonds are unsecured bonds with no pre-determined maturity date, and are issued to raise long-term
capital.
✓ There will be a slight increase in outstanding Tier-I bonds, which are expected to reach Rs 1.3 lakh crore
by March 2024 compared to Rs 1.2 lakh crore as of March 2023.
✓ As for Tier-II bonds, a moderation is expected after the peak observed in FY23, which amounted to Rs
49,600 crore. However, some banks may still opt to raise infrastructure bonds.
✓ According to the latest RBI’s money market operation data, liquidity surplus has risen from around Rs
56,000 crore to Rs 2.11 lakh crore
Q. According to the International Energy Agency's (IEA) Renewable Energy Market Update 2023, how
many gigawatts of global renewable capacity is expected to be added?
A) 120 GW B) 380 GW
C) 107 GW D) 210 GW
Answer : C
✓ IEA report: Global Renewable Capacity Additions to increase by 107 GW in 2023 .
✓ According to Renewable Energy Market Update – June 2023 of International Energy Agency (IEA), the
Global renewable capacity additions are set to increase by 107 gigawatts (GW), the largest absolute
increase ever, to more than 440 GW in 2023.
✓ The growth in renewable capacity is fueled by the expanding policy support, growing energy security
concerns and improving competitiveness against fossil fuel alternatives.
✓ India’s renewable capacity additions are expected to increase again in 2023 and 2024, owing to faster
onshore wind, hydropower and distributed solar PV deployment.
✓ International Energy Agency (IEA)
✓ Founded : 1974
✓ Headquarters : Paris, France
✓ Executive Director : Fatih Birol
✓ Membership : 30

Follow us: Official Site, Telegram, Facebook, Instagram, Instamojo 366


Q. Which of the following entity has approved the acquisition of an additional 4.04% shareholding of Acko
Technology and Services Private Limited (Acko Tech) by GASACK?
A) Reserve Bank of India (RBI)
B) Competition Commission of India (CCI)
C) Securities and Exchange Board of India (SEBI)
D) Confederation of Indian Industry (CII)
Answer : B
✓ CCI approves acquisition of 4.04% shareholding in Acko Tech by GASACK
✓ The Competition Commission of India (CCI) approved the acquisition of an additional 4.04%
shareholding of Acko Technology and Services Private Limited (Acko Tech) by General Atlantic
Singapore ACK Pte. Ltd. (GASACK).
✓ The CCI has approved the proposed combination subject to compliance with amendments jointly
provided by GASACK and GAP Bermuda, LP pursuant to Regulation 19(2) of the Rules of the
Competition Commission of India (Procedure in Commercial Transactions Related to Combinations),
2011.
✓ Competition Commission of India (CCI)
✓ It is the chief national competition regulator in India.
✓ It was set up under the Competition Act, 2002.
✓ It comes under the Ministry of Corporate Affairs.
✓ It promotes healthy market competition and prevents activities that have an adverse effect on competition
in India.
✓ CCI also approves merger and acquisitions of companies in India so that the two merging entities do not
dominate the market unfairly.
✓ Headquarter - New Delhi
✓ Chairman : Ravneet Kaur
Q. Which institution has conceptualized ‘Lightweight Payments System’?
A) RBI B) SEBI
C) SIDBI D) NPCI
Answer : A
✓ The Reserve Bank of India (RBI) is developing a lightweight and portable payment system designed to
operate during catastrophic events.
✓ About Lightweight and portable payment system:
✓ The RBI has conceptualized this system which it is calling a bunker which is an equivalent of digital
payments that can be operated from anywhere by a bare minimum staff in exigencies such as natural
calamities or war.
✓ It is expected to operate on minimalistic hardware and software and would be made active only on a
need basis.
✓ The infrastructure for this system will be independent of the technologies that underlie the existing
systems of payments such as UPI, NEFT, and RTGS.
✓ The system is expected to process transactions that are critical to ensure the stability of the economy,
including government and market-related transactions.
✓ The existing conventional payment systems such as RTGS, NEFT, and UPI are designed to handle large
volumes of transactions while ensuring sustained availability.
✓ What is NEFT?
✓ The National Electronic Funds Transfer is an electronic method of transferring money online.

Follow us: Official Site, Telegram, Facebook, Instagram, Instamojo 367


✓ It enables transferring funds from the account maintained with any bank to any other bank branch,
provided the transaction is attempted between the banks that participate in the NEFT payment system.
✓ The payments made via NEFT are processed and settled half hourly batches and transactions can be
performed 24*7.
✓ Minimum Transfer Value: Rs. 1
✓ Maximum transfer value: No limit
✓ Money transfer made through NEFT does not require any additional transaction costs.
✓ What is RTGS?
✓ It stands for Real-time Gross Settlement, which is a payment mode where the money is transferred from
one bank account to the other in real time, without any delay.
✓ It is mostly used for transactions of high value.
✓ When using the banking method, RTGS is the fastest possible way to transfer money.
✓ Transactions made through RTGS are processed on a one-to-one basis and transactions can be performed
24*7.
✓ Minimum Transfer Value: 2 lakh
✓ Maximum transfer value: No upper limit is there, but can vary between banks.
Q. Which of the following has introduced Account Aggregator (AA) services following the receipt of
Reserve Bank of India (RBI) Non-Banking Finance Companies (NBFC) licence?
A) PhonePe B) Google Pay
C) Paytm D) Mobikwik
Answer : A
✓ Fintech major PhonePe has introduced Account Aggregator (AA) services following the receipt of
Reserve Bank of India (RBI) Non-Banking Finance Companies (NBFC) licence.
✓ The company launched it through its wholly-owned subsidiary PhonePe Technology Services.
✓ PhonePe received the Reserve Bank of India’s (RBI) approval to operate as an account aggregator in
2021.
✓ The account aggregator services will allow Indian consumers to consent to and share all their financial
data such as bank statements, insurance policies and tax filings with regulated financial institutions.
✓ PhonePe Technology Services has also launched its product suite and application programming interface.
The fintech company built an account aggregator micro-application within the PhonePe consumer
application
✓ PhonePe
✓ Founded : 2015
✓ Headquarters : Bangalore, Karnataka
✓ Founder & CEO : Sameer Nigam
✓ Co-Founder & CTO : Rahul Chari
✓ It is owned by Flipkart, a subsidiary of Walmart.
✓ PhonePe app, based on the Unified Payments Interface (UPI), went live in August 2016.
Q. Which company has recently launched a new ‘voice chat’ feature to its Al-powered Bing Chat on the
desktop?
A) Google B) IBM
C) Microsoft D) Tesla
Answer : C
✓ Microsoft has launched a new ‘voice chat’ feature to its Al-powered Bing Chat on the desktop.
✓ It will help users to talk to the Al chatbot by clicking on the microphone icon in the Bing Chatbox.

Follow us: Official Site, Telegram, Facebook, Instagram, Instamojo 368


✓ Currently, the voice chat feature supports five languages — English, Japanese, French, German, and
Mandarin. The company said it will add more languages soon.
✓ Bing Chat supports text-to-speech answers, which will respond to your questions in its own voice.
Q. Which Union Ministry implements the City Investments to Innovate, Integrate and Sustain 2.0 (CITIIS
2.0)?
A) Ministry of Commerce and Industry
B) Ministry of Coal
C) Ministry of Housing and Urban Affairs
D) Ministry of Micro, Small and Medium Enterprises
Answer : C
✓ Recently, the Union Cabinet chaired by the Indian Prime Minister has approved the City Investments to
Innovate, Integrate and Sustain 2.0 (CITIIS 2.0) programme.
✓ About CITIIS 2.0 Program:
✓ It is a program conceived by the Ministry of Housing and Urban Affairs (MoHUA) in partnership with
the French Development Agency (AFD), Kreditanstalt für Wiederaufbau (KfW), the European Union
(EU), and National Institute of Urban Affairs (NIUA).
✓ The program will run for four years, i.e., from 2023 to 2027.
✓ Objective: The program envisages supporting competitively selected projects promoting a circular
economy with a focus on integrated waste management at the city level, climate-oriented reform actions
at the State level, and institutional strengthening and knowledge dissemination at the National level.
✓ Funding: The funding for this program would include a loan from AFD and KfW and a technical
assistance grant of Rs.106 cr from the EU.
Q. Which of the following has approved the First Loss Default Guarantee (FLDG) framework, allowing
fintech to partner with banks and non-Banking financial companies (NBFCs)?
A) RBI B) SEBI
C) SIDBI D) IRDAI
Answer : A
✓ The Reserve Bank of India (RBI) has approved the First Loss Default Guarantee (FLDG) framework,
allowing fintech to partner with banks and Non-Banking financial companies (NBFCs).
✓ The central bank has allowed a 5% default loss cover for bank-fintech deals.
✓ The RBI had earlier specified that banks and NBFCs must abide by securitization norms before entering
into any guarantee arrangement.
✓ Subject to certain guidelines, the RBI has decided to permit arrangements between regulated entities and
lending service providers or between two regulated entities involving default loss guarantee.
✓ The limits are aimed at reining in the risk of large slippages in tie-ups between lenders and fintech.
Q. State Bank of India (SBI) has approved raising of funds aggregating how many crores via issuance of debt
instruments, including capital instruments in FY24?
A) Rs 50,000 crore B) Rs 80,000 crore
C) Rs 10,000 crore D) Rs 20,000 crore
Answer : A
✓ SBI to raise Rs 50,000 crore in FY24.

Follow us: Official Site, Telegram, Facebook, Instagram, Instamojo 369


✓ The central board of State Bank of India (SBI) has approved raising of funds aggregating ₹50,000 crore
via issuance of debt instruments, including capital instruments in FY24.
✓ The board’s approval is for raising funds in the Indian rupee and/or any other convertible currency by
issue of debt instruments, including but not limited to Long Term Bonds, Basel III-compliant Additional
Tier 1 Bonds, Basel III-compliant Tier 2 Bonds.
✓ The resources will be raised through private placement mode to Indian and/or overseas investors during
FY24, subject to the government’s approval wherever required.
✓ SBI had raised funds aggregating about ₹38,850 crore through issuance of infrastructure bonds, Tier-2
bonds and AT-1 bonds in FY23.
✓ Though the issuing banks of tier 1 bonds can be called after a period of 5 or 10 years.
Q. Which bank has become the first public sector bank to introduce an IVR-based UPI solution – UPI
123PAY?
A) State Bank of India B) Punjab National Bank
C) Indian Overseas Bank D) Kotak Mahindra Bank
Answer : B
✓ Punjab National Bank (PNB) has become the first public sector bank to introduce an IVR-based UPI
solution – UPI 123PAY.
✓ What is UPI 123PAY
✓ Unified Payments Interface (UPI) is a 24-hour payment channel enabling customers to perform fast,
encrypted and real-time payments.
✓ Till now, the solution was accessible only through smartphones or through Unstructured Supplementary
Service Data (USSD) service and was dependent on good internet connectivity.
✓ The UPI 123PAY solution would extend the functionality and facilitate the feature to any phone users
and those in low internet connectivity zones to perform UPI transactions
✓ How to use UPI 123PAY
✓ The UPI 123PAY functionality is simple and involves the following steps:
✓ Step 1-Dial the bank’s easy-to-remember IVR number “9188-123-123″
✓ Step 2-Choose the beneficiary
✓ Step 3-Authenticate the transaction
✓ UPI 123PAY will be multilingual and be available in the customer’s preferred language.
✓ The solution is available in multiple languages and can be accessed by dialing 9188-123-123.
✓ IDFC First Bank, City Union Bank, and NSDL Payments Bank have also gone live on IVR payments
✓ About Punjab National Bank (PNB)
✓ It is an Indian public sector bank based in New Delhi and it is the third largest public sector bank in India.
✓ PNB has a banking subsidiary in the UK as well as branches in Dubai, Kabul, Kowloon and Hong Kong.
✓ Established - May 1894
✓ Founder - Dayal Singh Majithia, Lala Lajpat Rai
✓ Tagline - "The Name You Can Bank Upon"
✓ MD & CEO - Atul Kumar Goel
✓ Punjab National Bank Merge – (Oriental Bank of Commerce & United Bank of India)

Q. Who has approved US$ 255.5 million with an aim to enhance the quality of technical education in India?
A) World Bank B) Asian Development Bank
C) International Monetary Fund D) New Development Bank
Answer : A
✓ World Bank has granted its approval for a USD 255.5 million loan aimed at enhancing the quality of
technical education in government-run institutions across India.

Follow us: Official Site, Telegram, Facebook, Instagram, Instamojo 370


✓ This funding will support approximately 275 selected government-run technical institutions over the
course of the next five years, benefiting more than 350,000 students annually.
✓ The Multidisciplinary Education and Research Improvement in Technical Education Project will focus
on improving student skills and employability through various measures.

Q. Which bank has recently planned to raise Rs 750 crore from bonds to fund business growth?
A) Karnataka Bank B) Indian Overseas Bank
C) Saraswat Bank D) Punjab & Sind Bank
Answer : D
✓ State-owned Punjab & Sind Bank has planned to raise Rs 750 crore from bonds to fund business growth.
✓ The board has approved the raising of capital up to Rs 750 crore through the issuance of Basel-Ill
compliant Additional Tier-I Bonds or Tier-Il Bonds.
✓ The fundraising will be done in one or more tranches within a period of 12 months
✓ Punjab & Sind Bank
✓ Founded : 24 June 1908
✓ Headquarters : New Delhi
✓ MD & CEO : Swarup Kumar Saha
✓ Executive Director : Kollegal V Raghavendra
✓ Tagline : Where Service is a Way of Life
Q. Which entity has introduced a regulatory framework for online investment platforms that facilitate
investments in commission-free mutual fund (MF) schemes?
A) RBI B) SEBI
C) SIDBI D) NPCI
Answer : B
✓ The Securities and Exchange Board of India (Sebi) has introduced a regulatory framework for online
investment platforms that facilitate investments in commission-free mutual fund (MF) schemes.
✓ The regulation brings clarity on the obligations of such platforms while opening an avenue to monetize
their business. It also addresses investor concerns on associated risks and introduces grievance redressal
mechanism.
✓ As per the framework, platforms providing MF investment services in direct (commission-free) plans will
have to apply for ‘execution only platform’ (EOP) licence either with Sebi or the Association of Mutual
Funds in India (Amfi) within three months of the regulation coming into force, which happens in
September 2023.
✓ Platforms of investment advisors and stock brokers will not need an EOP registration if it is open only
for their advisory or broking clients.

Q. Who is ranked 10 on Forbes list of top 50 global CMOs?


A) Sunalini Menon B) Sumit Virmani
C) Sunil A. D’Souza D) Harish Bhat
Answer : D
✓ Forbes has ranked Tata Group’s brand custodian Harish Bhat as among the top ten most influential Chief
Marketing Officers (CMO) for the second consecutive year.
✓ Last year (2022), he was in the ninth position.
✓ William White, CMO of Walmart has topped the Forbes list of most influential CMOs 2023.

Follow us: Official Site, Telegram, Facebook, Instagram, Instamojo 371


✓ While Ashimita Dubey, chief digital and marketing officer at L'Oreal, is ranked 21 and Infosys chief
marketing officer Sumit Virmani is at 41.
Q. Which bank has launched a digital platform for hassle-free account opening platform that eliminates the
hassle of paper-based documentation?
A) Indian Overseas Bank B) Kotak Mahindra Bank
C) South Indian Bank D) Karur Vysya Bank
Answer : C
✓ South Indian Bank has introduced SIB SWIFTe, an innovative and customer-centric digital account
opening platform that eliminates the hassle of paper-based documentation.
✓ The platform prioritizes customer convenience and transforms the account opening process, enabling the
bank to swiftly on-board customers in less than five minutes.
✓ This facility brings the branch-like experience to the comfort of customers’ homes, allowing them to
connect with bankers at their convenience.
✓ SIB SWIFTe, a user-friendly and intuitive interface, provides clear instructions and visual cues to guide
both customers and bank representatives through each step.
✓ South Indian Bank
✓ Founded : 1929
✓ Headquarters : Thrissur, Kerala
✓ Chairman : Salim Gangadharan
✓ MD & CEO : Murali Ramakrishnan
✓ Tagline : Experience Next Generation Banking.
Q. Which payments bank has entered into a tie-up with Sequoia Capital-backed fintech Hubble to introduce
India’s first spending account?
A) India Post Payments Bank B) Fino Payments Bank
C) Paytm Payments Bank D) Airtel Payments Bank
Answer : B
✓ Fino Payments Bank has entered into a tie-up with Sequoia Capital-backed fintech Hubble to introduce
India’s first spending account.
✓ The bank customers can park their funds, spend (on categories such as food ordering, shopping, travel,
entertainment, etc.) and save up to 10 percent on all purchases made from the account.
✓ Further, users can track spends across categories of over 50 brands that Hubble has partnered with and
save up to ₹20,000 a year.
✓ The spending account is integrated with the bank’s digital savings account operated through its mobile
app FinoPay. Users earn an interest rate of 2.75 percent per annum on the funds parked in the account
Q. Which IT company has bagged a digital transformation deal from Denmark-based Danske Bank
estimated to be around USD 454 million for five years?
A) IBM B) HCL
C) TCS D) Infosys
Answer : D
✓ Infosys bags $454-million deal from Danske Bank.
✓ IT major Infosys has bagged a digital transformation deal from Denmark-based Danske Bank estimated
to be around USD 454 million for five years.

Follow us: Official Site, Telegram, Facebook, Instagram, Instamojo 372


✓ As part of the strategic collaboration with Danske Bank, Infosys will acquire Danske Bank’s IT centre in
India, which employs 1400 digitally skilled professionals.
✓ This collaboration will help Danske Bank achieve its strategic priorities towards better customer
experiences, operational excellence, and a modernized technology landscape, powered by next-gen
solutions.
✓ INFOSYS
✓ Founded : 1981
✓ Headquarters : Bangalore, Karnataka
✓ Chairman : Nandan Nilekani
✓ MD & CEO : Salil Parekh

Q. Which bank has introduced “MY ACCOUNT MY NAME” for Savings Bank account customers?
A) Indian Overseas Bank B) Kotak Mahindra Bank
C) South Indian Bank D) Karur Vysya Bank
Answer : A
✓ Indian Overseas Bank has introduced “MY ACCOUNT MY NAME” for Savings Bank account
customers.
✓ This is the first time such a feature has been introduced in the banking industry.
✓ In ‘My Account My Name’ customer can choose any name (alias) for his/her account in addition to the
account name. This ‘Alias Name’ name can be shared by customers to others instead of sharing the 15-
digit account numbers.
✓ The account name can be a combination of seven letters only or seven numbers only or seven
alphanumeric — like AJIT007, PRADHAN or 2424707 and others.
✓ Indian Overseas Bank (IOB)
✓ Founded : 1937
✓ Founder : M. Chidambaram Chettyar
✓ Headquarters : Chennai, Tamil Nadu
✓ MD & CEO : Ajay Kumar Srivastava
✓ Tagline : Good People to Grow With
Q. Who Joined Hands to Accelerate Sustainable Development in India?
A) NITI Aayog and United Nations
B) DRDO and United Nations
C) Planning Commission and United Nations
D) Ministry of Power and United Nations
Answer : A
✓ NITI Aayog, United Nations join hands to accelerate sustainable development in India.
✓ NITI Aayog and the United Nations in India signed the Government of India-United Nations
Sustainable Development Cooperation Framework 2023-2027 in New Delhi .
✓ The Government of India-United Nations Sustainable Development Cooperation Framework (GoI-
UNSDCF) for the period 2023-2027 was signed by CEO of NITI Aayog, B V R Subrahmanyam, and
UN Resident Coordinator India, Shombi Sharp.
✓ The signing ceremony took place in the presence of Vice Chairman of NITI Aayog, Suman Bery, senior
representatives from NITI Aayog, central ministries, and heads of UN agencies in India.
✓ GoI-UNSDCF 2023-2027

Follow us: Official Site, Telegram, Facebook, Instagram, Instamojo 373


✓ The GoI-UNSDCF 2023-2027 represents the collective commitment of the UN development system to
support the Government of India in achieving the Sustainable Development Goals (SDGs) while
promoting gender equality, youth empowerment, and human rights.
✓ It aligns with the national vision for development and aims to contribute to the country's progress.
✓ The Government of India-UNSDCF, through its partnerships, outcomes and outputs, will play a key role
in realizing the national development priorities.
Q. Which bank has signed a Memorandum of Understanding (MoU) with the Tamil Nadu State Rural
Livelihood Mission (TNSRLM) to foster the growth of Self-Help Groups (SHGs) in the State?
A) Canara Bank B) Kotak Mahindra Bank
C) Indian Bank D) Indian Overseas Bank
Answer : D
✓ Indian Overseas Bank (IOB) has signed a Memorandum of Understanding (MoU) with the Tamil Nadu
State Rural Livelihood Mission (TNSRLM) to foster the growth of Self-Help Groups (SHGs) in the State
and provide them with vital financial support.
✓ Under the terms of the MoU, TNSRLM will facilitate eligible SHGs for credit linkage with IOB.
✓ This strategic collaboration aims to streamline access to financial resources for SHGs, enabling them to
undertake entrepreneurial activities, expand their ventures, and contribute to the economic development
of the State.
✓ The collaboration is expected to catalyse the growth of SHGs targeting ₹1,875 crore and increase the
disbursal under DAY-NRLM (Deendayal Antyodaya Yojana-National Rural Livelihoods Mission)
during this fiscal, effectively doubling the impact of financial assistance
✓ Indian Overseas Bank (IOB)
✓ Founded : 1937
✓ Founder : M. Chidambaram Chettyar
✓ Headquarters : Chennai, Tamil Nadu
✓ MD & CEO : Ajay Kumar Srivastava
✓ Tagline : Good People to Grow With

Q. Who has reappointment of MD and CEO of Equitas Small Finance Bank?


A) M. V. Nair
B) Arun Kumar Mishra
C) Rekha Sharma
D) P N Vasudevan
Answer : D
✓ Equitas Small Finance Bank the Reserve Bank of India gave its approval for the re-appointment of P N
Vasudevan as Managing Director and Chief Executive officer of the bank.
✓ Vasudevan has been reappointed for a period of three years, from July 23, 2023, to July 23, 2026.
✓ Equitas Small Finance Bank
✓ Founded : 2016
✓ Headquarteres : Chennai
✓ MD & CEO : Vasudevan P N
✓ Tagline : It's Fun Banking
✓ Rani Rampal & Smriti Mandhana : Brand ambassador of Equitas Small Finance Bank

Follow us: Official Site, Telegram, Facebook, Instagram, Instamojo 374


Q. Which organization has approved a $150 million loan to support the Resilient Kerala program to
continue strengthening the state’s preparedness against natural disasters, climate change impacts, and
disease outbreaks?
A) World Bank B) ADB
C) IMF D) AIIB
Answer : A
✓ World Bank approves $150-million loan for Resilient Kerala programme.
✓ The World Bank has approved a $150 million loan to support the Resilient Kerala program to continue
strengthening the state’s preparedness against natural disasters, climate change impacts, and disease
outbreaks.
✓ This additional financing will further deepen Kerala’s resilience in the critical areas of coastal erosion
and water resource management.
✓ The loan, from the International Bank for Reconstruction and Development (IBRD), has a final maturity
of 14 years, including a grace period of six years.
Q. Who has published IS 18267: 2023 “Food Serving Utensils Made from Agri By-Products Specification,”
aimed at reducing plastic pollution and promoting sustainability?
A) Bureau of Indian Standards (BIS)
B) Food Corporation of India (FCI)
C) International Organization for Standardization (ISO)
D) Food Safety and Standards Authority of India (FSSAI)
Answer : A
✓ Bureau of Indian Standards (BIS) has issued standards for agricultural by-product utensils with an aim
to reduce plastic pollution and promote sustainability.
✓ The published standard IS 18267: 2023 “Food Serving Utensils Made from Agri By-Products –
Specification” provides comprehensive guidelines to manufacturers and consumers to ensure uniformity
in quality requirements across the country.
✓ BIS stated that the use of biodegradable agri by-product utensils will contribute to environmental safety,
conservation of natural resources and promotion of a circular economy.
✓ IS 18267: 2023 covers several aspects for the production of biodegradable utensils; including raw
materials, manufacturing techniques, performance and hygiene requirements.
✓ It specifies the use of agricultural by-products, such as leaves and sheaths, as preferred materials for
making plates, cups and bowls.
✓ About Bureau of Indian Standards
✓ Bureau of Indian Standards is thenational standards body of India under the Department of Consumer
Affairs, Ministry of Consumer Affairs, Food and Public Distribution, Government of India.
✓ Headquarter- Manak Bhawan, Old Delhi.
✓ Director General- Pramod Kumar Tiwari.
✓ Established- 23 December 1986
Q. The Department of Legal Affairs under the Union law ministry has set up a panel to recommend reforms
in the Arbitration and Conciliation Act. The committee would be led by __.
A) T K Vishwanathan B) Rajesh Verma
C) Ramesh Kandula D) Mahendra Kumar Sharma
Answer : A

Follow us: Official Site, Telegram, Facebook, Instagram, Instamojo 375


✓ The Government of India has constituted an expert panel headed by former Law Secretary TK
Vishwanathan to propose reforms in the Arbitration and Conciliation Act and reduce the burden on the
courts.
✓ The expert committee comprises Attorney General N Venkataramani and members from various
government departments, private law firms and legal entities.
✓ The mandate of the Committee includes evaluating the current arbitration ecosystem in India, analyzing
the strengths, weaknesses and challenges in comparison to foreign jurisdictions.
✓ The panel is expected to submit its report within 30 days and recommend a framework for a cost-effective
arbitration system.
✓ The Committee will focus on developing a competitive environment both domestically and
internationally in the arbitration services market, and will aim at establishing a cost-effective arbitration
regime.
Q. Which financial institution has approved $391 million in financing to improve access to high-quality
healthcare services and to economic opportunities for people in Assam and Tripura?
A) World Bank B) New Development Bank
C) Asian Development Bank D) International Monetary Fund
Answer : A
✓ The World Bank has approved $391 million in financing to improve access to high-quality healthcare
services and to economic opportunities for people in Assam and Tripura.
✓ The Assam State Secondary Healthcare Initiative for Service Delivery Transformation (ASSIST) Project,
for which $251 million was approved, will improve access to high-quality secondary healthcare services
in Assam and directly serve at least 1.8 million people

Q. Who has unveiled its version of the metaverse called Bharatmeta?


A) india.ai B) Wistron.ai
C) Better.ai D) Kiya.ai
Answer : D
✓ India is poised to become a major player in the metaverse technology landscape, as showcased by Kiya.ai,
an India-based company specializing in Artificial Intelligence and emerging technology solutions.
✓ At the prestigious VivaTech event held in Paris 2023, Kiya.ai unveiled its version of the metaverse called
Bharatmeta, making it the first of its kind from India.
✓ The demonstration took place at the India Pavilion during the event ‘VivaTech’ , which is recognized as
Europe's largest annual technology and startup gathering.
Q. Indian Government has provided financial incentives of ___ crore rupees to 12 states to accelerate power
sector reforms.
A) Rs. 60,000 Crores B) Rs. 66,000 Crores
C) Rs. 73,000 Crores D) Rs. 79,000 Crores
Answer : B
✓ Centre has provided financial incentives to 12 states to accelerate power sector reforms.
✓ These states will receive over 66 thousand crore rupees incentives for reforms in the power sector.
✓ This initiative aims to support and motivate states in implementing reforms that enhance the efficiency
and performance of the power sector.
✓ The announcement regarding this initiative was made in the Union Budget 2021-22 by the Union Finance
Minister.

Follow us: Official Site, Telegram, Facebook, Instagram, Instamojo 376


✓ As part of the initiative, states are granted an additional borrowing space of up to 0.5 percent of their
Gross State Domestic Product (GSDP) annually for a period of four years, from 2021-22 to 2024-25.
✓ Based on the recommendations of the Ministry of Power, the Ministry of Finance has granted permission
for the reforms undertaken by 12 state governments in the years 2021-22 and 2022-23.
✓ As a result, these states have been allowed to raise financial resources of Rs 66,413 crore through the
additional borrowing permissions over the past two financial years.
Q. Who has been appointed as the non-executive chairman by Axis Bank?
A) Shaktikanta Das B) Urjit Patel
C) NS Vishwanathan D) Ajay Mehra
Answer : C
✓ Axis Bank has appointed NS Vishwanathan, former deputy governor of the Reserve Bank of India, as
non-executive chairman.
✓ The board of directors of the bank has appointed Vishwanathan for a period of three years. Apart from
Axis Bank, Vishwanathan was appointed as the head of the advisory board at Razorpay in April 2023.
✓ Vishwanathan had joined RBI in 1981 and retired as Deputy Governor in March 2020
Q. The Department of Expenditure under the Finance Ministry has approved capital investment proposals
of Rs 56,415 cr for how many states in the current financial year 2023-24?
A) 10 B) 16
C) 12 D) 14
Answer : B
✓ The Department of Expenditure under the Finance Ministry has approved capital investment proposals
of Rs 56,415 cr for 16 states in the current financial year 2023-24.
✓ The amount has been approved for 16 states including Arunachal Pradesh, Bihar, Chhattisgarh, Goa,
Gujarat, Haryana, Himachal Pradesh, Karnataka, Madhya Pradesh, Mizoram, Odisha, Rajasthan,
Sikkim, Tamil Nadu, Telangana, and West Bengal.
✓ The approval has been given under the scheme titled ‘Special Assistance to States for Capital Investment
2023-24’.
✓ The scheme, which was announced in the Budget for 2023-24 in continuation of a similar push for capex
from the last three years, special assistance is being provided to the state governments in the form of 50-
year interest-free loan up to an overall sum of Rs 1.3 lakh crore during the financial year 2023-24.
Q. Which bank has recently launched its ActivMoney feature, which gives customers the benefit of fixed
deposit (FD)-like interest up to 7 percent per annum?
A) Indian Overseas Bank B) Kotak Mahindra Bank
C) Yes Bank D) HDFC Bank
Answer : B
✓ Kotak Mahindra Bank launches ‘ActivMoney’
✓ Kotak Mahindra Bank launched its ActivMoney feature, which gives customers the benefit of fixed
deposit (FD)-like interest up to 7 percent per annum, and the flexibility to access their funds at any time
from the linked savings account.
✓ This is an auto sweep feature that is offered by several other banks, including Axis Bank, HDFC Bank,
ICICI Bank, and IndusInd Bank.

Follow us: Official Site, Telegram, Facebook, Instagram, Instamojo 377


Q. Which company has signed an agreement with the German government for investing more than 30
billion euros ($32.8 billion) in a chip manufacturing site in Magdeburg, Germany?
A) Intel B) HP
C) IBM D) Samsung
Answer : A
✓ Intel has signed an agreement with the German government for investing more than 30 billion euros
($32.8 billion) in a chip manufacturing site in Magdeburg, Germany.
✓ Intel acquired the land for two semiconductor facilities in Magdeburg in eastern Germany in November
2022 and is expected to start production in four or five years.
✓ The investment in Germany means a significant expansion of Intel’s production capacity in Europe and
is the biggest investment ever made by a foreign company in Germany.
✓ About Intel
✓ Founded : 1968
✓ Founders : Gordon Moore, Robert Noyce, Arthur Rock
✓ Headquarters : Santa Clara, California, United States
✓ CEO : Patrick P. Gelsinger
✓ Chairperson : Omar Ishrak

Q. Which bank has partnered with Wise to offer low-cost & fast inward remittance to India?
A) IndusInd Bank B) Bank of Baroda
C) Punjab National Bank D) Canara Bank
Answer : A
✓ IndusInd Bank has forged a strategic partnership with Wise, a global technology company specializing
in secure money transfers.
✓ The collaboration aims to provide convenient and efficient online inward remittance services specifically
tailored for Non-Resident Indians (NRIs) residing in the United States and Singapore.
✓ Through this partnership, IndusInd Bank’s customers can transfer funds from these two major global
markets. The remittance service, known as Indus Fast Remit (IFR), will integrate with the Wise Platform,
allowing NRIs to enjoy a multi-currency inward remittance service
✓ INDUSLND BANK
✓ Founded : 1994
✓ Founder : S.P. Hinduja
✓ Headquarters : Pune, Maharashtra,
✓ MD & CEO : Sumant Kathpalia
✓ Tagline : We Make You Feel Richer
Q. Which has become the first Indian airline to achieve market capitalization of Rs 1 lakh crore?
A) Vistara B) Air India
C) Spice Jet D) Indigo Airline
Answer : D
✓ InterGlobe Aviation Ltd, which operates budget carrier Indigo, has become India's leading airline to
cross ₹1 trillion in market capitalization (Mcap) on June 28, 2023.
✓ The stock hit a 52-week high of ₹2,634.25 on the BSE earlier in the session 28th June 2023, with a market
value of ₹1.01 trillion.
✓ Shares of IndiGo settled 3.55 percent higher at ₹2,619.85 apiece on the BSE, compared to the previous
close of ₹2,529.95.

Follow us: Official Site, Telegram, Facebook, Instagram, Instamojo 378


Q. Who has been appointed as chairman of GIFT (Gujarat International Finance Tec) City?
A) Ajit Doval B) Satya Pal Malik
C) Nripendra Misra D) Hasmukh Adhia
Answer : D
✓ Hasmukh Adhia has been appointed as the chairman of GIFT City Limited.
✓ Hasmukh Adhia is a retired bureaucrat and former Union Finance Secretary and Revenue Secretary.
✓ Before Hasmukh Adhia, former Chief Secretary of Gujarat Sudhir Mankad was in the post of president.
✓ GIFT City aims to promote international financial services and develop infrastructure for the finance and
technology sectors.
✓ Hasmukh Adhia has been appointed as the director and chairman of the boards of Gujarat Alkalies and
Chemicals Limited (GACL) and Gujarat Mineral Development Corporation Limited (GMDC).
✓ About GIFT City
✓ Gujarat International Finance Tec (GIFT) City is a central business district under construction in
Gandhinagar district of Gujarat, India.
✓ It is India's first operational Greenfield Smart City and International Financial Services Centre.
✓ GIFT City has been promoted as a greenfield project by the Government of Gujarat.
✓ GIFT City is divided into two zones: the International Financial Services Center (IFSC) and the
Domestic Tariff Area (DTA).
✓ IFSC is designed for global financial institutions and international transactions.
✓ DTA caters to home businesses in a variety of industries.
Q. World Bank has approved How many billions in financing to accelerate the development of India's low-
carbon energy sector?
A) $2.6 billion B) $1.9 billion
C) $1.5 billion D) $1.2 billion
Answer : C
✓ World Bank has approved $1.5 billion in financing to accelerate the development of India’s low carbon
energy sector.
✓ The financing will help India promote low-carbon energy by scaling up renewable energy, developing
green hydrogen and stimulating climate finance for low-carbon energy investments, the bank said in a
statement.
✓ The program will support the successful implementation of the National Green Hydrogen Mission that
aims to stimulate $100 billion in private sector investment by 2030,” said Auguste Tano Kouame, World
Bank Country Director for India
Q. Which bank has signed a Memorandum of Understanding (MoU) with the Tamil Nadu State Rural
Livelihood Mission (TNSRLM) to foster the growth of Self-Help Groups (SHGs) in the State?
A) Karur Vyasya Bank B) Union Bank of India
C) Indian Overseas Bank D) Bank of Baroda
Answer : C
✓ Indian Overseas Bank (IOB) has signed a Memorandum of Understanding (MoU) with the Tamil Nadu
State Rural Livelihood Mission (TNSRLM) to foster the growth of Self-Help Groups (SHGs) in the State
and provide them with vital financial support.
✓ Under the terms of the MoU, TNSRLM will facilitate eligible SHGs for credit linkage with IOB.

Follow us: Official Site, Telegram, Facebook, Instagram, Instamojo 379


✓ The collaboration is expected to catalyse the growth of SHGs targeting ₹1,875 crore and increase the
disbursal under DAY-NRLM (Deendayal Antyodaya Yojana-National Rural Livelihoods Mission)
during this fiscal, effectively doubling the impact of financial assistance.
Q. Which fintech company has signed an MoU with Arunachal Pradesh Innovation and Investment Park
(APIIP) to create a startup ecosystem for the youth in the northeastern state?
A) PhonePe B) Google Pay
C) Paytm D) BHIM
Answer : C
✓ Paytm Payment Services Limited (PPSL) has signed an MoU with Arunachal Pradesh Innovation and
Investment Park (APIIP) to create a startup ecosystem for the youth in the northeastern state.
✓ According to the MoU, Paytm Incubation Centre will provide its product to early-stage startups at a
discounted price and offer free credits through the Paytm Startup toolkit to aid the business ventures of
youths in their early stage of growth.
✓ Paytm is India’s payment app, which offers consumers and merchants payment services. It is a pioneer
of mobile QR payments in the country.
Q. Which of the following has launched Anti Money Laundering AI (AML AI) to help financial institutions
detect money laundering more effectively and efficiently?
A) Microsoft Azure B) Amazon Web Services
C) Google Drive D) Google Cloud
Answer : D
✓ Google Cloud has launched Anti Money Laundering AI (AML AI) to help financial institutions detect
money laundering more effectively and efficiently.
✓ The product utilizes machine learning to assist banks and other financial institutions in meeting
regulatory requirements for identifying and reporting suspicious activities.
✓ The tool is designed to aid global financial institutions to detect money laundering more effectively and
efficiently.
✓ The risk score is based on the bank’s data including network behavior, and Know Your Customer (KYC)
data.
Q. Which Company has decided to extend financial assistance of Rs. 3,045 crores to Bangalore Metro Rail
Corporation Limited (BMRCL)?
A) GAIL B) REC Limited
C) NTPC Limited D) CESC Limited
Answer : B
✓ REC Limited, a Maharatna Central Public Sector Enterprise under the Union Ministry of Power, has
decided to extend financial assistance of Rs. 3,045 crores to Bangalore Metro Rail Corporation Limited
(BMRCL), for establishment and development of Metro Lines under Phase-II project of Bangalore
Metro.
✓ Phase-II project of Namma Metro
✓ The Phase-II project of Namma Metro includes the extension of the existing East-West Corridor and
North-South Corridor of Phase-I, as well as the addition of two new lines: RV Road to Bommasandra
and Kalena Agrahara to Nagawara.

Follow us: Official Site, Telegram, Facebook, Instagram, Instamojo 380


✓ REC becomes the 12th ‘Maharatna’ CPSE.
✓ Rural Electrification Corporation is a Non-Banking Finance Company (NBFC) under the Union
Ministry of Power.
✓ It was set up in 1969 and it focuses on Power Sector Financing and Development across India. Its
headquarters is in New Delhi

Q. Which city has Emerges as India’s Top Choice for Foreign Direct Investment (FDI)?
A) Singapore B) Dubai
C) New York D) London
Answer : B
✓ Dubai Emerges as India’s Top Choice for Foreign Direct Investment (FDI).
✓ According to the latest fDi Markets Report and the Dubai FDI Monitor, Dubai has secured its position
as the leading destination for foreign direct investment (FDI) from India in the calendar year 2022. India
ranked among the top five source countries for announced FDI projects and estimated FDI capital in
Dubai, solidifying the emirate’s appeal to Indian investors
Q. SEBI approved a framework for application supported by blocked amount (ASBA) ice facility being made
available to investor, for secondary market trading for top 100 firms which will come into effect from ?
A) 1 December 2023 B) 1 April 2023
C) 1 November 2023 D) 1 October 2023
Answer : D
✓ Securities and Exchange Board of India (SEBI) approved a broad framework for Application Supported
by Blocked Amount or ASBA-like facility being made available to investors for secondary market trading.
✓ The facility shall be optional for investors as well as stock brokers.
✓ The ASBA framework applies to initial public offers (IPO) and other primary market issues, where the
blocked fund remains in your account earning interest.
✓ SEBI tells top 100 listed firms to clarify market rumours from October 1, 2023.

Follow us: Official Site, Telegram, Facebook, Instagram, Instamojo 381


BEST MCQ BANKING, ECONOMY AND FINANCIAL AWARENESS
MAY 2023

Q. The Union Cabinet has approved the second edition of the Production Linked Incentive (PLI 2.0) scheme
for IT hardware with how much rupees?
A) Rs 17,000 crore B) Rs 23,000 crore
C) Rs 15,000 crore D) Rs 10,000 crore
Answer : A
✓ Union Cabinet, chaired by the Prime Minister Narendra Modi on 17th May approved the Production
Linked Incentive Scheme 2.0 for IT Hardware with a budgetary outlay Rs 17,000 crore.
✓ Electronics manufacturing in India has seen a steady growth with a CAGR (Compound Annual Growth
Rate) of 17% in the last 8 years.
✓ This year it crossed a major benchmark in production of USD 105 billion (about Rs 9 lakh crore).
✓ India has become the second largest mobile phone manufacturer in the world.
✓ Mobile phone exports have crossed a major milestone of US $ 11 billion (about Rs 90 thousand crore)
this year.
✓ The global electronics manufacturing ecosystem is coming to India, and India is emerging as a major
electronics manufacturing country.

✓ Features of PLI Scheme 2.0 for IT hardware


✓ It covers laptops, tablets, all-in-one PCs, servers and ultra small form factor devices.
✓ The tenure of this scheme is 6 years.
✓ Expected incremental production is Rs 3.35 Lakh crore.
✓ Expected incremental investment is Rs 2,430 crore.
✓ Expected incremental direct employment is 75,000.

✓ What is Production Linked Incentive Scheme?


✓ It is an initiative that provides incentives to domestic industries to promote production at the local level.
✓ Through this scheme, the government aims to incentivize companies to increase the sales of
manufactured products in domestic units.
✓ The objective of the scheme is to make domestic manufacturing globally competitive and to become a
global champion in manufacturing.
✓ The government has launched this scheme with an outlay of around Rs 2 lakh crore for 14 sectors
including Automobiles and Auto Components, White Goods, Pharma, Textiles, Advanced Chemistry
Cell and Specialty Steel.
✓ The support under the Scheme shall be provided for a period of five (5) years, i.e. from FY 2021-22 to
FY 2025-26.
Q. Which bank has been selected as a clearing and settlement bank by the Indian Clearing Corporation Ltd
ICCL?
A) Bank of India B) Canara Bank
C) Yes Bank D) Indian Bank
Answer : D

Follow us: Official Site, Telegram, Facebook, Instagram, Instamojo 382


✓ Indian Bank Joins ICCL as Clearing and Settlement Bank.
✓ Indian Bank announced that it has been selected as a Clearing and Settlement Bank by the Indian
Clearing Corporation Ltd (ICCL).
✓ As a result, this public sector bank is now authorized to provide banking services to members of the
Bombay Stock Exchange (BSE) for clearing and settlement operations
Q. Reserve Bank of India has approved how much rupees dividend payout to the central government for
2022-23?
A) Rs 74,896 Crore B) Rs 67,137Crore
C) Rs 87,416 Crore D) Rs 91,567 Crore
Answer : C
✓ RBI Board approved Transfer of Rs 87,416 Crore as Surplus to Government.
✓ the board of the Reserve Bank of India (RBI) approved the transfer of Rs 87,416 crore as surplus to the
Government for the year 2022-2023 at the 602nd meeting of the RBI’s Central Board of Directors,
Chaired by Governor Shaktikanta Das. It also decided to raise the contingency risk buffer to 6% from
5.5%.
✓ In the financial year (FY) 2021-2022, the RBI transferred Rs 30,307 crore to the government.
Q. Windfall tax which was in news recently has been introduced by government of India on which of the
following?
A) Crude Oil B) Entertainment
C) Corporate D) Agriculture
Answer : A
✓ India recently slashed the windfall tax on domestically produced crude oil to ₹4,100 per tonne from
₹6,400 per tonne.
✓ About Windfall Tax:
✓ What is it? It is a higher tax levied by the government on specific industries when they experience
unexpected and above-average profits.
✓ When is it imposed?
✓ When the government notices a sudden increase in an industry's revenue, they impose this tax.
✓ However, these revenues cannot be linked to anything the company actively pursued, such as its business
strategy or expansion.
✓ Consequently, a Windfall Tax is imposed on an industry's profits when it experiences a sharp increase in
revenue due to unrelated external events.
✓ Rationale behind the imposition of windfall tax:
✓ Redistribution of unexpected gains, when high prices benefit producers at the expense of consumers;
✓ To fund social welfare schemes;
✓ As a supplementary revenue stream for the government;
✓ As a way for the Government to narrow the country’s widened trade deficit;
Q. The Reserve Bank of India has decided to withdraw the ₹2000 denomination banknotes from circulation
but they will continue to remain as legal tender. This banknote was introduced in November 2016 under
which section of RBI Act 1934?
A) Section 14(1) B) Section 45a (1)
C) Section 31b D) Section 24(1)
Answer : D

Follow us: Official Site, Telegram, Facebook, Instagram, Instamojo 383


Q. Reserve Bank of India decided to withdraw the ₹2000 notes unde its ___ policy ?
A) Remove Note Policy B) Clean Note Policy
C) New Note Policy D) Clean and Remove Policy
Answer : B
✓ The Reserve Bank of India (RBI) has decided to withdraw Rs 2,000 denomination banknotes from
circulation under the "Clean Note Policy".
✓ Rs 2,000 notes will continue to be legal tender.
✓ Individuals can deposit the Rs 2,000 banknotes into their bank accounts or exchange them for other
denominations at any bank branch.
✓ From May 23, 2023, Rs 2,000 notes can be exchanged for other denominations up to a limit of Rs 20,000
at a time at any bank branch.
✓ RBI's 19 regional offices with issue departments will also provide the facility to exchange Rs 2,000
banknotes up to a limit of Rs 20,000 at a time from May 23.
✓ All banks to offer deposit and/or exchange services for Rs 2,000 notes by September 30, 2023.
✓ Clean Note Policy:
✓ The clean note policy was introduced by the Reserve Bank of India (RBI) in 1999 to address the issue of
soiled and mutilated notes.
✓ Banks are instructed not to give soiled, torn and mutilated notes to customers and instead deposit them
with RBI.
✓ Stapling of currency notes/packets is prohibited as per the policy.
✓ The objective of the Clean Note Policy is to provide good quality currency notes and coins to the
customers.
✓ Soiled and mutilated currency notes can be easily exchanged at teller counters under the RBI's note refund
rules of 2009.
✓ The ₹2000 denomination banknote was introduced in November 2016 under Section 24(1) of RBI Act,
1934.
✓ RS 2000
✓ Dimension : 66*166 mm
✓ Colour : Magenta
✓ Design : Mangalyaan
✓ Angular bleed lines for Visually Impaired : 7
Q. What is the corpus of the Start-up India Seed Fund Scheme (SISFS), which was approved recently?
A) Rs 868 crore B) Rs 777 crore
C) Rs 945 crore D) Rs 589 crore
Answer : C
✓ The Department for Promotion of Industry and Internal Trade (DPIIT) is undertaking a third-party
assessment of the Rs 945-crore Startup India Seed Fund Scheme to see its impact on the ground.
✓ About Startup India Seed Fund Scheme (SISFS):
✓ It is a flagship scheme launched in April 2021 under the Startup India
✓ Aim: Financial assistance to startups for proof of concept, prototype development, product trials, market
entry, and commercialization.
✓ It is implemented with effect from 1st April 2021 for the period of 4 years with a corpus of 945 crores.
✓ Nodal Department: Department for Promotion of Industry and Internal Trade (DPIIT), under the
Ministry of Commerce & Industry.
✓ Funding:
✓ An Experts Advisory Committee (EAC) has been created by DPIIT to execute and monitor the SISFS.
✓ The EAC will select eligible incubators who will be provided grants of up to Rs 5 Crores each.

Follow us: Official Site, Telegram, Facebook, Instagram, Instamojo 384


✓ In turn, the selected incubators will provide startups with up to Rs 20 lakhs for validation of Proof of
Concept, prototype development, and product trials to startups.
✓ Eligibility criteria for Startups:
✓ A startup recognized by DPIIT incorporated not more than 2 years ago at the time of application.
✓ The startup must have a business idea to develop a product or a service with a market fit, viable
commercialization, and scope of scaling.
✓ The startup should be using technology in its core product or service, or business model, or distribution
model, or methodology to solve the problem being targeted.
✓ Startup should not have received more than Rs 10 lakh of monetary support under any other Central or
State Government scheme.
✓ Shareholding by Indian promoters in the startup should be at least 51% at the time of application to the
incubator for the scheme
Q. Which company only Indian firm in BCG list of Top 50 innovative companies 2023 List?
A) Adani Group B) Larsen & Toubro
C) Tata Group D) Aditya Birla Group
Answer : C
✓ Tata Group only Indian firm in BCG list of top 50 Most Innovative Firms 2023; Apple Top.
✓ Boston Consulting Group (BCG) has released a report namely ‘Most Innovative Companies 2023:
Reaching New Heights in Uncertain Times’ showcasing top 50 companies in 2023 that are most
innovative in their operations. In this, Tata Group is the only Indian company to feature in this list at
20th position
✓ The top 10 ranks : In the list went to Apple, Tesla, Amazon, Alphabet, Microsoft, Moderna, Samsung,
Huawei, BYD and Siemens, in that order.
✓ BCG's list parameters : Superior performance, higher resilience to shocks and disruptions, and the ability
to exploit innovation faster for value-creating growth, among other things
Q. Consider the following statements regarding Central Bank Digital Currency (CBDC):
1. Their value is fixed by the central bank and equivalent to the country's fiat currency.
2. BDCs decrease the cost of maintenance that a complex financial system requires.
Which of the statements given above is/are correct?
A) 1 only B) 2 only
C) Both 1 and 2 D) Neither 1 nor 2
Answer : C
✓ The Reserve Bank of India (RBI) Deputy Governor recently said that Central Bank Digital Currency
(CBDC) platforms could bring about a substantive change in the sphere of cross-border payments.
✓ About Central Bank Digital Currency (CBDC):
✓ CBDCs are a form of digital currency issued by a country's central bank.
✓ Examples of central banks include the Reserve Bank of India (RBI), the US Federal Reserve System, the
Bank of Japan.
✓ CBDCs are similar to stablecoins, except that their value is fixed by the central bank and equivalent to
the country's fiat currency.
✓ It provides businesses and consumers with privacy, transferability, convenience, accessibility, and
financial security.
✓ It also decreases the cost of maintenance that a complex financial system requires.
✓ It reduces cross-border transaction costs.

Follow us: Official Site, Telegram, Facebook, Instagram, Instamojo 385


✓ It would also reduce the risks associated with using digital currencies, or cryptocurrencies, in their current
form. CBDCs, backed by a government and controlled by a central bank, would give households,
consumers, and businesses a secure means of exchanging digital currency
✓ What are Stablecoins?
✓ They are cryptocurrencies whose value is pegged or tied to that of another currency, commodity, or
financial instrument.
✓ Stablecoins aim to provide an alternative to the high volatility ofthe most popular cryptocurrencies,
including Bitcoin (BTC).
Q. How many million loan agreement has been signed by the Government of India with the Asian
Development Bank for infrastructure development in Andhra Pradesh?
A) USD 105.50 Million B) USD 122.75 Million
C) USD 141.12 Million D) USD 167.37 Million
Answer : C
✓ The Asian Development Bank (ADB) and the Government of India signed a $141.12 million loan
agreement for the development of an industrial corridor in Andhra Pradesh.
✓ The agreement has been signed to support the development of high quality indoor infrastructure facilities
in three industrial areas in Andhra Pradesh. These are: roads, water supply plants and power distribution
systems.
✓ The loan is the second tranche of a $500 million Multi-tranche Financing Facility (MFF) under the
program approved by ADB in 2016.
✓ With this loan facility, infrastructure facilities will be developed in three industrial clusters under the
Visakhapatnam and Srikalahasti-Chittoor areas sanctioned in Andhra Pradesh
Q. How many million loan agreement has been signed by the Government of India with the Asian
Development Bank for infrastructure development in Andhra Pradesh?
A) USD 105.50 Million B) USD 122.75 Million
C) USD 141.12 Million D) USD 167.37 Million
Answer : C
✓ The Asian Development Bank (ADB) and the Government of India signed a $141.12 million loan
agreement for the development of an industrial corridor in Andhra Pradesh.
✓ The agreement has been signed to support the development of high quality indoor infrastructure facilities
in three industrial areas in Andhra Pradesh. These are: roads, water supply plants and power distribution
systems.
✓ The loan is the second tranche of a $500 million Multi-tranche Financing Facility (MFF) under the
program approved by ADB in 2016.
✓ With this loan facility, infrastructure facilities will be developed in three industrial clusters under the
Visakhapatnam and Srikalahasti-Chittoor areas sanctioned in Andhra Pradesh
Q. Who is the Chairperson of Financial Stability and Development Council (FSDC) Sub-Committee?
A) Finance Minister B) Finance Secretary
C) RBI Governor D) RBI Deputy Governor
Answer : C
✓ Finance Minister recently chaired the 27th meeting of the Financial Stability and Development Council
(FSDC).
✓ About Financial Stability and Development Council (FSDC):

Follow us: Official Site, Telegram, Facebook, Instagram, Instamojo 386


✓ It is an apex-level forum constituted by the Government of India in December 2010.
✓ Status: FSDC is not a statutory body. No funds are separately allocated to the council for undertaking its
activities.
✓ Objective: Strengthening and institutionalizing the mechanism for maintaining financial stability,
enhancing inter-regulatory coordination and promoting financial sector development.
✓ Composition:
✓ It is chaired by the Union Finance Minister of India.
✓ Its members include the heads of financial sector Regulators (RBI, SEBI, PFRDA, IRDA & FMC)
Finance Secretary and/or Secretary, Department of Economic Affairs, Secretary, Department of
Financial Services, and Chief Economic Adviser.
✓ The Council can invite experts to its meeting if required.
✓ Sub-committee of FSDC:
✓ A sub-committee of FSDC has also been set up under the chairmanship of Governor RBI.
✓ It discusses and decides on a range of issues relating to financial sector development and stability,
including substantive issues relating to inter-regulatory coordination.
Q. The Foreign direct investment (FDI) into India has declined by 22% to how many billions in 2022-23?
A) $33 Billion B) $39 Billion
C) $46 Billion D) $53 Billion
Answer : C
✓ Foreign direct investment (FDI) into India declined by 22% to $46 billion in 2022-23, dragged by lower
inflows in computer hardware and software, and automobile industry, according to the Department for
Promotion of Industry and Internal Trade (DPIIT) data.
✓ The FDI inflows stood at $58.77 billion during 2021-22.
✓ Total FDI inflows, which include equity inflows, re-invested earnings and other capital, declined by 16%
to $70.97 billion in the last fiscal as against $84.83 billion in 2021-22.
✓ During April-March 2022-23, Singapore emerged as the top investor with $17.2 billion FDI.
✓ It was followed by Mauritius ($6.13 billion), the US ($6 billion), the UAE ($3.35 billion), the Netherlands
($2.5 billion).
✓ The FDI inflows have contracted in 2022-23 from Mauritius, the US, and the Netherlands, the Cayman
Islands, and Germany.
✓ State-wise, though Maharashtra received the highest inflows of $14.8 billion during the last financial year,
the inflows are down as compared to $15.44 billion in 2021-22.
Q. Which among the following is a primary benchmark for short term interest rates across the world?
A) PARIBA B) LIBOR
C) BARCLAYS D) WESTPAC
Answer : B
✓ The Reserve Bank of India (RBI) recently told banks and other regulated entities to ensure a complete
transition away from the London Interbank Offered Rate (LIBOR).
✓ About London Interbank Offered Rate (LIBOR):
✓ It is a benchmark interest rate at which major global banks lend to one another in the international
interbank market for short-term loans.
✓ It acts as a benchmark for short-term interest rates.
✓ It is an indicator of the health of the financial system and provides an idea of the trajectory of impending
policy rates of central banks.
✓ LIBOR is also the basis for consumer loans in countries around the world, so it impacts consumers just
as much as it does financial institutions.

Follow us: Official Site, Telegram, Facebook, Instagram, Instamojo 387


✓ The five currencies for which LIBOR is computed are the Swiss franc, euro, pound sterling, Japanese yen
and US dollar.
Q. Which bank has top performer in NPA management during FY23?
A) HDFC Bank B) Kotak Mahindra Bank
C) Bank of Maharashtra D) ICICI Bank
Answer : C
✓ Bank of Maharashtra (BoM) has emerged as the best bank as far as the management of bad loans is
concerned with its net non-performing assets (NPAS).
✓ The NPA fell as low as 0.25% during the financial year ended March 2023.
✓ This is the lowest ratio among all banks having total business above ₹3 lakh crore and not just public
sector banks (PSBs).
✓ While HDFC Bank’s net NPA fell to 0.27% followed by Kotak Mahindra Bank placed third spot with
✓ Bank of Maharashtra
✓ Founded : 1935
✓ Headquarters : Pune, Maharashtra
✓ MD & CEO : A. S. Rajeev
✓ Tagline : One Family One Bank/ Ek Parivar Ek Bank
Q. Ministry of Skill Development and Entrepreneurship (MSDE) will organise the Pradhan Mantri National
Apprenticeship Mela (PMNAM) in over how many districts across the country?
A) 200 B) 100
C) 150 D) 125
Answer : A
✓ Ministry of Skill Development and Entrepreneurship (MSDE) will organise the Pradhan Mantri National
Apprenticeship Mela (PMNAM) in over 200 districts across the country on May 9, 2023.
✓ The Ministry of Skill Development and Entrepreneurship will be organising the Pradhan Mantri National
Apprenticeship Mela every month now on.
✓ It's objective is to connect the youth with more opportunities of on-ground training within corporates and
further a chance to get employment.
✓ Individuals having a 5th-12th grade pass certificate, a skill training certificate, an ITI Diploma, or a
graduate degree can apply for an interview across these trades/opportunities.
✓ The candidates will be given a choice of 500+ trades, including welders, electricians, housekeepers,
beauticians, mechanics, and others.
✓ Candidates will also receive Apprenticeship certificates recognised by the National Council for
Vocational Education and Training (NCVET) at the end of their training period, which will give them
industry recognition

Q. Which country has become the largest recipient of ADB loans in Asia?
A) India B) China
C) Pakistan D) Bangladesh
Answer : C
✓ Pakistan emerges as largest recipient of ADB loans in 2022 in Asia.
✓ AS per the annual ADB Annual Report 2022, released by the Asian Development Bank (ADB), Pakistan
was the largest recipient of the ADB-funded programmes/projects in the year 2022.

Follow us: Official Site, Telegram, Facebook, Instagram, Instamojo 388


✓ The Manila-based lender, ADB had disbursed a total of over USD 31.8 billion to 40 Asian countries in
2022.
✓ Out of this, Pakistan received loans of USD 5.58 billion.

Q. Sanchar Saathi portal, which was launched recently, is used for which activity?
A) Cloud computing B) Cybersecurity
C) Tracking Lost or Stolen Mobile D) Artificial intelligence
Answer : C
✓ Union Minister of Communications recently launched a citizen-centric Sanchar Saathi portal.
✓ About Sanchar Saathi portal:
✓ The portal aims to provide various reforms and services related to mobile connections and
telecommunications.
✓ It has been developed by C-DoT under the Department of Tele-communications (DoT) to prevent frauds
such as identity theft, forged KYC, banking frauds etc.
✓ The three reforms are being introduced as part of the portal's framework:
✓ CEIR (Central Equipment Identity Register): It enables the tracking and blocking of lost or stolen phones
anywhere in the country.
✓ ASTR (Artificial Intelligence and Facial Recognition powered Solution for Telecom SIM Subscriber
Verification): This AI-based technology facilitates mobile connection analysis and includes features such
as IMEI-based phone theft information messaging to law enforcement agencies and the owner
Q. Which company Overtakes Infosys to become India's Sixth Most Valuable Company?
A) ITC B) Wipro
C) Asian Paints D) Larsen & Toubro
Answer : A
✓ FMCG giant ITC has overtaken IT giant Infosys in terms of market capitalization to become India’s sixth
most valuable company on the stock exchanges.
✓ ITC’s market cap currently stands at ₹5.11 trillion compared to Infosys’ nearly ₹5.08 trillion.
✓ List of Top 10 companies by market cap
1. RELIANCE INDUSTRIES ₹1606895.46
2. TCS ₹1161858.58
3. HDFC BANK ₹928407.83
4. ICICI BANK ₹637688.17
5. HUL ₹585048.22
6. ITC ₹511226.51
7. INFOSYS ₹508011.92
8. HDFC LTD. ₹505558.17
9. SBI ₹501250.83
10. BHARTI AIRTEL ₹430650.07
Q. The new flagship Pradhan Mantri Matsya Sampada Yojana (PMMSY) is being implemented by which
Ministry?
A) Ministry of Animal Husbandry, Dairying and Fisheries
B) Ministry of Corporate Affairs
C) Ministry of Agriculture & Farmers Welfare
D) Ministry of Ports, Shipping and Waterways

Follow us: Official Site, Telegram, Facebook, Instagram, Instamojo 389


Answer : A
✓ For giving fresh impetus to deliverables under the flagship Pradhan Mantri Matsya Sampada
Yojana(PMMSY), seven major field studies are being carried out by National Productivity Council
(NPC).
✓ The NPC is conducting field studies in seven areas such as best practices in fish marketing system of
Andhra Pradesh and dissemination through workshop; agro-climatic zone-specific mapping of
innovative fishing practices for enhancing production and productivity in Upper Gangetic Plain Region
etc.
✓ It will help strengthen the mechanism of delivery of the central scheme 'Pradhan Mantri Matsya Sampada
Yojana (PMMSY).
✓ National Productivity Council:
✓ It is an autonomous organization under the Department for Promotion of Industry and Internal Trade
(DPIIT), Ministry of Commerce & Industry.
✓ It was set up in 1958.
✓ The Pradhan Mantri Matsya Sampada Yojana (PMMSY) is a new flagship programme that is being
implemented by the Ministry of Fisheries, Animal Husbandry and Dairying envisages enhancing
fisheries exports to about Rs.1 lakh crore, and increasing the fish production to 22 million metric tons by
2024-25.
✓ Pradhan Mantri Matsya Sampada Yojana (PMMSY)
✓ PMMSY is being implemented by the Department of Fisheries (DoF), Ministry of Fisheries, Animal
Husbandry and Dairying (FAHD), Government of India.
✓ PMMSY is a scheme to bring about Blue Revolution through sustainable and responsible development
of fisheries sector in the country at an estimated investment of Rs. 20,050 crores.
✓ PMMSY is being implemented in all the States and Union Territories since FY 2020-21.
✓ It is designed to address critical gaps in the fisheries value chain from fish production, productivity and
quality to technology, post-harvest infrastructure and marketing.
Q. Which countries have agreed to explore the possibility of accepting RuPay and Mir cards for easy
payments?
A) India and Russia B) India and France
C) India and Japan D) India and United States
Answer : A
Q. Mir cards is a payment system in which country?
A) Japan B) China
C) Singapore D) Russia
Answer : D
✓ India & Russia to explore possibility of accepting RuPay and Mir cards for hassle-free payments.
✓ In the recent meeting of the High Level Inter-Governmental Commission on Trade, Economic, Scientific,
Technological and Cultural Cooperation (IRIGC-TEC), it was discussed and agreed to explore the
possibility of allowing acceptance of these cards.
✓ The mutual acceptance of RuPay (India) and Mir cards (Russia) will help Indian and Russian citizens to
make hassle-free payments in Indian rupees and Russian ruble in their respective countries
✓ The meeting was co-chaired by External Affairs Minister S Jaishankar and Russia Deputy Prime Minister
Denis Manturov.
✓ They agreed to explore the possibility of interaction with the Unified Payments Interface (UPI) of
National Payment Corporation of India and the Faster Payments System (FPS) of the Bank of Russia.

Follow us: Official Site, Telegram, Facebook, Instagram, Instamojo 390


✓ The two countries agreed to adopt the Russian Financial Messaging System, the Service Bureau of the
Bank of Russia's Financial Messaging System, for cross-border payments.
✓ Currently, the overseas payments from India and vice versa are through the SWIFT network
✓ What is Mir?
✓ Mir is a payment system in Russia.
✓ It waslaunched in 2017
✓ The Central Bank of Russia has established the function of this payment option.
✓ The Russian national card payment system operatesit.
✓ Sberbank was the first Russian bank to issue payment cards under the Mir brand
Q. What is the new milestone reached by the Jal Jeevan Mission scheme across the country?
A) 16 Crore B) 15 Crore
C) 10 Crore D) 12 Crore
Answer : D
✓ Under Azadi ka Amrit Kaal, JJM celebrates a new milestone of ensuring safe and clean drinking water
through taps to more than 12 Crore rural households of the country.
✓ About Jal Jeevan Mission (JJM):
✓ Mission Objective: To ensure HarGharJal (piped water supply) to all rural households by 2024 under the
Jal Jeevan Mission.
✓ As on date, 5 States (Goa, Telangana, Haryana, Gujarat and Punjab) and 3 UTs (Puducherry, D&D and
D&NH and A&N Islands), have reported 100% coverage. Himachal Pradesh at 98.35%, followed by
Bihar at 96.05%, are also poised to achieve saturation in the near future.
✓ Working on the motto of Sabka Saath, Sabka Vikas, Sabka Vishwas aur Sabka Prayas, Jal Jeevan Mission
is marching towards achieving the SDG 6, i.e., safe and affordable water to all, through provision of safe
water through taps to all households, schools, anganwadis and other public institutions, in rural areas.
✓ Funding Pattern:
✓ The fund sharing pattern is in the proportion of:
✓ 50:50 between Centre and States,
✓ 90:10 for Himalayan and North-Eastern States.
✓ In case of UTs, 100% funding is provided by the Central government.
Q. Who has been appointed as the Managing Director (MD) and Chief Executive Officer (CEO) of the Bank
of Baroda?
A) Atanu Kumar Das B) R K Chhibber
C) Sunil Mehta D) Debadatta Chand
Answer : D
✓ Debadatta Chand has been appointed as the Managing Director (MD) and Chief Executive Officer
(CEO) of the Bank of Baroda.
✓ He has been appointed for a period of three years with effect from July 2023.
✓ Prior to this, Debadatta served as the Executive Director of the bank.
✓ He succeeds Sanjiv Chadha, whose tenure will end on June 30, 2023.
✓ Bank of Baroda (BOB)
✓ Founded : 1908
✓ Founder : Sayajirao Gaekwad III
✓ Headquarters : Vadodara, Gujarat
✓ MD & CEO : Debadatta Chand
✓ Tagline : India’s International Bank.

Follow us: Official Site, Telegram, Facebook, Instagram, Instamojo 391


Q. Which Union Ministry is associated with the Nutrient Based Subsidy (NBS) scheme?
A) Ministry of Chemicals and Fertilisers
B) Ministry of Road Transport and Highways
C) Ministry of Finance
D) Ministry of Agriculture & Farmers Welfare
Answer : A
✓ Recently, The Union Cabinet has approved the proposal of the Department of Fertilizers for revision in
Nutrient Based Subsidy (NBS) rates for various nutrients for Rabi Season and Kharif Season 2023.
✓ Nutrient Based Subsidy (NBS) rates has been confirmed for various nutrients i.e. Nitrogen (N),
Phosphorus (P),Potash (K) and Sulphur (S) for Rabi Season 2022-23 and for Kharif Season, 2023 for
Phosphatic and Potassic (P&K) fertilizers.
✓ Nutrient Based Subsidy Scheme:
✓ The Subsidy on P&K fertilizers is governed by NBS Scheme under the Department of Fertilizers since
2010.
✓ Subsidy is fixed by an inter-ministerial committee taking into account the benchmark international prices
of finished fertilisers as well as raw materials.
✓ The subsidy is given to registered to P & K fertiliser manufacturers/importers which provides these
fertilisers at subsidised rates to farmers.
✓ Benefits:
✓ It helps farmers in ensuring availability of essential nutrients at subsidized prices.
✓ It rationalizes the subsidy on P&K fertilizers, ensuring effective and efficient utilization of government
resources.
Q. According to RBI, which state has topped the list of states with the highest market borrowings for the
third consecutive year?
A) Tamil Nadu B) Karnataka
C) Maharashtra D) Gujarat
Answer : A
✓ Tamil Nadu tops market borrowing for third consecutive year: RBI
✓ For third year in a row, Tamil Nadu has topped the list of States with the highest market borrowings.
✓ According to RBI data, Tamil Nadu’s gross market borrowings through State Development Loans
(SDLs) stood at ₹68,000 crore during April-February FY23.
✓ Andhra Pradesh is in 2nd place with Rs 51,860 crore. Maharashtra is at 3rd place with Rs.50 thousand
crore among the states that have borrowed the most in the country.
✓ It is noteworthy that Tamil Nadu was the state that borrowed the most in the previous 2 financial years
i.e. 2020-21 and 2021-22.
Q. The headquarter of European Bank for Reconstruction and Development (EBRD) is based at:
A) New York, United States B) London, United Kingdom
C) Helsinki, Finland D) Metro Manila, Philippines
Answer : B
✓ The European Bank for Reconstruction and Development (EBRD) recently pledged further support for
war-torn Ukraine with plans to increase its capital by 3 billion-5 billion euros.

Follow us: Official Site, Telegram, Facebook, Instagram, Instamojo 392


✓ About European Bank for Reconstruction and Development (EBRD):
✓ It is an international financial institution with the mandate to promote transition towards a sustainable,
open market economy and to foster innovation.
✓ The EBRD was created in April 1991 to help Eastern European and ex-Soviet countries transitioning
into democracies by developing free-market economies after the fall of communism.
✓ Headquarters: London
✓ It provides project financing mainly for private enterprises, usually together with other commercial
lending partners, in countries that are committed to, and apply, democratic principles.
✓ It also works with public partners to support privatisation, restructuring and improvement of municipal
services.
✓ It prioritizes support for countries in Central and West Asia, and North Africa.
Q. Which Ministry has launched the scheme, “Vivad se Vishwas I – Relief to MSMEs” for providing relief
to MSMEs for COVID-19 period?
A) Ministry of Finance B) Ministry of Home affairs
C) Ministry of Corporate Affairs D) Ministry of MSME
Answer : A
✓ The Department of Expenditure, Ministry of Finance, has launched the scheme, “Vivad se Vishwas I –
Relief to MSMEs” for providing relief to Micro, Small and Medium Enterprises (MSMEs) for COVID-
19 period.
✓ The scheme was announced in the Union Budget 2023-24 by Union Finance Minister Nirmala
Sitharaman.
✓ The relief provided under this scheme is in continuation to the efforts of the government in promoting
and sustaining the MSME sector.
✓ Under the scheme, ministries have been asked to refund performance security, bid security and liquidated
damages forfeited/ deducted during the Covid pandemic.
✓ Additional benefits to eligible MSMEs, affected during the COVID-19 period.
✓ 95% of the performance security forfeited shall be refunded.
✓ 95% of the Bid security shall be refunded.
✓ 95% of the Liquidated Damages (LD) deducted shall be refunded.
✓ 95% of the Risk Purchase amount realized shall be refunded.
✓ Eligibility Criterion
✓ Registered as a Medium, Small or Micro Enterprise as per relevant scheme of Ministry of MSME on the
date of claim by supplier/ contractor.
✓ MSME could be registered for any category of Goods and Services
Q. The ‘STARS’ project, which has been approved by the Union Cabinet, is partially funded by which
financial institution?
A) World Bank B) Reserve Bank of India
C) International Monetary Fund D) Asian Development Bank
Answer : A
✓ Recently, Ministry of Education and World Bank organised a one of its kind workshop on School-to-
Work Transition under the STARS Program in Mumbai.
✓ School-to-Work Transition workshop under the STARS Program was co-chaired by Secretaries of School
Education and Skill Development and Entrepreneurship.
✓ STARS Program:
✓ Strengthening Teaching-Learning and Results for States (STARS) Project was approved by the Cabinet
in October 2020 and to be implemented as a centrally sponsored scheme.

Follow us: Official Site, Telegram, Facebook, Instagram, Instamojo 393


✓ It became effective since 2021 for a period of five yearse. up to FY: 2024-25.
✓ STARS project would be implemented under Department of School Education and Literacy, Ministry of
Education with the financial support of World Bank.
✓ It is being implemented in six identified States Himachal Pradesh, Maharashtra, Odisha, Rajasthan,
Madhya Pradesh and Kerala. These states will be supported for various interventions for improving the
quality of education.
✓ It is carved out of Samagra Shiksha, with a focus on those elements of the scheme that will most directly
support school education enhancement.
✓ The overall focus and components of the STARS project are aligned with the objectives of National
Education Policy (NEP) 2020 of Quality Based Learning Outcomes.
✓ The STARS project also aims to focus on initiatives of PM e-Vidya, Foundational Literacy and
Numeracy Mission and National Curricular and Pedagogical Framework for Early Childhood Care and
Education as part of the Atmanirbhar Bharat Abhiyan
Q. Who has been named as the new President of the World Bank to succeed incumbent David Malpass?
A) Surjit Bhalla B) Ajay Banga
C) Suman Bery D) Abhijit Banerjee
Answer : B
✓ The former CEO of Mastercard, Ajay Banga, has been named as the new President of the World Bank
for a five-year term. His term will begin from June 2, 2023.
✓ Ajay Banga Selected 14th President of the World Bank.
✓ The Indian-origin Ajay Banga will succeed incumbent chief David Malpass, who plans to step down on
June 01, 2023.
✓ The 63-year-old Banga is currently serving as vice chairman at equity firm General Atlantic.
✓ Ajay Banga was born in Khadki cantonment of Pune, Maharashtra, India.
Q. ‘Open Network for Digital Commerce’ (ONDC) is the initiative of which government body?
A) CBDT B) CBIC
D) BIS D) DPIIT
Answer : D
✓ The Union government is looking to formally launch the Open Network for Digital Commerce (ONDC)
this year to “democratise e-commerce” and “to provide alternatives to proprietary e-commerce sites”.
✓ About Open Network for Digital Commerce (ONDC):
✓ It is an open-source network set up to enable buyers and sellers to transact with each other irrespective of
the e-commerce platform on which either of them are registered.
✓ It will enable local commerce across segments, such as mobility, grocery, food order and delivery, hotel
booking and travel, among others, to be discovered and engaged by any network-enabled application.
✓ It is an initiative of the Department for Promotion of Industry and Internal Trade (DPIIT) under the
Ministry of Commerce and Industry.
✓ It comprises of buyer-side apps where consumers can place orders, seller-side apps that onboard
merchants and display their listings, and logistics platforms that handle deliveries.

Q. Which bank has launched Digital Onboarding platform ‘Sarathi’ for POS Terminals?
A) Yes Bank B) Axis Bank
C) HDFC Bank D) ICICI Bank
Answer : B

Follow us: Official Site, Telegram, Facebook, Instagram, Instamojo 394


✓ Axis Bank Ltd has launched ‘Sarathi’, a first of its kind digital onboarding platform to enable easy
installation of Electronic Data Capture (EDC) or Point of Sale (POS) for merchants.
✓ Sarathi provides a seamless and hassle-free experience for merchants unlike the traditional onboarding
process that can take several days.

AXIS BANK IN NEWS 2023.


▪ Axis Bank launches Digital Onboarding platform ‘Sarathi’ for POS Terminals.
▪ Axis Bank and India Shelter Finance Corporation ltd announce strategic partnership under the co-lending
model.
▪ Axis Bank completes deal to buy Citibank’s India consumer business.
▪ Axis Bank Partners with OPEN to Launch a Fully Digital Current Account.
▪ AXIS BANK
▪ Founded : 1993
▪ HQ : Mumbai, Maharashtra
▪ Chairman : Rakesh Makhija
▪ MD & CEO : Amitabh Chaudhary
▪ Tagline : Badhti Ka naam Zindagi

Q. Which Union Ministry implements the ‘PM MITRA’ Scheme?


A) Ministry of Textiles B) Ministry of Home Affairs
C) Ministry of Defence D) Ministry of External Affairs
Answer : A
✓ New Mega Textile Park at Dhar district will strengthen Make in India and will create new job
opportunities: PM Modi.
✓ The Parks will come up in Tamil Nadu, Telangana, Gujarat, Karnataka, Madhya Pradesh, Uttar Pradesh
and Maharashtra.
✓ It is under PM Mega Integrated Textile Regions and Apparel (PM MITRA) scheme.
✓ It will offer the opportunity to create an Integrated Textiles Value Chain right from spinning, weaving,
processing/dyeing and printing to garment manufacturing etc. at one location and will reduce the
logistics cost of the Industry.
✓ A Special Purpose Vehicle owned by the Centre and State Government will be set up for each park which
will oversee the implementation of the project.
✓ Funding:
✓ The Central government’s budget outlay for the scheme, which is ₹4,445 crore, is to be spent till 2027-
28.
✓ Special purpose vehicles, with a 51% equity shareholding of the State government and 49% of the Centre,
will be formed for each park.
✓ The State governments will provide the land, be part of the SPV, and give the required clearances.
✓ The Central government will disburse Development Capital Fund of ₹500 crore in two tranches for each
of the seven facilities.
✓ This is for the creation of core and support infrastructure.
✓ It will also give a Competitive Incentive Support of ₹300 crore per park to be provided to the
manufacturing units.
✓ Nodal Ministry: Ministry of Textiles
Q. Who is taking over as Twitter's new CEO?
A) Raj Subramaniam B) Linda Yaccarino
C) Brie Carere D) Edwin Khodabakchian

Follow us: Official Site, Telegram, Facebook, Instagram, Instamojo 395


Answer : B
✓ Former NBCUniversal ad chief Linda Yaccarino named CEO of Twitter.
✓ Linda Yaccarino, former advertising chief at NBCUniversal, has been appointed as the new CEO of
Twitter, according to a tweet by Elon Musk, the current CEO of Twitter.
✓ About Twitter
✓ Twitter was founded in March 2006 by Jack Dorsey, Biz Stone, and Evan Williams as a social media
platform for short, real-time messages called "tweets."
✓ It launched publicly in July 2006 and quickly gained popularity.
✓ In its early years, Twitter became known for its role in political activism and citizen journalism during
events such as the Arab Spring.
✓ In October 20222, Twitter was acquired by Tesla CEO Elon Musk in a deal worth $44 billion.
✓ Best CEOs in the World in 2023
✓ Linda Yaccarino - CEO of Twitter
✓ Adam Mosseri - CEO of Instagram
✓ Neal Mohan - CEO of YouTube
✓ Tim Cook – CEO of Apple
✓ Sundar Pichai – CEO of Google
✓ Satya Nadella – CEO of Microsoft
✓ Mark Zuckerberg – CEO of Meta (Facebook)
✓ Elon Musk – CEO of Tesla
✓ Andy Jassy – CEO of Amazon
✓ Jensen Huang – CEO of Nvidia
✓ Michael Miebach – CEO of Mastercard
✓ Daniel Zhang – CEO of Alibaba
✓ Shantanu Narayen – CEO of Adobe
✓ Arvind Krishnan – CEO of IBM
✓ Lakshman Narasimhan – CEO of Starbucks
✓ Doug McMillon - CEO of Walmart
✓ Indian-origin CEOs leading top companies across the world.
✓ Sundar Pichai – CEO of Google
✓ Satya Nadella – CEO of Microsoft
✓ Neal Mohan - CEO of YouTube
✓ Arvind Krishnan – CEO of IBM
✓ Lakshman Narasimhan – CEO of Starbucks
✓ Shantanu Narayen – CEO of Adobe
Q. ‘NeVA’ which is in news recently, is related to the digitization of which of the following?
A) Legislatures B) Supreme court
C) High court D) Council of States
Answer : A
✓ Ministry of Parliamentary Affairs is set to organize a two-day National Workshop on the National e-
Vidhan Application (NeVA) on 24th & 25th of May, 2023 in New Delhi.
✓ The objective of this move is to encourage the legislatures of all states and union territories to move
towards the NeVA platform and bring in transparency, accountability and responsiveness in their
conduct of house business through the use of technology.
✓ It is the second workshop. The first workshop was organised in September 2018.
✓ What is National e-Vidhan Application?
✓ National e-Vidhan Application (NeVA) is one of the 44 Mission Mode Projects (MMPs) under the
“Digital India Programme” of Government of India.

Follow us: Official Site, Telegram, Facebook, Instagram, Instamojo 396


✓ Aim: to make the functioning of all the State Legislatures paperless by transforming them into ‘Digital
House’.
✓ It is a device neutral and member centric application created to equip them to handle diverse House
Business smartly by putting entire information regarding member contact details, rules of procedure,
notices, bills, starred/unstarred questions and answers, committee reports etc. in their hand held devices/
tablets.
✓ Till now, 21 State legislatures have signed Memorandum of Understanding for implementation of the
NeVA and the project has been sanctioned for 17 Legislatures.
✓ Among them, 9 Legislatures have already become fully digital and are Live on NeVA platform.
✓ Nodal Ministry: Ministry of Parliamentary Affairs.

Q. Which bank has recently launched the ‘100 Days 100 Pays’ campaign in every district of the country?
A) RBI B) SEBI
C) SIDBI D) NPCI
Answer : A
✓ Reserve Bank of India (RBI) has launched the ‘100 Days 100 Pays’ campaign for banks to trace and settle
the top 100 unclaimed deposits of every bank in every district of the country within 100 days.
✓ This measure will complement the ongoing efforts and initiatives by the Reserve Bank to reduce the
quantum of unclaimed deposits in the banking system and return such deposits to their rightful
owners/claimants.
✓ At present, the depositors and beneficiaries of unclaimed bank deposits of 10 years or more have to go
through the website of multiple banks to locate such deposits
✓ What is Unclaimed Deposit?
✓ An unclaimed deposit is one which does not see any activity like the infusion of funds, withdrawal, etc.
from the depositor for 10 years or more, and is deemed as an inactive deposit.
✓ Banks transfer these amounts to the Depositor Education and Awareness (DEA) Fund maintained by
the RBI

Q. Consider the following statements


1. Angel Tax is a tax levied on investments made by external investors in startups.
2. Entire investment is taxed subjected to over 30% tax if it is more than the fair market value (FMV).
Which of the statements given above is/are correct?
A) Only 1 B) Only 2
C) Both 1&2 D) None of these
Answer : A
✓ The Finance Ministry recently exempted investors from 21 countries from the levy of angel tax for non-
resident investment in unlisted Indian startups.
✓ About Angel Tax:
✓ It is a tax levied on the capital raised via the issue of shares by unlisted companies if the share price of
issued shares is seen in excess of the fair market value of the company.
✓ The excess funds raised at prices above fair value is treated as income, on which tax is levied.
✓ Angel tax essentially derives its genesis from section 56(2)(viib) of the Income Tax Act, 1961.
✓ The finance act, 2012 introduced section 56(2)(viib) in the IT act which taxes any investment, received
by any unlisted Indian company, valued above the fair market value by treating it as income.
✓ Rate: It is levied at a rate of 30.9% on net investments in excess of the fair market value.

Follow us: Official Site, Telegram, Facebook, Instagram, Instamojo 397


✓ The aggregate amount of paid-up share capital and share premium of the Startup cannot be more than
₹25 crores.
✓ This amount does not include the money raised from Non-Resident Indians (NRIs), Venture Capital
Firms, and specified companies.
✓ For angel investors, the amount of investment that exceeds the fair market value can be claimed for a
100% tax exemption. However, the investor must have a net worth of ₹2 crores or an income of more
than ₹25 Lakh in the past 3 fiscal years.
✓ During Union Budget 2023, Finance Minister Smt Nirmala Sita Raman proposed to amend the Angel
tax

Q. Who has been appointed as the new President of Paytm?


A) Ajay Sethi B) Manak Gupta
C) Abhinav Vashisht D) Bhavesh Gupta
Answer : D
✓ Paytm-owned One 97 Communications Limited has appointed Bhavesh Gupta as its new President and
Chief Operating Officer.
✓ He will report to Vijay Shekhar Sharma, Managing Director and Chief Executive Officer of Paytm.
✓ Bhavesh has over 25 years of experience, he joined Paytm in 2020.
✓ About Paytm
✓ Paytm is an Indian multinational financial technology company headquartered in Noida.
✓ The company specialises in providing a wide range of digital payment solutions and financial services.
✓ The primary focus of Paytm is to facilitate digital transactions and make payments convenient for the
users.
✓ Paytm provides a comprehensive platform that enables users to pay bills, recharge mobiles, book tickets
and shop online.
✓ Establishment - August 2010
✓ Founder/CEO - Vijay Shekhar Sharma
✓ Parent Organization - One97 Communications

Q. Pradhan Mantri Rashtriya Bal Puraskar (PMRBP) is given to children of which age?
A) 5 to 18 B) 6 to 14
C) 4 to 16 D) 8 to 18
Answer : A
✓ Recently, the Women and Child Development Ministry has invited nominations for Pradhan Mantri
Rashtriya Bal Puraskar.
✓ About Pradhan Mantri Rashtriya Bal Puraskar:
✓ The Government conducts the prestigious Pradhan Mantri Rashtriya Bal Puraskar every year to give due
recognition to children with exceptional abilities, who have achieved extraordinary recognition in the
fields of Bravery, Sports, Social Service, Science & Technology, Environment and others.
✓ Age Limit: From 5 years to 18 years (as on 31st August of respective year).
✓ It is given under two categories-
✓ Bal Shakti Puraskar To be given as recognition to children with exceptional abilities and outstanding
achievement in the fields of innovation, scholastic, sports, arts & culture, social service and bravery.
✓ The awards carry a Cash prize of one lakh rupees, a medal and a certificate

Follow us: Official Site, Telegram, Facebook, Instagram, Instamojo 398


Q. IRDAI has recently constituted a five-member expert committee comprising medical experts and insurers
to advise on matters related to mental health and insurance. Who will be headed by this committee?
A) Pravin Kutumbe B) K Ganesh
C) Deepak Mohanty D) Pratima Murthy
Answer : D
✓ The Insurance Regulatory and Development Authority of India (IRDAI) has recently constituted a five-
member expert committee comprising medical experts and insurers to advise on matters related to mental
health and insurance.
✓ The panel has been constituted for two years to advise on matters related to mental health and insurance.
✓ The committee will provide advice and input on existing insurance coverage and what is offered for
mental illnesses.
✓ It will also provide guidance on terminology and concepts from a medical perspective and aspects related
to mental illness from an insurance perspective.
✓ Pratima Murthy, director of the National Institute of Mental Health and Neurosciences (NIMHANS),
has been made the chairperson of the committee.
✓ About Insurance Regulatory and Development Authority (IRDAI).
✓ IRDAI is the regulatory body responsible for overseeing and regulating the insurance industry in India.
✓ It was set up to promote and regulate the insurance sector in India.
✓ It was established on April 1, 2000, under the provisions of the Insurance Regulatory and Development
Authority Act, 1999.
✓ The primary role of the IRDAI is to regulate and promote the insurance industry in India.
Q. ‘Atal Bhujal Yojana’, which was seen in the news, is implemented by which Union Ministry?
A) Ministry of Tribal Affairs B) Ministry of Jal Shakti
C) Ministry of Mines D) Ministry of Skill Development and Entrepreneurship
Answer : B
✓ Recently, The fourth meeting of the National Level Steering Committee (NLSC) of the Atal Bhujal
Yojana was held in New Delhi.
✓ Atal Bhujal Yojana:
✓ Atal Bhujal Yojana (ATAL JAL) is being implemented as a Central Sector Scheme since April, 2020 in
8220 water stressed Gram Panchayats of 229 administrative blocks/Talukas in 80 districts of seven States
Gujarat, Haryana, Karnataka, Madhya Pradesh, Maharashtra, Rajasthan and Uttar Pradesh.
✓ Implementation Period: for 5 years (2020-21 to 2024-25).
✓ Goal: To demonstrate community-led sustainable ground water management which can be taken to
scale.
✓ Objective: The major objective of the Scheme is to improve the management of groundwater resources
in select water stressed areas in identified states.
✓ Funding: Total outlay of INR 6000 crores. Out of this, INR 3,000 crores will be as loan from the World
Bank and INR 3,000 crores as matching contribution from the Government of India.
✓ Nodal agency: The Department of Water Resources, River Development & Ganga Rejuvenation,
Ministry of Jal Shakti.
✓ The Committee decides Atal Bhujal Yojana should be taken up for continuation for another two years
beyond 2025
Q. Which company has announced an investment of $1 million in the form of grants to the IIT Madras?
A) Google B) Microsoft
C) Meta D) Apple

Follow us: Official Site, Telegram, Facebook, Instagram, Instamojo 399


Answer : A
✓ US technology giant Google announced an investment of $1 million in the form of grants to the Indian
Institute of Technology Madras (IIT Madras).
✓ Google has been announced as the inaugural ‘platinum consortium’ member of the IIT Madras newly
formed Centre for Responsible Artificial Intelligence (CeRAI).
✓ CeRAI’s major goal will be to create high-quality research outputs such as research publications in high-
impact journals or conferences, white papers, and patents, among other things.

Q. ‘REWARD Program’ is assisted by which institution in India?


A) ADB B) AIIB
C) IMF D) World Bank
Answer : D
✓ The Secretary, Department of Land Resources (DoLR), Government of India (GoI) Shri Ajay Tirkey
reviewed the Implementation Support Mission of World Bank Assisted Rejuvenating Watersheds for
Agricultural Resilience through Innovative Development (REWARD) program.
✓ REWARD is a World Bank assisted Watershed development program which is being implemented from
2021 to 2026.
✓ The Development Objectives of REWARD program is to “Strengthen capacities of National and State
institutions to adopt improved watershed management for increasing farmers resilience and support value
chains in selected watersheds of participating States”.
✓ The program is being implemented to introduce modern watershed practices in the Department of Land
Resources in the Ministry of Rural Development and in the States of Karnataka and Odisha.
✓ The total budget outlay of REWARD program is USD 167.71 million over a program period of 4.5 years.
Q. RuPay has introduced the CVV free payment experience for its debit, credit, and prepaid cardholders.
What is the full form of CVV?
A) Card Verified Value B) Card Value Verification
C) Credit Verification Value D) Card Verification Value
Answer : D
✓ RuPay has introduced the CVV (Card Verification Value) free payment experience for its debit, credit
and prepaid cardholders who have tokenized their cards on the merchant application or webpage.
✓ This new CVV less experience ensures that the cardholder will not have to reach out to their wallet or
remember any card details, if they have saved (tokenized) their card on the e-commerce merchant which
supports this feature.
✓ They will just have to enter the OTP or their device will auto-populate the OTP during the domestic e-
commerce transaction and the payment will be done.
✓ Tokenization is a simple technology to secure card transactions without sharing clear or real card details
with the merchants.

Q. Which institution releases the Periodic Labour force survey (PLFS) results in India?
A) NSO B) CBDT
C) CBIC D) BIS
Answer : A
✓ According to the recent periodic labour force survey, the urban unemployment rate dipped further in the
January to March 2023 quarter.

Follow us: Official Site, Telegram, Facebook, Instagram, Instamojo 400


✓ The unemployment rate in current weekly status (CWS) for persons of age 15 years and above in urban
areas declined to 6.8% in the January to March 2023 quarter.
✓ The urban labour force participation rate (LFPR) for all ages, however, inched up to a high of 38.1% in
the fourth quarter of last fiscal from 37.9% in the October to December 2022 period.
✓ The urban LFPR for persons above 15 years of age was even higher at 48.5% in the fourth quarter of
FY23.
✓ About Periodic Labour force Survey (PLFS).
✓ Considering the importance of availability of labour force data at more frequent time intervals, National
Statistical Office (NSO) launched Periodic Labour Force Survey in April 2017.
✓ The objective of PLFS is:
✓ To estimate the key employment and unemployment indicators (viz. Worker Population Ratio, Labour
Force Participation Rate, Unemployment Rate) in the short time interval of three months for the urban
areas only in the ‘Current Weekly Status’ (CWS).
✓ To estimate employment and unemployment indicators in both ‘Usual Status’ (ps+ss) and CWS in both
rural and urban areas annually.
✓ It is conducted by the National Sample Survey (NSO), working under Ministry of statistics and
programme implementation (MoSPI).

Q. UNEP suggests measures to reduce 80% of world's plastic pollution by which year?
A) 2025 B) 2030
C) 2040 D) 2050
Answer : C
✓ Plastic pollution could be slashed by 80 per cent by 2040, according to a new report by the UN
Environment Programme (UNEP).
✓ About 400 million tonnes of plastic waste are generated every year.
✓ This ambitious target relies on major policy changes and the deployment of existing technologies in the
way we produce, use and dispose of plastics.
✓ Sustainable alternatives could reduce pollution by 17 per cent by 2040.
✓ United Nations Environment Programme (UNEP)
✓ Formation : 5 June 1972
✓ Headquarters : Nairobi, Kenya
✓ Executive Director : Inger Andersen (Denmark).
Q. Sakhalin-1 project, sometimes seen in news, is situated in which country?
A) UAE B) Qatar
C) Russia D) Saudi Arabia
Answer : C
✓ Oil and Natural Gas Corporation’s subsidiary, ONGC Videsh, recently said that oil output from the
Sakhalin-1 project in Russia had bounced back to its plateau level of about 200,000 barrels per day (bpd)
from zero.
✓ About Sakhalin-1 Project:
✓ Location: The Sakhalin-1 oil and gas development project is located in the Okhotsk Sea, off the north-
eastern coast of Sakhalin Island in the Russian Far East.
✓ It comprises three offshore fields namely Chayvo, Odoptu, and Arkutun-Dagi.
✓ It is operated by Exxon Neftegas (ENL), a subsidiary of Exxon Mobil which holds a 30% participating
interest. The other partners of the project are the Japanese consortium Sakhalin Oil and Gas
Development Company (SODECO, 30%), India’s state-owned ONGC Videsh (20%), and Russian oil
company Rosneft (20%).

Follow us: Official Site, Telegram, Facebook, Instagram, Instamojo 401


✓ The Sakhalin 1 project is estimated to hold approximately 2.3 billion barrels of oil and 17.1 trillion cubic
feet (Tcf) in recoverable reserves.
✓ ONGC Videsh Ltd (OVL):
✓ It is a Miniratna Schedule “A” Central Public Sector Enterprise (CPSE)of the Government of India and
is the wholly owned subsidiary and overseas arm of Oil and Natural Gas Corporation Limited (ONGC).
✓ It comes under the administrative control of the Ministry of Petroleum and Natural Gas.
Q. Defence production beats _ crore mark on back of crucial reforms.
A) ₹1 lakh crore B) ₹2 lakh crores
C) ₹3 lakh crores D) ₹4 lakh crores
Answer : A
✓ Defence production crosses Rs 1 lakh crore mark for the first time ever.
✓ The value of defense production in the financial year (FY) 2022-23 has crossed the Rs one lakh crore
mark for the first time.
✓ The value is currently Rs 1,06,800 crore and may increase further once data is received from the
remaining private defense industries.
✓ The present value of defense production in FY 2022-23 is over 12 per cent higher than Rs 95,000 crore in
FY 2021-22.
✓ Government's efforts to boost defense production
✓ Simplified defence industrial licensing, relaxation of export controls and grant of No Objection
Certificate.
✓ Specific incentives introduced under the foreign trade policy.
✓ Defence Production & Export Promotion Policy 2020.
✓ The government had issued two "positive indigenization lists" which included 209 items that could not
be imported.
✓ The government has also announced two dedicated corridors in Tamil Nadu and Uttar Pradesh to act as
clusters of defense manufacturing.
✓ The number of defense licenses issued by the government to industries has increased by almost 200 per
cent in the last 7-8 years.
✓ The vision of the government
✓ To achieve a turnover of $25 billion, including exports of $5 billion in aerospace and defence goods and
services by 2025
Q. Who has approved the proposed merger of Credit Suisse Group AG with UBS Group AG?
A) Reserve Bank of India (RBI)
B) Competition Commission of India (CCI)
C) Securities and Exchange Board of India (SEBI)
D) Confederation of Indian Industry (CII)
Answer : B
✓ Competition Commission of India (CCI) approves merger of Credit Suisse Group with UBS Group.
✓ UBS Group AG (UBS) is a multinational investment bank and financial services company
✓ It is a Switzerland based company and is active globally.
✓ UBS's businesses include wealth management, asset management, investment banking services, and retail
and corporate banking.
✓ In India, UBS's business is primarily focused on brokerage services.
✓ About Credit Suisse Group AG (Credit Suisse)

Follow us: Official Site, Telegram, Facebook, Instagram, Instamojo 402


✓ Credit Suisse Group AG (Credit Suisse) is a multinational investment bank and financial services
company established in Switzerland.
✓ In India, Credit Suisse's businesses include wealth management and investment banking services.
✓ Competition Commission of India (CCI)
✓ It is the chief national competition regulator in India.
✓ It was set up under the Competition Act, 2002.
✓ It comes under the Ministry of Corporate Affairs.
✓ It promotes healthy market competition and prevents activities that have an adverse effect on competition
in India.
✓ CCI also approves merger and acquisitions of companies in India so that the two merging entities do not
dominate the market unfairly.
✓ Headquarter - New Delhi
✓ Chairman : Ravneet Kaur
Q. Amazon Web Services has announced how much investment in India's cloud infrastructure by 2030?
A) $12.7 billion B) $13.7 billion
C) $14.7 billion D) $15.7 billion
Answer : A
✓ With an aim to meet the growing demand for cloud services, Amazon Web Services will invest $12.7
billion in India by 2030. Which is expected to contribute $ 23.3 billion to India's economy.
✓ Punit Chandhok, President, Commercial Business, AWS India and South Asia, reiterated AWS's
commitment to driving economic transformation in India
Q. Which entity has introduced a risk disclosure framework for individual traders for trading in the equity
Futures & Options (F&O) segment?
A) SEBI B) RBI
C) IRDAI D) SIDBI
Answer : A
✓ SEBI Introduced Risk disclosure Framework for Trading by Individual Traders in Equity F&O Segment..
✓ Securities and Exchange Board of India (SEBI) introduced a risk disclosure framework for individual
traders for trading in the equity Futures & Options (F&O) segment.
✓ To empower the traders with detailed information about the risks associated with trading in derivatives
and also to facilitate informed decision-making by the investors.
✓ The risk disclosure framework would come into force with effect from July 01, 2023.
Q. According to the World Bank's annual "State and Trends of Carbon Pricing" report, revenues from
carbon taxes and ETS reach approximately how many billions?
A) $73 Billion B) $81 Billion
C) $90 Billion D) $95 Billion
Answer : D
✓ According to the World Bank’s (WB) annual report ‘State and Trends of Carbon Pricing 2023’ released
on May 23, 2023, revenues collected from carbon taxes and Emissions Trading Systems (ETS) globally
have reached a record high, about US$95 billion in 2022 despite the challenging context for governments
facing high inflation, fiscal pressures, and energy crises.
✓ The revenues collected from carbon taxes and ETS in 2021 was about US$ 84 billion

Follow us: Official Site, Telegram, Facebook, Instagram, Instamojo 403


✓ As per the report, there are currently 73 global carbon pricing instruments are in operation, compared
with 68 in May 2022 when the World Bank issued its 2022 report.
✓ The 73 Carbon Pricing initiatives covered 11.66 GtCO2e (gigatonnes of carbon dioxide equivalent),
representing 23% of global greenhouse gas (GHG) emissions.
✓ As of April 1, 2023 less than 5% of global GHG emissions are covered by a direct carbon price at or above
the range recommended by 2030.
Q. With which company has BSNL joined hands to set up 4G network in India?
A) Reliance Communication B) Tech Mahindra
C) TCS D) Airtel
Answer : C
✓ BSNL has placed an order worth ₹15,000 crore to tech firm Tata Consultancy Services (TCS) to further
expand the 4G network in India.
✓ The deal includes Tejas Networks, a Tata Group telecom gear maker, which will provide Radio Access
Network (RAN) equipment.
✓ The TCS-led consortium includes Tejas Networks and C-DOT.
Q. How many million loan agreement has been signed by the Government of India with the Asian
Development Bank for infrastructure development in Andhra Pradesh?
A) USD 105.50 Million B) USD 122.75 Million
C) USD 141.12 Million D) USD 167.37 Million
Answer : C
✓ The Asian Development Bank (ADB) and the Government of India signed a $141.12 million loan
agreement for the development of an industrial corridor in Andhra Pradesh.
✓ The agreement has been signed to support the development of high quality indoor infrastructure facilities
in three industrial areas in Andhra Pradesh. These are: roads, water supply plants and power distribution
systems.
✓ The loan is the second tranche of a $500 million Multi-tranche Financing Facility (MFF) under the
program approved by ADB in 2016.
✓ With this loan facility, infrastructure facilities will be developed in three industrial clusters under the
Visakhapatnam and Srikalahasti-Chittoor areas sanctioned in Andhra Pradesh
Q. CBDT has raised the tax exemption Exemption for leave encashment raised to______ lakhs, effective
from 1 April 2023.
A) ₹15 lakhs B) ₹20 lakhs
C) ₹25 lakhs D) ₹30 lakhs
Answer : C
✓ Central Board of Direct Taxes (CBDT) has raised the tax exemption on leave encashment on retirement
of non-government salaried employees from ₹3 lakh to ₹25 lakh.
✓ The relief, which was announced in the union budget speech this year (2023), is effective from 1 April
2023.
✓ CBDT provides essential inputs for policy and planning of direct taxes in India and is also responsible
for administration of direct tax laws through the Income Tax Department.
✓ It is a statutory authority functioning under the Central Board of Revenue Act, 1963

Follow us: Official Site, Telegram, Facebook, Instagram, Instamojo 404


✓ Central Board of Direct Taxes (CBDT)
✓ Founded : 1860
✓ Headquarteres : New Delhi
✓ Chairman : Nitin Gupta
✓ Minister responsible : Ministry of Finance
Q. The National Tiger Conservation Authority (NTCA) has constituted an 11-member Cheetah Project
Steering Committee. Who will head this committee?
A) Abhijit Sen B) Bimal Jalan
C) Chandra Shekhar D) Rajesh Gopal
Answer : D
✓ The National Tiger Conservation Authority (NTCA) set up an 11-member Cheetah Project Steering
Committee.
✓ The committee works under the Ministry of Environment, Forest and Climate Change.
✓ Rajesh Gopal, general secretary of the Global Tiger Forum, has been appointed as the chairman of the
committee.
✓ The decision to form the committee was taken after the death of six cheetahs involved in the translocation
project.
✓ The objective of the committee is to address the challenges and issues related to the Cheetah
Translocation Project.
✓ The committee includes 10 other members with diverse backgrounds and expertise.
✓ National Tiger Conservation Authority (NTCA):
✓ It was established in December 2005.
✓ The establishment of NTCA was based on the recommendation made by the Tiger Task Force.
Q. Srikrishnan Hari Hara Sarma has been appointed as Managing Director and Chief Executive Officer
(MD & CEO) of which bank?
A) Karnataka Bank
B) Kotak Mahindra Bank
C) HDFC Bank
D) ICICI Bank
Answer : A
✓ Srikrishnan Hari Hara Sarma appoints as MD & CEO of Karnataka Bank
✓ Private sector lender Karnataka Bank has appointed Srikrishnan Harihara Sarma as the new managing
director and chief executive officer of the bank for three years.
✓ The appointment is subject to the approval of shareholders at the ensuing AGM or within a period of 3
months from the date of assuming charge whichever is earlier.
✓ He has also worked at HDFC Bank Limited, Yes Bank, and Jio Payments Bank
✓ KARNATAKA BANK
✓ Founded : 1924
✓ Headquarters : Mangaluru, Karnataka
✓ Chairmam : Pradeep Kumar Panja
✓ MD & CEO : Srikrishnan Hari Hara Sarma
✓ Tagline : Your Family Bank Across India

Follow us: Official Site, Telegram, Facebook, Instagram, Instamojo 405


Q. Union government has extended the Atal Bhujal Yojana (Atal Jal) for two more years until December
2025. Atal Bhujal Yojana (Atal Jal) is being implemented in 8,565 gram panchayats in 80 districts of how
many states?
A) 5 B) 7
C) 6 D) 4
Answer : B
✓ Union government has extended the Atal Bhujal Yojana (Atal Jal) for two more years until December
2025.
✓ Initially launched in 2019, Atal Jal is active across 8,220 water-stressed Gram Panchayats within 80
districts of seven Indian states, including Gujarat and Uttar Pradesh.
✓ The scheme is active across 8,220 water-stressed Gram Panchayats within 80 districts of seven Indian
states.namely, Gujarat, Haryana, Karnataka, Madhya Pradesh, Maharashtra, Rajasthan and Uttar
Pradesh .
✓ Atal Jal plans to bring 450,000 hectares of irrigated area under efficient water techniques like drip
irrigation and crop diversification..
✓ About Atal Bhujal Yojana.
✓ Atal Bhujal Yojana is a groundwater management scheme launched by Prime Minister Narendra Modi
on the 95th birth anniversary of former Prime Minister Atal Bihari Vajpayee, on 25 December 2019.
✓ The purpose of the scheme is to improve groundwater management in seven states of India.
✓ In order to acknowledge and contain the ever-growing problem of groundwater depletion, the Indian
Government introduced the Atal Bhujal Yojana (ABY) in December 2019 after getting financial approval
from the World Bank in 2018.
✓ It has been launched under the Jal Jeevan Mission of the government.
Q. SEBI has formed an Intermediary Advisory Committee to advise the market regulator on changes in the
legal framework. Who will be chaired this committee?
A) Amarjeet Singh B) Ruchi Chojer
C) Pramod Rao D) S Ravindran
Answer : D
✓ The Securities and Exchange Board of India (Sebi) has constituted an Intermediary Advisory Committee
to advise the markets regulator on changes in legal framework and increasing transparency in the
functioning of market intermediaries like stock exchanges, depository participants, clearing corporations,
among others.
✓ The committee will be chaired by former Sebi executive director S Ravindran who in his assignment with
the markets regulator handled the charge of Corporation Finance Department.
✓ He had also headed departments related to investment management, and foreign institutional investors
and custodians.
Q. Which company has launched a new generative AI-driven chatbot called ‘Jugalbandi’, designed for
farmers and other users living in rural India?
A) Google B) Microsoft
C) Amazon D) IBM
Answer : B
✓ Microsoft has unveiled a new generative AI-driven chatbot called ‘Jugalbandi’, designed for farmers and
other users living in rural India.

Follow us: Official Site, Telegram, Facebook, Instagram, Instamojo 406


✓ The chatbot is being developed in collaboration with Microsoft Research and the government-backed
AI4Bharat – an open-source language AI centre based at the Indian Institute of Technology (IIT) Madras
– and OpenNyAI.
Q. Who has been appointed as the Managing Director (MD) and Chief Executive Officer (CEO) of the Bank
of Baroda?
A) Atanu Kumar Das B) R K Chhibber
C) Sunil Mehta D) Debadatta Chand
Answer : D
✓ Debadatta Chand has been appointed as the Managing Director (MD) and Chief Executive Officer
(CEO) of the Bank of Baroda.
✓ He has been appointed for a period of three years with effect from July 2023.
✓ Prior to this, Debadatta served as the Executive Director of the bank.
✓ He succeeds Sanjiv Chadha, whose tenure will end on June 30, 2023.
✓ Bank of Baroda (BOB)
✓ Founded : 1908
✓ Founder : Sayajirao Gaekwad III
✓ Headquarters : Vadodara, Gujarat
✓ MD & CEO : Debadatta Chand
✓ Tagline : India’s International Bank.

Q. Who has released the "Best Practices in Social Sector: A Compendium, 2023"?
A) RBI B) FICCI
C) NITI Aayog D) FSSAI
Answer : C
✓ NITI Aayog has released the “Best Practices in Social Sector: A Compendium, 2023” in collaboration
with the United Nations Development Programme.
✓ Reason : To commemorate 75 years of India’s independence and highlight and laud the efforts of Union
Ministries and State Governments, this compendium includes 75 case studies cutting across 14 key social
sectors.
✓ The case studies have been sourced from all States and Union Territories and 30 Ministries and
Departments of the Government of India. The aim of this exercise has been to synthesise lessons for the
future to expand, enhance and improve life at the grassroots level.
Q. What is the interest rate on deposits made under ‘Mahila Samman Saving Certificate Account’?
A) 7.5 % B) 6.5 %
C) 7.0 % D) 8.5 %
Answer : A
✓ Finance Minister recently announced a new saving scheme ‘Mahila Samman Saving Certificate’ for
women and girls in the Union Budget.
✓ About Mahila Samman Saving Certificate’ Scheme:
✓ The scheme offers deposit facility up to Rs 2 lakh in the name of women or girls for a tenor of 2 years.
✓ It offers fixed interest rate of 7.5 per cent.
✓ There are no tax benefits, but partial withdrawal is allowedin this scheme.
✓ This is a one-time scheme announced in Budget 2023 and will remain available for a two-year period i.e.
up to March 2025.

Follow us: Official Site, Telegram, Facebook, Instagram, Instamojo 407


✓ Benefit: It will encourage more women to adopt formal financial saving instruments.
✓ Bank of India becomes first bank to offer Mahila Samman Savings Certificate.
✓ According to a statement by the bank, while the minimum investment is Rs 1,000, any sum in multiples
of Rs 100 can be deposited, up to a maximum limit of Rs 2 lakh.
✓ The account will mature after two years from the date of opening. Accounts under this scheme can be
opened until March 31, 2025
Q. Which Artificial Intelligence Company launched ‘Pi Chatbot’?
A) DeepMind Technologies B) Inflection AI Inc
C) OpenAI D) Tesla
Answer : B
✓ Inflection AI Company unveiled a modern talkbot named ‘Pi Chatbot’.
✓ The full form of Pi is Personal Intelligence which is different in its activities.
✓ The CEO of this 1-year-ago startup, Mustafa Suleyman announced that this agent is capable of solving
real-life problems of humans rather than coding complex terms and writing homework like other AI-
based systems.
✓ This has been launched collaboratively by the co-founders of Google DeepMind and LinkedIn
Q. Which bank has introduced a digital distribution platform Smart Saathi to connect business
correspondents (BCs) and business facilitators (BFs) to the bank?
A) Yes Bank B) HDFC Bank
C) ICICI Bank D) Axis Bank
Answer : B
✓ HDFC Bank has introduced a digital distribution platform HDFC Bank Smart Saathi to connect business
correspondents (BCs) and business facilitators (BFs) to the bank.
✓ HDFC Bank Smart Saathi will contribute by making banking products and services available, especially
credit for rural customers, which will help in the development of the country.
✓ This will help in enhanced income generation for the agents and also help customers to access easy credit.
✓ The platform will also boost capacity building for agents and further financial literacy for customers.
Q. Who is the highest paid athlete as per the latest Forbes’ highest-paid athletes list in 2023?
A) Lionel Messi
B) Cristiano Ronaldo
C) Canelo Álvarez
D) LeBron James
Answer : B
✓ According to the latest Forbes’ highest-paid athletes list in 2023, Portuguese football star Cristiano
Ronaldo is the world’s highest-paid athlete, with earning of $136 million.
✓ The report states that world’s ten highest-paid athletes collectively made $1.11 billion before taxes and
agents’ fees over the last 12 months, the highest total ever.
✓ With an earning of $130 million, Argentine professional footballer Lionel Messi is second highest paid
athlete.
✓ French professional footballer, Kylian Mbappé is third with total earning estimated at $120 million

Follow us: Official Site, Telegram, Facebook, Instagram, Instamojo 408


Q. The Maitree Super Thermal Power Plant (MSTPP) is NTPC LTD’s first overseas capacity addition. The
plant is located in which country?
A) Malaysia B) Nepal
C) Bangladesh D) Sri Lanka
Answer : C
✓ NTPC Group’s total installed capacity reaches 72,304 MW with first overseas capacity addition in
Bangladesh.
✓ The state-owned NTPC Ltd has marked its first overseas capacity addition with the installation of Unit-
1 of project named Maitree Super Thermal Power Plant (MSTPP) in Rampal, Mongla, Bagerhat,
Bangladesh.
✓ The project has two units of 660 MW capacity each (1320 MW (2×660)).
✓ With the new addition, thbe total Installed Capacity of NTPC Group has reached to 72304 MW.
✓ The MSTPP project was executed in collaboration with the Bangladesh-India Friendship Power
Company Pvt Ltd. (BIFPCL), a foreign joint venture company of NTPC Ltd.
Q. Recently which company got Telecom license in India?
A) Zoom B) Meta
C) Tesla D) RailTel
Answer : A
✓ Zoom Video Communications, the parent company of the video conferencing platform 'Zoom', has
received a nationwide telecom license from the Indian Telecom Department.
✓ Under this, the company will now provide telephone service to enterprise customers in the country, as
well as the company will also provide its cloud based private branch exchange service (Zoom Phone)
service to enterprise customers.
✓ Zoom is an American company based on video conferencing.
Q. India’s domestic coal production has risen by how much percent during the last five years?
A) 22% B) 25%
C) 16% D) 31%
Answer : A
✓ India’s domestic coal production rises by over 22 % during last 5 years.
✓ The country’s overall coal production has seen a quantum jump to 893.08 million tonnes in the Financial
Year 2022-23 as compared to 728.72 million tonnes in the Financial Year 2018-2019 with a growth of
about 22.6 percent.
✓ The priority of the Ministry is to enhance domestic coal production to reduce the dependence on
substitutable coal imports.
✓ The production of Coal India Limited (CIL) has increased by 703.21 million tonnes as compared to
606.89 million tonnes in the Financial Year 2018-2019 with a growth of 15.9 percent in the last five years
Q. SEBI has introduced the Legal Entity Identifier (LEI) system for issuers of non-convertible securities,
securitized debt instruments, and security receipts. How many digits are there in Legal Entity Identifier
(LEI)?
A) 16 B) 15
C) 21 D) 20

Follow us: Official Site, Telegram, Facebook, Instagram, Instamojo 409


Answer : D
✓ SEBI introduced the Legal Entity Identifier (LEI) system for issuers of non-convertible securities,
securitized debt instruments and security receipts.
✓ The Legal Entity Identifier (LEI) is a 20-digit number used to uniquely identify parties to financial
transactions worldwide to improve the quality and accuracy of financial data systems.
✓ LEI is a unique global identifier for legal entities participating in financial transactions.
✓ The system is primarily intended for issuers that have listed or plan to list non-convertible securities,
securitized debt instruments and security receipts.
✓ It aims to create a global reference data system that identifies each legal entity that is party to a financial
transaction.
✓ The LEI system will enable greater transparency and better risk management in the financial markets.
✓ Companies planning to list non-convertible securities, securitized debt instruments, and security receipts
will need to obtain an LEI from an authorised issuer.
✓ The LEI system will also help prevent fraud and financial crimes by properly identifying and verifying
legal entities participating in financial transactions.
✓ Entity Identifier India Limited is a subsidiary of Clearing Corporation of India Limited.
✓ The Reserve Bank of India (RBI) mandates non-individual borrowers having an aggregate exposure of
above Rs 25 crore to obtain the LEI code.
Q. Who announced the introduction of CVV-free online transactions for domestic tokenised credentials in
India?
A) Mastercard B) Capital One
C) Visa D) American Express
Answer : C
✓ Digital payment solution provider Visa on May 5, 2023 announced the introduction of CVV-free online
transactions for domestic tokenised credentials in India. The move is in accordance with Reserve Bank
of India’s (RBI) guidelines on tokenisation, Visa said in a press release.
✓ Tokenisation is a process that replaces a card’s 16-digit number with a unique alternate card number or
‘Token’, which can be used for online transactions, mobile point-of-sale (POS) transactions, or in-app
transactions.

Q. Which has rebranded its multi-brand loyalty program, Payback India to Zillion?
A) PhonePe B) Zerodha
C) Pine Labs D) BharatPe
Answer : D
✓ BharatPe Group, has announced that it will be rebranding Payback India, multi-brand loyalty program
to Zillion.
✓ This new brand identity is in line with the company’s vision to make Zillion ubiquitous to loyalty and
rewards across the country.
✓ Aim : The new brand identity aims to target customers across age groups and add a new dimension to
their overall shopping experience, across categories and brands.
✓ Zillion will offer a wide range of options for customers to earn and redeem at a wide range of partners
across the country. The customers will be able to earn ‘Zillion coins’ for their routine spends, that will in
turn add a moment of delight in their life.
✓ BharatPe becomes ICC’s official partner till 2023.
✓ BharatPe
✓ Founded : 2018
✓ Headquarteres : New Delhi

Follow us: Official Site, Telegram, Facebook, Instagram, Instamojo 410


✓ Founded / CEO : Ashneer Grover
✓ Developed By : NPCI
✓ Chairmam : Rajnish Kumar

Q. According to Everest Annual ITS rankings 2023, which company has topped the list?
A) TCS B) Accenture
C) Wipro D) HCL
Answer : B
✓ Accenture takes the number one spot for the seventh straight year in the Everest 2023 PEAK Matrix
Service Provider of the Year Awards for Information Technology (IT) services.
✓ The ranking of Tata Consultancy Services (TCS), Capgemini, and Wipro has risen to take the number 2,
3, and 4 spots, respectively. HCL Tech ranks fifth.
✓ Since Everest Group, a global IT research firm, launched the rankings in 2016, TCS has been in the top
five, and this year it climbed to second place. Capgemini and Wipro each rose three positions in the
rankings over last year.

Q. Which company has replaced BharatPe as the global sponsor of the International Cricket Council (ICC)?
A) MasterCard B) Paytm
C) Tata Neu D) Dream11
Answer : A
✓ American multinational financial services corporation, Mastercard has replaced BharatPe as the global
sponsor of the International Cricket Council (ICC).
✓ The sponsorship pact between ICC & Mastercard is a multi-year arrangement and will cover the
upcoming 4 years.
✓ For the past year, Mastercard has been aiming to bag some lucrative sponsorship deals as it also secured
title sponsorship rights of the Board of Control for Cricket in India (BCCI) international and domestic
home fixtures from Paytm.
✓ ICC primarily helms a three-year sponsorship and its partnership with BharatPe was kicked off on June
2021.
Q. Who launched “Open Challenge Program (OCP) 2.0” as part of a series of OCP programs by Kalpataru-
CoE (Center for Entrepreneurship)?
A) Indian Oil Corporation Limited B) Bharat Heavy Electricals Limited
C) NTPC Limited D) Rashtriya Ispat Nigam Limited (RINL)
Answer : D
✓ Rashtriya Ispat Nigam Limited (RINL) launched “Open Challenge Program (OCP) 2.0” as part of a
series of OCP programs by Kalpataru-CoE (Center for Entrepreneurship).
✓ The objective of this program is to select startups specialising in Industry 4.0 technologies for
collaboration with RINL.
✓ The startups' technologies are expected to provide faster return on investment for RINL, contribute to
energy savings and enhance process safety.
✓ The "Center of Excellence (CoE) on Industry 4.0" project involves collaboration between STPI (Software
Technology Parks of India), MeitY (Ministry of Electronics & IT), RINL and Government of Andhra
Pradesh.
✓ Over a period of 5 years, the CoE plans to incubate around 175 startup companies, of which 50 are
incubated in physical mode and 125 are incubated in virtual mode.

Follow us: Official Site, Telegram, Facebook, Instagram, Instamojo 411


✓ Rashtriya Ispat Nigam Limited (RINL)
✓ It is a government body that oversees the operations of the Visakhapatnam Steel Plant (VSP), India's first
shore-based integrated steel plant.
✓ The VSP plant was commissioned in 1992 and initially had a capacityto produce 3.0 MTPA of liquid
steel.
✓ Establishment -18 February 1982
✓ Headquarters - Visakhapatnam, India
✓ CMD - Atul Bhatt
Q. Which entity has announced its participation in the Global Financial Innovation Network (GFIN)
greenwashing techsprint?
A) SEBI B) RBI
C) NPCI D) SIDBI
Answer : B
✓ RBI has announced its participation in the Global Financial Innovation Network (GFIN) greenwashing
techsprint.
✓ The Reserve Bank of India (RBI) has invited firms to participate in the Greenwashing TechSprint to
develop technology solutions to detect incidents of greenwashing in financial products.
✓ TechSprint is part of RBI's efforts to promote sustainable finance and ensure that financial products
marketed as green or sustainable are actually aligned with environmental objectives.
✓ TechSprint is open to firms and individuals who can provide innovative technology solutions to detect
greenwashing in financial products using technologies such as natural language processing and artificial
intelligence.
✓ About Global Financial Innovation Network (GFIN).
✓ This is an international network of financial regulators that seeks to foster innovation in the financial
services sector.
✓ It was established in 2018 by a group of 12 financial regulators, including the UK Financial Conduct
Authority, the Monetary Authority of Singapore, and the US Consumer Financial Protection Bureau.
✓ It aims to provide a platform for regulators to collaborate and share best practices on innovative financial
products and services, as well as regulatory approaches to innovation
Q. Export lender Exim Bank to raise up to record _ billion in FY 2023-24 (FY24) for Trade Finance and
Term Loans.
A) $2.7 billion B) $3 billion
C) $3.3 billion D) $4 billion
Answer : D
✓ Export lender Exim Bank plans to raise up to record $4 billion in financial year 2023-24 (FY24) for
extending trade finance and term loans.
✓ Exim Bank raised $1 billion through sustainability bond in Jan-uary 2023, under its environm-ent social
governa-nce (ESG) fra-me-work. It later raised $100 million via a second offering of the bond.
Q. European Union regulators approve Microsoft's _ billion acquisition of Activision Blizzard.
A) $50 Billion B) $54 Billion
C) $63 Billion D) $69 Billion
Answer : D

Follow us: Official Site, Telegram, Facebook, Instagram, Instamojo 412


✓ The European Union approved Microsoft’s $69 billion purchase of video game maker Activision
Blizzard, deciding the deal won’t stifle competition for popular console titles like Call of Duty and
accepting the US tech company’s remedies to boost competition in cloud gaming.
✓ The blockbuster deal is still in jeopardy because British regulators have rejected it and U.S. authorities
are trying to thwart it.
Q. Sarbananda Sonowal has informed that the government will provide financial support of____ for the
promotion of Green Shipping.
A) 50% B) 40%
C) 30% D) 20%
Answer : C
✓ Union Minister for Ports, Shipping and Waterways, Sarbananda Sonowal has announced 30 per cent
financial assistance to promote green shipping.
✓ Ports, Shipping and Waterways Minister Sarbananda Sonowal made five important announcements
focusing on green shipping and digitization of ports at the end of the second Chintan Shivir of the ministry
held in Munnar, Kerala.
✓ Green Hydrogen hubs will be developed at the Deendayal Port at Kandla and Chidambaranar Port at
Tuticorin.
✓ Two tugs each will be procured for Jawaharlal Nehru, VO Chidambaranar, Paradip and Deendayal ports
under the Green Tug Transition Programme.
✓ Jawaharlal Nehru and Turicorin ports will be made smart ports by next year.
✓ What is Green Shipping?
✓ Green shipping is when people or goods are transported via ships using the minimum resources and
energy possible to protect the environment from pollutants generated by the ships.
✓ Green shipping promotes clean practices by implementing emission control, efficient port management
and equipment management

Q. Which Ministry has set up the Centre for Processing Accelerated Corporate Exit (C-PACE)?
A) NITI Aayog B) Ministry of Corporate Affairs
C) Ministry of Commerce and Industry D) Ministry of External Affairs
Answer : B
✓ The government established the Expedited Corporate Exit Processing Center (C-PACE) and C-PACE is
under the Ministry of Corporate Affairs (MCA).
✓ The C-PACE aims to streamline and speed up the process of winding up defunct companies.
✓ It centralises the process of winding up companies, making it easier and more efficient.
✓ This relieves the burden on the registry and ensures a clean and accurate database.
✓ Its establishment is part of the MCA's commitment to promote Ease of Doing Business and facilitate
smooth exit of companies.
✓ It functions under the Registrar of Companies (ROC) as per sub-section (1) of section 396.
✓ The C-PACE office was inaugurated on May 1, 2023.
✓ The office is supervised by the Director General of Corporate Affairs (DGCoA) in New Delhi.
Q. Ministry of Cooperation has informed that government will form How Many New Farmer Producer
Organizations (FPOs) in the Cooperative Sector?
A) 1,000 B) 1,100
C) 1,200 D) 1,300
Answer : B

Follow us: Official Site, Telegram, Facebook, Instagram, Instamojo 413


✓ Government to set up 1,100 new farmer producer organisations in the cooperative sector.
✓ The government will set up 1,100 new Farmer Producer Organizations (FPOs) in the cooperative sector
and the initiative is in line with Prime Minister Narendra Modi's vision of "prosperity through
cooperation".
✓ The National Cooperative Development Corporation (NCDC) will monitor the allotment of these
additional FPOs.
✓ Each FPO will get financial assistance of Rs 33 lakh.
✓ Cluster based business organisations associated with FPOs will be given Rs 25 lakh per FPO.
✓ The aim of setting up the FPO is to provide the necessary market linkages to the farmers.
✓ Its objective is to ensure remunerative prices for the produce of the farmers
Q. Which bank has partnered with National e-Governance Services Limited (NESL) to launch
the Electronic Bank Guarantee (BG) system on digital platforms?
A) Bank of Baroda B) Indian Overseas Bank
C) Kotak Mahindra Bank D) State Bank of India
Answer : A
✓ Bank of Baroda partnered with National e-Governance Services Limited (NESL) to launch the Electronic
Bank Guarantee (BG) system on digital platforms.
✓ It aims to significantly reduce the turnaround time for issuance of electronic bank guarantees as compared
to the traditional paper-based process.
✓ The electronic bank guarantee system is accessible through Baroda INSTA, the digital platform of Bank
of Baroda.
✓ The digital approach enhances convenience, security and efficiency in the issuance of inland bank
guarantees.
✓ About Bank of Baroda (BoB)
✓ It is an Indian public sector bank headquartered in Vadodara, Gujarat.
✓ It is the second largest public sector bank in India after State Bank of India and has a global presence with
over 100 overseas offices.
✓ The bank was established on 20 July 1908 by Sayajirao Gaekwad III, the Maharaja of Baroda.
Q. Who has launched ‘General Surety Bond Bima (Conditional & Unconditional)’ insurance product for
infrastructure projects?
A) SBI General Insurance B) Bharti AXA General Insurance
C) Bajaj Allianz General Insurance D) Cholamandalam MS General Insurance
Answer : A
✓ SBI General launches ‘Surety Bond Bima‘ for Infrastructure ProjectsOn May 23, 2023, SBI General
Insurance (SBIG), the non-life insurance arm of State Bank of India (SBI), launched ‘General Surety
Bond Bima (Conditional & Unconditional)’ insurance product for infrastructure projects.
✓ It will act as a security arrangement to provide protection for infrastructure projects and shield both the
contractor and the principal (the entity awarding the contract).
✓ This Surety Bond Insurance is available in two variants viz. Conditional and Unconditional.
✓ What is Surety Insurance?
✓ It provides an assurance to the project owner in the form of a Surety Bond that the contractor would
complete the project as per the agreed terms and conditions. It involves three parties: the principal
(contractor), the obligee (the individual or entity hiring the contractor), and the surety company (the
insurer).

Follow us: Official Site, Telegram, Facebook, Instagram, Instamojo 414


Q. Which company has become the first general insurance company to accept RBI’s Central Bank Digital
Currency (CBDC) e-Rupee (e₹) for premium payments?
A) Bharti AXA General Insurance B) Reliance General Insurance
C) Bajaj Allianz General Insurance D) Cholamandalam MS General Insurance
Answer : B
✓ Reliance General Insurance has become the first general insurance company to accept RBI’s Central
Bank Digital Currency (CBDC) e-Rupee ((e₹) for premium payments.
✓ The insurer has tied up with YES Bank to facilitate the collection of premiums in digital mode using the
bank’s e-rupee platform.
✓ Customers who have an active e-rupee wallet with any bank can scan Reliance General Insurance’s
CBDC QR code to make easy, safe, instant, and green payments.
✓ e-rupee is a digital token, equivalent to a banknote, and is legal tender or sovereign currency backed by
the RBI.
✓ Currently, Reliance General Insurance’s physical e-rupee QR code is available at select branches for
walk-in customers to scan and pay instantly.
Q. RBI announce withdrawal of RS 2000 denomination bank notes from circulation in pursuance of its
“clean note policy” all bank branches, will provide the facility of exchange of Rs 2000 bank note up to
limit of ___ at a time.
A) 12000 B) 20000
C) 30000 D) 50000
Answer : B
✓ Reserve Bank of India (RBI) announced that it will withdraw the Rs 2000 denomination banknotes from
circulation.
✓ While the existing notes will remain legal tender. The RBI has provided a generous timeframe, allowing
individuals to deposit or exchange the notes until September 30, 2023.
✓ Exchange of Rs.2,000 bank notes to notes of other denominations can be made up to a limit of Rs 20,000
at a time.
✓ The Rs 2000 banknotes were introduced in 2016 to meet the immediate currency requirements after the
withdrawal of Rs 500 and Rs 1000 notes during the demonetization exercise..
✓ As of March 31, 2023, the value of Rs 2000 banknotes in circulation has decreased to Rs 3.62 lakh crore,
constituting only 10.8% of the total notes in circulation.

Follow us: Official Site, Telegram, Facebook, Instagram, Instamojo 415


BEST MCQ BANKING, ECONOMY AND FINANCIAL AWARENESS
APRIL 2023

Q. What is the estimated global trade growth rate in 2023 as per the World Trade Organization (WTO)?
A) 1.0% B) 1.5%
C) 1.7% D) 1.9%
Answer : C
✓ WTO projects growth rate at 1.7% in 2023
✓ The World Trade Organization (WTO) estimates that the global trade growth in 2023 is expected to slow
to 1.7 percent.
✓ This was 2.7 percent growth in 2022.
✓ The decline in trade may be due to multiple factors, such as the ongoing Russia-Ukraine conflict, high
inflation, tighter monetary policy and financial uncertainty.
✓ The World Trade Organization said in its forecast that trade growth for 2024 is expected to be 3.2 percent.
✓ World Trade Organisation (WTO)
✓ It was set up on 1st January 1995 replacing the General Agreement on Tariff and Trade (GATT).
✓ Its main purpose is to promote rule based trading systems in the world and it also settles trade related
disputes between the member countries.
✓ The headquarters of WTO is in Geneva, Switzerland.
✓ There are 164 member countries.
✓ Director General of WTO : Dr Ngozi -Okonjo-Iweala of Nigeria
✓ Report released by WTO : World Trade Report
Q. Which bank has recently raised USD 750 million through bonds to fund offshore business growth?
A) Punjab National Bank B) HDFC Bank
C) ICICI Bank D) State Bank of India
Answer : D
✓ State Bank of India (SBI), the country’s biggest lender, has raised USD 750 million through bonds to
fund offshore business growth.
✓ SBI has concluded the issue of USD 750 million senior unsecured fixed rate notes having a maturity of 5
years and coupon of 4.875 percent payable semiannually under Regulation-S.
✓ Earlier this month, the bank’s board had approved a proposal to raise $2 billion (about Rs 16,000 crore)
from bonds to fund its global operations.
Q. RBI has decided to launch a secured web-based centralised portal to simplify and streamline application
processes. What is the name given to the portal?
A) PRAVAAH B) SATAT
C) UTHAAN D) PRAMUKH
Answer : A
✓ The Reserve Bank of India (RBI) has planned to launch a secured web-based centralised portal named
‘PRAVAAH’ to simplify and streamline application processes.
✓ PRAVAAH stands for Platform for Regulatory Application, Validation And AutHorisation.
✓ The portal will show time limits for deciding on the applications/approvals sought.

Follow us: Official Site, Telegram, Facebook, Instagram, Instamojo 416


✓ Currently, the application and approval processes for the same take place in varied on-line and off-line
modes.
✓ The new system will bring greater efficiencies into regulatory processes and facilitate ease of doing
business for the regulated entities of RBI.
✓ The facility will be extended to all types of applications made to RBI across all functions.
Q. The Solar Energy Corporation of India Limited (SECI) has been accorded which status by the
Government of India?
A) Maharatna B) Miniratna Category-I
C) Navratna D) Miniratna Category-II
Answer : B
✓ The Solar Energy Corporation of India Limited (SECI) has been accorded the status of ‘Miniratna
Category-I’ Central Public Sector Enterprise (CPSE) on April 10, 2023.
✓ SECI is a company of the Ministry of New and Renewable Energy (MNRE) to facilitate the
implementation of the National Solar Mission (NSM).
✓ It is the only Central Public Sector Undertaking (PSU) dedicated to the solar energy sector.
✓ It is the primary implementing agency of MNRE for renewable energy schemes
Q. Union Cabinet has approved the National Quantum Mission for 2023-2031. How much amount has been
allocated to this mission?
A) ₹6,005.95 crore B) ₹6,000.15 crore
C) ₹6,003.65 crore D) ₹6,001.35 crore
Answer : C
✓ The Union Cabinet has approved the National Quantum Mission, with the aim to strengthen India’s
research and development in the quantum arena alongside indigenously building quantum-based
(physical qubit) computers.
✓ The mission will work on developing quantum supercomputers and establishing secure quantum
communications with other nations.
✓ The National Quantum Mission has been launched for an eight-year period, through 2023 – 2031. The
total budget allocated to the mission is ₹6,003.65 crore.
✓ Till date only six other nations are doing research and development in quantum technologies.
✓ They are the US, Canada, China, Austria, Finland and Scotland.

Q. Which company has recently got the status of Navratna Central Public Sector Enterprise?
A) Rail Vikas Nigam Limited B) NMDC Limited
C) India Post D) NTPC Limited
Answer : A
✓ The Government of India upgraded the status of Rail Vikas Nigam Limited (RVNL) to 'Navratna' Central
Public Sector Enterprise (CPSE).
✓ Prior to this upgrade, RVNL was earlier classified as a 'Miniratna' CPSE.
✓ The decision to upgrade RVNL was approved by the Finance Minister and is effective from April 26,
2023.
✓ RVNL is a mid-cap company under the Ministry of Railways.
✓ For the year 2021-22, RVNL has an annual turnover of Rs 19,381 crore and a net profit of Rs 1,087 crore.
✓ This upgrade makes RVNL the 13th Navratna company among CPSEs in India.
✓ About Navratna CPSE

Follow us: Official Site, Telegram, Facebook, Instagram, Instamojo 417


✓ It is a select group of public sector enterprises in India that have been granted greater autonomy in
decision making and financial management by the government.
✓ The term "Navratna" means "Nine Gems" in Hindi and refers to the original nine CPSEs that were
granted this status in 1997.
✓ Navratna CPSEs are also subject to certain obligations, such as achieving certain performance targets
and maintaining minimum levels of public shareholding.
✓ Some examples of Navratna CPSEs include Bharat Electronics Limited, Hindustan Petroleum
Corporation Limited and Power Grid Corporation of India Limited.
✓ List of Navratna Companies in India 2023
1. Shipping Corporation of India Limited
2. Rashtriya Ispat Nigam Limited
3. Oil India Limited
4. NMDC Limited
5. Neyveli Lignite Corporation Limited
6. National Building Construction Corporation Limited
7. National Aluminum Company Limited
8. Mahanagar Telephone Nigam Limited
9. Hindustan Aeronautics Limited
10. Engineers India Limited
11. Container Corporation of India Limited
12. Bharat Electronics Limited

Q. Who has been appointed as Chairman of the Life Insurance Corporation of India till June 2024?
A) Siddhartha Mohanty B) TC Suseel Kumar
C) Vipin Anand D) Mukesh Kumar Gupta
Answer : A
✓ The Indian Government has approved the appointment of Siddhartha Mohanty as Chairperson of Life
Insurance Corporation of India (LIC) till June 29, 2024.
✓ Before this, Mohanty was to superannuate as LIC CEO and MD in June 2025.
✓ He took over as interim chairman of LIC in March 2023 for three months.
✓ Mohanty is one of the four managing directors at LIC.
✓ Before joining LIC as MD, Mohanty was serving as MD and chief executive officer of LIC Housing
Finance – one of the largest mortgage financiers in the country.

LIC IN NEWS 2023


▪ Siddhartha Mohanty appointed as Chairman of LIC Until June 2024.
▪ Ratnakar Patnaik appointed as new Chief Investment Officer of LIC.
▪ Tablesh Pandey and M. Jagannath have been appointed as the managing directors of the Life Insurance
Company (LIC).
▪ Life Insurance Corporation of India (LIC) has launched LIC Jeevan Azad Plan.
▪ The minimum basic sum assured under LIC Jeevan Azad Plan is Rs 2 lakh and the maximum basic sum
assured is Rs 5 lakh. The policy can be taken for term of 15 to 20 years.
▪ LIC to divest 60.72% share in IDBI Bank.
▪ LIC presently has four managing directors
▪ Life Insurance Corporation of India (LIC)
▪ Founded : 1956
▪ Headquarters : Mumbai
▪ Chairman : Siddhartha Mohanty
▪ It is India's largest life insurance company and also the country's largest investment company.
▪ Authorised Capital of LIC - Rs 25,000 Crore.

Follow us: Official Site, Telegram, Facebook, Instagram, Instamojo 418


▪ At Rs 21,000 crore, LIC is India's largest IPO.
Q. What is the additional corpus support provided in the Union Budget for FY 2023-24 to the CGTMSE
Scheme?
A) ₹2,000 crore B) ₹4,500 crore
C) ₹7,500 crore D) ₹9,000 crore
Answer : D
✓ Union Minister for MSME Shri Narayan Rane launched the revamped CGTMSE Scheme in Mumbai
27th April 2023.
✓ CGTMSE has been provided with an additional corpus support of ₹9,000 crore in the Union Budget for
FY 2023-24 to revamp its Scheme to provide guarantee for additional ₹2 lakh crore to Micro & Small
Enterprises.
✓ Accordingly, the major revamp measures were launched and disseminated to the lending institutions.
The modifications included reduction in guarantee fees for loans upto ₹1 crore by 50% bringing the
minimum guarantee fee to the level of 0.37% pa only.
✓ Another major change announced was raising of ceiling for guarantee from ₹2 crore to ₹ 5 crore and
enhancing the threshold limit for claim settlement without initiation of legal action to ₹10 lakh
Q. Who has been ranked as the top most influential person in the world in the TIME Magazine’s 2023
TIME100 poll?
A) Prince Harry B) Lionel Messi
C) Shah Rukh Khan D) Mukesh Ambani
Answer : C
✓ Bollywood actor Shah Rukh Khan has achieved the top spot on 2023 TIME100 poll, the TIME
magazine’s annual list of the most influential people.
✓ Out of the 1.2 million votes cast, 4% of them went to Shah Rukh.
✓ The Iranian women who are protesting for their rights stood second in the list, with 3% of the votes.
✓ The healthcare workers who’ve been at the frontlines of the pandemic since 2020, stood third with 2% of
the overall votes.
✓ Third position was shared by Prince Harry, the Duke of Sussex.
✓ Fourth place went to Meghan Markle, the Duchess of Sussex.
✓ Fifth spot was gained by Lionel Messi
TIME MAGAZINE'S IN NEWS 2023.
▪ Bollywood star Deepika Padukone has been featured on the latest cover of TIME magazine. The iconic
American magazine described Padukone as a ‘global star’ bringing ‘the world to Bollywood’.
▪ Shah Rukh Khan has achieved the top spot on 2023 TIME100 poll, the TIME magazine’s annual list of
the most influential people.
▪ Odisha’s Mayurbhanj and Ladakh among TIME magazine’s list of ‘World’s Greatest Places of 2023’.
▪ Time magazine named President Volodymyr Zelensky as well as “the spirit of Ukraine” as its 2022 person
of the year.
Q. RBI has announced the framework for the acceptance of green deposits of regulated entities (RE). It will
come into force from __
A) June 01 B) April 01
C) May 01 D) July 01

Follow us: Official Site, Telegram, Facebook, Instagram, Instamojo 419


Answer : A
✓ The Reserve Bank of India (RBI) has announced the framework for the acceptance of green deposits of
regulated entities (RE).
✓ The framework will come into effect from June 1, 2023.
✓ As per the guidelines of RBI, a Green Deposit means:
✓ An interest-bearing deposit, received by the RE for a fixed period and the proceeds of which are
earmarked for being allocated towards green finance.
✓ Lending to and investing in the activities or projects that contribute to climate risk mitigation, climate
adaptation and resilience, and other climate-related or environmental objectives – including biodiversity
management and nature-based solutions.
✓ The green deposits will be denominated in Indian rupees only.
✓ The Regulated entities (REs) will have to allocate proceeds raised from green deposits towards the green
activities or projects.
✓ These instructions are applicable for all scheduled commercial banks, including small finance banks.
However regional rural banks, local area banks and payments banks are excluded.
✓ The framework will also apply on all deposit-taking non-banking financial companies (NBFCs) registered
with the RBI, including housing finance companies.
✓ The aim of Green Deposit framework:
✓ To encourage regulated entities (REs) to offer green deposits to customers to foster and develop a green
finance ecosystem in the country.
✓ To protect the interest of the depositors, aid customers to achieve their sustainability agenda, address
greenwashing concerns and help augment the flow of credit to green activities or projects.
Q. Which country was the highest source of foreign portfolio investor (FPI) in India in FY23, as per National
Securities Depository?
A) Singapore B) China
C) United States D) Japan
Answer : C
✓ The US was largest source of foreign portfolio investment (FPI) inflows into India in FY23; Mauritius
witnessed sharpest decline.
✓ As per the data released by the National Securities Depository, the overseas FPI inflow into Indian capital
markets from Mauritius witnessed the sharpest decline in FY23.
✓ The total assets under custody (AUC) from Mauritius declined nearly 42 percent to Rs 3.25 trillion as of
March 2023. This was Rs 5.55 trillion in 2022.
✓ Overall, FPI AUC fell 4.44 percent in FY23.
✓ The US was the biggest source for foreign portfolio investor (FPI) flows in India in FY23, with total AUC
of Rs 19.19 trillion.
✓ Overall, FPI AUC fell 4.44 percent in FY23.
✓ The US was the biggest source for foreign portfolio investor (FPI) flows in India in FY23, with total AUC
of Rs 19.19 trillion.
✓ Singapore (4.56 trillion), Luxemburg (3.64 trillion), Mauritius (3.25 trillion) and the United Kingdom
(2.55 trillion), are the top 5 countries respectively.
✓ Norway (1.47 trillion) and Singapore witnessed highest growth in FPI AUC in FY23.

Q. Which country saw highest decline in the foreign portfolio investor (FPI) flows in India during FY23?
A) Mauritius B) Singapore
C) United States D) Norway
Answer : A

Follow us: Official Site, Telegram, Facebook, Instagram, Instamojo 420


✓ As per the data released by the National Securities Depository, the overseas equity inflow into Indian
capital markets from Mauritius witnessed the sharpest decline in FY23.
Q. Which company has emerged as the second-strongest tyre brand in the world according to the report by
Brand Finance on the ‘Most Valuable and Strongest Tyre Brands’ in the world?
A) JK Tyre & Industries B) CEAT Tyres
C) MRF Limited D) Apollo Tyres
Answer : C
✓ MRF Ltd has emerged as the second-strongest tyre brand in the world according to the latest report by
Brand Finance on the ‘Most Valuable and Strongest Tyre Brands’ in the world.
✓ MRF has scored high in almost all parameters and is also featured as the second fastest-growing tyre
brand in the world.
✓ It has scored 83.2 out of 100 in brand strength and was awarded a AAA- brand rating.

Q. Which country was the biggest trading partner of India in 2022-2023?


A) China B) Germany
C) France D) United States
Answer : D
✓ According to the provisional data of the Ministry of Commerce, the United States has emerged as India’s
biggest trading partner in 2022-23.
✓ The bilateral trade between India and the US increased by 7.65 percent to USD 128.55 in 2022-23
compared to USD 119.5 billion in 2021-22.
✓ Speaking about exports to the US, it rose by 2.81 percent to USD 78.31 billion in 2022-23 as against USD
76.18 billion in 2021-22.
✓ Imports from US grew by about 16 percent to USD 50.24 billion.
✓ In 2022-23, China was the second largest trading partner of India. However, the two-way commerce
between them declined by about 1.5 percent to USD 113.83 billion as against USD 115.42 billion in 2021-
22.
✓ The UAE (USD 76.16 billion), was the third largest trading partner of India, followed by Saudi Arabia
(USD 52.72 billion), and Singapore (USD 35.55 billion) respectively.
Q. Which country will be the top contributor to global growth over the next five years through 2028, as per
the data of IMF?
A) Indonesia B) United States
C) China D) India
Answer : C
✓ AS per the data by the International Monetary Fund (IMF), China will be the top contributor to global
growth over the next five years, and its share is set to be double than that of the US.
✓ The report by Bloomberg, based on the data of World Economic Outlook of IMF, reveals that China
share in the global gross domestic product expansion over the next five years, through 2028, is expected
to represent 22.6% of total world growth.
✓ India is on second spot, representing 12.9% of the total growth.
✓ The US will contribute 11.3%, which is half than that of China.
✓ International Monetary Fund (IMF)
✓ Founded : 1945
✓ Headquarters: Washington, D.C.

Follow us: Official Site, Telegram, Facebook, Instagram, Instamojo 421


✓ Managing Director : Kristalina Georgieva (Bulgaria)
✓ First Deputy MD : Gita Gopinath
✓ Chief Economist : Pierre-Olivier Gourinchas (France)
✓ Executive Director for India at IMF : Krishnamurthy Subramaniam
✓ Member countries : 190 (Andorra)
✓ IMF Released Report
✓ Global Financial Stability Report
✓ World Economic Outlook.
✓ The value of Special Drawing Right (SDR) is determined by the basket of 5 currencies.
✓ The currencies are, US Dollar, Japanese Yen, British Pound, Chinese Yuan and Euro.
✓ Special Drawing Right is known as the Paper Gold. The value of the SDR is based on a basket of key
international currencies reviewed by IMF every five years. SDR was introduced in the 1969 by the IMF
to solve the problem of International liquidity.
Q. Who has received a non-banking financial company (NBFC) license from the Reserve Bank of India?
A) InstantPay B) FamPay
C) Jupiter D) Fi Money
Answer : C
✓ RBI grants NBFC Licence to neobank Jupiter.
✓ The neobanking startup, Jupiter has secured the non-banking finance company (NBFC) licence from the
Reserve Bank of India (RBI).
✓ This NBFC licence will help Jupiter to lend credit from its book and further scale its business fast.
✓ Jupiter will lend loan through its Bengaluru-based Amica Financial Technologies Ltd., the private entity
which operates Jupiter.
Q. The Kirit Parikh Committee, which was seen in the news, is associated with which sector?
A) Gas Pricing B) Long Term Food Policy
C) On Small Scale Industries D) Increase in Motor Vehicle Tax
Answer : A
✓ The government on April 7 fixed natural gas price at $7.92 per MMBTU (million British thermal units)
for the rest of April under the new pricing formula suggested by the Kirit Parekh Committee.
✓ However, the final rates for consumers have been capped at $6.5 per MMBTU.
✓ The Petroleum Planning and Analysis Cell (PPAC) of the oil ministry said in the order that the price of
natural gas for April 8 to April 30 would be $7.92 per million British thermal units.
✓ This price has been fixed on the basis of 10 per cent of the average cost of imported crude oil.
✓ The Union Cabinet, while changing the pricing formula, capped the rates for consumers at $6.5 per
MMBTU. This limit will be applicable for two years till March 31, 2025.
✓ “The price for gas produced by ONGC/OIL from their old fields shall be subject to a ceiling of $6.5 per
MMBTU,” .
✓ The new rates will bring down the prices of CNG and compressed natural gas (CNG) transported through
pipelines and used in various industries including auto fuel and petrochemicals and fertilizer sector, by
up to 10 per cent.
Q. The Government of India aims to achieve nearly 9% share of electricity generation from nuclear sources
by which year?
A) 2047 B) 2025
C) 2030 D) 2027

Follow us: Official Site, Telegram, Facebook, Instagram, Instamojo 422


Answer : A
✓ India to get 9% of Electricity from Nuclear Sources by 2047
✓ AS per the information by the Union Minister Jitendra Singh, in India, the Government has set the target
that nearly 9% share of electricity is likely to be contributed from nuclear sources by 2047.
✓ The rising contribution of nuclear power in India’s energy sector will help the country to achieve the
commitment of net zero target by 2070.
✓ The other target laid down by the Department of Atomic Energy is achieving 20 gigawatts (GW) capacity
of nuclear power generation by 2030.
✓ This will make India the third-largest producer of atomic energy in the world after the US and France.
Q. The Department of Atomic Energy has set the target to achieve 20 gigawatts (GW) capacity of nuclear
power generation by which year?
A) 2030 B) 2040
C) 2050 D) 2060
Answer : A
✓ The other target laid down by the Department of Atomic Energy is achieving 20 gigawatts (GW) capacity
of nuclear power generation by 2030.
Q. As per the 2023 Foreign Trade Policy, the Government of India has set the vision to achieve the target
of $2 trillion goods and services exports by which year?
A) 2030 B) 2025
C) 2035 D) 2040
Answer : A
✓ Union Minister of Commerce & Industry, Piyush Goyal announced the Foreign Trade Policy 2023, on
March 31, 2023.
✓ The 2023 foreign trade policy has the vision to take India’s goods and services exports to $2 trillion by
2030.
✓ The FTP 2023 will come into effect from April 1, 2023.
✓ Key highlights of the 2023 FTP
1. The policy will be dynamic. There is no end date to the policy, unlike the previous FTP’s which has
duration of five years. The policy will be updated or changed as and when necessary, based on the
feedback received.
2. The new FTP seeks to make the Indian rupee a global currency and allow international trade settlement
in the domestic currency.
3. Target to achieve the goal of $1 trillion merchandise exports and $1 trillion services exports by 2030.
4. 4 new towns, namely, Faridabad, Moradabad, Mirzapur and Varanasi declared as towns of export
excellence. These are in addition to the existing 39 towns under the category.
✓ Faridabad will be the town of export excellence for apparels.
✓ Moradabad will be the town of export excellence for handicrafts.
✓ Mirzapur will be the town of export excellence for handmade carpet and dari.
✓ Varanasi will be the town of export excellence for handloom and handicraft
5. FTP 2023 will encourage e-commerce export, which is expected to grow to $200-300 billion by 2023.
6. The dairy sector has been exempted from maintaining average export obligation
7. The value limit for exports through courier services increased to Rs 10 lakh from Rs 5 lakh per
consignment
8. FTP 2023 aims at streamlining the SCOMET (Special Chemicals, Organisms, Materials, Equipment and
Technologies) policy, which covers special chemicals, organisms, materials, equipment and technologies.

Follow us: Official Site, Telegram, Facebook, Instagram, Instamojo 423


The policy for the export of dual -use items under the SCOMET has been consolidated in one place for
ease of understanding and compliance by the industry.
9. The policy has recommended restructuring of the Department of Commerce to make it future ready.
10. Foreign Trade Policy 2023 has introduced an amnesty scheme for one-time settlement of default in export
obligation.
Q. Which of the following interest rate on small savings scheme is not correct under the April to June quarter
of financial year 2023-2024?
A) Senior Citizen Savings Scheme – 8.2% B) National Savings Certificate – 7.7%
C) Public Provident Fund Scheme – 7.4% D) Sukanya Samriddhi Account – 8.0%
Answer : C
✓ Union Ministry of Finance has raised the interest rates on different small savings schemes between 10-70
basis points for the April to June quarter of financial year 2023-2024.
✓ The revised interest rates will be applicable from the financial year starting April 01, 2023.
✓ This is the third time in past 9 months, that the interest rates on small savings schemes have been
increased.
✓ The interest rates on small savings schemes range from 4.0 percent to 8.2 percent for April-June 2023.
✓ Only, the interest rate for Public Provident Fund (PPF) scheme and Post Office Savings Account have
been kept unchanged during this period.
✓ Various Interest Rates for Quarter-1 (April-June) of 2023-24 are listed below.
✓ Savings Deposit : 4.0%
✓ Senior Citizen Savings Scheme : 8.2%
✓ National Savings Certificate : 7.7%
✓ Public Provident Fund Scheme 7.1%
✓ Kisan Vikas Patra : 7.5% (115 months)
✓ Sukanya Samriddhi Account Scheme : 8.0%
Q. What is the rate of interest on Sukanya Samriddhi Account Scheme for the quarter 1 of the financial year
2023-24 (Apr-Jun 2023)
A) 7.1% B) 6.5%
C) 7.6% D) 8.0%
Answer : D
✓ Sukanya Samriddhi Yojana
✓ The scheme was launched by Prime Minister Narendra modi on 22 January 2015 as a part of the Beti
Bachao, Beti Padhao campaign.
✓ Who can open the account : A natural/ legal guardian on behalf of a girl child.
✓ Minimum Deposit per Year : Rs 250/- (then amount in multiple of Rs 100 can be deposited thereafter)
✓ Maximum Deposit per Year: Rs.1,50,000 /- (Any deposit above the maximum cap will not earn any
interest and can be withdrawn anytime by the depositor).
✓ Number of accounts under Sukanya Samriddhi Account Yojana:
✓ Only one account per girl child.
✓ Accounts can be opened for a Maximum of two girl children in one family.
✓ In case, the guardian has triplet daughters from first birth or twin daughters from the second birth,
maximum of 3 accounts can be opened by the parent, where one account is opened in the name of a
single girl child.
✓ When can an SSA account be opened : Any time between birth of girl child till she attains the age of 10
years.
✓ Till which year regular yearly deposit is required– For initial 15 years from the date of opening of account.

Follow us: Official Site, Telegram, Facebook, Instagram, Instamojo 424


✓ Example : SSA account is opened for Girl A, when her age is 1 year. The regular deposit will be required
till she turns 16 year. For Girl B, whose account is opened when she is 9 years, the regular deposit is
required till she turns 24 years.
✓ Maturity of deposit : After 21 years from date of opening of account. (No interest will be earned after
this.)
✓ Sukanya Samriddhi Accounts Interest rate : 7.6%
Q. The interest rate on Kisan Vikas Patra for April to June quarter of financial year 2023-2024 is set at 7.5
percent. What is the maturity period of this scheme?
A) 125 months B) 120 months
C) 115 months D) 100 months
Answer : C
✓ Kisan Vikas Patra : 7.5% (115 months)

Q. Who has been appointed as the new MD & CEO of Bank of India (BOI)?
A) Shyam Srinivasan B) Rajiv Lall
C) Rajneesh Karnatak D) Atanu Chakraborty
Answer : C
✓ Rajneesh Karnatak has been appointed as the MD & CEO of the Bank of India (BOI) with effect from
April 29, 2023.
✓ Prior to this, Karnatak was the Executive Director at the Union Bank of India.
✓ He has been appointed for a term of three years.
✓ He replaces Atanu Kumar Das, whose term came to an end in January 2023.
✓ Bank of India (BOI)
✓ Founded : 1906
✓ Headquarteres : Mumbai
✓ MD & CEO : Rajneesh Karnatak
✓ Tagline : Relationship beyond Banking
Q. Which entity has entered into a partnership with digital payments infrastructure provider PPRO to
facilitate global e-commerce payments through UPI?
A) NPCI Bharat BillPay B) NPCI PayAuth Challenge
C) NPCI International Payments D) Indian Financial Technology and Allied Services
Answer : C
✓ NPCI International Payments, a wholly-owned subsidiary of National Payments Corporation of India
(NPCI), entered into partnership with digital payments infrastructure provider PPRO to facilitate global
e-commerce payments through UPI.
✓ The agreement is aimed at expanding RuPay card and UPI acceptance across global clients such as
payment service providers (PSPs) and global merchant acquirers.
✓ It will drive NPCI International’s expansion into foreign markets and will add India to PPRO’s Local
Payment Method (LPM) coverage map.
✓ International payment service providers and their merchants can now easily tap into an e-commerce
market that is expected to reach an estimated $111 billion next year and almost double to $200 billion by
2026.
✓ The UPI network processes 60 percent of all domestic payments and 40 percent of instant payments
processed globally.

Follow us: Official Site, Telegram, Facebook, Instagram, Instamojo 425


✓ National Payments Corporation of India(NPCI)
✓ Founded : 2008
✓ HQ : Mumbai, Maharashtra
✓ MD & CEO : Dilip Asbe
✓ Chairmam : Biswamohan Mahapatra
Q. Who has been appointed as the new CEO of the Association of Asset Reconstruction Companies
(ARCs)?
A) Salil Parekh B) Hari Hara Mishra
C) C Vijayakumar D) Rajan Amba
Answer : B
✓ Hari Hara Mishra has taken charge as the Chief Executive Officer of the Association of Asset
Reconstruction Companies.
✓ Mishra, who worked with State Bank of India from 1982 to 2004, Officer, has been associated with Asset
Reconstruction Sector since those formative days (2004) till date, in various ARCs in executive and
director level roles.
✓ The Association, which is the voice of all Asset Reconstruction Companies (ARCs), has been active for
more than eight years. Currently, there are 28 ARCs registered with Reserve Bank of India.
Q. Which bank has launched the ‘MicroPay’, India’s first pocket-sized swipe machine for accepting digital
payments.
A) Kotak Mahindra Bank B) HDFC Bank
C) ICICI Bank D) Axis Bank
Answer : D
✓ India’s third largest private sector lender Axis Bank has launched ‘MicroPay’, country’s first pocket-sized
swipe machine, for accepting digital payments.
✓ The ‘MicroPay’ has been launched along with Ezetap by Razorpay and MyPinpad as the technical
solution partners.
✓ MicroPay is based on ‘Pin on Mobile’ technology, that converts a merchant’s smartphone into a Point-
of-Sale (POS) terminal.
✓ How it works?
✓ MicroPay is a pocket-sized, small, low-cost card reader which supports “insert” and “tap” options for
debit and credit cards.
✓ The card reader can be connected to the merchant’s smartphone through Bluetooth and allows customers
to punch in their PIN directly on the merchant’s smartphone, for accepting digital payments.
Q. According to reports, the Education Ministry has selected around ___schools from across the country for
its flagship PM SHRI scheme.
A) 9,000 B) 6,000
C) 5,000 D) 7,000
Answer : A
✓ Ministry of Education has shortlisted around 9,000 schools across the country for its flagship Pradhan
Mantri Schools for Rising India (PM SHRI).
✓ The institutions were selected based on six broad parameters including curriculum, access and
infrastructure, human resources, and gender equity were assessed.

Follow us: Official Site, Telegram, Facebook, Instagram, Instamojo 426


✓ They were assessed on the basis of six broad parameters: curriculum, pedagogy and assessment; access
and infrastructure; human resources – leadership; inclusive practices and gender equity; management,
monitoring and governance and beneficiary satisfaction.
✓ About PM SHRI
✓ PM Modi announced this scheme on 5th September 2022.
✓ PM SHRI school is a new centrally sponsored scheme that aims at developing more than 14500 schools
across the country by strengthening select existing schools being managed by all forms of government.
✓ The PM SHRI schools will aim at delivering quality teaching for the cognitive development of students
and creating and nurturing holistic and well-rounded individuals equipped with key 21st-century skills
Q. The central government has sanctioned how much rupees under the Faster Adoption and Manufacturing
of Electric Vehicles (FAME) Scheme Phase-II for setting up 7432 public fast charging stations across the
country?
A) Rs 600 crore B) Rs 800 crore
C) Rs 500 crore D) Rs 700 crore
Answer : B
✓ Union Government has sanctioned 800 crore rupees under the Faster Adoption and Manufacturing of
Electric Vehicles (FAME) Scheme Phase -2 for setting up 7432 public fast charging stations across the
country.
✓ The government has formulated Faster Adoption and Manufacturing of Electric Vehicles (FAME) India
Scheme in 2015 to improve the infrastructure required for the large-scale use of electric vehicles.
✓ At present, there are about 6,586 charging stations across the country. The addition of the new 7,432
public charging stations will be a significant push to Electric Vehicles charging ecosystem.
✓ This move will boost the electric vehicle ecosystem in India and encourage more people to switch to
cleaner modes of transportation.
Q. NPCI has introduced an interchange charge of up to how much percent on merchant UPI transactions
done using prepaid payment instruments from 1 April 2023?
A) 1.1% B) 1.6%
C) 2.1% D) 1.3%
Answer : A
✓ The National Payments Corporation of India (NPCI) has issued a circular suggesting "Prepaid Payment
Instruments (PPI)" charges on merchant transactions on Unified Payments Interface (UPI).
✓ NPCI has introduced an interchange charge of up to 1.1 per cent on merchant UPI transactions done
using prepaid payment instruments from April 1.
✓ It will be levied on UPI payments above Rs 2,000 made to online merchants, large merchants and small
merchants.
✓ NPCI has clarified that there will be no charges for UPI payment from bank account to bank account or
normal UPI payment.
✓ With UPI, customers will have the option of using any bank account, RuPay credit card and prepaid
wallet on a UPI-enabled app.
✓ What is the interchange fee?
✓ The interchange fee is levied to cover the costs of accepting, processing, and authorising transactions.
✓ This is likely to make the transaction costlier.
✓ Interchange fee is a fee paid by payment service providers to wallet issuers such as banks.
✓ These wallets are mainly online payments like Paytm, PhonePe and Google Pay.
✓ National Payments Corporation of India (NPCI).

Follow us: Official Site, Telegram, Facebook, Instagram, Instamojo 427


✓ NPCI, an umbrella organization for operating retail payment and settlement systems in India, has been
initiated by the 'RBI' and the 'Indian Banks' Association' (IBA) under the 'Payment and Settlement
Systems Act, 2007'.
✓ It is a 'not-for-profit' company established under the provisions of Section 25 of the Companies Act 1956
(as amended in 2013), with the objective of providing physical and electronic payment infrastructure to
the entire banking system in India.
Q. Which of these insurers have not been listed as the Domestic Systemically Important Insurers (D-SIIs)
for 2022-23 by IRDAI?
A) National Insurance Company Limited
B) Life Insurance Corporation of India
C) General Insurance Corporation of India
D) The New India Assurance Co. Ltd
Answer : A
✓ The Insurance Regulatory and Development Authority of India (IRDAI) has retained the following
insurers as Domestic Systemically Important Insurers (D-SIIs) for 2022-23:
✓ Life Insurance Corporation of India;
✓ General Insurance Corporation of India; and
✓ The New India Assurance Co. Ltd.
✓ Domestic Systemically Important Insurers (D-SIIs) are the insurance companies that are considered “Too
Big (or Too Important) to Fail”. i.e its failure will impact the economy and insurance sector to a great
extend.
✓ IRDAI has chosen these three insurers on the basis of their size, market importance and domestic and
global interconnectedness whose distress or failure would cause significant dislocation in the domestic
financial system.
✓ Insurance Regulatory and Development Authority (IRDAI)
✓ Founded : 1999
✓ Headquarteres : Hyderabad
✓ Chairman : Debasish Panda
Q. India’s agricultural exports grow by how much percent during April-January of the current financial year
to worth USD 43.37 billion?
A) 5.12% B) 3.76%
C) 6.04% D) 4.34%
Answer : C
✓ India’s agricultural exports rose 6.04 percent during April-January of the current financial year to worth
USD 43.37 billion. During the same period last year, it was USD 40.90 billion.
✓ During the full financial year 2021-22, India’s agricultural exports touched the highest ever at USD 50.21
billion.
✓ The Agricultural and Processed Food Products Export Development Authority (APEDA) engages in
export promotion activities for agricultural and processed food products.
Q. The Ministry of Electronics and Information Technology (MeitY) has constituted a nine-member task
force to make India a 'Product Developer and Manufacturing Nation'. who is chaired this committee?
A) Rajesh Verma B) Kaushal Pandey
C) Bhuvnesh Kumar D) Amaresh Goswami

Follow us: Official Site, Telegram, Facebook, Instagram, Instamojo 428


Answer : C
✓ The Ministry of Electronics and Information Technology (MeitY) has constituted a nine-member task
force to make India a ‘product developer and manufacturing nation’.
✓ It will be chaired by the additional secretary (MeitY), Bhuvnesh Kumar with the joint secretary
(electronics), Amitesh Kumar Sinha as the member convenor.
✓ This is an effort to position itself as an alternative to the likes of China and Vietnam.

Q. Which organization has set up 'Corporate Debt Market Development Fund'


A) RBI B) SEBI
C) NSE D) BSE
Answer : B
✓ SEBI decided to set up a corporate debt market development fund in the form of an Alternative
Investment Fund.
✓ Corporate Debt Market Development Fund will be set up with an initial corpus of Rs 3,000 crore.
✓ It will function as a backstop facility for buying investment grade corporate debt securities during stressful
times.

Q. Under the recently revamped Credit Guarantee Scheme, what is the ceiling limit for guarantees coverage?
A) Rs 5 cr B) Rs 2 cr
C) Rs 3 cr D) Rs 4 cr
Answer : A
✓ The Government of India has further revamped the Credit Guarantee Scheme to increase the flow of
credit to MSEs and to strengthen the MSE Sector.
✓ In the revamped scheme, the annual guarantee fees structure have been reduced to 0.37 percent.
✓ The ceiling for guarantees coverage has been enhanced from Rs 2 crore to Rs 5 crore.
✓ The initiative is a part of the government’s efforts to strengthen the micro, small and medium enterprises
(MSME) sector.
Q. Who is the head of the committee formed by Ministry of Housing and Urban Affairs to look into issues
related to completing stalled housing projects?
A) Bibek Debroy B) Arvind Panagariya
C) Sanjiv Mehta D) Amitabh Kant
Answer : D
✓ The Ministry of Housing and Urban Affairs has set up a 14-member committee to look into issues related
to completing stalled housing projects, and propose a future course of action to ensure that buyers receive
their homes.
✓ The committee will be headed by Amitabh Kant, former CEO of Niti Aayog and India’s G20 Sherpa. It
will comprise of top central and state government officials.
✓ The panel will suggest ways to complete and hand over the projects to homebuyers in a “time-bound
manner.
✓ The committee shall submit its report within six months from the date of its first meeting.
✓ India’s G-20 Sherpa Amitabh Kant.
✓ Amitabh Kant Chief Executive Officer (CEO) of the National Institution for Transforming India (NITI
Aayog) (2016-2022).

Follow us: Official Site, Telegram, Facebook, Instagram, Instamojo 429


Q. E-commerce app ‘Pincode’ has been launched by which company?
A) Amazon B) GPay
C) PhonePe D) Paytm
Answer : C
✓ Walmart-owned fintech company PhonePe has launched a new consumer application named ‘Pincode’,
which is powered by the government’s Open Network for Digital Commerce (ONDC) network.
✓ The service is currently live only in Bangalore, and will be launched in other cities soon.
✓ The app aims to promote local shopkeepers and sellers by digitally connecting consumers with all the
neighbourhood stores that they usually buy from offline.
✓ Pincode will launch in six major categories including grocery, food, pharma, electronics, home decor and
fashion.
✓ Customers can search for stores by browsing for category and can have multiple carts at the same time
with one active cart per store. Buyers can also save multiple carts.
✓ PhonePe
✓ Founded : 2015
✓ Headquarters : Bangalore, Karnataka
✓ Founder & CEO : Sameer Nigam
✓ Co-Founder & CTO : Rahul Chari
✓ It is owned by Flipkart, a subsidiary of Walmart.
✓ PhonePe app, based on the Unified Payments Interface (UPI), went live in August 2016.
✓ Walmart completed the acquisition of a 77% stake in Flipkart in August 2018
Q. According to the new joint report from the Institute for Energy Economics and Financial Analysis
(IEEFA) and JMK Research and Analytics, India could become the world’s second-largest solar
photovoltaic manufacturer by which year?
A) 2025 B) 2026
C) 2030 D) 2035
Answer : B
✓ According to the new joint report from the Institute for Energy Economics and Financial Analysis
(IEEFA) and JMK Research and Analytics, India could become the world’s second-largest solar
photovoltaic manufacturer by 2026, after China.
✓ India’s cumulative module manufacturing nameplate capacity more than doubled from 18GW in March
2022 to 38GW in March 2023.
✓ In terms of upcoming PV manufacturing installations, Gujarat is the leading state in India. It accounts
for nearly 57% of all the upcoming PV manufacturing capacity.
✓ Some major reasons manufacturers chose Gujarat for setting up their PV fabrication facilities include
cheaper industrial electricity prices and easy access to ports for imports and exports.
Q. Public Sector Banks (PSBs) have transferred unclaimed deposits worth how much rupees to the Reserve
Bank of India (RBI) by February 2023?
A) Rs 42,923 crore B) Rs 29,109 crore
C) Rs 35,012 crore D) Rs 51,347 crore
Answer : C
✓ Public sector banks (PSBs) have transferred unclaimed deposits of Rs 35,012 crore as of February 2023
to the Reserve Bank of India (RBI).
✓ These deposits have not been operated for 10 years or more.

Follow us: Official Site, Telegram, Facebook, Instagram, Instamojo 430


✓ SBI tops the list with an unclaimed amount of Rs 8,086 crore, while Punjab National Bank has Rs 5,340
crore and Canara Bank has Rs 4,558 crore.
✓ These unclaimed deposits belong to 10.24 crore accounts transferred to the RBI.
✓ Banks have been advised by RBI to play a more proactive role in finding the whereabouts of the account
holders whose accounts have remained inoperative.
Q. Which bank has recently launched a mobile application, ‘Digital Dukaan’ to empower merchants and to
accept payments through various digital modes?
A) Axis Bank B) Yes Bank
C) ICICI Bank D) HDFC Bank
Answer : A
✓ Axis Bank and VISA have jointly launched a mobile application, ‘Digital Dukaan’ to empower
merchants to accept payments through various digital modes.
✓ The app ‘Digital Dukaan’ has been specifically designed to address business requirements such as
accepting digital payments, inventory management and billing, among others.
✓ Digital Dukaan will also enable merchants to set up an online store and grow their business effortlessly
by creating new channels.
Q. Which bank has launched a new investment solution called ‘digiPortfolio’?
A) HSBC India B) Woori Bank
C) DBS Bank India D) SBM Bank India
Answer : C
✓ DBS Bank India has launched a new investment solution called ‘digiPortfolio’, which uses technology
and human expertise to provide customised investment options for investors.
✓ The platform is available on the bank’s digibank platform and offers a one-stop solution for investors to
put money into ready-made baskets of mutual funds, with diverse portfolios to cater to different investor
risk profiles.
✓ The digiPortfolio platform is run by Quantifeed, making it automated and easy to use, with two plans
starting at a minimum investment of ₹10,000 and ₹50,000, offering portfolio construction, monitoring,
and rebalancing services powered by Morningstar.
✓ About DBS Bank
✓ Its full document The Bank of Singapore Limited and The Bank Limited is a Singapore multinational
banking and financial services corporation based in Marina Bay, Singapore
Q. India has agreed with which country to settle trade in Indian Rupee?
A) Philippines B) Indonesia
C) Malaysia D) Vietnam
Answer : C
✓ Ministry of External Affairs announced on 1 April that India and Malaysia have agreed to settle trade in
the Indian rupee.
✓ In July 2022, the Reserve Bank of India (RBI) allowed settlement of international trade in Indian
currency.
✓ This step has been taken only after this decision of the RBI.
✓ According to the Ministry of External Affairs,trade between India and Malaysia can now be settled in
Indian Rupee (INR) in addition to the existing methods of settlement in other currencies.

Follow us: Official Site, Telegram, Facebook, Instagram, Instamojo 431


✓ Kuala Lumpur-based India International Bank of Malaysia (IIBM) has introduced this system in India
by opening a special Rupee Vostro account through Union Bank of India.
✓ Vostro accounts are used to make payments in domestic currency.
✓ What is a Vostro Account?
✓ Vostro is a Latin word meaning "your", hence, Vostro Account means "your account".
✓ Vostro account is defined as an account maintained by a correspondent bank on behalf of another bank.
✓ A Vostro account is established to enable a foreign correspondent bank to act as an agent or provide
services as an intermediary for a domestic bank.
✓ About Malaysia
✓ Prime Minister - Anwar Ibrahim
✓ Capital - Kuala Lumpur
✓ Currency - Malaysian ringgit
Q. Who will be the head of the committee formed by Ministry of Finance to review the National Pension
Scheme (NPS) and suggest any changes in it to meet demand of Government employees
A) T.V. Somanathan B) Samant Goel
C) Rajiv Gauba D) P.K. Mishra
Answer : A
✓ The Ministry of Finance has set up of a four-member committee to review the National Pension Scheme
(NPS) and suggest any changes in it, keeping in mind the interest of government employees.
✓ The committee will be headed by Finance Secretary T.V. Somanathan.
✓ The other members of the committee will comprise of Secretary, Personnel, Special Secretary, Personnel,
and Chairman of the Pension Fund Regulatory and Development Authority (PFRDA).
✓ The panel will suggest ways for higher pensionary benefits to the government employees covered under
the NPS, amid growing demand for the old pension system (OPS) with assured benefits without
contribution.
Q. Apple iPhone maker’s first company-owned store ‘Apple-BKC’ in the country will be opened in which
city?
A) Mumbai B) Hyderabad
C) Bengaluru D) Chennai
Answer : A
✓ Apple iPhone maker’s first company-owned store in the country will be opened in Mumbai. The store
will be named as Apple-BKC.
✓ The barricade of its first retail store in India at Jio World Drive Mall, Mumbai officially marking the
upcoming opening of Apple BKC.
✓ The move comes at a time when the smartphone major is ramping up plans to boost local manufacturing
in a fillip to the Central government’s ‘Make in India’ initiative.
Q. Which organization has published World Energy Transition Outlook report 2023?
A) United Nations Development Programme (UNDP)
B) United Nations Population Fund (UNPF)
C) United Nations High Commissioner for Refugees (UNHCR)
D) International Renewable Energy Agency (IRENA)

Follow us: Official Site, Telegram, Facebook, Instagram, Instamojo 432


Answer : D
✓ Recently, the International Renewable Energy Agency (IRENA) released the World Energy Transitions
Outlook report-2023.
✓ The global energy transition is still not on track and well short of the 1.5°C pathway.
✓ To survive 1.5°C, deployment levels must increase from 3,000 gigawatts (GW) at present to more than
10,000 GW in 2030, averaging 1,000 GW annually.
✓ Equally, more investment towards countries requires public sector intervention.
✓ Global investment in energy transformation technologies to reach a new record of $1.3 trillion in 2022.
✓ Annual investments must more than quadruple to more than $5 trillion to stay on the 1.5°C path.
✓ By 2030, cumulative investment should be US$44 trillion, with transition technologies representing 80
percent of total investment, or $35 trillion.
✓ Current pledges and plans fall well short of IRENA's 1.5°C pathway and would result in an emissions
gap of 16 gigatonnes (Gt) in 2050.
✓ About International Renewable Energy Agency (IRENA)
✓ It is an intergovernmental organisation.
✓ Founded - in Bonn, Germany on 26 January 2009.
✓ Headquarter - Abu Dhabi, United Arab Emirates.
✓ Member Countries - 167 and the European Union.
✓ India became a Member - In 2009 as 77th Founding Member.
✓ Purpose - To support countries in their transition to a sustainable energy future
Q. Under which scheme the government has sanctioned loans worth more than Rs 40,600 crore to more
than 1.8 lakh women entrepreneurs?
A) Stand Up India Scheme B) PM CARES
C) Beti Bachao Beti Padhao Scheme D) Mahila Shakti Kendras (MSK)
Answer : A
✓ The Government has sanctioned loans of more than 40,600 crore rupees to over 1.8 lakh women
entrepreneurs under the Stand Up India Scheme.
✓ Speaking on the 7th anniversary of the Stand-up India scheme, Union Finance Minister Nirmala
Sitharaman said that over 1.8 lakh women and SC/ST entrepreneurs received loans under the scheme.
✓ The scheme has created an eco-system which provides a supportive environment for setting up green field
enterprises by availing loans from bank branches of all scheduled commercial banks.
✓ About Stand Up India Scheme
✓ It was launched on April 5, 2016, to promote entrepreneurship among women, Scheduled Caste (SC)
and Scheduled Tribe (ST) categories.
✓ It has been launched to help them start a greenfield venture in manufacturing, services or business sector
and agriculture allied activities.
✓ The objective is to encourage all bank branches to extend loans to SC, ST and women for setting up their
own greenfield enterprises.
✓ The Stand-up India scheme is based on the third pillar of the National Mission for Financial Inclusion
namely "Funding the Unfunded".
✓ More than 80 per cent of the loans given under the scheme have been given to women.
✓ Eligibility for the scheme
✓ SC/ST women entrepreneurs above 18 years of age.
✓ Loans under the scheme are available only for greenfield projects.
✓ Borrower should not be in default to any bank or financial institution.

Follow us: Official Site, Telegram, Facebook, Instagram, Instamojo 433


Q. The Government has recently disbursed how much rupees to beneficiaries during 2022-23 under the PLI
Scheme for Drones and Drone Components?
A) Rs. 40 crore B) Rs. 30 crore
C) Rs. 70 crore D) Rs. 50 crore
Answer : B
✓ The Government has disbursed an amount of 30 crore rupees to beneficiaries during 2022-23 under the
PLI Scheme for Drones and Drone Components.
✓ As per Civil Aviation Ministry the Production Linked Incentive scheme for drones and drone
components in September 2021 to promote the indigenous drone industry.
✓ PLI rate is kept constant at 20% for all three years, which is an exceptional treatment for the drone
industry in the country.
✓ The Minimum value addition norm has been at 40% of net sales for drones and drone components instead
of 50% which is another exceptional treatment for the industry.
✓ The total incentive is Rs. 120 crore spread over three financial years. It is nearly double the combined
turnover of all domestic drone manufacturers in FY 2020-21
Q. The Jal Jeevan Mission has reached 60% of its countrywide target, adding 23.4 million households in the
fiscal year ended 2022-23. To achieve the target set for it of 100 per cent tap water coverage for households
by which year?
A) 2024 B) 2025
C) 2026 D) 2027
Answer : A
✓ The Jal Jeevan Mission has reached 60% of its countrywide target, adding 23.4 million households in the
fiscal year ended 2022-23,
✓ At the start of the Jal Jeevan Mission, a baseline survey showed that less than 17% or 32.3 million
households had a tap connection.
✓ Of India’s total of 190.4 million rural households, 116 million now have a functioning water tap.
✓ The country crossed another milestone in the journey towards ‘Har Ghar Jal’ on 4th April 2023, with
over 11.66 crore (60%) rural households provided with tap water supply in their homes.
✓ As on date, Gujarat, Telangana, Goa, Haryana, Punjab and the Union territories of Andaman & Nicobar
Islands, Daman Diu & Dadra Nagar Haveli and Puducherry have achieved 100% coverage.
✓ Under the mission, 2,078 water testing labs have been installed, of which 1,122 are accredited with
National Accreditation Board for Testing and Calibration Laboratories.
✓ The Jal Jeevan Mission (JJM), a flagship program of the Government of India, was launched on 15,
August 2019, by Prime Minister Narendra Modi.
✓ The Jal Jeevan Mission is one of the largest infrastructure-building programmes in the country, which
involves installing source-to-household water tap connections.
✓ Jal Jeevan Mission Launched in 15 August 2019, a flagship programme of the Modi government, aims
to provide tap water connections to rural households by 2024
Q. The Government of India has set the vision to become a one trillion dollar tourism economy by which
year?
A) 2023 B) 2047
C) 2030 D) 2045
Answer : B
✓ India aims to become one trillion dollar tourism economy by 2047.

Follow us: Official Site, Telegram, Facebook, Instagram, Instamojo 434


✓ The Government of India has set the vision to make India a one trillion dollar tourism economy by the
year 2047, which marks India’s 100th year of Independence.
✓ In line with this, India is observing ‘Visit India Year 2023′ in order to give a boost to the uniqueness of
Indian tourism and make India a seamless , modern, and smart tourism destination.
✓ The ‘Visit India Year 2023’ is a unique, collective movement that invites the world to explore India in
2023, the historical year of India’s G20 presidency.
✓ The year 2023 also marks the grand celebrations of 75 years of Indian Independence – India@75 Azadi
ka Amrit Mahotsav
Q. RBI has imposed monetary penalty of Rs 55 lakh on which bank recently for non-compliance with certain
provisions of the RBI (Know Your Customer) Directions, 2016?
A) Indian Bank B) Kotak Mahindra Bank
C) Indian Overseas Bank D) Bank of Maharashtra
Answer : A
✓ The Reserve Bank of India (RBI) imposed a fine of Rs 55 lakh on Indian Bank on April 06, 2023, for
non-compliance with certain provisions of the Reserve Bank of India (Know Your Customer) Directions,
2016.
✓ RBI has imposed this penalty in the exercise of powers vested in RBI under the provisions of section 47
A (1) (c) read with sections 46 (4) (i) and 51 (1) of the Banking Regulation Act, 1949.
✓ Indian Bank public sector bank, established in 1907 and headquartered in Chennai.
✓ Indian Bank
✓ Founded : 1907
✓ Headquarters : Chennai, Tamil Nadu
✓ MD & CEO : Shanti Lal Jain

Q. What is the rank of India in terms of number of billionaires as per the 2023 Forbes Billionaires list?
A) fifth B) third
C) fourth D) tenth
Answer : B
✓ The Forbes Billionaire List 2023 reveals that 2,640 billionaires of the world listed in the ranking hails
from 77 countries or territories.
✓ According to Forbes, the United States has topped the list for having the highest number of billionaire.
✓ The US has 735 billionaires in the ranking, similar to 2022.
✓ China stood second in terms of the number of billionaires, at 495, compared to 539 in 2022.
✓ India is on third spot in the rankings with 169 billionaires.
✓ Germany is on 4th place with 126 billionaires, while Russia with 105 billionaires has 5th highest
billionaires.
Q. Which institution has launched India’s first-ever Real Estate Investment Trusts (Reits) and Infrastructure
Investment Trusts (InvITs) Index?
A) Reserve Bank Innovation Hub
B) National Stock Exchange
C) Reserve Bank Information Technology Private Limited
D) Indian Financial Technology and Allied Services
Answer : B

Follow us: Official Site, Telegram, Facebook, Instagram, Instamojo 435


✓ NSE Indices Ltd, a subsidiary of National Stock Exchange (NSE), has launched India’s first-ever Real
Estate Investment Trusts (Reits) and Infrastructure Investment Trusts (InvITs) Index.
✓ The new index— Nifty Reits and InvITs index—aims to track the performance of Reits and InvITs that
are publicly listed and traded on the NSE.
✓ Objective – To track the performance of REITs and InvITs that are publicly listed and traded on the NSE
✓ A Real Estate Investment Trust (REIT) or an Infrastructure Investment Trust (InvIT) is an investment
vehicle that owns revenue-generating real estate or infrastructure assets.
✓ What is REIT :
✓ They are like Mutual Funds who pool the money of the investor and invest into real estate like Malls,
shopping complexes, office buildings etc. The money they earn through rents is distributed amongst the
investors in the form of dividends. Also, if the value of the invested real estate’s increases then the
investors will also benefit.
✓ They help an investor invest in Real estate with as little as Rs 10,000-Rs15, 000.
✓ Infrastructure Investment Trust (InvIT) :
✓ They are like Mutual Funds who pool the money of the investor and invest into Infrastructure, roads,
pipelines, power plants etc. They offer regular dividends and the benefit of capital appreciation.
✓ Both REIT and InvIT are regulated by SEBI.
✓ Money market is regulated by RBI while the Capital market is regulated by SEBI.
✓ National Stock Exchange of India (NSE).
✓ NSE was established in the year 1992 as the first dematerialized electronic exchange in the country.
✓ The National Stock Exchange of India Limited (NSE) is the largest financial market in India.
✓ Nifty 50 is the flagship index of the National Stock Exchange of India Limited (NSE).
✓ NIFTY 50 index was launched by NSE in the year 1996.
✓ Headquarters - Mumbai, Maharashtra
✓ Managing Director & CEO - Ashish Kumar Chauhan
Q. Which social media platform has launched a new campaign called “Stay Safe”?
A) Instagram B) WhatsApp
C) Twitter D)Telegram
Answer : B
✓ WhatsApp has launched a new campaign called “Stay Safe with WhatsApp,” which highlights the
several safety features that are pre-built on the messaging app.
✓ The campaign will run for a total of three months with a focus on several safety-centric features like
‘Block and Report,’ ‘Two-Step Verification,’ and ‘Privacy and group settings.’
✓ The campaign highlights why it is important to add two-step verification to WhatsApp, which requires a
6-digit pin and an OTP to authenticate a user.
✓ WhatsApp
✓ Founded : February 24, 2009
✓ Founders : Jan Koum & Brian Acton
✓ Headquarters : Mountain View, California, US
✓ CEO : Will Cathcart
✓ WhatsApp Parent organization: Facebook
Q. Citi Bank and the Japan International Cooperation Agency (JICA) have joined hands to provide a co-
financing loan to which bank?
A) Yes Bank B) HDFC Bank
C) IndusInd Bank D) Kotak Mahindra Bank
Answer : C

Follow us: Official Site, Telegram, Facebook, Instagram, Instamojo 436


✓ Citi Bank and the Japan International Cooperation Agency (JICA) have joined hands to provide a co-
financing loan to IndusInd Bank Limited, a private commercial bank.
✓ This includes a $30 million loan by Citi and a ¥13 billion loan by JICA.
✓ This high-impact social finance offering that Citi arranged for IndusInd Bank is expected to improve
financial access for farmers and catalyae capital investment in the agricultural sector in India.
Q. Which bank has recently launched “Parivar” (family) savings account in FY24 to minimize the gap
between deposit growth and credit growth?
A) Kotak Mahindra Bank B) Punjab National Bank
C) State Bank of India D) Canara Bank
Answer : C
✓ State Bank of India (SBI) is planning to launch new variants of current account (with balances of ₹50,000
and ₹50 lakh) and “Parivar” (family) savings account in FY24 to minimize the gap between deposit
growth and credit growth.
✓ Domestic deposits of India’s largest bank grew 8.86 percent year-on-year (y-o-y) by the end of 2022, while
domestic advances were up 16.91 percent y-o-y.
✓ SBI expects that its domestic deposits and domestic advances will grow about 12 per cent and 16 per cent
y-o-y, respectively, in FY24.

Q. Who has been appointed as the new CEO & MD of TCS to replace Rajesh Gopinathan?
A) Aarthi Subramanian B) Pradeep Kumar Khosla
C) N. Chandrasekaran D) K Krithivasan
Answer : D
✓ Tata Consultancy Services (TCS) has appointed K Krithivasan as its new Chief Executive Officer (CEO)
and Managing Director of the company.
✓ He will take over the post with effect from June 01, 2023 for a tenure of five years.
✓ Krithivasan will replace outgoing CEO Rajesh Gopinathan, who was appointed in 2017.
Q. Vishal Kanvaty has recently been appointed as the Chief Technology Officer (CTO) of which
organisation?
A) AAI B) ICAR
C) NITI Aayog D) NPCI
Answer : D
✓ The National Payments Corporation of India (NPCI) has roped in Vishal Kanvaty as Chief Technology
Officer (CTO).
✓ Kanvaty joined NPCI in 2017.
✓ Prior to this he managed the portfolio of Chief Market Innovation as well as Product and Innovations
profiles
Q. Recently, the bill to amend the Competition Act received President Droupadi Murmu’s assent. The
amended act will replace which act?
A) Competition (Amendment) Act, 2004 B) Competition (Amendment) Act, 2002
C) Competition (Amendment) Act, 2007 D) Competition (Amendment) Act, 2009
Answer : D

Follow us: Official Site, Telegram, Facebook, Instagram, Instamojo 437


✓ The bill to amend the Competition Act has received the assent of President Droupadi Murmu. The bill
was cleared by Parliament on April 3, 2023.
✓ The Competition (Amendment) Act, 2023 is an act to further amend the Competition Act, 2002.
✓ The legislation for the Competition Act was brought in 2002 and subsequently, it underwent amendments
in 2007 and 2009.
✓ This is the first time since the enforcement of the Competition Act in 2009 that amendments are being
made to the Act.
✓ Whats there in the updated legislation?
✓ It ensures regulatory certainty and fosters a trust-based business environment.
✓ It provides for faster timelines for approval of M&A filings.
✓ It introduces settlement and commitment framework for faster market correction.
✓ It incentivises parties in an ongoing cartel investigation in terms of lesser penalty to disclose information
regarding other cartels.
✓ It provides certainty to stakeholders by prescribing limitation period of three years for filing information
on anti-competitive agreements and abuse of dominant position.
✓ It also introduces “value of transaction” as another criteria for notifying mergers and acquisitions to CCI.

Q. Which organization publishes the “World Economic Outlook” (WEO) report ?


A) World Intellectual Property Organization (WIPO)
B) The Economist Intelligence Unit
C) Transparency International
D) International Monetary Fund (IMF)
Answer : D
✓ The latest World Economic Outlook Report by the International Monetary Fund (IMF) has lowered
India’s growth projection for 2023-24 (FY24) to 5.9 percent from 6.1 percent projected earlier.
✓ For 2024-2025 (FY25) – 6.3 percent
✓ For 2023-2024 (FY24) – 5.9 percent
✓ For 2022-23 (FY23) – 6.8 percent
✓ In case of global economy, the projection are as follows:
✓ 2022 – 3.4 percent
✓ 2023 – 2.8 percent
✓ 2024 – 3.0 percent

Q. RBI has recently approved the re-appointment of Sanjay Agarwal as the MD & CEO of which bank?
A) AU Small Finance Bank B) Paytm Payments Bank
C) Bank of Maharashtra D) ESAF Small Finance Bank
Answer : A
✓ The Reserve Bank of India has approved the re-appointment of Sanjay Agarwal as managing director
(MD) and CEO of AU Small Finance Bank.
✓ His re-appointment is valid with effect from April 19, 2023, for a period of three years, that means till
April 18, 2026.
✓ RBI has also cleared the re-appointment of Uttam Tibrewal as the whole-time director of the bank for a
period of three years.
✓ AU Small Finance Bank Limited
✓ Founded :1996
✓ Founder : Sanjay Agarwal
✓ Headquarters : Jaipur, Rajasthan

Follow us: Official Site, Telegram, Facebook, Instagram, Instamojo 438


✓ MD & CEO : Sanjay Agarwal
✓ Tagline : Chalo Aage Badhein
Q. Which bank has signed a "Master Inter Bank Credit Agreement" with the Export-Import Bank of Korea
for a USD 300 million line of credit?
A) HDFC Bank B) Yes Bank
C) ICICI Bank D) Kotak Mahindra Bank
Answer : A
✓ HDFC Bank has signed a “Master Inter Bank Credit Agreement” with Export-Import Bank of Korea for
a USD 300 million line of credit.
✓ The pact was signed by the two entities at GIFT City in Gujarat’s capital Gandhinagar.
✓ This agreement will help HDFC Bank raise foreign currency funds which it would extend to Korea-
related businesses.

Q. Which company has become the first Indian company to have over 50 lakh shareholders?
A) Axis Bank B) HDFC Bank
C) ICICI Bank D) Yes Bank
Answer : D
✓ Private sector lender YES Bank has become the first Indian company to have over 50 lakh shareholders.
✓ Tata Power was a far second with 38.5 lakh shareholders, followed by Reliance Industries with 33.6 lakh
shareholders, according to the December 2022 shareholding disclosures.
✓ The bank had 50.6 lakh shareholders as of March 31, 2023, all of whom were public shareholders.
✓ YES BANK
✓ Founded : 2004
✓ HQ : Mumbai, Maharashtra
✓ MD & CEO : Prashant Kumar
✓ Tagline : Experience our Expertise
Q. Which Country Gazprombank has expanded its ties with banks in India to expedite trade between the
two countries in national currencies?
A) China B) Russia
C) United States D) Germany
Answer : B
✓ Russia’s Gazprombank has expanded its links with banks in India to expedite trade between the two
countries in national currencies, a key executive told Reuter as Russia this year has become the biggest
supplier of oil to India.
✓ Trade between India and Russia has surged since the West imposed sanctions against Russia for its
invasion last year of Ukraine, which has altered flows of oil and other goods.
✓ Gazprombank is Russia’s third-largest lender by assets and a key conduit of the Russian energy trade
Q. Who has been appointed as the head of the advisory board formed by fintech Razorpay to strengthen the
corporate governance of the company?
A) Viral Acharya B) Arvind Subramanian
C) NS Vishwanathan D) Raghuram Rajan
Answer : C

Follow us: Official Site, Telegram, Facebook, Instagram, Instamojo 439


✓ Fintech unicorn Razorpay has set up an advisory board that will focus on setting up standards to
strengthen customer experience, governance, compliance, and risk management practices.
✓ NS Vishwanathan, the former deputy governor of Reserve Bank of India (RBI) has been roped in as the
chairperson of this 4-member advisory board.
✓ The other member of this board are:
✓ Arijit Basu – former managing director of State Bank of India;
✓ Aruna Sundararajan – former telecom secretary
✓ K.P. Krishnan – former secretary, Ministry of Skill Development and Entrepreneurship.
Q. How many crores of projects have been approved under the National Mission for Clean Ganga?
A) Rs 820 Crore B) Rs 761 Crore
C) Rs 705 Crore D) Rs 638 Crore
Answer : D
✓ 48th meeting of the Executive Committee of the National Mission for Clean Ganga (NMCG) was held
on 18th April under the chairmanship of Director General, National Mission for Clean Ganga (NMCG)
G. Asok Kumar.
✓ Eight projects worth around Rs. 638 crore were approved in the meeting.
✓ In an attempt to clean River Hindon, which is a tributary of River Yamuna, four projects worth Rs 407.39
crore were approved for pollution abatement in the Shamli district.
✓ These projects are part of the comprehensive Hindon Rejuvenation Plan.
✓ Hindon river is identified as Priority 1 polluted river stretch.
✓ The projects which were sanctioned are to prevent theflow of polluted water into Krishna River.
✓ Krishni is one of the major tributaries of Hindon which discharges pollution from Shamli district into
River Hindon.
✓ Two more sewerage management projects were approved in the meeting, one each in Bihar and Madhya
Pradesh.
✓ These projects will arrest the flow of polluted water into theKiul River, a tributary of the Ganges.
✓ .About Namami Gange
✓ It was approved as a 'Flagship Programme' by the Central Government in June 2014.
✓ It was launched to serve the twin objectives of effective abatement of pollution and conservation and
rejuvenation of the national river Ganga.
✓ It is being operated under the Ministry of Water Resources, Department of River Development and
Ganga Rejuvenation and Ministry of Jal Shakti.
✓ The program is being implemented by the National Mission for Clean Ganga (NMCG) and its State
Program Management Groups (SPMGs).
Q. Which State has bagged the national award for the best implementation of the Pradhan Mantri Fasal
Bima Yojana (PMFBY) in the country?
A) Maharashtra B) Karnataka
C) Uttar Pradesh D) Madhya Pradesh
Answer : B
✓ For the first time, Karnataka has bagged the national award for the best implementation of Pradhan
Mantri Fasal Bima Yojana (PMFBY) in the country.
✓ According to the authorities, between the kharif season 2021 and 2022, the state recorded a 47.74%
increase in the enrolment of farmers under the scheme.
✓ While the area covered under the scheme increased by 41.80%. The award was given to the state on 14
April 2023.

Follow us: Official Site, Telegram, Facebook, Instagram, Instamojo 440


✓ According to the data shared by the officials of the department of agriculture, in the kharif season 2021,
16.15 lakh farmers had enroled under the scheme.
✓ About 'Pradhan Mantri Fasal Bima Yojana' (PMFBY) .
✓ The Pradhan Mantri Fasal Bima Yojana (PMFBY) was launched on 13 January 2016.
✓ The scheme provides a comprehensive insurance cover to the farmers in the event of crop failure, which
helps in stabilising the income of the farmers.
✓ This includes all food and oilseed crops and annual commercial/horticultural crops for which past yield
data is available.
✓ Central and state governments pay more than 95% of the premium amount, while farmers bear 1.5-5%
of the premium amount.
✓ A 2% premium will be paid for Kharif crops (Paddy or Rice, Maize, Jowar, Bajra, Sugarcane etc.).
✓ For Rabi crops (wheat, barley, gram, lentil, mustard etc.) 1.5% premium will be paid.
✓ A 5% premium will be paid for annual commercial and horticultural crops insurance.
Q. Which Fintech company has launched its UPI-based peer-to-peer (P2P) payments, providing its users
with a new payment option?
A) CRED B) Freecharge
C) Paytm D) Razorpay
Answer : A
✓ Fintech major CRED has launched its UPI-based peer-to-peer (P2P) payments, providing its users with
a new payment option.
✓ The feature allows CRED members access to ‘pay anyone’ through their contact list, phone numbers, or
UPI IDs.
✓ The new payment experience has been introduced after the launch of Scan & Pay in October 2022. As
scan and pay gain traction, CRED expects the number of merchants to grow.
✓ Now with the launch of P2P UPI payments, CRED members will have multiple payment options
including offline payments (UPI P2P, Scan & Pay, Tap to Pay), online merchant payments (CRED Pay,
CRED flash) and bill payments.
Q. Which bank has recently launched India’s 1st Voice Biometric Authentication Banking App?
A) Canara Bank
B) City Union Bank
C) Indian Overseas Bank
D) Bank of Baroda
Answer : B
✓ City Union Bank (CUB) launched India's first Voice Biometric Authentication banking app for its mobile
banking app which uses it to improve security.
✓ The purpose of this feature is to provide an additional layer of security to customers while logging into
their accounts.
✓ Currently the Voice Biometric Authentication login option is available only for Mobile Banking App
users. Customers will have multiple authentication methods to choose from, including User ID/PIN,
Face ID, fingerprint authentication and now voice biometric authentication.
✓ The Voice Biometric Authentication feature was developed in collaboration with M/s Kaizen Secure
Voice Pvt Ltd, a Chennai based start-up and 5G Use Case Lab of Institute for Development and Research
in Banking Technology (IDRBT).
✓ The launch of the Voice Biometric Authentication facility came a day after CUB reported its provisional
business figures, in which the total business increased from Rs 88,846 crore to Rs 96,347 crore.

Follow us: Official Site, Telegram, Facebook, Instagram, Instamojo 441


Q. Which bank has reintroduced 400 days ‘Amrit Kalash’ retail term fixed deposit (FD) scheme?
A) State Bank of India B) Punjab National Bank
C) Indian Overseas Bank D) Kotak Mahindra Bank
Answer : A
✓ The country’s largest lender State Bank of India (SBI) has extended the validity of Amrit Kalash special
fixed deposit (FD) scheme.
✓ The bank previously introduced this retail term deposit programme, and it was valid from February 15,
2023, through March 31, 2023. But on April 12, 2023, SBI stated that its Amrit Kalash FD retail term
deposit programme has been extended till 30th June 2023.
✓ SBI Amrit Kalash FD Scheme comes with a special tenor of 400 days on which the general public will
get an interest rate of 7.10% and senior citizens will get an interest rate of 7.60% which is 50 bps higher
than the standard applicable rate and senior citizens, staff and staff pensioners are eligible for additional
interest rate applicable to them.
Q. Which organization has launched a pilot scheme “Mission 50K-EV4ECO” for better financing terms in
the electric vehicle space and strengthening the whole ecosystem?
A) NPCI B) SEBI
C) NABARD D) SIDBI
Answer : D
✓ Small Industries Development Bank of India (SIDBI) has launched a pilot scheme for better financing
terms in the electric vehicle space and strengthening the whole ecosystem.
✓ The pilot phase of ‘Mission 50K-EV4ECO’ aims at strengthening the EV ecosystem, including uptake
for two, three, and four-wheelers through direct and indirect lending.
✓ The pilot scheme, which is the precursor to EVOLVE scheme by SIDBI-World Bank, has two
components — direct lending and indirect lending.
✓ Direct lending: SIDBI will directly give loans to eligible MSMEs.
✓ Indirect lending: Targeted at NBFCs, including small unrated focused, and emerging NBFCs.
✓ Small Industries Development Bank of India (SIDBI)
✓ Founded : 2 April 1990
✓ Headquarteres : Lucknow, Uttar Pradesh
✓ Chairman & MD : Siva S Ramann
Q. Which company has partnered with 20 South Korean firms to establish an electronics manufacturing hub
in India?
A) Tata Consultancy Services B) Wipro
C) Infosys D) Vedanta Group
Answer : D
✓ Vedanta Group has partnered with 20 South Korean firms to establish an electronics manufacturing hub
in India.
✓ The Indian firm is also setting up a semiconductor plant in Gujarat in a joint venture (JV) with Foxconn.
✓ The partnership aims to boost India’s electronics manufacturing industry and reduce dependence on
imports.
✓ The hub will be set up in Tamil Nadu and will focus on manufacturing products such as smartphones,
TVs, and other electronic devices.

Follow us: Official Site, Telegram, Facebook, Instagram, Instamojo 442


Q. Which city has emerged as one of the top digital payment transactions cities in 2022 in the country, a
report by payment services firm Worldline India?
A) Delhi B) Mumbai
C) Chennai D) Hyderabad
Answer : C
✓ Chennai has emerged as one of the top digital payment transaction cities in the country in 2022, according
to a report by payment services firm Worldline India.
✓ In terms of volume, according to Worldline India, Chennai has done 14.3 million transactions with a
value of USD 35.5 billion. It is at number five.
✓ Bengaluru tops with 29 million transactions worth USD 65 billion, followed by New Delhi 19.6 million
transactions worth USD 50 billion, Mumbai (18.7 million transactions worth USD 49.5 billion), Pune
(15 million transactions worth USD 32.8 billion).
✓ In 2022, frequently visited physical merchant categories like grocery stores, restaurants, clothing and
apparel, pharmacy, household appliances together accounted for over 43 per cent in terms of volume and
about 40 per cent in terms of value, across the country.
✓ E-commerce space, gaming, utility and financial services contributed to over 85 per cent of transactions
and 25 per cent in terms of value.
✓ Education, travel and hospitality sector accounted for 15 per cent in volume and 75 per cent in terms of
value
✓ What is a Digital Payment System?
✓ A digital payment is sometimes called an electronic payment.
✓ It is the transfer of value from one payment account to another using a digital device such as a mobile
phone, POS (Point of Sales) or computer.
✓ Modes of Digital Payments
✓ Unified Payments Interface (UPI)
✓ Bharat Interface for Money (BHIM)
✓ UPI 123PAY
✓ UPI Lite
✓ Cards (RuPay Debit Cards)
✓ Immediate Payment Services (IMPS)
✓ Aadhaar Enabled Payment System (AePS)
Q. AS per the Hurun Global Unicorn Index 2023, what is the rank of India in the number of startup
unicorns?
A) third
B) sixth
C) fourth
D) fifth
Answer : A
✓ As per the Hurun Global Unicorn Index 2023, India has retained its position as the third-largest hub for
startup unicorns with total 68 unicorns. This is up by 14 compared to previous year.
✓ The United States has the highest number of unicorns. It is followed by China at second place.
✓ Israel and Russia are among top five respectively.
✓ The edtech firm BYJU’S has been named as India’s top unicorn with a valuation worth US$22 billion.
✓ It is followed by foodtech platform Swiggy and fantasy sports platform Dream11 with a valuation of
US$8 bn each.

Follow us: Official Site, Telegram, Facebook, Instagram, Instamojo 443


Q. The Asian Infrastructure Investment Bank (AIIB) has planned to set up its first overseas office in which
city?
A) New Delhi B) Abu Dhabi
C) Bangkok D) Seoul
Answer : B
✓ The Asian Infrastructure Investment Bank (AIIB) has signed a host member agreement with the United
Arab Emirates (UAE) to officially open its first overseas office in Abu Dhabi.
✓ AIIB Abu Dhabi will serve as an Interim Operational Hub (the Hub).
✓ The AIIB Hub in the UAE will give a robust platform to the bank to manage growing investment
portfolio, enhance client and member engagement, project monitoring and implementation services
across the globe.
✓ Asian Infrastructure Investment Bank (AIIB)
✓ Founded : 2016
✓ HQ : Beijing, China
✓ President : Jin Liqun
Q. Power Finance Corporation Ltd (PFC) has sanctioned how much rupees loan to Gensol Engg for the
purchase of 5,000 passenger electric vehicles (EVs) and 1,000 cargo EVs?
A) Rs 633 crore B) Rs 893 crore
C) Rs 567 crore D) Rs 779 crore
Answer : A
✓ PFC sanctions Rs 663 crore loan to Gensol Engineering for purchase of Electric Vehicles.
✓ Power Finance Corporation (PFC), a Maharatna company and NBFC in the Indian power sector has
sanctioned a loan of Rs 633 crore to Gensol Engineering, for the purchase of 5000 passenger Electric
Vehicles (EVs) and 1000 cargo EVs.
✓ PFC is a leading NBFC in the Indian power sector. The purchased passenger EVs will be leased to
Blusmart Mobility Pvt. Ltd (BMPL) to expand its fleet of ride-hailing cabs.
✓ The first tranche of the loan has already been disbursed, and the first lot of EV cabs have hit the Delhi
roads.
✓ 5000 Passenger E4W (Electric Four-Wheelers) funded by PFC are being deployed in Delhi and will result
in CO2 emission reduction.
✓ With a view to accelerate India’s net-zero target, PFC is exploring opportunities in debt financing of EVs
(OEMs and Fleet Acquisition), battery OEMs and EV charging infrastructure, apart from large-scale
renewable energy financing.
Q. The Ministry of Civil Aviation launched which version of Regional Connectivity Scheme (RCS) UDAN
in April 2023?
A) UDAN 5.0 B) UDAN 4.0
C) UDAN 3.0 D) UDAN 2.0
Answer : A
✓ The Ministry of Civil Aviation launched the fifth round of the Regional Connectivity Scheme (RCS)
UDAN 5.0 on April 21, 2023
✓ The fifth round of Ude Desh Ka Aam Nagrik (UDAN) or Regional Connectivity Scheme (RCS), will
focus on Category-2 (20-80 seats) and Category-3 (more than 80 seats) aircraft operations.
✓ The 600 km limit of the first phase has been done away with and there is no restriction for the distance
between the origin and destination of the flight.

Follow us: Official Site, Telegram, Facebook, Instagram, Instamojo 444


✓ The Viability Gap Funding (VGF) to be provided will be capped at a distance of 600 km for priority and
non-priority areas, which was 500 km.
✓ UDAN 5.0 will connect more than 50 wildlife and tourist destinations.
✓ The scheme was launched in 2017 to enhance the connectivity to remote and regional areas of the country
through air service.
✓ The first aircraft under UDAN Scheme was launched from Shimla to Delhi in April 2017.
✓ About UDAN scheme
✓ Ude Desh Ka Aam Nagrik (UDAN) was launched in 2016 as a Regional Connectivity Scheme (RCS)
under the Ministry of Civil Aviation.
✓ Its objective is to provide affordable, economically viable and profitable air travel on regional routes to
the common man even in small towns.
Q. Sumant Kathpalia has recently been appointed as the MD & CEO of which bank?
A) Bank of India B) Kotak Mahindra Bank
C) Canara Bank D) IndusInd Bank
Answer : D
✓ The Board of Directors of the private lender IndusInd Bank has reappointed Sumant Kathpalia as the
managing director and chief executive officer (MD & CEO) of IndusInd Bank Limited for a period of
two years with effect from 24 March, 2023.
✓ Kathpalia was first appointed as the MD & CEO of IndusInd Bank in March 2020.
✓ INDUSLND BANK
✓ Founded : 1994
✓ Founder : S.P. Hinduja
✓ Headquarters : Pune, Maharashtra,
✓ MD & CEO : Sumant Kathpalia
✓ Tagline : We Make You Feel Richer
Q. Reliance Retail has recently formed a Joint Venture with which company for local manufacturing of toys
for its toy business?
A) Buddha Crafts B) Circle E Retail
C) Concept Toys D) Funcorp
Answer : B
✓ Reliance Retail, a subsidiary of Reliance Industries and one of the leading toy distributors, has entered
into a joint venture with a Haryana-based firm, Circle E Retail, for local manufacturing of toys, to
vertically integrate its toy business.
✓ The brand currently owns iconic British toy brand Hamleys and home-grown toy brand Rowan.
✓ Reliance Retail will have control over the entire aspects of the toy ecosystem, right from designing and
manufacturing to retailing of the product.
✓ This new business venture will cater for both the toys brands, Hamleys and Rowan, owned by Reliance Retail.
✓ Circle E Retail has a specialisation in toy manufacturing, having modern manufacturing unit in Haryana
with a licence to manufacture and distribute a wide range of toys.
Q. What percent of stake each will be acquired by HDFC and AXIS bank in the GoDigit Life Insurance
company?
A) 9.94% B) 9.12%
C) 9.99% D) 9.43%
Answer : A

Follow us: Official Site, Telegram, Facebook, Instagram, Instamojo 445


✓ Private sector lenders Axis Bank and HDFC Bank have signed definitive agreements to acquire up to a
9.94 percent stake each in insurance tech startup GoDigit Life Insurance.
✓ Both the banks will invest ₹69.90 crore each in two tranches.
✓ The banks have already made the payment of first tranche of Rs 10.93 crore each
Q. Which bank has launched “Digitalised Submission of Form 15G/15H”, in collaboration with Reserve
Bank Innovation Hub (RBIH), to make it easier for senior citizens to submit their Tax waiver forms
through SMS and Website?
A) IndusInd Bank B) Bank of Baroda
C) Punjab National Bank D) Canara Bank
Answer : D
✓ Canara Bank Partners Reserve Bank Innovation Hub Leveraging Technology to simplify 15 G-15H
Submission for senior citizens.
✓ Bangalore-based public sector lender Canara Bank, has launched “Digitalised Submission of Form
15G/15H”, in collaboration with Reserve Bank Innovation Hub (RBIH).
✓ The new solution will enable all Deposit
✓ holders, including Senior citizens, to easily submit their Tax waiver forms through SMS and Website
(with their registered mobile number), Any Time Any Where.
✓ Shri Shaktikanta Das, Governor of RBI has inaugurated the Reserve Bank Innovation Hub (RBIH) on
March 24, 2022 in Bengaluru, Karnataka
✓ RBI has set up the RBIH with an initial capital contribution of Rs. 100 crore.
✓ The objective of RBIH is to encourage and nurture financial innovation in a sustainable manner through
an institutional set-up. It has been setup as a Section 8 company under Companies Act, 2013.
✓ Reserve Bank Innovation Hub (RBIH) is one of the five wholly-owned subsidiaries of the RBI.
✓ Canara Bank:
✓ It is a leading commercial bank of India.
✓ It was established in 1906, by Sri Ammembal Subba Rao Pai.
✓ Syndicate Bank was merged with Canara Bank on 1 April 2020. After this merger, Canara Bank has
become the fourth largest bank in the country.
✓ Headquarters Bangalore.
✓ K. Satyanarayana Raju has taken charge as Managing Director & CEO of Canara Bank
Q. RBI approves re-appointment of whom as Managing Director and CEO of City Union Bank?
A) K G Mohan B B) Atul Kumar Goel
C) Sashidhar Jagdishan D) N Kamakodi
Answer : D
✓ Reserve Bank of India (RBI) has approved the reappointment of N. Kamakodi as Managing Director and
CEO for a period of three years with effect from May 1, 2023.
✓ The RBI has given its approval vide letter dated April 26, 2023, for the appointment of N. Kamakodi as
MD of the bank, City Union Bank said in a regulatory filin.
✓ City Union Bank Limited is an Indian private sector bank headquartered in Kumbakonam, Tamil Nadu.
✓ City Union Bank
✓ Founded : 1904
✓ Headquarters : Kumbakonam, Tamil Nadu
✓ Chairman : R. Mohan
✓ MD & CEO : N. Kamakodi
✓ Tagline : Trust and Excellence since 1904

Follow us: Official Site, Telegram, Facebook, Instagram, Instamojo 446


BEST MCQ WITH FACTS
SEPTEMBER 2023

Q. In India, which city has been inaugurated as the first solar city of the country?
A) Alirajpur B) Barwani
C) Sanchi D) Nagda
Answer : C
✓ Sanchi Nagar of MP inaugurated as first solar city of India.
✓ Chief Minister of Madhya Pradesh, Shivraj Singh Chouhan inaugurated Sanchi Nagar, in the Raisen
district of Madhya Pradesh as the first solar city of India.
✓ Sanchi Nagar is a World Heritage town. In the programme, an agreement was signed between the state
department of renewable energy and IIT Kanpur to make Sanchi a net zero city.
✓ Sanchi Solar City will reduce the emissions of more than 14,000 tonnes of carbon dioxide annually,
which is equivalent to more than 238,000 trees.
INDIA'S FIRST IN NEWS 2023
▪ Telangana has launched India’s first Agricultural Data Exchange (ADeX) and Agriculture Data
Management Framework (ADMF) in Hyderabad.
▪ India's first-ever sports university to be set up in Manipur.
▪ India Launches Its 1st Indigenous e-Tractor Developed By the CMERI.
▪ Dr Jitendra Singh, had launched the CSIR Prima ET11, the first indigenous e-Tractor developed by the
Central Mechanical Engineering Research Institute (CMERI), Durgapur, a public engineering R&D
institution in Durgapur, West Bengal, and a constituent laboratory of the Indian Council of Scientific
and Industrial Research (CSIR).
▪ Kerala Launches India's First AI School, Pioneering Future-Focused Education.
▪ Sanchi, a world heritage site located in the Raisen district of Madhya Pradesh, is all set to become India's
first solar city.
▪ Hitachi Payment Services launched India's first UPI-ATM.
▪ India’s first Vedic-themed park unveiled in Noida.
▪ Google celebrates India's street food 'pani puri' with an interactive game Doodle, allowing users to help
a street vendor fulfill pani puri orders.
▪ OPPO India sets up first PPP-model Atal Tinkering Lab in Kerala.
▪ Kerala gets India’s first Construction Innovation Hub.
▪ Uttar Pradesh government announced that the Buddh international circuit in Greater Noida will host
India's first MotoGP racing event.
▪ India's First ‘Sahiwal’ Breed Calf Given Birth Through ‘Cow Surrogacy’ At TTD's Goshala In Tirupati.
▪ Recently, the PM has inaugurated India’s first facility to produce rare earth permanent magnets in
Visakhapatnam.
▪ India's first online gaming academy launched in Madhya Pradesh.
▪ Kerala to set up India's first fisheries Atal Incubation Centre.
▪ India’s first International Multi Modal Logistics park in Jogighopa, Assam.
▪ India’s First Undersea Tunnel in Mumbai Set to Open by November 2023.
▪ India's first private hill station Lavasa was sold to Darwin Platform Infrastructure for Rs 1.8 thousand
crore after the approval of the National Company Law Tribunal (NCLT).
▪ According to the Kolkata Metro Rail Corporation, India's first underwater metro service, the East-West
Metro Corridor project is expected to be completed by December 2023.
▪ Tech startup firm IG Drones has developed India's first 5G-enabled drone capable of vertical take-off and
landing. This drone has been named Skyhawk.

Follow us: Official Site, Telegram, Facebook, Instagram, Instamojo 447


▪ Union Minister for Road Transport and Highways Nitin Gadkari unveiled the country's first drone-air
traffic management system "Sky UTM".
▪ India’s First Centre of Excellence in Online Gaming to be set up in Shillong, Meghalaya.
▪ Luminous Power Technologies announced its plan to build the country's first green energy-based solar
panel manufacturing factory in Uttarakhand.
▪ India’s First AC Double Decker Electric Bus Introduced in Mumbai.
▪ NCERT Launched India’s First National Assessment Regulator “PARAKH”.
▪ PARAKH stands for The Performance Assessment, Review, and Analysis of Knowledge for Holistic
Development.
▪ Uttar Pradesh on track to be first Indian state with 100% EVs in govt depts.
▪ Shivangi Singh being the first woman pilot to fly the Rafale fighter aircraft, is a trailblazer in the Indian
Air Force.
▪ India’s first “Water Metro” services will be inaugurated by the Prime Minister Narendra Modi on April
25, 2023 in Kochi, Kerala.
▪ Mana village on India-China border in UK’s Chamoli district declared as ‘India’s first village’ by BRO.
▪ Narendra Modi has laid the foundation stone for India’s first Digital Science Park coming up at the
Technocity campus in Thiruvananthapuram, Kerala.
▪ PM Modi laid the foundation stone of the India’s first urban transport ropeway on March 24, 2023, in
Varanasi.
▪ City Union Bank (CUB) launched India's first Voice Biometric Authentication banking app for its mobile
banking app which uses it to improve security.
▪ India’s first semi high-speed regional rail services in NCR named ‘RAPIDX’.
▪ India's first 3D-printed post office is being built in Bengaluru.
▪ NSE Indices Ltd, a subsidiary of National Stock Exchange (NSE), has launched India’s first-ever Real
Estate Investment Trusts (Reits) and Infrastructure Investment Trusts (InvITs) Index.
▪ India’s first National Centre of Excellence in Green Port & Shipping (NCoEGPS) in Gurugram,
Haryana.
▪ IPL India’s first unicorn with a $1.1 billion valuation.
▪ Northern India’s first nuclear plant to come up in Haryana’s Gorakhpur.
▪ MobiKwik becomes India's first app to Support Credit Cards on UPI.
Q. Utkela Airport has recently been inaugurated in which state by Minister of Civil Aviation, Shri
Jyotiraditya M Scindia?
A) Bengaluru B) Kerala
C) Odisha D) Andhra Pradesh
Answer : C
✓ Minister of Civil Aviation, Shri Jyotiraditya M Scindia inaugurated Utkela Airport owned by the
Government of Odisha.
✓ It has been developed as a regional airport under the UDAN scheme of the Ministry of Civil Aviation,
Government of India at a cost of Rs. 31.07 Cr.
✓ He also inaugurated a direct flight between Utkela and Bhubaneshwar that will be operated by IndiaOne.

AIRPORT IN NEWS 2023


▪ Minister of Civil Aviation, Shri Jyotiraditya M Scindia inaugurated Utkela Airport owned by the
Government of Odisha.
▪ Guwahati's Lokpriya Gopinath Bordoloi International Airport became the first airport in the northeast
to get ‘Digi Yatra’ facility.

▪ Prime Minister Narendra Modi will inaugurate Gujarat's first greenfield airport at Hirasar near Rajkot.

Follow us: Official Site, Telegram, Facebook, Instagram, Instamojo 448


▪ Government of India has given 'in-principle' approval for setting up 21 new greenfield airports.
▪ Bangalore International Airport Limited (BIAL) has launched a new app called BLR Pulse to provide
hassle-free experience to the commuters at Kempegowda International Airport.
▪ Delhi's Indira Gandhi International (IGI) Airport has launched a new facility called 'DigiYatra' which
enables passengers to avail of its services without downloading a mobile application.
▪ Lal Bahadur Shastri International Airport is the first in India to offer a free reading lounge to the
passengers, with a huge collection of litreature.
▪ Indira Gandhi International Airport in New Delhi ranked 9th busiest airports globally in 2022 , as per
Airports Council International (ACI). United States-based Atlanta airport has retained its position as the
world’s busiest airport.
▪ According to the Airport Council International (ACI), Delhi International Airport has been ranked
among the cleanest airports in Asia-Pacific.
▪ Prime Minister Narendra Modi on 27 February 2023 launched the newly built Shivamogga airport along
with various development projects worth over Rs 3,600 crore in Shivamogga, Karnataka.

Q. ISRO used which launch vehicle to successfully launch its Solar mission Aditya L1 in September 2023?
A) PSLV C-55 B) PSLV C-54
C) PSLV C-56 D) PSLV C-57
Answer : D
✓ ISRO successfully launched its Solar mission Aditya L1 (PSLV C-57) from Sriharikota, Andhra Pradesh
on September 02, 2023.
✓ Aditya-L1 is a satellite dedicated to the comprehensive study of the Sun. The spacecraft carries seven
payloads to observe the photosphere, chromosphere and the outermost layers of the Sun (the corona)
using electromagnetic and particle and magnetic field detectors, all seven are developed indigenously.
Five by ISRO and two by Indian academic institutes in collaboration with ISRO.
✓ Aditya in Sanskrit means the Sun. L1 here refers to Lagrange Point 1 of the Sun-Earth system. L1 is a
location in space where the gravitational forces of two celestial bodies, such as the Sun and Earth, are in
equilibrium. This allows an object placed there to remain relatively stable with respect to both celestial
bodies.
✓ Following its scheduled launch on September 2, 2023, Aditya-L1 stays Earth-bound orbits for 16 days,
during which it undergoes 5 maneuvres to gain the necessary velocity for its journey.
✓ Aditya-L1 will stay approximately 1.5 million km away from Earth, directed towards the Sun, which is
about 1% of the Earth-Sun distance. The Sun is a giant sphere of gas and Aditya-L1 would study the
outer atmosphere of the Sun. Aditya-L1 will neither land on the Sun nor approach the Sun any closer.

Q. What will be the distance of Aditya-L1 satellite from the earth, for the comprehensive study of the Sun?
A) 1.5 million B) 2 million
C) 0.5 million D) 1 million
Answer : A
✓ Aditya-L1 will stay approximately 1.5 million km away from Earth, directed towards the Sun, which is
about 1% of the Earth-Sun distance. The Sun is a giant sphere of gas and Aditya-L1 would study the
outer atmosphere of the Sun. Aditya-L1 will neither land on the Sun nor approach the Sun any closer.
Q. ‘Bhaderwah Rajmash and Sulai honey’, which got Geographical Indication (GI) tag, is from which
State/UT?
A) Assam B) Jammu and Kashmir
C) Rajasthan D) West Bengal

Follow us: Official Site, Telegram, Facebook, Instagram, Instamojo 449


Answer : B
✓ Bhaderwah Rajmash and Sulai honey of Doda and Ramban districts of Jammu and Kashmir, have been
granted Geographical Indication (GI) tags.
✓ In 2015, Prime Minister Narendra Modi had gifted organic Sulai honey to Queen Elizabeth during his
visit to Britain.
GI TAG IN NEWS 2023.
▪ Recently, the Salem Starch and Sago Manufacturers Service Industrial Cooperative Society Ltd.
(popularly called SAGOSERVE) received a Geographical Indication Tag, or GI Tag, for Salem sago.
▪ Salem Sago, the wet starch powder extracted from Tapioca roots in Salem district, Tamil Nadu, now
proudly bears a GI tag.
▪ Bhaderwah Rajmash and Sulai honey of Doda and Ramban districts of Jammu and Kashmir, have been
granted Geographical Indication (GI) tags.
▪ Chokuwa rice of Assam recently earned a GI (Geographical Indication) tag for its exquisiteness.
▪ The Matti banana variety, native to Kanniyakumari district was recently granted the Geographical
Indication (GI) tag.
▪ Mushkbudji rice, which was granted the GI tag along with eight other products from Jammu and
Kashmir.
▪ Tamil Nadu's Authoor betel leaves get Geographical Indication (GI) certificate.
▪ Geographical Indications Registry, in June 2023, accorded the Geographical Indication (GI) tag to the
Kari Ishad mango predominantly grown in Ankola taluk of Uttara Kannada in Karnataka .
▪ Mircha rice variety from West Champaran district of Bihar has recently been granted the GI tag.
▪ With the recently added products in GI tag lists, has made the total number of GI tags in India 2023 to
over 430.
▪ The products included are Tandur red gram of Telangana, gamocha of Assam, Alibag white
onion of Maharashtra, and Raktsey Karpo apricot of Ladakh among others.
▪ After getting GI tag for three more One District One Product (ODOP) crafts, Uttar Pradesh has now
come second in the country in terms of maximum number of items with Geographical Indication tag.
▪ The ODOP crafts which have got GI tag include Mainpuri Tarkashi, Mahoba Gaura Stone Craft and
Sambhal Horn Craft.
▪ Tamil Nadu leads with 55 GI-tags, followed by UP and Karnataka with 48 and 46 GI products,
respectively.
▪ According to data shared by the GI Registry, Kerala received the highest number of Geographical
Indication (GI) tags for products among all states in India in the fiscal year 23 (FY23).
▪ Ladakh wood carvings, also known as 'Ladakh Shingskos', have been granted the Geographical
Indication (GI) tag.
▪ The GI tag was granted to Tamil Nadu's famous Cumbum Paneer Thirachai, also known as Cumbum
Grapes.
▪ Gond paintings from Madhya Pradesh have bagged the Geographical Indication (GI) tag.
▪ Banarasi paan has been awarded the GI tag, which signifies its unique identity and origin. Three other
foods from the Varanasi region, including Banarasi Langda Mango, Ramnagar Bhanta (Brinjal), and
Adamchini rice, have also received the GI tag.
▪ Chhattisgarh's Nagri Dubraj rice variety gets geographical indication tag.
▪ Gamocha’, a symbol of the culture and identity of Assam, has received the Geographical Indication (GI)
tag.
▪ European Commission (EC) has recently granted Protected Geographical Indication (PGI) status to
Kangra tea, aunique variety of tea grown in the Kangra district of Himachal Pradesh in India.
▪ The Central Government has granted Geographical Indication (GI) tag to Mithila Makhana, Bihar.
▪ GEOGRAPHICAL INDICATIONS (GI) :-

Follow us: Official Site, Telegram, Facebook, Instagram, Instamojo 450


▪ GI tags are issued as per the Geographical Indcations of Goods (Registration and Protection) Act,1999.
This tag is issued by the Geographical Indication Registry under the Department of Industry Promotion
and Internal Trade, Ministry of Commerce and Industry.
▪ Darjeeling Tea was the first Indian product to get the geographical indication tag in 2004.
▪ The second GI tag in India was given to Aranmula Kannadi (Handicraft) which is a mirror and made in
Kerala.
▪ A GI is registered for an initial period of ten years, which must be renewed after 10 years.
▪ Kerala has topped among Indian states for having most number of products securing the Geographical
Indication (GI) tag in financial year 2022-23.
▪ What is GI tag?
▪ A GI is a sign used on products that have a specific geographical origin and possess qualities or a
reputation that are due to that origin.
Q. Name the Indian who has been bestowed with the Ramon Magsaysay Award for 2023?
A) Palagummi Sainath B) Rajanikant Arole
C) Pramod Karan Sethi D) R Ravi Kannan
Answer : D
✓ Recently, Surgical oncologist Padma Shri Dr R. Ravi Kannan, Director of the Cachar Cancer Hospital
and Research Centre (CCHRC) in Assam, received the prestigious Ramon Magsaysay Award for 2023
Q. How many individuals have been awarded with the Ramon Magsaysay Award 2023?
A) 04 B) 02
C) 05 D) 03
Answer : A
✓ The 65th Ramon Magsaysay Award 2023 has been bestowed on the following 04 individuals:
✓ They are Korvi Rakshand from Bangladesh, Eugenio Lemos from Timor-Leste, Miriam Coronel-Ferrer
from the Philippines, and Dr Ravi Kannan R. from India.
✓ The award carries a certificate, a medallion bearing the likeness of the late President, and a cash prize of
USD 50,000.
✓ About Ramon Magsaysay Award:
✓ Established : 1957
✓ First award : 1958
✓ Vinoba Bhave was the 1st Magsaysay Award winner in 1958 (first Indian).
✓ This award was created to commemorate Ramon Magsaysay, the late president of Philippines.
✓ The Ramon Magsaysay Award is Asia’s premier prize and highest honour.
✓ It is considered as Asia’s equivalent of the Nobel Prize
✓ This award is given in 6 categories
✓ These categories are:
1. Government services (GS)
2. Public services (PS)
3. Community leadership(CL)
4. Journalism, literature & creative communication arts (JLCCA)
5. Peace and International Understanding (PIU)
6. Emergent leadership (EL)

Follow us: Official Site, Telegram, Facebook, Instagram, Instamojo 451


✓ Indian winners on the list
✓ Prominent Indians who have won the award include Vinoba Bhave in 1958, Mother Teresa in 1962,
Kamaladevi Chattopadhyay in 1966, Satyajit Ray in 1967, and Mahasweta Devi in 1997.
✓ In recent years, Arvind Kejriwal (2006), Anshu Gupta of Goonj (2015), human rights activist Bezwada
Wilson (2016), and journalist Ravish Kumar (2019) have won the award

Q. The Ramon Magsaysay Award is an annual award given by which country?


A) United States B) New Zealand
C) Germany D) Philippines
Answer : D
Q. India defeated which country to win the inaugural Men’s Hockey5s Asia Cup 2023?
A) Japan B) Malaysia
C) Pakistan D) Oman
Answer : C
✓ India has won the Men’s Hockey5s Asia Cup 2023 by defeating Pakistan in the final match. Mor
Mandeep was the captain of team India.
✓ The event was held in Salalah, Oman. With the win, India has booked their place in the FIH Men’s
Hockey5s World Cup 2024. It will be held in Muscat.

Q. Which edition of the ASEAN Summit was held in 2023 and in which city?
A) 43rd, Singapore B) 20th, Bangkok
C) 43rd, Jakarta D) 18th, Manila
Answer : C
✓ The 43rd Association of Southeast Asian Nations (ASEAN) summit will be held in Jakarta, Indonesia
from 5th to 7th of September 2023.
✓ Presently there are 10 countries as members of ASEAN.
✓ The annual ASEAN Tourism Forum will be hosted by Laos in January 2024.
✓ Theme : The theme of the forum will be "Quality and Responsible Tourism -- Sustaining ASEAN
Future".
✓ Previously, Laos hosted the ASEAN Tourism Forum in 2004 and 2013.
✓ The 42nd Summit of the Association of Southeast Asian Nations (ASEAN) started in Indonesia.
✓ Theme 2023 – ASEAN Affairs: Epicenter of Growth.
✓ Association of Southeast Asian Nations (ASEAN)
✓ Founded : 1967
✓ Headquarteres : Jakarta, Indonesia
✓ Secretary General : Lim Jock Hoi
✓ Member : 10 (Brunei, Cambodia, Indonesia, Laos, Malaysia, Myanmar, Philippines, Singapore,
Thailand & Vietnam)
✓ Trick to Remember Countries of ASEAN
✓ Trick : TV CLIP of MBBS
✓ T - Thailand
✓ V - Vietnam
✓ C - Cambodia
✓ L- Laos
✓ I - Indonesia
✓ P - Philippines of silent

Follow us: Official Site, Telegram, Facebook, Instagram, Instamojo 452


✓ M - Malaysia
✓ B - Burma (Myanmar)
✓ B - Brunei
✓ S - Singapore.
✓ Association of Southeast Asian Nations (ASEAN) has agreed ‘in principle’ to admit East Timor as the
group’s 11th member
Q. Recently, the Western Air Command (WAC) of the Indian Air Force launched its annual training
exercise along the China-Pakistan border in the north and west, covering area from Leh in Kashmir to
Nal in Rajasthan. What is the name of the exercise?
A) Kavach B) Trishul
C) Raksha D) Agni
Answer : B
✓ The Western Air Command (WAC) of the Indian Air Force (IAF) has launched its annual major training
exercise, codenamed ‘Trishul, from September 04 to 14, 2023, along the China-Pakistan border in the
north and west, covering area from Leh in Kashmir to Nal in Rajasthan.
✓ The main objective of this 10-days ‘Trishul’ exercise is to evaluate WAC’s overall combat preparedness.
✓ A variety of combat assets of IAF such as fighter jets such as Rafales, Jaguars, MiG-29s, and Sukhoi-
30MKIs, as well as transport aircraft such as C-130Js and C-17s, as well as surface-to-air guided weapons
such as S-400s, MR-SAMS, and Akash are being used during the drill

Q. Name the winner of the Bird Photographer of the Year 2023?


A) Jack Zhi B) Anton Trexler
C) Liron Gertsman D) Stefan Gerrits
Answer : A
✓ Jack Zhi wins Bird Photographer of the Year 2023
✓ Wildlife photographer Jack Zhi has been named as the overall winner for the Bird Photographer of the
Year 2023, for his image titled “Grab the Bull by the Horns”.
✓ Zhi won the prestigious honour which is worth £5,000.
✓ The image also won the Gold Award in Bird Behavior category.
✓ The image shows that how, during the breeding season, a female peregrine falcon fiercely protects her
young, attacking anything that comes near the nest.
Q. The world’s tallest Nataraja statue has been installed at Bharat Mandapam, the venue of the G20 summit
in Delhi. Who made this statue?
A) Ravinder Reddy
B) Ram V. Sutar
C) Subodh Gupta
D) Radhakrishnan Sthapathy
Answer : D
✓ The world’s tallest Nataraja statue has been installed at Bharat Mandapam, the venue of the G20 summit
in Delhi.
✓ The 28-foot-tall statue weighing 20 tonnes was made in a record time of seven months by sculptor
Radhakrishnan Sthapathy and his team at Swamimalai in Thanjavur District of Tamil Nadu.
✓ The statue was built in Ashtadhatu (8 metals) using the traditional casting method.

Follow us: Official Site, Telegram, Facebook, Instagram, Instamojo 453


STATUE IN NEWS 2023
▪ President Droupadi Murmu has unveiled a 12 feet high statue of the father of the nation Mahatma
Gandhi and a ‘Gandhi Vatika’ at Gandhi Darshan near Rajghat in New Delhi.
▪ Union Home Minister Amit Shah on laid the foundation stone of 108 feet tall statue of Lord Shri Ram
in Kurnool, Andhra Pradesh.
▪ Union Home Minister Amit Shah unveiled a 54 feet tall statue of Lord Hanuman at Sarangpur temple in
Botad district of Gujarat.
▪ Union Home Minister and Minister of Cooperation, Shri Amit Shah unveiled the statues of Lord
Basaveshwara ji and Nadaprabhu Kempegowda ji at the State Assembly premises in Bengaluru,
Karnataka.
▪ Samatha Kumbh is being held on the premises of the Statue of Equality at Muchintal, Telangana.
▪ Union Home Minister Amit Shah has unveiled a 120-feet-tall statue of a polo player riding a pony at
Marjing Polo Complex in Manipur’s Imphal East district.
▪ Tamil Nadu CM Stalin inaugurated statue of ‘Mahakavi Subramanian Bharathiyar’.
▪ Prime Minister Narendra Modi has unveiled a 108-feet tall bronze ‘Statue of Prosperity’
▪ of Sri Nadaprabhu Kempegowda in Bengaluru, Karnataka.
▪ 369-foot-tall Shiva statue called “Viswas Swaroopam” that has been built in Nathdwara, Rajasthan’s
Rajsamand district, will be dedicated.
▪ The Viswas Swaroopam statue was built by Tat Padam Sansthan.
Q. Moon Sniper is a moon lander mission launched by which country recently?
A) Brazil B) Spain
C) France D) Japan
Answer : D
✓ The Japanese space agency, Japan Aerospace Exploration Agency (JAXA) launched its moon -lander
mission SLIM (Smart Lander for Investigating Moon) on September 07, 2023. The mission is nicknamed
as Moon Sniper.
✓ The spacecraft took off aboard H-IIA rocket from Tanegashima Space Center in southern Japan.
✓ The mission is expected to touch down on the lunar surface by February 2024.
✓ The overall cost of the mission is estimated to be $100-million.
✓ If Japan succeeds in landing its SLIM spacecraft, it shall become only the fifth country in the world to do
so.
✓ Apart from this, the H-IIA rocket also carried X-Ray Imaging and Spectroscopy Mission (XRISM), a
research satellite meant for astronomical observations. It has been developed by the NASA, the European
Space Agency and the Japan Aerospace Exploration Agency (JAXA).
Q. Which city has topped in the Swachh Vayu Sarvekshan 2023?
A) Indore B) Surat
C) Patna D) Nagpur
Answer : A
✓ Indore Ranks first in Swachh Vayu Sarvekshan 2023, followed by Agra and Thane.
✓ Union Minister for Environment, Forest and Climate Change and Labour and Employment Shri
Bhupender Yadav distributed the awards of Swachh Vayu Sarvekshan 2023 in Bhopal ,Madhya Pradesh
on September 07, 2023, on the occasion of International Day of Clean Air for blue skies.
✓ Winners are mentioned below
✓ Category I – (Million plus population)
✓ Indore

Follow us: Official Site, Telegram, Facebook, Instagram, Instamojo 454


✓ Agra
✓ Thane.
✓ Category II – (3-10 lakhs population)
✓ Amravati
✓ Moradabad
✓ Guntur.
✓ Category III – (less than 3 lakhs population)
✓ Parwanoo
✓ Kala Amb
✓ Angul.

MADHYA PRADESH IN NEWS 2023


▪ The 'Sanchi' city of Madhya Pradesh has been established as the country's first solar city.
▪ International Aerospace Conference jointly organized by MoCA and CII in Gwalior, Madhya Pradesh.
▪ President of India, Droupadi Murmu has inaugurated ‘Unmesha’ – International Literature Festival and
‘Utkarsh’ – Festival of Folk and Tribal Performing Arts at Bhopal, Madhya Pradesh.
▪ India's first online gaming academy launched in Madhya Pradesh.
▪ Prime Minister Narendra Modi launched the National Sickle Cell Anemia Elimination Mission in
Shahdol, Madhya Pradesh.
▪ Tiger State Madhya Pradesh is poised to get its 7th tiger reserve with the state government going forward
with the notification of Nauradehi and Durgavati Wildlife Sanctuaries.
▪ Madhya Pradesh received the first prize in the best State category, while the best district award went to
Ganjam District in Odisha.
▪ Narmada Hydroelectric Development Corporation Ltd (NHDC Ltd.) will construct a 525 MW pumped
storage project near the Indira Sagar Dam in Khandwa, Madhya Pradesh.
▪ Madhya Pradesh Chief Minister Shivraj Singh Chouhan launched the 'Mukhya Mantri Ladli Bahna
Yojana'.
▪ Madhya Pradesh cabinet approves 'Mukhya Mantri Sikho Kamao Yojana' for unemployed youth.
▪ Gond paintings from Madhya Pradesh have bagged the Geographical Indication (GI) tag, a recognition
that protects and acknowledges the work of tribal artists.
▪ Madhya Pradesh government launched Chief Minister's Residential Land Rights Scheme.
▪ Madhya Pradesh government announced that Islam Nagar village located in Bhopal district has been
renamed as Jagdishpur.
▪ Madhya Pradesh Government on 5 March launched Ladli Bahna scheme under which each woman will
be given one thousand rupees per month..
▪ Madhya Pradesh Chief Minister Shivraj Singh Chouhan announced on March 4 to make Tehsil Mauganj
of Rewa district the 53rd district of the state.

Q. Name the winner of the Miss International India 2023?


A) Zoya Afroz B) Praveena Aanjna
C) Simrithi Bathija D) Tanishqa Bhosale
Answer : B
✓ Praveena Aanjna crowned as Miss International India 2023.
✓ Praveena Aanjna has won the Miss International India 2023 title during the Miss Divine Beauty 2023
final held at the Jawaharlal Nehru Stadium in New Delhi on August 26.
✓ She won the title from among 16 finalists.
✓ The 24-year-old Praveena Aanjna is a chartered accountant by profession.
✓ She hails from Udaipur, Rajasthan.
✓ Aanjna will now represent India at the 61st edition of Miss International 2023 in Japan in October.

Follow us: Official Site, Telegram, Facebook, Instagram, Instamojo 455


MISS WORLD / UNIVERSE IN NEWS 2023
▪ Praveena Aanjna crowned as Miss International India 2023.
▪ Priyan Sen, who hails from Rajasthan, won the Miss Earth India 2023 title during the Miss Divine Beauty
2023 National Finals held in New Delhi.
▪ Rikkie Kollé is the first transgender woman to win Miss Universe Netherlands.
▪ Rajasthan’s Nandini Gupta crowned 2023 Femina Miss India.
▪ America's R'Bonnie Gabriel won the Miss Universe 2022 title at the 71st Miss Universe pageant.
▪ South Korea’s Mina Sue Choi Crowned Miss Earth 2022.
▪ Divita Rai from Karnataka won the prestigious title of Miss Diva Universe 2022.
▪ Indian-American Aarya Walvekar crowned Miss India USA 2022.
▪ Pallavi Singh wins the Mrs Universe Divine Crown in South Korea.
▪ Khushi Patel from UK is crowned Miss India Worldwide 2022.
▪ Karolina Bielawska from Poland has won the title of Miss World 2021.
▪ List of Miss World winners from India
1. Reita Faria : 1966 2. Aishwarya Rai : 1994
3. Diana Hayden : 1997 4. Yukta Mookhey : 1999
5. Priyanka Chopra : 2000 6. Manushi Chhillar : 2017.

▪ ➢ List of Miss Universe winners from India

1. Sushmita Sen : 1994 2. Lara Dutta : 2000


3. Harnaaz Sandhu : 2021.
▪ Reita Faria is the first Indian to win the Miss World in 1966.
▪ Sushmita Sen first Indian to win the Miss Universe title in 1994.
▪ Nicole Faria became the first Indian to win the ‘Miss Earth’ in 2010
Q. Lunar Reconnaissance Orbiter (LRO), was launched by which agency?
A) NASA
B) ISRO
C) ROCOSMOS
D) JAXA
Answer : A
✓ NASA's Lunar Reconnaissance Orbiter (LRO) recently captured the image of India's Chandrayaan-3
lander.
✓ About Lunar Reconnaissance Orbiter (LRO):
✓ It is a NASA spacecraft that was launched on June 18, 2009.
✓ Primary Objective: To make a 3D map of the Moon's surface from lunar polar orbit.
✓ It has also been used to study the Moon's geology, mineralogy, and environment.
✓ It orbits the Moon in an eccentric polar mapping orbit.
✓ LRO is equipped with 7 science instruments, the most well-known of which is a 195-millimeter (7.7-inch)
telescope and camera system that can see details up to 2.5 meters across.
✓ LRO is equipped with a laser altimeter that produces 3D maps by shooting lasers at the surface and
measuring reflection times.

Follow us: Official Site, Telegram, Facebook, Instagram, Instamojo 456


Q. What is the theme of ASEAN Summit in 2023?
A) We Care, We Prepare, We Prosper
B) ASEAN Matters: Epicentrum of Growth
C) ASEAN A.C.T.: Addressing Challenges Together
D) Cohesive and Responsive ASEAN
Answer : B
✓ PM Modi attends 20th ASEAN-India Summit and 18th East Asia Summit (EAS) in Jakarta.
✓ Prime Minister Narendra Modi attended the 20th ASEAN-India Summit and the 18th East Asia Summit
(EAS) in Jakarta, Indonesia on 7 September 2023.
✓ The summit was held on the sidelines of the 43rd ASEAN Summit 2023.
✓ The theme of ASEAN Summit in 2023 is ‘ASEAN Matters: Epicentrum of Growth.
✓ At the ASEAN-India Summit, Prime Minister presented a 12-point proposal for strengthening India –
ASEAN cooperation.
✓ About the Association of Southeast Asian Nations (ASEAN):
✓ It is a regional intergovernmental organisation of ten Southeast Asian countries.
✓ It was established on 8 August 1967 in Bangkok, Thailand, with the signing of the ASEAN Declaration
(Bangkok Declaration).
✓ Member States (10) : Indonesia, Malaysia, Philippines, Singapore, Thailand, Brunei, Laos, Myanmar,
Cambodia and Vietnam.
✓ It promotes intergovernmental cooperation and facilitates economic, political, security, military,
educational, and sociocultural integration between its members and other countries in Asia.
✓ Secretariat: The ASEAN Secretariat, located in Jakarta, Indonesia, and led by a secretary-general,
coordinates the work of ASEAN member states and their associated bodies.

Q. Which edition of the East Asia Summit (EAS) was held in 2023 in Jakarta?
A) 21 B) 18
C) 30 D) 10
Answer : B
✓ Prime Minister Narendra Modi attended the 20th ASEAN-India Summit and the 18th East Asia Summit
(EAS) in Jakarta, Indonesia on 7 September 2023.
✓ What is the East Asia Summit?
✓ It is a unique Leaders-led forum of 18 countries of the Asia-Pacific region formed to further the objectives
of regional peace, security and prosperity.
✓ Established in 2005, EAS allows the principal players in the Asia-Pacific region to discuss issues of
common interest and concern in an open and transparent manner at the highest level.
✓ The membership of EAS consists of ten ASEAN Member States: Australia, China, India, Japan, New
Zealand, the Republic of Korea, the Russian Federation and the USA.
✓ EAS is an initiative of ASEAN and is based on the premise of the centrality of ASEAN.
Q. Which Railway Station was recently awarded the ‘Green Railway Station’ certification with the highest
rating of Platinum by the Indian Green Building Council?
A) Chengalpattu Railway Station, Tamil Nadu
B) Hubbali Railway Station, Karnataka
C) Kalyan Railway Station, Maharashtra
D) Vijaywada Railway Station, Andhra Pradesh

Follow us: Official Site, Telegram, Facebook, Instagram, Instamojo 457


Answer : D
✓ Vijayawada Railway Station was awarded the ‘Green Railway Station’ certification with the highest
rating of Platinum by the Indian Green Building Council (IGBC).
✓ About Indian Green Building Council:
✓ The Indian Green Building Council (IGBC) is part of the Confederation of Indian Industry (CII) and was
formed in the year 2001.
✓ It is India's Premier certification body.
✓ The council offers a wide array of services, which include developing new green building rating
programmes, certification services and green building training programmes.
✓ The rating is based on six environmental categories, which include sustainable station facility, health,
hygiene and sanitation, energy efficiency, water efficiency, smart and green initiatives and innovation
and development.
✓ Headquarter: Hyderabad

Q. What is the theme of the 2023 G20 Leaders’ Summit held in New Delhi?
A) Recover Together, Recover Stronger
B) One Earth, One Family, One Future
C) People, Planet, Prosperity
D) Realizing Opportunities of the 21st Century for All
Answer : B
✓ Global Biofuel Alliance and India-Middle East-Europe Economic Corridor announced at 2023 G20 New
Delhi Summit .
✓ The 2023 G20 Leaders’ Summit began in New Delhi on September 09, 2023.
✓ The two day summit from 9 – 10 September 2023, is taking place in Bharat Mandapam International
Exhibition-Convention Centre, Pragati Maidan, New Delhi.
✓ It is the 18th meeting of G20 (Group of Twenty), and for the first time G20 summit is being held in India
as well as in South Asia.
✓ The Summit is chaired by the Prime Minister of India, Shri Narendra Modi.
✓ The theme of 2023 G20 Leaders’ Summit is One Earth, One Family, One Future
✓ Indonesia held the presidency in 2022.
✓ Two major global partnerships announced on Day 1 of the Summit under India’s presidency are
✓ Global Biofuel Alliance (GBA)
✓ India-Middle East-Europe Economic Corridor.
✓ INDIAN PRESIDENCY OF G 20
✓ India officially assumed the Presidency of G20 on 1 December 2022 from Indonesia.
✓ India will hold the G20 Presidency from 1 December 2022 to 30 November 2023, and host the 18th G20
Summit in 2023.
✓ The summit meeting of the Head of the States and the leaders of the G20 is scheduled to be held in
September 2023 in New Delhi.
✓ The theme of India’s G20 Presidency—“Vasudhaiva Kutumbakam” or “One Earth One Family One
Future”—is drawn from the ancient Sanskrit text of the Maha Upanishad.
✓ India’s G-20 Sherpa Amitabh Kant.
✓ Group of Twenty (G-20)
✓ The G-20 was founded back in 1999 after the Asian financial crisis.
✓ The G20 or Group of 20 is an intergovernmental forum comprising 19 sovereign countries, the European
Union (EU), and the African Union (AU).It works to address major issues related to the global economy,
such as international financial stability, climate change mitigation and sustainable development.

Follow us: Official Site, Telegram, Facebook, Instagram, Instamojo 458


✓ The Group of Twenty (G-20) comprises 19 countries (Argentina, Australia, Brazil, Canada, China,
France, Germany, India, Indonesia, Italy, Japan, Republic of Korea, Mexico, Russia, Saudi Arabia,
South Africa, Turkiye, the United Kingdom, and the United States), European Union and
✓ The G-20 members represent around 85 per cent of the global GDP, over 75 per cent of the global trade,
and about two-thirds of the world population.
✓ India has been a member of the G20 since its inception in 1999.
✓ LIST OF G20 SUMMIT
✓ 1st G20 Summit 2008 – USA
✓ 16th G20 summit 2021 – Italy
✓ 17th G20 summit 2022 – Indonesia
✓ 18th G20 summit 2023 – India
✓ 19th G20 summit 2024 – Brazil
✓ 20th G20 Summit 2025 – South Africa
Q. Which among these countries or union has been included as the 21st permanent member of the Group
of 20 (G20) nation?
A) African Union B) Spain
C) World Trade Organization (WTO) D) Britain
Answer : A
✓ The African Union (AU), has been formally included as a permanent member of the Group of 20 (G20),
during the 18th G20 meeting 2023, being held in India on September 09 and 10, 2023. AU is a continental
body of 55 member states.
✓ During the opening remarks at the summit on September 9, 2023, Prime Minister Modi invited the AU
Chairperson and President of the Union of Comoros, Azali Assoumani, to take a seat at the table of G20
leaders as a permanent member representing AU.
✓ As of 2023, there are 21 members in the Group of 20 (G20).
✓ About African Union (AU):
✓ The African Union (AU) is a continental body consisting of the 55 member states that make up the
countries of the African Continent.
✓ It was officially launched in 2002 and replaced its predecessor, the Organization of African Unity (OAU),
which was founded in 1963.
✓ It aims to strengthen political, economic, and social integration among African countries, with a vision
of achieving an integrated, prosperous, and peaceful Africa.
✓ Headquarters: Addis Ababa, Ethiopia.
✓ The African Union (AU) is a continental union consisting of 55 member states located on the continent
of Africa. In 2017, the AU admitted Morocco as a member state.
Q. How many scientists have been selected for the Shanti Swarup Bhatnagar Award for the year 2022?
A) 10 B) 11
C) 12 D) 13
Answer : C
✓ 12 scientists have been selected for the Shanti Swarup Bhatnagar Award for the year 2022.
✓ Shanti Swarup Bhatnagar is the top annual award in the field of science.
✓ 12 scientists will be awarded awards in seven categories. Dr.N. CSIR Director General Kalaiselvi met
Union Science and Technology Minister Dr. The awards for the year 2022 were announced in the
presence of Jitendra Singh.
✓ Named after Shanti Swarup Bhatnagar, the first director of CSIR, the awards are given in seven scientific
disciplines – Physics, Biology, Engineering, Mathematics, Medicine, Chemistry and Earth Sciences.

Follow us: Official Site, Telegram, Facebook, Instagram, Instamojo 459


✓ About Shanti Swarup Bhatnagar Prize for Science and Technology:
✓ It is a science award in India given annually by the Council of Scientific and Industrial Research (CSIR).
✓ It is the most coveted award in multidisciplinary science in India, recognizing outstanding Indian work
(according to the view of the CSIR awarding committee) in science and technology.
✓ It is given for notable and outstanding research in applied or fundamental science, biology, chemistry,
environmental science, engineering, mathematics, medicine and physics.
✓ It was awarded first in 1958 and the names of the recipients are traditionally declared by the Director
General on every 26 September, which is the CSIR Foundation Day. The prize is distributed by the Prime
Minister of India.
✓ Any citizen of India engaged in research in any field of science and technology up to the age of 45 years
is eligible for the prize.
✓ The prize is awarded on the basis of contributions made through work done in India only during the 5
years preceding the year of the prize.
✓ The prize comprises a citation, a plaque, and a cash award of ₹5 lakh. In addition, recipients also receive
Rs. 15,000 per month up to the age of 65 years.
Q. Sabudana of Salem district of which state has recently been given GI tag?
A) Assam
B) Tamil Nadu
C) Rajasthan
D) Uttar Pradesh
Answer : B
✓ Salem district in the state of Tamil Nadu has gained acclaim for its sago production, which is widely
recognized as Sabudana.
✓ Salem Sago, locally known as Javvarisi, is derived from the wet starch powder extracted from tapioca
roots. Indian tapioca roots are known to contain approximately 30-35% starch content.
✓ Sago production has been a cornerstone of Salem’s economic development since 1967.
✓ Currently, over 80% of sago in India is produced in the Salem region, with a significant portion marketed
through Sagoserve.
TAMIL NADU IN NEWS 2023
▪ Taiwanese firm Foxconn Technology Group, a key supplier to Apple Inc., signed a letter of Intent with
the Tamil Nadu government to invest ₹1,600 crore to establish a manufacturing facility in Kancheepuram
district.
▪ Centre has approved the establishment of a new spaceport in Kulasekarapattinam, Tamil Nadu for
carrying out the launches of the Small Satellite Launch Vehicles (SSLV) developed by the ISRO.
▪ Tamil Nadu's Authoor betel leaves get Geographical Indication (GI) certificate.
▪ Tamil Nadu bagged the Best Performing State award 2023 in cadaver organ donation .
▪ Tamil Nadu Youth Welfare and Sports Development Minister "Udhayanidhi Stalin" has unveiled the
trophy and the ‘Bomman’ mascot for the prestigious Asian Champions Trophy 2023 hockey tournament
to be held in Chennai.
▪ Muthamizh Selvi, first Tamil Nadu woman to scale Mt Everest.
▪ Tamil Nadu government declared Thanthai Periyar Wildlife Sanctuary as the 18th wildlife sanctuary in
the state..
▪ Tamil Nadu becomes first state to launch its own Climate Change Mission.
▪ Tamil Nadu govt to launch five-year Tamil Nadu Millet Mission.
▪ In a first of its kind initiative in India, the Government of Tamil Nadu announced the 'Nilgiri Tahr
Project' .

Follow us: Official Site, Telegram, Facebook, Instagram, Instamojo 460


Q. Which team won the title of SAFF U-16 Championship 2023?
A) India B) Bangladesh
C) Bhutan D) Pakistan
Answer : A
✓ India's Under-16 football team has won the title of SAFF U-16 Championship 2023 by defeating
Bangladesh 2-0 in the final.
✓ The final of the tournament was held at Changlimithang Stadium in Thimphu, Bhutan.
✓ India's Suraj Singh was awarded as the best goalkeeper of SAFF U16 Championship.
Q. Which Ministry has launched a Portal for 'Bharat: The Mother of Democracy' at G20 exhibition?
A) Ministry of Culture B) Ministry of Finance
C) Ministry of Corporate Affairs D) Ministry of Home Affairs
Answer : A
✓ On the eve of the G20 leadership summit, the culture ministry launched a portal of the mega exhibition—
‘Bharat: The Mother of Democracy’—to be held at Bharat Mandapam for global leaders attending the
conference.
✓ About “Bharat: The Mother of Democracy” Portal:
✓ The portal showcases the content on the history of democracy in India, showcasing 7,000 years of
democratic ethos starting from the Sindhu-Saraswati civilisation to 2019.
✓ It is available in 16 languages, including German, French, Spanish, Arabic, Portuguese, Italian, Turkish,
and Russian besides English and Hindi.
✓ The portal is divided into five sections and 22 sub-sections starting from Sindhu-Saraswati Civilisation
(6000-2000BCE), Mahajanapada and Gantantra (7-8 BCE), Vijaynagar Empire (14-16 century), and
Mughal emperor Akbar reign (1556-1605) to Constitution of India (1947) and Elections in Modern India
(1952 onwards).
Q. Who has been selected as the brand ambassador by women's fashion brand 'W'?
A) Alia Bhatt B) Priyanka Chopra
C) Deepika Padukone D) Anushka Sharma
Answer : D
✓ India's leading women’s fashion brand - W, on-boarded famous Bollywood actor and entrepreneur
Anushka Sharma as its brand ambassador

LATEST BRAND AMBASSADOR IN NEWS 2023


▪ Anushka Sharma : W
▪ Mahendra Singh Dhoni : Swaraj Tractors
▪ Iga Świątek : Infosys
▪ Rafael Nadal : Infosys
▪ Rahul Dravid : Bharat Petroleum Corporation Limited (BPCL)
▪ Shah Rukh Khan : ICC World Cup 2023
▪ Shraddha Kapoor: Asics India Private Limited
▪ Rashmika Mandanna : Printer company Epson India
▪ Smriti Mandhana : Wrangler
▪ Hrithik Roshan : Zebronics TV
▪ Sachin Tendulkar : Smile Ambassador
▪ Sania Mirza : Sony Sports Network

Follow us: Official Site, Telegram, Facebook, Instagram, Instamojo 461


▪ Sourav Ganguly : Brand ambassador of Tripura Tourism
▪ Virat Kohli : Duroflex
▪ Alia Bhatt : Gucci, the luxury fashion brand from Italy
▪ Ayushman Khurana : Wakefit
▪ Rishabh Pant : Star Sports
▪ Rohit Sharma : JioCinema
▪ Virat Kohli : HSBC India
▪ Ranveer Singh : Star Sports Network
▪ Shreya Ghodawat : India's ambassador for She Changes Climate
▪ MC Mary Kom, Farhan Akhtar : Brand ambassadors of IBA Women's World Championships 2023
▪ Sachin Tendulkar : World’s first ‘Hand Ambassador’ for Savlon India
▪ Ranveer Singh : Pepsi
▪ Ayushmann Khurrana : UNICEF India child rights
▪ Yastika Bhatia and Renuka Singh Thakur : Hyundai Motor India Ltd
▪ Vir Das : Plant-based meat brand UnCrave
▪ Harmanpreet Kaur : Puma
▪ Nikhat Zareen : National Mineral Development Corporation (NMDC)
▪ Saurav Ganguly : Bandhan Bank 'Jahaan Bandhan, Wahaan Trust' campaign
▪ Anushka Sharma : Puma
▪ Deepika Padukone : Pottery Barn
▪ Virat Kohli : Smart watch Noise
▪ Prasoon Joshi : Uttarakhand State
▪ Lionel Messi : Byjus
Q. Who is the first woman to head Indian Mission in Pakistan?
A) Meera Shankar B) Sujatha Singh
C) Preeti Saran D) Geetika Srivastava
Answer : D
✓ Geetika Srivastava is first woman to head Indian mission in Pakistan.
✓ Geetika Srivastava will be India’s new charge d’Affaires at its high commission in Islamabad.
✓ Geetika, who is currently serving as the Joint Secretary at the Ministry of External Affairs (MEA), is the
first woman to head India’s mission in Pakistan.
✓ Geetika is currently handling the Indo-Pacific division in MEA and will succeed Dr M Suresh Kumar,
who is expected to return to India soon.

FIRST'S FEMALE IN NEWS 2023


▪ Jaya Varma Sinha has been appointed as the new Chairperson and Chief Executive Officer (CEO),
Railway Board (Ministry of Railways) w.e.f Sept 01, 2023. She has replaced AK Lahoti.
▪ She is the first woman to be appointed to this Apex post of Indian Railways.
▪ Geetika Srivastava is first woman to head Indian mission in Pakistan.
▪ Phangnon Konyak became the first woman to chair the Rajya Sabha from Nagaland.
▪ Tenzing Yangki has become the first female IPS officer of Arunachal Pradesh.
▪ Lisa Franchetti becomes 1st woman to be top Navy officer in US history.
▪ Nusrat Chowdhary will become the first Muslim woman federal judge in US history.
▪ Celeste Saulo of Argentina has been appointed as the first female Secretary-General of the World
Meteorological Organization (WMO).
▪ Shalini Singh from Lucknow has become the first woman NCC cadet in India to complete a
mountaineering course in the Himalayan region of Uttarakhand.
▪ Parminder Chopra becomes first woman Chairman and Managing Director (CMD) of Power Finance
Corporation.

Follow us: Official Site, Telegram, Facebook, Instagram, Instamojo 462


▪ Amy Pope of the United States has been selected as the first female Director-General of the United
Nations Migration Agency.
▪ Lieutenant Shivangi Singh is the first woman Rafale pilot to take part in Orion Exercise 2023.
▪ Wing Commander Deepika Mishra has become the first woman Air Force officer to receive the Gallantry
Medal.
▪ New Zealand-born Kim Cotton has become the first female umpire to officiate in a Men’s T20
International (T20I) cricket match involving two ICC full-member nations.
▪ Sheenu Jhawar becomes first woman president of TiE Rajasthan.
▪ American Space Agency, NASA has selected the first woman astronaut, Christina Koch for its Artemis
II mission.
▪ Surekha Yadav, Asia’s 1st woman loco pilot who now operates Vande Bharat, From Solapur to
Chhatrapati Shivaji Maharaj Terminal (CSMT) in Maharashtra.
▪ Shalija Dhami becomes first woman officer to command a fighter unit in Indian Air Force (IAF).
▪ Colonel Geeta Rana became the first woman officer to command a field workshop.
▪ Captain Shiva Chauhan becomes the first women officer to be operationally deployed in the highest
battleground in Kumar Post on the Siachen glacier.
▪ The first woman officer to be posted on foreign assignment in the Border Roads Organization (BRO) -
Captain Surabhi Jakhmola.
▪ Rae Bareli's hockey stadium is named after hockey star Rani Rampal, first woman player from India to
achieve this feat.
▪ Hekani Jakhalu became the first female MLA of Nagaland Assembly.
▪ Equatorial Guinea appoints Manuela Roka Botey as first female PM.
▪ Indian-origin Manpreet Monica Singh sworn in as 1st female Sikh judge of USA.
▪ Santhi Kumari Appointed as First Woman Chief Secretary of Telangana.
▪ Saudi Arabia's Rayna Barnawi first female astronaut is set to go into space this year .
▪ Dina Boluarte became Peru’s first female president.
▪ FIFA has announced that Stephanie Frappart of France will be the first woman to referee a men’s’ World
Cup match.
▪ PT Usha Become First Women President of Indian Olympic Association.

Q. Who has won the 8th Dr M S Swaminathan Award for the period 2021-2022 ?
A) Dr. A. Krishnaiah B) Dr. P V Satyanarayana
C) Dr. K. Nagaiah D) Dr.C.M.Senthil Kumar
Answer : B
✓ Dr P V Satyanarayana, Principal Scientist at the Agricultural Research Station, Acharya N.G. Ranga
Agricultural University, Ragolu, has been recently awarded the prestigious Dr MS Swaminathan Award.
✓ About M S Swaminathan Award:
✓ It was instituted in 2004 with the objective to recognise the lifetime contributions of eminent persons who
have made great impacts and outstanding contributions to agricultural research and development and to
the overall food security and sustainability of agriculture, with special reference to India.
✓ It is named after the doyen of Indian Agriculture, Prof. M.S. Swaminathan, Chairman, M.S
Swaminathan Research Foundation, Chennai.
✓ It is a biennial award constituted by the Retired Indian Council of Agricultural Research Employees
Association (RICAREA) and Nuziveedu Seeds Limited (NSL).
✓ The award carries a Cash prize of Rs.2 lakhs (Rupees Two Lakhs only), a Medal, and a Citation.
✓ The award is open to all, irrespective of his/her nationality.
✓ Who was M S Swaminathan?
✓ Mankombu Sambasivan Swaminathan, often referred to as Dr. M.S. Swaminathan, is a renowned Indian
agricultural scientist and geneticist.
✓ Green Revolution in India:

Follow us: Official Site, Telegram, Facebook, Instagram, Instamojo 463


✓ He is often referred to as the "Father of the Green Revolution in India."
✓ His research and work in crop breeding, particularly for wheat and rice, led to the development of high-
yielding varieties that significantly increased agricultural productivity.
✓ This Green Revolution helped India overcome food shortages and played a crucial role in the country's
agricultural transformation.
✓ In recognition of his outstanding contributions, he was awarded the first World Food Prize, the Ramon
Magsaysay Award and many other national and international honours.

Q. Who has been appointed as the Brand Ambassador of Swaraj Tractors?


A) Rohit Sharma B) Sachin Tendulkar
C) MS Dhoni D) Virat Kohli
Answer : C
✓ Industrialist Anand Mahindra appointed Mahendra Singh Dhoni as the Brand Ambassador of Swaraj
Tractors, a company of Mahindra & Mahindra.
✓ Swaraj Tractors is an Indian tractor manufacturing company headquartered in Mohali, Punjab, and a
subsidiary of Mahindra & Mahindra.
✓ Established in 1974, its mission was to be self-reliant and develop India’s first indigenous tractor.
✓ Presently Swaraj Tractors is one of the leading tractor manufacturers in India with a market share of over
10%.
Q. Who has been appointed as the Principal Director General of Akashvani and News Services Division?
A) Rajat Sharma B) Ritu Dhawan
C) Dr. Vasudha Gupta D) Vinay Maheshwari
Answer : C
✓ Dr. Vasudha Gupta takes charge as Principal Director General of Akashvani and News Services Division
✓ Vasudha Gupta previously worked as Director General in All India Radio.
✓ About Akashvani (AIR)
✓ Akashvani, also known as All India Radio, is India's national public radio broadcaster, established in
1936.
✓ It is part of Prasar Bharati along with Indian television broadcaster Doordarshan.
✓ It is headquartered at Akashvani Bhawan, New Delhi, which houses various sections such as Drama,
FM and National Service as well as Doordarshan Kendra (Delhi).
✓ AIR is the world's largest radio network and one of the largest broadcasting organizations globally,
serving a wide variety of languages and cultures.
✓ AIR produces programming in 23 languages and 179 dialects.
Q. Who has been named as the chairman of the Film and Television Institute of India (FTII), Pune?
A) Aamir Khan B) Kamal Haasan
C) Gautham Vasudev Menon D) R Madhavan
Answer : D
✓ R Madhavan has been named as the chairman of the Film and Television Institute of India (FTII), Pune.
✓ Madhavan's nomination follows the national recognition his film 'Rocketry: The Nambi Effect' received.
✓ 'Rocketry: The Nambi Effect' won the prestigious Best Feature Film award at the 69th National Film
Awards.
✓ 'Rocketry: The Nambi Effect' throws light on the life of former ISRO scientist S Nambi Narayanan and
his remarkable contribution to India's space agency.

Follow us: Official Site, Telegram, Facebook, Instagram, Instamojo 464


✓ Film and Television Institute of India (FTII):-
✓ It is a film school affiliated with the Ministry of Information and Broadcasting, Government of India,
and is supported by the Central Government.
✓ FTII is located in Pune and was established in 1960 with the objective of training individuals for careers
in the film and television industry

Q. Which Countries Launched the RETAP Under the Umbrella of the Strategic Clean Energy Partnership?
A) India and United States B) India and Australia
C) India and New Zealand D) India and United Kingdom
Answer : A
✓ India and the United States have taken a bold step towards fostering renewable energy technology
innovation by launching the Renewable Energy Technology Action Platform (RETAP) under the
umbrella of the Strategic Clean Energy Partnership.
✓ RETAP's initial focus will revolve around the development of green and clean hydrogen, wind energy,
and long-duration energy storage solutions

Q. India's Largest Home-Built 700 MW Nuclear Plant Starts Operations in Which State?
A) Punjab B) Maharashtra
C) Gujarat D) Uttar Pradesh
Answer : C
✓ Prime Minister Narendra Modi announced the commencement of full-capacity operations at India's
largest domestically built 700 MW nuclear power plant located at Kakrapar in Gujarat.
✓ The nuclear facility at Kakrapar is the largest nuclear facility of its kind to be built entirely within India.
✓ Initially, the Kakrapar Atomic Power Project (KAPP) started commercial operations on 30 June but was
operating at 90% capacity.
✓ Nuclear power development in Kakrapar and all over India:-
✓ Nuclear Power Corporation of India Limited (NPCIL) is responsible for the construction of two 700 MW
Pressurized Heavy Water Reactors (PHWR) at Kakrapar in Gujarat.
✓ Kakrapar also has two power plants of 220 MW each.
✓ NPCIL currently operates 23 commercial nuclear power reactors.
✓ Future plans and projects:-
✓ KAPP Unit 4 had achieved 97.56% progress as of July while commissioning activities were in progress.
✓ NPCIL has plans to build 16 more PHWRs of 700 MW each across the country and has secured financial
and administrative approvals for these projects.
✓ Other 700 MW nuclear power plant projects are in progress at Rawatbhata, Rajasthan (RAPS 7 & 8),
and Gorakhpur, Haryana (GHAVP 1 & 2).
✓ The government has approved the construction of 10 indigenously developed PHWRs in fleet mode at
four locations: Gorakhpur (Haryana), Chutka (Madhya Pradesh), Mahi Banswara (Rajasthan), and
Kaiga (Karnataka).

Q. Who has been elected as the new President of Singapore?


A) Halima Yakub B) Tharman Shanmugaratnam
C) Lee Hsien Loong D) Ding Xuexiang
Answer : B
✓ Bharatvanshi Tharman Shanmugaratnam has registered a historic victory in the recently held presidential
election in Singapore.

Follow us: Official Site, Telegram, Facebook, Instagram, Instamojo 465


✓ He will be the 9th President of the country.
✓ Thurman won the election by defeating two rivals of Chinese origin.
✓ He got a total of 70.4% votes in this election.
✓ He will replace Halima Yakub. Singapore is an island country in Southeast Asia
✓ Singapore
✓ Capital : Singapore city
✓ Currency : Dollar
✓ President : Bharatvanshi Tharman Shanmugaratnam
✓ Prime Minister : Lee Hsien Loong
NEWLY PRESIDENT IN NEWS 2023
▪ Bharatvanshi Tharman Shanmugaratnam : Singapore
▪ Emmerson Mnangagwa : Zimbabwe
▪ Edgars Rinkevics : Latvian
▪ Tayyip Erdogan : Turkey
▪ Bola Tinubu : Nigeria
▪ Mohammed Shahabuddin Chuppu : Bangladesh
▪ Ram Chandra Paudel : Nepal
▪ Xi Jinping : China (third term)
▪ Vo Van Thuong : Vietnam
Q. Which state government has increased the amount of the Mukhyamantri Kanya Sumangla Yojana from
Rs 15,000 to Rs 25,000 from the financial year 2024-2025?
A) Uttar Pradesh B) Madhya Pradesh
C) Rajasthan D) Haryana
Answer : A
✓ UP govt increases amount of Mukhyamantri Kanya Sumangla Yojana.
✓ On the auspicious occasion of Raksha Bandhan, Uttar Pradesh Chief Minister Yogi Adityanath
announced an increase in the amount of the Mukhyamantri Kanya Sumangla Yojana from Rs 15,000 to
Rs 25,000 from the financial year 2024-2025.

UTTAR PRADESH IN NEWS 2023


▪ Uttar Pradesh government has decided to organize a campaign under Road Safety Fortnight from July
17 to 31.
▪ Uttar Pradesh Police Department has launched ‘Operation Conviction’ to combat criminals and mafias
in the state.
▪ Uttar Pradesh government has decided to cancel all Traffic Chalan for private and commercial vehicle
owners pending from 2017 to 2021.
▪ UP holds 2nd position in GI tagged products.
▪ After getting GI tag for three more One District One Product (ODOP) crafts, Uttar Pradesh has now
come second in the country in terms of maximum number of items with Geographical Indication tag.
▪ The ODOP crafts which have got GI tag include Mainpuri Tarkashi, Mahoba Gaura Stone Craft and
Sambhal Horn Craft.
▪ Tamil Nadu leads with 55 GI-tags, followed by UP and Karnataka with 48 and 46 GI products,
respectively.
▪ The state government has given green signal to the project of setting Uttar Pradesh a state's first Pharma
Park in Lalitpur district of Bundelkhand.
▪ Uttar Pradesh introduces “School Health Program” digital health cards for children.
▪ The online rural education initiative ‘Pahal’ was officially launched by Chief Secretary Durga Shankar
Mishra at Government UP Sainik Inter College in Sarojini Nagar.

Follow us: Official Site, Telegram, Facebook, Instagram, Instamojo 466


▪ Uttar Pradesh on track to be first Indian state with 100% EVs in govt depts.
▪ Prime Minister Shri Narendra Modi laid the foundation stone of the India’s first urban transport ropeway
on March 24, 2023, in Varanasi.
▪ Uttar Pradesh government has launched the portal for the creation of ‘Family ID – One Family One
Identity’.
▪ Uttarakhand won the first prize for its tableau, followed by Maharashtra and Uttar Pradesh in the second
and third positions respectively.
▪ Uttar Pradesh came in third with its tableau commemorating Ayodhya's Deepotsav.
▪ Uttar Pradesh Government Launched ‘Samagra Shiksha Abhiyan’ Campaign.
▪ Uttar Pradesh tops charts for mobile gaming destination in India.
▪ Taj Mahotsav Starts In Agra of Uttar Pradesh.
▪ UP first all-woman PAC battalions formed by Uttar Pradesh Govt.
▪ Uttar Pradesh tops in India, 8462 lakes developed under Mission Amrit Sarovar.
▪ Chief Minister Yogi Adityanath inaugurates north India’s first data centre at Greater Noida.
▪ Uttar Pradesh bags top honours at PMAY-U Awards 2021.
▪ Agriculture Minister of India, Narendra Singh Tomar, has introduced a new platform called ‘DigiClaim’
on the national crop insurance portal.
▪ Kannuj will be developed into a tourist destination as per the decision taken by the Government of Uttar
Pradesh.
▪ Centre has approved the setting up of the Terai Elephant Reserve (TER) at Dudhwa-Pilibhit in Uttar
Pradesh. The Terai Elephant Reserve is the 3rd Elephant Reserve in India.
▪ Uttar Pradesh cabinet has gave a green signal for the development of the first tiger reserve in the
Bundelkhand region
Q. Union Education Minister Dharmendra Pradhan has launched a comic book developed by NCERT &
UNESCO on good health and wellness. What is the name of this book?
A) Comic for Health B) Let’s Move Comic
C) Comic Forward D) Let’s Move Forward
Answer : D
✓ Union Education Minister Dharmendra Pradhan has launched a comic book on good health and
wellness.
✓ The comic book ‘Let’s Move Forward’ is designed to cater to the diverse requirements of adolescents and
encapsulates crucial information pertaining to the 11 thematic components of the School Health and
Wellness Programme.
✓ Launching the comic book, developed by the National Council of Educational Research and Training
(NCERT) and UNESCO, the minister hoped the book would motivate students to prioritize holistic well-
being while entertaining them through storytelling, with a key focus on good health and wellness.
✓ An electronic version of the comic book will also be accessible on the ministry website and also on the
websites of NCERT, UNESCO, and DIKSHA platforms.
✓ This comic book signifies a significant stride towards augmenting the goals of the School Health and
Wellness Programme (SHWP).
Q. Diversified group ITC will invest around Rs 1,500 crore to set up an integrated food manufacturing and
logistics facility in which state?
A) Uttar Pradesh B) Chhattisgarh
C) Madhya Pradesh D) Jharkhand
Answer : C

Follow us: Official Site, Telegram, Facebook, Instagram, Instamojo 467


✓ Diversified group ITC will invest around Rs 1,500 crore to set up an integrated food manufacturing and
logistics facility and sustainable packaging products manufacturing facility at Sehore in Madhya Pradesh.
✓ The two projects spread over an area of nearly 57 acres will give a boost to agricultural and manufacturing
sectors in Madhya Pradesh.
✓ The two projects, when complete, would entail a total investment outlay of Rs 1,500 crores which will
support livelihoods across sustainable value chain.
✓ While the food plant will manufacture ITC’s products including atta brand Aashirvaad, Sunfeast biscuits,
and ‘YiPPee!’ noodles, the moulded fibre products facility will be a pioneer in sustainable packaging,
contributing to plastic substitution in areas such as packaging for electronic items, FMCG, and the food
and beverage sector.
Q. Who has been appointed as a member of the prestigious International Mediator Panel of the Singapore
International Mediation Centre (SIMC)?
A) Ranjan Gogoi B) Dipak Misra
C) NV Ramana D) Tirath Singh Thakur
Answer : C
✓ Former Chief Justice of India (CJI) NV Ramana has been appointed as a member of the prestigious
International Mediator Panel of the Singapore International Mediation Centre (SIMC).
✓ Justice Ramana participated in the ‘Singapore Convention Week’, an annual conference organized by
Singapore’s Ministry of Law, the United Nations Commission on International Trade Law
(UNCITRAL), and over 20 partner organizations.
✓ About Singapore International Arbitration Center (SIMC)
✓ SIMC is an independent non-profit organization in Singapore.
✓ It specializes in providing arbitration services for cross-border commercial disputes.
✓ It maintains a panel of international arbitrators who facilitate the resolution of disputes.
Q. Who has been conferred the prestigious Emerging Entrepreneur of the Year 2023 award by the renowned
Entrepreneur India business magazine?
A) Manoj Bubna B) Vishnu Som
C) Shibani Dandekar D) Nidhi Kulpati
Answer : A
✓ Manoj Bubna Recognized as ‘Emerging Entrepreneur of the Year 2023
✓ In a remarkable recognition of visionary leadership and outstanding contributions to the e-commerce
technology landscape, N7 – The Nitrogen Platform’s CEO Manoj Bubna has been conferred the
prestigious Entrepreneur of the Year 2023 award by the renowned Entrepreneur India business
magazine..
✓ N7 – Nitrogen Platform is a new Digital experience and Streaming platform that delivers better digital
experiences, drives conversions, and increases engagement.
Q. Who has received the Dr. V G Patel Memorial Award 2023 for promoting and strengthening
entrepreneurship in India?
A) Satyajit Majumdar B) Abhigyan Prakash
C) Rini Simon Khanna D) Afshan Anjum
Answer : A
✓ Mumbai Professor Satyajit Majumdar, dean of School of Management and Labour Studies from Tata
Institute of Social Sciences (TISS), has received the ‘Dr V G Patel Memorial Award-2023 for

Follow us: Official Site, Telegram, Facebook, Instagram, Instamojo 468


Entrepreneurship Trainer, Educator and Mentor’ for his work in promoting and strengthening
entrepreneurship in India.
✓ Dr. Patel is widely acknowledged as the Father of the Entrepreneurship Movement in India.
Q. Danielle McGahey will become the first transgender cricketer to play in an international match for the
Women’s T20 World Cup 2024 in Bangladesh. Which country will she play for?
A) Australia B) Canada
C) South Africa D) England
Answer : B
✓ Danielle McGahey of Canada will become the first transgender cricketer to play in an international match
when she represents her adopted country next month in a regional qualifying tournament for the 2024
Women’s T20 World Cup in Bangladesh
Q. Which day is observed as the World Coconut Day globally?
A) 04 September B) 03 September
C) 01 September D) 02 September
Answer : D
✓ World Coconut Day is observed on September 02 every year since 2009.
✓ It is observed for promoting this tropical fruit and bringing awareness regarding its health benefits.
✓ The theme for World Coconut Day 2023 is 'Coconuts: Transforming Lives'.
✓ The WCD also commemorates the formation of the Asian Pacific Coconut Community (APCC), which
aims to promote, coordinate and harmonize all activities of the coconut industry
Q. Recently, Koraput Kalajeera rice, an aromatic rice, has received Geographical Indication status of which
state?
A) Assam B) Manipur
C) Odisha D) Chhattisgarh
Answer : C
✓ Recently, Koraput Kalajeera Rice,’ an aromatic rice, has got Geographical Indications status.
✓ About Koraput Kalajeera Rice:
✓ The farmers of Koraput district in Odisha have domesticated Kalajeera rice over generations.
✓ It is known as the ‘Prince of Rice’ and is an aromatic variety of rice.
✓ It is popular among rice consumers for its black colour, good aroma, taste and texture.
Q. Which state government has recently proposed to develop Gilbert Hill into a tourist spot on the lines of
Burj Khalifa in Dubai?
A) Uttar Pradesh B) Maharashtra
C) Rajasthan D) Delhi
Answer : B
✓ The Maharashtra Government recently proposed to develop Gilbert Hill into a tourist spot on the lines
of Burj Khalifa in Dubai.
✓ About Gilbert Hill:
✓ It is a 200-foot monolithic column of black basalt rock located at Andheri in Mumbai.

Follow us: Official Site, Telegram, Facebook, Instagram, Instamojo 469


✓ The rock has a sheer vertical face and was formed when molten lava was squeezed out of the Earth's
clefts during the Mesozoic Era about 66 million years ago.
✓ It was declared a National Park in 1952, while the Brihan Mumbai Municipal Corporation categorised
it as a Grade II heritage structure in 2007.

MAHARASHTRA IN NEWS 2023.


▪ Maharashtra Government recently proposed to develop Gilbert Hill into a tourist spot on the lines of
Burj Khalifa in Dubai.
▪ Maharashtra signs MoU with Germany’s professional association football.
▪ Maharashtra government has launched the Bhagwan Birsa Munda Road Scheme which aims to enhance
road connectivity in tribal areas of the 17 districts of the state.
▪ Maharashtra Industries Minister Uday Samant will present the first 'Udyog Ratna' award to veteran
industrialist and Tata Sons Chairman Emeritus Ratan Tata.
▪ Maharashtra topped the list with the highest number of missing women in 2021, with a reported count
of 56,498.
▪ Maharashtra’s Economy will become a trillion dollar by 2028, said N Chandrashekhar, president of the
State Economy Development Council.
▪ Maharashtra Prisons Department has launched a “video call” facility for foreign nationals lodged in
different jails across the state.
▪ NCP leader Ajit Pawar has been sworn in as the new Deputy Chief Minister of Maharashtra.
▪ Maharashtra Government launched Namo Shetkari Mahasanman Yojana.
▪ Maharashtra becomes the first state in the country to approve Good Governance Regulations.
▪ Uttar Pradesh surpasses Maharashtra in sugar production this season.
▪ A new initiative called ‘beggar-free city’ started in Nagpur, Maharashtra.
▪ Amit Shah, the Union Home Minister, is scheduled to present the ‘Maharashtra Bhushan’ Award to
Dattatreya Narayan Dharmadhikari, also known as Appasaheb Dharmadhikari, on April 16 for his
notable work as a social worker and reformer.
▪ Maharashtra Government has introduced a 4% quota for employees with disabilities in promotions.
▪ Bank of Maharashtra inaugurated its first dedicated Branch for Start-ups in Pune.
▪ Ramesh Bais Appointed as the New Governor of Maharashtra.
▪ Maharashtra became the first state in the country to pass the Lokayukta Bill.
▪ Recently a rare low altitude basalt plateau has been discovered in the Western Ghats of Maharashtra..
▪ Nilesh Sambre conferred with 'Maratha Udyog Ratna 2023' by Maharashtra State.
▪ Talon Grikspoor of the Netherlands won the singles title at the Maharashtra Open tennis tournament in
Pune.
Q. Which drone company has recently secured type certification approvals from the Director General of
Civil Aviation (DGCA) for its AG-365S kisan drone for use in agriculture?
A) Garuda Aerospace B) Aero360
C) Asteria Aerospace D) Marut Drones
Answer : D
✓ Hyderabad-based drone manufacturer Marut Drones has become the first player in the market to receive
the DGCA type certification in both small and medium-category battery-operated drones.
✓ Marut Drones recently secured type certification approvals from the Director General of Civil Aviation
(DGCA) for its AG-365S kisan drone for use in agriculture.
✓ About AG 365S:
✓ It is a multi-utility agricultural drone in the small category (less than 25kg).
✓ It has been designed and developed by Marut Drones; an Indian drone technology company based in
Hyderabad.

Follow us: Official Site, Telegram, Facebook, Instagram, Instamojo 470


✓ Purpose: The made-in-India kisan drone – AG 365 has been developed particularly for agricultural
purposes to reduce crop loss, lower agrochemical usage, and better yield and profits to the farmers.
✓ It is the first multi-utility agriculture small-category drone to receive the DGCA-approved type certificate
in India.
✓ Features:
✓ The AG-365S drone has the highest endurance of 22 minutes.
Q. Which State Uses AI-Enabled Tech to Clean Up Tourist Hotspot Umiam Lake?
A) Nagaland B) Meghalaya
C) Uttarakhand D) Maharashtra
Answer : B
✓ The Meghalaya government recently adopted AI-enabled robotic technology to keep its tourist hotspot,
Umiam Lake, free of pollutants.
✓ Umiam Lake, also known as Barapani Lake, is a lake in the state of Meghalaya 15 km (9.3 miles) north
of Shillong, the capital of Meghalaya.
Q. RAMBHA-LP payload on board Chandrayaan-3’s lander Vikram has completed the first in-situ
measurements of the surface-bound lunar ___ environment over the south polar region.
A) Plasma B) Superconductor
C) Photonic matter D) Superfluid
Answer : A
✓ Recently, the RAMBHA-LP payload on board Chandrayaan-3’s lander Vikram has completed the first
in-situ measurements of the surface-bound lunar plasma environment over the south polar region.
✓ About RAMBHA-LP Payload:
✓ The Radio Anatomy of Moon Bound Hypersensitive ionosphere and Atmosphere - Langmuir Probe,
named after American chemist and physicist Irving Langmuir, who was awarded the Nobel Prize in
Chemistry.
✓ It is a device used for characterising a plasma.
✓ The probe operates without interruption, aiming to explore the changes occurring in the near-surface
plasma environment throughout the lunar day.
✓ The device features a 5 cm metallic spherical probe mounted on a 1-metre boom attached to the
Chandrayaan-3 lander’s upper deck.
✓ It was led by Space Physics Laboratory (SPL), Vikram Sarabhai Space Centre (VSSC),
Thiruvananthapuram.
✓ Key findings
✓ The initial assessment indicates that the plasma near the lunar surface is relatively sparse.

Q. Which of the following has Introduced ENTAZIA biofungicide to Safeguard Crops in India?
A FMC India B) ICAR
C) CSIR D) BDL
Answer : A
✓ FMC India recently announced the launch of its latest product, ENTAZIA bio fungicide.
✓ About ENTAZIA:
✓ It is a bio fungicide crop protection product formulated with Bacillus subtilis.
✓ It was developed by FMC India.

Follow us: Official Site, Telegram, Facebook, Instagram, Instamojo 471


✓ It will provide farmers with a powerful and sustainable tool to protect their crops from fungi while
maintaining environmental integrity
Q. Who has unveiled a 12 feet high statue of the father of the nation Mahatma Gandhi and a ‘Gandhi Vatika’
at Gandhi Darshan near Rajghat in New Delhi?
A) Narendra Modi B) Amit Shah
C) Draupadi Murmu D) Rajnath Singh
Answer : C
✓ President Droupadi Murmu has unveiled a 12 feet high statue of the father of the nation Mahatma
Gandhi and a ‘Gandhi Vatika’ at Gandhi Darshan near Rajghat in New Delhi.
✓ To enhance the visitor experience, a selfie point has been established within Gandhi Vatika.
✓ The Gandhi Vatika features multiple statues of Mahatma Gandhi depicting him in various postures.
Speaking on the occasion, the President said that Mahatma Gandhi’s ideals and values have given a new
direction to the whole world.

Q. The First WHO Traditional Medicine Global Summit 2023 held in Which State?
A) Karnataka B) Maharashtra
C) Gujarat D) West Bengal
Answer : C
✓ The World Health Organization (WHO) released the outcome document of the first WHO Traditional
Medicine Global Summit 2023 as the "Gujarat Declaration".
✓ The "Gujarat Manifesto" reaffirms the global commitment to traditional medicine for health and
wellness.
✓ This emphasizes the need for rigorous scientific methods to understand and implement holistic and
context-specific healthcare approaches.
✓ India, as the host of the WHO Global Center for Traditional Medicine in Gujarat, plays an important
role in supporting Member States and taking forward the work agenda of the summit.
✓ The summit was organized by WHO and co-hosted by the Ministry of AYUSH in Gandhinagar, Gujarat
on 17 - 18 August 2023.
Q. Which state government has launched the Mukhya Mantri Sporting Abilities, Rebuilding Aspirations
and Livelihood Scheme (SABAL) Scheme to Empower specially-Abled Children?
A) Uttar Pradesh B) Himachal Pradesh
C) Rajasthan D) Uttarakhand
Answer : B
✓ Chief Minister Sukhwinder Singh Sukhu inaugurated the Mukhya Mantri Sporting Abilities, Rebuilding
Aspirations and Livelihood Scheme (SABAL) at Nadaun in the Hamirpur district of Himachal Pradesh.
✓ The goal of SABAL Yojana is to improve the lives of differently-abled children by connecting them to
the mainstream of society.
✓ Around 400 schools in Hamirpur, Solan, and Shimla districts will get government aid, which will include
necessary facilities and opportunities for specially-abled children.
✓ A 2% reservation quota has been allotted in government jobs for specially-abled persons.
✓ The education stipend for disabled children will be increased to Rs.1000.
✓ The Himachal Pradesh government aims to empower over 7,000 specially-abled children through the
SABAL scheme.

Follow us: Official Site, Telegram, Facebook, Instagram, Instamojo 472


✓ In the same program, Chief Minister Sukhwinder Singh Sukhu also launched 'Abhyas Himachal' and
'Teacher Sahayata' chatbots on the Swift Chat App.
HIMACHAL PRADESH IN NEWS 2023
▪ HIMACHAL PRADESHHimachal Pradesh Chief Minister Launches Scheme For Specially-Abled
Children, ‘SABAL’.
▪ Himachal Pradesh Chief Minister Sukhwinder Singh Sukhu launched 'Sashakt Mahila Loan Yojana', a
collateral-free loan scheme for women.
▪ Himachal Pradesh Chief Minister Sukhvinder Singh Sukhu has inaugurated “The Tribune Real Estate
Expo, 2023, Shimla”.
▪ Himachal Pradesh Chief Minister Sukhwinder Singh Sukhu informed that Hindustan Petroleum
Corporation Limited (HPCL) will set up an ethanol plant at Jeetpur Baheri in Una district at a cost of Rs
500 crore.
▪ European Commission (EC) has awarded protected geographical indication (PGI) status to Kangra tea,
a unique variety of tea grown in the Kangra district of India’s Himachal Pradesh.
▪ Himachal Pradesh (HP) has become the first Indian state to develop a DNA database for unidentified
bodies.
▪ Himachal Pradesh launched Sanjeevani Project to uplift the livelihood of small dairy farmers and
livestock owners.
▪ Himachal Pradesh targets to be first ‘green energy state’ by 2025.
▪ Himachal Cabinet approves monthly incentive of Rs 1,500 for women of Spiti Valley.
▪ Himachal Pradesh Chief Minister Sukhvinder Singh Sukhu announced to set up of Rs 101 crore CM’s
Sukhashraya Sahayata Kosh for the destitute in the state.
▪ Himachal Chief Minister Sukhvinder Singh Sukhu laid the foundation stone of ‘Himachal Niketan‘
which will provide accommodation facilities to the students and residents of Himachal Pradesh visiting
New Delhi.
▪ Himachal Pradesh celebrates its 53rd Statehood Day with joy and enthusiasm across the state on 25th
January 2023
▪ About Himachal Pradesh
▪ Formation - 1 November 1956
▪ Capital - Shimla and Dharamshala (Winter)
▪ District - 12
▪ Governor- Shiv Pratap Shukla
▪ Chief Minister - Sukhwinder Singh Sukhu
▪ Rajya Sabha - 3 seats
▪ Lok Sabha - 4 seats
▪ Symbol of Himachal Pradesh
▪ Bird - Western Tragopan
▪ Fish - Golden Mahseer
▪ Flowers - Pink Rhododendron
▪ Mammal - Snow Leopard
▪ Tree -Deodar Cedar

Q. Who has been named as the official sponsor of the Indian contingent for the 19th Asian Games?
A) Amul B) Paytm
C) Jio D) Airtel India
Answer : A
✓ Amul has been named as the official sponsor of the Indian contingent for the 19th Asian Games 2022 to
be held in Hangzhou, China.
✓ The 19th Asian Games will be held from September 23 to October 8, 2023

Follow us: Official Site, Telegram, Facebook, Instagram, Instamojo 473


Q. In India, the Teachers Day is celebrated every year ob 5th of September to mark the birth anniversary of
which leader?
A) V. V. Giri B) Rajendra Prasad
C) A. P. J. Abdul Kalam D) Dr Sarvepalli Radhakrishnan
Answer : D
✓ In India, the Teachers’ Day is celebrated every year on September 05 since 1962.
✓ The day marks the birth anniversary of Dr Sarvepalli Radhakrishnan, an Indian philosopher, academic,
and statesman, who defined the role of the teacher as someone who acts not only as an educator but also
as a moral mentor and inculcates values among students.
✓ He was the second President of India (1962 to 1967) and the first Vice President of India (1952–1962).

Q. Which team has won the 2023 Durand Cup?


A) Kerala Blasters B) Mohun Bagan Super Giant
C) Indian Army D) East Bengal
Answer : B
✓ The Kolkata club Mohun Bagan Super Giant defeated East Bengal FC by 1-0 to win the 2023 Durand
Cup.
✓ It was the 132nd edition of Durand Cup, the oldest football tournament in Asia.
✓ This was the 17th Durand Cup title won by Mohun Bagan Super Giant and the first since 2000.
✓ Golden Ball Award – Nandhakumar Sekar (East Bengal)
✓ Golden Boot Award – David Lalhlansanga (Mohammedan Sporting)
✓ Golden Glove Award – Vishal Kaith (Mohun Bagan Super Giant)

Q. Which player has won the 2023 Tata Steel Chess India Women’s Rapid Tournament?
A) Divya Deshmukh B) Dronavalli Harika
C) Koneru Humpy D) Vantika Agrawal
Answer : A
✓ Divya Deshmukh emerges winner of 2023 Tata Steel Chess India Women’s Rapid Tournament
✓ Divya Deshmukh, India’s 17-year-old Chess Woman Grandmaster from Maharashtra, has won the
women’s title in the rapid section of the Tata Steel Chess India tournament.
✓ Divya won all the three rounds of the tournament, which was held at the National Library in Kolkata on
September 01, 2023.
✓ Deshmukh finished on top with 7 points, followed by the world champion Ju Wenjun of China with 6.5
points and Russia’s Polina Shuvalova on 5.5 points.
Q. India defeated which country to win the inaugural Men’s Hockey5s Asia Cup 2023?
A) Japan B) Malaysia
C) Pakistan D) Oman
Answer : C
✓ India has won the Men’s Hockey5s Asia Cup 2023 by defeating Pakistan in the final match. Mor
Mandeep was the captain of team India.
✓ The event was held in Salalah, Oman. With the win, India has booked their place in the FIH Men’s
Hockey5s World Cup 2024. It will be held in Muscat.

Follow us: Official Site, Telegram, Facebook, Instagram, Instamojo 474


Q. SpaceX is in news recently launching its record breaking mission in 2023. How many orbital launch
missions have been launched in 2023 by SpaceX?
A) 65 B) 67
C) 61 D) 62
Answer : D
✓ SpaceX breaks record, launches 62nd mission of the year 2023
✓ Elon Musk-owned SpaceX has set a new launch record, when the company sent 21 of its Starlink internet
satellites to orbit on September 03, 2023.
✓ The launch took place atop a Falcon 9 rocket from NASA’s Kennedy Space Center in Florida.
✓ It was the 62nd orbital mission of SpaceX in 2023, breaking its own orbital launch record for most flights
in a year.
✓ In 2022, SpaceX launched 61 orbital missions for the full year.
✓ SpaceX
✓ Founded : 2002
✓ Founder : Elon Musk
✓ Headquarters : Hawthorne, California, United States
✓ CEO : Elon Musk
✓ President : Gwynne Shotwell

Q. Where was the International Aerospace Conference as a part of G20 initiative held?
A) Maharashtra B) Madhya Pradesh
C) Rajasthan D) Uttar Pradesh
Answer : B
✓ International Aerospace Conference jointly organized by MoCA and CII in Gwalior, Madhya Pradesh.
✓ ‘International Aerospace Conference: Moving Towards Inclusive Global Value Chains’ is being jointly
organised by the Ministry of Civil Aviation and Confederation of Indian Industry (CII) on 1st and 2nd
September 2023 in Gwalior, Madhya Pradesh.
✓ Shri Jyotiraditya M Scindia, Union Minister for Civil Aviation & Steel today inaugurated the event.
✓ The International Aerospace Conference is a G20 initiative in the aerospace sector under G20 and B20
priority, focusing on developing inclusive global value chains for Global trade and Investment.
Q. When is Ministry of Education celebrating the literacy week in 2023?
A) 1 September to 7 September B) 1 September to 8 September
C) 2 September to 8 September D) 8 September to 14 September
Answer : B
✓ Ministry of Education will organise a literacy week from 1st September to 8th September 2023 to
celebrate International Literacy Day and to create awareness among all citizens about the ULLAS- Nav
Bharat Saaksharta Karyakram.
✓ ULLAS (Understanding of Lifelong Learning for All in Society) of Nav Bharat Saksharta Karyakram
was launched by the Union Education Minister Shri Dharmendra Pradhan on 29.07.2023 in New Delhi
during the celebration of 3rd anniversary of National Education Policy (NEP) 2020.
✓ International Literacy Day is celebrated on 8 September.
✓ Ullas- New India Literacy Program is a Centrally Sponsored Scheme in line with the National Education
Policy (NEP) 2020, which will run from 2022 to 2027.
✓ The scheme comprises five components: Foundational Literacy and Numeracy, Important Life Skills,
Basic Education, Vocational Skills and Continuing Education.

Follow us: Official Site, Telegram, Facebook, Instagram, Instamojo 475


✓ The scheme's logo and slogan, "Understanding of Lifelong Learning for All in Society" (ULLAS) and
"Jan Jan Sakshar," were launched by the Union Education Minister, Shri Dharmendra Pradhan, on July
29, 2023, during the celebration of the 3rd anniversary of NEP 2020 in New Delhi.
✓ The ULLAS Mobile App serves as a platform where learners and volunteers can connect with literacy
and educational programs
Q. Which Northeastern state of India has become the first in the region to initiate Aadhaar Linked Birth
Registration (ALBR)?
A) Mizoram B) Nagaland
C) Assam D) Meghalaya
Answer : B
✓ Nagaland has become the first State in the North Eastern Region to initiate Aadhaar Linked Birth
Registration (ALBR).
✓ Nagaland aims to streamline birth registration by integrating Aadhaar enrolment for children in the age
group of 0 to 5 years.
✓ Nagaland has collaborated with the Unique Identification Authority of India (UIDAI) as a Registrar cum
Enrolment Agency for Child Enrolment via the ALBR process.
Q. Which US State has declared to observe the month of October as ‘Hindu Heritage’ month?
A) Virginia B) Florida
C) Georgia D) California
Answer : C
✓ Governor of Georgia Brian Kemp has declared October as ‘Hindu Heritage’ month.
✓ This declaration highlights the intention to celebrate Hindu heritage by emphasizing its rich culture and
the diverse spiritual traditions that originate from India.
✓ In April 2023, Georgia passed a historic resolution condemning Hinduphobia, making it the first US state
to do so.
Q. The Union Law Ministry has launched ‘One nation, One election’ committee. Who is the head of this
committee?
A) Ram Nath Kovind B) Amit Shah
C) Nitin Gadkari D) Rajnath Singh
Answer : A
✓ Government constitutes 8-member committee to examine ‘One nation, One election’: Head- Ram Nath
Kovind.
✓ Law Ministry has constituted an 8-member committee to examine ‘One nation, One election’. This
committee will be chaired by Ram Nath Kovind.
✓ The seven other members are- seven other members which includes Home Minister Amit Shah, Senior
Congress leader Adhir Ranjan Chowdhury, Ghulam Nabi Azad, NK Singh, Subhash C. Kashyap,
Harish Salve and Sanjay Kothari.
✓ The committee will examine and recommend specific amendments to the Constitution, the
Representation of the People Act and any other laws and rules which would require amendments for the
purpose of holding simultaneous elections.

Follow us: Official Site, Telegram, Facebook, Instagram, Instamojo 476


Q. The Ministry of Education and, Ministry Skill Development & Entrepreneurship have partnered with
which company to launch 3-year partnership “Education to Entrepreneurship?
A) IBM B) Microsoft
C) Meta D) Alphabet
Answer : C
✓ Govt partners with Meta for ‘Education to Entrepreneurship’ to skill 500,000 entrepreneurs
✓ Ministry of Education, Ministry Skill Development & Entrepreneurship along with Meta have launched
a 3-year partnership “Education to Entrepreneurship: Empowering a generation of students, educators
and entrepreneurs”. It was launched by Union Minister for Education and Skill Development &
Entrepreneurship Shri Dharmendra Pradhan.
✓ Under this partnership, 5 lakh entrepreneurs will get access to digital marketing skills by Meta over the
next 3 years. In the initial stage, the budding and existing entrepreneurs will be trained in digital
marketing skills using Meta platforms in 7 regional languages.
✓ For this, 3 Letters of Intent (LoI) were exchanged between Meta and National Institute for
Entrepreneurship and Small Business Development (NIESBUD), AICTE and CBSE
Q. The famous ISRO scientist who was responsible to give voice to the countdown of numerous ISRO
launches, including the recent Chandrayaan-3 mission. has passed away. Name the scientist?
A) Ritu Karidhal B) N Valarmathi
C) Tessy Thomas D) V. R. Lalithambika
Answer : B
✓ N Valarmathi, an ISRO scientist, who was the voice behind the countdown for the recent Chandrayaan-
3 mission, has passed away due to hear attack. She was 64.
✓ Valarmathi had been responsible for the countdowns for numerous launches by the Indian Space
Research Organisation (ISRO).
✓ She was the project director for the remote sensing RISAT-1 project in 2012.
Q. Which day is observed as the International Day of Charity to commemorate the death anniversary of
Mother Teresa?
A) 03 September B) 04 September
C) 05 September D) 02 September
Answer : C
✓ The International Day of Charity is observed annually on September 05. It was declared by the United
Nations General Assembly in 2012.
✓ September 5 was chosen in order to commemorate the death anniversary of Mother Teresa, who had
always been engaged in charitable work.
✓ The prime purpose of the International Day of Charity is to raise awareness and provide a common
platform for charity related activities all over the world for individuals, charitable, philanthropic and
volunteer organizations for their own purposes on the local, national, regional and international level.
✓ About Mother Teresa
✓ Mother Teresa was born in Macedonia on 26 August 1910.
✓ She was honoured in the Catholic Church as Saint Teresa of Calcutta.
✓ She received the Ramon Magsaysay Peace Prize in 1962 and Nobel Peace Prize in 1979.
✓ She became the Indian citizen in 1950 and was awarded Bharat Ratna in 1980.
✓ Mother Teresa Is known as the ‘Saint of the Gutters

Follow us: Official Site, Telegram, Facebook, Instagram, Instamojo 477


Q. How many teachers have been conferred with the National Teachers Award in 2023 by the Government
of India?
A) 75 B) 45
C) 65 D) 55
Answer : A
✓ National Teachers Awards 2023 conferred upon 75 teachers across the country.
✓ The Ministry of Education presents the National Teachers Awards on the occasion of Teachers’ Day
every year to acknowledge and encourage extraordinary and meritorious efforts of teachers from across
the length and breadth of the country for their excellent and unique contributions.
✓ In 2023, the President of India, Droupadi Murmu will confer National Awards on 75 teachers from across
the country.
✓ They include 50 School Teachers, 13 teachers from Higher education and 12 teachers from Ministry of
Skill Development & Entrepreneurship
Q. Which IIT’s researchers have developed a novel Cold-plasma Detergent in Environment (CODE) device
for good indoor air quality?
A) IIT Madras B) IIT Bombay
C) IIT Kanpur D) IIT Jodhpur
Answer : D
✓ Indian Institute of Technology Jodhpur researchers have developed a novel Cold-plasma Detergent in
Environment (CODE) device for good indoor air quality.
✓ The CODE device consumes less power, deactivates bacteria, moulds and viruses, captures dust and
pollen, and also removes odours.
✓ The device has been tested extensively which deactivates more than 99.99 percent of harmful pathogens
and provides quality indoor air.
✓ Air pollution is basically one of the top 5 risks causing chronic diseases, according to the World Health
Organisation
Q. Malini Rajurkar, who has passed away recently was well known for her which style of music?
A) Hindustani classical vocal B) Carnatic music
C) Indian folk music D) Khyal
Answer : A
✓ Noted Hindustani classical vocalist Malini Rajurkar, passed away at the age of 82 years.
✓ She was very popular for her ‘tappa’ style of music. Tappa is a captivating form of Indian semi-classical
vocal music known for its fast-paced and intricate melodies.
Q. The Exercise BRIGHT STAR- 23 is a multinational joint military exercise, taking place in which
country?
A) Indonesia B) India
C) Egypt D) South Africa
Answer : C
✓ Indian Army Participates in ‘Exercise BRIGHT STAR- 23’ in Egypt.
✓ The Indian Army is participating in the ‘Exercise BRIGHT STAR- 23’, which is the largest ever joint
military exercise in Middle East & North Africa region.

Follow us: Official Site, Telegram, Facebook, Instagram, Instamojo 478


✓ The first edition of the Exercise was conducted in the year 1980 in Egypt.
✓ Indian Army contingent comprising of 137 personnel is participating with INS Sumedha in the exercise.
✓ The exercise has been organised at Mohammed Naguib Military Base in Egypt from 31 August to 14
September 2023.
✓ Overall 34 countries are participating in the 2023 edition of the multinational Tri-Services military
exercise.
✓ The joint military exercise will be led by US CENTCOM and Egyptian Army.
✓ Exercise BRIGHT STAR- 23 will provide opportunity to Indian Army to share best practices and
experiences with other armies for enhancing overall defence cooperation.
Q. What is the name of the World’s Most Advanced Arabic Large Language Model, which was released
recently?
A) Gyaan B) Ruez
C) Jais D) Rooh
Answer : C
✓ Inception, a unit of Abu Dhabi AI company G42, has released ‘Jais’, the world’s most advanced Arabic
large language model.
✓ Jais is a bilingual Arabic-English model that has been trained on a massive dataset of text and code.
✓ It can be used for a variety of tasks, such as machine translation, text summarization, and question
answering.
✓ It was trained on the Condor Galaxy, the world’s largest AI supercomputer, using 116 billion Arabic
tokens and 279 billion English tokens
Q. Who are the main beneficiaries of 'Project Naman' launched by the Indian Army?
A) Veterans and families of deceased personnel
B) Service Before Self
C) May the Lord of Water be auspicious unto us
D) Touch the Sky with Glory
Answer : A
✓ The Indian Army recently launched 'Project Naman', which will entail establishing facilitation and
grievance redressal centres for veterans and the kin of personnel who have lost their lives.
✓ About Project Naman:
✓ The Project will entail establishing facilitation and grievance redressal centres for veterans and the kin of
personnel who have lost their lives.
✓ The first such centre will be established in Delhi Cantonment.
✓ 'Naman' will house a Common Service Centre, which will facilitate the veterans and their relatives.
✓ It will offer all government-to-customer services and also facilitate the updating of pensioners' accounts
on the SPARSH portal of veterans, next of kin and dependents.
✓ What is the SPARSH portal?
✓ It is being implemented by the Ministry of Defence to meet the pension sanction and disbursement
requirements for Armed Forces viz. Army, Navy, Air Force and Defence Civilians.
Q. The state government of Uttarakhand observes the Himalayan Day every year on which day?
A) 07 September B) 10 September
C) 08 September D) 09 September
Answer : D

Follow us: Official Site, Telegram, Facebook, Instagram, Instamojo 479


✓ The Himalayan Day is observed every year on September 09 in the state of Uttarakhand.
✓ The day was officially declared in 2010, by the then Chief Minister of Uttarakhand, Ramesh Pokhriyal
“Nishank”.
✓ The aim of celebrating Himalayan Day is to save and maintain the Himalayas and mountain nature.
✓ The year 2023 marks the 14th anniversary of the Himalayan Day.

UTTARAKHAND IN NEWS 2023


▪ Harela Festival 2023 begins in Uttarakhand.
▪ Uttarakhand has become India’s first state to launch the Resham Keet Bima scheme for sericulturists.
▪ Uttarakhand tableau which showcased the state’s wildlife and religious sites at the 74th Republic Day
parade has won the top prize.
▪ India’s First Green Solar Panel Factory to Build by Luminous in Uttarakhand.
▪ Uttarakhand awarded for adventure tourism & all round development of tourism.
▪ Uttarakhand Chief Minister launches Lakhpati Didi Yojana.
▪ The Uttarakhand High Court will be shifted from Nainital to Haldwani.
▪ Uttarakhand Plans Genetic Enhancement of Its Indigenous Badri Cow.
▪ Uttarakhand Government appoints Prasoon Joshi as its Brand Ambassador.
▪ Uttarakhand Gaurav Samman: NSA Ajit Doval and late CDS Gen Rawat to receive award.
▪ Uttarakhand CM Launched SBI Foundation and HESCO’s Project for Chamoli.
▪ Uttarakhand Foundation Day is observed on November 9 every year.
▪ Uttarakhand Governor Approved 30% Women’s Reservation Bill.
▪ PM Modi has inaugurated connectivity projects worth 3,400 crores in Mana village of Chamoli district
of Uttarakhand.
▪ Uttarakhand recently received the first prize in two categories - the best adventure tourism destination
and all-round development of tourism - from the Union tourism ministry.
▪ India's first commercial space observatory will be set up in the Garhwal division of Uttarakhand.
▪ Uttarakhand Chief Minister Pushkar Singh Dhami has launched the “Mukhyamantri Udyaman Khiladi
Unnayan Yojana” on the occasion of National Sports Day.
▪ Uttarakhand education department launched an e-governance portal “Samarth”.
Q. Which city in United States has declared to observe September 03 as the Sanatana Dharma Day every
year?
A) Seattle B) Boston
C) Miami D) Louisville
Answer : D
✓ United States city Louisville declares September 3rd as Sanatana Dharma Day.
✓ The Louisville city in the US state of Kentucky, has declared September 3 to be observed as Sanatana
Dharma Day in the city.
✓ The Mayor of Louisville, Craig Greenberg attended a re-consecration ceremony at the Hindu Temple of
Kentucky where his deputy Barbara Sexton Smith read out the official proclamation on September 3.

Q. Wide Field Survey Telescope (WFST) is being developed by which country?


A) China B) Russia
C) India D) United States
Answer : A
✓ China is set to launch the Wide Field Survey Telescope (WFST) in September.
✓ About Wide Field Survey Telescope:
✓ It is the largest time-domain survey facility in the Northern Hemisphere.

Follow us: Official Site, Telegram, Facebook, Instagram, Instamojo 480


✓ Objectives:
✓ The core scientific objective is to explore and monitor dynamic astronomical events and conduct time-
domain astronomical observation research.
✓ This will enable researchers to detect faint and distant celestial signals, including those from galaxies
beyond the Milky Way and galaxy clusters.
✓ It will also seek out planets or their moons in the Kuiper Belt and beyond.
✓ The WFST project was launched in July 2019 through a collaboration between the University of Science
and Technology of China and the Purple Mountain Observatory under the Chinese Academy of
Sciences.
Q. Foxconn pulls out of joint venture with Vedanta to make semiconductors. Foxconn is company from
which country?
A) China B) Russia
C) United States D) Taiwan
Answer : D
✓ India's Make in India initiative suffered a major loss when Vedanta Limited's agreement with Taiwanese
company Foxconn to manufacture semiconductors in India was cancelled.
✓ It was established production for semiconductors in India.
✓ According to a report, the company has expressed confidence about India's semiconductor development.
The company will fully support the "Make in India" campaign of the Government of India.
✓ According to Vedanta, it will work with local partners to meet the needs of its stakeholders.
✓ Foxconn and Vedanta last year signed an agreement to invest $19.5 billion to build a semiconductor and
display production plant in Gujarat.
✓ Vedanta Limited:
✓ Vedanta Resources Limited
✓ Founded : 2003
✓ Founder : Anil Agarwal
✓ Headquarters : London, United Kingdom
✓ Executive Chairman : Anil Agarwal
✓ Executive Vice Chairman : Navin Agarwal
✓ CEO : Srinivasan Venkatakrishnan
Q. When is the International Literacy Day observed globally?
A) September 09 B) September 08
C) September 10 D) September 07
Answer : B
✓ International Literacy Day is observed annually on September 8 across the world to highlight the
importance of literacy to individuals, communities and societies.
✓ The day was declared by UNESCO on 26 October 1966 at 14th session of UNESCO’s General
conference. It was celebrated for the first time in 1967.
✓ The theme of the International Literacy Day in 2023 is ‘Promoting literacy for a world in transition:
Building the foundation for sustainable and peaceful societies’.
Q. Sabudana of Salem district of which state has recently been given GI tag?
A) Assam B) Tamil Nadu
C) Rajasthan D) Uttar Pradesh
Answer : B

Follow us: Official Site, Telegram, Facebook, Instagram, Instamojo 481


✓ Salem district in the state of Tamil Nadu has gained acclaim for its sago production, which is widely
recognized as Sabudana.
✓ Salem Sago, locally known as Javvarisi, is derived from the wet starch powder extracted from tapioca
roots. Indian tapioca roots are known to contain approximately 30-35% starch content.
✓ Sago production has been a cornerstone of Salem’s economic development since 1967.
✓ Currently, over 80% of sago in India is produced in the Salem region, with a significant portion marketed
through Sagoserve.

TAMIL NADU IN NEWS 2023


▪ Taiwanese firm Foxconn Technology Group, a key supplier to Apple Inc., signed a letter of Intent with
the Tamil Nadu government to invest ₹1,600 crore to establish a manufacturing facility in Kancheepuram
district.
▪ Centre has approved the establishment of a new spaceport in Kulasekarapattinam, Tamil Nadu for
carrying out the launches of the Small Satellite Launch Vehicles (SSLV) developed by the ISRO.
▪ Tamil Nadu's Authoor betel leaves get Geographical Indication (GI) certificate.
▪ Tamil Nadu bagged the Best Performing State award 2023 in cadaver organ donation .
▪ Tamil Nadu Youth Welfare and Sports Development Minister "Udhayanidhi Stalin" has unveiled the
trophy and the ‘Bomman’ mascot for the prestigious Asian Champions Trophy 2023 hockey tournament
to be held in Chennai.
▪ Muthamizh Selvi, first Tamil Nadu woman to scale Mt Everest.
▪ Tamil Nadu government declared Thanthai Periyar Wildlife Sanctuary as the 18th wildlife sanctuary in
the state..
▪ Tamil Nadu becomes first state to launch its own Climate Change Mission.
▪ Tamil Nadu govt to launch five-year Tamil Nadu Millet Mission.
▪ In a first of its kind initiative in India, the Government of Tamil Nadu announced the 'Nilgiri Tahr
Project' .
Q. To which country has India handed over the presidency of G20 during the New Delhi G20 summit?
A) Indonesia B) USA
C) Japan D) Brazil
Answer : D
✓ During the New Delhi G20 summit, PM Modi has handed over the responsibility of the next presidency
of the G20 to Brazil.
✓ PM Modi handed over the formal responsibility of the post of President to Brazilian President Luiz Inacio
Lula da Silva and promised all possible help. On December 1 last year, India took over the chairmanship
of G-20 from Indonesia, which was the chairperson at that time.
✓ India will remain the President of G20 till 30 November

Q. G20 leaders commit to WTO Reforms To Have a Fully Functional Dispute Settlement System by __.
A) 2023 B) 2024
C) 2026 D) 2028
Answer : B
✓ The G20 leaders expressed their commitment to conduct discussions for having a "fully and well-
functioning" dispute settlement system of the World Trade Organisation (WTO) by 2024.
✓ Besides formulating norms for global exports and imports, the Geneva-based 164-member multi-lateral
body adjudicates trade disputes among the member countries. The dispute settlement mechanism of the
WTO has been derailed due to a non-functional appellate body since December 2019.

Follow us: Official Site, Telegram, Facebook, Instagram, Instamojo 482


✓ MC, which is the highest decision-making body of the WTO, is scheduled to meet in February next year
(2024) in Abu Dhabi.
✓ There are two main ways to settle a dispute once a complaint has been filed in the WTO -
✓ First : The countries find a mutually agreed solution, particularly during the phase of bilateral
consultations;
✓ Second : Through adjudication which includes ruling by a panel and if not satisfied, challenging that
ruling at the appellate body

Q. Novak Djokovic, who won the US Open 2023 men's singles title, is a player from which country?
A) Serbia B) Spain
C) Belarus D) Georgia
Answer : A
✓ Serbia's Novak Djokovic won his fourth US Open men's singles title by defeating Daniil Medvedev in
the final at Flushing Meadows, New York. This was his 24th Grand Slam title.
✓ Djokovic won his first major title at the 2008 Australian Open
✓ Grand Slam Tennis Winners 2023
✓ US Open 2023
Men’s Single – Novak Djokovic (Serbia) defeated Daniil Medvedev (Russia)
Women’s Single – Coco Gauff (United States) defeated Aryna Sabalenka (Belarus)
✓ Wimbledon Open 2023
Men’s Single – Carlos Alcaraz (Spain )defeated Novak Djokovic ( Serbia)
Women’s Single –Markéta Vondroušová (Czech Republic ) defeated Ons Jabeur (Tunisia)
✓ French Open 2023
Men’s Single – Novak Djokovic (Serbia ) defeated Casper Ruud (Norway)
Women’s Single –Iga Świątek (Poland ) defeated Karolína Muchová (Czech Republic)
✓ Australian Open 2023
Men’s Single – Novak Djokovic (Serbia) defeated Stefanos Tsitsipas (Greece)
Women’s Single –Aryna Sabalenka (Belarus) defeated Elena Rybakina (Kazakhstan)

Q. Who won the US Open 2023 women's singles title?


A) Arina Sabalenka B) Erin Routliff
C) Coco Gauff D) Gabriela Dabrowski
Answer : C
✓ American tennis player Coco Gauff has won the US Open 2023 women's singles title. She won this title
by defeating Aryna Sabalenka of Belarus in the final. This is Gauff's first Grand Slam title
Q. Where is the world's highest fighter airfield being developed?
A) Srinagar B) Baku
C) Kathmandu D) Ladakh
Answer : D
✓ India is building the world's highest fighter airfield at Nyoma in Ladakh.
✓ This airfield, being prepared by the Border Road Organization, is located in eastern Ladakh, which is an
important location to compete with China.
✓ This project is being prepared at a cost of Rs 218 crore

Follow us: Official Site, Telegram, Facebook, Instagram, Instamojo 483


Q. Which player has completed the fastest 13000 runs in One Day International cricket?
A) Babar Azam B) Steve Smith
C) Virat Kohli D) David Warner
Answer : C
✓ India's star batsman Virat Kohli has become the fastest batsman to complete 13000 runs in One Day
International cricket. Kohli achieved this milestone in just 267 innings.
✓ Kohli has become the fifth batsman in the world to score 13,000 ODI runs.
✓ Along with this, Virat Kohli also scored his 47th ODI century. Kohli is just two centuries away from
equaling Sachin's record of ODI centuries.
✓ Before this, Sachin Tendulkar, Ricky Ponting, Sanath Jayasuriya and Mahela Jayawardene have
achieved this position.

Q. With whom has NTPC Green Energy Limited signed an agreement for the Green Hydrogen Project?
A) Nayara Energy B) Tata Green
C) Adani Green D) Reliance Green
Answer : A
✓ NTPC Green Energy Limited, a subsidiary of NTPC Limited, has signed an MoU with downstream
energy company Nayara Energy.
✓ The objective of this agreement is to explore opportunities in the green hydrogen and green energy sector.
This agreement is in line with PM Modi's vision of self-reliant India.
✓ Nayara Energy is an international downstream company with strong capabilities in hydrocarbons from
refining to retail

Q. Which player won the men's singles title at Indonesia Masters 2023?
A) Kiran George B) Lakshya Sen
C) Koo Takahashi D) Priyanshu Rawat
Answer : A
✓ Indian badminton player Kiran George has won the men's singles title in Indonesia Masters 2023.
✓ Kiran George, ranked 50th in badminton rankings, defeated Ku Takahashi of Japan.
✓ Kiran George had won the Odisha Open title last year by defeating Priyanshu Rawat in the final.
✓ Kiran George was the only Indian to reach the final of Indonesia Masters 2023
Q. Odisha government has announced to provide financial assistance of how much rupees to the athletes of
the state participating in the upcoming Asian Games?
A) 10 lakhs B) 20 lakhs
C) 30 lakhs D) 40 lakhs
Answer : A
✓ The Odisha government has announced financial assistance of Rs 10 lakh for each of the 13 athletes from
the state participating in the upcoming Asian Games in Hangzhou, China

Q. Which aircraft manufacturer has started delivery of C295 transport aircraft to the Indian Air Force?
A) Lockheed Martin B) Airbus
C) Dassault Aviation D) Boeing

Follow us: Official Site, Telegram, Facebook, Instagram, Instamojo 484


Answer : B
✓ Airbus Defense and Space on Wednesday handed over the first of 56 C295 transport aircraft to the Indian
Air Force, beginning the delivery of the aircraft under the Rs 21,935 crore project.
Q. When is Hindi Diwas celebrated every year?
A) 12 September B) 13 September
C) 14 September D) 15 September
Answer : C
✓ Hindi Diwas is celebrated every year on 14 September. Hindi is the fourth most spoken language in the
world.
✓ More than 80 crore people all over the world speak and understand Hindi language.
✓ Hindi was adopted as the official language by the Constituent Assembly of India in the year 1949.
✓ National Hindi Day was celebrated for the first time in the year 1953.
✓ World Hindi Day is celebrated every year on January 10.
✓ As per Article 343(1) of the Constitution of India, Hindi in Devanagari script shall be the official language
of the Union

Q. In which state will the fourth edition of the World Trade Expo be organized?
A) Maharashtra B) Uttar Pradesh
C) Assam D) Bihar
Answer : A
✓ Maharashtra Governor Ramesh Bais will inaugurate the fourth edition of the World Trade Expo 2023.
It is India's largest trade platform to assist global trade relations especially for MSMEs.
✓ The expo will be organized on 3-4 October 2023 at the World Trade Center, Mumbai.
✓ This event will be jointly organized by the World Trade Center Mumbai and All India Association of
Industries (AIAI).

Q. In which state the mobile application app 'Sarpanch Samvad' was unveiled?
A) Bihar B) Rajasthan
C) Assam D) Haryana
Answer : C
✓ Assam Governor Gulab Chand Kataria unveiled 'Sarpanch Samvad', a mobile application of Quality
Council of India (QCI) at Raj Bhavan in Guwahati.
✓ QCI has launched Sarpanch Samvad Abhiyaan as an initiative aimed at connecting approximately 2.5
lakh Sarpanches across India acting as a holistic platform for networking, knowledge dissemination and
collaboration.

Q. When is International Democracy Day celebrated every year?


A) 13 September B) 14 September
C) 15 September D) 16 September
Answer : C
✓ International Day of Democracy is celebrated across the world on 15 September.
✓ Theme 2023 – Empowering the next generation.

Follow us: Official Site, Telegram, Facebook, Instagram, Instamojo 485


✓ It was implemented through a resolution by the United Nations General Assembly (UNGA) in the year
2007.
✓ International Democracy Day was celebrated for the first time in the year 2008.

Q. When is National Engineers Day celebrated every year?


A) 14 September B) 15 September
C) 16 September D) 17 September
Answer : B
✓ National Engineers' Day in India is celebrated every year on 15 September to commemorate the birth
anniversary of the famous Indian engineer Sir Mokshagundam Visvesvaraya.
✓ The theme of National Engineers Day 2023 is 'Engineering for a sustainable future'.
Q. Which of the following has organized the “Vaishali Festival of Democracy” at Bihar’s Nalanda
University exhibiting India’s rich democratic traditions?
A) NITI Aayog B) Finance Ministry
C) Indian Council for Cultural Relations D) Election Commission of India
Answer : C
✓ The Indian Council for Cultural Relations (ICCR) has organized the “Vaishali Festival of Democracy”
at Bihar’s Nalanda University exhibiting India’s rich democratic traditions.
✓ The festival – the ‘Vaishali Festival of Democracy’, will be held on Friday (September 15), which is also
the International Day of Democracy.
✓ The ICCR chose the university as the venue due to its great significance for Bihar, the democratic ethos
of the country, and the presence of an international audience.
✓ Vaishali in Bihar is the place where the roots of democracy are traced.
Q. Which state government has initiated a special Gift Deed Scheme?
A) Odisha B) Assam
C) Uttar Pradesh D) Rajasthan
Answer : C
✓ UP Govt launches Gift Deed scheme to reduce property related Issues.
✓ In a bid to foster harmony and alleviate property-related family disputes, the Yogi government initiated
a special Gift Deed Scheme which has benefited over 43,574 people from 5 August 2023, to 12 September
2023.
✓ The public got a benefit of Rs 1807.31 crore under this scheme.
✓ Under the new scheme, the stamp duty for such transactions has been reduced to Rs 5,000.
Q. Which state government has recently approved nine industrial projects involving a cumulative investment
of Rs 1,01,804 crore?
A) Odisha B) Madhya Pradesh
C) Rajasthan D) Karnataka
Answer : A
✓ The Odisha government has approved nine industrial projects involving a cumulative investment of Rs
1,01,804 crore.
✓ The projects are expected to create employment for over 28,000 people.

Follow us: Official Site, Telegram, Facebook, Instagram, Instamojo 486


✓ The High-Level Clearance Authority (HLCA) headed by Chief Minister Naveen Patnaik has approved
the investment projects during its meeting.
✓ These projects will be set up in the districts of Jagatsinghpur, Jajpur, Jharsuguda, Khurda, Paradeep,
Bhadrak, Ganjam, and Koraput districts.
Q. Shipping and Waterways Minister Sarbananda Sonowal has represented India at the eighth Eastern
Economic Forum in which country?
A) Iran B) Bangladesh
C) Afghanistan D) Russia
Answer : D
✓ Shipping and Waterways Minister Sarbananda Sonowal has represented India at the eighth Eastern
Economic Forum in the Russian port city of Vladivostok.
✓ The objective of the meeting is to maintain strong ties and foster strategic, trade, and logistical
cooperation across various sectors.
✓ Sonowal expressed confidence that his visit will catalyze the efforts to develop an alternative trade route
between Vladivostok and Chennai
Q. Which state government has launched social welfare scheme “Kalaignar Magalir Urimai Thogai
Thittam”?
A) Tamil Nadu B) Rajasthan
C) Haryana D) Bihar
Answer : A
✓ Tamil Nadu government has launched social welfare scheme “Kalaignar Magalir Urimai Thogai
Thittam”.
✓ Under the scheme, Tamil Nadu government will provide monthly assistance of Rs 1,000 to more than
1.06 crore female family heads through Kalaignar Magalir Urimai Thogai Thittam.
✓ Beneficiaries will receive financial assistance through direct bank transfers and will be provided with
ATM cards for easy access.
✓ The government emphasizes on effective implementation and will use SMS notifications to communicate
with beneficiaries.
Q. India and which country have recently signed a Memorandum of Understanding (MoU) on cooperation
in the Energy Sector?
A) Kuwait B) Russia
C) UAE D) Saudi Arabia
Answer : D
✓ India and Saudi Arabia have signed a Memorandum of Understanding (MoU) on cooperation in the field
of energy.
✓ The MoU is expected to result in enhanced ties between India and Saudi Arabia in renewable energy,
energy efficiency, hydrogen, electricity and grid interconnections, petroleum, natural gas, strategic
petroleum reserves, and energy security.
Q. Who has been presented with the Lifetime Achievement Award in Academia?
A) Najma Akhtar B) Deepak Shenoy
C) Shanti Lakra D) Neeli Bendapudi

Follow us: Official Site, Telegram, Facebook, Instagram, Instamojo 487


Answer : A
✓ Jamia Millia Islamia (JMI) Vice Chancellor, Professor Najma Akhtar was presented ‘The Lifetime
Achievement Award-Academia.
✓ The award was presented to her by Dr. Raghunath Anant Mashelkar (Padma Vibhushan) at the Grand
Conference and award function titled “Making India Employable” was organized by TeamLease
EdTech.
✓ Akhtar was selected for the award by a jury panel comprising of educationists and industry experts. She
has been awarded for her “great efforts to connect education to employment”.
Q. Who has been awarded the prestigious “Dr AM Gokhale Award” for ophthalmology?
A) Ashvini Kumar Dwivedi B) RG Patel
C) AM Gokhale D) Sanjay Kumar Mishra
Answer : D
✓ Renowned ophthalmologist Brig Sanjay Kumar Mishra of Indian Army’s Research and Referral Hospital
was conferred with the prestigious “Dr AM Gokhale Award” in Pune.
✓ Dr AM Gokhale was a renowned teacher of Ophthalmology and his students progressed to become the
doyens of Ophthalmology in the country.
✓ Brig Mishra is one of the highest decorated officers of Indian Armed Forces Medical Services.

Q. Which country has won the Men’s FIBA Basketball World Cup 2023 after defeating Serbia?
A) Germany B) Finland
C) Sweden D) Argentina
Answer : A
✓ Germany won their first-ever men’s FIBA Basketball World Cup 2023 title by beating Serbia in Manila.
✓ The 19th edition of FIBA’s flagship event, co-hosted by the Philippines, Indonesia, and Japan.
✓ Germany became the first team to win a FIBA World Cup crown in their final debut since Spain in 2006.

Q. Dennis Austin passed away recently. He was the co-creator of which software?
A) Power Point B) Excel
C) Tally D) Notepad
Answer : A
✓ One of the co-creators of Microsoft PowerPoint, Dennis Austin died at the age of 76.
✓ Dennis Austin, along with Robert Gaskins, developed PowerPoint and introduced it in 1987.
✓ Austin and Gaskins were both part of the software company Forethought, which was later acquired by
Microsoft for $14 million shortly after the launch of PowerPoint, which was originally named Presenter.
✓ Initially, PowerPoint was available exclusively for the Macintosh platform

Q. Indian Airforce has received First C-295 Transport Plane from which company?
A) Airbus B) Boeing
C) Dassault Aviation D) HAL
Answer : A
✓ The Air Chief Marshal VR Chaudhari, Chief of the Indian Air Force, has received the first C-295
transport plane for India from global aircraft manufacturer Airbus at a facility in Seville, Spain.

Follow us: Official Site, Telegram, Facebook, Instagram, Instamojo 488


✓ India has contracted 56 of these planes, with 16 being manufactured in Spain and the remaining 40
produced by a Tata-Airbus joint venture in Vadodara, Gujarat.
Q. Thiruvonam is a festival prevalently celebrated in which state?
A) Kerala B) Andhra Pradesh
C) Tamil Nadu D) Karnataka
Answer : A
✓ Thiruvonam, the most auspicious day during the Onam festival, is celebrated throughout Kerala. Onam
is a harvest festival that celebrates the return of King Mahabali/Maveli to his beloved state.
✓ The festival is celebrated for ten days and each day holds a lot of significance. Onam also marks the
beginning of the Malayalam year, called Kolla Varsham
Q. Which state government will provide Rs 10 lakh as compensation to the kin of person killed in a mob
lynching incident?
A) Madhya Pradesh B) Gujarat
C) Rajasthan D) Haryana
Answer : A
✓ Madhya Pradesh government will provide Rs 10 lakh as compensation to the kin of any person killed in
a mob lynching incident.
✓ There will be a provision of compensation of Rs 4 to Rs 6 lakh for the injured.
✓ This will also include all those incidents of mob lynching that occur due to religion, caste, language or
any other reason. Under the scheme, mob lynching will be considered in which five or more accused are
involved.
Q. India launched ‘Infrastructure Financing Bridge’ along with which country?
A) USA B) UK
C) Australia D) New Zealand
Answer : B
✓ India and the UK have announced the launch of an infrastructure financing bridge to facilitate major
infrastructure projects, as revealed by India’s Finance Minister, Nirmala Sitharaman.
✓ The two nations have also agreed to conduct investment negotiations concurrently with goods
negotiations as part of their free trade agreement (FTA) discussions

Q. India recently organised the meeting of Strategic Partnership Council (SPC) formed with which country?
A) Brazil B) Argentina
C) Egypt D) Saudi Arabia
Answer : D
✓ Saudi Arabia’s Crown Prince Mohammed bin Salman and India’s Prime Minister Narendra Modi held
a landmark summit, marking the first meeting of the India-Saudi Arabia Strategic Partnership Council
(SPC), which was established in 2019.
✓ The council consists of two main pillars: the Committee on Political, Security, Social, and Cultural
Cooperation, and the Committee on Economy and Investments, each with several levels of engagement.
The two nations agreed to expedite the USD 50-billion West Coast refinery project.

Follow us: Official Site, Telegram, Facebook, Instagram, Instamojo 489


Q. According to US News & World Report’s annual Best Countries rankings, Which country has once again
claimed the title of the world’s best country?
A) Norway B) Denmark
C) Switzerland D) Sweden
Answer : A
✓ Switzerland has once again claimed the title of the world’s best country, according to the latest U.S. News
& World Report’s annual Best Countries rankings.
✓ This marks Switzerland’s second consecutive year at the pinnacle and its sixth time overall as the No. 1
nation on the list.
✓ Switzerland is followed by Canada at No. 2, Sweden at No. 3, Australia at No. 4, and the United States
at No. 5. The United States has slipped one spot compared to its 2022 ranking.
✓ In 2023, India’s ranking improved by one position, securing the 30th spot with an overall score of 40.8

Q. Which indian Company in TIME's "The World's Best Companies of 2023" List?
A) TATA B) HCL
C) Infosys D) Mahindra & Mahindra
Answer : C
✓ Infosys, the Bengaluru-headquartered IT services provider, has secured a coveted spot on TIME
magazine’s ‘The World’s Best Companies of 2023’ list.
✓ Infosys stands out as the only Indian company to make it into the top 100 rankings, claiming the 64th
position with an impressive overall score of 88.38.
✓ About Infosys : Founded in 1981 by four engineers, including NR Narayana Murthy, Infosys has grown
into India’s second-largest IT company in terms of revenue
1. Microsoft ( United States ) 2. Apple (United States )
3. Alphabet ( United States ) 4. Meta Platforms (United States)
5. Accenture (Ireland )

Q. Who will inaugurate a 108-foot-tall statue of Adi Shankaracharya in the temple town of Omkareshwar ?
A) Narendra Modi B) Amit Shah
C) Draupadi Murmu D) Shivraj Singh Chouhan
Answer : D
✓ Madhya Pradesh Chief Minister Shivraj Singh Chouhan will inaugurate a 108-foot-tall statue of Adi
Shankaracharya in the temple town of Omkareshwar on September 18, 2023.
✓ Omkareshwar is famous for being home to one of the 12 Jyotirlingas, which are sacred shrines dedicated
to Lord Shiva. The towering structure has been named "Ekatmata Ki Pratima" (statue of oneness).
Q. Which state is host to the 14th World Spice Congress in 2023?
A) Gujarat B) Maharashtra
C) Kerala D) Tamil Nadu
Answer : B
✓ Recently, the 14th edition of the World Spice Congress (WSC) began at Vashi in Navi Mumbai,
Maharashtra.

Follow us: Official Site, Telegram, Facebook, Instagram, Instamojo 490


✓ The theme of the WSC 2023 is VISION 2030: S-P-I-C-E-S, which stands for sustainability, productivity,
innovation, collaboration, excellence and safety.
✓ The World Spice Congress, which is organized by Spices Board India, will be conducted on the sidelines
of India’s G20 presidency
Q. Which state government has signed an MoU with six Public Sector Banks for providing banking services
in all unbanked Gram Panchayats?
A) Bihar B) Chhattisgarh
C) Odisha D) Jharkhand
Answer : C
✓ Odisha Govt has signed an MoU with six Public Sector Banks, namely SBI, PNB, Union Bank, UCO
Bank, Bank of India, Bank of Baroda for providing banking services in all unbanked Gram Panchayats
of the State through CSP Plus banking outlets.
✓ The scheme will be implemented in a phased manner to cover all unbanked GPs of State within the
current financial year 2023-24.
✓ Odisha Govt will provide rent free banking space for 5 years and also bear one-time expenses for Fixed
Cost & Recurring expenses for a period of 3 years.
✓ A provision of ₹500 Cr has been made for the scheme..
Q. Which company has sponsored ICC Cricket World Cup 2023 on Star Sports and Disney+Hotstar?
A) Adani Group B) TATA Group
C) Reliance Industries D) Mahindra and Mahindra
Answer : D
✓ Mahindra & Mahindra has unveiled its new partnership as an associate sponsor for the upcoming ICC
Cricket World Cup 2023.
✓ The automotive giant will take on the role of broadcast associate sponsor for Star Sports, while also
serving as the co-powered sponsor for Disney+Hotstar during the ICC Men's Cricket World Cup 2023

Follow us: Official Site, Telegram, Facebook, Instagram, Instamojo 491


BEST MCQ WITH FACTS
AUGUST 2023

Q. Warship 'Mahendragiri' has been built by which Shipbuilders Limited?


A) Goa Shipyard Limited B) Cochin Shipyard Limited
C) Mazagon Dock Shipbuilders D) ABG Shipyard Limited
Answer : C
✓ The wife of the Vice President will launch India’s latest warship, Mahendragiri, at the Mazagon Dock
Shipbuilders Limited, Mumbai.
✓ About Mahendragiri Frigate:
✓ It is the seventh and last stealth frigate of Project 17A Frigates.
✓ It is named after a mountain peak in Eastern Ghats located in Odisha.
✓ The ship is being built by the Mazagon Dock Shipbuilders Limited (MDL) in Mumbai.
✓ What is Project 17A?
✓ The project was launched by the defence forces of India to construct a series of stealth guided-missile
frigates.
✓ Under the Project 17A programme, four ships by Mumbai-based Mazagon Dock Shipbuilders Limited
(MDL) and three by Garden Reach Shipbuilders and Engineers Limited (GRSE) are being built.
✓ These warships follow the Project 17 Class Frigates (Shivalik Class) and boast enhanced stealth features,
advanced weapons, sensors, and platform management systems.
✓ Project 17A ships have been designed in-house by the Indian Navy’s Warship Design Bureau WDB.
✓ As much as75% of the orders for equipment and systems of Project 17A ships are from indigenous firms,
including MSMEs.
✓ The first six ships of the project have been launched so far by MDL & GRSE between 2019-2023.

Q. Who developed 'Nabhmitra' device for fishers’ safety successfully tested at Neendakara?
A) ISRO B) DRDO
C) IIT Madras D) IIT Bombay
Answer : A
✓ The Indian Space Research Organisation (ISRO) has achieved a significant milestone by successfully
testing its innovative device named ‘Nabhmitra’.
✓ This device, developed at ISRO’s Space Applications Centre in Ahmedabad, aims to enhance the safety
of fishermen during their sea expeditions.
✓ This device is aimed at enhancing the safety of fishermen. It operates through satellite-based
communication and enables two-way messaging between boats at sea and authorities on land.
✓ The system can convey weather alerts, cyclone warnings, and other information in the local language.
✓ In cases of emergencies like boat accidents or fires, fishermen can activate the device to alert the control
centre. The control centre receives the boat’s location and the crew onboard gets a response from the
control centre
Q. Who has been appointed as the first woman Chairman and CEO of Railway Board?
A) Samiksha Singh B) Jaya Verma Sinha
C) Jayanti Sharma D) Hemlata Kushwaha
Answer : B

Follow us: Official Site, Telegram, Facebook, Instagram, Instamojo 492


✓ The Central Government has appointed Jaya Verma Sinha as the first woman Chairman and CEO of the
Railway Board.
✓ She will replace Anil Kumar Lahoti at this position.
✓ Railway Board is the apex decision making body in Railways. Jaya Verma Sinha has been an alumnus
of Allahabad University.
✓ Jaya joined the Indian Railway Traffic Service (IRTS) in 1988.
✓ Jaya Verma Sinha has also worked as Chief Commercial Manager in South Eastern Railway.

Q. Who won the title of Miss Earth India 2023?


A) Priyan Sen B) Vanshika Parmar
C) Sweta Sharda D) Rakhi Kapoor
Answer : A
✓ Priyan Sen, who hails from Rajasthan, won the Miss Earth India 2023 title during the Miss Divine Beauty
2023 National Finals held in New Delhi.
✓ She was crowned by Korea's current Miss Earth Mina Soo Choi.
✓ While Praveena Anjana was named Miss International India 2023, both Pema Choden Bhutia and
Tejaswini Srivastava were declared runners-up.
Q. Which country will host the first edition of the Global India AI 2023 conference?
A) India B) Germany
C) Japan D) France
Answer : A
✓ India will host the first edition of the Global India AI 2023 conference in October this year.
✓ It is a good platform for AI applications in next-generation learning, artificial intelligence models, electric
vehicles and computing systems.
✓ The event is being organized by the Ministry of Electronics and Information Technology (MeitY) which
will see participation from prominent AI experts, researchers, startups, and investors from across the
world.
Q. Who has become the first Indian to win a gold medal in the World Athletics Championships?
A) Neeraj Chopra B) DP Manu
C) Kishore Jena D) Rohit Yadav
Answer : A
✓ Neeraj Chopra won the first gold medal for India at the World Athletics Championships.
✓ Javelin thrower Neeraj Chopra won the first gold medal for India at the World Athletics Championships
in Budapest, Hungary
✓ In the men's javelin final, Neeraj Chopra won the Gold medal with a throw of 88.17 meters.
✓ Pakistan's Arshad Nadeem won the silver medal in the men's javelin throw event with a throw of 87.82
metres.
✓ Jakub Vadlejch of the Czech Republic won the bronze medal in the same event.
✓ In the javelin throw, Indian player Kishor Jena finished fifth, while Indian player DP Manu finished
sixth.
✓ Success in women's 3000m steeplechase:
✓ Representing India, Parul Chowdhary finished 11th in the women's 3000m steeplechase event.
✓ Men's 4x400m Relay:

Follow us: Official Site, Telegram, Facebook, Instagram, Instamojo 493


✓ The Indian team of Muhammad Anas Yahiya, Amoj Jacob, Muhammad Ajmal Variathodi, and Rajesh
Ramesh finished fifth in the men's 4x400m relay
NEERAJ CHOPRA IN NEWS 2023
▪ Javelin thrower Neeraj Chopra won the first gold medal for India at the World Athletics Championships
in Budapest, Hungary with a throw of 88.17 meters.
▪ Neeraj Chopra, will lead the country’s challenge at the World Athletics Championships from August 19
to 27, 2023 in Budapest.
▪ India observes 3rd ‘Javelin Throw Day’ on August 07, 2023.
▪ Neeraj Chopra wins second straight Diamond League title in Lausanne, Switzerland with 87.66m throw.
▪ Olympic gold medal winner Neeraj Chopra has secured victory with 88.67 m throw at Doha Diamond
League 2023.
▪ Neeraj Chopra makes history, becomes World No 1 in men's javelin throw.
▪ Neeraj Chopra is on top with 1455 points, 22 ahead of Anderson Peters of Granada.
▪ The Mission Olympic Cell (MOC) accepted Neeraj Chopra's proposal to train in Kuortane, Finland.
▪ The men's javelin must weigh at least 800g and be 2.6m-2.7m long while the women's javelin must weigh
600g and be 2.2m-2.3m long.
▪ Neeraj Chopra presently holds the rank of Subedar in the Indian Army.
▪ Neeraj Chopra is from Khandra village, Panipat district, Haryana
Q. What is the name of the specific spot where Chandrayaan-2 was landed on the Moon?
A) Shivshakti B) India Point
C) Shakti Point D) Tiranga Point
Answer : D
✓ Prime Minister Narendra Modi has declared August 23 as the National Space Day to commemorate the
remarkable achievement of Chandrayaan-3’s landing on the moon.
✓ The specific spot where Chandrayaan-3 landed on the Moon will be named ‘Shivshakti’, and the spot
where Chandrayaan-2 landed will be called ‘Tiranga Point’.
✓ PM Modi made this announcement during his visit to the ISRO Telemetry Tracking and Command
Network Mission Control Complex in Bengaluru, Karnataka.
✓ On 23 August 2023, Chandrayaan-3 achieved a historic feat by becoming the first mission to land on the
South Pole of the Moon.
✓ Mission goals included a safe lunar landing, rover maneuverability, and the performance of scientific
experiments on-site.
✓ India has now joined the ranks of the United States, Russia, and China in successfully landing on the
Moon.

Q. Which day has been announced to be celebrated as National Space Day?


A) 22 August B) 23 August
C) 24 August D) 25 August
Answer : B
✓ Prime Minister Narendra Modi has announced to celebrate August 23 as 'National Space Day'.
✓ Chandrayaan-3 made a successful soft landing on the South Pole of the Moon on 23 August. To make
this day historic, it has been declared as National Space Day.
✓ Also, the point (surface) of Chandrayaan-3 mission's Vikram Lander on the Moon's south pole was
named as 'Shiv Shakti Point'.
✓ The point at which the hard landing of Chandrayaan-2 took place in 2019 has been named as 'Tiranga'
point.

Follow us: Official Site, Telegram, Facebook, Instagram, Instamojo 494


✓ PM Modi made this announcement during his visit to the ISRO Telemetry Tracking and Command
Network Mission Control Complex in Bengaluru, Karnataka
✓ Chandrayaan-3 Achievements:
✓ On 23 August 2023, Chandrayaan-3 achieved a historic feat by becoming the first mission to land on the
South Pole of the Moon.
✓ Mission goals included a safe lunar landing, rover maneuverability, and the performance of scientific
experiments on-site.
✓ India has now joined the ranks of the United States, Russia, and China in successfully landing on the
Moon.
✓ Objectives of Chandrayaan-3:
✓ Chandrayaan-3 is set to operate on the lunar surface for one lunar day (equivalent to 14 Earth days).
✓ The Pragyan rover will detect, conduct experiments, and transmit the data to the lander within a radius
of 500 meters around the landing site.
✓ The Vikram lander will relay data and images to the orbiter, which will then transmit them back to Earth.
✓ Both the lander and the rover carry advanced scientific instruments for a variety of lunar investigations
including terrain analysis, mineralogical composition, surface chemistry, atmospheric studies, and
water/resource exploration.
✓ Additional Payloads and Studies:
✓ The propulsion module that propelled the lander and rover into a 100 km lunar orbit contains the SHAPE
payload.
✓ SHAPE (Spectro-Polarimetry of the Habitable Planet Earth) is designed to study the spectral and
polarimetric characteristics of Earth from the orbit of the Moon.
✓ Chandrayaan-1
✓ Chandrayaan-1 was the first Indian lunar probe launched by ISRO.
✓ It was part of the Chandrayaan program and was launched in October 2008 and the mission lasted till
August 2009.
✓ Chandrayaan-1 consisted of a Lunar Orbiter and an Impactor.
✓ The Lunar Orbiter conducted scientific research and collected data about the Moon.
✓ The objective of the mission was to make a detailed map of the Moon's surface and study its composition.
✓ Chandrayaan-1 had advanced instruments to test the presence of water ice and minerals.
✓ The spacecraft carried both Indian and international scientific payloads.
✓ Chandrayaan-1 made important discoveries including evidence of water molecules on the lunar surface.
✓ Chandrayaan-2:
✓ India's second lunar exploration mission developed by ISRO.
✓ Components: Lunar Orbiter, Vikram Lander, and Pragyan Rover.
✓ Scientific Objective: Study the composition of the lunar surface and find lunar water.
✓ Launch: July 22, 2019, from Satish Dhawan Space Centre.
✓ Landing Site: Intended for the south polar region of the Moon at a latitude of 70°S.
✓ Planned Landing Date: September 6, 2019.
✓ Landing Result: The lander crashed due to a software glitch.

Q. Which state has been awarded the best ‘State Award’ for 2022?
A) Madhya Pradesh B) Rajasthan
C) Uttar Pradesh D) Tamil Nadu
Answer : A
✓ Indore bagged the best “National Smart City Award”, followed by Surat and Agra.
✓ The central government announced the India Smart Cities Awards for 2022.
✓ Madhya Pradesh has won the best ‘State Award’ and Tamil Nadu bagged the second spot. Rajasthan
and Uttar Pradesh were in the third spot jointly.
✓ Chandigarh has been ranked the number one in the Union territory category.

Follow us: Official Site, Telegram, Facebook, Instagram, Instamojo 495


✓ Among the 100 smart cities in the country, Indore bagged the top spot, Surat in Gujarat the second place,
and Agra the third spot.
Q. Who has been appointed as the first woman field director of Kaziranga National Park?
A) Rita Chauhan B) Dr. Sonali Ghosh
C) Aditi Sinha D) Premlata Thakur
Answer : B
✓ As the first woman Field Director of Kaziranga National Park, Dr. Sonali Ghosh has been appointed.
✓ She will replace Indian Forest Service officer Field Director Jatindra Sarma who is retiring on August
31.
✓ Dr. Sonali will assume her charge from 01 September.
✓ According to a forest department official, she will be the first woman to take charge as the director of the
118-year-old Kaziranga National Park.
✓ About Kaziranga National Park:
✓ Location: It is located in the Golaghat and Nagaon districts of Assam.
✓ It was declared a national park in 1974.
✓ It is also a UNESCO World Heritage Site and houses two-thirds of the total world population of greater
one-horned rhinoceros.

Q. Which Country has handed over the B20 presidency to Brazil to host G20 in 2024?
A) India B) Canada
C) Maldives D) Australia
Answer : A
✓ India has handed over the B20 presidency to Brazil to host G20 in 2024. B20 India Chair N
Chandrasekaran said that India's B20 Presidency under the its G20 Presidency worked under the theme
of Vasaudeva Katumbakam (One Earth, One Family, One Future).
✓ B20 Summit India 2023 began in New Delhi on 25 August.
✓ Eminent leaders including Tata Sons Executive Chairman and B20 India Chairman N Chandrasekaran,
Mastercard CEO Michael Meibach, and World Economic Forum President Borge Brende will be
present.
✓ The B20 India 2023 dialogue will take place under the theme of 'RAISE', an acronym for Responsible,
Accelerated, Innovative, Sustainable & Equitable Businesses.
Q. Which country has hosted the first 'Karnataka Cultural Festival'?
A) Maldives B) Sri Lanka
C) Thailand D) Bhutan
Answer : B
✓ Sri Lanka hosts 'Karnataka Cultural Festival' for the first time.
✓ Sri Lanka hosted the Karnataka Cultural Festival on August 27, 2023.
✓ This program is organized by the International Cultural Council of India, Swami Vivekananda Cultural
Center, and M.E. Organized in association with the Global Peace Foundation.
✓ The festival featured a variety of performances from over 30 artists from Karnataka: traditional dances,
folk music, multilingual poems, ballet, and puppet shows
✓ The focus was on highlighting the cultural heritage and heritage of Karnataka.
✓ The cast included lead artist Madhuri Bondre and her troupe.

Follow us: Official Site, Telegram, Facebook, Instagram, Instamojo 496


FESTIVAL IN NEWS 2023
▪ Karnataka Cultural Festival : Sri Lanka .
▪ Unmesha’ – International Literature Festival and ‘Utkarsh’ – Festival of Folk and Tribal Performing Arts
: Bhopal, Madhya Pradesh
▪ Festival of Libraries 2023 : New Delhi .
▪ Chachin Grazing Festival : Arunachal Pradesh
▪ Harela Festival : Uttarakhand
▪ Bonalu Festival : Telangana
▪ Kharchi Puja : Tripura
▪ Palkhi festival : Maharashtra
▪ Raja festival : Odisha
▪ Kheer Bhawani Mela : Jammu and Kashmir
▪ Great Prayer Festival : Ladakh
▪ Sinthan Snow Festival : Jammu and Kashmir
▪ Heritage Festival : Goa
▪ Thawe Festival : Bihar
▪ Bohag Bihu festival : Assam
▪ 49th Khajuraho Dance Festival : Madhya Pradesh
▪ Yaoshang festival : Manipur
▪ Purple Fest : Goa
▪ Orange Festival : Nagaland
▪ Kala Ghoda Arts Festival : Mumbai, Maharashtra
▪ Losar Festival : Ladakh
▪ 23rd Hornbill Festival : Nagaland
▪ Gaan Ngai festival : Manipur
▪ Cherchera Festival : Chhattisgarh
▪ 26th National Youth Festival 2023 : Hubbali, Karnataka
▪ International Yoga Festival 2023 : Rishikesh, Uttarakhand
▪ Tribal Dance Festival & Military Tattoo : New Delhi
▪ Mongeet Festival : Assam
▪ International Kite Festival 2023 : Ahmedabad, Gujarat
▪ Vigyanika Science Literature Festival : Bhopal, Madhya Pradesh
▪ Baisakhi is a harvest festival : Punjab and Haryana
Q. Road Transport and Highways Minister Nitin Gadkari has launched the world’s first 100 percent
Ethanol-fueled car in which city?
A) Pune B) Bengaluru
C) New Delhi D) Hyderabad
Answer : C
✓ Union road transport and highways minister Nitin Gadkari has unveiled a 100 per cent ethanol-fueled
variant of Toyota's Innova HyCross car. This car is said to be the world's first BS-VI (Stage-II), electrified
flex-fuel vehicle.
✓ The government has launched the Ethanol Blended Petrol (EBP) Programme to mix this biofuel with
petrol to reduce the consumption of fossil fuels.
✓ The car, which runs on 100 per cent ethanol (E100), is based on the Toyota Innova HyCross. It can cover
40 per cent of its distance on ethanol and the remaining 60 per cent on electric, with the petrol engine
shut off.
✓ India has set a target of 20 per cent ethanol blending with petrol by 2025

Follow us: Official Site, Telegram, Facebook, Instagram, Instamojo 497


Q. Which country's second- highest honor 'Grand Cross of the Order of Honour' was awarded to PM Modi?
A) France B) South Africa
C) Greece D) Germany
Answer : C
✓ The Indian Prime minister was recently conferred with the Grand Cross of the Order of Honour by the
Greece President in Athens.
✓ About Grand Cross of the Order of Honour:
✓ It is the second-highest civilian honour in Greece.
✓ It is conferred to "eminent personalities who, by reason of their distinguished position, have contributed
to enhancing the stature of Greece".
✓ The Order of Honour was established in 1975.
✓ The head of the goddess Athena is depicted on the front side of the Star with the inscription “ONLY
THE RIGHTEOUS SHOULD BE HONOURED”.
✓ Key Facts about Greece:
✓ Location: It is located in southeastern Europe
✓ Capital : Athens
✓ Currency : Euro
✓ President : Katerina Sakellaropoulou
✓ Prime Minister : Kyriakos Mitsotakis
✓ STATE HONOURS GIVEN TO NARENDRA MODI
15. PM Modi conferred with Greece's Grand Cross of the Order of Honour (2023)
14. Grand Cross of the Legion of Honour : France's highest civilian and military honour (2023)
13. Order of the Nile : Egypt's highest honor ( 2023)
12. Companion of the Order of Logohu: the highest civilian award of Papua New Guinea (2023)
11. Companion of the Order of Fiji: Highest honour of Fiji ( 2023)
10. Ebakl Award by the Republic of Palau (2023)
9. Order of the Dragon King is the highest civilian honour of Bhutan ( 2021)
8. Legion of Merit is a military award of the USA (2020)
7. Order of Zayed is the highest civilian honour of the United Arab Emirates (UAE) (2019)
6. Order of St. Andrew is the highest civilian honour of Russia (2019)
5. Order of the Distinguished Rule of Izzuddin is the highest honour of the Maldives ( 2019)
4. King Hamad Order of the Renaissance is the highest honour of Bahrain (2019)
3. Grand Collar of the State Of Palestine is the highest civilian order of Palestine (2018)
2. Order of Abdulaziz Al Saud is the Highest honour of Saudi Arabia (2016)
1. Amir Amanullah Khan Award is the highest civilian award of Afghanistan (2016)

Q. Gond' paintings are from which region of India?


A) Western B) Northeastern
C) Southern D) Central

Follow us: Official Site, Telegram, Facebook, Instagram, Instamojo 498


Answer : D
✓ Recently, during the BRICS meeting, the Prime Minister of India gifted a Gond painting to the Brazilian
president Lula da Silva.
✓ About Gond Painting
✓ It is a famous folk art of the Gond tribal community of central India.
✓ It is done to preserve and communicate the culture of the Gond tribal community.
✓ Themes: Gond tribes are highly interlinked with nature, and this appears in their paintings too. They
include animals, the mahua tree, mythological stories, Hindu gods, local deities and folktales, etc.
✓ The artist uses his distinctive pattern and style to fill the images. These style signatures are used in collages
to make a complete picture, such as dots, fine lines, curved lines, dashes, fish scales, etc.
✓ Gond paintings were created by the aboriginals from Central Indian region like Madhya Pradesh. These
paintings depict the day to day lives of their community and are rich with images of plants and animals

Q. Who has won the Chess World Cup Final 2023?


A) Garry Kasparov B) Magnus Carlsen
C) R Praggnanandhaa D) Fabiano Caruana
Answer : B
✓ The Norwegian chess grandmaster, Magnus Carlsen won the Chess World Cup Final 2023 title for the
first time, after defeating India’s chess grandmaster R Praggnanandhaa in a tie-breaker.
✓ He has also become the second Indian after Viswanathan Anand to reach final in Chess World Cup
history.
✓ Magnus Carlsen Norwegian chess grandmaster who is a five-time World Chess Champion.
✓ Carlsen has held the No. 1 position in the FIDE world chess rankings since 1 July 2011.
✓ Carlsen became World Chess Champion in 2013 by defeating Viswanathan Anand.
✓ R Praggnanandhaa (18) was the youngest World Cup finalist ever and the youngest World Cup winner.
✓ He managed to secure the silver medal after defeating Hikarua Nakamura and Fabiano Caruana.
✓ He also became the second Indian after Viswanathan Anand to enter the Chess World Cup.
✓ The Chess World Cup 2023 was a 206-player single-elimination chess tournament that took place in
Baku, Azerbaijan from 30 July to 24 August 2023. It was the 10th edition of the Chess World Cup
✓ India's Viswanathan Anand and Levon Aronian have won the FIDE World Cup title 2-2 times
✓ The 18-year-old R Praggnanandhaa has jumped nine places to achieve a career-high 20th rank in the
world and has become India's new No. 3 behind his Chennai mates D Gukesh and mentor Viswanathan
Anand.
✓ In 2016, Praggnanandhaa became the youngest international master in history, at the age of 10 years, 10
months, and 19 days.
✓ Praggnanandhaa was born in Chennai, Tamil Nadu.
Q. Prime Minister Narendra Modi has recently gifted South African President Cyril Ramaphosa a pair of
'Surahi', the Bidri work of which state?
A) Nagaland B) Madhya Pradesh
C) Telangana D) Tamil Nadu
Answer : C
✓ PM Modi gifts Bidri Surahi, Nagaland Shawl, and Gond Painting to BRICS leaders.
✓ Prime Minister Narendra Modi, who was on a three-day visit to South Africa to attend the 15th BRICS
summit, presented special gifts to the organization’s leaders including South African president Cyril
Ramaphosa, First Lady of South Africa Tshepo Motsepe, and Brazilian president Luiz Inácio Lula da
Silva. PM Modi gifts Bidri Surahi, Nagaland Shawl, and Gond Painting to BRICS leaders.
✓ PM Modi gifted Bidri a pair of ‘Surahi’ from Telangana to South African President Cyril Ramaphosa.

Follow us: Official Site, Telegram, Facebook, Instagram, Instamojo 499


✓ The Prime Minister also gifted a Nagaland shawl to the First Lady of South Africa Tshepo Motsepe.
✓ PM Modi also gifted Gond Painting from Madhya Pradesh to the Brazilian President, Luiz Inácio Lula
da Silva
✓ About Bidriware:
✓ It is a form of metal handicraft that has Persian influences and has been made for centuries by artisans
from Karnataka’s Bidar district.
✓ It is renowned for its intricate, handcrafted designs.
✓ Origin:
✓ The origin of Bidriware as a craft is attributed mostly to the Bahamani Sultans who ruled the region
during the 14th and 15th centuries.
✓ It was first brought to India by the noted Sufi Khwaja Moinuddin Hasan Chisti in the form of utensils.
✓ The art form developed in the kingdom was a mix of Turkish, Persian and Arabic influences, which were
intermingled with the local styles, and thus a unique style of its own was born.
✓ This native art form has obtained a Geographical Indications (GI) registry.
✓ Bidar in Karnataka and Hyderabad in Telangana are the most vibrant centers of Bidriware.
✓ How is Bidriware made?
✓ Bidri Ware is manufactured from an alloy of copper and zinc (in the ratio 1:16) by casting.
✓ The zinc content gives the alloy a deep black colour.
✓ The craftsman uses small chisels to engrave the design over the freehand etching.
✓ Fine wire or flattened strips of pure silver are then carefully hammered into these grooves.
Q. Which company signs women's tennis champ Iga Swiatek as brand ambassador?
A) TATA B) Infosys
C) Jio D) Wipro
Answer : B
✓ Infosys announced on has entered into a multi-year partnership with women’s tennis World No. 1 Iga
Świątek, making her its global brand ambassador to promote Digital Innovation and inspire women
worldwide.
✓ This news comes a day after the IT major signed up Rafael Nadal for the same role
Q. Recently, which of the following crop was accorded with Geographical Indication (GI) Tag?
A) Chokuwa rice of Assam B) Tur dal of Gulbarga
C) Cotton of Raichur D) Wheat of Bidar
Answer : A
✓ Chokuwa rice recently earned a GI (Geographical Indication) tag for its exquisiteness.
✓ About Chokuwa rice.
✓ It is also known as Magic rice cultivated in Assam.
✓ It is a part of Assam's culinary heritage; this unique rice has been a staple of the troops of the mighty
Ahom dynasty.
✓ This unique and healthy rice is cultivated around the Brahmaputra River area.(In several parts of Assam
like Tinsukia, Dhemaji, Dibrugarh, etc.)
✓ It is basically semi-glutinous winter rice, known as Sali rice.
✓ The sticky and glutinous variety is categorised as Bora and Chokuwa based on their amylose
concentration.
✓ The low amylase Chokuwa rice variants are used to make soft rice, which is known as Komal Chaul or
soft rice.
✓ This whole grain can be consumed after soaking the rice in cold or lukewarm water. This rice variety is
widely consumed for its convenience of preparation and nutritional value.

Follow us: Official Site, Telegram, Facebook, Instagram, Instamojo 500


✓ This unique rice variety is consumed with curd, sugar, jaggery, and bananas to name a few.
✓ This rice is also used in making several Assamese delights like Pithe and other local dishes.
Q. President Smt. Draupadi Murmu released a postal stamp in whose memory?
A) Parkash Singh Badal B) Arun Jaitley
C) Grandmother Prakashmani D) Atal Bihari Vajpayee
Answer : C
✓ President Smt. Draupadi Murmu released a postal stamp in the memory of Grandmother Prakashmani,
former head of Brahma Kumaris at the Rashtrapati Bhavan Cultural Centre.
✓ This postage stamp was released under the 'My Stamp' initiative of the Department of Posts, Ministry of
Communications, on the occasion of the 16th death anniversary of Dadi Prakashmani.
✓ Dadi Prakashmani, former chief of Brahma Kumaris, played a pivotal role in the Brahma Kumaris
organization.
✓ Brahma Kumaris to become the world’s largest women-led spiritual institution.
✓ Brahma Kumaris, founded in 1937 by Prajapita Brahma Baba, is a global spiritual organization.

Q. Which country has become the first country in the world to reach the South Pole of the Moon?
A) Russia B) India
C) USA D) China
Answer : B
✓ Creating history, India has become the first country in the world to land a lander on the south pole of the
Moon.
✓ Recently Vikram Lander has landed near the south pole of the Moon. With the successful landing of
Chandrayaan-3, India has become the fourth country in the world to make a soft landing on the moon.
✓ America, China and the former Soviet Union (now Russia) have been successful in soft landing on the
moon. Now India has also joined this list.

Q. Who is the project director of Chandrayaan-3 mission launched by ISRO?


A) S Somnath B) S. Unnikrishnan Nair
C) M Sankaran D) P Veeramuthuvel
Answer : D
✓ The project director of the Chandrayaan-3 mission is P Veeramuthuvel, he took over as the project
director of Chandrayaan-3 in the year 2019.
✓ Prior to this, he has served as Deputy Director in the Space Infrastructure Program Office of ISRO. P
Veeramuthuvel had earlier played an important role in the Chandrayaan-2 mission.
✓ He hails from Villupuram, Tamil Nadu and has studied at the Indian Institute of Technology Madras
(IIT-M).

Q. Professor Kalyampudi Radhakrishna Rao passed away recently, he was associated with which field?
A) Medicine B) Space
C) Statistics D) Journalism
Answer : C
✓ Professor Kalyampudi Radhakrishna Rao, one of the eminent statisticians, has passed away. He was also
known as CR Rao.

Follow us: Official Site, Telegram, Facebook, Instagram, Instamojo 501


✓ The Government of India honored CR Rao with the Padma Bhushan in the year 1968 and the Padma
Vibhushan in 2001, India's second highest civilian award.
✓ Indian-American statistician Calyampudi Radhakrishna Rao has been awarded the 2023 International
Prize in Statistics, which is statistics’ equivalent of the Nobel Prize.

Q. What name has ISRO given to the rover sent with Chandrayaan-3 mission?
A) Vikram B) Gyan Ganga
C) Jigyasha D) Pragyan
Answer : A
✓ The Chandrayaan-3 mission was launched by ISRO on 14 July 2023 from the Satish Dhawan Space
Center in Sriharikota, Andhra Pradesh.
✓ ISRO has named the lander of Chandrayaan-3 mission as 'Vikram' and 'rover' as 'Pragyan' which is a
scientific payload.
✓ 'Pragyan' is a robotic vehicle which translates to 'knowledge' in Sanskrit.
✓ The Chandrayaan-3 mission is a follow-up to the 2019 Chandrayaan-2 mission. Chandrayaan-2's Vikram
Lander crashed on the surface of the Moon

Q. Who was honored with the first 'Udyog Ratna' award instituted by the Government of Maharashtra?
A) Ratan Tata B) Mukesh Ambani
C) Gautam Adani D) Adar Poonawala
Answer : A
✓ Renowned industrialist Ratan Tata was honored with the first 'Udyog Ratna' award instituted by the
Government of Maharashtra.
✓ Chief Minister Eknath Shinde, Deputy Chief Minister Devendra Fadnavis and Ajit Pawar were present
on the occasion. In the financial year 2021-22, the combined revenue of Tata companies was $ 128 billion.
✓ Indian Industrialist Shri Ratan Tata appointed in ‘Order of Australia’ for distinguished service.
✓ In 2008, he received the Padma Vibhushan, the second highest civilian honour in India, after receiving
the Padma Bhushan, the third highest civilian honour in 2000.
Q. Which country has won the FIFA Women's World Cup 2023 title?
A) Argentina B) Brazil
C) England D) Spain
Answer : D
✓ Spain has won the title of FIFA Women's World Cup 2023 Final. In the final, the women's team of Spain
registered a historic victory by defeating England 1-0. Spain's women's football team has become the
world champion for the first time
✓ 2023 FIFA Women's World Cup
✓ Host countries : Australia & New Zealand
✓ Teams : 32
✓ Champions : Spain (1st title)
✓ Runners-up : England
✓ Third place : Sweden
✓ Fourth place : Australia
✓ Top scorer : Japan Hinata Miyazawa (5 goals)
✓ Best player : Spain Aitana Bonmatí
✓ Best young player : Spain Salma Paralluelo

Follow us: Official Site, Telegram, Facebook, Instagram, Instamojo 502


✓ Best goalkeeper : England Mary Earps
✓ Fair play award : Japan
Q. In which research institute the new variety of lotus flower 'NAMOH 108' has been developed?
A) National Botanical Research Institute, Lucknow
B) FSSAI
C) National Botanical Research Institute, Howrah
D) Indian Council of Agricultural Research
Answer : A
✓ Union Science and Technology Minister Jitendra Singh unveiled a new variety of lotus flower with 108
petals 'NaMoh 108'.
✓ This variety of lotus has been developed by the National Botanical Research Institute (NBRI) located in
Lucknow.
✓ Flowers of 'NBRI Namoh 108' variety of lotus bloom from March to December and this is the first flower
of this variety
Q. Who is the Best Actor Award at the 69th National Film Awards 2023?
A) Prabhas B) Allu Arjun
C) Vijay Deverakonda D) Nagarjuna
Answer : B
✓ Best Actor - Allu Arjun (Pushpa)
✓ Best Actress - Alia Bhatt (Gangubai Kathiawadi) and Kriti Sanon (Mimi)
Q. Which film recently won the best feature film award in the 69th National Film Award?
A) Rocketry B) The Kashmir Files
C) Sardar Udham D) Pushpa
Answer : A
✓ Best Feature Film - R Madhavan's Rocketry
Q. Who won the award for Best Direction in the 69th National Film Award?
A) R Madhavan B) Nikhil Mahajan
C) Sanjay Leela Bhansali D) Sriram Raghavan
Answer : B
✓ Best Direction - Nikhil Mahajan - Godavari (Marathi)
Q. Which film won the award for Best Popular Film Providing Wholesome Entertainment in the 69th
National Film Award?
A) Rocketry B) Sardar Udham
C) RRR D) The Kashmir Files
Answer : C
✓ Best Popular Film Providing Wholesome Entertainment - RRR

Follow us: Official Site, Telegram, Facebook, Instagram, Instamojo 503


Q. Which film won the award for Best Hindi Film in the 69th National Film Award?
A) Sardar Udham B) Rocketry
C) RRR D) The Kashmir Files
Answer : A
✓ Best Feature Film - Hindi - Sardar Udham
✓ The 69th National Film Awards was announced on Thursday during a press conference by the Jury at
the National Media Center New Delhi.
✓ The Jury members for National Film Awards handed over the list of awardees for feature, non-feature
and best script category to Union I&B Minister Anurag Thakur.
✓ A total of 280 films from 28 languages were submitted for the awards in year 2021. 31 categories in
Feature film, 24 in non-feature and 3 in script writing on cinema.
✓ 69th National Film Awards 2023
✓ Best Feature Film- Rocketry
✓ Best actor – Allu Arjun (Pushpa)
✓ Best actress – Alia Bhatt (Gangubai Kathiawadi) & Kriti Sanon (Mimi)
✓ Best Director – Nikhil Mahajan Godavari
✓ Best Popular Film – RRR
✓ Nargis Dutt award for best film on national integration – Kashmir Files
✓ Best Supporting Actor – Pankaj Tripathi (Mimi)
✓ Best Supporting Actress – Pallavi Joshi (Kashmir Files)
✓ Best Music Direction (Background Music) – MM Keeravaani (RRR)
✓ Best Male Playback Singer – Kaala Bhairava (RRR)
✓ Best Female Playback Singer – Shreya Ghoshal (Iravin Nizhal)
✓ Best Choregraphy – RRR
✓ Best Hindi Film – Sardar Udham
✓ Best Gujarati Film – Chhello Show
✓ Best non feature film – Ek Tha Gaon
✓ Special Jury Award- Shershaah, Vishnuvardhan
✓ Special Jury Award (Non-feature film) – Shekhar Bapu Rankhambe, Rekha
MOST IMPORTANT FLIMFARE AWARDS IN NEWS 2023
▪ 69th NATIONAL FILM AWARDS
Best Feature Film : Rocketry
Best actor : Allu Arjun (Pushpa)
Best actress : Alia Bhatt (Gangubai Kathiawadi) & Kriti Sanon (Mimi)
Best Director : Nikhil Mahajan Godavari
Best Popular Film : RRR
Best Hindi Film : Sardar Udham
▪ INTERNATIONAL INDIAN FILM ACADEMY AWARDS (IIFA) 2023
Best Film: Drishyam 2
Best Director: R Madhavan for Rocketry: The Nambi Effect
Best Actor in a Leading Role (Female): Alia Bhatt for Gangubai Kathiawadi
Best Actor in a Leading Role (Male): Hrithik Roshan for Vikram Vedha
▪ 68TH FILMFARE AWARDS, (also known as HYUNDAI FILMFARE AWARDS 2023)
Best Film: Gangubai Kathiawadi
Best Actor : Rajkummar Rao for Badhaai Do
Best Actress : Alia Bhatt for Gangubai Kathiawadi
Best Director: Sanjay Leela Bhansali for Gangubai Kathiawadi.

Follow us: Official Site, Telegram, Facebook, Instagram, Instamojo 504


▪ 95TH ACADEMY AWARDS (OSCARS AWARDS 2023)
Best Picture : Everything Everywhere All At Once
Best Actress : Michelle Yeoh (Everything Everywhere All at Once)
Best Actor : Brendan Fraser (The Whale)
Best Director : Daniel Kwan and Daniel Schienert (Everything Everywhere All at Once)
Best Original Song : “Naatu Naatu” (RRR)
Best Documentary Short Film : ‘The Elephant Whisperers’ by Kartiki Gonsalves and Guneet Monga
▪ 76TH BRITISH ACADEMY FILM AWARDS ( BAFTAs)
Best Film : All Quiet on the Western Front
Best Actress : Cate Blanchett, (Tar)
Best Actor : Austin Butler, (Elvis)
Best Director : Edward Berger, (All Quiet on the Western Front)
▪ DADASAHEB PHALKE INTERNATIONAL FILM AWARDS - 2023
Best Film: The Kashmir Files
Best Actor: Ranbir Kapoor for Brahmastra: Part 1
Best Actress: Alia Bhatt for Gangubai Kathiawadi
Best Director: R Balki for Chup: Revenge of The Artist
Film of The Year : RRR
OTHERS FLIMFARE AWARDS IN NEWS 2023
▪ South Cinema's film RRR has created history at the 80th 'Golden Globe Awards 2023' award ceremony,
the film's Telugu song 'Naatu Naatu' has won the Best Song award.
▪ RRR has bagged three big awards at the Hollywood Critics Association (HCA) Film Awards 2023.
▪ SS Rajamouli's film 'RRR' won the 28th Critics' Choice Awards 2023 for Best Foreign Language Film
and Best Song.
▪ The film is based on the lives of real-life Indian revolutionaries Alluri Sitarama Raju and Komaram
Bheem.
▪ The song 'Naatu Naatu' is composed by MM Keeravani and sung by Kaal Bhairav and Rahul Sipligunj.
Q. Which city has secured the first position in Swachh Vayu Sarvekshan-2023 conducted by the Central
Pollution Control Board, securing an impressive 187 out of 200 points given by the CPCB?
A) Surat B) Navi Mumbai
C) Indore D) Vijayawada
Answer : C
✓ Indore secures 1st position, Bhopal 5th in Swachh Vayu Sarvekshan 2023.
✓ Indore has secured the first position in Swachh Vayu Sarvekshan-2023 conducted by the Central
Pollution Control Board, securing an impressive 187 out of 200 points given by the CPCB.
✓ In the 10-lakh category, Gwalior seems to have made the least effort as it secured 117 points, ending up
43rd.
✓ Bhopal came fifth with 181 points and Jabalpur seventh with 172 points.
✓ Madhya Pradesh has secured two of the first five positions for cities with a population of more than 10
lakhs.
✓ Indore stood first, Agra second, Thane third, Srinagar fourth and Bhopal fifth.

Q. What is the theme of B20 Summit India 2023 which started recently in New Delhi?
A) RAISE B) Vasudhaiva Kutumbakam
C) One Earth, One Family, One Future D) Realising Opportunities of the 21st Century for
Answer : A

Follow us: Official Site, Telegram, Facebook, Instagram, Instamojo 505


✓ The Prime Minister Narendra Modi addressed the B20 Summit India 2023 in New Delhi.
✓ B20 Summit India brings policymakers, business leaders and experts from across the world to deliberate
and discuss the B20 India Communique.
✓ The B20 India Communique includes 54 recommendations and 172 policy actions for submission to
G20.
✓ The Summit is based on the theme of B20 India R.A.I.S.E: Responsible, Accelerated, Innovative,
Sustainable, Equitable Businesses.
✓ Chairperson of the Tata Group, N Chandrasekaran was appointed the chair of B20 India, to lead the
business agenda during India’s G20 presidency. The Confederation of Indian Industry (CII) is appointed
as the B20 India Secretariat by the Centre to lead the B20 India process.

Q. Rafael Nadal has been named as the ambassador for which Indian technology brand?
A) TCS B) Wipro
C) Infosys D) Reliance Industries
Answer : C
✓ Infosys announced entering into a three-year partnership with tennis legend Rafael Nadal.
✓ The 22-time Grand Slam champion will serve as the Brand Ambassador for the brand and Infosys Digital
Innovation.
✓ Infosys will collaborate with Nadal on the development of cutting-edge digital innovation projects.
✓ Rafael Nadal is a Spanish professional tennis player.
✓ Nadal has won 22 Grand Slam men's singles titles, including a record 14 French Open titles.
✓ Nadal has won the Stefan Edberg Sportsmanship Award five times and was the Laureus World
Sportsman of the Year in 2011 and 2021. Time named Nadal one of the 100 most influential people in
the world in 2022
Q. India’s first 3D-printed post office built with a robotic printer using 3D printing technology has been
opened in which city?
A) Hyderabad B) Bengaluru
C) Chennai D) Mumbai
Answer : B
✓ India’s 1st state-of-the-art 3D-Printed post office opens in Bengaluru.
✓ India’s first 3D-printed post office built with a robotic printer using 3D printing technology that deposits
concrete layer-by-layer in accordance with computerized 3D model drawing input was inaugurated in
Bengaluru.
✓ Cost and time savings make 3D-concrete printing technology a viable alternative to conventional building
practices.
✓ The post office building was constructed by construction company Larsen & Toubro Limited while IIT
Madras provided technical guidance.
BENGALURU IN NEWS 2023
▪ India’s 1st state-of-the-art 3D-Printed post office opens in Bengaluru.
▪ Bengaluru becomes the first Indian city to join the World Cities Culture Forum (WCCF).
▪ India is all set to host the 5th World Coffee Conference (WCC) for the first time from 25-28 September
2023 in Bengaluru, Karnataka.
▪ Sberbank’s branch in India has been granted permission by the Reserve Bank of India (RBI) to set up an
IT unit in Bengaluru.
▪ India’s first 3D-printed post office to come up in Bengaluru.

Follow us: Official Site, Telegram, Facebook, Instagram, Instamojo 506


▪ The second Trade and Investment Working Group meeting under the G20 India Presidency began in
Bengaluru.
▪ Foxconn to invest nearly 1 billion USD in Bengaluru.
▪ Nitin Gadkari unveiled first methanol run buses in Bengaluru.
▪ India Energy Week 2023 (IEW 2023), being organised during India's G20 Presidency, will be held in
Bengaluru between February 6 and 8, 2023. .
▪ The Union Ministry of Defence has announced on 27 November 2022 that the biennial air show
AeroIndia-2023 will be held at the Air Force Station Yelahanka in Bengaluru's from 13-17 February,
2023. This would be the 14th edition of Aero India.
▪ The first G-20 Environment meeting under the G-20 India Presidency will be held at the Taj West End
in Bengaluru.
▪ Union Minister of Agriculture and Farmers Welfare Narendra Singh Tomar on 15 March inaugurated a
5-day cultural program "AgriUnifest" in Bengaluru..
▪ The first meeting of the G-20 Energy Transition Working Group under the chairmanship of India will be
held in Bengaluru from 5 to 7 February
Q. In which city the warship 'INS Vindhyagiri' was inaugurated by President Draupadi Murmu?
A) Mumbai B) Kolkata
C) Chennai D) Visakhapatnam
Answer : B
✓ President Droupadi Murmu has launched ‘Vindhyagiri’, an advanced stealth frigate, at Garden Reach
Shipbuilders and Engineers Ltd’s (GRSE) facility on the banks of Hooghly River, Kolkata.
✓ This is the sixth of seven ships built under ‘Project 17 Alpha’ for the Navy. The first five ships of the
project were launched between 2019 and 2022.
✓ Project-17 Alpha is also known as the Nilgiri class frigate project. INS Vindhyagiri is the sixth stealth
frigate constructed under Project-17 Alpha.
✓ Sixth vessel to be rolled out as part of the Project 17A Frigate
✓ Stealth frigate Vindhyagiri is the sixth vessel rolled out as part of the Project 17A Frigate.
✓ The five other ships – INS Nilgiri, Udaygiri, Himgiri, Taragiri, and Dunagiri – were launched between
2019 and 2022.
✓ Built by: Garden Reach Shipbuilders and Engineers Ltd (GRSE) facility at Kolkata.
✓ Named after: The ship is named after the Vindhyagiri hills in Karnataka.
✓ These frigates are being built by two state-run shipyards:
– Mazagon Docks Limited(MDL) is responsible for manufacturing four frigates — INS Nilgiri,
Udaygiri, Taragiri and Mahendragiri.
– Garden Reach Shipbuilders and Engineers Ltd (GRSE) will manufacture the remaining three
frigates, INS Himgiri, Dunagiri and Vindhyagiri.
✓ These ships are follow-on versions of the three Project-17 vessels, also known as the Shivalik class frigates
in the Indian Navy.

Q. Indian National Center for Ocean Information Services has launched which app for seafarers?
A) SAGAR B) MAHASAGAR
C) SAMUDRA D) SETHUSAMUDRAM
Answer : C
✓ Recently, Indian National Centre for Ocean Information Services (INCOIS) launched a new mobile
application called SAMUDRA for seafarers and the fishing community.
✓ About SAMUDRA App:

Follow us: Official Site, Telegram, Facebook, Instagram, Instamojo 507


✓ Smart Access to Marine Users for Ocean Data Resources and Advisories (SAMUDRA) application offers
comprehensive information on all ocean related services.
✓ It is a cutting-edge tool helping users to navigate the marine domain with confidence on their safety, and
profitable fishing operations.
✓ It embodies the institute vision and mission in serving the nation with ocean data, information, and
advisory services.
✓ It will catalyse sustainable ocean activities, thus enabling a way forward for the Blue Economy,
✓ It empowers users with real-time updates and critical alerts on oceanic disasters such as tsunamis, storm
surges, high waves, and swell surge alerts, for the individuals and communities to stay informed and take
necessary precautions towards the protection of lives and property.
✓ What is INCOIS?
✓ It was established as an autonomous body in 1999 under the Ministry of Earth Sciences (MoES).
✓ Mandate: To provide the best possible ocean information and advisory services to society, industry,
government agencies and the scientific community through sustained ocean observations and constant
improvements through systematic and focused research

Q. Consider the following statements about ‘Floodwatch’ Mobile App.


The App provides real time information of flood like situation up to 7 days.
The App is launched by Central Water Commission (CWC).
Choose the incorrect statements:
A) 1 only
B) 2 only
C) Both 1 and 2
D) Neither 1 nor 2
Answer : C
✓ Chairman, Central Water Commission (CWC), Shri Kushvinder Vohra today launched the mobile
application, "FloodWatch” with the aim of using mobile phones to disseminate information related to
the flood situation and forecasts up to 7 days on a real-time basis to the public.
✓ About Floodwatch App:
✓ This app gives information related to the flood situation and forecasts up to 7 days on a real-time basis to
the public.
✓ The in-house developed user-friendly app has readable and audio broadcast and all the information is
available in 2 languages, viz. English and Hindi.
✓ Other feature of the app includes real-time flood monitoring where users can check up-to-date flood
situation throughout the country.
✓ The app utilizes near real-time river flow data from various sources.
✓ The app also provides flood forecast at nearest location where users can check the flood advisory at the
station nearest to them on the Home Page itself.
✓ The app will also provide State-wise/Basin-wise Flood Forecast (up to 24 hours) or Flood Advisory (up
to 7 days) which can be accessed by selecting specific stations, state wise or basin wise from the dropdown
menu.
✓ This app utilizes advanced technologies such as satellite data analysis, mathematical modelling, and real-
time monitoring to deliver accurate and timely flood forecasts.
✓ Key facts about Central Water Commission
✓ It is a premier Technical Organization of India in the field of Water Resources.
✓ It is presently functioning as an attached office of the Ministry of Jal Shakti, Department of Water
Resources, River Development and Ganga Rejuvenation

Follow us: Official Site, Telegram, Facebook, Instagram, Instamojo 508


Q. India’s first indigenous e-Tractor Prima ET11 is developed by which institution?
A) CSIR B)Tata Motors
C) Ashok Leyland D) Eicher Motors
Answer : A
✓ India’s first indigenously developed e-Tractor CSIR Prima ET11, was launched by the Union Minister
of State (Independent Charge) Science & Technology Dr. Dr.Jitendra Singh.
✓ The e-Tractor CSIR Prima ET11 has been developed by the Central Mechanical Engineering Research
Institute (CMERI), Durgapur, West Bengal.
Q. Asia’s largest Tulip Garden opens for public in which state/UT?
A) Ladakh B) Himachal Pradesh
C) Uttarakhand D) Jammu and Kashmir
Answer : D
✓ The Indira Gandhi Memorial Tulip Garden in Srinagar recently entered the World Book of Records
(London) as Asia's largest such park.
✓ About Indira Gandhi Memorial Tulip Garden
✓ Indira Gandhi Memorial Tulip Garden, previously Model Floriculture Center, is a tulip garden in
Srinagar, Jammu and Kashmir.
✓ It is the largest tulip garden in Asia, spread over an area of about 30 hectares.
✓ It is situated on the foothills of the Zabarwan Range with an overview of Dal Lake.
✓ The garden was opened in 2007 with the aim to boost floriculture and tourism in Kashmir Valley.
✓ The garden is built on a sloping ground in a terraced fashion consisting of seven terraces.
✓ The garden houses about 48 varieties of tulip flowers. The garden also has several types of other flowers,
including daffodils, hyacinths, roses, narcissus, and other ornamental plants.
✓ Tulip festival:
✓ It is an annual celebration that aims to showcase the range of flowers in the garden as a part of tourism
efforts by the Government of Jammu and Kashmir.
✓ It is organized during the onset of the spring season in Kashmir valley
Q. Matti banana variety granted the Geographical Indication (GI) tag of ___ District.
A) Theni B) Thiruvattaru
C) Dharmapuri D) Kanniyakumari
Answer : D
✓ The Matti banana variety, native to Kanniyakumari district was recently granted the Geographical
Indication (GI) tag
✓ About Matti banana:
✓ There are six known types of the Matti banana and they are indigenous to Kanniyakumari, Tamil Nadu,
where it thrives in the unique climate and soil.
✓ They are known as ‘Baby Banana’ which flourishes mainly in Kalkulam and Vilavancode taluks.
Q. NTCA has approved India's India's____tiger reserve in Rajasthan's Karauli and Dholpur districts.
A) 52nd B) 54th
C) 56th D) 59th
Answer : B

Follow us: Official Site, Telegram, Facebook, Instagram, Instamojo 509


✓ India got its 54th tiger reserve in Rajasthan's Karauli and Dholpur districts with the National Tiger
Conservation Authority (NTCA) approving a proposal in this regard.
✓ This is Rajasthan's fifth tiger reserve after Ranthambore, Sariska, Mukundra Hills and Ramgarh
Vishdhari.
✓ The number of tigers in India has increased from 2,967 in 2018 to 3,682 in 2022, an annual rise of 6 per
cent, according to government data.
✓ With a 50 per cent increase in the last four years, Madhya Pradesh has the maximum number of tigers in
the country (785), followed by Karnataka (563), Uttarakhand (560), and Maharashtra (444).
✓ The number of tigers in Rajasthan has increased from 32 in 2006 to 88 in 2022
Q. How many new countries will join the BRICS alliance from 1 January 2024?
A) 5 B) 6
C) 8 D) 4
Answer : B
✓ 6 Countries to join BRICS from 1 January 2024
✓ In a significant geopolitical development, the BRICS alliance has announced the inclusion of six new
member countries.
✓ These countries are Egypt, Ethiopia, Iran, Argentina, the United Arab Emirates, and Saudi Arabia, and
will join Brazil, Russia, China, India, and South Africa. The membership will take effect from 1 January
2024.
✓ President Cyril Ramaphosa of South Africa, the host nation of the recent BRICS summit.
✓ The BRICS group of nations comprises Brazil, Russia, India, China, and South Africa. It accounts for
more than 40% of the world’s population and about 26% of the global economy.
✓ About BRICS
✓ The full form of BRICS is Brazil, Russia, India, China, and South Africa.
✓ Goldman Sachs economist Jim O'Neill coined the term BRIC (without South Africa) in 2001.
✓ He claimed that by 2050, the four BRIC economies would dominate the global economy by 2050.
✓ South Africa was included in the list in 2010.
✓ The presidency of the Forum is rotated annually among the members.
✓ BRICS accounts for about 40% of the world’s population.
✓ It accounts for 30% of the world's GDP (Gross Domestic Product).
✓ In the year 2014, during the 6th BRICS Summit in Fortaleza, Brazil, the BRICS leaders signed an
agreement to establish a New Development Bank (NDB).

Q. 26th National Conference on e-Governance to begin in Which City?


A) Bhopal B) Mumbai
C) Indore D) Bengaluru
Answer : C
✓ The 26th National Conference On E-governance Began In Indore To Deliberate On Providing Better
Facilities To The Citizens.
✓ Theme : ‘Developing India: Empowering Citizens’.

Q. Which company is set to launch ‘India’s first kerosene-oxygen-powered rocket’?


A) Agnikul Cosmos B) Dhruv Cosmos
C) Pixxel Cosmos D) Orbit Cosmos
Answer : A

Follow us: Official Site, Telegram, Facebook, Instagram, Instamojo 510


✓ Indian firm Agnikul Cosmos is on its way to becoming the first in the country to privately design, develop
and launch a liquid-fuelled rocket.
✓ Agnikul is targeting to launch their maiden rocket known as ‘Agnibaan SOrTeD’ (SubOrbital
Technological Demonstrator).
✓ It is a single-stage launch vehicle driven by Agnikul’s patented Agnilet engine- an entirely 3D-printed,
single-piece, 6kN semi-cryogenic engine.
Q. What is the name of India’s first long-range revolver?
A) Prahaar B) Prithvi
C) Prabal D) Akash
Answer : C
✓ India’s first long-range revolver “Prabal” has been launched.
✓ The revolver is manufactured by the state-owned enterprise Advanced Weapons and Equipment India
(AWEIL), based in Kanpur.
✓ Distinguished by its .32 bore, the Prabal revolver boasts an impressive firing range of up to 50 meters,
more than twice the reach of other revolvers in India
✓ The noteworthy feature of Prabal is its side-swing cylinder, marking a significant advancement in revolver
design.
✓ Moreover, the prevailing market revolvers have a limited range of only up to 20 meters.
Q. Nehru Memorial Museum and Library has been renamed as?
A) Patel Museum and Library B) Gandhi Memorial Museum and Library
C) Chandra Bose Museum and Library D) Prime Ministers' Museum and Library
Answer : D
✓ The Nehru Memorial Museum and Library (NMML) has officially been renamed the Prime Ministers'
Museum and Library Society recently.
✓ About Prime Ministers' Museum and Library (PMML) Society.
✓ Status: It is an autonomous institution under the Ministry of Culture, Government of India.
✓ Location: It is housed in the historic Teen Murti campus located south of Rashtrapati Bhavan in New
Delhi.
✓ PMML is dedicated to the objective of promoting advanced research on Modern and Contemporary
India.
✓ History:
✓ Designed by Robert Tor Russel and built in 1929-30 as part of Edwin Lutyens' imperial capital, Teen
Murti House was the official residence of the Commander-in-Chief in India.
✓ PMML has four major constituents, namely-
✓ Memorial Museum;
✓ Library on modern India;
✓ Centre for Contemporary Studies;
✓ Nehru Planetarium;
✓ Administration: The General Council and the President and the Vice-President of the PMML Society
are nominated by the Central Government

Q. Which country is the host of the 15th BRICS Summit in 2023?


A) India B) Brazil
C) China D) South Africa
Answer : D

Follow us: Official Site, Telegram, Facebook, Instagram, Instamojo 511


✓ The BRICS group of major emerging economies - Brazil, Russia, India, China and South Africa is
holding its 15th heads of state and government summit in Johannesburg between August 22 and August
24.
✓ South African President Cyril Ramaphosa also invited the leaders of 67 countries to the summit,
including 53 other African Countries, Bangladesh, Bolivia, Indonesia and Iran
Q. Union Health Minister Dr. Mansukh Mandaviya has inaugurated India’s first medical technology expo
‘India MedTech Expo 2023’ in which city?
A) Kolkata B) Chennai
C) Gandhinagar D) Hyderabad
Answer : C
✓ Union Health Minister Dr. Mansukh Mandaviya has inaugurated India’s first medical technology expo,
‘India MedTech Expo 2023’, in Gandhinagar, Gujarat.
✓ The inaugural event was organized on the sidelines of the G20 Health Ministers’ Meeting.
✓ India has been recognized as the ‘Pharmacy of the World’. It is now time for India to take the lead in the
medical devices sector and become the leader in the manufacturing of affordable, innovative, and quality
Medical Devices
✓ The “MedTech Expo 2023 draws inspiration from PM Modi’s vision of making India Aatmanirbhar.
✓ The Expo is set to become an unparalleled and comprehensive platform for demonstrating the capabilities
and promise of the Indian Medical Devices ecosystem.
✓ Gujarat’s pharmaceutical industry ranks number one in India with a 33% share in drug manufacturing
and 28% share of drug exports.
GANDHINAGAR IN NEWS 2023
▪ Union Health Minister Dr. Mansukh Mandaviya has inaugurated India’s first medical technology expo,
‘India MedTech Expo 2023’, in Gandhinagar, Gujarat.
▪ The first WHO Traditional Medicine Global Summit is set to unfold on August 17 and 18, 2023, in the
city of Gandhinagar, Gujarat, India.
▪ The second edition of ‘SemiconIndia 2023’ in Gandhinagar, Gujarat, was inaugurated by Chief Minister
Bhupendrabhai Patel.
▪ Prime Minister, Shri Narendra Modi, inaugurated SemiconIndia 2023 in Gandhinagar, Gujarat. With
the theme “Catalysing India’s Semiconductor Ecosystem”.
▪ The Third G20 Finance Ministers and Central Bank Governors (FMCBGs) meeting, held during the
Indian G20 Presidency, is slated to occur in Gandhinagar, Gujarat from July 17th to 18th, 2023.
▪ The first 15 meetings of the Business 20 (B20) India Inception Meeting will be held in Gandhinagar from
January 22nd to 24th, 2023.
Q. The Centre has approved the establishment of a new spaceport in which state for carrying out the
launches of the Small Satellite Launch Vehicles (SSLV) developed by the Indian Space Research
Organization (ISRO)?
A) Andhra Pradesh B) Karnataka
C) Tamil Nadu D) Telangana
Answer : C
✓ The Centre has approved the establishment of a new spaceport in Kulasekarapattinam, Tamil Nadu for
carrying out the launches of the Small Satellite Launch Vehicles (SSLV) developed by the Indian Space
Research Organization (ISRO).

Follow us: Official Site, Telegram, Facebook, Instagram, Instamojo 512


✓ The Indian Space Policy 2023 has the provision for the utilization of a spaceport for carrying out launch
activities by Non-Government Entities (NGEs), subject to technical feasibility and range safety
constraints.
✓ The Indian Space Policy 2023 has the provision for the utilization of a spaceport for carrying out launch
activities by Non-Government Entities (NGEs), subject to technical feasibility and range safety
constraints.
✓ The Policy opens up the sector for enhanced participation of NGEs across the entire value chain of the
space economy, while clearly delineating the roles of various stakeholders like IN-SPACe, ISRO, New
Space India Limited and Department of Space.
✓ The Government has set up the Indian National Space Promotion and Authorization Centre (IN-SPACe)
as a single-window agency for promotion and authorization of space activities.
✓ The budget allocations for IN-SPACe are 2023-24 ₹95 crore; 2022-23 ₹33 crore and 2021-22 ₹10 crore.
Q. The Supreme Court of India has launched an online portal facility to generate online electronic passes
for visitors to enter the apex court’s premises. What is the name of this portal?
A) Yuktdhara B) SuSwagatam
C) e-Gazette D) TAPAS Portal
Answer : B
✓ The Supreme Court of India has launched ‘SuSwagatam’, an online facility to generate online electronic
passes for visitors to enter the apex court’s premises.
✓ “‘SuSwagatam’ is a web-based and mobile-friendly application that allows users to register themselves
online and request for e-passes for various purposes such as attending court hearings, and meeting
advocates.
✓ The portal also provides role-based secure logins for different types of users where they can upload their
proof of identity and capture their live photo during the registration process.
✓ SuSwagatam’ will enable a user to avoid long queues, and have a paperless entry pass (e-pass) for access
at the apex court

Q. Mushkbudji Rice of which state has recently received the Geographical Indication (GI) tag?
A) Maharashtra B) Karnatata
C) Tamil Nadu D) Jammu and Kashmir
Answer : D
✓ Recently, Mushkbudji Rice received the Geographical Indication (GI) tag.
✓ About Mushkbudji Rice:
✓ It is short bold aromatic rice grown in higher reaches of Kashmir valley.
✓ The cooked rice is unique and possesses a harmonious blend of taste, aroma and rich organoleptic
properties.
✓ It is mainly is grown in areas of Sagam, Panzgam and Soaf Shali of district Anantnag and Beerwah belt
of district Budgam.
✓ The consumption of aromatic rice in Kashmir has now been limited to special occasions, marriages, and
festivals.
Q. NASA’s Curiosity Rover recently captured the crepuscular rays on which planet?
A) Saturn B) Jupiter
C) Neptune D) Mars
Answer : D

Follow us: Official Site, Telegram, Facebook, Instagram, Instamojo 513


✓ NASA's Curiosity Mars rover recently spotted distinctive hexagonal mud cracks on Mars that scientists
speculate may offer the first evidence of wet-dry cycles on the planet.
✓ About Mars Curiosity Rover:
✓ It is a S. robotic vehicle designed to explore the surface of Mars.
✓ It was launched aboard an Atlas V rocket from Cape Canaveral Air Force Station, Florida on Nov. 26,
2011, and landed on Aug. 5, 2012, after taking eight months and 10 days to reach the Red Planet.
✓ The rover is currently roaming Mars' landscape looking for signs of life and learning about the Red
Planet's unique environment.
✓ The rover is part of NASA's Mars Science Laboratory mission which tested a novel landing method that
saw the spacecraft descend on a parachute before its landing system fired up its rockets and hovered as
the rover was lowered down onto the surface.
✓ It is a robotic explorer to land on Mars as part of NASA’s ongoing Mars 2020 Mission.
Q. For the first time in history, which of the following spacecrafts has touched the corona of the Sun?
A) Hinode B) Parker Solar Probe
C) TRACE D) Explorer 33
Answer : B
✓ NASA’s Parker Solar Probe recently executed a short maneuver that kept the spacecraft on track to reach
the aim point for the mission’s sixth Venus flyby.
✓ About Parker Solar Probe
✓ It is a NASA spacecraft designed to study the Sun and its atmosphere.
✓ It was launched on August 12, 2018, from Cape Canaveral Air Force Station, Florida, and is currently
orbiting the Sun in a highly elliptical orbit that takes it closer to the Sun than any previous spacecraft.
✓ The mission objectives of the Parker Solar Probe are to study the structure and dynamics of the Sun's
corona, the Sun's magnetic field, and the solar wind.
Q. What is PUShP?
A) A portal for surplus power trading
B) A platform for demand-response management
C) A marketplace for energy storage solutions
D) An online community for energy sector professionals
Answer : A
✓ Recently,the National Power Committee (NPC) has asked States to provide suggestions for offering
incentives to buyers and sellers on the surplus power portal PUSHp.
✓ About PUShP portal:
✓ A High Price Day Ahead Market (HP-DAM) and Surplus Power Portal (PUShP) was launched by the
Ministry of Power.
✓ It was launched to ensure greater availability of power during the peak demand season at a price higher
than the ceiling of Rs 12 per unit by certain category of sellers.
✓ The power distribution companies (DISCOMs) will be able to indicate their surplus power in block times
/ days / months on portal.
✓ Those DISCOMs who need power will be able to requisition the surplus power.
✓ The new buyer will pay both variable charge (VC) and fixed cost (FC) as determined by Regulators. Once
power is reassigned, the original beneficiary shall have no right to recall as entire FC liability is also
shifted to the new beneficiary.
✓ This will reduce the fixed cost burden on the DISCOMs, and will also enable all the available generation
capacity to be utilized.

Follow us: Official Site, Telegram, Facebook, Instagram, Instamojo 514


Q. Guyana will purchase two Dornier 228 aircraft from which Indian company?
A) Larsen & Toubro (L&T) B) Reliance Industries Limited (RIL)
C) Bharat Electronics Limited (BEL) D) Hindustan Aeronautics Limited (HAL)
Answer : D
✓ Guyana to the purchase of two Dornier 228 aircraft from Hindustan Aeronautics Limited (HAL).
✓ Guyana is also interested in purchasing patrol vessels, a fleet of armoured vehicles, and radar systems.
✓ Guyana and India will also step-up personnel training to improve military-to-military contacts and
Guyana is collaborating with India on cybersecurity and IT operations.
✓ India and Guyana will improve military-to-military contacts through personnel training.
HAL IN NEWS 2023
▪ Guyana to the purchase of two Dornier 228 aircraft from Hindustan Aeronautics Limited (HAL).
▪ Argentina to acquire light and medium utility helicopters from HAL.
▪ Ministry of Defence has recently signed a contract with Hindustan Aeronautics Limited (HAL) for the
procurement of two Dornier Aircraft, aimed at enhancing the aerial surveillance capabilities of the Indian
Coast Guard (ICG). With an overall cost of Rs 458 crore.
▪ United States, General Electric (GE) Aerospace has signed a Memorandum of Understanding (MoU)
with Hindustan Aeronautics Limited (HAL), a state-run Indian aerospace company. The MoU paves the
way for the joint production of fighter jet engines for the Indian Air Force (IAF) and signifies a new era
in the India-US partnership.
▪ HAL received approval from DGCA for indigenously developed ‘black boxes’.
▪ The defence ministry sealed a deal with Hindustan Aeronautics Limited (HAL) to procure six Dornier
aircraft at a cost of Rs 667 crore for the Indian Air Force.
▪ Hindustan Aeronautics Limited (HAL) has unveiled the Hindustan Lead-in Fighter Trainer (HLFT-42)
design of the scale model at the 14th edition of Aero India 2023 held in Bengaluru.
▪ Union Cabinet approved the procurement of 70 HTT-40 basic trainer aircraft from Hindustan
Aeronautics Limited (HAL). The approval was given to the Indian Air Force (IAF). The procurement
would cost around Rs 6,828 crore.
▪ Hindustan Aeronautics (HAL), an aerospace company in India, and EDGE, the top defence company
in the UAE, inked a Memorandum of Understanding at the International Defense Exhibition and
Conference (IDEX).
▪ Hindustan Aeronautics Limited (HAL):
▪ Establishment: 23 December 1940 (Aircraft manufacturing started from 1942).
▪ Founder: Incorporated as Hindustan Aircraft Limited in Bangalore by Walchand Hirachand.
▪ It was formed on 1 October 1964 as Hindustan Aeronautics Limited (HAL).
▪ Headquarters: Bangalore
▪ Management: by a Board of Directors appointed by the President of India through the Ministry of
Defence, Government of India.
▪ The HAL HF-24 Marut fighter-bomber was the first indigenously built fighter aircraft in India.
Q. Indian Army inducted the lighter and more compact version of the developed Weapon Locating Radar
(WLR-M) called “Swathi Mountains” in the Army. It is developed by which company?
A) Hindustan Aeronautics Limited B) Bharat Electronics Limited (BEL)
C) Hindustan Aeronautics Limited D) Larsen & Toubro (L&T)
Answer : B
✓ Indian Army inducted the lighter and more compact version of the indigenously developed Weapon
Locating Radar (WLR-M) called "Swathi Mountains."

Follow us: Official Site, Telegram, Facebook, Instagram, Instamojo 515


✓ About Swathi Mountains.
✓ The Swathi Mountains WLR is an advanced electronically scanned phased array radar.
✓ It is specifically designed for operations in mountainous and high-altitude areas.
✓ It was developed by Bharat Electronics Limited (BEL) in Bengaluru.

Q. ‘Maya’ is in the news recently, what is it?


A) ISRO’s Satellite B) Drone System
C) Operating System D) Missile System
Answer : C
✓ The Defence Ministry recently decided to drop the Microsoft operating system and go for a local one
named Maya operating system.
✓ About Maya OS:
✓ It is a new Operation System (OS) based on an open-source platform.
✓ It has the interface and all functionality like Windows and users will not feel much difference as they
transition to it.
✓ Maya was developed by government development agencies within six months.
✓ Aim: To prevent malware attacks and other cyberattacks which had seen a steep increase.
✓ Currently, Maya is being installed only in Defence Ministry systems and not on computers connected to
the networks of the three Services.
Q. ‘Luna 25’ is the name of the Lunar Mission of which country?
A) Israel B) Russia
C) Japan D) China
Answer : B
✓ The Russian space agency, Roscosmos, launched its moon mission Luna 25 for the lunar south pole on
August 11.
✓ Luna 25 (Luna-Glob lander) is a planned lunar lander mission by Roscosmos.
✓ The Russian space agency, launched Luna 25 spacecraft with a Soyuz 2 rocket from the Vostochny
Cosmodrome.
✓ Luna-Glob lander was the initial name of the mission which was later renamed Luna 25 to emphasize
continuity with the Soviet Luna programme from the 1970s.
✓ Luna 25 consisted of a lander and its mission was to soft-land near the moon's south pole and study the
optical, physical, and chemical properties of moondust and moon soil and the atmosphere.
✓ According to Director General of Roscosmos Yuri Borisov, a maneuvering engine could not be shut
down, and ran for 127 seconds instead of 84. The lander crashed on the lunar surface following the failed
manoeuvre, which created a trajectory that intersected with the Moon instead of a planned elliptical orbit
with a minimum distance of 18km.
✓ The Luna-25 mission cost an estimated $200 million.

Q. United States has officially given the nod for the delivery of M1 Abrams Tank to which country?
A) Pakistan B) Afghanistan
C) Ukraine D) Egypt
Answer : C
✓ The United States has officially given the nod for the delivery of Abrams tanks to Ukraine.
✓ About M1 Abrams Tank.
✓ It is the United States main battle tank.

Follow us: Official Site, Telegram, Facebook, Instagram, Instamojo 516


✓ The first M1 tank was manufactured by General Dynamics Land Systems (GDLS) in 1978 and was
delivered to the US Army in 1980.
✓ It was named in honor to General Abrams, commander of US forces during the Vietnam War.
✓ The export version of the Abrams is used by the armies of Australia, Egypt, Iraq, Kuwait, Poland, and
Saudi Arabia.

Q. Which state/UT has been given GI tag to 'Rajouri Chikri Woodcraft'?


A) Assam B) Odisha
C) Jammu and Kashmir D) Himachal Pradesh
Answer : C
✓ Rajouri Chikri Wood Craft from Rajouri district and Mushqbudji Rice of Anantnag district of Jammu
and Kashmir have now received the Geographical Indication (GI) Tag.
Q. Which company set to expand bandwidth with exclusive capacity on GSAT-24?
A) Sun Direct B) Tata Play
C) Bharti Airtel D) Dish TV
Answer : B
✓ Direct-to-home (DTH) company Tata Play is set to expand its bandwidth capacity by 50% by adding 24
new Ku-band transponders on the communication satellite GSAT-24.
✓ About GSAT-24
✓ It was built by ISRO for NewSpace India Limited (NSIL) which was successfully placed into
geostationary orbit by the Ariane 5 rocket.
✓ t is a 24-Ku band communication satellite weighing 4,180 kg with pan-India coverage for meeting DTH
application needs.
✓ It was the first demand driven communication satellite mission undertaken by NSIL post space sector
reforms.
✓ It is configured on ISRO's proven I-3k Bus with a mission life of 15 years.
✓ NewSpace India Limited (NSIL) NewSpace India Limited headquartered in Bengaluru is a Central
Public Sector Enterprise (CPSE).
✓ What is DTH?
✓ DTH stands for "Direct-to-Home." It is a satellite television broadcasting service that delivers television
signals directly to subscribers' homes through satellite transmission. DTH allows viewers to access a wide
range of TV channels and services without the need for cable connections

Q. Who has laid the foundation stone for an archeological museum at Adichanallur in Tamil Nadu?
A) Anurag Thakur B) Nirmala Sitharaman
C) Smriti Irani D) Rajnath Singh
Answer : B
✓ Nirmala Sitharaman laid the foundation stone for an archeological museum at Adichanallur in Tamil
Nadu’s Tuticorin district.
✓ Adichanallur was one of the five archaeological sites declared to be developed as ‘Iconic Sites’ in the
Union Budget 2020-21. Adichanallur is an archaeological site located on the banks of river Tamirabarani
in this district.
✓ The main objective of the museum would be to focus on establishing the significance of the archaeological
sites identified as part of the cultural landscape of the Tamirabharani valley, not limiting to the site of
Adichanallur

Follow us: Official Site, Telegram, Facebook, Instagram, Instamojo 517


Q. Who has become the first-ever Indian to clinch the individual World Title at the World Archery
Championships final 2023 in Berlin?
A) Deepika Kumari B) Aditi Swami
C) Tarundeep Rai D) Dola Banerjee
Answer : B
✓ Indian archer, Aditi Swami has become the first-ever Indian to clinch the individual World Title at the
World Archery Championships final 2023 in Berlin.
✓ 17-year-old Aditi, became the senior world champion when she defeated Andrea Becerra of Mexico, 149-
147 in the summit clash of the compound women’s event.
✓ Conversely, India’s Ojas Deotale also created history after defeating Lukasz Przybylski of Poland in a
thrilling final to become the new world champion in Men’s Individual compound event.
Q. Monitoring App for Seamless Inspection (MASI), which was seen in the news, is associated with which
action?
A) Monitoring of Child Care Institutions
B) Indian Railway e-Procurement System
C) trade-related payments to foreign
D) encourage inclusive growth and development of various sectors
Answer : A
✓ Recently, the Minister of Women and Child Development informed the Rajya Sabha about MASI Portal.
✓ About the MASI Portal:
✓ Monitoring App for Seamless Inspection (MASI) was developed for synchronous monitoring of the Child
Care Institutions (CCIs) and their inspection mechanisms across the country.
✓ The National Commission for Protection of Child Rights (NCPCR) has developed this application.
✓ The effective and efficient functioning of the mechanism for inspection of CCIs provided under the
Juvenile Justice Act, 2015 (as amended in 2021).
✓ The app is linked to the monitoring Portal where the automatic reports are generated.
✓ This App enables unified inspections by Child Welfare Committees (CWCs), State Inspection
Committees, District Inspection Committees, Members of Juvenile Justice Boards (JJBs) and State
Commissions for Protection of Child Rights (SCPCRs) as laid down under the JJ Act, 2015.
✓ Key Facts about National Commission for Protection of Child Rights
✓ It is a statutory body established by an Act of Parliament, the Commission for Protection of Child Rights
(CPCR) Act, 2005.
✓ It works under the aegis of the Ministry of Women and Child Development.

Q. Consider the following statements regarding JALDOST airboat


1. It is designed to remove excess aquatic weed and floating waste from water bodies.
2. It is developed by the National Aerospace Laboratories (NAL).
Which of the statements given above is/are correct?
A) 1 Only B) 2 Only
C) Both 1 and 2 D) Neither 1 nor 2
Answer : C
✓ Recently, the National Aerospace Laboratories (NAL) unveiled the JALDOST airboat.

Follow us: Official Site, Telegram, Facebook, Instagram, Instamojo 518


✓ About JALDOST:
✓ It is an airboat that operates on water.
✓ It is designed to remove excess aquatic weed and floating waste from water bodies.
✓ It has a closed airtight pontoon type hull to make it inherently unsinkable.
✓ According to NAL, it has a hybrid propulsion system, comprising air propulsion and paddle wheel
propulsion
✓ How it works?
✓ The ability to travel through weed makes JALDOST an ideal platform to collect them and bring them to
the shore.
✓ A steel mesh belt conveyor system fixed in the front collects the waste. The collected waste falls on the
horizontal deck conveyor.
✓ After reaching the shore, the collected waste is unloaded by a rear conveyor system to trucks or tractors.
✓ NAL has developed two versions of the airboat — JALDOST Mark-1 and an upgraded version
JALDOST Mark-2.
✓ Key facts about National Aerospace Laboratories (NAL)
✓ It is a constituent of the Council of Scientific and Industrial Research (CSIR), India, established in the
year 1959.
✓ It is the only government aerospace R&D laboratory in the country's civilian sector.
✓ CSIR-NAL is a high-technology-oriented institution focusing on advanced disciplines in aerospace.

Q. The kuril Island dispute or Northern terriories dispute is a dispute between Japan and ___.
A) China B) Russia
C) United States D) Philippines
Answer : B
✓ In Japan, conservative voices are hinting that Russian and Ukraine war could give Japan a chance to take
control of disputed Kuril Islands.
✓ About Kuril Islands:
✓ These are a set of four islands situated between the Sea of Okhotsk and the Pacific Ocean near the north
of Japan's northernmost prefecture, Hokkaido.
✓ Japan refers to them as Northern territories, Russia calls them the Kuril Islands and South Korea named
them as Dokdo islands.

Q. UNESCO has suggested adding which city to its list of world heritage centres in danger?
A) Venice B) Milan
C) Florence D) Bologna
Answer : A
✓ Recently, experts from the United Nations Educational, Scientific and Cultural Organization (UNESCO)
have stated in a new report that the Italian city of Venice should be added to a list of world heritage sites
in danger.
✓ Venice in Italy is known as “Queen of the Adriatic,” because of its location on the Adriatic coast
✓ About UNESCO heritage danger list:
✓ The list highlights a number of UNESCO World Heritage Sites which are threatened due to factors like
armed conflict and war, earthquakes and other natural disasters, pollution, poaching, uncontrolled
urbanisation and unchecked tourist development.
✓ The UNESCO also sets certain guidelines and criteria under the 1972 World Heritage Convention, which
decide, whether or not, a property is faced with specific and proven imminent danger or threat.

Follow us: Official Site, Telegram, Facebook, Instagram, Instamojo 519


✓ Key facts about UNESCO
✓ UNESCO stands for United Nations Educational, Scientific and Cultural Organization.
✓ It is specialized agency of the United Nations(UN).
✓ The constitution, which entered into force in 1946, called for the promotion of international collaboration
in education, science, and culture.
✓ The agency’s permanent headquarters are in Paris, France.
✓ Parent Organisation - United Nations Economic and Social Council
✓ Goal: The primary goals of UNESCO are to contribute to peace and security by promoting collaboration
among nations through education, science, and culture, and to promote sustainable development and
intercultural dialogue.
✓ UNESCO believes that these areas are crucial for building a more just, peaceful, and inclusive world.

UNESCO IN NEWS 2023


▪ UNESCO have stated in a new report that the Italian city of Venice should be added to a list of world
heritage sites in danger.
▪ The historic Byculla Railway Station, located in Mumbai received the prestigious UNESCO Asia Pacific
Cultural Heritage award.
▪ United States will rejoin UNESCO, the United Nations cultural organization, in July 2023.
▪ Jagdish Bakan wins UNESCO’s 2023 Michel Batisse Award .
▪ UNESCO World Heritage Centre has recommended Santiniketan, the cultural site in West Bengal,
India, for inclusion in the UNESCO World Heritage List.
▪ According to a report published by UNESCO at the United Nations 2023 Water Conference in New
York, 26 percent of the world's population does not have access to safe drinking water.
▪ Former German chancellor Angela Merkel has been awarded the Felix Houphouet-Boigny UNESCO
Peace Prize in Yamoussoukro, Ivory Coast.
▪ Visva-Bharati University is expected to get the status of the world's first "Living Heritage University"
from UNESCO.
▪ UNESCO listed Ukraine’s Odesa a World Heritage Site in Danger.
▪ Assam Chief Minister Himanta Biswa Sarma announced that the Centre has nominated the maidams of
Ahom Kingdom in Charaideo to seek a UNESCO World Heritage Site tag.
▪ The historic maidams of Ahom Kingdom in Charaideo were chosen out of 52 sites for the UNESCO
World Heritage Site tag.
▪ United Nations Educational, Scientific and Cultural Organization (UNESCO)
▪ Formation : 16 November 1945
▪ Headquarters : Paris, France
▪ Director-General : Audrey Azoulay (France)
▪ Member countries : 193
▪ Total UNESCO sites in india : 40.

Q. Who has launched the mobile application Rajmargyatra '?


A) Airports Authority of India (AAI)
B) Border Roads Organisation (BRO)
C) Central Bureau of Investigation (CBI)
D) National Highways Authority of India (NHAI)
Answer : D
✓ Recently, the National Highways Authority of India (NHAI) takes a significant stride towards improving
the highway user experience with the launch of 'Rajmargyatra application.
✓ About Rajmargyatra Application:

Follow us: Official Site, Telegram, Facebook, Instagram, Instamojo 520


✓ It empowers travellers with comprehensive information on Indian National Highways while also offering
an efficient complaint redressal system.
✓ This is user-friendly app which is now available for download on both Google Play Store and iOS App
Store.
✓ The app is currently available in Hindi and English.

Q. India's tiger population estimated to 3,925 with annual growth rate of __ per annum.
A) 5.4% B) 6.1%
C) 7.5% D) 9.2%
Answer : B
✓ India’s tiger population is estimated to be 3,925 with an annual growth rate of 6.1% per annum. This was
revealed in a report released by Minister of State for Environment, Forest and Climate Change Ashwini
Kumar on the occasion of Global Tiger Day 29th July 2023.
✓ Last year (2022) during the celebration of 50 years of the Project Tiger at Mysuru, Prime Minister
Narendra Modi had declared the minimum tiger population of 3 thousand 167, which is the population
estimated from the camera-trapped area
✓ Akashvani correspondent reports that India currently harbors almost 75 percent of the world’s wild tiger
population.
✓ The largest population of tigers is found to be in Madhya Pradesh with 785 in numbers followed by 563
in Karnataka, 560 in Uttarakhand, and 444 tigers in Maharashtra.
✓ As per the report, the tiger abundance within the Tiger Reserves is the highest of 260 tigers in Corbett
followed by 150 in Bandipur and 141 tigers in Nagarhole.
✓ The reports highlight that 35 percent of the tiger reserves urgently require enhanced protection measures,
habitat restoration and subsequent tiger reintroduction.
✓ According to the Wildlife Institute of India's (WII) 5th tiger census (quadrennial), India’s tiger population
increased to 3,682 in 2022 (revised from 3,167 recently), up from 1,411 in 2006, 1,706 in 2010, 2,226 in
2014 and 2,967 in 2018.
Q. Union Education Minister, Dharmendra Pradhan has launched the “ULLAS” mobile application on the
3rd anniversary of NEP. What is the meaning of second “L” in “ULLAS”?
A) Learning B) Lesson
C) Lifelong D) Library
Answer : A
✓ Union Education Minister Dharmendra Pradhan has launched the ULLAS (Understanding Lifelong
Learning for All in Society) initiative’s logo, slogan, and mobile application on the 3rd anniversary of
NEP.
✓ The app serves as a digital gateway for learners to engage in diverse learning resources through the
DIKSHA portal of NCERT.
✓ The ULLAS initiative focuses on promoting functional literacy, vocational skills, & digital literacy.
✓ The newly-unveiled logo and slogan, “ULLAS: Nav Bharat Saksharta Karyakram”, symbolize the
campaign’s enthusiasm and vigor in spreading knowledge and education to every corner of the country.
✓ The initiative aims to ignite the flame of curiosity and continuous learning in each individual, fostering
a culture of knowledge-sharing in communities across India

Q. The first week of August from 01 to 07, is dedicated to which of these observation every year?
A) World Anti-Corruption Week B) World Allergy Week
C) World Breastfeeding Week D) World Tourism Week

Follow us: Official Site, Telegram, Facebook, Instagram, Instamojo 521


Answer : C
✓ The World Breastfeeding Week (WBW) is marked every year in the first week of August, between 1 to 7
August, to raise awareness on the importance of breastfeeding for mothers and infants
✓ The theme of the World Breastfeeding Week 2023 is "Let's make breastfeeding and work, work!"
✓ The week is organized by WABA, WHO and UNICEF since 1991.
Q. What is the theme of the World Breastfeeding Week 2023?
A) Protect Breastfeeding:A Shared Responsibility
B) Support breastfeeding for a healthier planet
C) Empower Parents, Enable Breastfeeding
D) Let's make breastfeeding and work, work!
Answer : D

Q. ISRO launched seven satellites of which country with PSLV-C56 rocket?


A) Israel B) Brazil
C) Singapore D) Sri Lanka
Answer : C
✓ Indian Space Research Organization (ISRO) launched seven satellites of Singapore with PSLV-C56
(PSLV-C56) rocket from Satish Dhawan Space Center in Sriharikota.
✓ This is the 58th flight of PSLV and 17th flight of PSLV in Core Alone configuration.
✓ PSLV-C56 vehicle launched all seven satellites precisely into their intended orbits.
✓ DS-SAR, a radar imaging earth observation satellite is a primary satellite for the mission
✓ The 360-kg satellite, upon deployment at an altitude of 535 km into a Near-equatorial Orbit (NEO),
would be used to support the satellite imagery requirements of various agencies within the Government
of Singapore

Q. Which country is the host of the 5th World Coffee Conference?


A) Chennai B) Hyderabad
C) Bengaluru D) Mumbai
Answer : C
✓ India to host 5th World Coffee Summit in Bengaluru for the first time.
✓ India is all set to host the 5th World Coffee Conference (WCC) for the first time from 25-28 September
2023 in Bengaluru, Karnataka. During the event, tennis player Rohan Bopanna will serve as the brand
ambassador.
✓ The theme of the World Coffee Summit 2023 is "Sustainability through circular economy and
regenerative agriculture".
✓ The conference will bring together growers, farmers, roasters, exporters, policy makers and researchers
from over 80 countries to discuss important topics related to the global coffee sector.
✓ About International Coffee Organization (ICO)
✓ The International Coffee Organization (ICO), established in 1963 under the auspices of the United
Nations following the ratification of the first International Coffee Agreement in 1962, is the organising
body behind the World Coffee Conference.
✓ ICO is headquartered in the UK, and currently has 75 member countries and top-level experts working
towards building a circular coffee economy, promoting regenerative agriculture, promoting sustainability
through consumption and increasing access to finance. work in.

Follow us: Official Site, Telegram, Facebook, Instagram, Instamojo 522


✓ The World Coffee Conference is a high-level event held every four to five years, previously held in the
United Kingdom (2001), Brazil (2005), Guatemala (2010) and Ethiopia (2016).
✓ Its main goal is to facilitate discussion on important issues affecting the global coffee industry.
✓ Brazil is the world's largest coffee producer.
✓ India is the sixth largest producer of coffee in the world.
✓ Karnataka is the largest coffee-producing state in India.
Q. Which Indian-origin author debut novel on Booker Prize longlist 2023?
A) Jhumpa Lahiri B) Chetna Maroo
C) Kiran Desai D) Preeti Shenoy
Answer : B
✓ London-based Indian-origin author Chetna Maroo's debut novel 'Western Lane' is among 13 books to
make the cut for the 2023 BookerPrize longlist revealed
✓ Kenya-born Maroo's novel, set within the context of the British Gujarati milieu, has been praised by the
Booker judges for its use of the sport of squash as a metaphor for complex human emotions.
✓ The longlist comprises 13 books written by authors from various countries, including Ireland, Canada,
the US, and the UK, explore universal and topical themes.
✓ The 2023 Booker Prize winner will be announced on November 26 at an award ceremony in London.
Q. Who has launched India's first Indigenously developed, Magnetic Resonance Imaging (MRI) Scanner in
New Delhi?
A) Jitendra Singh B) Manoj Sinha
C) Piyush Goyal D) Anurag Thakur
Answer : A
✓ Union Minister, Dr Jitendra Singh launched India’s first indigenously developed, affordable, lightweight,
ultrafast, high field (1.5 Tesla), next generation Magnetic Resonance Imaging (MRI) Scanner.
✓ Dr Jitendra said, as an indigenous MRI scanner is made available, the cost of MRI scanning is expected
to reduce considerably for the common man thus allowing a wider access to the otherwise highly priced
MRI scans.
✓ Under the National Biopharma Mission, Bengaluru-based startup Voxelgrids Innovations Pvt Ltd has
developed a compact, lightweight, next generation MRI Scanner to solve the unmet need of the country.
Q. President Droupadi Murmu has inaugurated ‘Unmesha’ – International Literature Festival and ‘Utkarsh’
– Festival of Folk and Tribal Performing Arts in which state?
A) Rajasthan B) Madhya Pradesh
C) Uttar Pradesh D) Uttarakhand
Answer : B
✓ The President of India, Droupadi Murmu has inaugurated ‘Unmesha’ – International Literature Festival
and ‘Utkarsh’ – Festival of Folk and Tribal Performing Arts at Bhopal, Madhya Pradesh.
✓ Culture Department Madhya Pradesh along with Sangeet Natak Akademi and Sahitya Akademi of the
Ministry of Culture is organizing this event from 3rd to 5th August.
✓ The international literature festival Unmesh is Asia’s largest literary convention.
✓ 15 countries will participate in the Unmesh festival.
✓ Literature gave strength to the ideals of our freedom struggle. In every corner of the country, many writers
gave expression to the ideals of freedom and renaissance

Follow us: Official Site, Telegram, Facebook, Instagram, Instamojo 523


Q. Consider the following statements regarding ZARTH application
1. It is an augmented reality transient hunter application
2. It is developed by the National Aeronautics and Space Administration (NASA).
Which of the statements given above is/are correct?
A)1 only B) 2 only
C) Both 1 and 2 D) Neither 1 nor 2
Answer : A
✓ Recently, a team of researchers at the Center for Data Driven Discovery, California Institute of
Technology has developed ZARTH app that allows anyone with a smartphone to ‘hunt’ for transients
✓ About ZARTH App:
✓ The ZTF Augmented Reality Transient Hunter ( ZARTH) is built along the lines of the augmented reality
mobile game.
✓ It allows the user to do serious science while playing a game.
✓ The app uses the open-source Sky Map and adds data daily from the Zwicky Transient Facility (ZTF)’s
robotic telescope at the Palomar Observatory in California.
✓ Palomar is also home to one of the oldest, largest, and most powerful telescopes in the world: the 200-
inch Hale reflector.
✓ The ZTF scans the entire northern sky every two days and uses the data to make large area sky maps that
have important applications in tracking near-earth asteroids and studying supernovae.
Q. Consider the following statements regarding PM SWANidhi Scheme:
1. It was launched by the Ministry of Housing and Urban Affairs on June 01, 2020.
2. It provided as collateral -free loan of Rs 10,000 with low rates of interest for a period of one year.
Which of the statements given above is/are correct?
A) 1 Only B) 2 Only
C) Both 1 and 2 D) Neither 1 nor 2
Answer : C
✓ The Union Housing and Urban Affairs Ministry recently set a new target for its PM SVANidhi scheme
for street vendors.
✓ About PM Street Vendor’s AtmaNirbhar Nidhi (PM SVANidhi) Scheme:
✓ It was launched by the Ministry of Housing and Urban Affairs on June 01, 2020.
✓ Purpose: To provide affordable Working Capital loans to street vendors to resume their livelihoods that
have been adversely affected due to the Covid-19 lockdown.
✓ It is a micro-credit facility that provides street vendors with a collateral-free loan of Rs 10,000 with low
rates of interest (below 12%) for a period of one year, aiding the vendors in getting back on their feet
financially.
✓ The duration of the scheme initially was until March 2022. It has been extended till December 2024, with
a focus on enhanced collateral-free affordable loan corpus, increased adoption of digital transactions and
holistic socio-economic development of the Street Vendors and their families.
✓ Who is Eligible for the Loan?
✓ All vendors who have been vending from or before (March 24, 2020) and with a certificate of vending
can avail the loan.

Follow us: Official Site, Telegram, Facebook, Instagram, Instamojo 524


✓ As per the Street Vendors Act 2014, the Town Vending Committees(which comprises the local authorities
and vendors from an area) issue a certificate of vending after a survey has been conducted of all the
vendors.
✓ Scheme Benefits:
✓ Vendors can avail of a working capital loan of up to Rs. 10,000, which is repayable in monthly instalments
in the tenure of one year.
✓ On timely/ early repayment of the loan, an interest subsidy @ 7% per annum will be credited to the bank
accounts of beneficiaries through Direct Benefit Transfer on a quarterly basis.
✓ There will be no penalty on early repayment of loan.
✓ The scheme promotes digital transactions through cash back incentives up to an amount of Rs. 100 per
month.
✓ The vendors can avail the facility of escalation of the credit limit on timely/ early repayment of loan.
✓ Implementation agency: Small Industries Development Bank of India (SIDBI)

Q. The World’s first ‘Spotless giraffe’ was born in which country?


A) India B) USA
C) China D) New Zealand
Answer : B
✓ A spotless baby giraffe was born at The Brights Zoo, U.S and is believed to be the only singularly coloured
giraffe in the world.
✓ The female giraffe is uniformly brown in colour and lacks the distinctive patched pattern commonly
found in giraffes and also the exceptionally long necks, they are famous for.
✓ Brights Zoo stated that the giraffe already has a height of 6 feet.
Q. Which state government has announced 27% reservation for OBCs in local bodies like panchayats,
municipalities, and civic corporations?
A) Maharashtra B) Gujarat
C) Punjab D) Uttar Pradesh
Answer : B
✓ Gujarat government has introduced 27 per cent reservation for the Other Backward Classes (OBCs) in
local governing bodies, including panchayats, municipalities, and civic corporations, based on
recommendations from the Justice Jhaveri Commission.
✓ Previously, the OBC reservation in local bodies in Gujarat was set at 10 per cent.
GUJARAT IN NEWS 2023.
▪ London-based company OneWeb and the Gujarat Government’s Science and Technology Department
have signed a Memorandum of Understanding (MoU) today for the establishment of a ‘Satellite Network
Portal Site’ in Gujarat.
▪ Union Home and Cooperation Minister Amit Shah organized a virtual ceremony to lay the foundation
stone of the country's first cooperative-run Sainik School.
▪ Sarbananda Sonowal inaugurated the best tourism facilities in the historical lighthouses of Gujarat.
▪ Under the Sagarmala programme, the Ministry of Ports, Shipping and Waterways is developing a
National Maritime Heritage Complex, a world-class facility at Lothal, Gujarat.
▪ Gujarat government signs MoU with US chip maker firm Micron Technology for semiconductor
assembly and testing facility in Ahmedabad.
▪ Indian cabinet has given its approval to Micron Technology’s plan to invest $2.7 billion in setting up a
semiconductor testing and packaging unit in Gujarat.
▪ Urban-20 City Sherpas’ meet begins at Ahmedabad in Gujarat.

Follow us: Official Site, Telegram, Facebook, Instagram, Instamojo 525


▪ India’s largest power company NTPC Ltd has commissioned India’s first green hydrogen blending
project at Kawas, Gujarat.
▪ BJP leader Bhupendra Patel took oath as the Chief Minister of Gujarat for a second straight term.
▪ Madhvendra Singh has been appointed as the first Chief Executive Officer (CEO) of Gujarat Maritime
Cluster by Gujarat Ports Infrastructure Company Limited.
▪ First G20 Tourism Working Group meeting at Dhordo in Kutch region of Gujarat.
▪ Former Gujarat governor and veteran Bharatiya Janata Party (BJP) leader Om Prakash Kohli passed
away.
▪ Gujarat Day is celebrated on May 1 every year. The state was established on May 1, 1960.
▪ Chief Minister Bhupendra Patel has declared the last week of April as “Swagat Saptah” in
commemoration of the completion of 20 years of the “State Wide Attention on Grievances by
Application of Technology” (SWAGAT) initiative.
▪ Deakin University of Australia to set up campus in GIFT city.
▪ Two Australian Universities Wollongong and Deakin to set up Campuses in Gujarat’s GIFT City.
▪ The Youth 20 India Summit will be held at the Maharaja Sayajirao University Vadodara in Gujarat.
▪ International Kite Festival 2023 begins in Ahmedabad, Gujarat.

Q. Which medal did Neeraj Chopra win in the Zurich Diamond League Championship 2023?
A) Gold B) Silver
C) Bronze D) No medal
Answer : B
✓ India's star javelin thrower Neeraj Chopra qualified for the Diamond League final.
✓ The final of this league will be held next month in Eugene, America on 16 and 17 September.
✓ Neeraj Chopra secured second place in the Zurich Diamond League Championships 2023 with a best
throw of 85.71 metres.
✓ Jakub Wadlech of Czech Republic secured first place with 85.86 metres.
✓ At the same time, Germany's Julian Weber finished third with a throw of 85.04 metres.
✓ Neeraj Chopra's best throw is 89.94 metres, which is a national record.
Q. In which state has the Griha Lakshmi Yojana been recently launched for women?
A) Madhya Pradesh B) Uttar Pradesh
C) Karnataka D) Assam
Answer : C
✓ The state government of Karnataka has launched a social welfare scheme for women - Griha Lakshmi
Yojana - in Mysore.
✓ Under this, the female head of a family will get Rs 2,000 per month from the government.
✓ The registration for Griha Lakshmi Yojana started from 19th July.
Q. Which Ministry and the National Safai Karamcharis Finance and Development Corporation (NSKFDC)
signed an MoU for 2023-25?
A) Ministry of Tribal Affairs B) Ministry of Home Affairs
C) Ministry of Labour and Employment D) Ministry of Social Justice and Empowerment
Answer : D
✓ A momentous collaboration has been cemented as the Ministry of Social Justice & Empowerment and
the National Safai Karamcharis Finance and Development Corporation (NSKFDC) inked a
Memorandum of Understanding (MoU) for the fiscal years 2023-24 and 2024-25.

Follow us: Official Site, Telegram, Facebook, Instagram, Instamojo 526


✓ This strategic partnership aims to enhance the socio-economic upliftment of Safai Karamcharis, Manual
Scavengers, Waste Pickers and their dependants.
✓ NSKFDC also provides loans upto Rs.1.00 lacs under its Mahila Samridhi Yojana and Micro Credit
Finance Scheme. The overall rate of interest is 4% and 5%, respectively. The other loan schemes are
General Term Loan and Green Business Schemes under which loans upto Rs.15.00 lacs and upto
Rs.30.00 lacs respectively are provided with rate of interest at 6% and 1% rebate for women beneficiaries.
✓ NSKFDC has provided Skill development Training to 20686 sanitation workers and their dependents
under SRMS and PM DAKSH Yojana during FY 2022-23.
Q. Which Countries have signed a MoU to boost cooperation in civil aviation in New Delhi?
A) India and Australia B) India and Switzerland
C) India and United Kingdom D) India and New Zealand
Answer : D
✓ Recently, India and New Zealand signed a Memorandum of Understanding (MoU) to boost cooperation
in civil aviation.
✓ Furthermore, a meeting took place between the Agriculture Ministers of both countries, where they not
only reaffirmed their commitment to boost agricultural collaboration and invited India to join the Global
Research Alliance(GRA).
✓ New Zealand
✓ Capital : Wellington
✓ Currency : Dollar
✓ Prime Minister : Chris Hipkins
Q. The amount of Chief Minister Kanya Sumangala Yojana has been increased, it is related to which state?
A) Bihar B) Uttar Pradesh
C) Assam D) Madhya Pradesh
Answer : B
✓ Uttar Pradesh Chief Minister Yogi Adityanath has announced to increase the amount fixed for the
beneficiaries of Mukhyamantri Kanya Sumangala Yojana from Rs 15,000 to Rs 25,000.
✓ Chief Minister Kanya Sumangala Yojana was started in the year 2019.
✓ Under this scheme, financial assistance is provided to girls to complete their education and have a better
future
Q. In which city will the Miss World 2023 event be organized?
A) Kolkata B) Mumbai
C) New Delhi D) Kashmir
Answer : D
✓ The 71st Miss World 2023 contest will be held in Kashmir later this year, in which 140 countries will
participate.
✓ Julia Morley, CEO of Miss World Organization has given information about this.
✓ Sini Shetty will represent India in Miss World 2023.
✓ This prestigious international beauty pageant is being organized in India after 27 years.
✓ India would be hosting the pageant after nearly three decades, the last time being in 1996

Follow us: Official Site, Telegram, Facebook, Instagram, Instamojo 527


Q. Who has launched Tele-Law- 2.0 which Integrates Tele-Law and Nyaya Bandhu App to ensure access
to Justice in New Delhi?
A) Meenakshi Lekhi B) G. Kishan Reddy
C) Rattan Lal Kataria D) Arjun Ram Meghwal
Answer : D
✓ Law and Justice Minister Arjun Ram Meghwal launched Tele-Law- 2.0 that Integrates Tele-Law and
Nyaya Bandhu App to ensure access to Justice in New Delhi.
✓ The Department of Justice (DOJ) hosted the Tele-Law 2.0 event at Siri Fort Auditorium, New Delhi.
✓ The event marks the achievement of helping 5 million citizens and highlights the use of technology in
providing pre-litigation advice to the public.
✓ Tele-law services are being integrated with legal representation services under the Nyaya Bandhu (Pro
Bono) program.
✓ This integration streamlines access to legal advice, assistance, and representation through a unified
registration process on the Tele-Law platform.
✓ Screening of a documentary showing the journey of tele-law from 2017 to 2022.
✓ "Tele-Law-2.0" is being launched, which combines the Tele-Law and Nyay Bandhu apps, Also an e-
tutorial has been released.
✓ Unveiling of “Voices of Beneficiaries”, a compilation of firsthand experiences shared by people who have
benefited from Tele-Law.
✓ Presenting the "Achievers Catalogue", which acknowledges outstanding performers such as Paralegal
Volunteers, Village Entrepreneurs, Panel Lawyers, and State Coordinators for the years 2022-2023 and
April to June 2023.
Q. Indian women’s blind cricket team has won the gold medal after defeating which country in the IBSA
World Games 2023?
A) England B) Pakistan
A) Australia D) South Africa
Answer : C
✓ Indian women’s blind cricket team created history as it defeated Australia in the finals by 9 wickets and
won the gold medal at the IBSA World Games.
✓ The women’s cricket team showed exceptional resilience in the rain-spoiled match and ultimately became
the first-ever cricket winners at the IBSA World Games.
✓ The Indian men’s blind cricket team earned a silver medal at the IBSA World Games 2023 after losing
to Pakistan in the men’s T20 cricket final held at Edgbaston, Birmingham.
✓ The team became the first-ever cricket winners at the IBSA World Games
Q. Who has released books based on speeches of Prime Minister Narendra Modi titled “Sabka Saath, Sabka
Vikas, Sabka Vishwas” at the Kushabhau Thackeray International Convention Centre in Bhopal?
A) Anurag Thakur B) Nirmala Sitharaman
C) Smriti Irani D) Rajnath Singh
Answer : A
✓ Union Minister for Information and Broadcasting, Youth Affairs and Sports Anurag Singh Thakur and
Chief Minister of Madhya Pradesh Shivraj Singh Chouhan have released books based on speeches of
Prime Minister Narendra Modi titled “Sabka Saath, Sabka Vikas, Sabka Vishwas” at the Kushabhau
Thackeray International Convention Centre in Bhopal.

Follow us: Official Site, Telegram, Facebook, Instagram, Instamojo 528


✓ The speeches of Prime Minister Narendra Modi are always connected with the public, make relations,
create motion, and are based on Nation-First policy and that is why they are perfect and reach the heart
of the public.
✓ The second volume of the book covers 86 speeches from the second year of Mr. Modi’s second term in
the Prime Minister’s Office, i.e. from June 2020 to May 2021.
✓ Whereas, the third volume is a collection of 80 speeches from June 2021 to May 2022, the third year of
Mr. Modi’s second term in the Prime Minister’s Office.

Q. Which Indian player won the bronze medal in the World Badminton Championship 2023?
A) Shrikant Kidambi B) Lakshya Sen
C) H. S. Prannoy D) Parupalli Kashyap
Answer : C
✓ India's star badminton player H. S. Prannoy has won bronze medal in BWF World Championship 2023.
✓ He has become the fifth Indian to win a medal in the men's singles matches of the World Championships.
✓ India has so far won one silver and 4 bronze in men's singles in the history of the World Badminton
Championships.
✓ Kidambi Srikanth is the only Indian to reach the men's singles final (in 2021) of the same event.
✓ The 28th edition of BWF World Badminton Championships 2023, sponsored by TotalEnergies, was held
at Royal Arena, Copenhagen, Denmark from 21st to 27th August.
✓ Copenhagen has been chosen as the host for the BWF World Championships for the fifth time.
✓ India's star shuttler HS Prannoy won the gold medal in the Malaysia Masters Badminton Championship
2023 played in Kuala Lumpur, Malaysia.
Q. Who is the President of Zimbabwe who has been re-elected for a second five-year term?
A) Emmerson Mnangagwa B) Robert Pereiro
C) Jacob Zuma D) Tino Janakika
Answer : A
✓ Zimbabwe's President Emmerson Mnangagwa has been re-elected for a second and final five-year term
after the results of Zimbabwe's presidential election were announced.
✓ By the way, there has been a history of irregularities in elections in Zimbabwe, with the help of which
former President Robert Mugabe remained in power for almost four decades.
✓ Zimbabwe is a South African country, whose capital is Harare.
✓ About Zimbabwe
✓ It is a landlocked country in Southern Africa.
✓ The Zambezi River flows through Zimbabwe and is home to Victoria Falls.
✓ Victoria Falls plunges 108 meters down the Batoka Gorge.
✓ Downstream of the Zambezi River are the Matusadona and Mana Pools National Parks.
✓ Capital - Harare
✓ Currency - United States Dollar

Q. Which Indian state is the first to launch ‘Breakfast Scheme’ for students of Government schools?
A) Kerala B) Madhya Pradesh
C) Bihar D) Tamil Nadu
Answer : D
✓ Tamil Nadu Chief Minister MK Stalin launched the expanded formed of the state government's breakfast
scheme for government schools.

Follow us: Official Site, Telegram, Facebook, Instagram, Instamojo 529


✓ Stalin launched the expansion at a school at Tirukkuvalai in Nagapattinam district. He served food to
students to mark the launch.
✓ Video from the launch showed Stalin sitting on the floor and eating the meal with the children.
✓ The scheme was initially launched last year for 1,545 government primary schools, but has now been
expanded to 31,000 schools across the state.
✓ Here we explain what's Tamil Nadu's CM Breakfast Scheme and how it has performed so far
✓ Tamil Nadu CM MK Stalin had launched the CM Breakfast Scheme on September 15, 2022. It provides
free breakfast to children primary government schools
Q. Which state government has Issues Notification Banning Plastic Water Bottles Below 1 Litre from 2
October 2023?
A) Bihar B) Odisha
C) Assam D) Andhra Pradesh
Answer : C
✓ The Assam Environment and Forest Department has taken a significant step by announcing the ban on
the use and production of plastic water bottles below 1000 ml capacity within the state.
✓ This transformative initiative, set to be enforced from 2nd October 2023, reflects Assam’s commitment
to combat plastic pollution and promote sustainable practices.
✓ Recognizing the environmental hazards posed by such smaller plastic bottles, the Assam government’s
decision aligns with its commitment to safeguard the state’s natural beauty and biodiversity.

BAN IN NEWS 2023


▪ Indian sprinter Dutee Chand gets four-year ban for failing dope test.
▪ Assam Govt Issues Notification Banning Plastic Water Bottles Below 1 Litre.
▪ FIFA lifts ban on Sri Lanka Football Federation.
▪ Ministry of Information and Broadcasting (I&B) in India has recently cracked down on over 150 websites
and YouTube-based news channels since May 2021.
▪ These actions were taken in response to the production of content deemed “anti-India” and in violation
of Section 69A of the Information Technology (IT) Act.
▪ India bans export of non-basmati white rice.
▪ New Zealand becomes first country to ban plastic produce bags.
▪ After ChatGPT, Italy plans to ban English language.
▪ Seattle created history becomes first city in US to ban caste discrimination.
Q. Which state has emerged as the top state in the Nutrition Awareness Index 2023, scoring 11% above the
national average?
A) Punjab B) Uttar Pradesh
C) Uttarakhand D) Madhya Pradesh
Answer : A
✓ Punjab emerged as the top state in Nutrition Awareness Index 2023
✓ The ‘Right To Protein’ campaign released the ‘Nutrition Awareness Index 2023’, a state-wise index
assessing awareness, affordability, and accessibility of nutrition in India as perceived by end-consumers
(citizens).
✓ The Nutrition Awareness Index aims to serve as an important tool to advance the vision of nutrition
security in India – with regards to commitments, practices, and management.
✓ Punjab emerged as the top state, scoring 11% above the national average, while Bihar ranked at the
bottom, scoring 10% below the national average.

Follow us: Official Site, Telegram, Facebook, Instagram, Instamojo 530


Q. When is the National Sports Day observed in India?
A) 29 August B) 28 August
C) 27 August D) 26 August
Answer : A
✓ In India, the National Sports Day is observed every year on 29 August, on the occasion of the birth
anniversary of hockey legend Major Dhyan Chand.
✓ The National Sports Day, also known as Rashtriya Khel Divas, was celebrated for the first time in 2012.
✓ who was known as ‘The Wizard’ or ‘The Magician’ of hockey for his superb ball control.
✓ The legendary hockey player was born on 29 August 1905 in Allahabad, Uttar Pradesh
✓ He won gold medals in Olympics for India in the years 1928, 1934 and 1936. He scored over 400 goals
in his career, from 1926 to 1948. It is believed that after India defeated Germany in the 1936 Olympics
final by 8-1, Adolf Hitler offered him a senior post in the German Army, to which Chand refused.
✓ His autobiography “Goal!” was published by Sport & Pastime (Madras) in 1952.
✓ Government of India (GoI) awarded him the 3rd highest civilian honour of Padma Bhushan in 1956.
Q. The National Sports Day is observed in India to mark the birth anniversary of which Indian sports person?
A) K. D. Jadhav B) Dhanraj Pillay
C) Milkha Singh D) Major Dhyan Chand
Answer : D
✓ National Sports Day 2023 Theme : The theme for this year's National Sports Day celebration is "Sports
are an enabler to an inclusive and fit society".
Q. Union Home and Cooperation Minister Amit Shah has chaired the 26th meeting of the Western Zonal
Council in which city?
A) Gandhinagar B) Ahmedabad
C) New Delhi D) Bengaluru
Answer : A
✓ Union Home and Cooperation Minister Amit Shah has chaired the 26th meeting of the Western Zonal
Council at Gandhinagar, Gujarat.
✓ The meeting was organized by the Inter-State Council Secretariat under the Ministry of Home Affairs in
collaboration with the Government of Gujarat.
✓ The Western Zonal Council comprises the states of Gujarat, Goa, and Maharashtra, and the Union
Territories of Dadra and Nagar Haveli and Daman & Diu.
✓ Western Zone is an important zone of the country and with a contribution of 25% to the country's GDP,
this region is the hub of finance, IT, diamond, petroleum, automobile and defence.
Q. Who signs MoU with NRL for Export of Petroleum Cargo via National Waterways-2 (Brahmaputra)
and Indo-Bangladesh Protocol Route?
A) IWAI B) IWNI
C) IWGI D) IBAI
Answer : A
✓ The Numaligarh Refinery Limited (NRL) on signed an MoU with the Inland Waterways Authority of
India (IWAI) for transporting petroleum products to Bangladesh and southeast Asian countries from
Assam.

Follow us: Official Site, Telegram, Facebook, Instagram, Instamojo 531


✓ The MoU will facilitate the transportation of NRL's petroleum products from IWAI's jetty at Jogighopa
in Assam to neighbouring Bangladesh and southeast Asian nations.
✓ Speaking on the occasion, Sonowal said the National Waterways-2 (Brahmaputra) and Indo Bangladesh
Protocol Route (IBPR) will be used for exporting petroleum products to Bangladesh and southeast Asian
countries.
✓ The NRL will export around 10,000 MT of petroleum and petrochemical products per month from IWAI
Jogighopa Multi Modal terminal.
✓ It will also set up a Port of Loading (POL) Oil Terminal at Jogighopa Logistics Park with rail
connectivity.
✓ The Act East policy is empowering the region towards unlocking the full potential of Hydrocarbon vision
2030..
✓ National Waterway 1 (NW-1): The Ganges River between Allahabad in Uttar Pradesh and Haldia in
West Bengal.
✓ National Waterway 2 (NW-2): The Brahmaputra River in Assam, from Sadiya to Dhubri.
✓ National Waterway 3 (NW-3): The West Coast Canal in Kerala.
✓ National Waterway 4 (NW-4): The Krishna River between Wazirabad and Vijayawada.
✓ National Waterway 5 (NW-5): The Brahmani River in Odisha.
Q. Kaliveli Bird Sanctuary, which was named a bird sanctuary recently, is located in which state?
A) Tamil Nadu B) Uttar Pradesh
C) Rajasthan D) Telangana
Answer : A
✓ Recently, the Southern Bench of the National Green Tribunal (NGT) has withheld the environmental
clearance granted for fishing harbours at Alamparaikuppam and Azhagankuppam, located in the
intertidal area of Kaliveli bird sanctuary.
✓ About Kaliveli Bird Sanctuary:
✓ This wetland is the second-largest brackish water lake in South India after Pulicat Lake.
✓ The first declaration was issued by the Villupuram District Administration in Tamil Nadu under Section
18 of the Wildlife Protection Act, 1972, for setting up this sanctuary.
✓ The Kaliveli Lake is connected to the Bay of Bengal by the Uppukalli Creek and the Edayanthittu
sanctuary and is visited for nesting by migratory birds on the Central Asian flyway.
✓ The southern part of the wetland has been reserved land since 2001.
Q. The Jambughoda Wildlife Sanctuary (JWS) is located in which state?
A) Kerala B) Gujarat
C) Madhya Pradesh D) Rajasthan
Answer : B
✓ Recently, the Gujarat State Board for Wildlife (GSBWL) deferred a decision on a proposal of the Gujarat
State Minerals Development Corporation (GSDMA) seeking more land for “removing manganese
dump” from the eco-sensitive zone (ESZ) of Jambughoda Wildlife Sanctuary in Panchmahal district.
✓ About Jambughoda Wildlife Sanctuary.
✓ It is located in Gujarat’s south-central part.
✓ Vegetation: It contains Grasslands, water-borne plants, medicinal herbs, dry southern tropical rain
forests, dry deciduous mixed and secondary forests, and dry scrubs of the deciduous type.
✓ What are Eco-Sensitive Zones?
✓ Land within 10 km of the boundaries of national parks and wildlife sanctuaries is to be notified as eco-
fragile zones or Eco-Sensitive Zones (ESZ).
✓ While the 10-km rule is implemented as a general principle, the extent of its application can vary.

Follow us: Official Site, Telegram, Facebook, Instagram, Instamojo 532


✓ The Union government can also notify areas beyond 10 km as ESZs if they hold larger ecologically
important “sensitive corridors”.
Q. The Indian Air Force (IAF) has sent a contingent to participate in Exercise Bright Star-23, a biennial
multilateral tri-service exercise held in which country?
A) Australia B) Egypt
C) Singapore D) Japan
Answer : B
✓ Recently, an Indian Air Force (IAF) contingent departed to participate in Exercise BRIGHT STAR-23,
scheduled to be held at Cairo (West) Air Base, Egypt, from 27 August to 16 September 2023.
✓ About EXERCISE BRIGHT STAR-23:
✓ It is a biennial multilateral tri-service exercise.
✓ This multinational exercise was launched in 1980 as part of the US-brokered peace treaty between Egypt
and Israel.
✓ This is the first time that IAF is participating in Ex BRIGHT STAR-23.
✓ Participating countries: United States of America, Saudi Arabia, Greece and Qatar.
✓ The Indian Air Force contingent will consist of five MiG-29, two IL-78, two C-130 and two C-17 aircraft.
✓ India and Egypt have had an exceptional relationship and deep cooperation wherein the two jointly
undertook the development of aero-engine and aircraft in the 1960s, and training of Egyptian pilots was
done by Indian counterparts
Q. United World Wrestling has recently suspended the wrestling association of which country?
A) India B) Pakistan
C) China D) Iran
Answer : A
✓ United World Wrestling has suspended the Wrestling Federation of India (WFI) for not holding the
Wrestling Association elections on time.
✓ Due to which Indian wrestlers will not be allowed to compete in the upcoming World Championships
under the Indian flag.
✓ The IOA had appointed a panel on April 27 and the committee had to conduct elections within 45 days.
Q. What is the rank of India at the 15th International Olympiad on Astronomy and Astrophysics (IOAA)
2023?
A) 2nd B) 3rd
C) 4th D) 5th
Answer : A
✓ India has secured the second rank in the 16th International Olympiad of Astronomy and Astrophysics
(IOAA) to be held in Chorzów, Poland from 10 to 20 August 2023.
✓ India won four gold and one silver medal.
✓ United Kingdom secured the first position with five gold.
✓ A total of 236 students from 50 countries participated in this Olympiad, and collectively won 27 gold, 41
silver and 50 bronze medals in all categories.

Follow us: Official Site, Telegram, Facebook, Instagram, Instamojo 533


Q. The Central Reserve Police Force (CRPF) recently switched to indigenously developed “COPSYSS
“software for psychological screening of personnel prior to their induction in dignitary protection teams.
What is the full form of COPSYSS?
A) Commercial Psychological Screening System
B) Computerized Psychological System Screening
C) Computerized Psychological Survey System
D) Computerized Psychological Screening System
Answer : D
✓ The Central Reserve Police Force (CRPF) recently switched to indigenously developed software for
psychological screening of personnel prior to their induction in dignitary protection teams.
✓ Computerised Psychological Screening System (COPSYSS) is a software designed and developed by the
Defence Institute of Psychological Research (DIPR) of the Defence Research & Development
Organisation (DRDO).
✓ The indigenous physiological test system will help in the selection of candidates for escorting dignitaries.
✓ The force has a tough system in place for physical fitness but this test will ensure sharp mental fitness.
✓ However, the Special Protection Group (SPG) has been undergoing the Vienna Test System (VTS) for at
least 15 years, and the National Security Guard (NSG) has switched to VTS in 2017.

Q. Who has won the Cincinnati Open 2023 title after defeating Carlos Alcaraz of Spain?
A) Novak Djokovic B) Rafael Nadal
C) Alexander Zverev D) Daniil Medvedev
Answer : A
✓ Serbia’s Novak Djokovic secured the title of Cincinnati Open 2023 after defeating Carlos Alcaraz of
Spain.
✓ The win delivered Djokovic his 95th career title and 39th Masters 1000 crown.
✓ Serbian Tennis Player, Novak Djokovic has won the French Open 2023 men’s singles final, clinching his
23rd Grand Slam title.
✓ Novak Djokovic defeated Stefanos Tsitsipas in the final of men’s singles match of the Australian Open
2023

Q. Dr. V S Arunachalam passed away recently. He was the former Director General of which organization?
A) ISRO B) DRDO
C) L&T D) BARC
Answer : B
✓ Dr V S Arunachalam, founder chairman of Bengaluru-based think tank Center for Science, Technology
and Policy (CSTEP), passed away at the age of 87 in California.
✓ Arunachalam helmed the Defence Research and Development Organisation (DRDO) and was the
Scientific Advisor to the Defence Minister from 1982-92.
✓ He was conferred the Shanti Swarup Bhatnagar Award (1980), Padma Bhushan (1985), and Padma
Vibhushan (1990) for his contribution to engineering science and technology
Q. The World Photography Day is celebrated on which day?
A) 16 August B) 17 August
C) 18 August D) 19 August

Follow us: Official Site, Telegram, Facebook, Instagram, Instamojo 534


Answer : D
✓ World Photography Day is celebrated on 19 August every year to promote photography as a hobby and
also inspire photographers around the globe to share a single photo with the rest of the world.
✓ Theme 2023 – LANDSCAPES
✓ The first official World Photo Day was observed on August 19, 2010.
✓ The origin of World Photo Day comes from the invention of the Daguerreotype, a photographic process
developed by Frenchmen Louis Daguerre and Joseph Nicephore Niepce in 1837.

Q. Which day in the year is celebrated as World Humanitarian Day?


A) 19 August B) 20 August
C) 18 August D) 17 August
Answer : A
✓ The World Humanitarian Day (WHD) is observed every year on 19 August to pay tribute to
humanitarian personnel and those workers who lost or risked their lives while doing humanitarian
service.
✓ Theme 2023 – No matter what

Q. When is the World Mosquito Day observed globally?


A) 20 August B) 19 August
C) 18 August D) 17 August
Answer : A
✓ World Mosquito Day is observed on 20 August annually to raise awareness about the causes of malaria
and how it can be prevented.
✓ The day also commemorates the discovery made by British doctor Sir Ronald Ross in 1897 that female
mosquitoes transmit malaria between humans.
✓ In 1902. Ross won Nobel Prize for Medicine becoming the first British person to receive the award.
Q. Sadbhavana Diwas is observed in India annually to commemorate the birth anniversary of which Indian
leader?
A) Syama Prasad Mukherjee B) Atal Bihari Vajpayee
C) Sardar Vallabhbhai Patel D) Rajiv Gandhi
Answer : D
✓ Every year India observes Sadbhavana Diwas on August 20 to commemorate the birth anniversary of
late erstwhile Prime Minister, Rajiv Gandhi.
✓ ‘Sadbhavana’ means goodwill and bonafide.
✓ August 20, 2023 marks the 79th birth anniversary of Rajiv Gandhi who was the youngest Prime Minister
of India at the age of 40.
✓ The day aims to promote National Integration and Communal Harmony among people of all relgions,
languages and regions and stop violence.
✓ Rajiv Gandhi was the 6th Prime Minister of India, serving from 1984 to 1989.
Q. In India, when is the Akshay Urja Diwas celebrated?
A) 19 August B) 20 August
C) 18 August D) 17 August

Follow us: Official Site, Telegram, Facebook, Instagram, Instamojo 535


Answer : B
✓ Akshay Urja Diwas (Renewable Energy Day) is observed every year on 20 August since 2004 to raise
awareness about the developments and adoption of renewable energy in India.
✓ The Akshay Urja Day was initiated by the Indian Ministry for New & Renewable Energy Sources in
2004.
✓ The energy such as Biogas, Solar Energy, Wind energy, hydroelectrical power are few example of Akshay
Urja.
✓ The main motive of Akshya urja Diwas is to make people aware that they have to think about the
renewable energy (Akshya Urja) apart from tradition energy.
Q. Who has been appointed as the new brand ambassador of Bharat Petroleum Corporation Limited
(BPCL)?
A) Virat Kohli B) Rohit Sharma
C) Rahul Dravid D) Sachin Tendulkar
Answer : C
✓ Bharat Petroleum Corporation Limited (BPCL), the second-largest Indian oil marketing company
(OMC), has announced Rahul Dravid as its latest brand ambassador.
✓ Rahul Dravid’s remarkable sportsmanship, role model status, integrity, dependability, and
trustworthiness perfectly reflect the values we stand for and make him the perfect fit for brand BPCL.
✓ Rahul Dravid will represent BPCL’s Pure for Sure initiative and its range of MAK lubricants.
✓ CMD of BPCL – G. Krishnakumar

Q. NHPC signs MoU with RITES for 2,880 MW Dibang Multipurpose Project in Which State?
A) Uttarakhand B) Uttar Pradesh
C) Arunachal Pradesh D) Himachal Pradesh
Answer : C
✓ State-owned hydro power giant NHPC has inked an initial pact with Rail India Technical and Economic
Service (RITES) to construct railway siding for a 2,880 MW Dibang multipurpose project in Arunachal
Pradesh.
Q. The All-India Council for Technical Education (AICTE) has partnered with which institute for a faculty
development programme on artificial intelligence (AI) and data science?
A) Indian Institute of Science B) Jawaharlal Nehru University
C) Amrita Vishwa Vidyapeetham D) Jio Institute
Answer : D
✓ The All-India Council for Technical Education (AICTE) has partnered with Jio Institute for a faculty
development programme on artificial intelligence (AI) and data science.
✓ The partnership aims to upskill and reskill faculty members from AICTE-approved institutions. It will
also explore emerging opportunities within the technological landscape.
✓ The collaborative initiative aims to empower academic leaders and senior faculty members with a deep
understanding of AI and data science while also emphasizing the ethical considerations of utilizing AI
tools for immersive learning.

Follow us: Official Site, Telegram, Facebook, Instagram, Instamojo 536


Q. The World Senior Citizen Day is marked every year on which day?
A) 20 August B) 21 August
C) 22 August D) 23 August
Answer : B
✓ World Senior Citizen Day is observed globally on August 21 every year to raise awareness about issues
affecting older people, such as deterioration with age and the abuse of the elderly and support, honor and
show appreciation to seniors and to recognize their achievements.
✓ The Day was proclaimed by the United Nations General Assembly on December 14, 1990.
✓ The day was officially founded by the former President of the United States of America, Ronald Reagan
in 1988 to dedicate a day to older adults and their issues.
Q. Who has been designated as a “national icon” by the Election Commission (EC) to encourage greater
voter participation in the electoral process?
A) Mahendra Singh Dhoni B) Sachin Tendulkar
C) Kapil Dev D) Sourav Ganguly
Answer : B
✓ Cricketing legend Sachin Tendulkar has been designated as a “national icon” by the Election
Commission (EC) to encourage greater voter participation in the electoral process.
✓ A memorandum of understanding has been signed between Tendulkar and the poll panel. As part of the
three-year agreement, Tendulkar will spread voter awareness.
✓ This collaboration would mark a significant step towards leveraging Tendulkar’s unparalleled impact
with the youth for increasing voters’ participation in the forthcoming elections, especially in general
elections (to Lok Sabha), 2024.
✓ Last year, the commission had recognized actor Pankaj Tripathi as a national icon.

Q. Who has been appointed as the new Chairman of the National Green Tribunal (NGT)?
A) Advait Thakur B) Trishneet Arora
C) Utkarsh Kawatra D) Prakash Shrivastava
Answer : D
✓ Former Chief Justice of the Calcutta High Court, Justice Prakash Shrivastava has been appointed as the
new Chairman of the National Green Tribunal (NGT)
✓ The Cabinet Appointments Committee of the Central Government approved the appointment of Justice
Prakash Srivastava as the Chairman of the NGT.
✓ NGT was set up under the National Green Tribunal Act in 2010.
✓ Earlier, Justice Shiv Kumar Singh was appointed as the executive chairperson after the retirement of the
then NGT Chairman Justice Adarsh Kumar Goel in July.
✓ About National Green Tribunal (NGT)
✓ Founded – 18 October 2010 under the National Green Tribunal Act, 2010.
✓ Headquarters- Delhi
✓ Chairman – Prakash Shrivastavs
✓ Regional Offices – 4 (Bhopal, Pune, Kolkata and Chennai)
✓ Tenure- Term of 3 years or till the age of 65 years and not eligible for reappointment.
✓ Objective- To speed up disposal of environmental issues
✓ India is the third (first developing) country in the world after Australia and New Zealand to set up a
Special Environment Tribunal.

Follow us: Official Site, Telegram, Facebook, Instagram, Instamojo 537


Q. Who is the author of the book titled “Drunk on Love: The Life, Vision and Songs of Kabir”?
A) Anne Frank B) Vipul Rikhi
C) Christopher Paolini D) Alexander Pope
Answer : B
✓ A new book titled “Drunk on Love: The Life, Vision and Songs of Kabir” is written by Vipul Rikhi.
✓ A new book aims to capture the life of mystic poet Kabir through popular legends, his vision and his
poetry that has been quoted and translated extensively.
✓ This book was published by HarperCollins India and presents the 15th-century poet, as he is described,
quoted, and loved in popular imagination.
Q. Which country has demanded India to implement the PM Bharatiya Janaushadhi Kendra Model in its
country?
A) Indonesia B) Malaysia
C) Philippines D) Singapore
Answer : A
✓ Indonesia seeks to emulate India’s Jan Aushadhi Kendra model
✓ Indonesian Health Minister Budi Gunadi Sadikin visited a Jan Anushadhi Kendra in Gujarat’s
Gandhinagar.
✓ Sadikin praised India’s healthcare model and sought to replicate the same PM Bhartiya Janaushadhi
Pariyojana in Indonesia.
✓ Jan Aushadhi Kendras provide affordable medicines to senior citizens and middle-class families in India,
with prices ranging from Rs 10 to Rs 20.
✓ Notably, Prime Minister Narendra Modi during his Independence Day speech from Red Fort this year
said that the government has plans to increase the number of ‘Jan Aushadhi Kendras’ from 10,000 to
25,000.
✓ Jan Aushadhi Kendras have provided new strength to senior citizens and middle-class families in our
nation. We provide medicines which cost Rs 100 in the market for just Rs 10, Rs 15, Rs 20 through Jan
Aushadhi Kendras,” PM Modi had said from Red Fort.
Q. India’s first indigenous e-Tractor Prima ET11 is developed by which institution?
A) CSIR B)Tata Motors
C) Ashok Leyland D) Eicher Motors
Answer : A
✓ India’s first indigenously developed e-Tractor CSIR Prima ET11, was launched by the Union Minister
of State (Independent Charge) Science & Technology Dr. Dr.Jitendra Singh.
✓ The e-Tractor CSIR Prima ET11 has been developed by the Central Mechanical Engineering Research
Institute (CMERI), Durgapur, West Bengal.

Q. Khelo India Women's League has been renamed as?


A) Asmita Mahila League B) Kiran Mahila League
C) Beti Mahila League D) Jagrati Mahila League
Answer : A
✓ Anurag Singh Thakur, the Union Minister for Youth Affairs and Sports, has unveiled that the esteemed
Khelo India Women’s League will be recognized as the “Asmita Women’s League” from now on.

Follow us: Official Site, Telegram, Facebook, Instagram, Instamojo 538


✓ This purposeful transformation underscores the government’s commitment to advancing gender equality
and nurturing the vibrant engagement of women in the realm of sports.
✓ ASMITA: Achieving Sports Milestone by Inspiring Women Through Action :
✓ The new name, “Asmita Women’s League,” carries a profound meaning. ASMITA stands for
“Achieving Sports Milestone by Inspiring Women Through Action”, encapsulating the spirit of
resilience, determination, and achievement that defines women athletes across the nation.
Q. Who has been elected as the new 30th Prime Minister of Thailand?
A) Chan-o-cha used B) Shretha Thavisin
C) Yingluck Shinawatra D) Abhisit Vejjajiva
Answer : B
✓ Srettha Thavisin from the populist Pheu Thai party secured enough votes in Parliament to become
Thailand’s 30th Prime Minister.
✓ Earlier, former Prime Minister Thaksin Shinawatra returned from years of self-imposed exile and began
an eight-year prison sentence.
✓ THAILAND
✓ Capital : Bangkok
✓ Currency : Baht
✓ Prime Minister : Shretha Thavisin
Q. How many new countries will join the BRICS alliance from 1 January 2024?
A) 5
B) 6
C) 8
D) 4
Answer : B
✓ 6 Countries to join BRICS from 1 January 2024
✓ In a significant geopolitical development, the BRICS alliance has announced the inclusion of six new
member countries.
✓ These countries are Egypt, Ethiopia, Iran, Argentina, the United Arab Emirates, and Saudi Arabia, and
will join Brazil, Russia, China, India, and South Africa. The membership will take effect from 1 January
2024.
✓ President Cyril Ramaphosa of South Africa, the host nation of the recent BRICS summit.
✓ The BRICS group of nations comprises Brazil, Russia, India, China, and South Africa. It accounts for
more than 40% of the world’s population and about 26% of the global economy.
✓ About BRICS
✓ The full form of BRICS is Brazil, Russia, India, China, and South Africa.
✓ Goldman Sachs economist Jim O'Neill coined the term BRIC (without South Africa) in 2001.
✓ He claimed that by 2050, the four BRIC economies would dominate the global economy by 2050.
✓ South Africa was included in the list in 2010.
✓ The presidency of the Forum is rotated annually among the members.
✓ BRICS accounts for about 40% of the world’s population.
✓ It accounts for 30% of the world's GDP (Gross Domestic Product).
✓ In the year 2014, during the 6th BRICS Summit in Fortaleza, Brazil, the BRICS leaders signed an
agreement to establish a New Development Bank (NDB).

Follow us: Official Site, Telegram, Facebook, Instagram, Instamojo 539


Q. India has partnered with which country for the MeitY- National Science Foundation (NSF) research
collaboration?
A) Japan B) UK
C) USA D) South Korea
Answer : C
✓ The U.S. National Science Foundation (NSF) and the Ministry of Electronics and Information
Technology (MeitY) of the Government of India have signed an Implementation Arrangement on
research cooperation.
✓ Under the agreement, the first Joint Call for proposals has been made by the MeitY, in the areas of
semiconductor research, next generation communication technologies/networks/systems, cyber-
security, sustainability and green technologies and Intelligent Transportation Systems.

Q. Which country is the host of ‘World Water Week 2023’ event?


A) Finland B) Sweden
C) Ireland D) Switzerland
Answer : B
✓ The Director General, National Mission for Clean Ganga (NMCG) recently chaired an online session in
from India, during the World Water Week.
✓ The session was titled ‘Peer Networking for Integrated River Basin Planning and Management’ having a
discussion on the adoption of the river basin management approach.
✓ The World Water Week 2023 (from 20-24 August 2023) is an annual event held online and at Stockholm
(Sweden) with the theme ‘Seeds of Change: Innovative Solutions for a Water-Wise World’.

Q. Which Union Ministry has notified the ‘Green hydrogen standards’ for India?
A) Ministry of External Affairs B) Ministry of Tribal Affairs
C) Ministry of Coal D) Ministry of New and Renewable Energy
Answer : D
✓ The Ministry of New and Renewable Energy notified the green hydrogen standard for India, outlining
the emission thresholds for production of hydrogen that can be classified as ‘green’.
✓ The notification also specifies that the Bureau of Energy Efficiency (BEE) under the ministry of power
will be the nodal authority for accreditation of agencies for the monitoring, verification and certification
for green hydrogen production projects.
Q. Singapore-headquartered BOC Aviation Limited and which airline company have entered into a finance
lease transaction involving 10 Airbus A320NEO aircraft to further expand its fleet?
A) SpiceJet B) Go First
C) Air India D) IndiGo
Answer : D
✓ Singapore-headquartered BOC Aviation Limited and India’s largest airline IndiGo have entered into a
finance lease transaction involving 10 Airbus A320NEO aircraft to further expand its fleet.
✓ All 10 aircraft powered by CFM LEAP-1A engines are scheduled for delivery in 2023, said BOC
Aviation.
✓ India is currently among the fastest-growing civil aviation markets in the world and these aircraft will
help IndiGo consolidate its position in the region.

Follow us: Official Site, Telegram, Facebook, Instagram, Instamojo 540


✓ BOC Aviation is a leading global aircraft operating leasing company with a fleet of 652 aircraft owned,
managed, and on order. Its owned and managed fleet was leased to 91 airlines in 42 countries and regions
worldwide as of June 30, 2023
✓ IndiGo
✓ Founded : 2005
✓ Headquarters : Gurgaon, Haryana
✓ Chairman : Venkataramani Sumantran
✓ MD : Rahul Bhatia
✓ CEO : Pieter Elbers (Netherlands)
✓ Founders : Rahul Bhatia and Rakesh Gangwal
✓ IndiGo is India’s biggest airline by market share
Q. Which of the following company has become the first Indian EV company to start Production in Africa?
A) TVS iQube Electric B) Hero Electric Photon
C) Okinawa Ridge Plus D) One Electric
Answer : D
✓ One Electric Motorcycles announced that the company has started manufacturing its flagship electric
motorcycle “KRIDN” locally in Africa.
✓ The company has entered into a JV with a local established vehicle manufacturing entity.
✓ One Electric has provided complete components, technology, and know-how for the assembly of its
motorcycles.
✓ One Electric also has plans to set up a battery manufacturing plant in Africa by end of the current year
as volumes soar. After consolidation in Africa, the company is already in talks to expand into other
markets like South America and Southeast Asia.
Q. The state of Kerala has introduced its first AI school situated in which city of state?
A) Thrissur B) Kozhikode
C) Thiruvananthapuram D) Kochi
Answer : C
✓ Kerala’s first AI school launched in Thiruvananthapuram
✓ The state of Kerala has introduced its first AI school situated at Santhigiri Vidyabhavan in
Thiruvananthapuram.
✓ It was inaugurated by the Former President of India, Ram Nath Kovind.
✓ The pioneering AI School has been developed in collaboration with iLearning Engines (ILE) USA, a
globally renowned educational platform, and Vedhik eSchool.
Q. Which state has become the first Indian state to give free In Vitro Fertilization (IVF) treatment in
government hospitals?
A) Goa B) Kerala
C) Tamil Nadu D) Karnataka
Answer : A
✓ Goa has become the first Indian state to offer free in vitro fertilization (IVF) treatment.
✓ The inauguration of this service, alongside assisted reproductive technology (ART) and intrauterine
insemination (IUI), was done by Chief Minister Pramod Sawant at the Goa Medical College (GMC) in
Bambolim.

Follow us: Official Site, Telegram, Facebook, Instagram, Instamojo 541


✓ Depending on the hospital and the kind of IVF treatment, the cost of IVF in India comes between Rs
70,000 to Rs 3 lakh per cycle
GOA IN NEWS 2023
▪ Goa has become the first Indian state to offer free in vitro fertilization (IVF) treatment.
▪ GI tags for Goan mangoes and bebinca, crafts from Rajasthan and U.P.
▪ The products are ‘Jalesar Dhatu Shilp’ (a metal craft), ‘Goa Mankurad Mango’, ‘Goan Bebinca’,
‘Udaipur Koftgari Metal Craft’, ‘Bikaner Kashidakari Craft’, ‘Jodhpur Bandhej Craft’, and ‘Bikaner Usta
Kala Craft’.
▪ ‘India’s First Village Atlas’ Is Of Mayem In Goa.
▪ The tourism department of the Goa government is set to hold the ‘Heritage Festival 2023’ from April 28
to 30 at Saligao village in North Goa.
▪ Bharti Institute of Public Policy, a renowned think tank under the Indian School of Business (ISB), and
the Government of Goa have entered into a Memorandum of Understanding (MoU) to establish a
knowledge partnership.
▪ The Comptroller and Auditor General (CAG) of India, Shri Girish Chandra Murmu, holds the position
of Chair of the Supreme Audit Institutions-20 (SAI20) Engagement Group during India’s G20
Presidency. The SAI20 Summit is scheduled to take place in Goa from 12th to 14th June 2023.
▪ Union Cabinet has approved the naming of Greenfield International Airport at Goa’s Mopa as Manohar
International Airport after former Defence Minister and Goa Chief Minister Manohar Parrikar.
▪ Purple, First Inclusion Festival of India starts in Goa.
▪ Goa Manohar International Airport wins best sustainable greenfield airport award.
▪ Government of Goa has launched the Vision for All School Eye Health program.
▪ The first Prime Minister GatiShakti regional workshop for the Western and Central Zone was held in
Goa.
▪ India’s second longest cable-stayed eight-lane Zuari Bridge opens in Goa.
▪ 23rd Commonwealth Law Conference begins in Goa.
▪ Goa Statehood Day 2023 observed on 30th May.
▪ This year Goa celebrates its 36th anniversary of statehood.
Q. The government is planning skill development training for how many women under the ‘Lakhpati Didi’
scheme, which aims to encourage them to start micro-enterprises?
A) Rs 2 crore B) Rs 1 crore
C) Rs 5 crore D) Rs 3 crore
Answer : A
✓ Govt planning skill training for 2 crore women under ‘Lakhpati Didi’ scheme.
✓ The government is planning skill development training for two crore women under the ‘Lakhpati Didi’
scheme that aims to encourage them to start micro-enterprises.
✓ Prime Minister Narendra Modi, in his Independence Day speech, said “It is my dream to make two crore
lakhpati didis in villages”.
✓ The ‘Lakhpati Didi’ scheme has been in place in some states and now the government is planning to train
two crore women under it.
Q. How many Police personnel have been awarded Police Medals on the occasion of the 77th Independence
Day?
A) 787 B) 839
C) 689 D) 954
Answer : D

Follow us: Official Site, Telegram, Facebook, Instagram, Instamojo 542


✓ A total of 954 Police personnel has been awarded Police Medals on the occasion of Independence Day.
✓ The President’s Police Medal for Gallantry has been awarded to CRPF personnel and the Police Medal
for Gallantry has been awarded to 229 personnel.
✓ President’s Police Medal for Distinguished Service has been awarded to 82 and the Police Medal for
Meritorious Service to 642 personnel.
✓ 125 personnel from Left Wing Extremism affected areas, 71 personnel from Jammu and Kashmir region
and 11 personnel from North East region are being awarded for their gallant action.
✓ Among the majority of the 230 Gallantry Awards, 125 personnel from Left Wing Extremism affected
areas, 71 personnel from Jammu & Kashmir region and 11 personnel from North East region are being
awarded for their gallant action.
✓ Among the personnel receiving Gallantry Awards, 28 are from CRPF, 33 are from Maharashtra, 55 are
from J&K Police, 24 are from Chhattisgarh, 22 are from Telangana and 18 are from Andhra Pradesh the
remaining from the other States/UTs and CAPFs.
Q. How many personnel have been awarded Fire Service Medals on the occasion of Independence Day
2023?
A) 22 B) 47
C) 53 D) 36
Answer : C
✓ A total of 53 personnel has been awarded Fire Service Medals on the occasion of Independence Day
2023.
✓ Out of these, President’s Fire Service Medal for Gallantry is awarded to 03 personnel and Fire Service
Medal for Gallantry is awarded to 01 personnel for their respective acts of valour and gallantry.
✓ President’s Fire Service Medal for Distinguished Service is awarded to 8 personnel and Fire Service
Medal for Meritorious Service is awarded to 41 personnel for their respective distinguished and
meritorious records of services.
✓ In addition, 48 personnel/volunteers are also awarded Home Guards & Civil Defence Medals on the
occasion of Independence Day, 2023.
Q. Union Cabinet has approved how many multi-tracking projects targeted at Indian railways with an outlay
of Rs 32, 500 crore?
A) 6 B) 8
C) 9 D) 7
Answer : D
✓ The Cabinet, chaired by Prime Minister Narendra Modi, has approved seven multi-tracking projects
targeted at Indian railways with an outlay of Rs 32, 500 crore.
✓ The projects will be built completely on Engineering, procurement, and construction (EPC) mode, and
will add 2,339 kilometres to the existing network of Indian Railways.
✓ The projects will receive 100 percent funding from the central government. They will cover 35 districts
across Andhra Pradesh, Bihar, Gujarat, Jharkhand, Maharashtra, Odisha, Telangana, Uttar Pradesh,
and West Bengal
Q. Which state government has launched the Annapurna Food Packet Scheme, which aims to distribute
food packets free of cost to over 10.4 million families every month?
A) Odisha B) Bihar
C) Rajasthan D) Chhattisgarh
Answer : C

Follow us: Official Site, Telegram, Facebook, Instagram, Instamojo 543


✓ Rajasthan Chief Minister Ashok Gehlot has launched the state government’s Annapurna Food Packet
Scheme which seeks to distribute food packets to more than 10.4 million families for free every month.
✓ The scheme is part of the ₹19,000 crore ‘Inflation Relief Package’, which was announced by the chief
minister in February during the state budget for 2023-24 to provide relief to the public from rising prices.
✓ Under the scheme, each food packet will contain one kilogram each of chana dal, sugar, salt, one litre
soybean refined edible oil, 100 gm each of chilli powder and coriander powder, and 50 gm of turmeric
✓ The chief minister also urged the Centre to regularly implement the Pradhan Mantri Garib Kalyan Anna
Yojana – providing 5kg free dry rations to 800 million poor every month – instead of extending it for six
months each time.

Q. Who has launched the One District One Product 'ODOP Wall'?
A) Piyush Goyal B) Charanjit Singh
C) Som Parkash D) Sunil Barthwal
Answer : B
✓ One District One Product (ODOP) and Deendayal Antyodaya Yojna - National Rural Livelihoods
Mission (DAY-NRLM) joined hands to launch ‘ODOP wall’.
✓ The One District One Product (ODOP) program, an initiative under the Department for Promotion of
Industry and Internal Trade (DPIIT), Ministry of Commerce & Industry, aims at manifesting the vision
of the Prime Minister to make the country and its people self-reliant by fostering balanced regional
development across all districts of the country.
✓ Launching the 'ODOP Wall', Shri Charanjit Singh.
Q. Who has laid the foundation stone of Sant Ravidas Temple in Madhya Pradesh?
A) Narendra Modi B) Amit Shah
C) J. P. Nadda D) Rajnath Singh
Answer : A
✓ Prime Minister Narendra Modi laid the foundation stone of Sant Shiromani Gurudev Shri Ravidas ji
Memorial that will be constructed in more than ₹100 crors at Badtuma village of Sagar district in Madhya
Pradesh.
✓ The memorial is steeped in the spirit of ‘Samrasta’ as the soil of more than 20,000 villages and 300 rivers
have been used in it.
✓ Families from Madhya Pradesh have sent grain for ‘Samrast Bhoj’ and five yatras also concluded here at
Sagar.
✓ These yatras mark a new era of social harmony.
Q. Which state government has launched its flagship scheme "Gramin Mitra" to provide doorstep delivery
of government services across the state?
A) Goa
B) Assam
C) Odisha
D) Kerala
Answer : A
✓ The government of Goa launched its flagship scheme "Gramin Mitra" to provide doorstep delivery of
government services across the state and promote digital empowerment in rural areas.

Follow us: Official Site, Telegram, Facebook, Instagram, Instamojo 544


Q. Which airport becomes first in northeast to launch 'Digi Yatra' facility?
A) Mumbai, Chhatrapati Shivaji Maharaj International Airport
B) Delhi, Indira Gandhi International Airport
C) Kolkata, Netaji Subhash Chandra Bose International Airport
D) Guwahati, Lokpriya Gopinath Bordoloi International Airport
Answer : D
✓ Guwahati's Lokpriya Gopinath Bordoloi International Airport became the first airport in the northeast
to get ‘Digi Yatra’ facility.
✓ The Digi Yatra service will be available in three major areas of the airport, namely - first entrance, check-
in and boarding areas.
✓ What is Digi Yatra initiative?
✓ According to a release from the Ministry of Civil Aviation, Digi Yatra uses facial recognition technology
to provide a seamless and hassle-free experience to passengers at airports. The aim of Digi Yatra facility
is to eliminate the need for verification of tickets and IDs at various touch points.

Q. Which country has signed an agreement with the Indian government for sharing the India Stack?
A) Trinidad and Tobago B) Nigeria
C) Morocco D) Senegal
Answer : A
✓ India signs MoU with Trinidad and Tobago on sharing INDIA STACK
✓ Trinidad and Tobago has signed an agreement with the Indian government for sharing the India Stack,
which is a collection of open application programming interfaces and digital public goods for identity.
✓ The agreement was signed in the presence of officials from the Ministry of Electronics and IT (MeitY),
the National E-Governance Division, and the Ministry of External Affairs.
✓ A similar agreement was signed with Papua New Guinea also last month, while the UPI — part of India
Stack — has been accepted in France, UAE, Singapore and Sri Lanka.
✓ Recently, the Ministry of Electronics and IT (MeitY) and the Ministry of Information and
Communication Technology (MICT) of Papua New Guinea signed a memorandum of understanding
(MoU) for sharing India Stack.
✓ Trinidad and Tobago
✓ Capital : Port of Spain
✓ Currency : Trinidad and Tobago dollar (TTD)
✓ President : Christine Kangaloo
Q. In which city did Power Minister, RK Singh dedicate to the nation the 660 MW unit of NTPC’s Super
Thermal Power Project?
A) Darlipali B) Barh
C) Barauni D) Dadri
Answer : B
✓ Union Minister for Power and New and Renewable Energy RK Singh has dedicated to the nation a 660
MW unit of NTPC’s Barh Super Thermal Power Project in Barh, Bihar.
✓ The 660 MW unit being inaugurated is Unit 2 of Stage I of the project. The commissioning of this unit
would be another milestone in the government’s endeavour to provide reliable and affordable power to
the nation.

Follow us: Official Site, Telegram, Facebook, Instagram, Instamojo 545


✓ The Union Minister for Power and New & Renewable Energy also lay the foundation stone for the
extension of POWERGRID’s (Power Grid Corporation of India Limited) 400/132 kV Lakhisarai Sub-
station.
Q. Which state government has launched the Bhagwan Birsa Munda Road Scheme which aims to enhance
road connectivity in tribal areas of the 17 districts of the state?
A) Chhattisgarh B) Jharkhand
C) Madhya Pradesh D) Maharashtra
Answer : D
✓ Maharashtra government has launched the Bhagwan Birsa Munda Road Scheme which aims to enhance
road connectivity in tribal areas of the 17 districts of the state.
✓ The scheme aims to connect remote tribal areas to the main roads.
✓ Maharashtra cabinet also decided to repeal the 45-year-old Maharashtra Casinos (Control and Tax) Act,
1976, announcing that the state will not have casinos in the future.
✓ The State Cabinet has decided to give special rations during the upcoming Ganeshotsav and Diwali
festivals under the Antyodaya scheme
Q. Who won the gold medal in the women's 76kg freestyle at the U20 World Wrestling Championships
2023 in Jordan?
A) Sakshi Malik B) Vinesh Phogat
C) Priya Mallik D) Sonam Malik
Answer : C
✓ Indian grappler Priya Mallik won a Gold medal at the 2023 U20 World Wrestling Championships in
Jordan.
✓ Priya became just the second woman wrestler from India to be crowned U20 world championship. Antim
Panghal had been the first woman from India to win a junior world title in wrestling at last year’s edition.
✓ Priya Mallik beat Laura Celine Kuehn of Germany by 5-0 in the women’s 76kg freestyle wrestling final.
Q. Which district of Uttar Pradesh has become the first district in the state to adopt e-office?
A) Kannauj B) Mathura
C) Aligarh D) Gorakhpur
Answer : A
✓ UP’s Kannauj has become the first district in the state to adopt e-office – the new digital gateway that
cuts on manual handling of files and documents.
✓ The e-office is a Mission Mode Project (MMP) under the National e-Governance Plan.
✓ E-office is a workflow-based system that extends features of existing manual handling of files in addition
to the more efficient electronic system.
✓ The efficiency of e-office not only helps departments in taking quick decisions but also makes them
paperless.
Q. How many Indian youths have been named among the International Young Eco-Hero Award 2023 to
tackle the world’s most pressing environmental challenges?
A) 4 B) 5
C) 6 D) 8
Answer : B

Follow us: Official Site, Telegram, Facebook, Instagram, Instamojo 546


✓ Five Indian youths named among International Young Eco-Hero award 2023.
✓ Five youths from India have been named among 17 teen environmental activists from across the globe
to receive the 2023 International Young Eco-Hero Award who have taken initiatives to tackle the world’s
most pressing environmental challenges.
✓ The young eco-warriors who were recognized for their efforts by the US-based non-profit organization,
“Action For Nature”, are Eiha Dixit, Manya Harsha, Nirvaan Somany, Mannat Kaur, and Karnav
Rastogi.
✓ The International Young Eco-Hero Awards programme recognizes and encourages children and teens
from ages 8 to 16 who have taken action towards solving the most critical environmental issues.
✓ The awardees are eco-conscious youth who took crucial steps to solve tough environmental problems.
✓ Winners of the International Young Eco-Hero Award are selected by a panel of independent judges,
including experts in environmental science, biology, and education.
Q. Which Company has inked to supply 300 MW solar power for 25 years to Rajasthan?
A) NLC India Limited B) NTPC Limited
C) Coal India D) Power Grid Corporation of India
Answer : A
✓ State-owned NLC India (NLCIL) has inked a pact to supply 300 MW of solar power for 25 years to
Rajasthan Urja Vikas Nigam under the CPSU scheme.
✓ NLCIL is presently having 1,421 MW of Renewable Energy Capacity. As per its corporate plan, it
contemplates establishing 6,031 MW capacity by 2030, a statement said.
✓ NLC India has entered into a long-term power usage agreement with Rajasthan Urja Vikas Nigam
Limited to supply 300 MW of solar power under the CPSU Scheme in Rajasthan, it added.
✓ The company has secured 510 MW solar project capacity in the CPSU Scheme Phase-II Tranche-III,
floated by Indian Renewable Energy Development Agency (IREDA) through competitive bidding.
✓ The 300 MW solar project capacity is under execution at Barsingsar, Bikaner District, Rajasthan.

Q. Which state Government has launched the India's First-Ever Night Street Racing Circuit?
A) Karnataka B) Kerala
C) Tamil Nadu D) Andhra Pradesh
Answer : C
✓ A new street circuit, which will also hold night races, was launched by the Tamil Nadu government.
✓ The state government has partnered with Racing Promotions Private Limited (RRPL) for the project.
RRPL is also the promoter of the Indian Racing League and F4 Indian Championship.
Q. Which state government has launched India’s first Agricultural Data Exchange (ADeX) and Agriculture
Data Management Framework (ADMF)?
A) Himachal Pradesh B) Telangana
C) Rajasthan D) Uttar Pradesh
Answer : B
✓ Telangana has launched India’s first Agricultural Data Exchange (ADeX) and Agriculture Data
Management Framework (ADMF).
✓ It is developed as a digital public infrastructure (DPI) for the agriculture sector. The ADeX is a
collaboration between the Telangana Government, the World Economic Forum, and the Indian Institute
of Science.

Follow us: Official Site, Telegram, Facebook, Instagram, Instamojo 547


Q. Which state government has recently approved the disbursement of 6,000 rupees to eligible farmers under
the Chief Minister’s Kisan Kalyan Yojana for the financial year 2023-24?
A) Uttarakhand B) Bihar
C) Chhattisgarh D) Madhya Pradesh
Answer : D
✓ Madhya Pradesh Cabinet has approved the disbursement of 6,000 rupees to eligible farmers under the
Chief Minister’s Kisan Kalyan Yojana for the financial year 2023-2024.
✓ The decision was taken in a cabinet meeting chaired by Chief Minister Shivraj Singh Chouhan in Bhopal.
✓ The Cabinet also decided to give the benefit of the seventh pay scale to Gram Panchayat Secretaries. An
additional expenditure of 178.88 crore rupees on it will be met from minor mineral heads.
✓ The council of ministers further approved the construction of 53 CM Rise Schools and 19 Kanya Shiksha
Parisar with a total cost of over 2,491 crore rupees.

Q. Global Maritime India Summit (GMIS) 2023 will be held in which city from 17 to 19 October?
A) Chennai B) Hyderabad
C) New Delhi D) Mumbai
Answer : C
✓ Global Maritime India Summit (GMIS), 2023 is slated for October 17 to October 19 at Pragati Maidan,
New Delhi.
✓ The meeting, with participation from more than 40 countries, including 21 Ambassadors, High
Commissioners, Deputy High Commissioners, Trade Commissioners, and 23 other diplomats,
underscored the global interest in the upcoming GMIS 2023.
✓ GMIS 2023 is a premier maritime sector focussed event to bring together pivotal figures from the industry
to explore opportunities, understand challenges, and stimulate investment within India’s maritime sector.
Q. Who has laid the foundation stone for the Indian Farmers Fertilizer Cooperative Limited Nano Urea
Plant at Gandhidham in Kutch?
A) Piyush Goyal B) Dharmendra Pradhan
C) Rajnath Singh D) Amit Shah
Answer : D
✓ Union Home Minister Amit Shah has laid the foundation stone for the Indian Farmers Fertilizer
Cooperative Limited Nano Urea Plant at Gandhidham in Kutch, Gujarat.
✓ India needs another green revolution in the form of natural farming under the leadership of Prime
Minister Narendra Modi which aims to ensure the highest value addition to farmers and protect
farmlands from degradation.
✓ The nano urea plant will provide multi-dimensional benefits to the agricultural economy. It will also help
conserve the nutrients of the soil and ensure judicious use of fertilizers.

Q. Which state government has launched the ‘Sinh Suchna’ Web App on the occasion of World Lion Day?
A) Madhya Pradesh B) Gujarat
C) Maharashtra D) Tamil Nadu
Answer : B
✓ On the occasion of World Lion Day, Gujarat CM Bhupendra Patel has virtually launched Lion Anthem’
and unveiled the ‘Sinh Suchna’ Web App.

Follow us: Official Site, Telegram, Facebook, Instagram, Instamojo 548


✓ The application will help the state forest department track the movement of lions through the information
shared by common people using the app.
✓ Using the ‘Sinh Suchna’ app, common people can directly inform the state forest department about the
movement of lions in their area so that timely steps can be taken.
✓ This mobile will help us in locating lions in real-time and also help the authorities in avoiding human-
animal conflicts,” Patel said after launching the app.
✓ The central government under Prime Minister Narendra Modi has approved a budget of Rs 2,900 crore
under ‘Project Lion’ for the conservation of Asiatic Lions, which are found only in Gujarat.
Q. India has won the fourth Asian Champions Trophy title by defeating which country at the Mayor
Radhakrishnan Stadium in Chennai?
A) Malaysia B) Indonesia
C) Pakistan D) Thailand
Answer : A
✓ India beat Malaysia 4-3 and clinch a record fourth Asian Champions Trophy title at the Mayor
Radhakrishnan Stadium in Chennai.
✓ With four Asian Champions Trophy titles, India is the most successful team in the history of the
tournament.
✓ Pakistan, with three titles, is in the second spot now.
✓ Japan take 3rd spot after beating South Korea 5-3. Japan finished behind Malaysia in the Asian
Champions Trophy tournament after the beat defending champion South Korea 5-3 in the third-place
playoff tie in Chennai.
Q. Partition Horrors Remembrance Day’ or ‘Vibhajan Vibhishika Smriti Diwas’ is observed every year on
which day?
A) 13 August B) 12 August
C) 14 August D) 11 August
Answer : C
✓ Partition Horrors Remembrance Day’ or ‘Vibhajan Vibhishika Smriti Diwas’ is observed on August 14
to remember all those who lost their lives due to partition.
✓ This day was announced by the Prime Minister in 2021 to remember all those who lost their lives due to
the partition of the nation and were uprooted from their origins.
✓ The partition caused one of the largest migrations in human history, affecting about 20 million people.
Millions of families had to abandon their ancestral villages/towns/cities and were forced to start anew
as refugees.

Q. World Organ Donation Day is observed every year on which day?


A) 13 August B) 12 August
C) 10 August D) 14 August
Answer : A
✓ World Organ Donation Day is observed every year on 13 August.
✓ Objective – To raise awareness about the importance of and help people understand organ donation.
✓ Theme 2023 – Donate Organs, Save Lives

Follow us: Official Site, Telegram, Facebook, Instagram, Instamojo 549


. Q. Which state government has launched the ‘Mo Jungle Jami Yojana’ for indigenous people of the state to
ensure individual and community rights over forest land for eligible beneficiaries?
A) Odisha B) Haryana
C) Karnataka D) Telangana
Answer : A
✓ Odisha Chief Minister Naveen Patnaik has launched the ‘Mo Jungle Jami Yojana’ for indigenous people
of the state to ensure individual and community rights over forest land for eligible beneficiaries.
✓ It was launched on the International Day of the World’s Indigenous Peoples.
✓ All welfare activities of the government will be linked to the scheme to create stability in the livelihoods
of tribal people.
Q. Who has been recognized with the Lifetime Achievement Award at the first-ever RICS South Asia
Awards?
A) Subhash Runwal B) Radhakishan Damani
C) Bhavish Aggarwal D) Karsanbhai Patel
Answer : A
✓ Subhash Runwal, Chairman – Runwal, a leading real estate developer with a legacy of over four decades,
was recognized with the Lifetime Achievement Award at the first-ever RICS South Asia Awards.
✓ RICS (Royal Institution of Chartered Surveyors) is a global industry body representing professionals
across the country.
✓ This recognition is a testimony to Runwal’s exceptional contributions, outstanding achievements, and
unwavering dedication to the real estate sector

Q. Who has been awarded the Tech Startup of the Year (AI) at the 13th Annual Entrepreneur India Awards?
A) Cred B) Jio Haptik
C) Dream 11 D) Bharti Airtel
Answer : B
✓ Jio Haptik has been honored with the Tech Startup of the Year (Artificial Intelligence) award at the
prestigious 13th Annual Entrepreneur India Awards.
✓ Swapan Rajdev, Haptik’s co-founder & CTO, received the award as the ceremony organized by
Entrepreneur India, in media partnership with Entrepreneur magazine and Entrepreneur com, was held
at the Hotel JW Marriott, Aerocity, New Delhi.

Q. What is the aim of the National Social Assistance Programme?


A) To provide financial assistance to women
B) Old-age pension to very poor
C) To provide education and training to the weaker sections of the society
D) To provide financial security to the vulnerable sections of society
Answer : B
✓ Recently, the Comptroller and Auditor General of India submitted the report on the performance audit
of the National Social Assistance Programme from 2017-18 to 2020-21.

Follow us: Official Site, Telegram, Facebook, Instagram, Instamojo 550


✓ About National Social Assistance Programme:
✓ It was launched on 15th August, 1995.
✓ Objective
✓ It is is a social security and welfare programme to provide support to aged persons, widows, disabled
persons and bereaved families on death of primary bread winner,
✓ belonging to below poverty line households.
✓ Eligibility
✓ For getting benefits under NSAP the applicant must belong to a Below Poverty Line (BPL) family
according to the criteria prescribed by the Govt. of India.
✓ The other eligibility criteria and the scale of central assistance under the sub - schemes of NSAP are as
follows.
✓ Presently NSAP comprises of five schemes, namely –
✓ Indira Gandhi National Old Age Pension Scheme (IGNOAPS): The eligible age for IGNOAPS is 60
years. The pension is Rs.200 p.m. for persons between 60 years and 79 years. For persons who are 80
years and above the pension is Rs.500/ - per month.
✓ Indira Gandhi National Widow Pension Scheme (IGNWPS): Widow aged 40 years will get pension is
Rs.300 per month. After attaining the age of 80 years, the beneficiary will get Rs.500/ - per month.
✓ National Family Benefit Scheme NFBS) : In the event of death of the bread – winner Rs. 20000/ - will
be given as a lumpsum assistance to the bereaved household. The breadwinner should have been between
18-60 years of age.
✓ Annapurna: The scheme aims at provide food security to meet the requirements of those eligible old aged
persons who have remained uncovered under the IGNOAPS. Under this programme 10 kgs of food
grains (wheat or rice) is given per month per beneficiary.
✓ Nodal Ministry: Ministry of Rural Development

Q. Which day of the year has been dedicated to be celebrated as the International Youth Day annually?
A) 10 August B) 11 August
C) 12 August D) 13 August
Answer : C
✓ International Youth Day (IYD) is observed on August 12 annually to draw attention of governments and
others towards youth issues worldwide.
✓ 2023 Theme : Green Skills for Youth: Towards a Sustainable World
✓ The day was designated by the United Nations in 1999.
✓ The first IYD was observed on 12 August, 2000.
✓ National Youth Day is celebrated in India every year on 12 January to commemorate the birth
anniversary of Swami Vivekananda. The day was declared by the Government of India in 1984 while
the event was first celebrated in 1985.

Q. When is the World Elephant Day observed globally?


A) 13 August B) 11 August
C) 10 August D) 12 August
Answer : D
✓ World Elephant Day is observed every year on August 12 globally for the preservation and protection of
the global elephants.
✓ The main objective of World Elephant Day is to create awareness on elephant conservation, and to share
knowledge and positive solutions for the better protection and management of wild and captive
elephants.
✓ The day is being officially celebrated since 2012.

Follow us: Official Site, Telegram, Facebook, Instagram, Instamojo 551


Q. What is the theme of the 2022 International Youth Day (IYD)?
A) Intergenerational Solidarity: Creating a World for All Ages
B) Transforming Food Systems: Youth Innovation for Human and Planetary Health
C) Youth Engagement for Global Action
D) Green Skills for Youth: Towards a Sustainable World
Answer : D
✓ The theme for International Youth Day in 2023 is 'Green Skills for Youth: Towards a Sustainable World'
Q. IIT-Roorkee and Geological Survey of India (GSI) have discovered the oldest fossil remains of
dicraeosaurid dinosaur in which city?
A) Jodhpur B) Jaisalmer
C) Hanumangar D) Dungarpur
Answer : B
✓ Recently, scientists from IIT-Roorkee and Geological Survey of India (GSI) have discovered the oldest
fossil remains of dicraeosaurid dinosaur in Jaisalmer.
✓ About Dicraeosaurid dinosaur.
✓ It is a long-necked, plant-eating dinosaur species.
✓ The remains are 167 million years old and belong to a new species, unknown to scientists thus far.
✓ It has been named ‘Tharosaurus indicus’, the first name referring to the ‘Thar desert’ where the fossils
were found, and the second after its country of origin.
✓ According to the scientists, fossils of dicraeosaurid dinosaurs have been found previously in North and
South Americas, Africa and China, but such fossils were not known from India.
Q. The International Day of the World’s Indigenous Peoples, also called as World Tribal Day, is observed
on which day?
A) 8 August B) 9 August
C) 7 August D) 6 August
Answer : B
✓ The International Day of the World’s Indigenous Peoples (World Tribal Day) is observed on 9 August
annually to protect the rights of the world’s indigenous population and recognize the achievements and
contributions that indigenous people make to improve world issues such as environmental protection.
✓ The day was recognised by the General Assembly of the United Nations in December 1994.
✓ The date to marks the day of the inaugural session of the Working Group on Indigenous Populations at
the United Nations in 1982.
✓ The theme picked for World Tribal Day 2023 is "Indigenous Youth as Agents of Change for Self-
determination."

Q. Which nationwide campaign has been started in honor of the martyred brave soldiers of the country?
A) Desh ke Veer Saput B) Desh ke Veer Jawan
C) Meri Mati Mera Desh D) Mera Bharat Mahan
Answer : C
✓ Nation -wide “Meri Mati Mera Desh” Campaign will be launched on 9th August 2023, to pay tribute to
the ‘Veers’ who laid down their lives for the country.

Follow us: Official Site, Telegram, Facebook, Instagram, Instamojo 552


✓ From 9th to 30th August , 2023, the 'Meri Maati Mera Desh' campaign will include programs at the
village, and block levels , by local urban bodies, as well as state and national level.
✓ The campaign will include various programs across the country to remember the bravehearts (Veers).

Q. World Trade Organization (WTO) 2023, where does India rank in world merchandise trade (2022)?
A) 20th B) 18th
C) 15th D) 12th
Answer : B
✓ According to the World Trade Statistical Review 2023, the annual flagship publication of the World
Trade Organization (WTO), India is ranked 18th among the Leading exporters in world merchandise
trade (2022).
✓ It gives data on merchandise and services trade broken down by geographical origin, main product groups
and sectors, along with related data on key economic developments such as gross domestic product
(GDP) growth, commodity prices, and exchange rate fluctuations, mostly for 2022.
✓ On a global scale, India holds the 9th spot in merchandise importing and ranks 7th in merchandise
exporting (2022).
✓ The leading exporters in world merchandise trade for the year 2022 are China, the USA, and Germany,
occupying the top three positions.

Q. Which of the following has recently joined the INTERPOL Global Academy Network?
A) Intelligence Bureau Academy
B) Central Bureau of Investigation Academy
C) Research and Analysis Wing Academy
D) Enforcement Directorate Academy
Answer : B
✓ Recently, the Central Bureau of Investigation (CBI) Academy joined the Interpol Global Academy
Network at a virtual event.
✓ About Interpol Global Academy Network.
✓ It was launched in 2019 with the aim of supporting Interpol in leading a global approach to Law
Enforcement Training.
✓ It is a network of trusted law enforcement education institutions, both regional and national in scope.
✓ Members of the Global Academy support the creation and delivery of select digital and face-to-face
training courses, covering INTERPOL’s tools and services, crime areas, and other law enforcement
topics.
✓ They also help identify and coordinate responses to training needs and contribute to a knowledge
exchange platform to advance law enforcement education through cutting-edge training tools and
methodologies.
✓ The CBI Academy had become the 10th member of Interpol Global Academy Network.

Q. The World Biofuel Day is celebrated all over the globe on which day?
A) 8 August B) 10 August
C) 9 August D) 11 August
Answer : B

Follow us: Official Site, Telegram, Facebook, Instagram, Instamojo 553


✓ The International Biofuel Day (World Biofuel Day) is observed every year on August 10 to raise
awareness about the importance of non-fossil fuels as an alternative to conventional fossil fuels and
highlight the various efforts made by Government in the biofuel sector.
✓ The Day is being observed by the Ministry of Petroleum & Natural Gas since 2015.

Q. Which day of the year is observed as Nagasaki Day?


A) 6 August B) 7 August
C) 8 August D) 9 August
Answer : D
✓ In 2023, the world is observing the 78th anniversary of the Nagasaki bombing that took place on August
9, 1945.
✓ United States dropped the atomic bomb on Nagasaki, Japan and was codenamed “Fat Man” due to the
design of the bomb as it had a wide, round shape.
✓ The atomic bombing of Nagasaki and Hiroshima (August 6) forced Japan to surrender, thus ending
World War II.

Q. August 10 is dedicated to observing which of these days?


A) World Elephant Day B) World Lion Day
C) World Tiger Day D) World Fish Day
Answer : B
✓ World Lion Day is observed globally on August 10 every year with the following objective:
✓ raise awareness of lion and the issues that the species faces in the wild.
✓ find ways to protect the big cat’s natural environment
✓ educate people who live near wild cats on the dangers and how to protect themselves
✓ The World Lion Day was launched in 2013.
✓ The big cat is listed as an endangered species in the IUCN Red List.

Q. Who will lead the Indian team in the World Athletics Championships?
A) Neeraj Chopra B) Hima Das
C) Jeswin Aldrin D) Tajinderpal Singh Toor
Answer : A
✓ Neeraj Chopra, will lead the country’s challenge at the World Athletics Championships from August 19
to 27, 2023 in Budapest.
✓ India will send a 28-member contingent to the prestigious event.

Q. Which State bagged the Best Performing State award 2023 in cadaver organ donation?
A) Kerala B) Karnataka
C) Telangana D) Tamil Nadu
Answer : D
✓ Tamil Nadu bagged the Best Performing State award 2023 in cadaver organ donation .
✓ Tamil Nadu leads all Indian states in organ donations with 1,705 people donating their organs and saving
6,247 human lives.

Follow us: Official Site, Telegram, Facebook, Instagram, Instamojo 554


Q. Cabinet Secretary "Rajiv Gauba" has been granted extension for how many years by the central
government?
A) 1 Year B) 2 Years
C) 3 Years D) 5 Years
Answer : A
✓ The Central government relaxed key rules to give a one-year extension to Cabinet Secretary Rajiv Gauba
beyond August 30, 2023, his extension was extended, as per a government order.
✓ With this extension, Rajiv Gauba now becomes the longest-serving cabinet secretary.
✓ Before Rajiv Gauba, BD Pande had been the longestserving cabinet secretary from November 2, 1972,
to March 31, 1977

FAMOUS SECRETARY IN NEWS


▪ Sanjay Malhotra : Revenue Secretary
▪ Aramane Giridhar : Defence Secretary
▪ Rajiv Gauba - Cabinet Secretary
▪ Vivek Kumar - Private Secretary to PM
▪ Vinay Mohan Kwatra - Foreign secretary
▪ Tarun Kapoor - Advisor to PM
▪ T V Somanathan - Finance Secretary.
▪ Ajay Seth - Economic Affairs Secretary.
▪ TV Somanathan - Expenditure Secretary.
Q. What is the title of the short film made by the Sports Authority of India on the athletes participating in
the Asian Games?
A) Khelo India B) Halla Bol
C) Chak De India D) Namaste India
Answer : B
✓ Sports Authority of India (SAI) under the Umbrella Campaign ‘Cheer4India’ has launched a short movie
series ‘Halla Bol’ on the journey of Asian Games bound athletes to motivate them for Hangzhou Asian
Games and spread awareness about the upcoming Asian Games.
✓ The Sports Authority of India (SAI) is the Apex National Sports Body of India, established in 1984 by
the Ministry of Youth Affairs and Sports of the Government of India for the development of sports in
India
Q. Which State Cabinet approved the proposal of a high-level committee to form 19 new districts and 3 new
divisions in the State?
A) Rajasthan B) Maharashtra
C) Madhya Pradesh D) Uttar Pradesh
Answer : A
✓ The Rajasthan government has greenlit the formation of 19 new districts and 3 new divisions. The
proposal put forward by a high-level committee was approved by the Ashok Gehlot-led Rajasthan
cabinet.
✓ The western state will now have 50 districts - up from 33 - and 10 divisions.
✓ Jaipur district has been divided into Jaipur and Jaipur Rural, and Jodhpur district into Jodhpur and
Jodhpur Rural.

Follow us: Official Site, Telegram, Facebook, Instagram, Instamojo 555


✓ The other new districts are Apoongarh, Balotra, Beawar, Deeg, Didwana-Kuchaman, Dudu, Gangapur
City, Kekri, Kotputli-Behror, Khairthal-Tijara, Neem ka Thana, Phalodi, Slumber, Sanchore and
Shahpura.
✓ The new divisions are Banswara, Pali and Sikar.

Q. When is National Javelin Day celebrated annually?


A) August 5 B) August 6
C) August 7 D) August 8
Answer : C
✓ August 7 is celebrated as National Javelin Day in India. The Athletics Federation of India, the apex body
for the athletics in India, took the decision to celebrate August 7 as the National Javelin Day to
commemorate Neeraj Chopra's historic gold medal during the Tokyo Olympics.
✓ It was on August 7 2021, that Chopra hurled javelin at a distance of 87.58m during the men's Javelin
Throw final of the Tokyo Games to win the gold medal.
✓ Chopra scripted history as it was India's first-ever gold medal in Track and Field events in Olympics.
With the gold medal at Tokyo, Chopra also became the second individual gold medalist from India after
Abhinav Bindra's gold-medal winning performance at the 2008 Beijing Games.
✓ India observes 3rd ‘Javelin Throw Day’ on August 07, 2023

Q. Which State has introduced the 'Shubhayatra' scheme to support and facilitate Overseas Emigrants?
A) Goa B) Kerala
C) Uttar Pradesh D) Uttarakhand
Answer : B
✓ State government of kerala has launched a groundbreaking scheme called ‘Shubhayatra’. The scheme
aims to provide much-needed financial support to first-time overseas emigrants from Kerala, facilitating
a positive and productive migration ecosystem.
✓ In support of the ‘Shubhayatra’ scheme, the Kerala government has allocated ₹2 crore in the current
financial year for providing tax holidays and interest subvention to emigrants
Q. Which state government will set up a ‘Rhino Task Force’ to propose measures to restart the rhino
conservation plan under the National Rhino Conservation Plan?
A) Assam B) Bihar
C) Odisha D) Jharkhand
Answer : B
✓ The Bihar government will set up a ‘Rhino Task Force’ to propose measures to restart the rhino
conservation plan in ‘Valmiki Tiger Reserve’ (VTR) located in West Champaran district.
✓ At present there is only one rhino in VTR and 14 rhinos in Patna Zoo.
✓ The VTR has seen a significant increase in the tiger population, prompting state wildlife officials to focus
on reviving the rhino population in the region.
✓ Valmiki Tiger Reserve has been selected as a potential site under the National Rhino Conservation Plan.
✓ This national-level scheme allows rhinos to be introduced into Valmiki Tiger Reserve from other reserves
across the country to boost their conservation.
✓ India plays a critical role in the conservation of the one-horned rhinoceros species. Approximately 75%
of the world’s rhino population is found in India.
✓ Notably, over 93% of India’s rhino population resides in just one protected area, the Kaziranga National
Park in the state of Assam

Follow us: Official Site, Telegram, Facebook, Instagram, Instamojo 556


Q. Which ministry has implemented the ‘Naya Savera’ scheme also known as the ‘Free Coaching and
Allied’ scheme recently?
A) Ministry of Panchayati Raj B) Ministry of Food Processing Industries
C) Ministry of Earth Sciences D) Ministry of Minority Affairs
Answer : D
✓ The Ministry of Minority Affairs implemented the ‘Naya Savera’ scheme also known as the ‘Free
Coaching and Allied’ scheme.
✓ Objective – To assist students/candidates belonging to the six minority communities namely Sikh, Jain,
Muslim, Christian, Buddhist, and Parsi by way of special coaching for qualifying examinations.
✓ The coaching period under the scheme ranged from 3 months to 2 years.
✓ Preparing for qualifying examinations for admission into technical/professional courses like engineering,
medical, law, management, information technology, etc., and language/aptitude examinations required
for seeking admission to foreign universities.
Q. Which company has handed over the first Radio Frequency (RF) Seeker of Akash – the Next Generation
Weapon System to Defence Research and Development Organization (DRDO)?
A) ICOMM Tele Limited B) Hindustan Aeronautics Ltd (HAL)
C) Bharat Dynamics Ltd (BDL) D) Antrix Corporation Ltd (ACL)
Answer : C
✓ The Bharat Dynamics Limited (BDL) has handed over the first Radio Frequency (RF) Seeker of Akash
– the Next Generation Weapon System to Defence Research and Development Organization (DRDO).
✓ Seeker is a critical and technology-intensive subsystem used in Surface-to-Air Missiles and Air-to-Air
Missiles for target tracking in the terminal phase.
✓ The RF Seeker has been designed by DRDO’s Research Centre Imarat, and produced by BDL at its state-
of-the-art Seeker Facility Centre set up at BDL, Kanchanbagh Unit in Hyderabad.
✓ The establishment of the Seeker Facility Centre at BDL has enabled India to achieve self-reliance in the
field of RF seeker production and contribute greatly towards realizing the goal of Atmanirbhar Bharat.
✓ Chairman and Managing Director of BDL – A Madhavarao
Q. Who is the author of a book titled ‘Monsoon’?
A) Abhay K
B) Vijay Tendulkar
C) Ruskin Bond
D) Samvad Kaumudi
Answer : A
✓ Poet-diplomat Abhay K launched his new book ‘Monsoon’ at Kathika Cultural Centre in Old Delhi.
✓ The book-length poem takes readers on a journey through the monsoon.
✓ This poem actually takes you along the real path of the monsoon and introduces you to various things;
monuments, biodiversity, cuisine, languages, and so on.
✓ The book is a poetic symphony that transcends cultural boundaries and captures the beautiful emotions
evoked by monsoons. The book creates a poetic tapestry out of love, reflection, and nostalgia.
✓ His translation of the 1st Magahi novel ‘Phool Bahadur’ by Jayanath Pati will be published by Penguin
in Jan 2024

Follow us: Official Site, Telegram, Facebook, Instagram, Instamojo 557


Q. Jose Paulino Gomes, touted as the world’s oldest man, passes away at age 127. He was from which
country?
A) Japan B) Brazil
C) Germany D) France
Answer : B
✓ Brazilian supercentenarian, Jose Paulino Gomes, touted as world’s oldest man, passes away at age 127.
✓ Gomes possessed a marriage certificate dating back to 1917, which indicated his birth on August 4, 1895,
making him a witness to two world wars and three pandemics.
✓ A legal adviser from the civil registry, responsible for maintaining people’s records, confirmed the
accuracy of Jose’s details and verified that he was indeed born before 1900.

Q. Centre owes ___ crores in MGNREGS wages to 18 States and Union Territories
A) ₹5,413 Crores B) ₹5,768 Crores
C) ₹6,010 Crores D) ₹6,366 Crores
Answer : D
✓ The Union Ministry of Rural Development informed Parliament that it owes ₹6,366 crore under the wage
component of the Mahatma Gandhi National Rural Employment Guarantee Scheme (MGNREGS) to
States.
✓ There are a total of 14.42 crore active MGNREGS workers, most of whom are women, dependent on
the wages to supplement their family’s income. Of the total sum, ₹2,770 crore is owed to West Bengal
alone.
✓ Invoking Section 27 of the Mahatma Gandhi National Rural Employment Guarantee Act
(MGNREGA), the Centre has blocked payment of over ₹7,500 crore to West Bengal for “non-
compliance of directives of the Central government” since December 2021.
✓ MGNREGS aims : To enhance livelihood security in rural areas by providing at least 100 days of wage
employment in a financial year to unskilled manual work

Q. When is the Hiroshima Day celebrated every year?


A) 03 August
B) 05 August
C) 06 August
D) 04 August
Answer : C
✓ Hiroshima Day takes place every year on August 6 to commemorate the day in August 1945 when an
atomic bomb was dropped on the Japanese city of Hiroshima on August 6 by United States, followed a
few days later by another drop on the city of Nagasaki on August 9.
✓ Although these bombings effectively ended World War II, the two cities were terribly destroyed and
casualties were estimated at around 200,000, mostly civilians.
✓ This was the first time in the history of warfare when a nuclear bomb was used against a nation.
✓ The Day is observed to advocate, pray and work for a ban on all nuclear weapons and an end to war and
violence through anti-war and anti-nuclear discussions and demonstrations around the world.
✓ 2023 marks the 78th anniversary of the world’s first atomic bombin
✓ On 6th August 1945, a US bomber dropped the uranium fission bomb, codenamed Little Boy, on
Hiroshima, a city in Japan
✓ Three days later it dropped another bomb codenamed Fat Man, on Nagasaki

Follow us: Official Site, Telegram, Facebook, Instagram, Instamojo 558


Q. Which of the following state has launched the “Amrit Brikshya Andolan ” web portal/mobile
application?
A) Odisha B) Assam
C) Chhattisgarh D) Jharkhand
Answer : B
✓ Assam Chief Minister Himanta Biswa Sarma has launched the Department of Forest & Environment’s
Amrit Brikshya Andolan web portal/mobile application and the Wood-based Industry (WBI)
registration web portal, along with the theme song for the Amrit Brikshya Andolan.
✓ It is an initiative aimed at planting a total of 1 crore (10 million) commercially-viable saplings across the
State by individuals from various walks of life on coming September 17.
✓ Individuals who register on the ‘Amrit Brikshya Andolan’ app or portal and upload their geo-tagged
photographs planting saplings will receive a reward of Rs 100 in their bank account, with an aim to
encourage more people to participate in the tree plantation drive.

Q. India’s first beauty and lifestyle festival NYKAALAND will be held in which city in November 2023?
A) Hyderabad B) Chennai
C) Bengaluru D) Mumbai
Answer : D
✓ India’s First Beauty and Lifestyle Festival NYKAALAND debuts In Mumbai.
✓ Omnichannel consumer tech company Nykaa has collaborated with Book My Show to give India its first
beauty and lifestyle festival Nykaaland.
✓ The two-day event will be held in Mumbai on November 5 and 6 at the Royal Western India Turf Club
in Mahalaxmi Racecourse.
✓ The inaugural beauty and lifestyle festival will see the participation of leading makeup brands, makeup
artists, entrepreneurs, dermatologists, beauty experts, and content creators.
Q. Which of the following has partnered with the Indian Institute of Management Ahmedabad (IIM-A)’s
startup incubator CIIE to support startups with meritorious ideas and technologies?
A) RBI B) SEBI
C) PFRDA D) SIDBI
Answer : D
✓ Small Industries Development Bank of India (SIDBI), the principal financial institution for MSMEs in
the country has partnered with the Indian Institute of Management Ahmedabad (IIM-A)’s startup
incubator CIIE.
✓ Through this tie-up, SIDBI will set up a SIDBI Accelerator Fund at the incubator with an initial corpus
of Rs 20 crore.
✓ Objective: To support startups with meritorious ideas and technologies important for India’s goals for
2047 to become self-reliant
Q. The union government has recently launched which edition of the Vivad se Vishwas scheme for settling
contractual disputes involving the government and government undertakings?
A) 1st B) 2nd
C) 3rd D) 4th
Answer : B

Follow us: Official Site, Telegram, Facebook, Instagram, Instamojo 559


✓ The government has launched the Vivad se Vishwas-2 scheme for settling contractual disputes involving
the government and government undertakings.
✓ The scheme was announced in the Union Budget 2023-24.
✓ The scheme aims to resolve the government’s contractual disputes with private parties, clear the backlog
of litigation, and improve the ease of doing business.
✓ The deadline for submitting claims under the scheme is 31 October 2023.
✓ The scheme will apply to all domestic contractual disputes where one of the parties is either the
Government of India or an organization operating under its control.
✓ The central government is aiming to resolve approximately 500 cases, involving an estimated Rs 1 trillion,
under this new voluntary settlement scheme.
✓ The one-time settlement scheme will cover cases where the court order was passed by 30 April 2023, and
the arbitral order was given by 30 January 2023
Q. The Union government has launched Digital Crop Survey (DCS) in how many states on a pilot basis
from Kharif 2023 for better sowing data collection?
A) 17 B) 12
C) 11 D) 15
Answer : B
✓ The Union government has launched Digital Crop Survey (DCS) in 12 states on a pilot basis from Kharif
2023 (summer-sown) season of this year for better sowing data collection.
✓ DCS reference application has been developed as an open source, open standard, and interoperable public
good.
✓ The 12 states selected for the pilot on DCS are Madhya Pradesh, Karnataka, Telangana, Andhra Pradesh,
Uttar Pradesh, Rajasthan, Tamil Nadu, Maharashtra, Odisha, and Assam.
✓ The states have been selected based on the preparedness in respect of pre-requisite criteria for DCS i.e.,
geo-referencing of village map and digitized Record of Rights (RoR) with ownership extent.

Q. Prime Minister Narendra Modi has inaugurated the Semicon India 2023 conclave in which state?
A) Rajasthan B) Gujarat
C) Kerala D) Madhya Pradesh
Answer : B
✓ Prime Minister Narendra Modi has inaugurated the Semicon India 2023 conclave at Mahatma Mandir
in Gujarat.
✓ The three-day conclave is being organized by the Ministry of Electronics and Information Technology to
showcase India’s semiconductor strategy and development in the field of Semiconductors.
✓ The theme of the Conference is ‘Catalysing India’s Semiconductor Ecosystem’.
Q. Prime Minister Narendra Modi has launched “Urea Gold”–a new variety of urea that is coated with
sulphur in which state?
A) Uttar Pradesh B) Madhya Pradesh
C) Rajasthan D) Haryana
Answer : C
✓ Prime Minister Narendra Modi has launched “Urea Gold”–a new variety of urea that is coated with
sulphur in Rajasthan’s Sikar.
✓ A bag of urea, which is sold in India at Rs 266, is being sold at Rs 800 in Pakistan, Rs 720 in Bangladesh,
Rs 2,100 in China, and Rs 3,000 in the US.”

Follow us: Official Site, Telegram, Facebook, Instagram, Instamojo 560


✓ What is Urea Gold?
✓ It is a new variety of urea that is coated with sulphur, which is expected to address sulphur deficiencies
in the soil.
✓ The application of sulphur-coated urea, known as Urea Gold, will address sulphur deficiencies in the
soil.
Q. India has signed five MoUs with which country for Quick Impact Projects (QIPs) in education, health,
water facility, and IT?
A) Maldives B) Laos
C) Vietnam D) Malaysia
Answer : B
✓ India has signed five MoUs with Laos for Quick Impact Projects (QIPs) in education, health, water
facility, and IT.
✓ The MoUs were signed during the 3rd round of Foreign Office Consultations (FOC) held between India
and Lao PDR in Vientiane.
✓ Both sides discussed ways to bolster cooperation in various domains, including agriculture, health, and
trade ties.
✓ The next round of Foreign Office Consultations will be held in New Delhi.
Q. Amazon India will open its first-ever floating store in which lake under the last point delivery programme
‘I Have Space’?
A) Chilika Lake B) Lonar Lake
C) Wular Lake D) Dal Lake
Answer : D
✓ Amazon India to open first floating store in Dal Lake
✓ As a part of the last point delivery programme ‘I Have Space’, Amazon India has announced to open its
first-ever floating store in famous Dal Lake in Srinagar.
✓ The store should be open in a houseboat and will provide deliveries to its customers every day in Srinagar.
✓ Murtaza Khan Kashi, owner of houseboat Selec Town, will deliver packages every day to customers at
the doorstep of their houseboats as part of the onboarding
✓ This will provide faster and more reliable deliveries to customers across Srinagar, opportunities for small
businesses and strengthen Amazon’s delivery network.
✓ Launched in 2015, the ‘I Have Space’ programme has 28,000 neighbourhood and kirana partners in about
420 towns and cities in India.
✓ It partners with local stores and small businesses for delivering products within a 2-4 kilometres radius.
✓ The largest freshwater lake in India is Wular Lake (also coined as Wullar). It is also one of the largest
freshwater lakes in Asia. Wular Lake is located in Bandipora district of Jammu and Kashmir, India.
Tectonic activity has formed the lake basin and it is fed by the Jhelum River
Q. When is the Muslim Women’s Rights Day observed in India?
A) August 02 B) August 01
C) July 31 D) July 30
Answer : B
✓ In India, “Muslim Women’s Rights Day” is celebrated across the nation on August 01 to celebrate the
enactment of the law against Triple Talaq.
✓ The first Muslim Women’s Rights Day was observed in 2020.

Follow us: Official Site, Telegram, Facebook, Instagram, Instamojo 561


✓ The Government of India enacted the law against Triple Talaq on August 01, 2019 to make the social
malpractice of Triple Talaq a criminal offense.
✓ The law is formally called the Muslim Women (Protection of Rights on Marriage) Act, 2019.
✓ It prohibits the practice of instant divorce by Muslim men and any violation of the law is punishable by
a jail term of up to three years
Q. James Ferguson ‘Jim’ Skea was elected as the new chairman of the Intergovernmental Panel on Climate
Change (IPCC) in Nairobi, Kenya. He is from which country?
A) United States B) United Kingdom
C) Germany D) France
Answer : B
✓ James Ferguson ‘Jim’ Skea of the United Kingdom was elected as the new chairman of the
Intergovernmental Panel on Climate Change (IPCC) in Nairobi, Kenya.
✓ IPCC (Intergovernmental Panel on Climate Change):
✓ It was established in 1988 through a collaboration between the United Nations Environment Program
(UNEP) and the World Meteorological Organization (WMO).
✓ Its main headquarters is located in Geneva

Q. Who has launched the book titled ‘Dr APJ Abdul Kalam: Memories Never Die’ in Rameshwaram?
A) Narendra Modi B) Nirmala Sitharaman
C) Piyush Goyal D) Amit Shah
Answer : D
✓ Union Home Minister, Amit Shah has launched the book titled ‘Dr. APJ Abdul Kalam: Memories Never
Die’ in Rameshwaram.
✓ This book is an English translation of the Tamil book ‘Ninaivugalukku Maranamillai’. Written by two
people closest to A.P.J. Abdul Kalam, his niece Dr. Nazema Maraikayar and the distinguished ISRO
scientist Dr. Y.S. Rajan, who was a close confidante of Kalam.
✓ This is the story of how a small-town boy from Rameswaram ascended to the highest echelons of the
Indian political world.
✓ The book is about the history of Indian rocketry, science, technology and innovation, beautiful
representation of Indian politics and administrative system and description of many incidents related to
wishes and imagination of Dr. Kalam

Q. The Great Barrier Reef is located in which country?


A) Brazil B) Egypt
C) Peru D) Australia
Answer : D
✓ A UNESCO heritage committee recently stopped short of listing Australia’s Great Barrier Reef as a site
that is “in danger” but warned the world’s biggest coral reef ecosystem remained under “serious threat”
from pollution and the warming of oceans.
✓ About Great Barrier Reef.
✓ Location: It lies in the Pacific Ocean off the northeastern coast of Queensland, Australia, in the Coral
Sea.
✓ It is the longest and largest coral reef system in the world.
✓ It was designated as a UNESCO World Heritage Site in 1981.

Follow us: Official Site, Telegram, Facebook, Instagram, Instamojo 562


Q. Who has been elected as a member of the Board of Directors and a Constituency Director at BRICS'
New Development Bank?
A) Raja Al Gurg B) Aisha Bint Buti Bin Bisher
C) Maryam Buti Al Suwaidi D) Thuraiya Hamid Alhashmi
Answer : D
✓ BRICS’ New Development Bank (NDB) has elected Thuraiya Hamid Alhashmi, Director of
International Financial Relations and Organisations Department at the UAE Ministry of Finance, as a
member of its Board of Directors and a Constituency Director.
✓ Alhashmi will become the first Emirati and Arab woman to assume a Director’s position at the
multilateral bank, established by the grouping by Brazil, Russia, India, China and South Africa (BRICS)
in July 2015.
✓ Her appointment followed an internal voting process during NDB’s 8th annual meeting, held recently in
Shanghai, China.
✓ NDB, also known as BRICS bank, is a multilateral financial institution set up by the five BRICS
countries, namely Brazil, Russia, India, China, South Africa.
✓ Egypt has been inducted as the fourth new member of the BRICS New Development Bank(NDB).
✓ New Development Bank approves UAE, Bangladesh and Uruguay as a new member
✓ New Development Bank (NDB)
✓ Founded : 2015
✓ Headquarters : Shanghai, China
✓ Membership (9) : Brazil ,Russia, India, China , South Africa, Bangladesh, UAE , Uruguay & Egypt.
✓ President : Dilma Vana Rousseff ( Brazil )
✓ Vice President : Anil Kishora (Indian)
✓ The initial authorized capital of the bank is $100 bln divided into 1 mln shares having a par value of
$100,000 each.
✓ The initial subscribed capital of the NDB is $50 bln divided into paid-in shares ($10 bln) and callable
shares ($40 bln).

Q. Which country Tiger population has Increased by 27 percent since its last count in 2015?
A) Nepal B) Bhutan
C) Sri Lanka D) Bangladesh
Answer : B
✓ Over a decade later on Global Tiger Day 2023, the Royal Government of Bhutan has announced that
tiger populations continue to increase in the country, a fantastic achievement.
✓ The latest survey estimates there now to be 131 tigers in the country, a 27 percent increase from the last
survey in 2015.
✓ It reflected the success of Tiger conservation efforts in the South Asian Himalayan landlocked nation
situated between China and India.
✓ It was in 2010 that Bhutan continued its commitment to protecting its wildlife when the country joined
the global effort to double the number of tigers in the wild by 2022

Follow us: Official Site, Telegram, Facebook, Instagram, Instamojo 563


BEST MCQ WITH FACTS
JULY 2023

Q. Byculla Railway Station has received the prestigious UNESCO Asia Pacific Cultural Heritage award.
Byculla Railway station is located in which state?
A) Gujarat B) Kerala
C) Maharashtra D) Uttar Pradesh
Answer : C
✓ The historic Byculla Railway Station, located in Mumbai received the prestigious UNESCO Asia Pacific
Cultural Heritage award.
✓ The 169-year-old Byculla railway station in Mumbai is one of the oldest railway stations in India still in
use.
✓ In fact, the Chhatrapati Shivaji Maharaj Vastu Sangrahalaya (CSMVS) also won the highest award of
excellence in the UNESCO Asia Pacific awards 2022, for cultural heritage conservation last year.
✓ The project, launched in July 2019 by Shaina NC, trustee of ‘I Love Mumbai’ and in memory of her
father Nana Chudasama, aimed to revive the station’s original architectural splendor.
Q. Phangnon Konyak has become the first woman to chair the Rajya Sabha. She is from which state?
A) Nagaland B) Tripura
C) Manipur D) Meghalaya
Answer : A
✓ Phangnon Konyak became the first woman to chair the Rajya Sabha from Nagaland.
✓ Phangnon Konyak became the first woman member to be appointed to the panel of Vice-Chairpersons.
✓ Rajya Sabha Chairman Jagdeep Dhankhar nominated four women members (50% of the total) to the
panel of Vice-Chairpersons to promote gender equality.
✓ All the nominated women members are first time MPs
✓ The reconstituted panel now comprises a total of eight members, half of whom are women, a first in the
history of the Upper House
✓ The nominated women members are as follows:
✓ PT Usha: Renowned athlete and Padma Shri awardee, nominated to Rajya Sabha in July 2022.
✓ S. Phangnon Konyak: First woman elected as Rajya Sabha member from Nagaland (April 2022) and
serves on various committees.
✓ Dr. Fauzia Khan: Nominated to Rajya Sabha in April 2020, Member of various committees.
✓ Sulta Dev: Elected to Rajya Sabha in July 2022, served on various committees
Q. Who has become the 83rd Grandmaster of India?
A) Vidit Santosh
B) Aditya Samant
C) Nihal Sarin
D) Parimarjan Negi
Answer : B
✓ Aditya Samant from Maharashtra becomes India’s 83rd Grandmaster.

Follow us: Official Site, Telegram, Facebook, Instagram, Instamojo 564


Q. Who has becomes the first Indian city to join the World Cities Culture Forum (WCCF)?
A) Hyderabad B) Bengaluru
C) Mumbai D) Chennai
Answer : B
✓ Bengaluru becomes the first Indian city to join the World Cities Culture Forum (WCCF).
✓ Karnataka capital Bengaluru has achieved a significant milestone by becoming the first Indian city to join
the prestigious World Cities Culture Forum (WCCF).
✓ The World Cities Culture Forum (WCCF) is a global network of cities that collaboratively research and
share intelligence to explore the important role of culture in shaping future prosperity.
✓ Bengaluru is the 41st city to join WCCF, which already includes 40 cities across six continents such as
New York, London, Paris, Tokyo and Dubai.
✓ In the past year, the cultural essence of Bengaluru has been showcased through multimedia assets by
WCCF as part of a collaborative project called 'Unboxing Bangalore'.
✓ The 'Unboxing BLR Foundation' established in April 2022 played a key role in this achievement. The
Foundation is a non-profit initiative, founded by Prashant Prakash, founding partner of Accel India, and
Malini Goyal, a philanthropist and journalist.
✓ World Cities Culture Forum (WCCF)
✓ WCCF was established in 2012.
✓ Justin Simmons, the Deputy Mayor of London for Culture and Creative Industries, was instrumental in
its setting up.
✓ In 2015, Queen Elizabeth recognized Justin Simmons' significant contribution to culture in London.
✓ As a token of appreciation, Justin Simmons was awarded the Order of the British Empire (OBE).
Q. Which organisation has published its 2023 Global Education Monitoring Report titled "Technology in
Education: A Tool on Whose Terms"?
A) UNDP B) UNEP
C) WEF D) UNESCO
Answer : D
✓ UNESCO urges global ban on smartphones in schools to promote human-centred education approach.
✓ The United Nations Educational, Scientific and Cultural Organization (UNESCO) published its 2023
Global Education Monitoring Report titled "Technology in Education: A Tool on Whose Terms", which
supports a worldwide ban on smartphones in schools where technology integration Does not improve
learning outcomes.
✓ UNESCO advocates a more "human-centred approach" to education and highlights the need to reduce
reliance on digital technology.
✓ Excessive use of smartphones has negative effects on children's academic performance and emotional
competence.
✓ UNESCO's Manos Antoninis raised concerns about data leaks in educational technology, revealing that
only 16 percent of countries have laws guaranteeing data privacy in the classroom.
✓ The shift to online-only learning during the COVID-19 pandemic has resulted in the loss of
approximately 500 million students worldwide.
✓ The survey conducted by UNESCO reveals that only 11 out of 51 governments have implemented a
curriculum for artificial intelligence (AI) in schools
Q. Which state to set up India's first fisheries Atal Incubation Centre?
A) Telangana B) Kerala
C) Karnataka D) Tamil Nadu

Follow us: Official Site, Telegram, Facebook, Instagram, Instamojo 565


Answer : B
✓ Kerala to set up India's first fisheries Atal Incubation Centre.
✓ After receiving a grant of ₹10 crore from NITI Aayog, the Kerala University of Fisheries and Ocean
Studies (KUFOS) will set up India's first Atal Incubation Center (AIC) in fisheries.
✓ NITI Aayog, as the premier policy think tank of the Government of India, provided a grant to KUFOS
to promote innovation and entrepreneurship in the fisheries sector.
✓ The AIC initiative is part of the Atal Innovation Mission, which aims to foster a culture of innovation
and entrepreneurship across various industries in the country.
✓ Fisheries Atal Incubation Centre:
✓ Fisheries Atal Incubation Centre will act as a focal point to foster innovation and inspire young
individuals to create advanced technologies and solutions that will address challenges faced by our
marine ecosystem and fishing communities .
✓ Its primary objective is to promote progress in the fisheries industry by providing a supportive and
encouraging environment for startups and innovative initiatives within the sector.
✓ The centre will play a key role in creating employment opportunities, creating a conducive environment
for startups and entrepreneurs to flourish and succeed.
✓ Vice Chancellor of KUFOS - T. Pradeepkumar

KERALA IN NEWS 2023


✓ Kerala to set up India's first fisheries Atal Incubation Centre.
✓ Piramal Finance opened the first all women branch named "Maitreyi" at Thripunithura near Kochi to
cater to the unique needs of women customers.
✓ Kerala has become the first state in the country to launch its own internet service.
✓ Kerala has become the first and only state in the country to have its own internet services.
✓ According to data shared by the GI Registry, Kerala received the highest number of Geographical
Indication (GI) tags for products among all states in India in the fiscal year 23 (FY23).
✓ Kerala has adopted a water budget, the first of its kind in the country, to deal with the problem of water
scarcity in summer.
✓ Dharmadam, Kerala becomes first complete library constituency in India.
✓ Kerala CM Pinarayi Vijayan launches 'One Panchayat, One Playground' project.
✓ India’s first Graphene Innovation center to be set up in Kerala.
✓ Recently, a group of researchers discovered a butterfly subspecies from the fringes of Akkulam and
Vembanad lakes in Kerala.
✓ Prime Minister Narendra Modi on 25th April flagged off Kerala’s first Vande Bharat Express between
Thiruvananthapuram and Kasargod at Thiruvananthapuram Central Station.
✓ Kerala declared first State to enable digital banking service.
✓ M T Vasudevan Nair was awarded the "Kerala Jyoti", the highest civilian honour in Kerala.
✓ The Ministry of Women and Child Development has launched the 2nd G20 Empower meeting in
Thiruvananthapuram, Kerala under the theme “Women Empowerment: Win-win for Equality and
Economy”.
✓ Kerala to become first State to use robotics tech extensively for manhole cleaning.
✓ National Green Tribunal (NGT) in New Delhi has imposed a fine of Rs 10 crore on the Kerala
government for failing to protect the Vembanad and Ashtamudi lakes, listed as Ramsar sites.
Q. Which state government has topped the list with the highest number of missing women in 2021, with
a reported count of 56,498?
A) Maharashtra B) West Bengal
C) Madhya Pradesh D) Uttar Pradesh
Answer : A

Follow us: Official Site, Telegram, Facebook, Instagram, Instamojo 566


✓ Among all states, Maharashtra topped the list with the highest number of missing women in 2021, with
a reported count of 56,498.
✓ In 2021, a total of 375,058 women (above 18 years) were reported missing in India, according to data
from the National Crime Records Bureau (NCRB) as mentioned in the Ministry of Home Affairs (MHA)
press release.
✓ Additionally, 90,113 girls (under the age of 18) were also reported missing in India during the same year.
✓ The second-highest number of missing women was in Madhya Pradesh with 55,704 cases.
✓ Following closely, West Bengal reported 50,998 missing women, while Odisha recorded 29,582 cases in
the same year.
✓ Compared to the previous year, in 2020, there were 320,393 women and 71,204 girls reported missing
across the country.
✓ Madhya Pradesh and Maharashtra were the two states that witnessed the highest number of missing girls
and women from 2019 to 2021.
✓ In the three-year period from 2019 to 2021, a concerning total of 10,61,648 women and 2,51,430 girls
went missing in India.
Q. Batagaika, the world's largest permafrost crater, is located in the Far East Siberian taiga and is expanding
rapidly due to global warming in which country?
A) Ukraine B) Russia
C) United States D) United Kingdom
Answer : B
✓ World's largest permafrost crater 'Batagaika' is expanding rapidly due to global warming in Russia.
✓ This massive crater, also known as the "Gateway to the Underworld", initially appeared in the 1960s
when the area was cleared for timber.
✓ The crater is about one kilometre long and is classified as a 'mega-slump', indicating significant geological
changes on Earth.
✓ One of the primary factors contributing to the formation of permafrost craters is the thawing of
permafrost, which is being caused by deforestation, land use change, and rising temperatures.
✓ Russia's warming rate is at least 2.5 times faster than the global average, melting its long-frozen tundra
and contributing to the expansion of the 'Batagaika' crater at a rate of about 10 metres per year.
✓ Permafrost Crater:
✓ Permafrost refers to land that is frozen at or below 0°C for at least two consecutive years.
✓ It is mainly found in areas with high terrain and close to the Earth's poles, that is, the North and South
Poles. Permafrost is made up of a mixture of soil, rock, and sand that is bound together by ice.
Q. India's first online gaming academy launched in which state government?
A) Uttar Pradesh B) Rajasthan
C) Haryana D) Madhya Pradesh
Answer : D
✓ Madhya Pradesh is launching its first online gaming industry "MP State Esports Academy".
✓ The Academy aims to provide a platform for aspiring gaming and esports professionals to advance their
skills and reach a professional level.

Q. Whic of the following has launched the world's largest private communications satellite 'Jupiter 3'?
A) SpaceX B) ISRO
C) JAXA D) CNES
Answer : A

Follow us: Official Site, Telegram, Facebook, Instagram, Instamojo 567


✓ SpaceX launches 'Jupiter 3', world's largest private communication satellite.
✓ SpaceX launched the world's largest private communications satellite 'Jupiter 3' using a Falcon Heavy
rocket from Launch Complex-39A of NASA's Kennedy Space Center in Florida.
✓ The 'Jupiter 3' satellite, developed by Maxar Technologies in Palo Alto, California, holds the record as
the largest commercial communications satellite ever built.
✓ This upcoming launch marks the seventh mission of SpaceX's triple-booster rocket, the Falcon Heavy,
which first attracted widespread attention during its debut in 2018.
✓ About 'Jupiter 3'
✓ Its size would be comparable to the wingspan of a commercial airliner, measuring between 130 and 160
feet (40 to 50 m). Its larger size will enable it to double current internet capacity to 500 Gbps, significantly
improving connectivity in areas with limited cable and fibre options.
✓ Hughes will integrate the 'Jupiter 3' into its existing satellite network, allowing HughesNet customers in
the United States and Latin America to access high-speed satellite broadband at speeds of up to 100
Mbps.
✓ Space Exploration Technologies Corporation (SpaceX) is an American spacecraft manufacturer, launch
service provider and satellite communications company headquartered in Hawthorne, California.
✓ The company was founded in 2002 by Elon Musk.
✓ CEO & chairman : Elon Musk
Q. The government has extended the unemployment scheme, the Atal Beemit Vyakti Kalyan Yojana, under
the Employees’ State Insurance Corporation for a period of how many years?
A) 3 B) 2
C) 4 D) 5
Answer : B
✓ Government extends Atal Beemit Vyakti Kalyan Yojana for two years
✓ The government has extended the unemployment scheme, the Atal Beemit Vyakti Kalyan Yojana, under
the Employees’ State Insurance Corporation for a period of two years up to June 30, 2024.
✓ It has been decided to extend the Atal Beemit Vyakti Kalyan Yojana for the period 01.07.2022 to
30.06.2024 with the relaxed eligibility conditions and enhanced rate of relief.
✓ This is the third extension to the scheme with two extensions given each in 2020 and 2021 to financially
support ESIC beneficiaries who had lost jobs during the pandemic.
✓ Launched in 2018 on a pilot basis for a period of two years, the Atal Beemit Vyakti Kalyan Yojana
provides relief in the form of cash compensation up to 90 days to the insured persons (IPs) under ESIC
who have become unemployed.
✓ As per the government data, 66,013 employees availed unemployment benefits under the scheme Atal
Beemit Vyakti Kalyan Yojana (ABVKY) and an amount of Rs 87.83 crores were disbursed between July
1, 2018 and December 12, 2022
Q. Who has won the Miles Franklin Literary Award 2023 for her novel Chai Time at Cinnamon Gardens?
A) Shankari Chandran B) Meghna Pant
C) Meena Kandasamy D) Janice Pariat
Answer : A
✓ Shankari Chandran has won the Miles Franklin Literary Award 2023 for her novel Chai Time at
Cinnamon Gardens.
✓ The novel is set in the Cinnamon Gardens Nursing Home in the suburb of Westgrove, Sydney –
populated with residents with colourful histories.
✓ Ten years ago, her first novel was rejected by several Australian publishers for not being sufficiently
“Australian”. It was published in 2017 in Sri Lanka with the title Song of the Sun God.

Follow us: Official Site, Telegram, Facebook, Instagram, Instamojo 568


✓ The Miles Franklin Literary Award is an annual prize given to “a novel which is of the highest literary
merit and presents Australian life in any of its phases”.
Q. Who has become the first unseeded female player to win the Wimbledon Grand Slam?
A) Marketa Vondrousova B) Ons Jabeur
C) Barbora Strycova D) Hsieh Su-wei
Answer : A
✓ Marketa Vondrousova of the Czech Republic has become the first unseeded female player to win the
Wimbledon Grand Slam.
✓ In the final match, Marketa defeated Ons Jabeur of Tunisia by 6-4, 6-4.
✓ She achieved this feat as the first unseeded woman in the Wimbledon final in 60 years.
✓ This is Vondrousova's first Grand Slam title
Q. Which player won the Men's title of Wimbledon Grand Slam 2023?
A) Novak Djokovic B) Carlos Alcaraz
C) Daniil Medvedev D) Rafael Nadal
Answer : B
✓ Spain's young player Carlos Alcaraz created history by winning his first Wimbledon title.
✓ Alcaraz defeated veteran Novak Djokovic in the men's singles final of Wimbledon 2023.
✓ This is his second Grand Slam title.
✓ He has become the third youngest player to win the Wimbledon Men's title.
✓ Alcaraz won the US Open title in the year 2022.
AUSTRALIAN OPEN 2023
▪ Novak Djokovic won the Australian Open title for the 10th time by defeating Stefanos Tsitsipas of Greek.
▪ In the Australian Open 2023, Aryna Sabalenka of Belarus has won the women's singles title by defeating
Elena Rybakina of Kazakhstan. Sabalenka has won a Grand title for the first time.
▪ FRENCH OPEN 2023
▪ Serbian Novak Djokovic won the French Open 2023 men's singles final, clinching his 23rd Grand Slam
title .
▪ He won the title by defeating in Casper Rudd of Norway.
▪ The win gives Djokovic his third French Open title.
▪ In French Open tennis, World No 1 and top seed Iga Swiatek of Poland has won the Women's Singles
defeated Czech Republic’s Karolina Muchova.
▪ WIMBLEDON OPEN 2023 ( TENNIS)
▪ Carlos Alcaraz (Spain)defeated the four-time defending champion Novak Djokovic (Serbia) in the final.
▪ Markéta Vondroušová defeated Ons Jabeur win the ladies' singles tennis title at the 2023 Wimbledon
Championships.
Q. Which country has announced the successful elimination of rubella?
A) Bhutan B) Myanmar
C) Sri Lanka D) Bangladesh
Answer : A
✓ Bhutan has announced the successful elimination of rubella. On 21 July 2023, the World Health
Organisation recognised Bhutan's efforts to tackle this highly contagious and potentially devastating
disease, The Bhutan Live reported.

Follow us: Official Site, Telegram, Facebook, Instagram, Instamojo 569


✓ In the past 20 years, Bhutan's health workers have worked tirelessly and fought against rubella, also
known as German Measles, according to the report.
✓ This infectious disease poses a threat to those infected and also causes severe complications for pregnant
women, resulting in irreversible birth defects in newborns.
✓ This infectious disease poses a threat to those infected and also causes severe complications for pregnant
women, resulting in irreversible birth defects in newborns.
Q. The International Tiger Day falls on which day of the year annually?
A) 30 July B) 28 July
C) 26 July D) 29 July
Answer : D
✓ International Tiger Day is observed globally on July 29 every year.
✓ It serves as a platform to raise awareness about the endangered tiger species.
✓ It is celebrated worldwide to promote comprehensive efforts in safeguarding tigers and their natural
habitats, fostering harmonious coexistence between humans and tigers
✓ Global Tiger Day, commonly referred to as International Tiger Day, was first established in 2010 during
the Saint Petersburg Tiger Summit by nations that are home to tigers.
Q. Tenzing Yangki has become the first female IPS officer of which state?
A) Sikkim B) Tripura
C) Arunachal Pradesh D) Manipur
Answer : C
✓ Tenzing Yangki has become the first female IPS officer of Arunachal Pradesh who cracked the Union
Public Service Commission (UPSC) Civil Services Examination 2022.
✓ Tenzin Yangki achieved a rank of 545 and has become the first lady IPS officer from Arunachal Pradesh.
✓ Yangki had earlier cleared the Arunachal Pradesh Public Service Commission (APPSC) exam in 2017.
✓ Robin Hibu was the first IPS officer from Arunachal Pradesh. He joined as an IPS officer in 1993.
ARUNACHAL PRADESH IN NEWS 2023
▪ Tenzing Yangki has become the first female IPS officer of Arunachal Pradesh.
▪ Chachin Grazing Festival celebrated in Arunachal Pradesh.
▪ The Chief Minister of Arunachal Pradesh launched an app named 'Arunpol App' of Arunachal Pradesh
Police..
▪ Assam and Arunachal Pradesh have signed an agreement for the settlement of an inter-state boundary
dispute between the two States.
▪ A tripartite committee recommended that some areas be transferred from Assam to Arunachal..
▪ Union Home Minister and Cooperation Minister Amit Shah on 10 April launched the 'Vibrant Villages
Programme' at Kibithu, a border village in Anjaw district of Arunachal Pradesh.
▪ Defence Minister Rajnath Singh inaugurates Siyom bridge in Arunachal Pradesh.
▪ A fish museum, the first of its kind in the Northeast, would soon be built in Arunachal Pradesh.
▪ The Chief Minister of Arunachal Pradesh, Pema Khandu, inaugurated the newly renovated Shar Nyima
Tsho Sum Namyig Lhakhang (Gonpa) at Gyangkhar, his native village in Tawang district.
▪ February 20, 2023, Arunachal Pradesh is celebrating its 37th Statehood Day.
▪ A bipartisan resolution was passed by the US, formally recognizing the McMahon Line as the
international boundary between China and India’s Arunachal Pradesh.
▪ The trailer of 'Love in 90s', the first film made in Tagin language of Arunachal Pradesh.

Follow us: Official Site, Telegram, Facebook, Instagram, Instamojo 570


Q. “Akira” which is in the news recently, what is it?
A) Missile System B) Government’s Scheme
C) Internet Ransomware D) Diseases
Answer : C
✓ CERT-In (Indian Computer Emergency Response Team) has issued a warning about new Internet
ransomware named ‘Akira’.
✓ This computer malware is targeting Windows and Linux-based systems.
✓ A recently emerged ransomware operation dubbed Akira is reportedly active in cyberspace. This group
first steals the information from victims, then encrypt data on their systems and conducts double extortion
to force the victim into paying the ransom.
✓ In case the victim does not pay, they release their victim’s data on their dark web blog.
✓ Steps to protect against 'Akira' ransomware:
✓ Maintain up-to-date offline backups of important data to minimise potential data loss in the event of an
infection.
✓ Regularly update the operating system and applications to strengthen protection against vulnerabilities
and potential cyber threats. Consider using "virtual patching" to protect legacy systems and networks,
making it harder for cybercriminals to exploit legacy software.
✓ Indian Computer Emergency Response Team (ICERT)
✓ It functions as a division within the Ministry of Electronics and Information Technology under the
Government of India.
✓ ICERT is primarily tasked with dealing with and combating various cyber security threats including
hacking and phishing.
✓ Director General - Sanjay Behl
Q. India and which country have agreed to a bilateral exchange of young researchers and Startups,
particularly in the field of Biotechnology and Agriculture?
A) Indonesia B) Vietnam
C) Argentina D) Thailand
Answer : C
✓ India and Argentina have agreed to a bilateral exchange of young researchers and Startups, particularly
in the field of Biotechnology and Agriculture.
✓ The two countries also decided to hold comprehensive talks between delegations comprising academia,
Research, and Development Institutes, and Industry in the field of Science and Technology and
Biotechnology.
✓ India is the 4th largest trading partner of Argentina, with bilateral trade touching a historic peak of 6.4
billion US dollar in the year 2022, recording a growth rate of 12 percent over 2021.

Q. Which Companies signed MoU to foster advancements in AI & Emerging Technologies?


A) India AI and Google B) India AI and Meta
C) India AI and Amazon D) India AI and Microsoft
Answer : B
✓ In order to foster collaboration in the field of AI and Emerging Technologies, ‘India AI’ – an IBD under
Digital India Corporation and Meta, India have signed an MoU here 26th July 2023.
✓ The MoU was signed by Shri Abhishek Singh, CEO of India AI, and Shri Shivnath Thukral, Director &
Head of Public Policy, Meta in India.

Follow us: Official Site, Telegram, Facebook, Instagram, Instamojo 571


✓ The objective of the MoU is to establish a framework for collaboration and cooperation between ‘India
AI’ and Meta in the field of Artificial Intelligence & Emerging Technologies including to make Meta’s
open-source AI models available for use by Indian AI ecosystem.

Q. Which Company becomes largest single-location PSU refinery in India for the year 2022-23?
A) VRPL B) GRPL
C) ORPL D) MRPL
Answer : D
✓ Mangalore Refinery and Petrochemicals Limited (MRPL), a mini-Ratna CPSE PSU refinery based in
coastal Karnataka and a subsidiary of ONGC, has become the single largest PSU-refinery (single
location) in the country for the year 2022-23, a release from the company.
✓ MRPL achieved this feat by processing 17.14 million metric tonnes of crude oil during the past financial
year.
✓ This is also the highest-ever throughput processed by any single-location PSU refinery in India's
petroleum refining history.
✓ MRPL processes 10 per cent of the total crude oil refined by the PSU petroleum refineries in the country.
✓ Set up as a joint venture refinery in 1988 with a 3.69 MMTPA (million metric tonnes per annum)
capacity, MRPL later underwent a second and third-phase expansion to raise its capacity to 15 MMTPA.
✓ The refinery configuration has a Nelson Complexity Index of 11.3, one of the highly complex PSU
refineries.
✓ The MRPL petrochemical intensity is currently at 9.5 per cent, aiming to reach 15 per cent in the medium
term.
✓ The Nelson Complexity Index (NCI) refers to the types of petroleum products can be produced by a
refinery.
✓ Measured on a scale from 1 to 20, the higher the value on the NCI, the more sophisticated and complex
products the refinery can produce

Q. The World Hepatitis Day is observed annually on which day?


A) 26 July B) 27 July
C) 28 July D) 25 July
Answer : C
✓ The World Hepatitis Day is observed on 28 July every year to raise global awareness of the burden of
viral hepatitis, an inflammation of the liver that causes a range of health problems, including liver cancer.
✓ The day 28 July was chosen for World Hepatitis Day because it is the birthday of Nobel-prize winning
scientist Dr Baruch Blumberg, who discovered hepatitis B virus (HBV). He also developed a diagnostic
test and vaccine for the virus.
✓ Theme of World Hepatitis Day 2023 "One life, one liver,"
Q. Where will PM Modi inaugurate Gujarat's first greenfield airport?
A) Surat B) Ahmedabad
C) Gandhinagar D) Hirasar
Answer : D
✓ Prime Minister Narendra Modi will inaugurate Gujarat's first greenfield airport at Hirasar near Rajkot
on 27 July.
✓ This airport, built in an area of 1500 acres at a cost of 1,405 crores, has been renamed as Rajkot
International Airport.

Follow us: Official Site, Telegram, Facebook, Instagram, Instamojo 572


✓ The greenfield airport at Hirasar is 30 km from Rajkot city, the commercial capital of the Saurashtra
region.
✓ 14 aircraft can be parked here

Q. The central government has approved the decision to set up how many new greenfield airports?
A) 16 B) 21
C) 25 D) 30
Answer : B
✓ The Government of India has given 'in-principle' approval for setting up 21 new greenfield airports. It
includes Mopa in Goa, Navi Mumbai, Shirdi and Sindhudurg in Maharashtra, Kushinagar and Noida
(Jewar) in Uttar Pradesh, etc. Of these, 11 greenfield airports Durgapur, Shirdi, Kannur, Pakyong,
Kalaburagi, Oravakal (Kurnool), Sindhudurg, Kushinagar, Itanagar, Mopa and Shivamogga have been
made operational.

Q. Who has laid the foundation stone of 108 feet tall statue of Lord Shri Ram in Kurnool, Andhra Pradesh?
A) Narendra Modi B) Amit Shah
C) Nitin Gadkari D) Draupadi Murmu
Answer : B
✓ Union Home Minister Amit Shah on laid the foundation stone of 108 feet tall statue of Lord Shri Ram
in Kurnool, Andhra Pradesh through video conferencing.
✓ The statue will be built at a cost of over 500 crore rupees. In his address, Mr. Shah said the statue to be
installed in Mantralayam will give the message of Sanatan Dharma to the entire world, and it will
strengthen the Vaishnava tradition.
✓ The Home Minister said this project is spread over an area of 10 acres in Mantralayam village. It would
be completed in two and a half years.
✓ He said the village is very famous for the temple of Raghavendra Swamy and the place has historical
significance.
Q. In which state 'Mission Shakti Scooter Yojana' has been approved?
A) Odisha B) Jharkhand
C) Bihar D) Telangana
Answer : A
✓ Odisha Cabinet, led by Chief Minister Naveen Patnaik approved Mission Shakti Scooter Yojana, under
which interest subvention on bank loans up to Rs 1 lakh will be provided to beneficiaries to purchase a
scooter.
✓ Odisha Mission Shakti Minister Tukuni Sahu said around 75,000 SHG community support staff and
nearly 1,25,000 SHG federation leaders will be benefited under the new scheme.
✓ The State government has made budgetary provision of Rs 528.55 crore for execution of the scheme over
a period of next five years, she added.
ODISHA IN NEWS 2023
▪ Naveen Patnaik of Odisha has become the second longest serving chief minister of a state in India with
a tenure of 23 years and 139 days.
▪ Odisha Cabinet, led by Chief Minister Naveen Patnaik approved Mission Shakti Scooter Yojana, under
which interest subvention on bank loans up to Rs 1 lakh will be provided to beneficiaries to purchase a
scooter.

Follow us: Official Site, Telegram, Facebook, Instagram, Instamojo 573


▪ Odisha government, on July 2,2023, launched a scheme named, Mo Jungle Jami Yojana to effectively
implement the provisions of Scheduled Tribes and other Traditional Forest Dwellers (Recognition of
Forest Rights) Act 2006.
▪ India’s biggest natural arch formed 184 million years ago discovered in Odisha by Geological Survey of
India (GSI).
▪ Odisha becomes the first state to notify a committee for conservation of seeds preserved by tribal farmers.
▪ Odisha government launched the 'Mo Ghara' (My Home) housing scheme.
▪ PM Modi launches railway projects worth over 8,000 crore rupees in Odisha.
▪ Odisha government has expanded the jurisdiction of Special Development Councils (SDCs) from nine
to 23 districts to improve the lives of 84 lakh tribal people.
▪ Odisha has its 88th foundation day on 1st April 2023.
▪ World Bank’s Board of Executive Directors has approved USD 100 million loan under Odisha State
Capability and Resilient Growth Program.
▪ Odisha won the UN-Habitat's World Habitat Awards 2023 for Jaga Mission, a 5T initiative of the state.
▪ Aska police station of Odisha's Ganjam district became the country's number one police station.
▪ Gangapur police station in Odisha's Ganjam district has been awarded the second best police station
award.
▪ Gold Deposits Found at Different Location in Three Districts of Odisha , including Deogarh, Keonjhar,
and Mayurbhanj.
▪ Chief Minister Naveen Patnaik announced that the Odisha government aims to make Odisha slum-free
by the end of 2023.
Q. Who has become the second longest serving Chief Minister in India?
A) Nitish Kumar B) Mamta Banerjee
C) Naveen Patnaik D) Yogi Adityanath
Answer : C
✓ Naveen Patnaik of Odisha has become the second longest serving chief minister of a state in India with
a tenure of 23 years and 139 days.
✓ Surpassing the record of former West Bengal chief minister Jyoti Basu. Patnaik, the five-time Chief
Minister of Odisha, took charge on March 5, 2000, and has been holding the post for the last 23 years
and 139 days
✓ Past Records :
✓ Patnaik is now second only to Pawan Kumar Chamling of Sikkim, who holds the enviable record of
helming a state for the longest time of 24 years and 166 days between December 1994 and May 2019.
✓ Basu left office in 2000 after ruling the eastern state for over 23 continuous years, while Chamling lost
the assembly polls in May 2019 in the Himalayan state.
✓ List of the longest-serving chief ministers in India :
✓ 1st - Pawan Kumar Chamling Sikkim 24 years, 205 days (12 December 1994 – 27 May 2019).
✓ 2nd - Naveen Patnaik Odisha 23 years, 139 days (5 March 2000 – Present).
✓ 3rd - Jyoti Basu West Bengal 23 years, 137 days (21 June 1977 – 6 December 2000)

Q. Which state government has unveils 'Bomman', the mascot for Asian Champions Trophy 2023?
A) Karnataka B) Gujarat
C) Maharashtra D) Tamil Nadu
Answer : D
✓ Tamil Nadu Youth Welfare and Sports Development Minister "Udhayanidhi Stalin" has unveiled the
trophy and the ‘Bomman’ mascot for the prestigious Asian Champions Trophy 2023 hockey tournament
to be held in Chennai from August 3 to 12, 2023.

Follow us: Official Site, Telegram, Facebook, Instagram, Instamojo 574


✓ On the occasion, the minister also launched the ticket sales by receiving the first ticket from Hockey India
General Secretary Bhola Nath Singh at the Mayor Radhakrishnan Hockey Stadium in Egmore.
✓ Six teams from the continent – India, China, Pakistan, Malaysia, Japan and Korea – will compete for the
coveted trophy.
✓ This will be the first time since the Asia Cup in 2007, the stadium is hosting a major event. Tamil Nadu
government has allocated Rs 17 crore for the conduct of the event.
Q. Which has unveiled the memorial named VC Yeshwant Ghadge Sundial Memorial?
A) Italy B) France
C) Spain D) Germany
Answer : A
✓ Italy has unveiled the memorial named VC Yeshwant Ghadge Sundial Memorial at Montone Italy.
✓ Reason : To pay tribute to the Indian troops who fought in the Italian Campaign during the Second
World War (1939-1945) and also honour Naik Yeshwant Ghadge, Victoria Cross, who was killed in
action fighting on the heights of Upper Tiber Valley
✓ The memorial is in the form of a live sundial.
✓ Motto : Omines Sub Eodem Sole (Translates to 'We all live under the same sun').
Q. Which country has banned sex change and transgender marriage?
A) China B) USA
C) UK D) Russia
Answer : D
✓ Russian President Vladimir Putin has signed a new law that deals with the LGBTQ+ community in the
country.
✓ Under this, sex change and transgender marriage have been banned. This law has been passed
unanimously by both the Houses of the Parliament.
✓ The new law imposes comprehensive restrictions on various aspects related to gender change
Q. Who has created history with best ever performance by an Indian at 2023 Open Golf Championship?
A) Anirban Lahiri B) Jyoti Randhawa
C) Shubhankar Sharma D) Gaganjeet Bhullar
Answer : C
✓ Shubhankar Sharma achieved the best ever performance by an Indian golfer at the 2023 Open Golf
Championship held at the Royal Liverpool Golf Club in Merseyside,England.
✓ Showcasing remarkable skill and determination, Shubhankar Sharma tied for eighth place with Cameron
Young of the United States.
✓ The hometown of Shubhankar Sharma is Jhansi, which is located in the state of Uttar Pradesh.
✓ Brian Herman of the USA has won the 2023 Open Golf Championship.
Q. Which state government has introduced the Platform-Based Gig Workers Bill, 2023, making it the first
state in India to ensure social security assurance for gig workers?
A) Uttar Pradesh B) Rajasthan
C) Chhattisgarh D) Madhya Pradesh
Answer : B

Follow us: Official Site, Telegram, Facebook, Instagram, Instamojo 575


✓ The Rajasthan government has introduced the Rajasthan Platform-Based Gig Workers Bill, 2023, making
it the first state in India to ensure social security assurance for gig workers.
✓ The Welfare Board will enable gig workers to register with all state aggregators, thereby ensuring better
access to opportunities and benefits.
✓ An integrated fee deduction mechanism will be set up within aggregator apps, and non-compliance with
the provisions of the Bill will attract a fine of ₹5 lakh for the first offence and ₹50 lakh for subsequent
offences.

Q. Which government has launched first 'Water ATM' to provide clean water in water-scarce areas?
A) Kerala B) Bihar
C) Delhi D) Uttar Pradesh
Answer : C
✓ Delhi Chief Minister Arvind Kejriwal recently inaugurated the city's first 'Water ATM', which aims to
provide clean water to areas lacking piped water supply.
✓ The 'Water ATM' initiative aims to reduce dependence on water tankers and ensure that underprivileged
sections of society have access to the same quality of reverse osmosis (RO) water as more affluent areas.
✓ The Delhi government, through the Delhi Jal Board, plans to tackle water scarcity by setting up 500 RO
plants with integrated 'water ATM machines' in areas facing water supply issues.
✓ Each of the 500 RO plants will have a capacity of 30,000 litres and will be strategically located depending
on the availability of tubewells. The government will provide the necessary land, and the cost of ₹10 lakh
per plant will be borne by the Delhi Jal Board.
Q. Which country has banned the export of non-basmati white rice to stabilize the volatility in retail prices
within the country?
A) India B) China
C) Russia D) Ukraine
Answer : A
✓ India banned the export of non-basmati white rice to stabilise the volatility in retail prices within the
country.
✓ This decision was taken by the Ministry of Food and Consumer Affairs.
✓ The ban applies to all exports of "non-basmati white rice".
✓ India's decision is significant as it is the world's leading rice exporter.
✓ The primary objective of the ban is to stabilise the volatility of retail prices within India.
✓ Directorate General of Foreign Trade (DGFT):
✓ DGFT is an important agency under theMinistry of Commerce and Industry.
✓ Its headquarter is in New Delhi, India.
✓ DGFT plays an important role in regulating and enforcing laws related to foreign trade.
✓ One of its major responsibilities is to formulate and implement foreign trade policy.
✓ The foreign trade policy aims to promote India's export activities and create a conducive environment
for international trade.
✓ The current Director General of DGFT is Amit Yadav.

Q. Which state had the highest crime rate against Scheduled Caste groups in the year 2021?
A) Uttar Pradesh B) Madhya Pradesh
C) Maharashtra D) Assam
Answer : B

Follow us: Official Site, Telegram, Facebook, Instagram, Instamojo 576


✓ Madhya Pradesh had the highest crime rate against people belonging to Scheduled Caste (SC) groups in
2021, according to data from the National Crime Records Bureau (NCRB).
✓ The data shows that the state had the highest crime rate against SCs in 2020, and was second in 2019
(after Rajasthan).
✓ The National Crime Records Bureau is an Indian government agency that collects crime data. It was
established in 1986.
Q. The Kargil Vijay Diwas is observed every year since 1999 on which day?
A) 24 July B) 25 July
C) 27 July D) 26 July
Answer : D
✓ Kargil Vijay Diwas is celebrated every year on July 26 since the year 1999, to mark India’s victory over
Pakistan in the Kargil conflict.
✓ The day is dedicated to our Kargil War’s Heroes and is named after the successful Operation Vijay.
✓ It is celebrated to in view of Operation Vijay and Kargil War that ended on July 26, 1999. After this,
India successfully took command of the high outposts which had been lost to Pakistan.
✓ ‘Operation Vijay’, was the code name given to India’s offensive against Pakistan to flush out the Pakistani
infiltrators from the Indian territory in Kargil.
✓ This year 24th Kargil Vijay Diwas is being organized.
Q. India’s first Cannabis Medicine Project is set to be pioneered in which city?
A) Jammu B) New Delhi
C) Lucknow D) Bengaluru
Answer : A
✓ Union Minister of State Science & Technology Jitendra Singh confirmed that India’s first Cannabis
Medicine Project is set to be pioneered in Jammu.
✓ This project will play a vital role in Atma-Nirbhar Bharat, as it will be capable of producing export-quality
drugs designed for various neuropathies etc.
✓ The project will spread awareness that this substance of abuse has diverse medicinal uses, especially for
patients suffering from malignancies.
✓ The ‘Cannabis Research Project’ of CSIR-IIIM Jammu is a first-of-its-kind in the country in public-
private partnership with a Canadian firm, which has a “great potential to put substance of abuse for the
good of mankind especially for patients suffering from neuropathies, cancer and epilepsy
✓ This project of CSIR-IIIM is also important from the perspective of ‘Atmanirbhar Bharat’ (self-reliant
India) as after getting all the approvals, it will be able to produce export quality drugs meant for different
kinds of neuropathies and diabetic pains

Q. Which Football Club became the first Indian club to win the Gothia Cup in Sweden?
A) Spartan Academy B) Minerva Academy
C) Velocity Football Academy D) Champion Football Academy
Answer : B
✓ India’s Minerva Academy FC scripted history as they became the first team from the subcontinental
nation to lift the prestigious Gothia Cup in Sweden.
✓ The Gothia Cup, an international youth football tournament organized has been held annually at
Gothenburg in the Scandinavian country of Sweden.
✓ The prestigious tournament is organized under the aegis of Allsvenskan club BK Hacken.

Follow us: Official Site, Telegram, Facebook, Instagram, Instamojo 577


✓ The Indian team triumphed in the B13 category of the illustrious event with a 3-1 win over Brazilian side
Ordin FC in the final of the tournament on Saturday.
✓ The tournament is the world’s largest and most international youth football tournament with 1700 teams.

Q. The 'Meri Mati Mera Desh Abhiyan' campaign is an initiative of which ministry?
A) Ministry of Panchayati Raj B) Ministry of Rural Development
C) Ministry of Mines D) Ministry of Health and Family Welfare
Answer : B
✓ 'Meri Mati Mera Desh Abhiyan' is an initiative of the Ministry of Rural Development.
✓ This campaign will be run across the country from 09 to 15 August.
✓ Selected youths from around 7,500 blocks across the country will gather for this program at Delhi's
'Kartavya Path'.
✓ This campaign has been started for the conclusion of the Amrit Mahotsav of Independence.
✓ The Amrit Mahotsav of Independence was started on March 12, 2021.

CAMPAIGN IN NEWS 2023


▪ President Droupadi Murmu, in a historic virtual ceremony, launched the Ayushman Bhav campaign and
the Ayushman Bhava portal from the Raj Bhavan in Gandhinagar, marking a significant stride towards
achieving Universal Health Coverage (UHC) in India.
▪ This campaign aims to enhance the accessibility and affordability of healthcare services, especially for
underserved communities.
▪ 'Meri Mati Mera Desh Abhiyan' is an initiative of the Ministry of Rural Development.
▪ Union Minister Anurag Thakur flagged off "Bharat in Paris" campaign.
▪ Federal Bank launches 'I am Adyar, Adyar is me' campaign.
▪ Union Minister Hardeep S Puri launched the campaign 'Meri LiFE, Mera Swachh Shahar' in New Delhi.
▪ RBI Launches ‘100 Days 100 Pays’ Campaign to Settle Unclaimed Deposits.
▪ Union Minister Giriraj Singh launched the SAMARTH campaign to promote digital transactions at the
Gram Panchayat level.
▪ A 100-day "Uttar Pradesh Global City" campaign has been launched by the 'Uttar Pradesh Government'
ahead of the Global Investors Summit and G-20 Summit.
▪ UAE's Sharjah Police launched a traffic campaigncalled "Stay in Your Lane"to tackle road violations..
▪ Union Agriculture Minister Narendra Singh Tomar inaugurated the National Conference on Agriculture
for Kharif Campaign 2023-24 on 3rd May at NASC Complex New Delhi.
▪ Union Minister for Environment, Forest and Climate Change, and Labour and Employment Bhupender
Yadav participated in G20 Mega Beach Clean Up campaign at Juhu Beach in Mumbai.
▪ Union Minister for Environment, Forest and Climate Change, Bhupender Yadav on 4 February launched
the ‘Save Wetlands Campaign’.
▪ Union Minister for Science & Technology, Jitendra Singh, who is also the vice president of CSIR
(Council of Scientific & Industrial Research), on 17 December announced the launching of "One Week,
One Lab" countrywide campaign from January 6, 2023.
▪ Meta Launches #DigitalSuraksha campaign in partnership with MeitY for the G20 campaign.
▪ Uttar Pradesh Government Launched ‘Samagra Shiksha Abhiyan’ Campaign.
▪ Bandhan Bank launched the ‘Jahaan Bandhan, Wahaan Trust’ campaign along with the bank’s brand
ambassador Sourav Ganguly.
▪ WhatsApp launches ‘Stay Safe’ campaign to educate on online safety.
▪ The ‘Catch the Rain 2023’ campaign was introduced by President Droupadi Murmu in New Delhi.

Follow us: Official Site, Telegram, Facebook, Instagram, Instamojo 578


Q. Who has been re-appointed as the MD and CEO of Tata Steel for a period of five years?
A) T V Narendran B) Dilip Shanghvi
C) Bhavish Aggarwal D) Karsanbhai Patel
Answer : A
✓ T V Narendran has been re-appointed as the MD and CEO of Tata Steel, for a period of five years
effective 19 September 2023.
✓ He has over 35 years of experience in the mining and metals industry. Narendran joined Tata Steel in
1988.
✓ He was appointed as Managing Director of Tata Steel for India and South East Asia on November 1,
2013.
✓ In October 2017, Tata Steel elevated Narendran as Chief Executive Officer and Managing Director
globally
✓ Tata Steel Limited is an Indian multinational steel-making company, based in Jamshedpur, Jharkhand
and headquartered in Mumbai, Maharashtra. It is a part of the Tata Group
✓ Tata Steel Limited
✓ Founded : 1907
✓ Headquarters : Mumbai, Maharashtra
✓ Founder : Jamsetji Tata
✓ Dorabji Tata
✓ Chairperson : Natarajan Chandrasekaran
✓ Vice Chairman : Noel Tata
✓ MD & CEO : T. V. Narendran
Q. Noor Shekhawat became the first transgender person in which state to be issued a birth certificate with
her gender listed as transgender?
A) Haryana B) Kerala
C) Rajasthan D) Madhya Pradesh
Answer : C
✓ Noor Shekhawat became the first transgender person in Rajasthan to be issued a birth certificate with the
gender recorded as transgender.
✓ In her old birth certificate, her gender was marked as male.
✓ Shekhawat was handed her new birth certificate by municipal and state government officials.
✓ She runs an NGO for the transgender community and also wants to complete her graduation.

RAJASTHAN IN NEWS 2023


▪ Noor Shekhawat became the first transgender person in Rajasthan to be issued a birth certificate with the
gender recorded as transgender.
▪ For the first time in India, the Rajasthan government introduced the Rajasthan Platform Based Gig
Workers Bill, 2023.
▪ Rajasthan Assembly passed a bill to provide minimum guaranteed income to all adult citizens as part of
a welfare package for the state.
▪ Rajasthan government is set to take stringent measures against recruitment exam paper leaks by
introducing a bill in the upcoming assembly session.
▪ Union Minister for Road Transport and Highways Shri Nitin Gadkari inaugurated and laid the
foundation stone of 11 National Highway projects worth Rs 5600 crore in Pratapgarh, Rajasthan.
▪ Rajasthan government has sanctioned Rs 13.48 crore to promote religious tourism at Nagaur, Jaisalmer,
and Alwar regions of the state.

Follow us: Official Site, Telegram, Facebook, Instagram, Instamojo 579


▪ PM Modi Launches Projects Worth Rs 5,500 Crore In Rajasthan, Including 114 Km Long Six-Lane
Udaipur-Shamlaji Section Of NH-48.
▪ Lithium reserves have been discovered in Rajasthan’s Degana following the recent discovery in Jammu
and Kashmir’s Reasi.
▪ Prime Minister Narendra Modi inaugurated the Jal Jan Abhiyan virtually on Abu Road in the Sirohi
district of Rajasthan.
▪ Indian Air Force's MiG-21 crashes near Hanumangarh in Rajasthan.
▪ Chief Minister of Rajasthan, Ashok Gehlot has announced the launch of 19 new districts and 3 new
divisions in Rajasthan.
▪ Presently there are 33 districts in the state.
▪ With the creation of these 19 new districts, the total number of districts in Rajasthan will increase to 52.
Q. Which organization has launched an Action Plan designed to support the implementation of its
ambitious Strategy on Climate Change 2022-2031?
A) World Economic Forum (WEF)
B) Food and Agriculture Organisation (FAO)
C) United Nations Development Programme (UNDP)
D) United Nations Education Scientific & Cultural Organization (UNESCO)
Answer : B
✓ The Food and Agriculture Organisation of the United Nations (FAO) has launched an Action Plan
designed to support the implementation of its ambitious Strategy on Climate Change 2022-2031.
✓ The Strategy, which was endorsed in June 2022 by FAO’s executive body, the Council, envisages
agrifood systems as sustainable, inclusive, resilient and adaptive to climate change.
✓ Global agrifood systems, which encompass the production of food and non-food agricultural products,
as well as their storage, transportation, processing, distribution, marketing, disposal and consumption,
are currently responsible for about a third of total greenhouse gas emissions.
✓ Food and Agriculture Organization (FAO)
✓ Formation : 16 October 1945
✓ Headquarters : Rome Italy
✓ Director-General : Qu Dongyu ( China )
✓ Total Member : 197
Q. Asian Paints has appointed whom as the new chairman of the company?
A) Raviranjan Singh B) R. Seshasayee
C) Mahesh Agarwal D) Ajay Sinha
Answer : B
✓ R Seshasayee has been appointed as the new chairman of Asian Paints.
✓ The board of Asian Paints has approved the appointment of Seshasayee as the chairman of the company
with effect from October 1, 2023.
✓ He will remain on this post till January 22, 2027.
✓ Seshasayee was the Managing Director of Ashok Leyland Limited from 1998 to 2011, Executive Vice-
Chairman from 2011 to 2013 and Non-Executive Vice-Chairman from 2013 to 2016
✓ Asian Paints Ltd
✓ Founded : 1942
✓ Headquarters : Mumbai, Maharashtra
✓ Chairman : R Seshasayee

Follow us: Official Site, Telegram, Facebook, Instagram, Instamojo 580


Q. According to the Henley Passport Index 2023, Which country passport becomes the world's most
powerful passport?
A) Germany B) Austria
C) Singapore D) United Kingdom
Answer : C
✓ Singapore on top and India at 80th position in Henley Passport Index 2023.
✓ Singapore has topped the Henley Passport Index 2023, becoming the most powerful passport in the
world.
✓ Singapore passport holders are granted visa-free access to an impressive 192 out of 227 global travel
destinations.
✓ Three European countries, namely Germany, Italy and Spain, now rank second in the index. Their
passports provide visa-free access to 190 destinations.
✓ Japan, which held the top position for the last five years, is now ranked third in the index. Apart from
this, Austria, Finland, France, Japan, South Korea, Luxembourg, and Sweden are also in third place,
their passport holders can travel to 189 destinations without visa.

Q. What is the rank of Indian passport as per Henley Passport Index 2023?
A) 80th B) 81st
C) 82nd zD) 83rd
Answer : A
✓ India's rank in Henley Passport Index
✓ India has shown remarkable progress on the Henley Passport Index, improving its ranking by 5 places as
compared to the previous year.
✓ India is currently ranked 80th along with Togo and Senegal.
✓ Indian passport holders now enjoy visa-free access to 57 countries, reflecting the country's enhanced
global travel privileges.
✓ Pakistan ranks 100th on the Henley Passport Index and its passport gives holders visa-free access to 33
destinations.
✓ About Henley Passport Index
✓ Introduced by Dr. Christian H. Kaelin, President of Henley & Partners, almost 20 years ago.
✓ It calculates passport rankings based on exclusive and official data from the International Air Transport
Association (IATA).
✓ The index considers the number of destinations that passport holders can travel to without prior visas.
✓ The methodology used by the Henley Passport Index differs from other passport rankings, such as those
published by financial advisors Arton Capital.
✓ The index relies entirely on data from the International Air Transport Association to determine its
rankings.
✓ DON'T CONFUSED
✓ Singapore has world’s most powerful passport in 2023; India ranks 80th: Henley Passport Index.
✓ India’s Rank drop by 6 positions at 144 in The Passport Index 2023 by Arton Capital : UAE top
Q. Who has been appointed as the 25th Director General of the Indian Coast Guard?
A) Ajay Sinha B) Rannvijay Singh
C) Mahesh Sharma D) Rakesh Pal
Answer : D
✓ Rakesh Pal has been appointed as the 25th Director General of the Indian Coast Guard (ICG).

Follow us: Official Site, Telegram, Facebook, Instagram, Instamojo 581


✓ He has the distinction of being the first gunner of ICG.
✓ Shri Pal is an alumnus of the Indian Naval Academy and had joined the Indian Coast Guard in January
1989.
✓ Mr. Pal has done Electro-Optics Fire Control Solution courses from the United Kingdom.
✓ Rakesh Pal has served the nation in various capacities in his illustrious career spanning over 34 years.
✓ Prominent among them are the post of Commander Coast Guard Region (North West), Gandhinagar,
Deputy Director General (Policy and Plans), and Additional Director General, Coast Guard at Coast
Guard Headquarters in New Delhi.
✓ Shri Pal was awarded the Tatrakshak Medal in 2013 and President Tatrakshak Medal in 2018 for his
distinguished service
✓ Indian Coast Guard (ICG)
✓ Founded : 1977
✓ Headquarters : New Delhi
✓ 24th Director General : Virender Singh Pathania
✓ 25th Director General : Rakesh Pal
Q. Who has been elected as the new Deputy Chairman of Rajya Sabha?
A) Smriti Irani B) Kuldeep Singh
C) Abhishek Awasthi D) S Phangnon Konyak
Answer : D
✓ S Phangnon Konyak, the lone MP from Nagaland in the Upper House of Indian Parliament, has been
nominated as a Deputy Chairman of the Rajya Sabha.
✓ Under the Rules of Procedure and Conduct of Business in Rajya Sabha, Rajya Sabha Chairman Jagdeep
Dhankhar nominated Phangnon to the panel of Deputy Chairmen.
✓ Phangnon is a resident of Oting village in Mon district of Nagaland
Q. Which State government has signed a MoU with London-Based 'OneWeb Company' to set up a 'Satellite
Network Portal Site'?
A) Karnataka B) Madhya Pradesh
C) Gujarat D) Maharashtra
Answer : C
✓ Gujarat signed a Memorandum of Understanding (MoU) on 19th July 2023 here between London-based
company "OneWeb" and the Gujarat Government’s Science and Technology Department regarding the
establishment of a ‘Satellite Network Portal Site’.
✓ It will offer affordable connectivity to the sectors including the government and offer continuous and
secure satellite internet access at affordable rates nationwide.
✓ According to a note shared by the State government, OneWeb will establish one of the two private
‘satellite network portal sites’ in Mehsana district of Gujarat.
✓ This satellite network portal site, to be initiated in Jotana taluka of Mehsana district, will provide high-
speed, low-latency, and affordable connectivity to the government, businesses, consumers, schools, and
more.
✓ The satellite network portal to be established in Katosan and Tejpura in Jotana taluka of Gujarat is
scheduled to launch in 2023, with an estimated cost of over Rs. 100 crores for its Phase-1. It is also
expected to create approximately 500 direct and indirect employment opportunities in the state.

Follow us: Official Site, Telegram, Facebook, Instagram, Instamojo 582


Q. India has donated ____ million to the United Nations to support the promotion of the Hindi language.
A) $0.5 million B) $1 million
C) $1.7 million D) $2.5 million
Answer : B
✓ India has donated $1 million to the UN for the promotion of Hindi.
✓ India’s Commitment to Promote Hindi at the UN:
✓ India’s Permanent Representative Ruchira Kamboj, who gave her the cheque last Friday, said in a
statement that New Delhi “will continue to promote the use of the Hindi language in the United
Nations”.
✓ She highlighted the efforts of the United Nations to mainstream and consolidate news and multimedia
content in Hindi language have been appreciated in India and in countries where a Hindi-speaking
population resides.
✓ Hindi was introduced in the UN by former Prime Minister Atal Bihari Vajpayee who became in 1977 the
first Indian official when he was the external affairs minister to speak at the UN in the language
Q. Which Company has bagged orders worth Rs 800 crore from the Indian Army?
A) Tata Motors B) BharatBenz
C) Ashok Leyland D) Mahindra & Mahindra
Answer : C
✓ Mumbai, Commercial vehicle maker Ashok Leyland on has bagged orders worth Rs 800 crore from the
Indian Army.
✓ The contracts awarded also include the procurement of the Field Artillery Tractor (FAT 4x4) and the
Gun Towing Vehicle (GTV 6x6), Ashok Leyland said in a statement.
Q. Which Indian Building Surpasses Pentagon To Become World's Largest Office?
A) Mumbai Diamond Bourse B) Surat Diamond Bourse
C) Delhi Diamond Bourse D) Bengaluru Diamond Bourse
Answer : B
✓ The Surat Diamond Bourse has also surpassed the Pentagon office complex of the US Department of
Defense as the world's largest office complex.
✓ The full functioning of the Surat Diamond Bourse will give a boost to the dynamism and growth of the
diamond industry.
✓ The exchange will serve as a hub of trade, innovation and collaboration, boosting the country's economy
and creating over 1.5 lakh jobs.
✓ Inaugurated by the Prime Minister on 21 November:
✓ Surat Diamond Bourse will start functioning from November 21, 2023. The official inauguration will be
done in the presence of Prime Minister Narendra Modi.
✓ 90% of the world's diamonds get their final form in Surat:
✓ Surat is famous for producing 90 percent of the world's diamonds. According to the Union Ministry of
Finance, 14 out of 15 diamonds in the world are finally cut in Surat.
✓ Borse Cost:
✓ This office complex has been built at a cost of more than Rs 3,000 crore. This Diamond Bourse has been
constructed in 6.7 million square feet.
✓ Chief Architect:
✓ The office complex was designed by Manit Rastogi, founding partner of architecture firm Morphogenesis,
who was commissioned following an international design competition.

Follow us: Official Site, Telegram, Facebook, Instagram, Instamojo 583


✓ Complex Structure:
✓ The Surat Diamond Bourse consists of nine interconnected buildings connected by 24 feet wide spine
corridors. Each of these buildings is 15 floors high including the ground floor.
✓ Each office ranges from 300 to 75,000 square feet in size.
✓ There is also a 2 million square feet parking area in the basement of the exchange.
✓ Largest group of professionals in the world:
✓ The complex of 4,500 diamond trading offices is spread over 35.54 acres and can house 67,000 diamond
professionals including cutters, polishers and traders.
✓ IGBC Accredited:
✓ The Diamond Bourse has been designed to be eco-friendly and has also been given a platinum ranking
from the Indian Green Building Council (IGBC).
✓ All the offices overlook the landscaped court, designed on the concept of "panchtatva" – the five elements,
air, water, fire, earth and sky – which is approximately 200 feet wide and 300 feet long.
Q. India Nuclear power capacity has set to increase from over 7000 Mega Watt to over 22480 Mega Watt
by which year?
A) 2025 B) 2027
C) 2029 D) 2031
Answer : D
✓ Minister of State for Atomic Energy and Space "Dr Jitendra Singh" 20th July 2023 said that nuclear
power capacity has set to increase from over 7000 Mega Watt to over 22,480 Mega Watt by 2031
✓ The present installed nuclear power capacity in the country is 7480 Mega Watt comprising 23 nuclear
power reactors
✓ The share of nuclear power in the total electricity generation in the country was about 2.8 percent in
2022-23.
Q. Government Gujarat has inaugurated the 'One District One Product' Wall at Garvi Gujarat Bhawan in
Which City?
A) Mumbai B) Pune
C) New Delhi D) Lucknow
Answer : C
✓ DPIIT and Government of Gujarat jointly launch 'One District One Product' wall.
✓ One District One Product (ODOP) programme, an initiative of the Department for Promotion of
Industry and Internal Trade (DPIIT), Ministry of Commerce and Industry collaborated with the
Government of Gujarat to promote indigenous crafts and artisans of the state.
✓ Gujarat, with its 33 districts, offers a wide range of unique products, including traditional crafts such as
Gamathi block prints and Mata-ni-Pachedi, as well as agricultural goods such as groundnut and cumin.
✓ The collaboration between ODOP and the Government of Gujarat includes product tagging and story
cards to enhance the promotion and recognition of Gujarat's unique products.
✓ The partnership aims to increase the visibility of Gujarat products at the national level and drive
consumers towards the emporia, thereby boosting sales.
✓ Specific interventions have been made to enhance the market presence of certain products in Gujarat.
For example, Sujani Handloom, Jamnagari Bandhini, and Patan Patola have been onboarded on the
Government e-Marketplace (GeM), and support from the National Institute of Design (NID) workshop
has been provided for Agate Stone in Khambat district and Sujani from Bharuch district.

Follow us: Official Site, Telegram, Facebook, Instagram, Instamojo 584


Q. India has signed an agreement with which country for joint development of semiconductor ecosystem?
A) Brazil B) Japan
C) Argentina D) Germany
Answer : B
✓ India has signed an agreement with Japan for joint development of semiconductor ecosystem and
maintaining the resilience of the global supply chain.
✓ Japan has become the second Quad partner after the United States.
✓ With about 100 semiconductor manufacturing plants, Japan is among the top five countries with a
semiconductor ecosystem.
Q. In which state will the 69th edition of the Filmfare Awards be organized in the year 2024?
A) Uttar Pradesh B) Maharashtra
C) Gujarat D) Goa
Answer : C
✓ In the year 2024, the 69th edition of the Filmfare Awards will be organized in Gujarat, for which
preparations have been started.
✓ This is the first time that the Filmfare Awards will be organized in Gujarat.
✓ Its organization will promote film destination and tourism in the state.
✓ Filmfare Awards is an annual award ceremony, first organized in 1954

Q. The Chachin grazing festival was celebrated with great enthusiasm by local graziers in which state?
A) Assam B) Nagaland
C) Sikkim D) Arunachal Pradesh
Answer : D
✓ Chachin Grazing Festival celebrated in Arunachal Pradesh
✓ The Chachin grazing festival was celebrated with great enthusiasm by local graziers in the Tawang region
near Bumla Pass, Arunachal Pradesh.
✓ The festival featured a medical camp for aiding local graziers and a veterinary camp for their yaks.
✓ The two-day event held at Chachin saw enthusiastic participation from Graziers from all over Tawang
region.
✓ Chachin and other traditional grazing areas, near Bumla Pass have historically served as the backbone of
the local Monpa lifestyle that is largely dependent on nomadic herding – a primitive form of subsistence
farming – as means of livelihood.
Q. Which state government has launched 'Sashakt Mahila Loan Yojana', a collateral-free loan scheme for
women?
A) Uttarakhand B) Himachal Pradesh
C) Uttar Pradesh D) Madhya Pradesh
Answer : B
✓ Himachal Pradesh Chief Minister Sukhwinder Singh Sukhu launched 'Sashakt Mahila Loan Yojana', a
collateral-free loan scheme for women.
✓ The initiative has been launched by the Himachal Pradesh State Cooperative Bank (HPSCB) and aims
to help women fulfil their daily needs to realise their entrepreneurial aspirations, engage in livelihood
activities and uplift their families.

Follow us: Official Site, Telegram, Facebook, Instagram, Instamojo 585


✓ 'Sashakt Mahila Loan Yojana' will provide financial assistance to women for self-employment and
livelihood activities, which will contribute to their economic empowerment and financial independence.
✓ Through this scheme, women can improve their socio-economic status, which will have a positive impact
on their lives and community.
✓ The Himachal Pradesh State Cooperative Bank offers three credit options under this scheme: Rs 21,000,
Rs 51,000 and Rs 1,01,000, all at a relatively low interest rate of 8.51 per cent.
Q. Tata Motors-owned Jaguar Land Rover (JLR) has appointed whom as its Chief Executive Officer for a
3 Year term?
A) Adrian Mardell B) Ola Kallenius
C) Oliver Zipse D) Zak Brown
Answer : A
✓ Adrian Mardell has been appointed as the Chief Executive Officer of Jaguar Land Rover (JLR) for a
period of three years.
✓ The appointment was made by Tata Motors, which owns JLR.
✓ Prior to becoming CEO, Mardell served as interim CEO through November 16, 2022.
✓ With the appointment of Adrian Mardell as CEO, Richard Molyneux was announced as the new Chief
Financial Officer of JLR.
✓ Richard Molyneux was previously the Acting Chief Financial Officer from December 12, 2022. Prior to
this, he held the position of Finance Director of Operations at Jaguar Land Rover for six years.
✓ Adrian Mardell's journey with Jaguar Land Rover began in 1990, and he held various financial positions
within the company until 2008.
✓ About Tata Motors Limited
✓ It is an Indian multinational automotive manufacturing company.
✓ It operates as a part of the Tata Group conglomerate.
✓ Tata Motors produces a wide range of vehicles including passenger cars, trucks, vans, coaches and buses.
✓ Establishment - 1945
✓ Founder - Jehangir Ratanji Dadabhoy Tata
✓ Headquarters - Mumbai, Maharashtra
✓ Chairman - Natarajan Chandrasekaran

Q. Who has received the prestigious 'Eni Award' for scientific research in energy and environment?
A) Jhumpa Adhikari B) Rajdip Bandyopadhyaya
C) Avijit Chatterjee D) Thalappil Pradeep
Answer : D
✓ Professor Thalappil Pradeep of IIT Madras has received the prestigious 'Eni Award' for scientific research
in energy and environment.
✓ The Eni Award was established in 2007 and is a globally recognized honour.
✓ Professor Pradeep's research focuses on cost-effective and clean water solutions using advanced
nanoscale materials.
✓ His groundbreaking work led to the discovery of sustainable materials that remove toxic pollutants from
water.
✓ These water purification technologies benefit 1.3 million people in India every day.
✓ The President of Italy will confer the Eni Award on Professor T. Pradeep.
✓ About the Eni Award
✓ They are awarded in three categories: Energy Transitions, Energy Frontiers and Advanced
Environmental Solutions.

Follow us: Official Site, Telegram, Facebook, Instagram, Instamojo 586


✓ Yu Huang from the University of California (Los Angeles, USA) and Jeffrey R. Long from the University
of California (Berkeley, USA) were awarded in the Energy Transition category.
✓ Matthew Rosensky from the University of Liverpool (UK) received the award in the Energy Frontiers
category.
✓ The primary purpose of the Eni Award is to promote and recognize unprecedented progress in energy
efficiency, renewable energy, decarbonization and environmental protection.
✓ The purpose of this award is to inspire and support the work of young researchers in these important
areas
Q. Who has set the Guinness world record for the fastest hit by a male player in badminton, recording a
mindboggling 565 km/h with his smash?
A) Srikanth Kidambi B) Chirag Shetty
C) Lakshya Sen D) Satwiksairaj Rankireddy
Answer : D
✓ Indian badminton player Satwiksairaj Rankireddy created a new Guinness World Record for the fastest
hit in badminton during the Korea Open 2023.
✓ Satwiksairaj Rankireddy's smash reached an incredible speed of 565 km/h, breaking a decade-old record
previously held by Malaysian player Tan Boon Heong, who clocked 493 km/h.
✓ The record-breaking event took place at the Yonex factory gym in Soka, Saitama, Japan, and was
officially verified by Guinness World Records under controlled conditions.
✓ Satwik's smash also surpassed the top speed of 372.6 km/h achieved by a Formula 1 car.
✓ In the women's section, Malaysia's Tan Perli set the Guinness World Record for the fastest women's
badminton hit with a remarkable speed of 438 km/h.
✓ In Korea Open 2023, Satwiksairaj Rankireddy and his doubles partner Chirag Shetty defeated Thailand's
Supak Jomkoh and Kittinpong Kedren with a score of 21-16, 21-14 to enter the pre-quarterfinals.
✓ Among all the Indian participants in the Korea Open 2023, Satwik and Chirag were the only players to
enter the pre-quarterfinals.
✓ Among all the Indian participants in the Korea Open 2023, Satwik and Chirag were the only players to
enter the pre-quarterfinals.
✓ Korea Open 2023 is scheduled to take place from 18 July 2023 to 23 July 2023. The host city of Korea
Open 2023 is Yeosu, South Korea.
✓ About South Korea
✓ Capital - Seoul
✓ Prime Minister - Han Duck-soo
✓ President - Yoon Suk Yeol
Satwiksairaj Rankireddy IN NEWS 2023
▪ Indian badminton player Satwiksairaj Rankireddy created a new Guinness World Record for the fastest
hit in badminton during the Korea Open 2023.
▪ Satwiksairaj Rankireddy and Chirag Shetty created history by becoming the first Indian men's doubles
pair to win a gold medal at the Badminton Asia Championships.
▪ Satwiksairaj Rankireddy and Chirag Shetty became the first Indian pair to win the doubles title at a BWF
Super 1000 event at the Indonesia Open 2023.
Q. Who has presented the Bhoomi Samman awards to nine state secretaries and 68 district collectors for
their achievements in the implementation of the Digital India Land Records Modernization Programme?
A) Draupadi Murmu B) Narendra Modi
C) Rajnath Singh D) Amit Shah

Follow us: Official Site, Telegram, Facebook, Instagram, Instamojo 587


Answer : A
✓ Indian President Draupadi Murmu presented the "Bhoomi Samman" 2023 at a function organized by the
Union Ministry of Rural Development at Vigyan Bhawan in New Delhi.
✓ The awards were received by the State Secretaries and District Collectors along with their teams who
have shown excellence in achieving completion of major components of the Digital India Land Records
Modernization Program (DILRMP).
✓ Honor to the secretaries of nine states and officers of 68 districts:
✓ Officers of the team of total 68 districts including nine secretaries of Odisha, Madhya Pradesh, Gujarat,
Bihar, Assam, Chhattisgarh, Jharkhand, Tripura and West Bengal were honored at the Bhoomi
Samman-2023 function held at Vigyan Bhawan, New Delhi.
✓ Most awards to Odisha and Madhya Pradesh:
✓ Among these, 57 officers from 19 districts of Odisha, 35 officers from 15 districts of Madhya Pradesh,
who received maximum awards, were awarded Bhoomi Samman by the President
✓ Digital India Land Records Modernization Program (DILRMP):
✓ DILRMP was launched by the Union Ministry of Panchayati Raj in August 2008 for the digitization of
land records
✓ Main components of DILRMP:
✓ Change of land ownership;
✓ Digitization of maps and integration of curriculum and spatial data;
✓ Survey/Re-survey and
✓ Computerization of all land records including original land records.
✓ Land Parcel Identification Number:
✓ Under the Digital India Land Information Management System, a unique Land Parcel Identification
Number is being provided, which can be used like an Aadhaar card.
✓ This number will be helpful in making and implementing new welfare schemes along with proper use of
land.
✓ Linking e-courts with the land records and registration data-base would have many benefits.
✓ Digitization of land will bring transparency in administration.
✓ Unethical and illegal activities related to land will be curbed.
✓ Now getting land related information in a free and convenient way will have many other benefits.
Q. Tata Sons planned to build about Rs 42,300 crore electric vehicle battery factory in which country?
A) Israel B) United Kingdom
C) United States D) Japan
Answer : B
✓ Tata Sons plans to build a four-billion-pound (about Rs 42,300 crore) electric vehicle battery factory in
the UK, which will be its first gigafactory outside India.
✓ Last month, it announced plans to establish a Rs 13,000-crore battery manufacturing facility in Gujarat.
✓ In Britain, the Tata group is working on plans to make batteries for Jaguar Land Rover (JLR) as well as
other vehicle companies.
✓ The factory will be located in Bridgwater, Somerset, England.
✓ The UK plant, which will cater to the requirements of Tata Group company Jaguar Land Rover (JLR),
will create up to 4,000 jobs and have an annual output of 40-gigawatt hours. Production at the gigafactory
will start in 2026.
Q. Who has been appointed the Director General of the Railway Protection Force (RPF)?
A) Rishav Singh B) Abhishek Kumar
C) Manoj Yadava D) Rajesh Verma

Follow us: Official Site, Telegram, Facebook, Instagram, Instamojo 588


Answer : C
✓ Senior IPS officer Manoj Yadava has been appointed the Director General of the Railway Protection
Force (RPF).
✓ Yadava, a 1988 batch Indian Police Service (IPS) officer of the Haryana cadre, will succeed Sanjay
Chander who retires on July 31.
Q. When is the World Brain Day observed globally?
A) July 22 B) July 21
C) July 20 D) July 19
Answer : A
✓ The World Brain Day is celebrated on July 22 every year to raise awareness and promote advocacy
related to brain health.
✓ The day is organised by the World Federation of Neurology (WFN).
✓ It was observed for the first time in 2014 and 2021 is the eighth World Brain Day.
✓ The theme of 2023 World Brain Day is “brain health and disability”
Q. Which IIT has signed an MoU with the Indian Council of World Affairs (ICWA) to build capacity/skills
in international affairs and foreign policy?
A) IIT Madras B) IIT Kanpur
C) IIT Guwahati D) IIT Bombay
Answer: C
✓ The Indian Institute of Technology Guwahati has signed MoU with the Indian Council of World Affairs
(ICWA) to build capacity/skills in international affairs and foreign policy.
✓ The collaboration aims to expand awareness and knowledge of international affairs and Indian foreign
policy.
✓ The agreement will be valid for a period of 3 years.
Q. India has gifted its in-service missile corvette “INS Kirpan” to which country to robust the strategic
partnership between the two nations, especially in the maritime domain?
A) Maldives B) Vietnam
C) Indonesia D) Malaysia
Answer : B
✓ India has gifted its in-service missile corvette INS Kirpan to Vietnam to robust the strategic partnership
between the two nations, especially in the maritime domain.
✓ Navy Chief Admiral R Hari Kumar presided over the ceremony at Cam Ranh where the frontline warship
was handed over to Vietnam People’s Navy.
✓ It is for the first time that India gifted a fully operational corvette to any friendly foreign country.
✓ INS Kirpan, since its commissioning in 1991, has been an integral part of Indian Navy’s Eastern Fleet
and has participated in many operations over the last 32 years.
✓ Manned by about 12 officers and 100 sailors, the ship is 90 metres long and 10.45 metres in width with
maximum displacement of 1,450 tons.
✓ The transfer of INS Kirpan from the Indian Navy to Vietnam People’s Navy symbolizes the status of
Indian Navy of being the ‘preferred security partner in the Indian Ocean Region.

Follow us: Official Site, Telegram, Facebook, Instagram, Instamojo 589


Q. Who is the author of the book titled “As the wheel turns”?
A) Amrita Mahale B) Ranjit Pratap
C) Judy Balan D) Kanishk Tharoor
Answer : B
✓ Former Vice President of India M Venkaiah Naidu has released a book named ” As the wheel turns ” in
Chennai, Tamilnadu.
✓ The book is authored by Rayala Corporation CEO Ranjit Pratap.
✓ The book chronicles Ranjit Pratap’s triumphs and challenges in the corporate world.
✓ The book’s title is a metaphor for the cyclical nature of life and business, highlighting the importance of
adaptability and resilience in the face of change.
✓ Rayala Corporation was founded in 1948 as an automotive spare parts and office equipment product
company.
Q. India and which country have signed an MoU on cooperation in renewable energy and another one for
economic development projects?
A) Maldives B) Bhutan
C) Sri Lanka D) Nepal
Answer : C
✓ India and Sri Lanka have signed an MoU on cooperation in renewable energy and another one for
economic development projects in the Trincomalee district of Sri Lanka.
✓ This year both countries are celebrating the 75th anniversary of diplomatic relations. Apart from this, the
Tamil community of Indian origin is completing 200 years of its arrival in Sri Lanka.

Q. Who has been roped in as the brand ambassador for the ICC World Cup 2023?
A) Akshay Kumar
B) Shah Rukh Khan
C) Salman Khan
D) Aamir Khan
Answer : B
✓ Bollywood actor Shah Rukh Khan was appointed as the brand ambassador of ICC World Cup 2023.
✓ He launched the World Cup 2023 campaign ‘It takes one day’ in his iconic voiceover.
✓ The ICC World Cup 2023 is scheduled to be held in India from 5 October to 19 November 2023.
✓ A total of 10 teams will participate in the tournament, with India, Afghanistan, Australia, England,
Bangladesh, New Zealand, Pakistan and South Africa qualifying directly based on their performance in
the 2020–2023 ICC Cricket World Cup Super League.
✓ The 2023 ICC Men’s Cricket World Cup will be the 13th edition of the tournament.
✓ India won the cricket World Cup title 2 times (1983 and 2011).
✓ Australia won the Cricket World Cup 5 times, no other team has done this yet.
✓ India won the cricket World Cup title 2 times (1983 and 2011).
✓ India is the first country to win the 50 overs cricket world cup at home.
✓ West Indies won the first two cricket World Cups in 1975 and 1979.
✓ England is the only country that has hosted the highest number of World Cup tournaments – 4 times
✓ 1983 India vs West Indies
✓ 2011 India vs Sri Lanka

Follow us: Official Site, Telegram, Facebook, Instagram, Instamojo 590


Q. When is the Income Tax Day observed by the CBDT?
A) 22 July B) 23 July
C) 24 July D) 21 July
Answer : C
✓ The Central Board of Direct Taxes (CBDT) observed the 163rd Income Tax Day (also known as Aaykar
Diwas) on 24 July, 2023.
✓ In India, the Income Tax Day is celebrated every year on July 24, as it was on 24 July 1980 that Income
Tax was introduced for the first time in India by Sir James Wilson.
✓ The purpose of this tax was to compensate for the losses incurred by the British regime during the first
war of independence against British Rule in 1857.
✓ July 24 was first celebrated as the Income Tax day in 2010.

Q. Which country's highest honor 'Grand Cross of the Legion of Honour' was awarded to PM Modi?
A) Germany B) France
C) Australia D) Brazil
Answer : B
✓ Grand Cross of the Legion of Honor awarded to Prime Minister Modi.
✓ Prime Minister Narendra Modi was awarded the highest French award 'Grand Cross of the Legion of
Honour' by the President of the Republic of France, Emmanuel Macron on 13 July 2023.
✓ Prime Minister Modi was honored at an award ceremony held at the Elysee Palace in Paris.
✓ The Prime Minister thanked President Macron on behalf of the people of India for this distinguished
honour.
✓ Prime Minister Modi is on an official visit to France and the United Arab Emirates (UAE) from 13 to 15
July 2023.
✓ This is the sixth visit of Prime Minister Modi to France.
✓ The Prime Minister is the Chief Guest at the Bastille Day Parade on 14 July 2023, in which a contingent
of the Indian Armed Forces from the three Services is also participating.
✓ This year also marks the 25th anniversary of the India-France Strategic Partnership and the Prime
Minister's visit will provide an opportunity to chart out a roadmap for the future of the partnership in
various areas such as strategic, cultural, scientific, educational and economic cooperation.
Q. An Odia-based news station, Odisha TV has unveiled India’s first regional AI news anchor. What is the
name of this AI news anchor?
A) Piya B) Lisa
C) Diya D) Sofia
Answer : B
✓ India welcomes its first regional Al news anchor, ‘Lisa’.
✓ In a significant milestone for the AI industry, Odisha TV, an Odia-based news station, has unveiled
“Lisa,” India’s first regional AI news anchor.
✓ Lisa’s introduction marks a groundbreaking moment in TV broadcasting and journalism, with the
potential to revolutionize the industry.
✓ Lisa possesses the remarkable ability to speak multiple languages, including Odia, English, and others.
✓ This unveiling follows a similar development earlier in March when the India Today Group introduced
their own AI news anchor named Sana. Described as “bright, gorgeous, ageless, and tireless,” Sana
boasted multilingual proficiency and the ability to be controlled effortlessly

Follow us: Official Site, Telegram, Facebook, Instagram, Instamojo 591


Q. Defence Minister Rajnath Singh has inaugurated the regional office of Hindustan Aeronautics Limited
(HAL) in which country to facilitate close defence industrial collaboration between both countries?
A) Indonesia B) Malaysia
C) Vietnam D) Maldives
Answer : B
✓ Defence Minister Rajnath Singh has inaugurated the regional office of Hindustan Aeronautics Limited
(HAL) in Kuala Lumpur to facilitate close defence industrial collaboration between India and Malaysia.
✓ It will also serve as a hub for the HAL’s engagement with the wider South-East Asian region and act as
a window for other Indian defence PSUs.
✓ Malaysia has shown interest in the acquisition of Tejas fighter jets manufactured by HAL.
✓ Malaysia is home to the second-largest number of Persons of Indian Origin (PIO)
✓ About Malaysia
✓ Prime Minister - Anwar Ibrahim
✓ Capital - Kuala Lumpur
✓ Currency - Malaysian ringgit

Q. Which day of the year is observed as the World Youth Skills Day (WYSD)?
A) July 12 B) July 13
C) July 14 D) July 15
Answer : D
✓ The World Youth Skills Day (WYSD) is observed every year on July 15 as a day to recognize the strategic
importance of equipping young people with skills for employment, decent work and entrepreneurship,
and to highlight the crucial role of skilled youth in addressing current and future global challenges.
✓ The United Nations General Assembly declared 15 July as World Youth Skills Day in November 2014.
✓ The theme for 2023 is skilling teachers, trainers, and youth for a transformative future
Q. Who has become the first Arab country to receive observer status in the Asia/Pacific Group (APG) on
Money Laundering?
A) Kuwait B) Qatar
C) Saudi Arabia D) UAE
Answer : D
✓ The United Arab Emirates (UAE) has become the first Arab country to receive observer status in the
Asia/Pacific Group (APG) on Money Laundering, a regional body similar to the Financial Action Task
Force (FATF).
✓ The UAE is participating as an observer at the plenary session of the APG, which is currently taking
place in Vancouver, Canada.
✓ Obtaining observer status reflects the UAE's commitment to international cooperation in combating
money laundering and the financing of terrorism.
✓ Observer status is awarded to countries that actively and cooperatively combat financial crime, and the
UAE has demonstrated this through its participation in organizations such as MENAFATF and other
multilateral fora.
✓ The UAE delegation attending the APG plenary session is led by Hamid Al Zaabi, Director-General of
the Executive Office for Anti-Money Laundering and Counter-Terrorism Financing (EO AML/CTF).
✓ The delegation includes representatives from the UAE Financial Intelligence Unit (FIU) and other
government officials.

Follow us: Official Site, Telegram, Facebook, Instagram, Instamojo 592


Q. Which state government has launched "Gajah Kotha" campaign to address the growing problem of
Human-Elephant Conflict (HEC)?
A) Odisha B) Assam
C) Haryana D) Bihar
Answer : B
✓ Assam government launched "Gajah Kotha" campaign.
✓ Assam launched "Gajah Kotha" campaign to address the growing problem of Human-Elephant Conflict
(HEC).
✓ The "Gajah Kotha" campaign involves over 1,200 participants and aims to promote coexistence between
humans and elephants.
✓ The focus of the campaign is on HEC affected villages in Eastern Assam.
✓ The main objective is to educate the residents about elephant behaviour, ecology and the cultural
importance of elephants in the region.
✓ The launch of the "Gajah Kotha" campaign reflects Assam's commitment to promote harmonious co-
existence between humans and elephants

Q. Who is the author of the book titled “Prism: The Ancestral Abode of Rainbow”?
A) Dom Moraes B) Anuja Chauhan
C) Vinod Mankara D) Anita Desai
Answer : C
✓ A new book 'Prism: The Ancestral Abode of Rainbow' by Vinod Mankara released.
✓ National Award winning filmmaker-writer Vinod Mankara's new book titled "Prism: The Ancestral
Abode of Rainbow" was released from the rocket launchpad of Satish Dhawan Space Center (SDSC) in
Sriharikota, Andhra Pradesh.
✓ The book release coincided with preparations for India's upcoming moon mission, Chandrayaan-3, at
SDSC-SHAR.
✓ ISRO Chairman S Somnath released the book by handing it over to Vikram Sarabhai Space Center
(VSSC) Director S Unnikrishnan Nair.
✓ The book launch was attended by several prominent personalities including SDSC-SHAR Director A
Rajarajan, LPSC Director V Narayanan, former ISRO Director K Radhakrishnan, Lipi Akbar,
Managing Director of Lipi Prakashan (publisher of the book) and many other space scientists.
✓ The book, "Prism: The Ancestral Abode of the Rainbow", is a collection of science articles, providing
readers with unique insights and perspectives.

Q. Which country has launched the world’s first methane-liquid oxygen space rocket into orbit?
A) Russia B) China
C) United States D) Japan
Answer : B
✓ China launches world’s first methane-fuelled space rocket.
✓ A private Chinese company has launched the world’s first methane-liquid oxygen space rocket into orbit.
✓ The Zhuque-2 carrier rocket was launched from the Jiuquan Satellite Launch Center in northwest China.
Q. In which state of India the annual 'Bonalu Festival' was organized?
A) Telangana B) Madhya Pradesh
C) Assam D) Sikkim

Follow us: Official Site, Telegram, Facebook, Instagram, Instamojo 593


Answer : A
✓ The annual Bonalu Festival was organized in Telangana's capital Hyderabad, especially in parts of the
old city.
✓ This festival is celebrated every year during the Hindu lunar month of Ashadha.
✓ Bonalu is an annual festival celebrated in parts of Telangana including Hyderabad and Secunderabad.
✓ The festival is dedicated to Goddess Mahakali, who is believed to be an incarnation of Goddess Durga.
✓ The word 'Bonam' in Telugu means 'food' or 'feast'.
Q. The 2023 Exercise Nomadic Elephant-XV is a joint military exercise conducted between Indian Army
and the Army of____
A) Mauritius B) Mongolia
C) Kazakhstan D) United States
Answer : B
✓ The 15th edition of India-Mongolia joint military exercise "Nomadic Elephant-2023" is scheduled to be
held from 17 to 31 July 2023 at Ulaanbaatar, Mongolia.
✓ Exercise Nomadic Elephant is an annual training program of India with Mongolia which is conducted
alternately in Mongolia and India.
✓ Earlier the military exercise program was organized at Special Forces Training School, Bakloh in October
2019.
✓ Troops from Mongolian Armed Forces Unit 084 and Indian troops from the Jammu and Kashmir Light
Infantry Regiment will participate in the 15th edition of the military exercise
✓ About Mongolia
✓ It is located in North Central Asia.
✓ Capital: Ulaanbaatar
✓ Currency: Tugrik
✓ President: Ukhnaagiin Khurelsukh
Q. Rare Bird 'Jerdon's Babbler' Spotted In Which Tiger Reserve in Uttar Pradesh?
A) Pilibhit Tiger Reserve B) Dudhwa Tiger Reserve
C) Amangarh Tiger Reserve D) Palamau Tiger Reserve
Answer : B
✓ Rare bird 'Jerdon's Babbler' spotted in Dudhwa Tiger Reserve, Uttar Pradesh.
✓ A rare and globally endangered bird species called 'Jerdon's Babbler' was recently spotted in the
grasslands of Dudhwa Tiger Reserve (DTR) in Uttar Pradesh.
✓ According to surveyors, more than 95% of 'Jerdon's Babbler' in India are from Assam and Arunachal
Pradesh.
✓ The Jordanian babbler lives in pairs in small flocks in tall grasslands.
✓ This globally threatened bird has been listed as 'vulnerable' by the International Union for Conservation
of Nature (IUCN) since 1994.
✓ Earlier, the Jordan Babbler was found along the Sutlej River in Haryana and Punjab. However, due to
habitat loss, this species is now mainly found in Assam and Arunachal Pradesh.
✓ About 30% of the global population of the Jordanian babbler is found in India
Q. Which Countries are signed historic Memorandum of Understanding on Local Currency Settlement
(LCS) system?
A) India and UAE B) India and UK
C) India and France D) India and USA

Follow us: Official Site, Telegram, Facebook, Instagram, Instamojo 594


Answer : A
✓ India and UAE signed historic Memorandum of Understanding on Local Currency Settlement (LCS)
system.
✓ This groundbreaking agreement, signed between the Reserve Bank of India and the Central Bank of the
UAE, aims to revolutionize cross-border transactions between the two nations.
✓ The LCS system allows for the use of the Indian Rupee and UAE Dirham in bilateral trade, marking
India's first-ever LCS arrangement.

Q. In which city IIT Delhi is going to open its first global campus?
A) Dubai B) Muscat
C) Havana D) Abu Dhabi
Answer : A
✓ IIT Delhi is going to set up its first Global Campus in Abu Dhabi (UAE). Bachelor's degree courses are
also expected to start from September 2024.
✓ The MoU related to this was signed in Abu Dhabi in the presence of Prime Minister Narendra Modi.
✓ Earlier, IIT Madras also announced setting up its first global campus in the African country of Tanzania.
Q. Which edition of the India Mobile Congress (IMC) will be held from October 27 -29 at Pragati Maidan
in New Delhi this year?
A) 8th B) 7th
C) 5th D) 4th
Answer : B
✓ The seventh edition of the India Mobile Congress (IMC), co-hosted by the Department of
Telecommunications (DoT) and Cellular Operators Association of India (COAI) will be held from
October 27 -29 at Pragati Maidan in New Delhi this year.
✓ The event will be themed around “Global Digital Innovation” and will showcase 6G, advancements in
5G networks, the increasing use of artificial intelligence (AI) in telecommunications, edge computing,
and developments related to India Stack.
✓ IMC 2023 would also highlight related technology domains such as broadcast technology, satellite
communication, manufacture of semiconductors.
✓ The India Mobile Congress 2023 will signify the vision of our Prime Minister, Narendra Modi, to
promote India’s pivotal role in the digital revolution impacting every sector globally. This includes
pioneering breakthroughs in 5G, 6G, broadcasting, satellite, semiconductor, drone, devices, and green
technologies
Q. Which of the following company has been awarded the ‘Golden Peacock Environment Management
Award’?
A) TATA Power B) JSW Energy
C) NTPC Limited D) Adani Transmission Limited
Answer : D
✓ Golden Peacock' Environment Management Award 2023 to Adani Transmission Limited.
✓ Indian company Adani Transmission Limited (ATL) has been awarded the 'Golden Peacock
Environment Management Award' by the 'Institute of Directors' in the 'Power Transmission Sector'.
✓ This year an assessment group comprising environment, health and safety, energy and climate change
experts assessed more than 520 applications.

Follow us: Official Site, Telegram, Facebook, Instagram, Instamojo 595


✓ These applications were approved by Justice M.N. The review was done by an eminent jury committee
headed by Venkatachaliah.
✓ SAIL has been the winner of this award for three consecutive years.
✓ Golden Peacock Environment Award:
✓ The award is given for reducing its ecological footprint through programs such as zero waste in landfills,
single-use plastic free, water-positive operations, promoting renewable energy integration and
implementation of best-in-class environmental management strategies.
Q. India has signed an MoU with which country for the construction of three additional submarines under
the P75 program?
A) Russia B) Germany
C) France D) Israel
Answer : C
✓ India and France have signed an MoU for the construction of three additional submarines under the P75
program.
✓ The MoU has been signed between Mazgon Dockyard Ltd and Naval Group for the construction of
submarines under the P75 programme.
✓ The Scorpene Submarines have advanced stealth features and are equipped with long-range guided
torpedoes and anti-ship missiles.
✓ These submarines have a state-of-the-art SONAR suite and sensor suite permitting outstanding
operational capabilities.
✓ Prime Minister Narendra Modi and French President Emmanuel Macron adopted ‘Horizon 2047: 25th
Anniversary of the India-France Strategic Partnership, Towards A Century of India-France Relations
Q. PM Modi inaugurated the Integrated Terminal of Veer Savarkar International Airport, in which
state/UT?
A) Puducherry B) Sikkim
C) Lakshadweep D) Andaman and Nicobar Islands
Answer : D
✓ Prime Minister Narendra Modi inaugurated the Integrated Terminal Building of Veer Savarkar
International Airport Integrated Terminal, Port Blair in virtual mode.
✓ About Rs 710 crore has been spent in completing this project.
✓ This new terminal building will facilitate ease of trade and increase connectivity in the region.
✓ The airport's terminal design resembles a conch-shaped structure depicting the sea and islands
Q. India has recently signed an agreement with which country to develop new generation military
equipment?
A) China B) UK
C) France D) Germany
Answer : C
✓ India and France have agreed to develop new generation military equipment as part of a long-term
roadmap.
✓ French Ambassador to India Emmanuel Lenain has informed about this and called it a reflection of "trust
and strong partnership".
✓ This agreement was done during the visit of Prime Minister Narendra Modi to Paris.
✓ FRANCE

Follow us: Official Site, Telegram, Facebook, Instagram, Instamojo 596


✓ Capital : Paris
✓ Currency : Euro
✓ President : Emmanuel Macron
✓ Prime Minister : Elisabeth Borne

Q. The Nelson Mandela International Day is observed annually on which day?


A) 16 July B) 17 July
C) 18 July D) 15 July
Answer : C
✓ Nelson Mandela International Day (also Mandela Day) is held every year on July 18 globally to honour
the legacy of the late President of South Africa, Nelson Mandela.
✓ The day 18 July marks the birth anniversary of the great South African leader, who was born in 1918.
✓ United Nations officially declared July 18 as UN Mandela Day in November 2009.
✓ The first UN Mandela Day was held on 18 July 2010.
✓ Theme 2023 : “The Legacy Lives on Through You: Climate, Food and Solidarity”
✓ It marks the 105th Birth anniversary of Nelson Mandela.
✓ India's Highest civillian honour 'Bharat Ratna' in 1990.
✓ The first UN Mandela Day was held on 18 July 2010
✓ Nelson Mandela South Africa’s first black President (1994-1999)
✓ He has served 27 years in prison.
✓ Long Walk to Freedom: The Autobiography of Nelson Mandela
Q. The geographical indication (GI) tag has finally been given to the Authoor vetrilai (betel leaf from
Authoor) of which state?
A) Telangana B) Tamil Nadu
C) Karnataka D) Kerala
Answer : B
✓ Tamil Nadu's Authoor betel leaves get Geographical Indication (GI) certificate.
✓ Authoor betel leaves from Thoothukudi district of Tamil Nadu have been awarded the Geographical
Indication (GI) certificate by Tamil Nadu State Agricultural Marketing Board and NABARD Madurai
Agribusiness Incubation Forum..
✓ The Authoor Vattara Vetrilai Vivasayigal Sangam, a body representing growers of the Authoor betel leaf,
has been awarded the GI certificate.
✓ This recognition as a Geographical Indication opens up new opportunities for the marketing of Authoor
betel leaves.
Q. Which company has been honoured with the prestigious Economic Times (ET) HR World Future Skills
Awards 2023?
A) Reliance Industries B) NTPC Limited
C) Adani Power D) JSW Energy
Answer : B
✓ Power major NTPC has been honoured with the prestigious Economic Times (ET) HR World Future
Skills Awards 2023.
✓ The company received awards in two categories: "Best Use of AI/AR/VR in Learning and Upskilling"
and "Best Advances in Creating Extended Enterprise Learning Programs."

Follow us: Official Site, Telegram, Facebook, Instagram, Instamojo 597


✓ These accolades are in recognition of NTPC's dedication to incorporate cutting-edge technologies,
especially Virtual Reality (VR), in its Learning and Development (L&D) initiatives.
NTPC IN NEWS 2023
▪ NTPC has been honoured with the prestigious Economic Times (ET) HR World Future Skills Awards
2023.
▪ NTPC climbs up 52 positions to 433rd rank in Forbes’ “The Global 2000” List.
▪ NTPC Kanti launches Girl Child Empowerment Mission (GeM)-2023.
▪ NTPC Group’s total installed capacity reaches 72,304 MW with first overseas capacity addition in
Bangladesh.
▪ Country's largest power generation company NTPC Limited has been conferred with 'ATD Best Awards
2023' by Association for Talent Development (ATD), USA.
▪ NTPC and Nuclear Power Corporation of India Limited (NPCIL) sign agreement for joint development
of nuclear power plants.
▪ India’s largest power company NTPC Ltd has commissioned India’s first green hydrogen blending
project at Kawas, Gujarat.
▪ NTPC hosts an international seminar on ‘Carbon Capture Utilization and Storage’ (CCUS) at First
Energy Transitions Working Group meeting in Bengaluru.
▪ NTPC Limited announced the signing of a Memorandum of Understanding (MoU) with Energy Vault
Holdings (“Energy Vault).
▪ NTPC Renewable Energy Limited (NTPC REL) signs MoU with Government of Tripura for
cooperation in renewable energy development.
▪ NTPC Renewable Energy Ltd has signed an MoU with the Indian Army for setting up Green Hydrogen
Projects in its establishments on Build, Own and Operate (BOO) model.
▪ About NTPC
▪ The NTPC which was earlier known as National Thermal Power Corporation of India is owned by the
Government of India. It was set up in 1975.
▪ The Vindhyachal Thermal Power Station in the Singrauli district of Madhya Pradesh, with an installed
capacity of 4,760MW, is currently the biggest thermal power plant in India.
▪ Headquarters: New Delhi
▪ Chairman and Managing Director: Gurdeep Singh
Q. Who has launched the sale of subsidized Chana Dal under the brand name 'Bharat Dal'?
A) Piyush Goyal B) Smriti Irani
C) Amit Shah D) Nirmala Sitharaman
Answer : A
✓ Piyush Goyal launches sale of subsidised Chana Dal under the brand name 'Bharat Dal' at affordable
prices.
✓ The objective of subsidised chana dal is to make pulses available to the consumers at affordable prices.
✓ The government's stock of gram is converted into chana dal for distribution.
✓ Availability and Outlets:
✓ Retail outlets of National Agricultural Cooperative Marketing Federation (NAFED) in Delhi-NCR
selling the chana dal.
✓ NAFED undertakes milling and packaging of the Chana Dal.
✓ Distribution through retail outlets of NAFED, NCCF, Kendriya Bhandar, and Safal.
✓ State governments can procure chana dal for their welfare schemes, police, jails, and consumer
cooperative outlets.
✓ Versatile Usage and Nutritiona Benefits of Chana:
✓ Chana is the most abundantly produced pulse in India, consumed in various forms.
✓ Chana whole: Soaked and boiled for salads.

Follow us: Official Site, Telegram, Facebook, Instagram, Instamojo 598


✓ Roasted chana: Served as snacks.
✓ Fried chana dal: Alternative to tur dal in curries and soups.
✓ Chana besan: Major raw material for namkeens and sweets.
✓ Nutritional benefits: Rich in fibre, iron, potassium, vitamin B, selenium, beta carotene, and choline.
✓ Supports various health aspects: Control of anaemia, blood sugar, bone health, and mental health.
Q. Which state’s transport department has introduced a UPI payments system at inter-state check posts to
curb corruption?
A) Uttar Pradesh B) Andhra Pradesh
C) Rajasthan D) Haryana
Answer : B
✓ Andhra Pradesh transport department has introduced a UPI payments system at inter-state check posts
to curb corruption.
✓ The policy will support the Chief Minister’s directives toward corruption-free administration without any
intermediaries.
✓ The new policy has been implemented at 15 check posts of the transport department across the state.
✓ Further, people can pay border tax, temporary permit tax, voluntary tax, compounding fee, etc by
scanning the QR code.

Q. Which state has celebrated the Harela festival recently?


A) Punjab B) Himachal Pradesh
C) Uttarakhand D) Chhattisgarh
Answer : C
✓ Harela Festival 2023 begins in Uttarakhand.
✓ Uttarakhand Chief Minister Pushkar Singh Dhami participated in a program on water conservation,
organized on the occasion of the Harela festival 2023.
Q. Who has been appointed as the first woman to lead the Reserve Bank of Australia (RBA) in the 63-year
history of the central bank?
A) Michele Bullock B) Fiona Geminder
C) Judith Neilson D) Naomi Milgrom
Answer : A
✓ Australia has appointed the first woman to lead the Reserve Bank of Australia (RBA) in the 63-year
history of the central bank.
✓ Michele Bullock, a veteran economist at the central bank, will take over from outgoing governor Philip
Lowe in September.
Q. India's Neeraj Chopra won the men's javelin throw event with a best throw of how many meters at the
Lausanne Diamond League 2023 in Switzerland?
A) 87.66m B) 87.03m
C) 86.13m D) 83.50m
Answer : A
✓ Olympic champion Neeraj Chopra clinched the prestigious Diamond League title for the second
consecutive time by throwing his spear to 87.66m

Follow us: Official Site, Telegram, Facebook, Instagram, Instamojo 599


✓ Men's javelin throw results at 2023 Lausanne Diamond League
1. Neeraj Chopra (India) - 87.66m
2. Julian Weber (Germany) - 87.03m
3. Jakub Vadlejch (Czech Republic) - 86.13m

Q. What is the position of India as per the latest FIFA Men's rankings June 2023?
A) 106 B) 104
C) 105 D) 100
Answer : D
✓ India ranked 100th in FIFA men's football rankings.
✓ 'World Men's Football Ranking' released by 'FIFA' in June 2023; In which FIFA World Cup 2022 winner
Argentina (with 1843.73 points) has topped the list.
✓ India has been ranked 100th (with a total score of 1204.90) in the 'World Men's Football Ranking' by
'FIFA'.
✓ Top 5 teams in FIFA rankings:
✓ Top - Argentina
✓ 2nd - France
✓ 3rd - Brazil
✓ 4th -England
✓ 5th - Belgium

Q. Which of the following has topped the FIFA Men's rankings June 2023?
A) Belgium B) France
C) Argentina D) Brazil
Answer : C

Q. India's biggest natural arch discovered in Which State by Geological Survey of India (GSI)?
A) Gujarat B) Kerala
C) Odisha D) Madhya Pradesh
Answer : C
✓ The Geological Survey of India (GSI) discovered a "Natural Arch" in Odisha's Sundergarh Forest
Division.
✓ This natural arch originated approximately 184 million years ago during the Lower to Middle Jurassic
period.
✓ The GSI has proposed a Geo Heritage tag for the arch, which aims to become the largest natural arch in
India with this recognition.
✓ The GSI's state unit has proposed to designate the natural arch in the Kanika range of Sundargarh forest
division as a Geo Heritage Site, making it the country's largest natural arch with the Geo Heritage tag.
✓ Currently, India has two other natural arches—one located in the Tirumala hills of Tirupati and the other
in the Andaman and Nicobar Islands.
✓ About Sundargarh Natural Arch
✓ The Sundargarh natural arch, formed in the ferruginous sandstone dates back approximately 184 to 160
million years during the lower to middle Jurassic age.
✓ The arch has an oval shape and measures 30 meters in length at the base, with a height of 12 meters.
✓ About Geological Survey of India (GSI)
✓ It is a scientific agency of India.
✓ Its parent organization is the Ministry of Mines with Pralhad Joshi as its Union Minister.

Follow us: Official Site, Telegram, Facebook, Instagram, Instamojo 600


✓ Establishment - 1851
✓ Headquarters - Kolkata, West Bengal
Q. Government to Develop National Maritime Heritage Complex in Which State for an estimated cost of
₹4,500 Cr?
A) Gujarat B) Punjab
C) Karnataka D) Andhra Pradesh
Answer : A
✓ Government to Develop National Maritime Heritage Complex in Lothal, Gujarat for an estimated cost
of ₹4,500 Cr. Under the Sagarmala programme, the Ministry of Ports, Shipping and Waterways is
developing a National Maritime Heritage Complex, a world-class facility at Lothal, Gujarat.
✓ Union Minister of Ports, Shipping and Waterways, reviewed the project process of National Maritime
Heritage Complex (NMHC), Lothal in Gandhinagar, Gujarat.
✓ NMHC complex will have Asia’s Biggest Under Water Marine Museum and India’s Grandest Naval
Museum.
✓ The NMHC is set to become the world's largest maritime museum complex and an international tourist
destination.
✓ India, Portugal sign agreement to develop National Maritime Heritage Complex in Gujarat

Q. Who appointed as Chairman and CEO of Audi, effective September 1, 2023?


A) Gernot Dollner B) Vikram Pawah
C) Martin Lundstedt D) Santosh Iyer
Answer : A
✓ Audi appoints Gernot Dollner as new CEO of the Management Board.
✓ German luxury car maker Audi AG has appointed Gernot Dollner as the new CEO of the management
board.
✓ Gernot Dollner, who currently heads Volkswagen Group's product and group strategy, will replace
Markus Deussmann as chairman of the management board of Audi AG with effect from September 1,
2023.
✓ The appointment of Gernot Dollner is part of Audi's strategy to strengthen its product strategy and market
position.
✓ Audi AG,
✓ It is headquartered in Ingolstadt, Bavaria, Germany.
✓ It is a renowned German automotive manufacturer specializing in luxury vehicles.
✓ Audi AG operates under its parent organization, the Volkswagen Group.
✓ Balbir Singh Dhillon heads Audi India, which represents the brand in the Indian market
Q. Which state government has launched the ‘Jagananna Amma Vodi’ scheme?
A) Assam B) Odisha
C) Chhattisgarh D) Andhra Pradesh
Answer : D
✓ Andhra Pradesh Chief Minister Y S Jagan Mohan Reddy has launched the ‘Jagananna Amma Vodi’
scheme for the fourth year.
✓ Under this financial assistance of Rs 6,393 crore will be deposited into the bank accounts of 43 lakh
beneficiaries.

Follow us: Official Site, Telegram, Facebook, Instagram, Instamojo 601


✓ Under the Amma Vodi scheme, eligible mothers who send their children to schools will receive Rs 15,000
financial aid to meet the educational expenses.
✓ The state has credited a total of Rs 26,067 crore under Jagananna Amma Vodi scheme, including Rs
6,393 crore disbursal set in motion.

Q. Which company has launched its first commercial space flight with paying customers?
A) Blue Origin B) Lockheed Martin
C) Virgin Galactic D) SpaceX
Answer : C
✓ Virgin Galactic completes first manned mission to space.
✓ Virgin Galactic, founded by Richard Branson, successfully completed its first manned mission to the edge
of space on June 29, marking a significant milestone for the company after two decades of dedicated
efforts.
✓ The mission had a research-oriented focus and featured two Italian Air Force personnel, Colonel Walter
Villadei and Lieutenant Colonel Angelo Landolfi, who were funded by Virgin Galactic.
✓ VSS Unity ignited its rocket engine and ascended directly upward, reaching an altitude of more than 80
kilometers, which is recognized as the edge of outer space by the United States government.
✓ Virgin Galactic is a space venture established by British entrepreneur Richard Branson.
✓ The company aims to provide commercial space travel experiences to customers
Q. Which country has become the first country in the world to ban thin plastic bags at supermarkets?
A) Switzerland B) Netherlands
C) New Zealand D) Finland
Answer : C
✓ New Zealand became the first country in the world to ban thin plastic bags at supermarkets on July 1,
2023.
✓ The ban applies to single-use plastic bags that are less than 70 microns thick.
Q. India beat which country in the Asian Kabaddi Championship 2023 in Busan, Korea?
A) Iran B) Japan
C) China D) Hong Kong
Answer : A
✓ India beat Iran 42-32 in the final of the Asian Kabaddi Championship 2023 in Busan, Korea.
✓ This is India’s eighth title in nine editions.
✓ Indian captain Pawan Sehrawat led from the front with a super 10.
✓ Earlier in the day, India had beaten Hong Kong 64-20 to end the league stage of the tournament
undefeated.
✓ The next big challenge for the Indian kabaddi teams will be the upcoming Asian Games in Hangzhou,
China.
✓ Six teams – India, Iran, Japan, Korea, Chinese Taipei and Hong Kong – participated in the
Championships.
Q. Which day of the year is observed as the National Doctors’ Day?
A) 29 June B) 30 June
C) 02 July D) 01 July

Follow us: Official Site, Telegram, Facebook, Instagram, Instamojo 602


Answer : D
✓ In India, the National Doctors’ Day is organized on 1st of July annually since 1991. The day is organised
by the Indian Medical Association (IMA).
✓ The theme for National Doctors' Day 2023 in India is "Celebrating Resilience and Healing Hands.
✓ The day also commemorates the birth anniversary of the second Chief Minister of West Bengal, Dr.
Bidhan Chandra Roy who was born on July 1, 1882, and died on the same date in 1962.

Q. National Doctors’ Day is celebrated in India to honour ____.


A) Dr. Hargovind Khurana B) Dr. Satish Chandra
C) Dr. Bidhan Chandra Roy D) Dr. Bhupendranath Datta
Answer : C
✓ In India the National Doctors' Day is celebrated on July 1 all across India to honour the legendary
physician and the second Chief Minister of West Bengal, Dr Bidhan Chandra Roy.
✓ Dr Roy was honoured with the country's highest civilian award, Bharat Ratna on February 4, 1961.
Q. India’s first indigenously developed 700 MW nuclear power reactor at the Kakrapar Atomic Power
Project (KAPP) in which state started commercial operations?
A) Maharashtra B) Gujarat
C) Madhya Pradesh D) Telangana
Answer : B
✓ India’s first indigenously developed 700 MW nuclear power reactor at the Kakrapar Atomic Power
Project (KAPP) in Gujarat started commercial operations.
✓ The Nuclear Power Corporation of India Limited (NPCIL) is building two 700 MW pressurised heavy
water reactors (PHWRs) at Kakrapar, which is also home to two 220 MW power plants.
Q. Who has been awarded the prestigious PEN Pinter Prize 2023 for his work in poetry and literature for
children?
A) Michael Rosen B) Hilary Mantel
C) Kazuo Ishiguro D) Zadie Smith
Answer : A
✓ Recently, the British children’s writer and performance poet Michael Rosen has been awarded the
prestigious PEN Pinter Prize 2023
✓ About PEN Pinter Prize:
✓ It was established in 2009 in memory of Nobel-Laureate playwright Harold Pinter.
✓ The winner must be the author of a significant body of plays, poetry, essays, or fiction of outstanding
literary merit, written in English.
✓ It is given to a writer from the UK, Ireland and the Commonwealth whose work is committed to a fearless
exposition of truth about contemporary life.
✓ The prize is shared with an international writer of courage selected by English PEN’s Writers at Risk
Committee in association with the winner.
Q. Prime Minister Narendra Modi has inaugurated the National Sickle Cell Anemia Eradication Mission
2047 in which state?
A) Uttar Pradesh B) Madhya Pradesh
C) Chhattisgarh D) Punjab

Follow us: Official Site, Telegram, Facebook, Instagram, Instamojo 603


Answer : B
✓ Prime Minister launched the National Sickle Cell Anaemia Elimination Mission in Madhya Pradesh.
✓ On July 1, Prime Minister Narendra Modi launched the National Sickle Cell Anemia Elimination
Mission in Shahdol, Madhya Pradesh.
✓ The mission aims to eliminate sickle cell anaemia by 2047.
✓ This mission will be implemented in 278 districts of 17 states of India.
✓ The targeted states include Gujarat, Maharashtra, Rajasthan, Madhya Pradesh, Jharkhand,
Chhattisgarh, West Bengal, Odisha, Tamil Nadu, Telangana, Andhra Pradesh, Karnataka, Assam, Uttar
Pradesh, Kerala, Bihar and Uttarakhand.
✓ The mission focuses on creating awareness and universal testing among about seven crore people in the
age group of 0-40 years in the affected tribal areas.
✓ Collaborative efforts of Central Ministries and State Governments will be employed for consultation and
implementation.
✓ According to the National Heart, Lung and Blood Institute (NHLBI), sickle cell anaemia is a collection
of inherited red blood cell conditions that affect haemoglobin, which carries oxygen in the body.

Q. Who reappointed as Solicitor General of India along with 6 ASGs for SC?
A) Ranjit Kumar B) Aman Lekhi
C) Tushar Mehta D) Harish Salve
Answer : C
✓ The Appointments Committee of the Cabinet has approved re-appointment of the Tushar Mehta as
Solicitor General of India.
✓ Solicitor General is the second most senior law officer after Attorney General..
✓ Mehta was appointed as Solicitor General for the first time in October 2018 and his period was later
extended from time to time.
✓ The government aims to maintain a strong legal framework in India and uphold justice through these
appointments.

Q. Who has taken oath as the new Deputy Chief Minister of Maharashtra?
A) Sharad Pawar B) Ajit Pawar
C) Sanjay Raut D) Aditya Thackeray
Answer : B
✓ NCP leader Ajit Pawar has been sworn in as the new Deputy Chief Minister of Maharashtra.
✓ This is the third time in the last 5-years that Ajit Pawar has taken oath as the Deputy Chief Minister of
Maharashtra. With this, he has become a part of Chief Minister Eknath Shinde's government.
✓ Pawar was sworn in as Deputy Chief Minister in November 2019 during the tenure of the then Chief
Minister Devendra Fadnavis.
Q. Who to be India's ambassador to WTO for 9 months till 31 March 2024?
A) C. P. Gurnani B) Brajendra Navnit
C) Sofia Boza Martinez D) Vineet Nayyar
Answer : B
✓ Indian government announced the extension of Brajendra Navnit’s tenure as Ambassador and
Permanent Representative of India to the World Trade Organization (WTO) by 9 months till 31 March
2024.
✓ This decision comes ahead of the crucial 13th ministerial conference of the WTO in 2024.

Follow us: Official Site, Telegram, Facebook, Instagram, Instamojo 604


✓ World Trade Organisation (WTO)
✓ It was set up on 1st January 1995 replacing the General Agreement on Tariff and Trade (GATT).
✓ Its main purpose is to promote rule based trading systems in the world and it also settles trade related
disputes between the member countries.
✓ The headquarters of WTO is in Geneva, Switzerland.
✓ There are 164 member countries.
✓ Director General of WTO : Dr Ngozi -Okonjo-Iweala of Nigeria.
✓ Report released by WTO : World Trade Report
Q. Who has been appointed as the new Chairman of Coal India Limited (CIL)?
A) Rajiv Jain B) Tapan Deka
C) Dineshwar Sharma D) P M Prasad
Answer : D
✓ P M Prasad has been appointed as the new Chairman of Coal India Limited (CIL).
✓ He succeeds Pramod Agrawal who stepped down on 30 June.
✓ Coal India Limited (CIL)
✓ Founded : 1975
✓ Headquarters : Kolkata, West Bengal
✓ Coal India Limited (CIL) is an Indian central public sector undertaking under the ownership of the
Ministry of Coal.
✓ It is the largest government-owned-coal-producer in the world. It is also the seventh largest employer in
India.
Q. A photo of an elephant taken at the Anamalai Tiger Reserve (ATR) has emerged as the winner of the
Siena Drone Photo Awards 2023. Anamalai Tiger Reserve is located in which state?
A) Kerala B) Karnataka
C) Tamil Nadu D) Telangana
Answer : C
✓ A photo of an elephant taken at the Anamalai Tiger Reserve (ATR) in Tamil Nadu has emerged as the
winner of the Siena Drone Photo Awards 2023
✓ The photo was taken by wildlife photographer, Mathanraj S and will receive a cash prize of $10,000.
✓ The photo shows a herd of elephants crossing a river in the reserve.
✓ The photo was chosen as the winner by a panel of judges from around the world.
Q. What is the rank of India in the Global Peace Index 2022 report released by the Institute for Economics
and Peace (IEP)?
A) 126 B) 133
C) 144 D) 110
Answer : A
✓ Afghanistan is the least peaceful country in the world for the fifth consecutive year, according to the
‘Global Peace Index 2022’ report released by the Institute for Economics and Peace (IEP).
✓ Yemen, Syria, Russia, and South Sudan are followed by Afghanistan in the Global Peace Index 2022
report.
✓ All these countries have remained among the ten least peaceful nations for the past three years, Tolo
News reported.
✓ India stands at 126 ranks out of 163 countries in the Global Peace Index.

Follow us: Official Site, Telegram, Facebook, Instagram, Instamojo 605


✓ While Iceland has most peaceful country followed by Denamrk, Ireland.
Q. The Ministry of Defense (MoD) has sealed an agreement worth Rs 2,725 crore with Mazagon Dock
Shipbuilders Limited (MDL) for refurbishment and life certification of which INS?
A) INS Karanj B) INS Khanderi
C) INS Sindhuraj D) INS Shankush
Answer : D
✓ Defence ministry inks a Rs 2725 cr deal with MDL for INS Shankush.
✓ The Ministry of Defence (MoD) has sealed an agreement worth Rs 2,725 crore with Mazagon Dock
Shipbuilders Limited (MDL) for the refurbishment and life certification of INS Shankush, a formidable
submarine from the Indian Navy’s sub-surface killer (SSK) class.
✓ The project, known as Medium Refit with Life Certification (MRLC), aims to upgrade the submarine’s
combat capabilities.
✓ The INS Shankush submarine is projected to be combat-ready and integrated into the active fleet of the
Indian Navy after the completion of MRLC.
✓ The upgraded vessel is scheduled for delivery in 2026, equipped with enhanced combat capabilities.
Q. Union Minister for Home and Cooperation Amit Shah has virtually launched the ‘Akshar River Cruise’
on which Riverfront?
A) Ganga Riverfront B) Narmada Riverfront
C) Sabarmati Riverfront D) Tapi Riverfront
Answer : C
✓ Union Minister for Home and Cooperation Amit Shah has virtually launched the ‘Akshar River Cruise’
on Sabarmati Riverfront.
✓ This cruise is developed under Public Private Partnership by Ahmedabad Municipal Corporation and
Sabarmati Riverfront Development Corporation Limited.
✓ The cruise is the first passenger catamaran built in the country under Make in India at a cost of Rs 15
crore with twin engines where 165 passengers can travel safely for one-and-a-half hours.
Q. Which Indian Armed Force is the host of ‘Tarang Shakti’ multinational air exercise?
A) Indian Army B) Indian Navy
C) Indian Air Force D) Indian Coast Guard
Answer : C
✓ Biggest air exercise 'Tarang Shakti'.
✓ Indian Air Force is making preparations to organize a large-scale multilateral exercise later this year,
aimed at enhancing military cooperation with 12 nations.
✓ The exercise, named Tarang Shakti, is set to be the largest air exercise ever conducted in India,
showcasing the country's commitment to strengthening international military partnerships.
✓ Among others, air forces of France, Australia, the US and the UK are likely to participate in the exercise.
✓ The participating countries will contribute their military combat jets, as well as transport aircraft and
other assets, showcasing their collective capabilities and fostering interoperability.
✓ Earlier in 2023, IAF took part in Exercise Orion in France and INIOCHOS in Greece.
✓ In April, the IAF and the USAF had taken part in a joint exercise in Cope India-2023 across Kalaikunda,
Panagarh and Agra bases

Follow us: Official Site, Telegram, Facebook, Instagram, Instamojo 606


Q. What is the rank of India in the 2023 Global Energy Transition Index (ETI)?
A) 87 B) 52
C) 67 D) 95
Answer : C
✓ India ranks 67th on WEF's Energy Transition Index.
✓ World Economic Forum (WEF) published a report titled 'Fostering Effective Energy Transition 2023'
which ranked 120 countries on the basis of energy transition.
✓ In this ranking, WEF has placed India at 67th position (a jump of 20 places) globally in the Energy
Transition Index (ETI). Because in the year 2021, India was at 87th position among 115 countries.
✓ According to WEF, India is the only major economy where the pace of energy transition is accelerating
in all dimensions.
✓ Despite sustained economic growth, India has successfully reduced the energy intensity of its economy
and the carbon intensity of its energy mix, achieved universal energy access and managed electricity
affordability effectively
✓ According to ETI the top five countries in the world are Sweden, Denmark, Norway, Finland and
Switzerland respectively.
✓ World Economic Forum (WEF):
✓ Establishment : 1971 in Geneva (Switzerland)
✓ Headquarters: Cologny, Switzerland
✓ Founder : Klaus Schwab
✓ Chairman : Borge Brende
Q. Which country has topped the 2023 Global Energy Transition Index (ETI)?
A) Singapore B) Norway
C) Germany D) Sweden
Answer : D
✓ According to ETI the top five countries in the world are Sweden, Denmark, Norway, Finland and
Switzerland respectively
Q. The Global Energy Transition Index (ETI) is released every year by which organisation?
A) World Economic Forum B) International Monetary Fund
C) World Bank D) United Nations Development Programme
Answer : A
✓ The report is published by the World Economic Forum (WEF), prepared in collaboration with Accenture
to track nations on the current performance of their energy systems across various aspects
✓ Major reports published by WEF:
✓ Global Gender Gap Report
✓ Energy Transition Index (published jointly by Accenture and WEF).
✓ Global Competitiveness Report
✓ Global Risk Report
✓ Global Travel & Tourism Report
✓ Global Information Technology Report (The report is published by WEF in association with INSEAD
and Cornell University)

Follow us: Official Site, Telegram, Facebook, Instagram, Instamojo 607


Q. Which financial institution has approved a substantial funding package of 700 million dollars for Sri
Lanka?
A) World Bank B) Asian Development Bank
C) New Developments Bank D) International Monetary Fund
Answer : A
✓ World Bank has approved a substantial funding package of 700 million dollars for Sri Lanka, providing
a significant boost to the country's financial situation.
✓ Out of the approved funds, approximately 500 million dollars will be allocated for budgetary support,
aimed at stabilizing the country's economy.
✓ The remaining 200 million dollars will be dedicated to welfare support, with the objective of assisting
those most affected by the ongoing crisis.

SRI LANKA IN NEWS 2023


▪ World Bank approves $700 million for crisis-hit Sri Lanka.
▪ Union Minister for Ports, Shipping and Waterways, Sarbananda Sonowal flagged off India's first
international cruise vessel - MV Empress, from Chennai to Sri Lanka.
▪ Bharti Airtel has announced its plan to merge its Sri Lanka operations with Dialog Axiata, the largest
telecom provider in Sri Lanka.
▪ The 10th annual SLINEX-2023 bilateral maritime exercise between India and Sri Lanka has commenced
in Colombo, Sri Lanka.
▪ Sri Lanka assent to Adani Green’s $442 Million Wind Power Project.
▪ India will support Sri Lanka’s debt restructuring plan as the island nation looks to trim its huge public
expenditure to win approval for a crucial bailout from the International Monetary Fund.
▪ India has supplied 50 buses to Sri Lanka on the occasion of the 75th anniversary of independence.
▪ Indian High Commission signed a memorandum of understanding (MoU) with the Sabaragamuwa
University of Sri Lanka to establish a Hindi Chair.
▪ India & Sri Lanka inaugurate exhibition ‘Geoffrey Bawa’ in New Delhi.
▪ Reliance Retail Ventures Limited announced a strategic partnership with Sri Lanka-headquartered
Maliban Biscuit Manufactories Limited.
▪ SRI LANKA
▪ Capital : Sri Jayawardenepura Kotte (legislative)
▪ Colombo (executive and judicial)
▪ Currency : Rupee
▪ President : Ranil Wickremesinghe
▪ Prime Minister : Dinesh Gunawardena
▪ Mitra Shakti Military Exercise : India and Sri Lanka
▪ Slinex Navel Exercise : India and Sri Lanka
▪ Parliament : parliamenthua (225 seats)
▪ National day : 4 February 1948

Q. Which city’s police has established a ‘Drone Police Unit’ to assist cops in aerial surveillance?
A) Chennai B) Bengaluru
C) New Delhi D) Hyderabad
Answer : A
✓ India's First 'Police Drone Unit' Launched in Chennai for Enhanced Aerial Surveillance and Quick
Detection of Criminal Activities.

Follow us: Official Site, Telegram, Facebook, Instagram, Instamojo 608


✓ The Greater Chennai City Police (GCP) has introduced the groundbreaking 'Police Drone Unit' to bolster
aerial surveillance capabilities and expedite the identification of criminal activities.
✓ The initiative, which involves an investment of approximately Rs 3.6 crore, was inaugurated by the
outgoing Tamil Nadu Director General of Police (DGP) C Sylendra Babu, in the presence of Chennai
Police Commissioner Shankar Jiwal at Besant Avenue, Adyar.
Q. Union Home Minister Amit Shah has approved the release of Rs 6,194.40 crore to how many states
under the State Disaster Response Fund (SDRF)?
A) 16 B) 19
C) 12 D) 15
Answer : B
✓ Union Home Minister Amit Shah has approved the release of Rs 6,194.40 crore to 19 states under the
State Disaster Response Fund (SDRF).
✓ The amount includes Rs 1,209.60 crore as a central share of the SDRF to four states (Chhattisgarh,
Meghalaya, Telangana, and Uttar Pradesh) for 2022-23.
✓ Rs 4,984.80 crore will also be given to 15 states (Andhra Pradesh, AP Assam, Bihar, Goa, Haryana, HP,
Kerala, Maharashtra, Manipur, Meghalaya, Odisha, Punjab, TN, and Tripura) for 2023-24.
✓ The Central government has already approved the release of ₹3,649.40 crore as Central share of SDRF
to nine States during 2023-24.
✓ Based on the 15th Finance Commission’s recommendations, the Central government has allocated
₹1,28,122.40 crore for SDRF for the years 2021-22 to 2025-26
Q. PM Narendra Modi has inaugurated Sai Hira Global Convention Centre in Puttaparthi, Andhra Pradesh
via video conferencing. In this, he has named ‘Amrit Kaal’ as _.
A) Kirti Kaal B) Krishi Kaal
C) Kartavya Kaal D) Aatmnirbhar Kaal
Answer : C
✓ Prime Minister Narendra Modi has virtually inaugurated the Sai Hira global convention centre at
Prasanthi Nilayam at Puttaparthi in Sri Sathya Sai district of Andhra Pradesh.
✓ The next 25 years of India’s independence is going to be the “Kartavya Kaal” as the country will Accord
top priority to performing its duties.
✓ Moving forward towards the journey of 100 years of Indian independence, we have named this “Amrit
Kaal” as the “Kartavya Kaal”.
✓ The new global convention centre established with the generous support of philanthropist Ryuko Hira,
is a testament to the vision of promoting cultural exchange, spirituality and global harmony
Q. In which country will the first IIT campus outside India be set up?
A) Kenya B) South Africa
C) Egypt D) Tanzania
Answer : D
✓ India and Tanzania have signed an MoU for setting up a new campus of IIT Madras.
✓ It is the first IIT campus to be established outside India.
✓ Union Education and Skill Development and Entrepreneurship Minister Dharmendra Pradhan has
described this decision as historic.
✓ Tanzania is an East African country, its capital is Dodoma.

Follow us: Official Site, Telegram, Facebook, Instagram, Instamojo 609


IIT IN NEWS 2023
▪ IIT Guwahati has signed MoU with the Indian Council of World Affairs (ICWA) to build capacity/skills
in international affairs and foreign policy.
▪ IIT Guwahati researchers develop Al-based model for knee X-ray.
▪ Infosys co-founder Nandan Nilekani donated ₹315 crores to IIT Bombay.
▪ IIT-Madras generates hydrogen from seawater using solar energy.
▪ India, Israel to jointly develop Center of Water Technology at IIT Madras.
▪ IIT Madras researchers partner for development of materials and micro-device processing techniques.
▪ IIT Kanpur’s C3iHub, a cybersecurity Technology Innovation Hub, has launched a Cybersecurity
Skilling Programme .
▪ IIT Madras to set up its first international campus in Tanzania.
▪ Indian Institute of Technology Madras will be provided a grant of Rs 242 crore over a period of five years
to carry out research on Lab Grown Diamonds (LGD).
▪ Indian Institute of Technology (IIT) Bombay has been ranked 1st in India and 47th globally in
engineering and technology by the QS World University Rankings for 2023.
▪ IIT Indore, in partnership with NASA-Caltech and the University of Gothenburg in Sweden, has
designed an inexpensive camera setup.
▪ Indian Institute of Technology Madras (IIT Madras) Researchers have developed a three-dimensional
(3D) paper-based portable device that can detect adulteration in milk within 30 seconds.
▪ IIT Kanpur to Host Youth20 Consultation under G20 Presidency of India.
▪ IIT Bombay’s SHUNYA secures second place in ‘Solar Decathlon’ Build Challenge in US.
▪ Students of the Indian Institute of Technology (IIT) Indore won an award of AED (Emirates Dirham) 1
million along with a gold medal at the World Government Summit in Dubai.
▪ IIT Indore Students Awarded with Global Best M-GOV Awards by Egyptian President.
▪ Startup Incubation and Innovation Centre (SIIC) at IIT Kanpur has entered into a Corporate Social
Responsibility (CSR) agreement with Advanced Weapons and Equipment India Limited.
▪ DRDO Industry Academia Centre of Excellence inaugurated at IIT Hyderabad.
▪ IIT Madras-incubated firm has developed an indigenous mobile operating system called BharOS.
▪ IIT Madras Centre of Excellence working with DRDO on Advanced Defence Technologies including
Combat Vehicle Technologies.
▪ IIT Madras researchers develop machine learning tool to detect tumour in brain, spinal cord.
▪ ISRO plans to develop astronaut training module with IIT Madras.
▪ IIT-Bombay and Unique Identification Authority of India (UIDAI) join hands to develop touchless
biometric system.
▪ IIT delhi has been ranked among the top 50 engineering institutions in the QS World University
Rankings by Subject .
▪ An IIT Madras incubated firm has developed an indigenous mobile operating system 'Bharos’ that can
benefit India's 100 crore mobile phone users.
Q. Who is the champion of FIH Hockey Pro League 2022/23 season?
A) Belarus B) Belgium
C) Netherlands D) India
Answer : C
✓ The Netherlands men's team has been crowned champions of the FIH Hockey Pro League 2022/23
season as they ended their season four campaign with 35 points.
✓ The Indian men's hockey team finished fourth in the FIH Pro League 2022-23 with 30 points from 16
matches.
✓ India captain Harmanpreet Singh was the top scorer in the FIH Pro League 2022-23 with 18 goals

Follow us: Official Site, Telegram, Facebook, Instagram, Instamojo 610


Q. Hewlett Packard Enterprise (HPE) has partnered with Indian manufacturer VVDN Technologies to begin
manufacturing some of its high-volume servers in India. Hewlett is the company of which country?
A) Israel B) Japan
C) United States D) Russia
Answer : C
✓ Hewlett Packard to manufacture high-end servers in India.
✓ American multinational information technology company, Hewlett Packard Enterprise (HPE) has
partnered with Indian manufacturer VVDN Technologies to begin manufacturing some of its high-
volume servers in India.
✓ This announcement comes in the wake of the recent Prime Minister Narendra Modi’s State visit to the
US.
✓ HPE has partnered with Indian manufacturer VVDN Technologies to manufacture HPE’s products from
its plant in Manesar in Haryana.

Q. Who has been named the AIFF Men’s Footballer of the Year for 2022-23?
A) Lallianzuala Chhangte B) Liston Colaco
C) Sunil Chhetri D) Naorem Mahesh Singh
Answer : A
✓ Indian football team midfielder Lallianzuala Chhangte has been named the AIFF Men’s Footballer of
the Year for 2022-23.
✓ He beat Nandhakumar Sekar and Naorem Mahesh Singh of East Bengal to win the award.
✓ While Manisha Kalyan won her second consecutive Women’s Footballer of the Year award.
✓ AIFF Men’s Coach of the Year award – Clifford Miranda.
✓ AIFF Women’s Coach of the Year – Priya Parathi Valappil.
✓ AIFF Men’s Emerging Footballer of the Year 2022-23- Akash Mishra.
✓ AIFF Women’s Emerging Footballer of the Year 2022-23- Shilji Shaji
Q. Which state government has launched the 'Mo Jungle Jami Yojana' to saturate recognition of rights
pertaining to the Forest Rights Act (FRA)?
A) Bihar B) Odisha
C) West Bengal D) Maharashtra
Answer : B
✓ The Odisha government will soon launch ‘Mo Jungle Jami Yojana’ to saturate recognition of rights
pertaining to the Forest Rights Act (FRA) and speed up the post-rights recognition activities within the
prescribed time frame.
✓ Under the scheme, designed by the ST & SC Development department, it has been decided to constitute
forest rights cells at the tehsil and district level with human resources to ensure and assist in the effective
implementation of FRA, 2006.

Q. Which State overtakes Tamil Nadu to become India's largest MFI market?
A) Kerala B) Odisha
C) Bihar D) Madhya Pradesh
Answer : C

Follow us: Official Site, Telegram, Facebook, Instagram, Instamojo 611


✓ Bihar has overtaken Tamil Nadu to become the state with the highest microlending borrowings in India
as of March 2023, according to a report.
✓ As of March 2023, Bihar's microfinance (MFI) borrowings amounted to Rs 48,900 crore, accounting for
14.5 percent of the overall portfolio. Meanwhile, Tamil Nadu's MFI borrowings stood at Rs 46,300 crore,
representing 13.7 percent of the total outstanding.

Q. China’s QU Dongyu was re-elected to a second term as Director-General of which organization?


A) World Economic Forum (WEF)
B) United Nations Development Programme (UNDP)
C) United Nations Environment Programme (UNEP)
D) Food and Agriculture Organization (FAO)
Answer : D
✓ China’s QU Dongyu was re-elected to a second term as Director-General of the Food and Agriculture
Organization of the United Nations (FAO).
✓ He was elected during the 43rd session of the conference of the United Nations Food and Agriculture
Organization (FAO) held in Rome, Italy.
✓ Food and Agriculture Organization (FAO)
✓ Founded: 16 October 1945
✓ Headquarters: Rome, Italy
✓ Director-General: QU Dongyu (China)
✓ Motto: “Fiat panis” (Let there be bread)

Q. Which of the following cities is the venue of the 3rd World Hindu Congress (WHC) 2023?
A) Chicago B) Bangkok
C) Washington D) New Delhi
Answer : B
✓ Third World Hindu Conference organized in Bangkok.
✓ According to the Word Hindu Foundation, the third 'World Hindu Congress' (WHC) 2023 will be
organized in Bangkok in November.
✓ The three-day Vishwa Hindu Sammelan will be organized by the Vishwa Hindu Foundation from
November 24 at the Convention Center in Bangkok.
✓ Organizer of World Hindu Congress:
✓ The World Hindu Congress is being organized by the World Hindu Foundation. Its trustee is Swami
Vigyananand.
✓ Swami Vigyanananda is the Managing Trustee of the Vishwa Hindu Foundation as well as the Joint
General Secretary of the Vishwa Hindu Parishad (VHP).
✓ Invited members from India:
✓ Chief Minister of Uttar Pradesh Yogi Adityanath, Sarsanghchalak of Rashtriya Swayamsevak Sangh
Mohan Bhagwat and Ma Amritanandamayi have been invited to this event.
✓ Representatives of Hindu organizations and prominent Buddhist teachers from all over the world have
been invited to the conference.
✓ Among politicians from India, only Chief Minister Yogi Adityanath has been invited.
✓ Former World Hindu Congress:
✓ The first World Hindu Congress was held in Delhi in 2014 and the second in 2018 in Chicago, USA.

Follow us: Official Site, Telegram, Facebook, Instagram, Instamojo 612


Q. Who has been presented with the British National Award for Best Director for 'What's Love Got To Do
With It'?
A) Shekhar Kapur B) Gurinder Chadha
C) Tarsem Singh D) Mira Nair
Answer : A
✓ Noted film director Shekhar Kapur has been presented with the British National Award for Best Director
for 'What's Love Got To Do With It'.
✓ At the award ceremony, the film 'What's Love Got To Do With It' was nominated in nine categories, out
of which the film won four.
✓ Apart from Best Director, the film was also awarded Best British Film, Best Screenplay and Best
Supporting Actor.
✓ Shekhar Kapur is best known for directing films ranging from Bandit Queen, Mr. India to Elizabeth.
Q. India and which country will co-host a two-day ‘India-Africa International Millet Conference’ on 30 –
31 August 2023?
A) Kenya B) Ethiopia
C) Nigeria D) Rwanda
Answer : A
✓ India and Kenya will co-host a two-day ‘India-Africa International Millet Conference’ on 30 – 31 August
2023.
✓ The event will witness participation from government leaders, researchers, farmers, entrepreneurs and
industry associations from around the globe.
✓ Through the international conference, the government of India and Kenya aim to raise public awareness
about millet as the world’s emerging smart food.
✓ The government of India spearheaded the United Nations General Assembly (UNGA) resolution for
declaring the year 2023 as the International Year of Millets.
✓ India produces all the nine commonly known traditional millets viz. Sorghum, Pearl Millet, Finger
Millet, Foxtail Millet, Proso Millet, Little Millet, Barnyard Millet, Browntop Millet and Kodo Millet.
✓ Millet is a common term for categorizing small-seeded grasses that are often called Nutri-cereals. Most
of the states in India grow one or more millet crop species.
✓ Rajasthan, Uttar Pradesh, Haryana, Gujarat, Madhya Pradesh, Maharashtra, Karnataka, Tamil Nadu,
Andhra Pradesh, and Telangana are the major millets producing states.
✓ In March, India held its first mega-global event Global Millets (Shree Anna) Conference in Delhi to
celebrate 2023 as the International Year of Millets (IYM)
Q. The Ministry of Home Affairs has launched the “Scheme for Expansion and Modernization of Fire
Services in the States” with a total budget of how many rupees?
A) Rs. 2,000 crore B) Rs. 3,000 crore
C) Rs. 4,000 crore D) Rs. 5,000 crore
Answer : D
✓ The Ministry of Home Affairs launched the “Scheme for Expansion and Modernization of Fire Services
in the States” with a total budget of Rs. 5,000 crore.
✓ The scheme, part of the preparedness and capacity building funding window under the National Disaster
Response Fund (NDRF), aims to strengthen fire services across the country.
✓ Out of the allocated amount, Rs. 500 crore has been reserved for incentivizing states based on their legal
and infrastructure-related reforms.

Follow us: Official Site, Telegram, Facebook, Instagram, Instamojo 613


Q. European Space Agency (ESA) has launched the specially designed Euclid Space Telescope to review
solar system bodies. This telescope will be installed at a distance of about ___ from the Earth.
A) 2.3 million kilometers B) 1.8 million kilometers
C) 2.2 million kilometers D) 1.5 million kilometers
Answer : D
✓ ESA launches 'Euclid Space Telescope' to review solar system bodies.
✓ The European Space Agency (ESA) has launched the specially designed Euclid Space Telescope to
review solar system bodies and obtain new information.
✓ Through this telescope, the scientist will prepare a three-dimensional map of billions of galaxies spread
over a wide area of 10 billion light years of the universe.
✓ Also, it is expected that its observations will help in solving the mysteries of dark matter and dark energy.
✓ This teslascope is named after the Greek mathematician Euclid.
✓ This telescope will be installed at a distance of about 1.5 million kilometers from the Earth by going to
the second Lagrange Point (L2) and from there it will also be able to observe those waves which are
difficult to reach the surface of the Earth.
Q. Who has been appointed as the chairman of the Research Advisory Committee (RAC) of the Indian
Council of Agricultural Research (ICAR)?
A) B. Neeraj Prabhakar B) Sri Konda Laxman
C) Himanshu Pathak D) Tarun Bajaj
Answer : A
✓ B. Neeraj Prabhakar has been appointed as the chairman of the Research Advisory Committee (RAC) of
the Indian Council of Agricultural Research (ICAR).
✓ Ms. Prabhakar, Vice-Chancellor of Sri Konda Laxman Telangana State Horticultural University, has
been appointed as the Chairperson of the Research Advisory Committee (RAC) for ICAR-Indian
Institute of Oil Palm Research (IIOPR) at Pedavegi, Andhra Pradesh.
✓ In addition, Ms. Prabhakar has also served as a member of the Telangana Oil Palm Advisory Committee,
which advises the state government on various issues for the cultivation and promotion of oil palm.
✓ Telangana boasts of the highest oil extraction from oil palm kernels produced among all states in India.
✓ The ICAR-Indian Institute of Oil Palm Research (IIOPR) at Pedavegi (Andhra Pradesh) is the only
dedicated institute in India dedicated to research on oil palm and developing technologies applicable to
all oil palm growing states.
✓ Indian Council of Agricultural Research (ICAR):
✓ Established: 16 July 1929
✓ Headquarters: New Delhi
✓ Director General: Himanshu Pathak
Q. India has signed a protocol document with which country to extend the current MoU on cooperation in
personnel management and public administration until 2028?
A) Singapore B) Malaysia
C) Vietnam D) Indonesia
Answer : A
✓ India signed a protocol document with Singapore to extend the current MoU on cooperation in personnel
management and public administration until 2028.
✓ The MoU aims to strengthen the partnership between India and Singapore through various forms of
cooperation between both countries’ public services.

Follow us: Official Site, Telegram, Facebook, Instagram, Instamojo 614


✓ The cooperation in areas such as administrative reforms & public sector transformation, public service
delivery, leadership, and talent development, e-governance, capacity building, and training form part of
the areas to be covered under the activities of MoU.

Q. Which Indian badminton player has won the title of Canada Open?
A) Parupalli Kashyap B) Srikant Kidambi
C) Lakshya Sen D) B Sai Praneeth
Answer : C
✓ Lakshya Sen has won the men's singles title in Canada Open 2023 by defeating world number-10 player
Li Shi Feng of China in straight sets 21-18, 22-20.
✓ Commonwealth Games gold medalist Lakshya Sen has clinched the men's singles title at the Canada
Open 2023 by defeating the current All England champion Li Shi Feng of China.
✓ This is Lakshya's second BWF World Tour title this season.
✓ He has already won the India Open title in January 2022.
✓ Lakshya's current world ranking in men's singles is 19.
✓ Women’s Singles: Akane Yamaguchi (Rank 1) of Japan won the Women’s Singles title by defeating
Thailand’s Ratchanok Intanon at the Canada Open 2023..
Q. Which state government has doubled the insurance cover under the Pradhan Mantri Jan Arogya Yojana
(PMAY)?
A) Madhya Pradesh B) Uttar Pradesh
C) Gujarat D) Haryana
Answer : C
✓ The Gujarat government has doubled the insurance cover under the Pradhan Mantri Jan Arogya Yojana
(PMAY) from 11 July 2023.
✓ Now the beneficiaries of PMAY will be able to get an insurance cover of Rs 10 lakh instead of Rs 5 lakh.
✓ According to state health minister Rushikesh Patel, the decision will benefit around 1.78 crore Ayushman
Bharat card holders in the state.
✓ Beneficiaries under PMAY will get free medical treatment in two thousand government hospitals and
795 private hospitals in the state.
✓ Gujarat has received over 39 lakh claims under PMAY.
✓ Gujarat ranks fifth in the country in terms of number of claims and second in terms of amount of claims
settled so far under PMAY
Q. What is the highest Grade in Performance Grading Index (PGI) 2.0?
A) Daksh B) Utkarsh
C) Pracheshta D) Ati Uttam
Answer : A
✓ Recently, the Union Ministry of Education released a report on Performance Grading Index 2.0 for
States/UTs for the year 2021-22.
✓ About Performance Grading Index:
✓ It was first released for the year 2017-18 and so far it has been released up to the year 2020-21.
✓ It assesses the performance of school education system at the State/UT level by creating an index for
comprehensive analysis.
✓ Aim of PGI 2.0: To propel States & UTs towards undertaking multi-pronged interventions that will bring
about the much-desired optimal education outcomes covering all dimensions.

Follow us: Official Site, Telegram, Facebook, Instagram, Instamojo 615


✓ The PGI 2.0 structure comprises of 1000 points across 73 indicators grouped into 2 categories viz.,
Outcomes, Governance Management (GM).
✓ These categories are further divided into 6 domains, viz., Learning Outcomes (LO), Access (A),
Infrastructure & Facilities (IF), Equity (E), Governance Process (GP) & Teachers Education and
Training (TE&T).
✓ PGI 2.0 for 2021-22 classified the States/UTs into ten grades viz., highest achievable Grade is Daksh,
which is for State/UT scoring more than 940 points out of total of 1000 points.
✓ The lowest grade is Akanshi-3 which is for score up to 460.
✓ Indicators of PGI 2.0 have been aligned to policy initiatives and interventions introduced post
implementation of National Education policy (NEP) 2020 for proper tracking the progress.
✓ The PGI 2.0 is expected to help States and UTs to pinpoint the gaps and accordingly prioritize areas for
intervention to ensure that the school education system is robust at every level.
Q. Which Indian badminton player has won the title of Canada Open?
A) Parupalli Kashyap B) Srikant Kidambi
C) Lakshya Sen D) B Sai Praneeth
Answer : C
✓ Lakshya Sen has won the men's singles title in Canada Open 2023 by defeating world number-10 player
Li Shi Feng of China in straight sets 21-18, 22-20.
✓ Commonwealth Games gold medalist Lakshya Sen has clinched the men's singles title at the Canada
Open 2023 by defeating the current All England champion Li Shi Feng of China.
✓ This is Lakshya's second BWF World Tour title this season.
✓ He has already won the India Open title in January 2022.
✓ Lakshya's current world ranking in men's singles is 19.
✓ He defeated world number four Kunlavut Witidsarn in the round of 32. Lakshya then got a win over
world number 11 Kento Nishimoto of Japan in the semi-finals.
✓ Women’s Singles: Akane Yamaguchi (Rank 1) of Japan won the Women’s Singles title by defeating
Thailand’s Ratchanok Intanon at the Canada Open 2023..
Q. Which state government has signed six MoUs with private industries for the mangrove plantation under
the MISTHI (Mangrove Initiative for Shoreline Habitats & Tangible Incomes) Scheme?
A) Rajasthan B) Gujarat
C) Haryana D) Bihar
Answer : B
✓ The Gujarat government has signed six MoUs with private industries for the mangrove plantation under
the MISTHI (Mangrove Initiative for Shoreline Habitats & Tangible Incomes) Scheme.
✓ Led a plantation drive for mangroves, under PM Narendra Modi’s visionary Mangrove Initiative for
Shoreline Habitats & Tangible Incomes program, at a site near Rukmini Temple.
✓ The importance of core activities such as mangrove plantation target, stock of mangrove nurseries,
livelihood opportunities, publicity and outreach programme as well as eco-tourism in mangrove areas
through self helps groups (SHGs) to help achieve 30% increase in the mangrove sink.
✓ The mangrove forest will be developed at a cost of Rs 510 lakhs in the next five years
✓ MISHTI (Mangrove Initiative for Shoreline Habitats and Tangible Incomes).
✓ The scheme was first announced by the Union finance minister Nirmala Sitharaman in the 2023-24
Union budget.
✓ This will facilitate mangrove plantation on salt lands along the sea coast of India.
✓ The scheme will operate through “convergence between MGNREGS, Campa Fund and other sources”.
✓ It aims at intensive afforestation of coastal mangrove forests.

Follow us: Official Site, Telegram, Facebook, Instagram, Instamojo 616


✓ There are such forests on both the east and west coasts of India, with the Sundarbans in Bengal being one
of the largest mangrove forests.
✓ Initially the mangrove cover will be restored in nine states across the country
✓ The scheme envisages the development of mangroves covering nearly 540 sq km, spreading across 11
states and two union territories over five years, starting from FY2023-24.
✓ The Centre will cover 80% of the project cost, while state governments will contribute the remaining 20%
Q. Which day is observed as the World Population Day every year?
A) 11 July B) 10 July
C) 12 July D) 9 July
Answer : A
✓ The World Population Day is observed every year on July 11 to raise awareness of global population
issues, including their relations to the environment.
✓ The day was established by the Governing Council of the United Nations Development Programme in
1989 in commemoration of the day in 1987, when the world reached the population of 5 billion.
✓ The purpose of observance of this day is to raise public awareness on the importance of various
population issues such as family planning, maternal health, gender equality, poverty and human rights.
✓ Theme of World Population Day 2023
✓ Its theme is "Unleashing the Power of Gender Equality: Elevating the Voices of Women and Girls to
Unlock the Infinite Potential of Our World."
✓ The theme underlines the importance of gender equality in harnessing the full potential of women and
girls and the immense opportunities they present for our global society.

Q. Which Countries are jointly conducted an 'Operation Broader Sword' to prevent illegal drugs shipment?
A) India and UK B) India and USA
C) India and UAE D) India and Germany
Answer : B
✓ Operation Broader Sword: Indo-US joint initiative to stop illegal shipments.
✓ India and the United States jointly launched 'Operation Broader Sword' with the goal of stopping the
illegal transportation of pharmaceuticals, equipment and precursor chemicals through the International
Mail System (IMS).
✓ The operation was aimed at disrupting the flow of illegal substances entering the United States from
India.
✓ The operation primarily focused on packages entering the United States from India through the two
major International Mail Facilities (IMF) located at New York (JFK International Airport) and Chicago
(ORD International Airport).
✓ During the operation, investigators thoroughly examined more than 1,500 shipments originating from
India, and took conclusive action on nearly 500 products. These operations primarily targeted illegal
drugs intended to treat serious illnesses.
✓ Many of the shipments were found to contain opioids and other controlled substances, highlighting the
importance of the operation in combating illegal drug trafficking

Q. 'Lord Hanuman' has been declared as the official mascot of which championship?
A) Asian Athletics Championship B) Cricket Asia Cup
C) Davis Cup D) World Athletics Championship
Answer : A

Follow us: Official Site, Telegram, Facebook, Instagram, Instamojo 617


✓ Lord Hanuman is the Official Mascot of the Asian Athletics Championships 2023 in Bangkok, Thailand.
✓ It is being organized on the 50th anniversary of the establishment of the Asian Athletics Federation.
✓ The logo of the 25th Asian Athletics Championships 2023 depicts the dedication and sportsmanship
displayed by the athletes participating in the Games.
✓ India is participating in the championship under the leadership of shot putter Tajinderpal Singh Toor and
long jumper Murali Sreesankar
Q. With which country India has started bilateral trade in rupees?
A) Nepal B) Bhutan
C) Bangladesh D) Sri Lanka
Answer : C
✓ India and Bangladesh have started bilateral trade in rupees. India and Bangladesh will conduct trade in
Indian Rupee along with the normal transaction method in US Dollars.
✓ Bangladesh is India's largest trading partner in South Asia and India is Bangladesh's second largest
trading partner in Asia.

BANGLADESH IN NEWS 2023


▪ India and Bangladesh have started bilateral trade in rupees.
▪ The 6th Indian Ocean Conference (IOC) is taking place in Dhaka, Bangladesh from May 12-13, 2023.
▪ India and Bangladesh have recently jointly launched the '50 Start-up Exchange Programme' to promote
collaboration and exchange between the start-up ecosystems of the two countries.
▪ Mohammad Shahabuddin elected President of Bangladesh .
▪ He will be the 22nd President of Bangladesh.
▪ Bangladesh will be connected to North-East by 2024.
▪ The 21st meeting of the Bangladesh–India Joint Steering Committee (BIJSC) on the power sector was
held on 4th May at Khulna, Bangladesh.
▪ a new border-haat was inaugurated between India and Bangladesh at Bholaganj under Companyganj
Upazila of Sylhet.
▪ The High Commission of India opened its 16th Visa application center in Kushtia town of Southwestern
Bangladesh.
▪ Bangladesh Power Development Board received 450 MW of electricity from Adani Power.
▪ Power is supplied from Adani's power plant in Godda, Jharkhand.
▪ India-Bangladesh Friendship Pipeline to be jointly inaugurated by PM Modi and Sheikh Hasina.
▪ Bangladesh, India, and Japan are set to hold a connectivity event in Tripura, India on April 11-12.
▪ Prime Minister Sheikh Hasina inaugurated the first submarine base of Bangladesh ‘BNS Sheikh Hasina’
at Pekua in Cox’s Bazar.
▪ NTPC Group’s total installed capacity reaches 72,304 MW with first overseas capacity addition in
Bangladesh.
▪ People’s Republic of Bangladesh
▪ Capital: Dhaka
▪ Currency : Taka
▪ Prime Minister: Sheikh Hasina Wazed
▪ President: Mohammad Shahabuddin
Q. Which Country Sheikh Talal Fahad Al Ahmad Al Sabah has been elected as the new President of the
Olympic Council of Asia (OCA)?
A) Kuwait B) Oman
C) Qatar D) Saudi Arabia

Follow us: Official Site, Telegram, Facebook, Instagram, Instamojo 618


Answer : A
✓ Sheikh Talal Fahd Al Ahmad Al Sabah of Kuwait has been elected as the new President of the Olympic
Council of Asia (OCA).
✓ Sheikh Talal was succeeded by his brother Sheikh Ahmad al-Fahd al-Sabah.
✓ Sheikh Talal succeeded his elder brother, Sheikh Ahmed Al-Fahad Al-Sabah, who led the OCA until
2021.
✓ Sheikh Ahmed was forced to step down after being found guilty of forgery and sentenced to at least 13
months in prison.
✓ Sheikh Talal defeated Kuwaiti Hussein Al-Musallam, OCA Director General and World Aquatics
President, at the OCA General Assembly in Bangkok.
✓ Olympic Council of Asia (OCA)
✓ The OCA was founded in 1982 by Sheikh Talal's father, Fahad Al-Ahmed Al-Jaber Al-Sabah.
✓ It serves as the governing body of the Games in Asia and currently consists of 45 member National
Olympic Committees.
✓ The International Olympic Committee recognized the OCA the same year it was founded in 1982.
✓ Established: 16 November 1982, New Delhi.
✓ Headquarters: Kuwait City, Kuwait
✓ Chairman: Talal Fahd Al-Ahmed Al-Sabah
✓ Membership: 45 National Olympic Committees
✓ Motto: Ever Onward

Q. 7th edition of the SALVEX exercise conducted in Kochi between India and which country?
A) United States B) United Kingdom
C) Japan D) France
Answer : A
✓ The Seventh edition of the Indian Navy – US Navy (IN – USN) Salvage and Explosive Ordnance
Disposal (EOD) exercise, SALVEX was conducted at Kochi.
✓ Objective – To enhance the skill sets of the Diving teams in a number of diverse disciplines such as mine
detection and neutralization, wreck location, and salvage.
✓ The exercise saw participation from both the navies which included the ships – INS Nireekshak and
USNS Salvor in addition to Specialist Diving and EOD teams.
Q. India and which country have recently started trade settlements in local currencies and this new initiative
will help in promoting commerce between the two countries?
A) Ethiopia B) Tanzania
C) Nigeria D) Egypt
Answer : B
✓ India and Tanzania have started trade settlements in local currencies and this new initiative will help in
promoting commerce between the two countries.
✓ India-Tanzania bilateral trade has seen very strong growth and stood at USD 6.4 billion in 2022-23.
✓ Bank of India, Bank of Baroda, and Canara Bank have operations in Tanzania.
✓ About Tanzania
✓ Tanzania, East African country situated just south of the Equator.
✓ President: Samia Suluhu Hassan
✓ Capital: Dodoma
✓ Currency: Tanzanian shilling

Follow us: Official Site, Telegram, Facebook, Instagram, Instamojo 619


Q. Which day has been designated as the World Malala Day by United Nations?
A) 11 July B) 13 July
C) 12 July D) 10 July
Answer : C
✓ The United Nations designated 12 July to be observed as World Malala Day, on July 12, 2013, in honour
of the young Pakistani human rights activist, Malala Yousafzai.
✓ The theme for World Malala Day 2023 is inspired by the book "I Am Malala", which focuses on the
influence of celebrities as heroes and role models for social progress.
✓ Malala Yousafzai and Kailash Satyarthi win the 2014 Nobel Peace Prize
✓ In 2014, aged just 17, Malala Yousafzai from Pakistan became the youngest Nobel Peace Prize winner.

Q. According to the Global Firepower report, what is the rank of India in the world's most powerful armies?
A) second B) third
C) fourth D) fifth
Answer : C
✓ According to the latest ranking of 2023 by Global Firepower, a data website containing information
related to the defense of the countries of the world, India is the fourth most powerful army country in the
world. USA is on top in this list.
✓ In this ranking, after the US, Russia and China are in second and third place respectively.
✓ 10 nations with the most powerful militaries in the world-
1. United States 2. Russia
3. China 4. India
5. United Kingdom 6. South Korea
7. Pakistan 8. Japan
9. France 10. Italy
Q. Which state cabinet has approved a proposal to recommend the inclusion of the Kui language in the 8th
schedule of the Constitution of India?
A) Odisha B) Bihar
C) Assam D) Nagaland
Answer : A
✓ The Odisha Cabinet recently recommended a proposal for inclusion of ‘Kui’ language in the 8th Schedule
of the Indian Constitution.
✓ About Kui Language:
✓ Kui (also known as Kandh, Khondi, Khond, Khondo), is a South-Eastern Dravidian language spoken
by the Kandha community.
✓ It is primarily spoken in the state of Odisha.
✓ It is closely related to other languages in the Dravidian family, such as Gondi and Kuvi. It was also
referred to as the Kuinga language during the historical period.
✓ With 941,988 registered native speakers, it figures at rank 29 in the 1991 Indian census.
✓ Script: Kui is traditionally written using the Odia script, which is also used for writing the Odia language.

Follow us: Official Site, Telegram, Facebook, Instagram, Instamojo 620


✓ 8th Schedule of the Indian Constitution:
✓ It lists the official languages of India.
✓ Although there are hundreds of languages spoken across the country, the eighth schedule recognises a
total of 22 languages as the official languages

Q. Who has been named first woman director of the 'IIT Zanzibar' campus in Tanzania?
A) Tejaswini Saraf B) Bhavya Sree
C) Preeti Aghalayam D) Swayam Shashank Chube
Answer: C
✓ The Zanzibar campus (Tanzania) of the Indian Institute of Technology (IIT) which is set to become the
first offshore campus of the prestigious institution will also be the first IIT campus to be led by a woman
director.
✓ Preeti Aghalayam to become 1st woman head of IIT, to lead Zanzibar campus.
Q. Indian Mango Festival ‘Aamras’ has been inaugurated by UP’s Minister of State Dinesh Pratap Singh in
which country?
A) Japan B) Russia
C) China D) United States
Answer : B
✓ MoS DP Singh inaugurates Indian Mango Festival ‘Aamrus’ in Russia.
✓ Minister of State (Independent Charge) for Uttar Pradesh Government, Dinesh Pratap Singh and
Ambassador of India to Russia Pavan Kapoor have participated in the inauguration of the Indian Mango
Festival ‘Aamrus’ in Moscow.
✓ In the festival, the “enthusiastic” Russian audience got to enjoy five samples of different varieties of
Mangoes from Uttar Pradesh – Dasheri, Langra, Chausa, Amrapali and Mallika.
✓ The festival was organized by the Embassy of India with the support of the Uttar Pradesh Government,
APEDA, and DISHA
Q. With whom has the Defense Ministry tied up for safe and nutritious food for the armed forces?
A) FCI B) MOFPI
C) FSSAI D) AMUL
Answer : C
✓ The Defense Ministry has signed an MoU with the Food Safety and Standards Authority of India
(FSSAI) to promote the use of millets and promote healthy eating practices among the armed forces and
ensure safe and nutritious food.
✓ It aims to create awareness among personnel about dietary diversity and nutritional benefits of millet-
based food products.
✓ FSSAI is a statutory body established under the Ministry of Health and Family Welfare, Government of
India.
Q. Who has inaugurated the indigenous Differential Global Navigation Satellite System (DGNSS) 'SAGAR
SAMPARK'?
A) Amit Shah B) Narendra Modi
C) Rajnath Singh D) Sarbananda Sonowal
Answer : D

Follow us: Official Site, Telegram, Facebook, Instagram, Instamojo 621


✓ Sarbananda Sonowal, the Union Minister of Ports, Shipping and Waterways, inaugurated the 'Sagar
Sampark' Differential Global Navigation Satellite System (DGNSS) to strengthen the maritime sector in
India.
✓ The DGNSS system is a terrestrial-based enhancement system that corrects errors and inaccuracies in the
Global Navigation Satellite System (GNSS), providing more accurate positioning information.
✓ It helps mariners navigate safely and reduces the risk of collisions, groundings, and accidents in port and
harbour areas.
✓ Enhancing Maritime Sector Capability with 'Sagar Sampark':
✓ The launch of 'Sagar Sampark - Differential Global Navigation Satellite System' at six locations enhances
the capability of the Directorate General of Lighthouses and Lightships (DGLL) in the field of Radio
Aids to Marine Navigation.
✓ This initiative reflects the Ministry's commitment to innovation, infrastructure development, and
strengthening the Indian Maritime sector.
✓ Benefits of the DGNSS System for Safe Navigation:
✓ The DGNSS system provides more accurate information to ships, enabling safe navigation and efficient
movement of vessels.
✓ It significantly improves the accuracy of GPS positioning by reducing errors caused by atmospheric
interferences, satellite clock drift, and other factors.
✓ Fulfilling International Obligations and Standards:
✓ The DGNSS system aligns with the international obligations of the International Maritime Organization
(IMO), Safety of Life at Sea (SOLAS), and the International Association of Marine Aids to Navigation
and Lighthouse Authorities (IALA).
✓ It serves as an important Radio Aid to Navigation, ensuring compliance with international standards.
✓ Advanced Features of the DGNSS System:
✓ The DGNSS system incorporates multiple satellite constellations, including GPS and Global Navigation
Satellite System (GLONASS), increasing availability and redundancy.
✓ It utilizes modern technology receivers and advanced software to transmit corrections, improving
positioning accuracy within a range of 5 meters
Q. World's largest International Temple Convention and Expo (ITCX) will be organized at the Rudraksh
Convention Center in which city?
A) Lucknow B) Prayagraj
C) Varanasi D) Kanpur
Answer : C
✓ World's largest International Temple Convention and Expo organized in Varanasi.
✓ The world's largest International Temple Convention and Expo (ITCX) will be organized from 22 to 24
July 2023 at the Rudraksh Convention Center in Varanasi.
✓ Conceived by Temple Connect (India), it is the world's first event solely dedicated to the management of
temples.
✓ The conference will focus on nurturing and empowering the governance, management and operations of
the world and temple ecosystem.
✓ Conference Participation:
✓ More than 450 temple functionaries from 25 countries are expected to participate in the conference.
✓ Along with Hindus, office bearers of Math-Temples and Gurudwaras belonging to Sikh, Buddhist and
Jainism have also been invited.
✓ Launch of the conference:
✓ The conference will be inaugurated by RSS chief Mohan Bhagwat on July 22.
✓ The event will be held at Rudraksh Convention Centre.
✓ For the first time, the International Temples Convention & Expo is being organized by Temple Connect.

Follow us: Official Site, Telegram, Facebook, Instagram, Instamojo 622


Q. The Ministry of Defense has approved the purchase of how many Rafale fighter jets for the Indian Navy?
A) 22 B) 24
C) 26 D) 30
Answer : C
✓ The Defense Ministry has approved a proposal to procure 26 naval variants of Rafale fighter jets from
France.
✓ The Defense Acquisition Council (DAC) headed by Defense Minister Rajnath Singh has approved.
✓ Which includes 26 Rafale fighter aircraft including 22 Rafale MS and 4 twin seater trainer versions.
✓ Along with this, the purchase of three Scorpene submarines has also been approved.
✓ The first batch of first 5 Rafales were delivered to Indian Air Force from France on 27 July 2020
✓ It is a French twin-engine multirole fighter aircraft designed by Dassault Aviation Company of France..
✓ India had signed deal for purchase of 36 Rafale jets from French aerospace major Dassault Aviation for
the IAF at a cost of Rs 59,000 crore.
✓ Out of 36 Rafale jets, 30 are fighter jets and six are trainers.
Q. Which state government has launched pilot project of the Antyodaya Shram Suraksha Accident
Insurance Scheme?
A) Madhya Pradesh B) Gujarat
C) Uttar Pradesh D) Uttarakhand
Answer : B
✓ Gujarat Chief Minister Bhupendra Patel and Union Minister of State for Communications Devusinh
Chauhan have launched the Antyodaya Shram Suraksha Accident Insurance Scheme from Nadia of
Kheda district in Gujarat.
✓ The scheme, launched on a pilot basis, is being implemented through the Indian Postal Department, the
Indian Post Payments Bank, and the Ministry of Labour and Employment.
✓ Gujarat became the first state in India to launch a scheme for the welfare of laborers.
✓ The objective of Antyodaya Shramik Suraksha Yojana is to give priority to the welfare and safety of our
Shram Yogis so that they can get financial security in case of an accident.
✓ In case of the laborer’s accidental death, ’10 lakh will be given to their family and the same amount will
be given to the worker in case of permanent disability. Also, in the event of the death, the deceased’s
children will be eligible for education assistance of ‘1 lakh.
Q. Which company has signed a teaming agreement with Spanish-based Navantia for the submission of a
techno-commercial bid for the Indian Navy’s prestigious P75 (India) submarine program?
A) Hindustan Aeronautics Limited B) Bharat Electronics Limited (BEL)
C) Hindustan Aeronautics Limited D) Larsen & Toubro (L&T)
Answer : D
✓ Larsen and Toubro (L&T) has signed a teaming agreement with Spanish-based Navantia for the
submission of a techno-commercial bid for the Indian Navy’s prestigious P75 (India) submarine program.
✓ For its dwindling submarine fleet, the Navy is looking to procure six advanced diesel-electric submarines
under Project 75 (I).
✓ P75 (India) Submarine Project is expected to be valued at Euro 4.8 billion.
✓ Last month, public sector shipyard Mazagon Dock Shipbuilders Limited (MDL) and Germany’s
Thyssenkrupp Marine Systems (TKMS) signed an agreement to be able to bid for producing the advanced
submarines under Project 75 (I).

Follow us: Official Site, Telegram, Facebook, Instagram, Instamojo 623


Q. Automatic train protection system developed by Indian Railway that is supposed to enhance the safety
of running train?
A) Raksha B) Avaran
C) Suraksha D) Kavach
Answer : D
✓ The railways recently confirmed that there was no ‘Kavach’ system installed on the trains involved in the
accident in Odisha's Balasore district.
✓ About Kavach System
✓ It is an indigenously developed Automatic Train Protection (ATP) system.
✓ Kavach was developed by the Research Design and Standards Organisation (RDSO) under Indian
Railway (IR) in collaboration with Medha Servo Drives Pvt Ltd, HBL Power Systems Ltd and Kernex
Microsystems.
✓ It is a set of electronic devices and Radio Frequency Identification devices installed in locomotives, in
the signalling system as well the tracks, that talk to each other using ultra-high radio frequencies to control
the brakes of trains and also alert drivers, all based on the logic programmed into them

Q. Who is not a NATO pluse five countries ?


A) Japan B) Australia
C) Egypt D) Israel
Answer : C
✓ A US Congressional Committee recently recommended strengthening Nato Plus by including India in
the five-member grouping.
✓ About Nato Plus.
✓ It is a grouping of the North Atlantic Treaty Organization (NATO) and five countries, including
Australia, New Zealand, Japan, Israel, and South Korea.
✓ The group works toward boosting global defence cooperation.
✓ Advantages for India by becoming a member of NATO Plus:
✓ India would gain access to seamless intelligence sharing between these countries.
✓ India would get access to the latest military technology without much of a time lag.
✓ It would further strengthen India’s defence partnership with the United States.
✓ What is North Atlantic Treaty Organization (NATO)?
✓ It is a transatlantic alliance of 31 like-minded North American and European countries.
✓ It was established by the North Atlantic Treaty (also called the Washington Treaty) on April 4, 1949.
✓ Aim : To protect peace and to guarantee the territorial integrity, political independence and security of
the member states.
✓ Headquarters : Brussels, Belgium
Q. Recently, the delicious Ishad mango from which state has earned its much-awaited geological indicator
(GI) tag?
A) Kerala B) Karnataka
C) Tamil Nadu D) Telangana
Answer : C
✓ Recently, the delicious Ishad mango from Uttara Kannada (Karnataka) has earned its much-awaited
geological indicator (GI) tag.
✓ About Ishad Mango.
✓ It is grown predominantly around Ankola, is tasty and contains a lot of pulp.

Follow us: Official Site, Telegram, Facebook, Instagram, Instamojo 624


✓ It is said that the mango variety has been cultivated for the last 400 years.
✓ It has two variants — Kari Ishad, which has thin skin, more pulp and is sweeter, and Bili Ishad, which
has thick skin and less pulp and sweetness.
Q. 1st state in india to bag five “Green Apple’ Environment award under the international beautiful building
category, released by London based “ The green bag”
A) Himachal Pradesh B) Uttarakhand
C) Telangana D) Madhya Pradesh
Answer : C
✓ Telangana has achieved yet another milestone by bagging five 'Green Apple Awards' in the 'International
Beautiful Buildings' under Urban and Real Estate Sector category, from London-based independent non-
profit 'The Green Organization'.

Follow us: Official Site, Telegram, Facebook, Instagram, Instamojo 625


BEST MCQ WITH FACTS
JUNE 2023

Q. Who is the architect for new Parliament building?


A) Bimal Patel B) Prasanna Pandian
C) Shiv Nath Prasad D) Ram V. Sutar
Answer : A
✓ Recently, the Indian Prime Minister has inaugurated India’s new Parliament building, part of the
revamped Central Vista project.
✓ The construction of the new building, designed by Architect Bimal Patel, began in 2019.
✓ The New Parliament House is located in the Central Vista area of New Delhi.
✓ It has been designed to accommodate the growing size and needs of the Indian Parliament.
✓ The building has seating arrangements for 888 members of the Lok Sabha and 384 members of the Rajya
Sabha.
✓ It also includes six committee rooms equipped with state-of-the-art audio-visual systems.
✓ Covering an area of approximately 65,000 square metres, the new building has a triangular shape to
optimise space utilisation.
✓ It is a four storied building constructed at an estimated cost of Rs 970 crore.
✓ The building was designed by Ahmedabad-based HCP Design, Planning and Management and
constructed by Tata Projects Limited.
✓ Prime Minister Narendra Modi also installed "Sengol" in a dedicated enclosure to the right of the
Speaker's chair in the Lok Sabha chamber.
✓ About Sengol:
✓ Sengol is made of silver and gold.
✓ It is a five feet long sceptre which was given by Lord Mountbatten to Pandit Jawaharlal Nehru on August
14, 1947 as a symbol of the transfer of power.
✓ Sengol is a symbol of rule in the Tamil kingdoms of the past.
✓ It was earlier displayed in the Allahabad Museum.
✓ Old Parliament House ( Purani Sansad Bhavan) in New Delhi was the seat of the Imperial Legislative
Council between 18 January 1927 and 15 August 1947.
✓ It was designed by British architects Sir Edwin Lutyens and Sir Herbert Baker between 1912 and 1913,
and completed in 1927.
Q. Who has won the men’s single title at the Roland Garros Tennis Tournament 2023, also known as French
Open?
A) Roger Federer B) Stefanos Tsitsipas
C) Daniil Medvedev D) Novak Djokovic
Answer : D
✓ In Tennis, Serbian Novak Djokovic won the French Open 2023 men's singles final, clinching his 23rd
Grand Slam title.
✓ Novak Djokovic has become the first man to reach the 23rd Grand Slam title.
✓ He won the title by defeating Casper Rudd of Norway in straight sets.
✓ The win gives Djokovic his third French Open title and surpasses Rafael Nadal's record for most title
wins.
✓ He overtook Rafael Nadal as the oldest Roland Garros champion at the age of 36 years and 20 days.
✓ He also became the first person to win all four Grand Slam events at least three times.

Follow us: Official Site, Telegram, Facebook, Instagram, Instamojo 626


✓ This win marked Djokovic's 23rd major men's singles crown, surpassing all other players in the history
of the sport. He also equaled Serena Williams' record for the most Grand Slam titles won in the Open
Era.
✓ In addition to 10 Australia Open, Djokovic won the French Open (3 title) in 2016, 2021 & 2023 a total
of 7 Wimbledon between 2011, 2014, 2015, 2018, 2019, 2021, 2022 while the US Open (3 title) in 2011,
2015, 2018.
✓ Novak Djokovic beat Andy Murray to win the French Open for the first time at Roland Garros on this
day in 2016.
✓ The Stade Roland Garros in Paris is hosting the Roland-Garros tournament also known as the French
Open, which took place from May 28, 2023, until June 11, 2023.

Q. Name the winner of women’s single title at French Open 2023


A) Iga Swiatek B) Elise Mertens
C) Kateřina Siniaková D) Barbora Krejcikova
Answer : A
✓ In French Open tennis, World No 1 and top seed Iga Swiatek of Poland has won the Women's Singles
crown at Paris on 10th June.
✓ She defended her title against unseeded Czech Republic’s Karolina Muchova 6-2, 5-7, 6-4. This is her
third French title in just 5 years.
✓ Poland’s Iga Swiatek became the first woman since 2007 to win back-to-back French Open titles.
✓ The 22-year-old Swiatek is just the third woman in the Open era to win each of her first four Grand Slam
finals.
✓ Monica Seles and Naomi Osaka are the only other players to achieve this feat.
✓ Swiatek, the world number 1, is also theyoungest woman to claim back-to-back French Open titles since
Monica Seles in the early 1990s.
Q. What is the rank of India in the WEF Gender Gap Index 2023?
A) 135 B) 140
C) 145 D) 127
Answer : D
✓ Global Gender Gap Index 2023: India ranked 127; topped by Iceland.
✓ According to Gender Gap Report 2023, India has been ranked 127 out of 146 countries in Gender Gap
Report 2023.
✓ In 2022, India was on 135.
✓ The Global Gender Gap Index was established in 2006 and this is its 17th edition. It is published annually
by the World Economic Forum (WEF).
✓ Iceland has been ranked first as the most gender-equal country in the world for the 14th consecutive year
with a score of 91.2%.
✓ While Norway ranked 2nd, followed by Finland and Sweden.
✓ Among India’s neighbouring countries, Pakistan is at 142, Bangladesh at 59, China at 107, Nepal at 116,
Sri Lanka at 115, and Bhutan at 103.
✓ Iceland is the only country that has reduced the gender gap by more than 90%.

Q. Which country has topped the Gender Gap Index 2023 released by WEF?
A) Noway B) Iceland
C) Singapore D) Greece
Answer : B

Follow us: Official Site, Telegram, Facebook, Instagram, Instamojo 627


✓ According to a Gender Gap Index 2023, released by World Economic Forum, India has been ranked 127
out of 146 countries in the Global Gender Gap Report 2023, up by 8 positions from 135 in 2022.
✓ For the 14th year running, Iceland (91.2%) takes the top position.
✓ While Norway ranked 2nd, followed by Finland, and Sweden

Q. What has been renamed as Nehru Memorial Museum and Library Society?
A) Kalam Memorial Museum and Library Society
B) PM Memorial Museum and Library Society
C) Mahatma Gandhi Memorial Museum and Library Society
D) Indian Memorial Museum and Library Society
Answer : B
✓ Nehru Memorial Museum and Library Society renamed as Prime Ministers’ Museum and Library
Society.
✓ The Nehru Memorial Museum and Library Society (NMML) in New Delhi has been officially renamed
as the Prime Ministers' Museum and Library Society.
✓ A decision in this regard was taken during a special meeting of the NMML Society presided over by
Defence Minister Rajnath Singh, who is its Vice-President.
✓ The idea of setting up a museum dedicated to all the prime ministers of India at the Teen Murti complex
in New Delhi was initially proposed by Prime Minister Narendra Modi in 2016.
✓ The NMML Executive Council approved the construction of the museum within Teen Murti Estate, and
the project was completed.
✓ About Nehru Memorial Museum and Library
✓ It was established in 1964 in the memory of Jawaharlal Nehru (1889-1964).
✓ It is an autonomous institute under the Ministry of Culture, Government of India.
✓ It is located in Teen Murti House, the official residence of the first Prime Minister of India.
✓ It has four major components, a Memorial Museum, a Library on Modern India, a Center for
Contemporary Studies and the Nehru Planetarium.
✓ It is a specialised library designed and developed as an exclusive research and reference centre on colonial
and post-colonial India. It has a rich manuscript section.
Q. Which country will rejoin UNESCO in July 2023 four years after leaving due to allegations of bias against
Israel?
A) Russia B) Germany
C) Japan D) United States
Answer : D
✓ The United States will rejoin UNESCO, the United Nations cultural organization, in July 2023.
✓ The move ends Washington's more than ten-year dispute with the UN cultural agency after the US left
UNESCO in 2018.
✓ US Deputy Secretary of State Richard Verma submitted a letter to UNESCO Director-General Audrey
Azoulay last week to formalize plans to rejoin.
✓ In addition, the US will also pay an outstanding balance of over $600 million
✓ America's reason for being out of UNESCO
✓ The United States exited UNESCO primarily due to the induction of Palestine as a member in 2011.
✓ This move resulted in the US suspending its funding to the agency, which amounted to millions of dollars,
during the presidency of Barack Obama.
✓ The reason behind the US halting funding was that Palestine is not recognized as a sovereign state by the
United Nations.

Follow us: Official Site, Telegram, Facebook, Instagram, Instamojo 628


✓ While Palestine was granted non-member observer state status in 2012, enabling its participation in
General Assembly proceedings, it did not possess voting rights.
✓ Why did the USA rejoin UNESCO?
✓ The absence of the United States from UNESCO allowed China to gain greater influence within the
organization.
✓ Recognizing the need to maintain a strong international presence and influence, the US has sought to re-
engage to counter China's growing authority.
✓ By rejoining, the US is seeking to regain its role in shaping and influencing UNESCO's policies.
Q. What is the name of the fourth Survey Vessel large warship of the Indian Navy which has been launched
recently?
A) Sanshodhak B) Bhima
C) Sadhak D) Ajay
Answer : A
✓ The fourth Survey Vessel Large (SVL) of the Indian Navy, named 'Sanshodhak', was launched at
Kattupalli, Chennai.
✓ About 'Sanshodhak' warship
✓ The ship's name, meaning 'Researcher', reflects its primary role as a Survey Vessel.
✓ The Ministry of Defence (MoD) highlighted the launch of the fourth SVL as a testament to India's
commitment to indigenous shipbuilding, aligning with the government's 'Make in India' and Aatma
Nirbhar Bharat (Self-Reliant India) vision.
✓ The project, undertaken by L&T/GRSE for the Indian Navy, demonstrates the nation's resolve in
promoting self-sufficiency in defense manufacturing.
✓ Purpose and features of SVL ships
✓ The SVL ships are designed to replace the existing Sandhayak Class survey ships and are equipped with
new-generation hydrographic equipment to collect oceanographic data.
✓ These ships have a length of 110 meters, a width of 16 meters, and a displacement of 3,400 tons.
✓ The hull of the ships is constructed using DMR 249-A steel, an indigenously developed material
manufactured by the Steel Authority of India Limited (SAIL).
✓ With over 80 percent indigenous content by cost, the SVL project promotes defense production by Indian
manufacturing units, contributing to employment generation and enhancing warship building
capabilities in the country.
✓ Construction and launch timeline
✓ The contract for building four SVL ships was signed between the MoD and Garden Reach Shipbuilders
& Engineers (GRSE), Kolkata, on October 30, 2018.
✓ As per the build strategy, the first ship was built at GRSE, Kolkata, while the construction of the
remaining three ships up to the outfitting stage was sub-contracted to L&T Shipbuilding, Kattupalli.
✓ The first three ships, named Sandhayak, Nirdeshak, and Ikshak, were launched on December 5, 2021,
May 26, 2022, and November 26, 2022, respectively
Q. Which railway station was recently awarded the 'Eat Right Station' certificate?
A) Varanasi Railway Station B) Guwahati Railway Station
C) Patna Railway Station D) Ahmedabad Railway Station
Answer : B
✓ Guwahati becomes first railway station in Northeast to get FSSAI's 'Eat Right Station' certification.
✓ Guwahati Railway Station located in Assam under the North East Frontier Railway recently received
the prestigious 'Eat Right Station' certification from the Food Safety and Standards Authority of India
(FSSAI).

Follow us: Official Site, Telegram, Facebook, Instagram, Instamojo 629


✓ FSSAI has acknowledged Guwahati Railway Station for its commitment to providing high-quality and
nutritious food options to passengers.
✓ As the first railway station within the North East Frontier Railway to receive this certification, Guwahati
Railway Station has successfully met the stringent guidelines laid down by FSSAI.
✓ The 'Eat Right Station' certification is a testament to the station's compliance with FSSAI's established
standards for food storage and hygiene practices.
FSSAI IN NEWS 2023
▪ Defence ministry, FSSAI to promote use of millets, healthy eating habits among armed forces.
▪ Guwahati Railway Station of the Northeast Frontier Railway has been awarded the 'Eat Right Station'
certification by FSSAI, for providing high-quality, nutritious food to passengers.
▪ Union Minister for Health and Family Welfare, Dr Mansukh Mandaviya has inaugurated the state-of-
the-art National Training Centre for Food Safety and Standards Authority of India (FSSAI) at
Ghaziabad, Uttar Pradesh.
▪ Varanasi Cantt Railway Station of the Indian Railways has been awarded the 5-Star 'Eat Right Station'
certification for providing high quality nutritious food to the passengers.
▪ The Food Safety and Standard Authority of India (FSSAI) has awarded the food provided by the
Bulandshahr prison of Uttar Pradesh a five-star rating and the tag 'Eat Right Campus'. Bulandshahr Jail
is the second prison after Farrukhabad jail to get this tag from Uttar Pradesh..
▪ G Kamala Vardhana Rao was appointed the chief executive officer (CEO) of the Food Safety &
Standards Authority of India (FSSAI).
▪ FSSAI has been established under the Food Safety and Standards Act, 2006, which is a consolidating
statue related to food safety and regulation in India.
▪ Food Safety and Standards Authority of India (FSSAI)
▪ Founded : 2008
▪ Headquarters : New Delhi
▪ Chairperson : Rajesh Bhusan
▪ CEO : G Kamala Vardhana Rao
Q. Which edition of the joint military exercise “Ex Ekuverin” between the Indian Army and the Maldives
National Defence Force has commenced at Chaubatia, Uttarakhand?
A) 10th B) 14th
C) 12th D) 15th
Answer : C
✓ The 12th edition of the joint military exercise “Ex Ekuverin” between the Indian Army and the Maldives
National Defense Force has commenced on 11 June 2023 at Chaubatia, Uttarakhand.
✓ Ekuverein, meaning 'friend', is a bilateral annual exercise held alternately in India and Maldives.
✓ The joint military exercise "EX EKUVERIN" between the two countries will be held from 11 to 24 June
2023.
✓ A platoon-strength contingent of the Indian Army and Maldives National Defense Force will participate
in the 14-day long exercise.
✓ The 14-day exercise aims to enhance interoperability in UN counter-insurgency/anti-terrorist operations
and promote joint humanitarian assistance and disaster relief operations.
✓ It will focus on sharing best practices, enhancing coordination and cooperation between the two forces
at the tactical level.
✓ The 11th edition of joint military exercise "EX EKUVERIN" was held in Maldives in December 2021.

Follow us: Official Site, Telegram, Facebook, Instagram, Instamojo 630


Q. The Education Ministry will host the 4th and final Education Working Group (EDWG) Meeting under
India’s G20 presidency in which city?
A) Chennai B) Pune
C) Hyderabad D) Bengaluru
Answer : B
✓ The Education Ministry will host the 4th and final Education Working Group (EDWG) Meeting under
India’s G20 presidency in Pune from the 19th to the 21st June.
✓ The theme of the Meeting is “Ensuring Foundation Literacy and Numeracy, especially in the context of
blended learning”.
✓ The Department of Higher Education is organizing a Seminar in collaboration with ELSEVIER on the
16th of June at IISER, Pune.
✓ The Seminar will be held on the theme ‘Accessible Science: Fostering collaboration’

PUNE IN NEWS 2023


✓ Education Ministry will host the 4th and final Education Working Group (EDWG) Meeting under
India’s G20 presidency in Pune.
✓ The city of Pune will soon hold the distinction of having the country's deepest underground metro
station..
✓ Pune's Civil Court interchange station will be 108.59 feet deep with a ceiling of 95 feet.
✓ India and 23 African nations on 21 March began a nine-day mega military exercise Africa-India Field
Training Exercise (AFINDEX) in Pune.
✓ The first joint conference of army chiefs of India and African countries was held in Pune.
✓ Maharashtra Anti-Terrorism Squad recently arrested a scientist of the Defense Research and
Development Organization (DRDO) on charges of espionage..
✓ The scientist named Pradeep Kurulkar was arrested in Pune.
✓ External Affairs Ministry’s annual flagship event on Geoeconomics, Asia Economic Dialogue began on
23 February in Pune, Maharashtra.
✓ Maharashtra’s first Electronics Manufacturing Cluster to be set up at Ranjangaon, Pune.
✓ Savitribai Phule Pune University won its first gold medal in the women's tennis event at the Khelo India
University Games.
✓ DHARA - Annual Meeting of Members of the River Cities Alliance to be held In Pune.
✓ Union Minister for Youth Affairs Anurag Singh Thakur inaugurated the fourth Y20 consultation meeting
in Pune.
✓ Talon Grikspoor of the Netherlands won the singles title at the Maharashtra Open tennis tournament in
Pune
Q. Which day of the year is celebrated as the World Environment Day (WED)?
A) 6 June B) 5 June
C) 4 June D) 3 June
Answer : B
✓ World Environment Day is an annual event celebrated on June 5th to raise global awareness about
environmental issues and encourage positive actions to protect the planet.
✓ World Environment Day 2023 is hosted by Côte d'Ivoire and supported by the Netherlands.
✓ Theme of World Environment Day 2023 is 'Solutions to plastic pollution'.
✓ The theme is chosen by the United Nations Environment Programme (UNEP) and serves as a guiding
principle for activities and discussions worldwide.

Follow us: Official Site, Telegram, Facebook, Instagram, Instamojo 631


✓ World Environment Day is observed by individuals, communities, organizations, and governments in
over 100 countries.
✓ World Environment Day was first celebrated by the United Nations in 1972.
✓ Its celebration on a global scale began on 5 June 1974 in Stockholm, the capital of Sweden, where an
environmental conference was organised in the presence of 119 countries.

Q. What is the theme of World Environment Day 2023?


A) Time For Nature B) Ecosystem Restoration
C) Beat Plastic Pollution D) Beating Plastic Pollution
Answer : C
✓ Theme 2023 – Beat Plastic Pollution
✓ The theme for this 50th edition of World Environment Day focuses on solutions to plastic pollution under
the campaign #BeatPlasticPollution.
Q. President Draupadi has been awarded the country’s highest civilian award Grand Order of the Chain of
the Yellow Star. It is the highest civilian award in which country?
A) Fiji B) Nigeria
C) Suriname D) Mauritius
Answer : C
✓ President Droupadi Murmu conferred with highest civilian honour of Suriname.
✓ Suriname awarded President Draupadi Murmu its highest civilian honour, the Grand Order of the Chain
of Yellow Star, on 5 June, making her the first Indian to receive the award.
✓ President Murmu was presented with the award by Chandrikaprasad Santokhi, President of the Republic
of Suriname.
✓ He dedicated the award to successive generations of the Indian-Surinamese community, in recognition
of the important role of the Indian diaspora in strengthening the fraternal ties between the two countries.
✓ During her state visit to Serbia and Suriname, President Draupadi Murmu arrived in Paramaribo,
Suriname on 4 June.
✓ In addition, India and Suriname signed four important Memorandum of Understandings (MoUs) in the
fields of health, agriculture and capacity building.
✓ ABOUT SURINAME
✓ Suriname is a country located on the northern coast of South America. It is one of the smallest countries
in South America.
✓ President: Chandrikapersad Santokhi
✓ Capital: Paramaribo
✓ Currency: Suriname dollar.
Q. Dennis Francis has been elected as the President of the 78th UN General Assembly (UNGA). He is from
which country?
A) Trinidad and Tobago B) Suriname
C) Guatemala D) Costa Rica
Answer : A
✓ Dennis Francis elected 78th UNGA president.
✓ Dennis Francis, a citizen of Trinidad and Tobago (TT), created history by becoming the first TT citizen
to be elected as the President of the UN General Assembly (UNGA).
✓ He will serve as the 78th President of the UNGA.

Follow us: Official Site, Telegram, Facebook, Instagram, Instamojo 632


✓ He will enforce the rules of procedure, such as opening and closing debates, regulating speaking times
for representatives, suspending or adjourning debates, and setting the agenda for the assembly.
✓ United Nations General Assembly (UNGA)
✓ It is the United Nation’s chief policy-making and representative organ and was created in 1945.
✓ It is one of the six principal organs of the United Nations (UN).
✓ It serves as the main deliberative, policy making and representative organ of the United Nations.
✓ Its powers, structure, functions and procedures are set out in Chapter IV of the United Nations Charter.
✓ Its main functions are to prepare the budget of the United Nations, appoint non-permanent members to
the Security Council, appoint the Secretary-General of the United Nations, receive reports from other
parts of the United Nations system, and make recommendations through resolutions.

Q. How many countries have been elected to the United Nations Security Council for a two-year term?
A) 4 B) 5
C) 6 D) 7
Answer : B
✓ Five new countries elected as non-permanent members of the UN Security Council.
✓ Algeria, Guyana, the Republic of Korea, Sierra Leone and Slovenia were elected non-permanent
members of the UN Security Council on 6 June for a two-year term.
✓ The newly elected members will take up their new responsibilities on January 1, 2024 and serve till
December 31, 2025.
✓ Slovenia defeated Belarus by a vote of 153 to 38, while Algeria, Guyana, Sierra Leone and the Republic
of Korea were elected unopposed.
✓ These new members will replace Albania, Brazil, Gabon, Ghana and the United Arab Emirates at present
when their two-year term ends on 31 December 2023.
✓ Election of non-permanent members.
✓ Voting is done by secret ballot and candidates have to get a two-thirds majority, or 128 votes.
✓ In all, 192 countries voted to fill five seats on the council allocated to the Africa and Asia-Pacific groups,
and one each to Eastern Europe and Latin America and the Caribbean.
✓ About United Nations Security Council (UNSC)
✓ It was established in 1945 by the United Nations Charter.
✓ It is one of the six major organs of the United Nations.
✓ The other 5 organs of the United Nations are - the United Nations General Assembly (UNGA), the
Trusteeship Council, the Economic and Social Council, the International Court of Justice and the
Secretariat.
✓ It has five permanent members - China, France, Russia, the United Kingdom, and the United States,
collectively known as the P5.
✓ Any one of them can veto a resolution.
✓ Headquarters - New York
Q. Who launched two schemes 'Amrit Dharohar' and 'MISHTI' to revive the country's wetlands and
mangroves?
A) Narendra Modi B) Amit Shah
C) Draupadi Murmu D) Rajnath Singh
Answer : A
✓ PM Modi launches Amrit Dharohar and MISHTI schemes for wetland, mangrove conservation.
✓ On the occasion of World Environment Day (5 June), PM Narendra Modi launched two schemes--Amrit
Dharohar and MISHTI (Mangrove Initiative for Shoreline Habitats and Tangible Incomes)--aimed at
reviving the country's wetlands and mangroves..

Follow us: Official Site, Telegram, Facebook, Instagram, Instamojo 633


✓ PM said in the last nine years, the number of wetlands and Ramsar sites in India has increased almost
three times as compared to earlier
✓ India currently has 75 Ramsar sites which are wetlands of international importance and have been
designated under the norms of the Ramsar Convention on Wetlands.
✓ India also has about 3% of South Asia's mangrove population.
✓ Apart from Sundarbans in West Bengal, Andaman region, Kutch and Jamnagar regions in Gujarat have
substantial mangrove cover.
✓ Amrit Dharohar Yojana
✓ The scheme was first announced by the Union finance minister Nirmala Sitharaman in the 2023-24
Union budget.
✓ The scheme will be implemented over the next three years to encourage optimum use of wetlands and
enhance biodiversity, carbon stock, ecotourism opportunities and income generation for local
communities.
✓ It will emphasise on the importance of lakes and their conservation.
✓ Amrit Dharohar aims to achieve sustainable ecosystem development over the next three years with the
help of local communities.
✓ MISHTI (Mangrove Initiative for Shoreline Habitats and Tangible Incomes).
✓ The scheme was first announced by the Union finance minister Nirmala Sitharaman in the 2023-24
Union budget.
✓ This will facilitate mangrove plantation on salt lands along the sea coast of India.
✓ The scheme will operate through “convergence between MGNREGS, Campa Fund and other sources”.
✓ It aims at intensive afforestation of coastal mangrove forests.
✓ There are such forests on both the east and west coasts of India, with the Sundarbans in Bengal being one
of the largest mangrove forests.
✓ Initially the mangrove cover will be restored in nine states across the country.
✓ The scheme envisages the development of mangroves covering nearly 540 sq km, spreading across 11
states and two union territories over five years, starting from FY2023-24.
✓ The Centre will cover 80% of the project cost, while state governments will contribute the remaining 20%.
Q. Which Airways has been named the Environmental Airline of the Year 2023?
A) Alaska Airlines B) Etihad Airways
C) Qatar Airways D) British Airways
Answer : B
✓ Etihad Airways named Environmental Airline of the Year 2023.
✓ Etihad Airways, the national airline of the UAE, has been named the Environmental Airline of the Year
2023 for the second year in a row at the Airline Rating Awards.
✓ The prestigious award acknowledges the airline's commitment to sustainable aviation through innovation
and collaboration.
✓ Etihad is ranked third in the 'Top 10 Airlines' category, which evaluates comfort, innovation, value and
safety in economy, business and first class.
✓ Etihad's sustainability strategy focuses on emissions reduction, industry alignment, collaboration with
the UAE ecosystem and proactive transparency.
✓ Key initiatives of the airline include the Greenliner program using Boeing 787 Dreamliners and the
deployment of Sustainable50 A350-1000 aircraft.
✓ Antonaldo Neves, Group CEO, Etihad Aviation Group
Q. Which missile was successfully tested by India from Odisha coast?
A) Agni prime B) Agni 5
C) Agni 4 D) Bramhos

Follow us: Official Site, Telegram, Facebook, Instagram, Instamojo 634


Answer : A
✓ Agni Prime’ ballistic missile successfully flight-tested by DRDO.
✓ New Generation Ballistic Missile ‘Agni Prime’ was successfully flight-tested by Defence Research and
Development Organisation (DRDO) from Dr APJ Abdul Kalam Island off the coast of Odisha on June
7, 2023.
✓ DRDO's successful test flight of 'Agni Prime' missile marks a significant achievement.
✓ About 'Agni Prime' Missile
✓ The missile is a two-stage canisterised missile.
✓ It is the latest and sixth variant of the Agni series missiles, developed under the Integrated Guided Missile
Development Program (IGMDP).
✓ The missile is equipped with multiple independently targetable re-entry vehicles, enabling it to deliver
warheads to separate locations. It has a range of 1,000 - 2,000 km.
✓ The missile has a diameter of 1.2 m and a height of 10.5 m.
✓ It has a payload capacity of up to 1.5 tonnes for carrying warheads.
✓ The missile is capable of performing high maneuvers while homing in on its targets.
✓ After a series of user-associated launches, these missiles will be officially inducted into the armed forces.

Q. Who has won the prestigious UNESCO Michel Batisse Award 2023 for biosphere reserve management?
A) Jagdish Sudhakar Bakan B) Chandi Prasad Bhatt
C) Sumaira Abdulali D) Jadav Payeng
Answer : A
✓ Jagdish Bakan, Ramanathapuram DFO, wins UNESCO’s 2023 Michel Batisse Award .
✓ Jagdish Sudhakar Bakan, wildlife warden and District Forest Officer (DFO) of Ramanathapuram district,
Tamil Nadu (TN) has been selected for United Nations Educational, Scientific and Cultural
Organization (UNESCO)’s 2023 Michel Batisse Award for Biosphere Reserve Management.
✓ He is the first Indian to win Michel Batisse Award.
✓ About Michel Batisse Award for Biosphere Reserve Management:
✓ The award was established in 2005 and is given once every two years during the Man and the Biosphere
Reserves (MAB) Programme Council, in memory of Dr.Michel Batisse, a French Scientist.
✓ The award carries a cash prize of USD 12000.
✓ About Jagdish Sudhakar Bakan:
✓ JagdishSudhakarBakan, 2017 batch Indian Forest Service (IFS) officer hails from Parbhani district,
Maharashtra.
✓ He is currently the Director of Gulf of Mannar Biosphere Reserve, Tamil Nadu.
Q. Which state topped the 5th State Food Safety Index among the large states?
A) Punjab B) Kerala
C) Telangana D) Tamil Nadu
Answer : B
✓ Union Minister for Health, Dr. Mansukh Mandaviya has unveiled the 5th State Food Safety Index on
the occasion of World Food Safety Day 2023.
✓ Among the larger states, Kerala secured the top ranking, followed by Punjab and Tamil Nadu.
✓ Smaller states : Goa, Manipur and Sikkim
✓ UTs : J&K, Delhi and Chandigarh
✓ The index has evaluated the performance of states and UTs across six different aspects of food safety.
✓ He has also unveiled Rapid Food Testing Kit (RAFT) portal

Follow us: Official Site, Telegram, Facebook, Instagram, Instamojo 635


Q. According to the Mercer's Cost of Living Survey 2023, which is the most expensive city for expats in
India?
A) Delhi B) Mumbai
C) Chennai D) Bengaluru
Answer : B
✓ Mumbai is the most expensive Indian city for expatriates and has been ranked at 147 in the Mercer’s
2023 Cost of Living survey out of 227 cities across five continents.
✓ According to this survey, the second and third place among expensive Indian cities are held by New
Delhi (169) and Chennai (184), respectively.
✓ The rankings of other cities listed are Bengaluru (189), Hyderabad (202), Kolkata (211), and Pune (213).
✓ Across the globe, Hong Kong, Singapore and Zurich are found to be the most expensive cities for expats.
Copenhagen and four cities from Switzerland, including Zurich, are among the global top 10.
✓ Havana is the least expensive city due to currency devaluations last year along with the Pakistani cities
of Islamabad and Karachi..
Q. Who has won the 2023 Royal Society of Literature(RSL) Christopher Bland Prize for his debut novel
titled “The Secret Diaries of Charles Ignatius Sancho”?
A) Philip Ardagh B) Simon Armitage
C) Richard Dawkins D) Paterson Joseph
Answer : D
✓ British Writer Paterson Joseph wins 2023 RSL Christopher Bland Prize.
✓ British actor and writer Paterson Joseph has won the 2023 Royal Society of Literature(RSL) Christopher
Bland Prize for his debut novel titled “The Secret Diaries of Charles Ignatius Sancho”.
✓ The book is a historical fiction about Charles Ignatius Sancho, who was the first Black man to vote in
England.
✓ The jury for RSL Christopher Bland Prize 2023: Lemn Sissay (chair), Meena Kandasamy and Simon
Savidge.
✓ RSL Christopher Bland Prize is an annual award which honours a debut writer for their work of fiction
or non-fiction, who is first published at the age of 50 or over.
✓ The award carries a cash prize of 10,000 Pound sterling.
Q. Which organization is going to start its largest air force deployment exercise in Europe, which will be
known as "Air Defender 23"?
A) ASEAN B) NATO
C) SCO D) G20
Answer : B
✓ NATO plans German-led air force exercise, "Air Defender 23".
✓ NATO is set to commence its largest air force deployment exercise in Europe, known as "Air Defender
23," according to German and US officials.
✓ The exercise will span approximately 10 days, starting from 12th June, and will involve the participation
of around 220 military aircraft from 25 NATO and partner countries.
✓ The exercise will be conducted across Germany, with some of the training taking place at Ramstein Air
Base, one of NATO's largest air bases.
✓ Air Defender 23
✓ The exercise is purely defensive in nature.

Follow us: Official Site, Telegram, Facebook, Instagram, Instamojo 636


✓ Its primary objective is to enhance the defense capabilities of participating nations and ensure
preparedness to respond to potential security challenges.
✓ The exercise also intends to send a message to other countries including Russia.
✓ The demonstration of NATO's unity and collective defense posture serves as a reminder of the Alliance's
commitment to address potential threats and maintain regional stability.
✓ By conducting this exercise, NATO seeks to improve interoperability and strengthen the effectiveness of
its air defense capabilities.

NATO IN NEWS 2023


▪ NATO Plus: It is a grouping of the North Atlantic Treaty Organization (NATO) and five countries,
including Australia, New Zealand, Japan, Israel, and South Korea.
▪ NATO is set to commence its largest air force deployment exercise in Europe, known as "Air Defender
23," according to German and US officials.
▪ Finland has become the 31st member of the North Atlantic Treaty Organization (NATO).
▪ "Locked Shields" cyber-defense exercise 2023 to be held by NATO in Tallinn (Estonia).
▪ NATO launches Arctic exercises, pledges protection of Finland.
▪ The founding members of NATO were the United States, Canada, and ten European countries, including
Belgium, Denmark, France, Iceland, Italy, Luxembourg, the Netherlands, Norway, Portugal, and the
United Kingdom.
▪ North Atlantic Treaty Organization (NATO)
▪ Formation : 4 April 1949
▪ Headquarters : Brussels, Belgium
▪ Secretary General : Jens Stoltenberg (Norway)
▪ Total Member : 31 (Finland)
▪ North Atlantic Treaty, also referred to as the Washington Treaty.
Q. Who launched the Girl Empowerment Mission (GEM)-2023?
A) NHPC B) GAIL
C) Coal India D) NTPC Kanti
Answer : D
✓ NTPC Kanti launches Girl Child Empowerment Mission (GeM)-2023.
✓ The program is a four-week residential workshop aimed at empowering 40 underprivileged rural girls
from Kanti Block.
✓ This initiative is the first of its kind organised by NTPC Kanti.
✓ The primary objective of the GeM program is to provide academic training, skill development and overall
personality enhancement to the participants.
✓ The program focuses on inculcating curiosity among the participants and developing better
communication and social skills.
✓ By empowering underprivileged rural girls, NTPC Kanti aims to contribute to their holistic development
and provide them opportunities for a better future.
✓ The GeM-2023 program reflects NTPC Kanti's commitment to social welfare and dedication to support
marginalised communities
Q. Which country has named 'Biparjoy Cyclone' in the Arabian Sea?
A) India B) Sri Lanka
C) Pakistan D) Bangladesh
Answer : D
✓ The India Meteorological Department (IMD) has recently issued an advisory for all the ports of Gujarat.

Follow us: Official Site, Telegram, Facebook, Instagram, Instamojo 637


✓ In the advisory, it has been directed to issue a Distant Warning (DW 2) signal in case a deep pressure in
the Arabian Sea intensifies into a cyclonic storm Biperjoy.
✓ How was Cyclone Biparjoy named?
✓ The cyclone was named "Biparjoy" by Bangladesh.
✓ The World Meteorological Organization (WMO) follows an alphabetical order for naming tropical
cyclones, with member countries submitting names.
✓ Bangladesh proposed the name "Biparjoy," which translates to "Disaster" in Bengali.
✓ The next cyclone after Biparjoy will be named ‘Tej’ based on India’s suggestion.
✓ Cyclones in the Arabian Sea.
✓ The month of June is considered favorable for the formation of cyclones in the Arabian Sea.

Q. In which country will the Miss World pageant 2023 be organized?


A) India B) USA
C) China D) Germany
Answer : A
✓ Miss World pageant will be organized in India after a gap of 27 years.
✓ The last time this international pageant was organized in India in the year 1996.
✓ Julia Morley, CEO and Chairperson of Miss World Organization has given information about this.
✓ This will be the 71st edition of Miss World pageant.
✓ The current Miss India World Sini Shetty will represent India in this year's Miss World pageant.
✓ The current Miss World is Karolina Belwańska of Poland

Q. Which country will release the world’s first digital government bond?
A) Russia B) Israel
C) South Korea D) Germany
Answer : B
✓ Israel to release world’s first digital government bond.
✓ The Tel Aviv Stock Exchange (TASE) and Israel’s Finance Ministry successfully completed the proof-
of-concept phase for a digital Israeli Bond traded on a dedicated blockchain platform.
✓ The development positions Israel to release the world’s first digital government bond.
✓ The blockchain used for this event was EVM-compatible, enabling potential integration with other
blockchain solutions in the future.
Q. India will host the 2025 International Institute of Administrative Sciences (IIAS) Annual Conference at
Which State?
A) Gujarat B) Telangana
C) Uttar Pradesh D) Kerala
Answer : D
✓ India expressed its willingness to host the 2025 IIAS (International Institute of Administrative Sciences)
Annual Conference at Kochi, Kerala, in February, 2025.
✓ The IIAS established in 1930, is a federation of member States, national sections and academic research
centres jointly elaborating public administration solutions to the policy challenges of the day with
headquarters in Brussels, Belgium.
✓ The 2025 Annual Conference of IIAS would be attended by 30 Member Countries, 18 National Sections
and over 50 IIAS Universities/ Institutes of Public Administration in Member Countries.

Follow us: Official Site, Telegram, Facebook, Instagram, Instamojo 638


✓ Theme : In accordance with India’s governance model, the theme for 2025 IIAS Annual Conference at
Kochi is proposed as “Next Generation Administrative Reforms - Empowering the Citizens and
Reaching the Last Mile”.

Q. Which state has recently celebrated the Raja festival?


A) Assam B) Odisha
C) Haryana D) Rajasthan
Answer : A
✓ Raja or Mithun Sankranti is a three-day festival of womanhood celebrated in Odisha, India.
✓ Duration and name of festival days.
✓ Raja Utsav is celebrated for three days.
✓ Day 1: 'The First King'
✓ Day 2: 'Majhi Raja' or 'Raja Sankranti' or 'Mithuna Sankranti'
✓ Third Day: 'Bhoodah' or 'Bassi Raja' or 'Shesh Raja'.
✓ Festival of Agriculture and Worship of Mother Earth
✓ This is a unique festival of Odisha where Mother Earth is revered.
✓ Agricultural activities like ploughing and digging are suspended during the festival.
✓ Like a menstruating woman, Mother Earth is given rest without any disturbance.
✓ Significance of Raja Mahotsav in relation to monsoon
✓ 'Pahili Raja' falls on the last day of the month 'Jyestha', while 'Raja Sankranti' falls on the first day of the
month 'Ashadha'.
✓ 'Raja Sankranti' marks the beginning of the rainy season.
✓ The ritual of 'Basumati Snan'
✓ The fourth day of the festival is called 'Basumati Snan'.
✓ Women, after completing three days of symbolic menstruation, take a ceremonial bath for the grinding
stone.
✓ The grinding stone represents Mother Earth or Basumati.
Q. A Book titled Rebels Against the Raj: Western Fighters for India’s Freedom has won the Elizabeth
Longford Prize for Historical Biography 2023. Who is the author of this book?
A) Jhumpa Lahiri B) Vikram Seth
C) Aravind Adiga D) Ramchandra Guha
Answer : D
✓ Historian and writer Ramchandra Guha’s book Rebels Against the Raj: Western Fighters for India’s
Freedom has won the Elizabeth Longford Prize for Historical Biography 2023.
✓ Guha has been awarded £5,000 (₹5 lakhs approximately) and a bound copy of Elizabeth Longford’s
memoir, The Pebbled Shore.
✓ The award was founded in 2003 by Flora Fraser and Peter Soros to commemorate British historian
Elizabeth Longford. Every year it rewards exemplary works in historical biography. The prize is
celebrating its 20th anniversary this year
Q. Which Indian documentary has won a special prize in the ‘Health for All’ category at the 4th Annual
Health for All Film Festival (HAFF) held at the World Health Organization (WHO) headquarters in
Geneva?
A) One in 36 Million B) The Man Who Sold His Skin
C) When Climate Change Turns Violent D) Communicating for health impact
Answer : C

Follow us: Official Site, Telegram, Facebook, Instagram, Instamojo 639


✓ WHO Award for 'When Climate Change Turns Violent.
✓ The documentary titled 'When Climate Change Turns Violent' has bagged the special award in the
'Health for All' category at the 4th annual Health for All Film Festival held at the World Health
Organization Headquarters in Geneva.
✓ This documentary has been directed by Vandita Saharia from Rajasthan.
✓ She was the only Indian in the list of winners.
✓ The 4.32-minute documentary 'When Climate Change Turns Violent' focuses on the relationship between
domestic violence, climate change and human trafficking.
✓ The ceremony announced the winning films for seven different categories, while four films received
special mentions by the jury.
✓ World Health Organization (WHO)
✓ Formation : 7 April 1948
✓ Headquarters : Geneva, Switzerland
✓ First President : Andrija Štampar
✓ Director General : Tedros Adhanom (Ethiopia)
✓ Deputy Director General : Soumya Swaminathan (Indian)
✓ Member Countries : 194.
Q. Which has recently been recognized as the most trusted electronic media organization in the country?
A) DD India and All India Radio B) Zee TV and Disney Star
C) StarPlus and Colors TV D) Zee News and Sony Max
Answer : A
✓ Akashvani and Doordarshan emerge as most trusted electronic media organization in country.
✓ According to Reuters Institute's Digital News Report 2023, DD India and All India Radio have been
recognized as the most trusted electronic media organizations in the country.
✓ The report highlights that although overall trust in news has decreased marginally by 3 percentage points,
public broadcasters and print brands have managed to maintain a relatively high level of trust among the
public.
✓ DD India and All India Radio have been specifically described as electronic media organizations with
the highest degree of trust.
Q. Which state has celebrated the Palkhi festival by Warkari community?
A) Assam
B) Rajasthan
C) Maharashtra
D) Gujarat
Answer : C
✓ Warkari community has celebrated the Palkhi festival in Pandharpur, Maharashtra.
✓ Palkhi festival is an annual yatra to Pandharpur in Maharashtra to honor the Hindu god Vithoba.
✓ The Palkhi festival is a tradition that dates back 1000 years and was initiated by saints from Maharashtra,
India.
✓ The Palkhi festival takes place in the month of Jyeshth (June) and lasts for a total of 22 days. It involves
a journey from the departure points to Pandharpur. It is still practiced today by their followers known as
varkaris.
✓ Devotees take a holy dip in the sacred Chandrabhaga River before visiting the Vithoba temple.

Follow us: Official Site, Telegram, Facebook, Instagram, Instamojo 640


Q. A recent survey conducted by the Geological Survey of India (GSI) has led to the discovery of India's
largest natural arch in the Kenduadihi block of Sundargarh district in which state?
A) Assam B) Odisha
C) West Bengal D) Chhattisgarh
Answer : B
✓ India’s biggest natural arch formed 184 million years ago discovered in Odisha by GSI.
✓ The GSI's state unit has proposed to designate the natural arch in the Kanika range of Sundargarh forest
division as a Geo Heritage Site, making it the country's largest natural arch with the Geo Heritage tag.
✓ Currently, India has two other natural arches—one located in the Tirumala hills of Tirupati and the other
in the Andaman and Nicobar Islands.
✓ However, both of these arches are smaller in size compared to the one found in Sundargarh.
Q. How many National Water Awards for the year 2023 have been presented by Vice President Jagdeep
Dhankhar?
A) 44 B) 41
C) 51 D) 35
Answer : B
✓ Vice President Jagdeep Dhankhar presented the fourth National Water Awards in New Delhi.
✓ The awards, announced by the Jal Shakti Ministry, recognized 41 winners across 11 categories for their
outstanding contributions to water conservation in India in 2022.
✓ Madhya Pradesh received the first prize in the best State category, while the best district award went to
Ganjam District in Odisha.
✓ The Jagannadhapuram Village Panchayat in Bhadradri Kothagudem district of Telangana was honored
with the best Village Panchayat award.
✓ Each winner was presented with a citation, a trophy, and cash prizes.
✓ National Water Awards
✓ The National Water Awards aim to acknowledge and encourage the efforts made by individuals and
organizations in realizing the government's vision of a water-prosperous India.
✓ They also seek to create awareness about the importance of water and promote the adoption of best
practices for its sustainable use.
✓ Vice-President stressed the significance of adopting the three Rs - Reduce, Reuse, Recycle - to revive
ancient water harvesting systems such as ponds.
✓ Union Jal Shakti Minister Gajendra Singh Shekhawat shared that approximately 56,000 Amrit Sarovars
(sacred ponds) have been constructed in the country.
Q. Which state has topped under the Best State category at the fourth National Water Awards?
A) Uttar Pradesh B) Madhya Pradesh
C) Karnataka D) Gujarat
Answer : B
✓ Madhya Pradesh received the first prize in the best State category, while the best district award went to
Ganjam District in Odisha.
✓ The Jagannadhapuram Village Panchayat in Bhadradri Kothagudem district of Telangana was honored
with the best Village Panchayat award.

Follow us: Official Site, Telegram, Facebook, Instagram, Instamojo 641


Q. Which will be awarded the Gandhi Peace Prize for 2021 for his extraordinary contribution to social,
economic, and political transformation through non-violent and Gandhian methods?
A) Ayana Charitable Trust B) Prarthana Samaj
C) Gita Press Gorakhpur D) Church of God Chenna
Answer : C
✓ Gita Press, Gorakhpur, awarded Gandhi Peace Prize for 2021.
✓ Gita Press, Gorakhpur, will be awarded the Gandhi Peace Prize for 2021 in recognition of its exceptional
contribution to social, economic, and political transformation through non-violent and Gandhian
methods.
✓ The jury, headed by Prime Minister Modi, unanimously selected Gita Press as the recipient of the Gandhi
Peace Prize.
✓ PM Modi highlighted Gita Press's efforts in promoting Gandhian ideals of peace and social harmony,
emphasizing that the award signifies recognition of their community service after a century of
establishment.
✓ About Gita Press
✓ Established in 1923, Gita Press is renowned as one of the world's largest publishers, having published
41.7 crore books in 14 languages, including 16.21 crore copies of the Shrimad Bhagavad Gita.
✓ About Gandhi Peace Prize
✓ The Gandhi Peace Prize, instituted in 1995 on the occasion of Mahatma Gandhi's 125th birth
anniversary, is an annual award that pays tribute to Gandhi's ideals.
✓ The award is open to individuals irrespective of nationality, race, language, caste, creed, or gender.
✓ The prize includes a monetary amount of ₹1 crore, a citation, a plaque, and an exquisite traditional
handicraft/handloom item.
✓ Previous recipients of the Gandhi Peace Prize include Sultan Qaboos Bin Said Al Said of Oman in 2019
and Bangabandhu Sheikh Mujibur Rahman of Bangladesh in 2020.
Q. Who has been awarded the 45th European Essay Prize for lifetime achievement by the Charles Veillon
Foundation?
A) Jhumpa Lahiri B) Arundhati Roy
C) Kiran Desai D) Shashi Deshpande
Answer : B
✓ Arundhati Roy received the 45th European Essay Prize for Lifetime Achievement and the award is
presented by the Charles Whelan Foundation.
✓ Arundhati Roy's book of essays, 'Azadi' (2021), especially the French translation, received acclaim.
✓ 'Azaadi' explores the concept of freedom in an increasingly totalitarian world, with an emphasis on fiction
and alternative imaginations.
✓ Roy is known for his notable works, including the Booker Prize-winning novel The God of Small Things.
✓ The award ceremony will be held on September 12 in Lausanne, Switzerland.
✓ Past recipients of the European Essay Prize include Alexander Zinoviev, Amin Malouf and Siri Hustvedt.
✓ European Essay Prize:
✓ It is the first literary award dedicated solely to the genre of the essay.
✓ The award recognizes outstanding essays that criticise current societies, their practices and ideologies.
✓ It has been awarded since 1975 highlighting its long standing presence in the literary scene.
✓ Award-winning essays shed light on social norms, political structures, cultural practices, and
socioeconomic challenges
✓ The first Indian to win the Booker Prize was the author and activist Arundhati Roy. Her first novel 'God
of Small Things' in 1997.
✓ Famous Books Written by Arundhati Roy

Follow us: Official Site, Telegram, Facebook, Instagram, Instamojo 642


✓ The God of Small Things.
✓ The Ministry of Utmost Happiness.
✓ The Doctor and the Saint.
✓ Azaadi.
✓ Capitalism.
✓ An Ordinary Person’s Guide to Empire.
✓ Walking with the Comrades.
✓ The Algebra of Infinite Justice.
✓ Broken Republic.
✓ Listening to Grasshoppers.

Q. Who has been appointed as the new chief of Research and Analysis Wing (RAW)?
A) Ravi Sinha B) Ajay Sinha
C) Vivek Kala D) Vinod Lahori
Answer : A
✓ Ravi Sinha, a 1988 batch IPS officer of the Chhattisgarh cadre, has been selected as the next chief of the
Research and Analysis Wing (RAW)..
✓ Sinha will take over from current chief Samant Kumar Goel, whose term ends on June 30, 2023.
✓ Presently, Ravi Sinha is holding the post of Special Secretary in the Cabinet Secretariat
✓ Research and Analysis Wing (RAW):
✓ It is the foreign intelligence agency of India.
✓ RAW's primary function is to gather foreign intelligence.
✓ It is responsible for counter-terrorism and counter-proliferation efforts.
✓ RAW advises Indian policy makers on strategic matters.
✓ Established - 21 September 1968
✓ Founder - RN Kao, Indira Gandhi
✓ Headquarters - New Delhi
✓ Officeholder - Samant Goyal (Director of Intelligence Agency)
Q. India has gifted indigenously built missile corvette 'INS Kirpan' to which country?
A) Vietnam B) Bangladesh
C) Bhutan D) Sri Lanka
Answer : A
✓ India gifts missile corvette INS Kirpan to Vietnam.
✓ Defense Minister Rajnath Singh on 19 June announced the gift of an indigenous in-service missile
corvette, INS Kirpan, to the Vietnam People's Navy.
✓ The announcement is expected to significantly enhance the capabilities of the Vietnamese Navy.
✓ About INS Kirpan
✓ INS Kirpan is a missile corvette belonging to the Khukri class with a displacement of about 1,350 tonnes.
✓ It was commissioned into the Indian Navy on 12 January 1991.
✓ The length of the corvette is 91 meters and the beam is 11 metres.
✓ It is capable of attaining speeds of over 25 knots.
✓ Equipped with a variety of weapons including medium-range cannon, 30 mm close-range gun, chaff
launcher and surface-to-surface missiles, INS Kirpan has the versatility to play multiple roles.
✓ The roles performed by INS Kirpan include coastal and offshore patrolling, coastal security, surface
warfare, anti-piracy operations and Humanitarian Assistance and Disaster Relief (HADR) operations.

Follow us: Official Site, Telegram, Facebook, Instagram, Instamojo 643


Q. Which author has been awarded the prestigious 'German Peace Prize'?
A) Chetan Bhagat B) Salman Rushdie
C) Arundhati Roy D) Amish Tripathi
Answer : B
✓ Salman Rushdie wins German Book Trade 2023 Peace Prize.
✓ The Peace Prize of the German Book Trade for 2023 has been awarded to British-American author
Salman Rushdie.
✓ Birth and Background: Salman Rushdie was born on June 19, 1947 in Bombay (now Mumbai), India.
✓ The Satanic Verses: Rushdie achieved international recognition through his controversial 1988 novel,
The Satanic Verses.
✓ History of the Peace Prize of the German Book Trade
✓ It was established in 1950 and the award recognizes individuals who have made significant contributions
to the promotion of international understanding between nations and cultures through their literary
works.
✓ The prize is endowed with €25,000 ($27,300).
✓ The award ceremony takes place in October during the Frankfurt Book Fair, one of the largest and most
important book fairs in the world.
✓ The purpose of the award is to encourage efforts to promote peace, dialogue and cooperation.
✓ Past recipients of the award include Margaret Atwood, Orhan Pamuk, Susan Sontag, Amos Oz, and
Vaclav Havel.
✓ Last year, the Peace Prize of the German Book Trade was awarded to Ukrainian author Serhiy Zhadan.
✓ List of famous books by Salman Rushdie:
✓ Midnight's Children
✓ Shame
✓ The Satanic Verses
✓ fury
✓ Shalimar The Clown
✓ The Golden House
✓ Quichotte
✓ Victory City
Q. Which pair won the doubles title of Indonesia Badminton Open?
A) Satwiksairaj Rankireddy and Chirag Shetty
B) B. Sai Praneeth and Lakshya Sen
C) Chirag and Lakshya Sen
D) Shrikant Kidambi and Satwik
Answer : A
✓ Satwik-Chirag become men's doubles champions at Indonesia Open 2023.
✓ Satwiksairaj Rankireddy and Chirag Shetty became the first Indian pair to win the doubles title at a BWF
Super 1000 event at the Indonesia Open 2023.
✓ They defeated the Malaysian world champion pair of Aaron Chia and Soh Wooi Yik in the final.
✓ The win made them the first doubles team from India to win the Indonesia Open.
✓ They also became the first pair from India to win a Badminton World Federation (BWF) Super 1000
title.
✓ Indonesia Open 2023 took place at Istora Gelora Bung Karno in Jakarta, Indonesia
✓ Recently, Satwiksairaj Rankireddy and Chirag Shetty created history by becoming the first Indian men's
doubles pair to win a gold medal at the Badminton Asia Championships.

Follow us: Official Site, Telegram, Facebook, Instagram, Instamojo 644


Q. UK's largest financial services providers 'Lloyds Banking Group', would set up a technology center in
Which City?
A) Chennai B) Bengaluru
C) Visakhapatnam D) Hyderabad
Answer : D
✓ Lloyds Banking Group, one of the UK’s largest financial services organisations, said on 21st June 2023
that it is setting up a technology centre in Hyderabad and will recruit 600 people by the end of 2023.
✓ Lloyds’s captive will come up in the Knowledge City. The group, which has brands such as Lloyds Bank,
Halifax, and Bank of Scotland under its belt, said it plans to expand its digital capabilities through the
tech centre.
✓ The new captive is part of the group’s strategic investment of £3 billion over the next three years to
transform its digital offerings.

Q. Who has been appointed as the new CEO of the Unique Identification Authority of India (UIDAI)?
A) N. K. Sodhi B) P. Vishwanatha Shetty
C) Krishnamurthy Subramanian D) Amit Agarwal
Answer : D
✓ Amit Agarwal has assumed charge as the Chief Executive Officer (CEO) of the Unique Identification
Authority of India (UIDAI).
✓ Amit Agarwal is a senior IAS officer of 1993 batch of Chhattisgarh cadre.
✓ He is a graduate of the Indian Institute of Technology, Kanpur.
✓ He has held important positions at the Center and in the states of Chhattisgarh and Madhya Pradesh.
✓ Before becoming the CEO of UIDAI, he served as Additional Secretary in the Ministry of Electronics
and Information Technology (MeitY).
✓ About Unique Identification Authority of India (UIDAI)
✓ Establishment and Authorization:
✓ Unique Identification Authority of India (UIDAI) is a government department established on 12 July
2016.
✓ It works under the Ministry of Electronics and Information Technology.
✓ UIDAI is a statutory authority created under the Aadhaar Act 2016.
✓ Objectives and Functions:
✓ UIDAI provides a 12 digit unique identification number called "Aadhaar" to all Indian residents.
✓ Aadhaar serves as a universal identity for residents across the country.
✓ UIDAI links the Aadhaar number with basic demographic and biometric information of residents.
✓ This information includes a photograph, ten fingerprints and two iris scans.
✓ The data is stored in a secure, centralized database managed by the UIDAI
Q. Who has been awarded Olympic Order by the International Olympic Committee (IOC)?
A) Tedros Adhanom Ghebreyesus B) Audrey Azoulay
C) Okonjo-Iweala D) António Guterres
Answer : A
✓ WHO Director General Tedros Adhanom Ghebreyesus
✓ awarded Olympic Order by IOC.
✓ The International Olympic Committee (IOC) awarded the Olympic Order to Dr. Tedros Ghebreyesus,
Director General of the World Health Organization (WHO).

Follow us: Official Site, Telegram, Facebook, Instagram, Instamojo 645


✓ The award was given to Dr. Tedros for his efforts in holding the Olympic Games Tokyo 2020 despite the
challenges of the COVID-19 pandemic.
✓ The Olympic Order was presented by IOC President Thomas Bach at the Olympic House.
✓ Dr. Tedros has been invited by President Bach to be the Guest of Honor at the Olympic Games Paris
2024.
✓ The award recognizes Dr. Tedros' support in promoting cooperation between the IOC and WHO, in
particular the Cooperation Agreement signed in 2020.
✓ The IOC and WHO have collaborated on various initiatives, including the Let's Move campaign for
Olympic Day.
✓ About the Let's Move campaign
✓ It stresses the importance of physical activity and regular exercise.
✓ It highlights the benefits associated with regular exercise, such as improved physical fitness and mental
health.
✓ The campaign is in line with the International Olympic Committee's (IOC) Olympism365 strategy, which
aims to increase access to sporting opportunities.
✓ The Let's Move campaign supports the IOC's aim of increasing access to sport for people of all ages and
backgrounds
✓ International Olympic Committee:
✓ Founded - 23 June 1894, Paris, France
✓ Founders - Pierre de Coubertin and D. Bikélas
✓ Headquarters - Lausanne, Switzerland
✓ President - Thomas Bach
✓ Director General - Christophe De Kepper.
✓ World Health Organization (WHO)
✓ Formation : 7 April 1948
✓ Headquarters : Geneva, Switzerland
✓ First President : Andrija Štampar
✓ Director General : Tedros Adhanom (Ethiopia)
✓ Deputy Director General : Soumya Swaminathan (Indian)
✓ Member Countries : 194.
Q. Which is the first Baltic country to recognize same-sex marriage?
A) Latvia B) Estonia
C) Lithuania D) Iceland
Answer : B
✓ Estonia becomes 1st Baltic nation to legalize same-sex marriage.
✓ Estonia’s parliament approved a law legalizing same-sex marriage, making it the first central European
country to do so.
✓ The law will come into effect in 2024.
✓ The bill received 55 votes in the 101-seat parliament, from the coalition of liberal and social democratic
parties.
✓ 53 percent of the population supported same-sex marriage in a 2023 poll by the Centre for Human Rights.
✓ Same-sex marriage is legal in much of Western Europe but not in central European countries which were
once under communist rule and members of the Moscow-led Warsaw Pact alliance but now members of
NATO and, largely, the EU.
Q. Who has been elected as the new Prime Minister of Finland?
A) Sanna Marin B) Antti Rinne
C) Petri Orpo D) Alexander Stubb

Follow us: Official Site, Telegram, Facebook, Instagram, Instamojo 646


Answer : C
✓ Petteri Orpo has been elected as the new Prime Minister of the country by the Parliament of Finland.
✓ In Finland, the leader of the Conservative Party, Petteri Orpo, was elected Prime Minister of the country
by the parliament.
✓ Orpo's election was supported by 107 members of parliament, while 81 opposed and 11 abstained.
✓ President Sauli Niinisto is set to formally appoint Petteri Orpo as the new Prime Minister.
✓ Orpo will replace Sanna Marin of the Social Democrats, who finished third in the polls.
✓ About Finland
✓ It is a northern European nation.
✓ It shares borders with Sweden, Norway and Russia.
✓ The capital of Finland is Helsinki.
✓ Helsinki is situated on a peninsula and is surrounded by islands in the Baltic Sea.

Q. Who will take over as the Chief Executive of the Alibaba Company?
A) Eddie Woo B) Eddie Wu
C) Jin yuanying D) Peng Lei
Answer : B
✓ Eddie Wu has been appointed as the new CEO and Joseph Tsai as Executive Chairman by Alibaba
Group.
✓ Eddie Wu is one of the co-founders of Alibaba and currently serves as the chairman of Taobao and Tmall
Group.
✓ This leadership change is the second major change for Alibaba in recent years after Jack Ma stepped
down in 2019.
✓ Alibaba CEO and Chairman Zhang to step down to focus on cloud business.
✓ Alibaba sells $125 million stake in Paytm through block deal.
✓ About Alibaba
✓ Tt is a Chinese multinational technology company.
✓ It specializes in e-commerce, retail, internet and technology.
✓ Alibaba operates in a variety of sectors including e-commerce, retail, Internet services and technology
solutions.
✓ The company's primary strength lies in its e-commerce platforms such as Alibaba.com, Taobao, Tmall
and AliExpress.
✓ Alibaba facilitates businesses and consumers to connect, buy and sell a wide range of products and
services.
✓ Alibaba Cloud, Alibaba's cloud computing division, provides infrastructure, data storage and AI
capabilities.
✓ Founded - 4 April 1999
✓ Headquarters - Hangzhou, China
✓ Founders - Jack Ma, Joseph C. Tsai, Zhang Ying, Jianhang Jin
Q. Which University has topped the Times Higher Education (THE) Asia University Rankings 2023
globally?
A) University of Hong Kong B) Tsinghua University
C) Peking University D) National University of Singapore
Answer : B
✓ 11th THE’s Asia University Rankings 2023: IISc top Indian institutes, Overall Tops by China’s Tsinghua
University.

Follow us: Official Site, Telegram, Facebook, Instagram, Instamojo 647


✓ 11th edition of ‘Times High Education (THE) Asia University Rankings 2023 has named Indian Institute
of Science (IISc) in Bengaluru (Karnataka) as the highest-ranked Indian institution in Asia. Overall it is
ranked at 48th in the Asian Region.
✓ On the whole regional front i.e. Asia, the list is topped by China’s Tsinghua University, and Peking
University at 1st and 2nd positions for the 4th year in a row.
✓ Singapore’s National University of Singapore is ranked at 3rd place for the 3rd consecutive year.
✓ Notably, IISc 2023 position at 48th is the lowest rank since 2015, when it first entered the ranking at 34th
spot. It was ranked 42nd in 2022.
Q. Which institution in India has topped among the Indian Universities in the Times Higher Education
(THE) Asia University Rankings 2023 ?
A) IIT Delhi B) IIT Ropar
C) IIT Indore D) IISC, Bengaluru
Answer : D
Q. Which of the following companies has successfully conducted the Flight Qualification Test of
the Raman-I Engine at the Liquid Propulsion Systems Centre (LPSC) of Indian Space Research
Organisation (ISRO), in Benglauru, Karnataka?
A) AgniKul Cosmos B) Skyroot Aerospace
C) Dhruva Space D) ABL Space Systems
Answer : B
✓ Skyroot successfully tested Raman-I Engine designed to use in Vikram-I rocketSkyroot Aerospace.
✓ Skyroot Aerospace, Hyderabad (Telangana) based private Indian Space company, successfully
conducted the Flight Qualification Test of the Raman-I Engine at the Liquid Propulsion Systems Centre
(LPSC) of Indian Space Research Organisation (ISRO), in Benglauru, Karnataka.
✓ ‘Raman’ is a series of engines or thrusters which uses the Earth storable hypergolic (MMH/N2O4)
propellants for upper stage engine and attitude or reaction control in Skyroot’s Vikram-1 launch vehicle
(rocket).
✓ The Raman-I engine underwent a test firing at the LPSC of ISRO, marking Skyroot as the first Indian
private company to launch a rocket with Vikram-S.
✓ The indigenous Vikram-S (named after Vikram Sarabhai, the father of India’s space programme) was
launched in November 2022, from the ISRO’s Satish Dhawan Space Center in Sriharikota, Andhra
Pradesh, marking India’s first privately developed rocket.
✓ The three variants of the Vikram rocket of Skyroot: Vikram-I can carry 480 Kilograms (kg) of payload to
Low Earth Orbit, Vikram-II is designed to lift off with 595 kg of cargo, and Vikram-III can launch with
815 kg to 500 Kilometre (km) Low Inclination Orbit.
✓ Raman-1 engine for roll control and Raman-2 engine as a single higher thrust regeneratively cooled will
be used in the upper stage of Vikram-1 launch vehicle, which may take place by the end of 2023.
Q. ‘Kheer Bhawani Mela’ is a festival celebrated in which Indian state/UT?
A) Kerala B) Ladakh
C) Chhattisgarh D) Jammu and Kashmir
Answer : D
✓ Every year on the day of Jyestha Ashtami, Kashmiri Pandits visit Mata Ragneya Devi Temple, also
known as Kheer Bhavani Temple, for darshan.
✓ This year the mela was celebrated with great enthusiasm not only by the Kashmiri Pandits but also by
the local people of the Kashmir Valley on 28th May.

Follow us: Official Site, Telegram, Facebook, Instagram, Instamojo 648


✓ The Kheer Bhavani fair held on Jyestha Ashtami in Kashmir holds a sacred place in the spiritual realm
of Kashmiri Pandits.
✓ More than 25 thousand devotees participated in the fair.
✓ The Kheer Bhawani Mela started on 26th May and concluded on 28th May i.e. Jyestha Ashtami.
✓ Kheer Bhavani is considered to be the deity of Kashmiri Pandits, who have a lot of recognition there.
✓ Over the years, the Kheer Bhawani Mela has become a symbol of communal harmony and brotherhood
in Kashmir

MELA IN NEWS 2023


▪ Kheer Bhavani Mela : Jammu and Kashmir
▪ SARAS Aajeevika Mela : Jammu and Kashmir
▪ Matua Mela : West Bengal
▪ Shrimp Mela : Punjab
▪ Maghi Mela : Punjab
▪ 36th Surajkund International Crafts Mela : Faridabad, Haryana
▪ Divya Kala Mela : Bhopal, Madhya Pradesh
▪ Gangasagar Mela : West Bengal
▪ Pusa Krishi Vigyan Mela : New Delhi
Q. Maharashtra government has named which Indian cricketer as ‘Smile Ambassador’ under the state
medical education & drug department’s ‘Swachh Mukh Abhiyan?
A) Saurav Ganguly B) Virat Kohli
C) Sachin Tendulkar D) Rohit Sharma
Answer : C
✓ Maharashtra govt names Sachin Tendulkar as Smile Ambassador’
✓ Maharashtra government has named cricket legend, Sachin Tendulkar as ‘Smile Ambassador’ under the
state medical education & drug department’s ‘Swachh Mukh Abhiyan.
✓ Objective – To spread awareness about oral health and hygiene.
✓ Maharashtra Chief Minister Eknath Shinde and Deputy Chief Minister Devendra Fadnavis signed a
Memorandum of Understanding (MoU) with Sachin Tendulkar. The former Indian cricketer will be the
campaign’s brand ambassador for the next five years.
Q. Which state government has recently launched Namo Shetkari Mahasanman Yojana?
A) Kerala B) Gujarat
C) Maharashtra D) Rajasthan
Answer : C
✓ Maharashtra Government launched Namo Shetkari Mahasanman Yojana.
✓ Maharashtra government has approved a new financial scheme under which Rs 6,000 will be paid
annually to over one crore farmers in the state.
✓ Namo Shetkari Mahasanman Yojana' was approved in a cabinet meeting chaired by Chief Minister
Eknath Shinde in Mumbai.
✓ Chief Minister Shinde said that this amount is in addition to the Rs 6,000 being given to farmers by the
Center under PM Kisan Yojana.
✓ The Maharashtra cabinet also approved the implementation of 'Gender Inclusive Tourism Policy' named
'Aai' to empower women in the tourism sector.
✓ The cabinet also approved the starting of women bike-taxi service at some tourist places.

Follow us: Official Site, Telegram, Facebook, Instagram, Instamojo 649


Q. Which state government launched a new scheme, Mo Ghara (My Home) – a credit-linked housing
scheme for rural poor in the state?
A) Assam B) Odisha
C) Chhattisgarh D) Madhya Pradesh
Answer : B
✓ The Odisha government launched the 'Mo Ghara' (My Home) housing scheme.
✓ The scheme provides credit-linked financial assistance to economically and socially weaker families and
the scheme is fully funded from the state budget.
✓ Applications for the loan will open from June 15, 2023 and the scheme covers families who were earlier
excluded from the housing schemes.
✓ Beneficiaries can avail housing loans up to Rs.3 lakh.
✓ The state government waives off registration fee, stamp duty and processing fee for bank loan approval.
✓ The 'Mo Ghara' housing scheme is in addition to the Biju Pakka Ghar Yojana (BPGY) and Pradhan
Mantri Awas Yojana-Gramin.
Q. Who has become the first female NCC cadet in India to complete the advanced mountaineering course
in the Himalayan region of Uttarkashi in Uttarakhand?
A) Shalini Singh B) Poonam Pandey
C) Shivani Singh D) Ritika Arora
Answer : A
✓ Shalini Singh has become the first female NCC cadet in India to complete the advanced mountaineering
course in the Himalayan region of Uttarkashi in Uttarakhand.
✓ She was the only female cadet in the team of 45 NCC cadets that was formed to complete the course.
✓ The Senior Wing NCC cadet, Shalini Singh, was enrolled in the 67 UP Battalion. She began the month-
long mountaineering course on April 26.
Q. Rail company Alstom will manufacture 100 aluminum-bodied Vande Bharat trains with sleeper facilities,
after bidding the lowest for the tender at a price of Rs 150.9 crores per train. Alstom is the company of
which country?
A) France B) Germany
C) Sweden D) Switzerland
Answer : A
✓ French railway major Alstom has emerged as the lowest bidder for manufacture and maintenance of 100
aluminium Vande Bharat trainsets, valued at Rs 30,000 crore.
✓ The company, which is a major player in the Indian railway and urban transport sector, bid Rs 150.99
crore as the cost to manufacture one trainset, followed by a consortium of Swiss rolling stock company
Stadler Rail and India’s Medha Servo, which quoted Rs 169.9 crore to build one trainset
✓ The basic manufacturing cost would be paid upon delivery of the trainsets, while the remaining portion
of close to Rs 17,000 crore would be paid for maintenance for a period of 35 years.
✓ The train is pegged to have a speed capability of 200 km per hour.

Q. How many railway stations to be upgraded under Amrit Bharat Station Scheme?
A) 1,275 B) 1,325
C) 1,678 D) 1,897

Follow us: Official Site, Telegram, Facebook, Instagram, Instamojo 650


Answer : A
✓ The Consultative Committee of Members of Parliament for the Ministry of Railways has resolved to
upgrade 1,275 railway stations under Amrit Bharat Station.
✓ The decision was taken at a meeting in New Delhi. The meeting was chaired by Railway Minister
Ashwini Vaishnaw.
✓ The catering services were given to passengers through either static or mobile units, adding that there
were 473 pairs of trains with pantry cars/mini pantries and 706 pairs of trains with a train-side vending
facility.
Q. Every Year, June 01 is dedicated to celebrate which of these days?
A) World Milk Day B) World Honey Day
C) World Water Day D) World Food Day
Answer : A
✓ World Milk Day is observed annually on 1 June and was created by the Food and Agriculture
Organization (FAO) of the United Nations (UN) in 2001.
✓ World Milk Day aims to promote the consumption and benefits of milk around the world.
✓ The day aims to raise awareness and support initiatives related to the dairy industry.
✓ The theme for World Milk Day 2023 is "Showcasing how dairy is reducing its environmental footprint,
while also providing nutritious foods and livelihoods."
✓ Every year November 26 is celebrated as National Milk Day in India.
✓ The day is being observed since 2014 to commemorate the birth anniversary of the Father of India’s
White Revolution, Dr. Verghese Kurien.
✓ Dr. Verghese Kurien is also nicknamed as Milkman of India.
✓ The day was fixed by all the dairy majors of the country including the National Dairy Development
Board (NDDB), the Indian Dairy Association (IDA), along with 22 state level milk federations
✓ India is the world's largest milk producer.
✓ Uttar Pradesh is the highest milk producing state in India.

Q. Who is the author of a book titled “Ringside”?


A) Vijay Darda B) Rima Huja
C) Shashi Tharoor D) Chetan Bhagat
Answer : A
✓ Renowned author and Congress MP Dr. Shashi Tharoor has released the book “Ringside” written by
Lokmat Media Group Editorial Board Chairman and former MP Dr. Vijay Darda.
✓ The book is a captivating compilation of Darda’s weekly articles, which were published in the Lokmat
Media Group newspapers and other prominent national and regional dailies of the country between 2011
and 2016.
✓ “RINGSIDE” will take readers on an intellectual journey encompassing science, environment, economy,
security, social developments, sports, arts, culture, foreign policy, and national and international affairs.

Q. Which of the following country has launched its first commercial-grade satellite?
A) North Korea B) South Korea
C) Iran D) Saudi Arabia
Answer : B
✓ South Korea has launched its first commercial-grade satellite.

Follow us: Official Site, Telegram, Facebook, Instagram, Instamojo 651


✓ The commercial-grade satellite was launched by the Naro Space Center in Gohang, South Korea, using
a Launch Nuri rocket.
✓ The main satellite objectives include verifying imaging radar technology and observing cosmic radiation
in near-Earth orbit.
✓ The main satellite, called “Next Generation Small Satellite 2”, was accompanied by seven cube-shaped
satellites.
✓ South Korea also plans to launch three more Nuri rockets by 2027.

Q. Which.country has become the first country to start putting health warnings on individual cigarettes?
A) Canada B) Australia
C) Sweden D) Norway
Answer : A
✓ Canada has become the first country in the world to stipulate that individual cigarettes carry mandatory
health warnings.
✓ While packets carry such warnings of the risks associated with smoking, the Canadian Government is
taking this measure to ensure that it becomes virtually impossible to avoid health warnings altogether.
✓ These regulations will come into effect on August 1 this year. These cigarettes will be introduced in a
phased manner.
✓ The individual health warnings will first appear on king-size cigarettes by the end of July 2024, and by
the end of April 2025, other varieties, regular-size cigarettes and little cigars with tipping paper, and tubes,
will follow.
Q. Who has been appointed as the new Chairman and Managing Director of Steel Authority of India Ltd
(SAIL)?
A) Ajit Kumar Saxena B) Amarendu Prakash
C) Rajesh Kumar D) Rajiv Dixit
Answer : B
✓ Amarendu Prakash has been appointed as the new Chairman and Managing Director of Steel Authority
of India Ltd (SAIL).
✓ He was earlier Director (In-Charge) of SAIL’s Bokaro Steel Plant.
✓ Prakash is heading the Bokaro Steel Plant as Director-in-Charge from September 2020.
✓ Under Prakash’s leadership, Bokaro Steel Plant achieved remarkable production records in FY22 and
FY23.
✓ Steel Authority of India Limited (SAIL)
✓ Founded : 19 January 1954
✓ Headquarters : New Delhi

Q. Edgars Rinkevics was elected as the new president of which country?


A) Latvia B) Kenya
C) Ethiopia D) Zimbabwe
Answer : A
✓ Edgars Rinkevics elected as new president of Latvia
✓ Edgars Rinkevics, Latvia’s foreign minister since 2011, has been elected as the country’s new president.
✓ The 100-seat Saeima legislature elected Edgars Rinkevics, the country’s top diplomat since 2011, as
president to serve for a four-year term.

Follow us: Official Site, Telegram, Facebook, Instagram, Instamojo 652


✓ In 2014, he announced that he is “proudly” homosexual, will be the first openly gay president in the
Baltic nations
✓ Latvia declared independence in 1918 but was later occupied by the Soviet Union in 1940.
Q. Who has been appointed as the first woman Secretary General of the World Meteorological
Organisation?
A) Soumya Swaminathan B) Kristalina Georgieva
C) Petteri Taalas D) Celeste Saulo
Answer : D
✓ Celeste Saulo of Argentina has been appointed as the first female Secretary-General of the World
Meteorological Organization (WMO).
✓ Saulo won a landslide victory at the Congress of the United Nations Climate and Weather Agency in
Geneva.
✓ She will take over from the outgoing secretary-general Petteri Taalas of Finland on January 1.
✓ Saulo has served as director of Argentina's National Meteorological Service since 2014.
✓ World Meteorological Organization (WMO)
✓ It was established in 1873 by the International Meteorological Organization.
✓ Headquarters – Geneva, Switzerland
✓ On March 23, 1950, the World Meteorological Organization became a specialised organisation of the
United Nations.
✓ This organisation uses the latest information on meteorology, climatology, hydrology and related
geophysical research to contribute to the safety and welfare of society.
Q. Which Country to host world's largest Conservation Conference in 2025?
A) UAE B) India
C) Bangladesh D) United States
Answer : A
✓ UAE to Host World's Largest IUCN World Conservation Congress 2025.
✓ The next International Union for Conservation of Nature (IUCN) World Conservation Congress will be
held from 9 to 15 October 2025 in Abu Dhabi, United Arab Emirates.
✓ The United Arab Emirates was selected as the host country based on a rigorous evaluation process of its
candidature.
✓ The United Arab Emirates has a long-standing collaboration with IUCN, with seven IUCN Member
organizations and the National Committee based there.
✓ The country is home to 50 scientific and policy experts who are members of IUCN Commissions.
✓ About IUCN World Conservation Congress
✓ The IUCN Congress is the world's largest conservation event, held every four years.
✓ The Congress brings together leaders from government, civil society, Indigenous peoples' organizations,
business, and academia.
✓ Its purpose is to determine the world's most pressing environmental and development challenges and
develop actions to address them.
✓ The Congress will focus on delivering the Global Biodiversity Framework adopted by over 190 countries
in December.
✓ The Framework includes targets to enhance the conservation of ecosystems, species, and genetic
diversity.

Follow us: Official Site, Telegram, Facebook, Instagram, Instamojo 653


Q. India defeated which team to win the Men's Hockey Junior Asia Cup title?
A) Sri Lanka B) Pakistan
C) Nepal D) Bangladesh
Answer : B
✓ India has won the Men's Hockey Junior Asia Cup-2023 title by defeating Pakistan 2-1. India has won
this title record for the fourth time.

Q. Who has become the first woman from Tamil Nadu to climb the world’s highest peak, Mount Everest?
A) Premlata Agarwal B) Krushnaa Patil
C) Malavath Purna D) Muthamizh Selvi
Answer : D
✓ Tamil Nadu’s Muthamizh Selvi climbs Mount Everest.
✓ A teacher from Chennai became the first woman from Tamil Nadu to climb Mount Everest. Muthamizh
Selvi set out on the quest to scale the world’s highest peak on April 2.
✓ Muthamizh Selvi, who teaches Japanese in Chennai.
Q. According to a new report by Standard Chartered, India is set to remain UAE’s largest export destination
by which year?
A) 2025 B) 2026
C) 2030 D) 2028
Answer : C
✓ India set to remain UAE’s largest export destination by 2030
✓ India is set to remain the United Arab Emirates (UAE) largest export destination by 2030, according to
a new report by Standard Chartered.
✓ The report, titled “Future of Trade: New opportunities in high-growth corridors”, forecasts that the
UAE’s exports will reach around USD 543 billion by 2030, showcasing a robust annual growth rate of
5.5 percent.
✓ The report also highlights that trade corridors anchored in Asia, Africa, and the Middle East are expected
to outpace the global trade growth rate by almost four percentage points, propelling the combined trade
volume in these regions to an astounding USD14.4 trillion, accounting for 44 percent of global trade by
2030.
✓ India is set to remain the UAE’s largest export destination by 2030, while exports to Turkiye, Vietnam
and Singapore are among the fastest growing
Q. Who has been appointed as the new Director General of the Geological Survey of India (GSI)?
A) S. K. Mohanty B) Ananta Barua
C) Ashwani Bhatia D) Janardan Prasad
Answer : D
✓ Janardan Prasad appointed new DG of Geological Survey of India.
✓ Prasad succeeds Dr. S. Raju, who has held the post since April 2022.
✓ Mr. Prasad is an MSc in Geology from Patna University and joined GSI, Gandhinagar, as a geologist in
1988.
✓ He was also posted in Shillong, Patna, Faridabad, Ranchi and Hyderabad in various capacities.
✓ About Geological Survey of India (GSI)

Follow us: Official Site, Telegram, Facebook, Instagram, Instamojo 654


✓ Founded : 1851
✓ Headquarters : Kolkata, West Bengal
✓ Director General : Janardan Prasad

Q. Who has been appointed as a brand ambassador for Zebronics TV?


A) Hrithik Roshan B) Virat Kohli
C) Rohit Sharma D) Sachin Tendulkar
Answer : A
✓ Hrithik Roshan appointed brand ambassador for Zebronics TV
✓ An IT peripheral, audio systems, and lifestyle accessories company, Zebronics has signed actor Hrithik
Roshan as the face of its range of Smart LED TVs.
✓ This is the sixth year of its collaboration with Roshan and it has been extended to its new range.
✓ According to a report by Kroll, Hrithik Roshan had a value of $71.6 million and was the tenth most-
valued celebrity in the country in 2022 as part of its Celebrity Brand Valuation Report.
✓ Co-founder and director of Zebronics – Rajesh Doshi

Q. Which institute has topped the tally of the 3rd Khelo India University games?
A) Lovely Professional University, Punjab
B) Jain University, Karnataka
C) Gujarat University
D) Punjab University, Chandigarh
Answer : D
✓ The third khelo India University games has been concluded at IIT BHU campus in Varanasi.
✓ Union Youth Affairs and Sports Minister Anurag Singh Thakur along with Union Minister of state for
Sports, Nishith Pramanik and Chief Minister of Uttar Pradesh, Yogi Adityanath were present at the
closing ceremony.
✓ Punjab University, Chandigarh regain champions crown as Guru Nanak Dev University fall short despite
Fencing sweep on final day, after a gap of one edition of the Khelo India University Games..
✓ PUC finished with an overall tally of 69 medals including 26 gold, 17 silver and 26 bronze medals.
✓ GNDU amassed 24 gold, 27 silver and 17 bronze medals to finish second and in the top three for the very
first time.
Q. Who is the author of the book titled “The Power of One Thought: Master Your Mind”?
A) Zuni Chopra B) Durjoy Dutta
C) BK Shivani D) Samhita Arni
Answer : C
✓ “The Power of One Thought: Master Your Mind” is a book by BK Shivani that aims to guide readers
toward personal transformation.
✓ The book is published by HarperCollins India.
✓ Shivani was also honored with ‘Nari Shakti Puraskaar’ in 2017.
✓ Since 2017, she has been appointed as a Goodwill Ambassador by the World Psychiatric Association.
✓ HarperCollins India was awarded the Publisher of the Year Award three times, in 2015, 2016, and 2018.

Follow us: Official Site, Telegram, Facebook, Instagram, Instamojo 655


Q. According to CSE Report, Which state ranks top on overall environmental performance 2023?
A) Bihar B) Meghalaya
C) Telangana D) Uttar Pradesh
Answer : C
✓ A non-profit Centre for Science and Environment (CSE) released its annual compendium, titled 'State of
India's Environment 2023.
✓ It is based on four parameters i.e. environment, agriculture, public health, and infrastructure.
✓ In terms of overall environmental performance, the report has ranked Telangana at the top for its progress
in increasing its forest cover and in municipal waste treatment.
✓ Gujarat, Goa and Maharashtra follow Telangana, in that order. The bottom rung is occupied by
Rajasthan, Nagaland and Bihar.

Q. Who received 'Climate And Health' award at UITP Awards 2023, Barcelona?
A) CESC Limited
B) Brihanmumbai Electric Supply and Transport (BEST)
C) Indian Railway Catering and Tourism Corporation (IRCTC)
D) Uttar Haryana Bijli Vitran Nigam (UHBVN)
Answer : B
✓ The Brihanmumbai Electric Supply and Transport (BEST) Undertaking was honored with the prestigious
'Climate And Health' award at the UITP Awards 2023 function held in Barcelona
✓ Additionally, BEST was recognized as one of the top three finalists in the 'Public And Urban Transport
Strategy' category.

Q. Which state government has announced the launch of 'Mukhyamantri Seekho Aur Kamao' scheme?
A) Madhya Pradesh B) Uttar Pradesh
C) Assam D) Bihar
Answer : A
✓ MP Govt to launch CM Learn and Earn scheme.
✓ In Madhya Pradesh, the Chief Minister's Learn and Earn scheme is set to be launched with the aim of
providing employable skills to the youth.
✓ Chief Minister's Learn and Earn scheme in Madhya Pradesh focuses on providing training, financial
support, and employment opportunities to the youth.
✓ By identifying numerous work areas and involving various industries, the scheme aims to equip the young
workforce with the necessary skills to enhance their employability and contribute to the state's
development.
✓ Participants in the scheme will receive a monthly allowance ranging from Rs 8,000 to 10,000.

Q. Which has become the first state in the country to start its own internet service?
A) Tamil Nadu B) Karnataka
C) Himachal Pradesh D) Kerala
Answer : D
✓ Kerala has become the first state in the country to launch its own internet service.

Follow us: Official Site, Telegram, Facebook, Instagram, Instamojo 656


✓ The Pinarayi Vijayan-led Kerala government officially launched the Kerala Fiber Optical Network
(KFON).
✓ Kerala is the first state to declare the right to internet as a fundamental right.
✓ The main objective of KFON is to bridge the digital divide and ensure high-speed broadband internet
access to all households and government offices in Kerala.
✓ KFON uses fibre optic technology to provide fast and reliable internet connection.
✓ The Government of Kerala aims to bridge the digital divide and promote digital inclusion through the
implementation of KFON.
✓ The initiative aims to create a robust digital infrastructure that fosters innovation, economic growth and
social development.
✓ By providing high-speed internet connectivity, KFON strives to ensure a level playing field for all citizens
in accessing digital services.
✓ Kerala Fiber Optical Network (KFON)
✓ It is an optical fibre cable network spanning 30,000 km with 375 Points-of-Presence (PoPs) across Kerala.
✓ The KFON infrastructure is shared with cable operators and all service providers.
✓ KFON services will be provided through local ISPs, Telecom Service Providers (TSPs) and Cable TV
providers.
Q. Who has released NHAI's first 'Sustainability Report for FY2021-22'?
A) Nitin Gadkari B) Amit Shah
C) Draupadi Murmu D) Narendra Modi
Answer : A
✓ Highlighting its commitment towards Environment sustainability, NHAI’s first ‘Sustainability Report for
FY 2021-22’ covers NHAI’s governance structure, stakeholders, environment and social responsibility
initiatives.
✓ The report was recently released by Shri Nitin Gadkari, Union Minister for Road Transport and
Highways.
✓ The Sustainability Report underlines the effectiveness of various efforts made by NHAI for environment
and energy conservation. From FY 2019-20 till 2021-22 direct emission reduced by 18.44% and 9.49%
due to less fuel consumption. NHAI continues to work towards reducing indirect emissions as well, by
transiting towards clean and green energy sources.
✓ Green House Gas (GHG) Emissions from energy consumption, operations, transport and travel
measured in Metric Tonnes CO2 equivalent /km constructed saw a decline of 9.7% in FY 2020-21 and
2% in FY 2021-22

Q. India's first international cruise ship - MV Empress has been flagged off for which country?
A) Maldives B) Thailand
C) Sri Lanka D) Bangladesh
Answer : C
✓ Union Minister for Ports, Shipping and Waterways, Sarbananda Sonowal flagged off India's first
international cruise vessel - MV Empress, from Chennai to Sri Lanka.
✓ This is the launch of the International Cruise Tourism Terminal at Chennai built at a cost of Rs 17.21
crore, ushering in a new era of cruise tourism and maritime trade in the country.
✓ On the occasion of Environment Day, Sonowal along with the port officials also planted 2,500 saplings
at the port.
✓ The launch of the first cruise service between Chennai and Sri Lanka has opened a new chapter in the
cruise tourism sector in the country.

Follow us: Official Site, Telegram, Facebook, Instagram, Instamojo 657


✓ This is the result of an MoU signed during the Incredible India International Cruise Summit in 2022
between Chennai Port and M/s Waterways Leisure Tourism Pvt Ltd to start domestic and international
cruise services.
✓ International Cruise Vessel, MV Empress
✓ It is a renowned and luxurious cruise ship that operates globally.
✓ It offers passengers a unique and unforgettable experience on the open seas, combining comfort,
entertainment, and leisure.
Q. Who has been appointed as new Chairperson of The Hindu Group Publishing Private Limited
(THGPPL)?
A) Arun Kumar Singh B) Roshni Nadar
C) Leena Tewari D) Nirmala Lakshman
Answer : D
✓ Nirmala Laxman has been appointed as the Chairperson of the Board of Directors of The Hindu Group
Publishing Private Limited (THGPPL) for a period of three years..
✓ He succeeds Malini Parthasarathy, who stepped down after completing her three-year term at the board
meeting on June 5, 2023.
✓ Nirmala Laxman During her time as joint editor of The Hindu, she was instrumental in re-launching and
setting up new features sections, including 'The Hindu Literary Review', 'Young World' and 'The Hindu
in School'. played an important role.
✓ Nirmala Laxman is the founder and curator of Lit for Life, a literary festival organized by The Hindu.
✓ The Hindu Group Publishing Private Limited (THGPPL):
✓ It is a media company.
✓ It is the publisher of The Hindu, a national daily newspaper, as well as other publications including
Frontline, Sportstar and Hindu BusinessLine.
✓ THGPPL operates the digital assets including the website and mobile app of The Hindu.
✓ The company was founded in 1878 by G. Subramania Iyer as The Hindu Religious and Public Trust,
which focused on religious and educational material.
✓ In 1905, the Trust acquired The Hindu newspaper, shifting the company's focus to journalism.
✓ Headquarters - Chennai, India

Q. Who was the CEO of alcohol maker Diageo, who has passed away?
A) Jensen Huang B) Ivan Menezes
C) Andy Jassy D) Debra Crew
Answer : B
✓ Ivan Menezes, CEO of the world's largest spirits company Diageo, has passed away.
✓ Ivan Menezes was born in Pune, India in July 1959, the son of Ivan Menezes, the chairman of the Indian
Railway Board.
✓ Menezes joined Diageo in 1997, following the merger of Guinness and Grand Metropolitan.

Q. Which Indian-origin scientist has been awarded the 'Spinoza Award', the highest honor in Dutch science?
A) Soumya Swaminathan B) Abhilasha Singh
C) Surekha Gupta D) Joyita Gupta
Answer : D
✓ Indian-origin scientist Joyita Gupta has been awarded the Spinoza Prize, the highest honor in Dutch
science.

Follow us: Official Site, Telegram, Facebook, Instagram, Instamojo 658


✓ This award is sometimes also called the 'Dutch Nobel Prize'.
✓ The University of Amsterdam said in a statement that he has been honored for his outstanding,
pioneering and inspiring scientific work.
✓ Gupta will get 1.5 million euros to spend on activities related to scientific research.

Q. Which district in Maharashtra is the development of India’s first carbon-neutral village taking place?
A) Nashik B) Aurangabad
C) Nagpur D) Thane
Answer : D
✓ India's first carbon neutral village to be developed in Bhiwandi taluka of Thane district.
✓ Union Minister of State, Kapil Patil, recently announced the development of India's first carbon-neutral
village in Bhiwandi Taluka, located in Thane district.
Q. Who is the current US Defence secretary?
A) Antony Blinken B) Lloyd Austin
C) Merrick Garland D) Janet Yellen
Answer : B
✓ Defence Minister Rajnath Singh, held discussions with US Defence Secretary Lloyd Austin in New Delhi
on 5th June.
✓ The meeting focused on strengthening defence cooperation in various areas, including aligning strategic
interests and enhancing security cooperation.
✓ During the talks, Minister Singh emphasized the significance of the India-US partnership in ensuring a
free, open, and rules-based Indo-Pacific region.
✓ He expressed India's commitment to closely collaborate with the United States across different domains,
aiming for capacity building and further strengthening strategic partnership.
✓ India and the US cooperate in the fields of science, technology and innovation

Q. Which of the following country has hosted the meeting of BRICS Foreign Ministers?
A) Brazil B) Russia
C) South Africa D) India
Answer : C
✓ BRICS countries kicked off its foreign ministers meeting on 1st June in Cape Town, the legislative capital
of South Africa.
✓ External Affairs Minister S Jaishankar participated in the meeting of foreign ministers of BRICS
countries.
✓ South Africa, which currently holds the chair of the bloc, hosted the BRICS Foreign Ministers' Meeting
in Cape Town on 1-2 June.
✓ The foreign ministers’ meeting will finalise the agenda for the 15th BRICS summit scheduled to be held
in South Africa in August.
✓ It is believed that 19 countries are in line to join BRICS.
✓ Joint Agreement
✓ The BRICS foreign ministers agreed BRICS cooperation should be strengthened to promote unity among
developing countries.
✓ Advance the global governance system reform.
✓ Accelerate the realization of the 2030 Sustainable Development Goals.
✓ Contribute to world peace and development.

Follow us: Official Site, Telegram, Facebook, Instagram, Instamojo 659


✓ About BRICS
✓ The full form of BRICS is Brazil, Russia, India, China, and South Africa.
✓ Goldman Sachs economist Jim O'Neill coined the term BRIC (without South Africa) in 2001.
✓ He claimed that by 2050, the four BRIC economies would dominate the global economy by 2050.
✓ South Africa was included in the list in 2010.
✓ The presidency of the Forum is rotated annually among the members.
✓ BRICS accounts for about 40% of the world’s population.
✓ It accounts for 30% of the world's GDP (Gross Domestic Product).
✓ In the year 2014, during the 6th BRICS Summit in Fortaleza, Brazil, the BRICS leaders signed an
agreement to establish a New Development Bank (NDB).
Q. Which state has becomes the first state to notify a committee for conservation of seeds preserved by tribal
farmers?
A) Odisha B) Assam
C) Uttar Pradesh D) Chhattisgarh
Answer : A
✓ Odisha becomes the first state to notify a committee for conservation of seeds preserved by tribal farmers.
✓ The Odisha government has established a committee called the Landrace Varietal Release Committee
(LVRC) to release traditional millet varieties in accordance with standard operating protocols (SoP) for
seed systems.
✓ Odisha is the first state in India to develop a protocol for releasing indigenous millet varieties conserved
by tribal farmers for centuries.
✓ Landraces refer to ecotypes that have been cultivated for a long time in their original habitats.
✓ It is crucial to preserve these landraces before they become extinct.
✓ The initiative aims to preserve traditional landraces, promote agrobiodiversity, and empower custodian
farmers in Odisha while supporting sustainable agricultural practices.
✓ Landrace Varietal Release Committee (LVRC)
✓ The objective of the committee is to recognize the efforts of custodian farmers, particularly tribals, in
conserving traditional millet landraces..
✓ Participatory varietal trials (PVTs)
✓ Participatory varietal trials (PVTs) conducted under the Odisha Millets Mission (OMM) have shown that
certain millet landraces perform better than released varieties under different agricultural practices.
✓ Standard Operating Protocol (SoP)
✓ The working group, in consultation with experts from various institutions, developed a standard
operating protocol (SoP) for seed systems for landraces.
Q. When is the World Food Safety Day celebrated?
A) June 5 B) June 6
C) June 7 D) June 4
Answer : C
✓ World Food Safety Day - 7 June.
✓ World Food Safety Day is observed annually on 7 June to highlight the importance of maintaining food
standards and protecting consumers from foodborne illnesses.
✓ The theme of World Food Safety Day 2023 is "Food Standards Save Lives".
✓ The theme emphasises the important role of food safety standards in ensuring the well-being of
individuals.
✓ World Food Safety Day encourages cooperation among nations and raises awareness of the importance
of food safety standards in protecting public health

Follow us: Official Site, Telegram, Facebook, Instagram, Instamojo 660


Q. What is the name of the world's first multi-sensor Earth observation satellite, GalaxEye Space?
A) Drishti B) Vidhi
C) Prakashan D) Kalyan
Answer : A
✓ GalaxEye Space to launch world’s 1st Multi-Sensor Earth Observation Satellite ‘Drishti’.
✓ ’An IIT (Indian Institute of Technology )-Madras (Tamil Nadu) incubated space-tech startup, GalaxEye
Space, is developing the world’s first multi-sensor earth observation satellite namely ‘Drishti’
incorporating a visible spectrum camera that allows synchronized imaging, capable of capturing multiple
images simultaneously.
✓ The startup is going to launch this, its first satellite, in 2024.
✓ It will be India’s first and the world’s highest resolution multi-sensor imaging satellite.
✓ GalaxEye is focused on deploying a satellite constellation featuring a unique sensor based on data fusion
to provide the most comprehensive dataset of imagery from space.

Q. The Central government sanctioned Rs 12,911 crore for the Polavaram project in which state?
A) Andhra Pradesh B) Telangana
C) Kerala D) Tamil Nadu
Answer : A
✓ The central Government sanctioned Rs 12,911 crore for the Polavaram project in Andhra Pradesh.
✓ The Central government sanctioned Rs 12,911 crore to Andhra Pradesh (AP) for completing the first
phase of the Polavaram Project, which includes an additional Rs 2,000 crore for the construction of the
diaphragm wall, which was washed away due to the Godavari floods.
✓ The Polavaram Irrigation Project is on the river Godavari near Ramayyapeta village of Polavaram
Mandal in Eluru district, where Godavari emerges out of the last range of the Eastern Ghats and enters
the plains in West Godavari District of Andhra Pradesh.
✓ This project also includes a hydroelectric power plant with generation capacity of 960 MW power (12 x
80 MW).
✓ This project interlinks Godavari and Krishna rivers by diverting 80 TMC of water to the Krishna river
basin.
Q. Which state government signed two Memoranda of Understanding (MOUs) with the Airports Authority
of India (AAI) for the construction of Civil Enclaves at Darbhanga Airport and Purnea Airport?
A) Chhattisgarh B) Bihar
C) Jharkhand D) Madhya Pradesh
Answer : B
✓ Bihar & AAI signs MOUs to construct new Civil Enclaves at Darbhanga & Purnea Airport.
✓ The Government of Bihar signed two Memoranda of Understanding (MOUs) with the Airports
Authority of India (AAI) for the construction of Civil Enclaves at Darbhanga Airport and Purnea Airport
in Bihar.
✓ Under this MoU, the Bihar government will provide undisputed land at free of cost to AAI for both the
airports.
✓ The construction will take place on 76.65 acres of land for New Civil Enclave and installation of CAT-II
approach lights in Darbhanga and 52.18 acres of land for Civil Enclave in Purnea.
✓ The construction of the new Civil Enclaves will bolster the infrastructure and also pave the way for the
commencement of air services at Purnea Airport

Follow us: Official Site, Telegram, Facebook, Instagram, Instamojo 661


Q. The 6th edition of the annual naval exercise 'Ektha' was held between India and which country?
A) Egypt B) UAE
C) Maldives D) Sri Lanka
Answer : C
✓ Sixth edition of India-Maldives Exercise ‘Ekatha’ begin in Maldives.
✓ The 6th edition of the annual Naval Exercise ‘Ekatha’ between India and Maldives was scheduled to be
conducted from 4th June to 3rd July in Maldives, 2023 to enhance interoperability in diving and special
operations.
Q. Which state government has launched the Nand Baba Milk Mission scheme at a cost of Rs 1,000 crore?
A) Gujarat B) Punjab
C) Uttar Pradesh D) Madhy Pradesh
Answer : C
✓ Uttar Pradesh government launches Nand Baba Milk Mission scheme.
✓ In a bid to make Uttar Pradesh a leading state in the field of milk development and milk production, the
state government has launched the Nand Baba Milk Mission scheme on 6th June at a cost of Rs 1,000
crore.
✓ The mission is implemented to facilitate milk producers to sell their milk at reasonable prices in villages
through dairy cooperatives.
✓ Under this mission, it is proposed to form a Dairy farmer producer organisation (Dairy FPO) in order to
facilitate sale of milk in producers’ villages itself.
✓ There is a plan to set up five dairy FPOs in five districts of the state as a pilot project in the financial year
2023-24, in which women will also play a major role.
Q. The Himachal Pradesh government has signed an agreement with which company for promoting green
hydrogen production?
A) Oil India Limited B) Bharat Petroleum
C) Hindustan Petroleum D) Oil and Natural Gas Corporation
Answer : A
✓ The Himachal Pradesh government has signed an agreement with Oil India Limited for promoting green
hydrogen production.
✓ State Government has imposed ban on single-use plastic in Himachal Pradesh since 2009 and now it was
working to find suitable alternatives to plastic.
✓ The government is committed to environmental protection and has made various provisions in the budget
to improve the state’s climate.
Q. Which Ministry has launched a National Mission titled "Mission on Advanced and High-Impact
Research (MAHIR)"?
A) Ministry of Power B) Ministry of Home Affairs
C) Ministry of Heavy Industries D) Ministry of Defence
Answer : A
✓ Ministry of Power and the Ministry of New and Renewable Energy have jointly launched a National
Mission to quickly identify emerging technologies in the power sector and develop them indigenously,
at scale, for deployment within and outside India.

Follow us: Official Site, Telegram, Facebook, Instagram, Instamojo 662


✓ The National Mission, titled “Mission on Advanced and High-Impact Research (MAHIR)”.
✓ Aim : To facilitate indigenous research, development and demonstration of the latest and emerging
technologies in the power sector.
✓ Planned for an initial period of five years from 2023-24 to 2027-28, the Mission will follow the technology
life cycle approach of Idea to Product.

Q. Which of the following will launch “One Student One Tree Campaign 2023”?
A) National Council of Educational Research and Training ( NCERT)
B) National Testing Agency (NTA)
C) University Grants Commission (UGC)
D) All India Council for Technical Education (AICTE)
Answer : D
✓ All India Council for Technical Education (AICTE) will launch “One Student One Tree Campaign 2023”
and that UGC too has taken initiatives for creating awareness about Mission LiFE.
✓ The celebration of World Environment Day across the country envisages involvement of youth in
spreading the message of LiFE, along with sensitizing school going children and others about LiFE.
✓ The mantra of Mission Life is ‘Lifestyle For Environment’ to make the fight against climate change
democratic, in which everyone can contribute within their capacity.

Q. Which country has introduced a digital birth certificate for someone who was born in country for free?
A) Israel B) Germany
C) Norway D) Switch
Answer : A
✓ Israel Government has introduced a digital birth certificate for someone who was born in Israel for free.
✓ The service allows parents to receive the birth certificate immediately as a digital file via email or mobile
device app
Q. BIMSTEC Energy Center will be set up in which country?
A) Bhutan B) Bangladesh
C) Thailand D) India
Answer : D
✓ For BIMSTEC energy cooperation, BIMSTEC Energy Center will be set up in India by the end of this
year. This center will act as the secretariat of BIMSTEC energy cooperation.
✓ BIMSTEC Secretary General Tenzin Lekfel announced this in Dhaka. BIMSTEC is an international
organization of seven South Asian and South East Asian countries.
✓ About BIMSTEC
✓ BIMSTEC whose full form is 'Bay of Bengal Initiative for Multi-Sectoral Technical and Economic
Cooperation' is an international organization of seven South Asian and South East Asian countries.
✓ It was established in 1997 to advance mutual trade, connectivity and cultural, technological and
economic development in the region.
✓ Out of its 7 member countries, 5 members – Bangladesh, Bhutan, India, Nepal and Sri Lanka are from
South Asia and two countries – Myanmar and Thailand are from South-East Asia.
✓ Initially, BIMSTEC included six areas - trade, technology, energy, transport, tourism and fisheries - for
regional cooperation, which was later expanded to 14 areas, including agriculture.
✓ Secretariat: Dhaka, Bangladesh

Follow us: Official Site, Telegram, Facebook, Instagram, Instamojo 663


✓ Bay of Bengal Initiative for Multi-Sectoral Technical and Economic Cooperation (BIMSTEC)
✓ Establishment :6 June 1997
✓ Headquarteres : Dhaka, Bangladesh
✓ Membership : 7 (Bangladesh, Bhutan, India, Myanmar, Nepal, Sri Lanka &Thailand)
✓ Chairmanship : Sri Lanka
✓ Secretary General : Tenzin lekhpel (Bhutan)
✓ Trick to remember BIMSTEC Countries
✓ TRICK -: MBBS NIT
✓ M - Myanmar
✓ B - Bangladesh
✓ B - Bhutan
✓ S - SriLanka
✓ N - Nepal
✓ I - India
✓ T - Thailand
Q. According to the report titled "Tracking SDG7: The Energy Progress 2023", approximately how many
people around the world are still using polluting fuels and techniques for cooking?
A) 2.9 billion B) 3.1 billion
C) 2.3 billion D) 1.8 billion
Answer : C
✓ Tracking SDG7: 2.3 people use polluting fuel & 675 million have no electricity.
✓ According to the report titled “Tracking SDG7: The Energy Progress 2023” released on 6th June 2023,
around 2.3 billion people across the globe are still using polluting fuels and technologies for cooking and
around 675 million people did not have access to electricity in 2021.
✓ The report states that the world is not on track to achieve the Sustainable Development Goal (SDG) 7
for energy by 2030.
✓ The report highlighted that despite the progress across various indicators of SDG 7 agenda, the current
rates of progress is insufficient to achieve the 2030 targets.

Q. Which day is observed as the World Oceans Day as recognised by UN?


A) June 6 B) June 5
C) June 7 D) June 8
Answer : D
✓ World Oceans Day is marked annually on the 8 June to celebrate the role of the oceans in our everyday
life and inspiring action to protect the ocean and sustainably use marine resources.
✓ The theme of World Oceans Day for the year 2023 is "Planet Ocean: The Tides Are Changing".
✓ World Oceans Day was officially recognised by the United Nations in 2008.
✓ The concept was originally proposed in 1992 by Canada’s International Centre for Ocean Development
(ICOD) and the Ocean Institute of Canada (OIC) at the Earth Summit – UN Conference on Environment
and Development (UNCED) in Rio de Janeiro, Brazil.
Q. Tata Elxsi has partnered with the ISRO to design and develop Crew Module Recovery Models (CMRM)
for recovery team training of which Mission?
A) Chandrayaan-3 B) Aditya-L1
C) Gaganyaan D) Shukrayaan-1
Answer : C

Follow us: Official Site, Telegram, Facebook, Instagram, Instamojo 664


✓ Tata Elxsi partners with ISRO to build Crew Module Recovery Models for Gaganyaan Mission
✓ Tata Elxsi, a leading global design and technology company has partnered with the Indian Space
Research Organization (ISRO) to design and develop Crew Module Recovery Models (CMRM) for
recovery team training of the Gaganyaan Mission.
✓ The project marked Tata Elxsi’s entry into mechanical design for space.
✓ This is the first time ISRO has outsourced the mechanical design and development work to an external
partner for a critical system.
✓ Currently, two Recovery-Models CMRM and Ground Support Fixture (GSF) have been delivered to
Naval training teams at Kochi (Kerala) and Visakhapatnam (Andhra Pradesh) to train the recovery
teams.
✓ ISRO’s Gaganyaan Mission aims to demonstrate Human Spaceflight capability by launching a three-
member crew onto an orbit of 400 km for a three-day mission and bringing them back to Earth safely by
landing in Indian sea waters
Q. Indian Navy and Defence Research and Development Organisation (DRDO) undertook the first
‘combat' test-firing of the Varunastra heavyweight torpedo. What is the operational range of Varunastra?
A) 50 km B) 70 km
C) 60 km D) 40 km
Answer : D
✓ Indian Navy undertakes first combat firing of Varunastra torpedo.
✓ The Indian Navy and the country's Defence Research and Development Organisation (DRDO)
undertook the first ‘combat' test-firing of the Varunastra heavyweight torpedo on 5 June.
✓ It will enhance the anti-submarine warfare capabilities of the indigenous Navy and give it a formidable
force.
✓ The torpedo was fired from a submarine and successfully hit the target at a distance of 40 km.
✓ The test was conducted in the Arabian Sea in the presence of senior officials from the Indian Navy and
the Defense Research and Development Organization (DRDO).
✓ About Varunastra torpedo
✓ It has been designed and developed by the Naval Science and Technological Laboratory at
Visakhapatnam under the Defense Research and Development Organisation.
✓ Bharat Dynamics Limited (BDL) is responsible for the production of the Varunastra missile system.
✓ This would become the mainstay of anti-submarine torpedoes for all Navy warships.
✓ It will replace the old torpedoes on all naval ships that can fire heavy-weight torpedoes.
✓ Features of the Varunastra
✓ It is seven to eight meters long, weighs 1,500 kg and has a diameter of 533 mm.
✓ When fired it can travel at 40 knots, or 74 kmph.
✓ The operational range is 40 km and it can carry a warhead weighing 250 kg.
✓ Varunastra was inducted by Indian Navy in 2016.
✓ It can be fired from all Anti-submarine warfare (ASW) ships capable of firing heavy-weight torpedoes in
an intense countermeasures environment.
✓ Benefits of the Varunastra Torpedo
✓ It is a powerful and sophisticated weapon that will significantly enhance the Navy’s ability to detect,
track, and engage enemy submarines.
✓ It is the first indigenously developed heavyweight torpedo that is capable of meeting the Navy’s
operational requirements

Follow us: Official Site, Telegram, Facebook, Instagram, Instamojo 665


Q. Which country has begun conducting initial tests of autonomous flying taxi that could one day serve as
public transportation?
A) Israel B) Australia
C) Germany D) France
Answer : A
✓ Israel tests first autonomous flying taxi.
✓ Israel has begun conducting initial tests of autonomous drones that could one day serve as public
transportation.
✓ The drones are part of Israel’s National Drone Initiative to ease traffic congestion.
✓ Israel hopes autonomous Electric Vertical Takeoff and Landing (eVTOL) aircraft will help to ease its
worsening traffic problems.
Q. ThyssenKrupp Marine Systems (TKMS) has signed an agreement with India’s biggest warship yard,
Mazagon Dock (MDL), to cooperate in building six advanced submarines for the Indian Navy.
ThyssenKrupp Marine is the company of which country?
A) France B) Germany
C) Belgium D) United States
Answer : B
✓ German shipbuilder ThyssenKrupp Marine Systems (TKMS) has signed an agreement with India’s
biggest warship yard, Mazagon Dock (MDL), to cooperate in building six advanced submarines for the
Indian Navy.
✓ These submarines will be powered by a new technology called air-independent propulsion (AIP). They
will be built under a Rs 45,000-crore programme called Project 75-I.
✓ The Project 75-I is being pursued under the so-called “strategic partner” model of equipment acquisition.
✓ This requires the Ministry of defence (MoD) to identify Indian firms that have the capability and capacity
to build and supply a particular weapons platform – such as a submarine, warship, fighter aircraft or tank

Q. Which countries are agreed to set a target for bilateral trade one billion euros?
A) India, Croatia B) India, Serbia
C) India, Albania D) India, Bulgaria
Answer : B
✓ India & Serbia agree to set bilateral trade of 1 Billion Euros.
✓ President Droupadi Murmu and her Serbian counterpart Aleksandar Vucic on 8 June in Belgrade agreed
to set a target for bilateral trade from the present 320 million euros to one billion euros by the end of the
decade.
✓ President Draupadi Murmu is on a three-day state visit to Serbia from June 7 to 9.
✓ During the visit, President Draupadi Murmu highlighted that the relations between the two sides have a
shared understanding of each other's core interests.
✓ About Serbia
✓ Serbia is a landlocked country in the west-central Balkans. For most of the 20th century, it was a part of
Yugoslavia.
✓ Prime Minister: Ana Brnabic
✓ Capital: Belgrade
✓ President: Aleksandar Vučić
✓ Currency: Serbian dinar (RSD)

Follow us: Official Site, Telegram, Facebook, Instagram, Instamojo 666


Q. Where is the first India-France-UAE Maritime Exercise being organized?
A) Bay of Bengal B) Gulf of Oman
C) Arabian Sea D) Gulf of Aden
Answer : B
✓ The first edition of the India, France and UAE maritime partnership exercise began in the Gulf of Oman.
✓ The exercise includes the participation of INS Tarkash and French Ship Surcouf, both equipped with
integral helicopters, as well as French Rafale aircraft and UAE Navy Maritime Patrol Aircraft.
✓ The exercise focused on a wide range of naval operations, including surface combat, tactical firing, missile
engagement on surface targets, helicopter cross deck landing operations, advanced air defence exercises
and boarding operations.
✓ OMAN
✓ Capital - Muscat
✓ Currency - Omani Rial
✓ Sultan - Haitham bin Tariq
✓ Crown Prince - Thiazin bin Haitham

Q. Which govt announces free bus travel 'Shakti' scheme for women, starting from June 11, 2023?
A) Punjab B) Kerala
C) Karnataka D) Madhya Pradesh
Answer : C
✓ Karnataka government has advised the women to apply for Shakti smart cards to avail the free travel in
state run buses, starting from June 11, 2023.
✓ The government has already issued a set of guidelines on the ‘Shakti’ scheme, which is one of the major
poll promises by the Congress party in Karnataka. According to the transport department of Karnataka,
women can apply for Shakti smart cards through sevasindhu.
✓ The Shakti scheme applies only on the ordinary state run bus services of Karnataka
Q. Who will inaugurate the integrated Water Resources Action Plan-2023-25 under the Amrit Jal Kranti
initiative?
A) Narendra Modi B) Piyush Goyal
C) Nirmala Sitharaman D) Manohar Lal
Answer : D
✓ Haryana Chief Minister Manohar Lal will inaugurate the integrated Water Resources Action Plan-2023-
25 under the Amrit Jal Kranti initiative.
✓ This plan, aimed at conserving water resources, represents a significant and proactive step in addressing
water conservation in the state.
Q. Who has inaugurated the first-ever National Training Conclave at the International Exhibition and
Convention Centre at Pragati Maidan in New Delhi?
A) Nirmala Sitharaman B) Draupadi Murmu
C) Narendra Modi D) Rajnath Singh
Answer : C
✓ Prime Minister Narendra Modi inaugurated the first-ever National Training Conclave at the
International Exhibition and Convention Centre at Pragati Maidan in New Delhi on 11 June.

Follow us: Official Site, Telegram, Facebook, Instagram, Instamojo 667


✓ The Conclave is aimed to enhance capacity building in the civil services and is organised under the
auspices of the National Programme for Civil Services Capacity Building (NPCSCB) - 'Mission
Karmayogi.'
✓ The Conclave is being hosted by the Capacity Building Commission.
✓ More than 1500 representatives from various training institutes, including Central Training Institutes,
State Administrative Training Institutes, Regional and Zonal Training Institutes, and Research institutes
are participating in the conclave.
✓ About Mission Karmayogi Scheme
✓ Mission Karmayogi Yojana was launched on 2nd September 2020.
✓ This scheme is operated under the leadership of Prime Minister Narendra Modi.
✓ The main objective of the Mission Karmayogi scheme is to develop the capabilities of government
employees.
✓ Under the Mission Karmayogi scheme, a budget of Rs 510.86 crore has been set by the government for
a period of 5 years for about 46 lakh central employees.
✓ For the successful operation of the scheme iGOT Karmayogi platform has also been formed through
which online contact is made available.
Q. Who has released the report highlighting the significant impacts of 'Har Ghar Jal' program on public
health and economic savings?
A) World Health Organization B) World Intellectual Property Organization
C) International Maritime Organization D) World Meteorological Organization
Answer : A
✓ A recently released report by the World Health Organization (WHO) has highlighted the significant
impacts of the 'Har Ghar Jal' program on public health and economic savings.
✓ Dr Richard Johnston and Dr Sophie Boisson of the WHO South East Asia presented the report titled
‘Health impact of Jal Jeevan Mission’.
✓ Highlight of the report
✓ The report pointed out that nearly four lakh diarrheal deaths could be prevented by ensuring safely
managed drinking water for all households in the country.
✓ Nearly 14 million disability-adjusted life years (DALYs) related to these diseases could be prevented.
✓ This achievement alone will result in estimated cost savings of up to US$101 billion.
✓ This report focuses on diarrheal diseases as water-borne diseases are a major reason for this.
✓ The report shows that in 2018, 36 per cent of India's total population did not have access to improved
drinking water sources in their premises.
✓ Direct use of unsafe drinking water has serious health and social consequences.
✓ The analysis indicates that in 2019, unsafe drinking water, along with inadequate sanitation and hygiene,
contributed to 1.4 million deaths and 74 million DALYs globally.
✓ Har Ghar Jal' Program
✓ Launched - By Prime Minister Narendra Modi on August 15, 2019
✓ Implementation - By the Jal Jeevan Mission under the Ministry of Jal Shakti
✓ Aims - To provide every rural household with affordable and regular access to an adequate supply of safe
drinking water through taps.
✓ SDG 6.1 - Ensuring universal access to safe and affordable drinking water .
✓ Achievement - 5 states (Goa, Telangana, Haryana, Gujarat and Punjab) and 3 union territories
(Puducherry, Daman and Diu and Dadra and Nagar Haveli) have reported 100% tap water coverage.

Follow us: Official Site, Telegram, Facebook, Instagram, Instamojo 668


Q. Which state government has recently launched a new scheme ‘Mukhyamantri Ladli Bahna Yojana’ for
women empowerment?
A) Uttar Pradesh B) Madhya Pradesh
C) Bihar D) Chhattisgarh
Answer : B
✓ Government of Madhya Pradesh launched Mukhyamantri Ladli Bahna Yojana.
✓ Madhya Pradesh Chief Minister Shivraj Singh Chouhan launched the 'Mukhya Mantri Ladli Bahna
Yojana'.
✓ The Ladli Bahna Yojana 2023 was officially inaugurated by CM Chouhan, who took the initiative to
deposit the first instalment of Rs 1,000 in the accounts of eligible women beneficiaries.
✓ This scheme was launched in Jabalpur, Madhya Pradesh with the primary objective of empowering
women and providing financial assistance to them.
✓ The chief minister outlined plans to increase the monthly support to Rs 1,200, Rs 1,500, Rs 1,700 and
eventually Rs 2,000, thereby increasing the financial assistance provided to beneficiaries.
✓ About Ladli Behna Scheme
✓ It was started on March 15 by Madhya Pradesh Chief Minister Shivraj Singh Chouhan.
✓ This scheme provides financial assistance to women in Madhya Pradesh.
✓ Under the scheme, an amount of Rs 12,000 per year ie Rs 1,000 per month is sent directly to the bank
accounts of eligible women.
✓ The state government has allocated a budget of Rs 60,000 crore for the implementation of the scheme.
✓ The scheme is open to women belonging to various categories including general, OBC, SC, ST,
abandoned, or widowed women.
✓ Eligible women will have to fulfill certain criteria including being economically weak and owning less
than five acres of land.
✓ The scheme targets women with an annual income of less than Rs 2.5 lakh.
✓ The age of the woman should be between 23 to 60 years to be eligible for the scheme.

Q. Who has been launched 'SAGAR SAMRIDDHI', an online dredging monitoring system?
A) Giriraj Singh B) Sarbananda Sonowal
C) Riniki Bhuyan Sarma D) Himanta Biswa Sarma
Answer : B
✓ Sarbananda Sonowal launches 'Sagar Samriddhi' online dredging monitoring system.
✓ In a significant move to accelerate the 'Waste to Wealth' initiative of the Ministry of Ports, Shipping and
Waterways (MoPSW), Union Minister Sarbananda Sonowal launched the 'Sagar Samriddhi' online
dredging monitoring system on 12 June.
✓ About 'Sagar Samriddhi' online dredging monitoring system
✓ This will help in making the seas and port clean. With this, real time information on dredging will be
available.
✓ This system has been developed by National Technology Centre for Ports, Waterways and Coasts
(NTCPWC) the technological arm of MoPSW.
✓ The new technology Draft and Loading Monitor (DLM) system is a significant improvement over the
older system.
✓ The 'Sagar Samriddhi' monitoring system aims to bring synergy among various input reports, including
the daily dredging report and pre and post dredging survey data.
✓ By integrating and processing these reports, the system can generate real-time dredging reports, providing
valuable insights and information.

Follow us: Official Site, Telegram, Facebook, Instagram, Instamojo 669


SARBANANDA SONOWAL IN NEWS 2023
▪ Sarbananda Sonowal lays Foundation Stone for the modernization and upgradation works of the Cochin
Fishing Harbour was laid at Willingdon Island in Kochi.
▪ In a significant move to accelerate the 'Waste to Wealth' initiative of the Ministry of Ports, Shipping and
Waterways (MoPSW), Union Minister Sarbananda Sonowal launched the 'Sagar Samriddhi' online
dredging monitoring system.
▪ Union Minister for Ports, Shipping and Waterways, Sarbananda Sonowal flagged off India's first
international cruise vessel - MV Empress, from Chennai to Sri Lanka.
▪ Ports, Shipping and Waterways Minister Sarbananda Sonowal will receive the first Indian cargo ship at
Sittwe Port in Myanmar .
▪ The real-time performance monitoring dashboard of the Ministry of Ports, Shipping and Waterways
(MoPSW), known as 'Sagar Manthan' was recently inaugurated by the Union Minister for Ports,
Shipping and Waterways and AYUSH, Sarbananda Sonowal.
▪ Union Minister for Ports, Shipping and Waterways and AYUSH Sarbananda Sonowal and Deputy
Prime Minister of Myanmar Admiral Tin Aung San jointly inaugurated Sittwe Port in Myanmar.
▪ Union Minister for Ports, Shipping and Waterways Sarbananda Sonowal inaugurated the Discovery
Campus of the National Technology Center for Ports, Waterways and Coasts at IITM, Chennai, Tamil
Nadu.
▪ Union Minister Sarbananda Sonowal, along with Tripura Chief Minister Dr. Manik Saha, inaugurated
the School of Logistics, Waterways and Communication in Agartala.
▪ Real-time Performance Monitoring Dashboard of MoPSW called ‘Sagar Manthan’ was virtually
launched by Union Minister for Ports, Shipping and Waterways and Ayush Shri Sarbananda Sonowal.
▪ Union Minister of AYUSH, Sarbananda Sonowal has inaugurated a two-day scientific convention on
the theme ‘Homoeopathy: People’s Choice for Wellness’ in New Delhi.
▪ Minister for Ports, Shipping and Waterways Sarbananda Sonowal inaugurated the National Logistics
Portal (marine) in New Delhi.
▪ Sarbananda Sonowal, the Union Minister for AYUSH, inaugurated the International Yoga Mahotsav at
the Dibrugarh University Playground to mark the upcoming International Day of Yoga 2023, which is
just 75 days away.

Q. Which Countries Forge 'Atlantic Declaration' to Boost Economic Ties?


A) US and UK B) US and UAE
C) UK and India D) US and Australia
Answer : A
✓ Atlantic Declaration’ between US and UK signed by Joe Biden and Rishi Sunak.
✓ Prime Minister of the United Kingdom (UK), Rishi Sunak and President of the United States (US), Joe
Biden announced new economic partnership called ‘Atlantic Declaration’ at a joint press conference at
the White House, Washington, D.C., US to strengthen economic ties between the two countries.
✓ The declaration aims to increase US-UK trade in areas such as defence, nuclear materials and the critical
minerals used in electric-car batteries.

Q. Which football club has become the most valuable football club in Europe after replacing Real Madrid?
A) Inter Milan B) Manchester City
C) Liverpool D) Chelsea
Answer : B
✓ Manchester City has become the most valuable football club in Europe after replacing Real Madrid.

Follow us: Official Site, Telegram, Facebook, Instagram, Instamojo 670


✓ City, who play Inter Milan in Champions League final in Istanbul, came out on top with a valuation of
4.073 billion euros ($4.39 billion)
✓ Real Madrid’s value was at 4.006 billion euros.
✓ Manchester City, who were taken over by the Abu Dhabi United Group in 2008, recorded revenue for
last season of £613 million ($769.5m), the second-highest revenue figure in English club history after
Manchester United’s annual revenues hit £627 million in 2019.
Q. Who has become the first team in the world to win all the ICC titles?
A) India B) England
C) Pakistan D) Australia
Answer : D
✓ Australia beat India by 209 runs to win its maiden ICC World Test Championship title at the Kennington
Oval in London on 11 June. Australia became the first team to win all ICC titles.
✓ With this, the Australian team has become the first team in the world to win all the ICC titles.
✓ The title of Player of the Match was given to Travis Head.
✓ Australia wins in ICC title matches
✓ 5 times ODI World Cup
✓ 2 times Champions Trophy
✓ T20 World Cup 1 time
✓ 1 time World Test Championship
✓ Prize money of World Test Championship
✓ Winners (Australia): $1.6 million
✓ Runners Up (India): $800,000

Q. Which team won the title of Women's Junior Hockey Asia Cup 2023?
A) India B) Pakistan
C) Australia D) Germany
Answer : A
✓ The Indian women's junior hockey team won its maiden Women's Junior Asia Cup title by defeating
South Korea 2-1 in Kakamigahara, Japan.
✓ Annu and Neelam scored one goal each for India.
✓ Hockey India has announced a cash prize of Rs 2 lakh for each player.
Q. Who has been elected as the chair-elect of the International Air Transport Association (IATA) Board of
Governors?
A) Camiel Eurlings B) Rakesh Gangwal
C) Rahul Bhatia D) Pieter Elbers
Answer : D
✓ IndiGo CEO, Peter Albers has been elected as the Chairman-elect of the International Air Transport
Association (IATA) Board of Governors.
✓ He will take up his post from June 2024 and will replace Yvonne Manji Makolo.
✓ International Air Transport Association (IATA)
✓ Founded: 19 April 1945, Havana, Cuba
✓ It is a trade association of the world's airlines and has 300 member airlines, which account for 83% of
total air traffic.

Follow us: Official Site, Telegram, Facebook, Instagram, Instamojo 671


✓ It is headquartered in Montreal, Canada and works to promote the safe and sustainable development of
the air transportation industry.
✓ It launched the "Fly Net Zero" initiative to achieve net zero carbon emissions from aviation by 2050.
✓ It is a non-profit organisation funded by its members and governed by a Board of Directors and a General
Assembly.
Q. Which Country Unveils World's Most Powerful Hypersonic Wind Tunnel for Advancing Aerospace
Ambitions?
A) Japan B) USA
C) China D) Russia
Answer : C
✓ China announced the successful completion of the JF-22 wind tunnel’s final evaluation on May 30, 2023,
marking a significant milestone in the country’s pursuit of hypersonic capabilities.
✓ Named the JF-22, the wind tunnel located in the Huairou District in northern Beijing is reportedly able
to simulate flights of up to 10 kilometers per second or 30 times the speed of sound, making it the fastest
in the world according to China’s Institute of Mechanics, its operator.
✓ The JF-22’s construction began in 2018 and was completed in August 2021. The tunnel is 167 meters
long and four meters in diameter.
Q. Which Country Unveils World's Most Powerful Hypersonic Wind Tunnel for Advancing Aerospace
Ambitions?
A) Japan B) USA
C) China D) Russia
Answer : C
✓ China announced the successful completion of the JF-22 wind tunnel’s final evaluation on May 30, 2023,
marking a significant milestone in the country’s pursuit of hypersonic capabilities.
✓ Named the JF-22, the wind tunnel located in the Huairou District in northern Beijing is reportedly able
to simulate flights of up to 10 kilometers per second or 30 times the speed of sound, making it the fastest
in the world according to China’s Institute of Mechanics, its operator.
✓ The JF-22’s construction began in 2018 and was completed in August 2021. The tunnel is 167 meters
long and four meters in diameter.
Q. Which company has successfully launched pilot study on vehicles using E27 fuel and Ethanol Blended
Diesel Fuel?
A) Coal India Limited B) Bharat Heavy Electricals Limited
C) Steel Authority of India Limited D) Hindustan Petroleum Corporation Limited
Answer : D
✓ Hindustan Petroleum Corporation Limited (HPCL) has successful launched pilot study on vehicles using
E27 fuel and Ethanol Blended Diesel Fuel.
✓ HPCL has become the first Oil Marketing Company in India to initiate such a comprehensive research
program.
✓ The move aligns with the roadmap for ethanol blending in India by 2025, which aims to promote the
adoption of ethanol blending in gasoline.
✓ HPCL successfully launched E20 fuel on February 6th, 2023, at 23 retail outlets (ROs) across the country.
✓ Hindustan Petroleum Corporation Limited (HPCL)
✓ Founded : 1974

Follow us: Official Site, Telegram, Facebook, Instagram, Instamojo 672


✓ Headquarters : Mumbai
✓ Chairman : Pushp Kumar Joshi
Q. Narmada Hydroelectric Development Corporation Ltd (NHDC Ltd.) will construct a 525 MW pumped
storage project in which state?
A) Uttarakhand B) Himachal Pradesh
C) Madhya Pradesh D) Uttar Pradesh
Answer : C
✓ Narmada Hydroelectric Development Corporation Ltd (NHDC Ltd.) will construct a 525 MW pumped
storage project near the Indira Sagar Dam in Khandwa, Madhya Pradesh.
✓ The project will use the existing reservoirs of Indira Sagar and Omkareshwar, which are part of the Indira
Sagar Project.
✓ The project is being executed to meet the increasing peak-hour demand for energy in Madhya Pradesh.
✓ The project is expected to generate 1.2 billion units of energy during peak hours and is estimated to cost
Rs 4,200 crore (approximately $550 million).
✓ NHDC Limited has been allotted this project by the Department of New & Renewable Energy,
Government of Madhya Pradesh.
Q. World Cup Squash Championship 2023 begins in Which City?
A) Chennai B) Bengaluru
C) Kochi D) Hyderabad
Answer : A
✓ The World Cup Squash Championships began in Chennai.
✓ The games were formally inaugurated by Minister of State for Youth Affairs and Sports Udhayanidhi
Stalin at Express Avenue Mall.
✓ Notable players who have participated in the tournament include Joshna Chinappa, Saurabh Ghosal,
Abhay Singh, Khanna and Jessica Turnbull.
✓ On the first day of the championship, India will play against Hong Kong and China in Pool B matches.
✓ In Pool A, Egypt will face Australia and Malaysia will take on Colombo.
✓ Tournament Chairperson - Jenna Woolridge
✓ Indian coach - Chris Walker
Q. Who has been appointed as brand ambassador for Printer company Epson India?
A) Rashmika Mandanna B) Alia Bhatt
C) Deepika Padukone D) Anushka Sharma
Answer : A
✓ Printer company Epson India has signed actor Rashmika Mandanna as its brand ambassador.
✓ The actress will collaborate with the company to promote its products in a multi-media campaign for its
‘EcoTank’ printers this month.
✓ She is one of the new entrants on the list of top 20 celebrities who endorse brands in India.
✓ The collaboration aims to spread awareness about printers and their benefits.
Q. World Blood Donor Day is observed every year on____
A) 13 June B) 14 June
C) 12 June D) 15 June

Follow us: Official Site, Telegram, Facebook, Instagram, Instamojo 673


Answer : B
✓ World Blood Donor Day is celebrated every year on 14 June to raise awareness as well as thank the
various blood donors around the world who do their bit to save a life.
✓ This day was first organized by the World Health Organization on June 14, 2005.
✓ The host nation for World Blood Donor Day 2023 is Algeria
✓ The theme for World Blood Donor Day 2023 is "Give blood, give plasma, share life, share often."
✓ The day marks the birthday of Karl Landsteiner who discovered the ABO blood group system in 1901.

Q. Which country is the host nation for 2023 World Blood Donor Day?
A) Sweden B) Japan
C) Italy D) Algeria
Answer : D
✓ The host country for the global event of World Blood Donor Day 2023 is Algeria through its National
Blood Transfusion Service

Q. In which state an app named 'Arunpol App' has been launched for the police?
A) Assam B) Andhra Pradesh
C) Arunachal Pradesh D) Kerala
Answer : C
✓ Arunachal Pradesh Chief Minister Pema Khandu launched the 'Arunpol App' of Arunachal Pradesh
Police .
✓ This app allows common people to register complaints without visiting the police station.
✓ The app also enables people to apply for clearance from the police.
✓ An APP Awareness Generation Vehicle named Arunpol Sewa Van was flagged off during the event.
✓ The vehicle will travel to various places, engage with the public and create awareness about the Arunpol
App.

Q. Which team won the FIFA U-20 World Cup title?


A) Italy B) Uruguay
C) Argentina D) Brazil
Answer : B
✓ Uruguay's Under-20 football team won their first Under-20 World Cup title by defeating Italy 1-0.
✓ Uruguay has become the first South American country to win this title since 2011.
✓ Uruguay lost the final of the tournament in 1997 and 2013
✓ 2023 FIFA U-20 World Cup : Host country : Argentina
✓ Uruguay
✓ Capital : Montevideo
✓ Currency : peso
✓ President : Luis Lacalle Pou

Q. In which state was the country's 'first' toy train powered by CNG inaugurated?
A) Rajasthan B) Tamil Nadu
C) Maharashtra D) Arunachal Pradesh
Answer : A

Follow us: Official Site, Telegram, Facebook, Instagram, Instamojo 674


✓ Toy train was started once again in Gulab Bagh of Udaipur (Rajasthan), which has been named
Maharana Pratap Express train.
✓ According to the engineer of the toy train, Anirudh Singh Nathawat, this is the country's first CNG-
powered toy train. Here tourists will be able to ride this toy train for ₹25/₹50.

Q. Who has flagged off the first Agri- SEZ project of Gabon from New Delhi?
A) Dharmendra Pradhan B) Smriti Irani
C) Mansukh L. Mandaviya D) Ashwini Vaishnaw
Answer : A
✓ Union Minister of Education and Skill Development and Entrepreneurship Shri Dharmendra Pradhan
flagged off Gabon’s first Agri- SEZ project from New Delhi.
✓ In the primary part of the programme, 30 farmers and 20
✓ B.Sc./M.Sc. Agri and B.Tech/M. Tech Engineering college students from the Gajapati district, will likely
be travelling collectively as agri-technical and technical consultants for the agriculture SEZ which is being
✓ developed underneath this undertaking.
✓ African coverage. Being a trusted accomplice within the journey of socio-economic growth, India has
prolonged concessional loans of over USD 12.3 billion to Africa and USD 700 mn of grant help with
growth tasks in numerous sectors, an official assertion quoted the Union minister as saying.
Q. Which institute has been awarded accreditation as ‘Athi Uttam’ by the National Accreditation Board of
Education and Training (NABET)?
A) Defence Research and Development Organisation (DRDO)
B) Geological Survey of India Training Institute (GSITI)
C) Indian Space Research Organisation (ISRO)
D) Archaeological Survey of India (ASI)
Answer : B
✓ GSITI Hyderabad recognized as "Athi Uttam".
✓ The Geological Survey Training Institute of India (GSITI) located at Hyderabad has been accredited as
“Athi Uttam” by the National Accreditation Board for Education and Training (NABET).
✓ The accreditation recognizes GSITI's excellent services and high standards in the field of Earth Science
training.
✓ The accreditation was granted to GSITI after an on-site assessment conducted by Capacity Building
Commission (CBC), NABET and Quality Control of India.
✓ GSITI, established in 1976, functions under the Ministry of Mines and is headquartered in Hyderabad.
✓ The institute consists of six Regional Training Divisions (RTDs) located at Hyderabad, Nagpur, Jaipur,
Lucknow, Kolkata and Shillong.
✓ Four Field Training Centers (FTCs) are also established at Chitradurga (Karnataka), Raipur
(Chhattisgarh), Jawar (Rajasthan) and Kuju (Jharkhand).
✓ The Institute conducts courses on Remote Sensing sponsored by the Indian Space Research Organization
(ISRO) under the NNRMS programme.
Q. Which Indian documentary has won a special prize in the ‘Health for All’ category at the 4th Annual
Health for All Film Festival (HAFF) held at the World Health Organization (WHO) headquarters in
Geneva?
A) One in 36 Million B) The Man Who Sold His Skin
C) When Climate Change Turns Violent D) Communicating for health impact

Follow us: Official Site, Telegram, Facebook, Instagram, Instamojo 675


Answer : C
✓ WHO Award for 'When Climate Change Turns Violent.
✓ The documentary titled 'When Climate Change Turns Violent' has bagged the special award in the
'Health for All' category at the 4th annual Health for All Film Festival held at the World Health
Organization Headquarters in Geneva.
✓ This documentary has been directed by Vandita Saharia from Rajasthan.
✓ She was the only Indian in the list of winners.
✓ The 4.32-minute documentary 'When Climate Change Turns Violent' focuses on the relationship between
domestic violence, climate change and human trafficking.
✓ The ceremony announced the winning films for seven different categories, while four films received
special mentions by the jury.
Q. Which government has appointed Jane Marriott as first woman envoy to Pakistan?
A) Australia B) Germany
C) United Kingdom D) United States
Answer : C
✓ Britain appoints Jane Marriott as first female envoy to Pakistan.
✓ The United Kingdom appointed Jane Marriott as the next British High Commissioner to Pakistan,
making her the first female British envoy to Islamabad.
✓ Jane Marriott previously served as the High Commissioner to Kenya since September 2019.
✓ She will replace Dr Christian Turner, who held the position of British Ambassador to Pakistan since
December 2019.
✓ Prior to his diplomatic career, he held roles in the Cabinet Office and the Home Office
✓ About United Kingdom
✓ It is an island nation located in northwestern Europe. It consists of four countries: England, Scotland,
Wales and Northern Ireland.
✓ Capital - London
✓ Prime Minister - Rishi Sunak
✓ Emperor - Charles III
Q. The Defence Ministry has signed a contract of 500 crore rupees with which company to strengthen the
Indian Army’s communication system?
A) ICOMM Tele Limited B) Hindustan Aeronautics Ltd (HAL)
C) Bharat Dynamics Ltd (BDL) D) Antrix Corporation Ltd (ACL)
Answer : A
✓ Defence Ministry signs Rs 500 crore contract to strengthen Indian Army’s communication system.
✓ In a significant move to promote indigenous manufacturing and support the vision of Aatmanirbhar
Bharat, the Ministry of Defence has signed a contract with ICOMM Tele Limited, Hyderabad, on June
15, 2023, in New Delhi.
✓ The contract is for the procurement of 1,035 units of 5/7.5 Ton Radio Relay Communication equipment
containers, with a contract value of approximately Rs 500 crore.
✓ The delivery of these containers is scheduled to commence in the current financial year 2023-24.
✓ The acquisition of these Radio Relay Containers fulfills a long-pending requirement for mobile
communication detachments of the Indian Army.

Follow us: Official Site, Telegram, Facebook, Instagram, Instamojo 676


Q. NTPC Barauni has achieved a significant milestone by securing the first rank in the Best Industry
Category at the 4th National Water Awards. NTPC barauni is situated in which state?
A) Jharkhand B) Bihar
C) Chhattisgarh D) Rajasthan
Answer : B
✓ NTPC Barauni secures First Rank at 4th National Water Awards, in Best Industry Category.
✓ NTPC Barauni has achieved a significant milestone by securing the first rank in the Best Industry
Category at the 4th National Water Awards.
✓ The award was presented to CMD, NTPC Gurdeep Singh, and Head of Project, NTPC Barauni,Rajeev
Khanna by the Vice President of India, Jagdeep Dhankhar, in the presence of Union Jal Shakti Minister
Gajendra Singh Shekhawat.
✓ The award ceremony took place at Vigyan Bhawan, New Delhi, on June 17, 2023.
✓ The recognition was given by the Ministry of Jal Shakti, Government of India, highlighting NTPC's
commitment and efforts in conserving and managing water resources.
✓ NTPC has demonstrated its dedication to sustainable practices and environmental stewardship, which
played a crucial role in this achievement.
✓ The company has implemented innovative initiatives to effectively conserve and manage water resources,
resulting in significant water savings and a reduced environmental footprint.

Q. Who has been unveiled the "Dugdh Sanakalan Sathi Mobile App" at Mussorie, Uttarakhand?
A) Narendra Modi B) Narendra Singh Tomar
C) Nirmala Sitharaman D) Mahendra Nath Pandey
Answer : D
✓ Union Minister of Heavy Industries Dr Mahendra Nath Pandey unveiled the "Dugdh Sanakalan Sathi
Mobile App" on 16th June at Mussorie, Uttarakhand.
✓ This innovative application has been developed by Rajasthan Electronics & Instruments Limited (REIL),
located in Jaipur.
✓ REIL is a Central Public Sector Enterprise under the Ministry of Heavy Industries and holds the
prestigious "Mini Ratna" status.
✓ Objective of the app
✓ Its primary objective is to enhance milk quality, promote transparency among stakeholders, and
streamline operations at the village level, particularly within Milk Cooperative Societies.
✓ Key features and benefits of the app
✓ Increased transparency among stakeholders.
✓ Online monitoring of daily milk kept in milk cooperatives.
✓ Real-time milk price updates from cloud servers, ensuring transparency and eliminating human errors.
✓ Direct transfer of milk payment and government subsidy to the bank accounts of beneficiary milk
producers through the app.
✓ Notifications in English, Hindi, Punjabi, Telugu and other languages on milk producers' app for milk
collection.
✓ Importance of app
✓ The app is poised to make a significant impact on the Indian dairy industry by addressing critical
challenges in the milk collection process.
✓ The 'Dudgh Sankalp Sathi' mobile app will provide transparency, efficiency and empowerment to all the
participants involved in the milk collection process.
✓ This will benefit the milk producers and contribute to the development of the dairy sector.

Follow us: Official Site, Telegram, Facebook, Instagram, Instamojo 677


Q. Union Minister of Ports, Shipping & Waterways Sarbananda Sonowal received the first over dimensional
cargo (ODC) transported via waterways at Numaligarh Refinery jetty is situated in which state?
A) Assam B) Odisha
C) Kerala D) Gujarat
Answer : A
✓ Union Minister of Ports, Shipping & Waterways Sarbananda Sonowal received the first over dimensional
cargo (ODC) transported via waterways at Numaligarh Refinery jetty (Assam) on 16th June.
✓ The ODC was transported by Inland Waterways Authority of India (IWAI) vessel MV Marine 66 from
Kolkata to Numaligarh Refinery Jetty via Indo Bangladesh Protocol Route (IBPR).
✓ The Diesel Hydrotreating (DHT) reactor had a net weight of 485 MT, while the gross weight was 521
MT.
✓ The length of this reactor is 31.5 m while the height is 8.250 m and the diameter is 8.00 m.
✓ This is the first consignment transported by the Inland Waterways Authority of India (IWAI), the nodal
agency in charge of inland waterways in the country, Ministry of Ports, Shipping and Waterways, to
augment the capacity of Numaligarh Refinery Limited (NRL) from 3 MMT to 9 MMT.

Q. Which IIT researchers devised a mathematical model on 'Prospect Theory'?


A) IIT Delhi B) IIT Kharagpur
C) IIT Madras D) IIT Roorkee
Answer : C
✓ Indian Institute of Technology Madras (IIT Madras) Researchers have devised a mathematical model on
contract farming to predict farmers' delivery of commodities using a decision-theoretic framework based
on the 'Prospect Theory.'
✓ The development of the model is targeted to aid policymakers in designing mechanisms that would
encourage more firms to offer advance-payment contracts
Q. Which State to get its 7th tiger by clubbing Nauradehi & Durgavati sanctuaries?
A) Madhya Pradesh B) Karnataka
C) Tamil Nadu D) Uttar Pradesh
Answer : A
✓ Tiger State Madhya Pradesh is poised to get its 7th tiger reserve with the state government going forward
with the notification of Nauradehi and Durgavati Wildlife Sanctuaries as a combined protected area.
✓ This new reserve, spanning an expansive 2,339 sq km and around 20km from Bhopal, is yet to be officially
named and will straddle three districts -- Narsinghpur, Sagar, and Damoh.
✓ MP is known for its six tiger reserves :
✓ Kanha, Bandhavgarh, Panna, Pench, Sanjay Dubri, and Satpura. Sources say the seventh will be named
‘Durgavati Tiger Reserve

Q. Which countries are Joins Hands with UN Habitat for Mission LiFE?
A) India and Oman B) India and Kuwait
C) India and Israel D) India and Libya
Answer : B
✓ Embassy of India in Kuwait, in association with the United Nations Habitat Office in Kuwait, organized
a Beach Cleaning Event.

Follow us: Official Site, Telegram, Facebook, Instagram, Instamojo 678


✓ The event, held at the Beach opposite Bneid Al Gar, received tremendous support from various
organizations, including the Office of the Hawally Governor, Kuwaiti Municipality, and Indian
community associations such as the Kuwait Kerala Muslim Cultural Centre (KKMCC), Thrissur
Association of Kuwait (TRASSK), Telugu Kala Samithi (TKS), Kuwait Tamil Peoples Service Centre
(KTPSC), Prvasandra Reddy Association - Kuwait (PRAK), and Kuwait Kannada Koota (KKK).
✓ The Beach Cleaning Event was organized as part of the Mission LiFE (Lifestyle For Environment)
initiative of the Government of India, which was announced by Hon’ble Prime Minister H.E Narendra
Modi at the 2021 UN Climate Change Conference (UNFCCC COP26).
✓ Aim : The mission aims to bring individual behaviors to the forefront of global climate action by
promoting a circular economy and mindful utilization.
✓ Under Mission LiFE, behavioral change solutions target water and waste management sectors,
encouraging climate-friendly practices at the individual, household, and community levels.
Q. Who has built the 'Solar Ultraviolet Imaging Telescope'?
A) C-DAC B) IUCAA
C) DRDO D) Tech Mahindra
Answer : B
✓ The Pune-based Inter-University Center for Astronomy and Astrophysics (IUCAA) has built a unique
space telescope, the Solar Ultraviolet Imaging Telescope (SUIT).
✓ It will be used in ISRO's first solar mission Aditya-L1.
✓ This space telescope will provide a full disk image of the Sun in the 2000 to 4000A wavelength range
which has never been possible before.
✓ This space telescope will be sent with seven payloads in the Aditya-L1 mission.

Q. In which state will the 37th edition of the National Games of India be organized?
A) Himachal Pradesh B) Uttar Pradesh
C) Haryana D) Goa
Answer : D
✓ Goa Chief Minister Pramod Sawant launched the mascot 'Moga' of the 37th edition of the National
Games at a launch ceremony held at the Goa Dr. Syama Prasad Mukherjee Stadium.
✓ The 37th edition of the National Games will be organized at various places in the state of Goa.
✓ National Games in India are organized by the Indian Olympic Association
Q. Which country won the title of Squash World Cup 2023?
A) India B) Iran
C) Turkey D) Egypt
Answer : D
✓ Egypt emerged victorious in the World Squash Championship, defeating Malaysia 2-1 in the final held
in Chennai.
✓ Malaysia secured the silver medal in the championship.
✓ The third place in the championship was shared between the host country, India, and Japan.
✓ Malaysia defeated India with a score of 3-0 in the semi-final held the previous day.
✓ The state of Tamil Nadu has gained recognition in international sports by successfully organizing the
Chess Olympiad within a remarkable four-month timeframe

Follow us: Official Site, Telegram, Facebook, Instagram, Instamojo 679


Q. Indian men’s football team has won the Intercontinental Cup 2023 by defeating which country?
A) Argentina B) Portugal
C) Lebanon D) Brazil
Answer : C
✓ The Indian men’s football team beat Lebanon 2-0 to win the Intercontinental Cup 2023 at the Kalinga
Stadium in Bhubaneswar.
✓ It was India’s first win over Lebanon in 47 years and second in eight meetings.
✓ This was the third edition of the Intercontinental Cup. The Indian football team upset Kenya to win the
inaugural Intercontinental Cup in 2018 while North Korea clinched the crown in 2019 by beating
Tajikistan.
✓ Indian captain – Sunil Chhetri

Q. The inaugural edition of India’s first-ever Women’s Kabaddi League (WKL) commenced in which city?
A) Dubai B) New Delhi
C) London D) New York
Answer : A
✓ The inaugural edition of India’s first-ever Women’s Kabaddi League (WKL) commenced with great
fanfare in Dubai.
✓ The league is being organized by the Kabaddi Federation of India and the Dream Sports Group.
✓ The league consists of eight teams, featuring a total of 96 players, each representing Indian states.
✓ These teams bear the names of various states, including the Delhi Dynamites, Gujarat Angels, Great
Marathas, Haryana Hustlers, Punjab Panthers, Rajasthan Riders, Uma Kolkata, and Bengaluru Hawks.

Q. World Refugee Day is observed on which day?


A) 18 June B) 20 June
C) 19 June D) 17 June
Answer : B
✓ The United Nations Refugee Agency (UNHCR) observes the World Refugee Day on June 20 each year
to raise awareness of the situation of refugees throughout the world.
✓ 2023 Theme: Hope Away from Home
✓ The United Nations General Assembly recognised the day on 4 December 2020.
✓ United Nations Refugee Agency (UNHCR) headquarter – Geneva, Switzerland

Q. In which country Indian Army is participating in 'Exercise Khan Quest 2023'?


A) France B) Japan
C) Mongolia D) Russia
Answer : C
✓ With the participation of military contingents and observers from more than 20 countries, the
multinational peacekeeping joint exercise "Ex Khan Quest 2023" began in Mongolia on 19 June.
✓ The ceremony was inaugurated by Mongolian President Ukhnagin Khurelsukh at the exercise site.
✓ The exercise is jointly sponsored by the Mongolian Armed Forces (MAF) and the United States Army
Pacific Command (USARPAC).
✓ The Indian Army is being represented by a contingent of the Garhwal Rifles.
✓ About Mongolia

Follow us: Official Site, Telegram, Facebook, Instagram, Instamojo 680


✓ It is located in North Central Asia.
✓ Capital: Ulaanbaatar
✓ Currency: Tugrik
✓ President: Ukhnaagiin Khurelsukh

Q. Which state government has launched free power scheme, 'Gruha Jyothi Scheme'?
A) Telangana B) Madhya Pradesh
C) Karnataka D) Uttar Praesh
Answer : C
✓ Karnataka government has launched a free power scheme named 'Gruha Jyothi scheme' for providing
free electricity up to 200 units a month.
✓ Karnataka CM Siddaramaiah also stated that the scheme also extends to tenants living on rent if their
electricity consumption matches the scheme's criteria.
✓ The beneficiaries will get a ‘zero bill’ starting August 1 if usage is within their entitlement.
✓ The Gruha Jyothi scheme is part of the Congress government's five poll promises.
Q. Which Government has implemented Anna Bhagya Scheme i.e. Free Rice Distribution Scheme for the
citizens of state?
A) Kerala B) Telangana
C) Karnataka D) Tamil Nadu
Answer : C
✓ Karnataka's Anna Bhagya Scheme.
✓ The Karnataka government is currently facing challenges in implementing the Anna Bhagya scheme
starting from July 1 due to the unavailability of the required amount of rice.
✓ The Food Corporation of India (FCI) has halted the sale of rice to state governments, including
Karnataka, since June 12.
✓ This discontinuation of rice supply has created difficulties for the implementation of the Anna Bhagya
scheme.
✓ However, Punjab has in principle agreed to supply rice to Karnataka in the required quantity.
✓ About Anna Bhagya Scheme
✓ The state government intends to increase the allocation of free rice from 5 kg to 10 kg per person on
Below Poverty Line (BPL) cards under the Anna Bhagya scheme.
✓ The aim of this increase in free rice allocation is to provide more support and food security to the persons
belonging to BPL category.
✓ The implementation of this revised allocation is scheduled to begin from July 1.
✓ The increased allocation of free rice will cost the state government an estimated ₹840 crore per month.
✓ Annually, the scheme is expected to cost Rs 10,092 crore to the state exchequer
Q. Which state’s chief minister has inaugurated “The Tribune Real Estate Expo 2023” in the state?
A) Rajasthan B) Madhya Pradesh
C) Himachal Pradesh D) Uttarakhand
Answer : C
✓ Himachal Pradesh Chief Minister Sukhvinder Singh Sukhu has inaugurated “The Tribune Real Estate
Expo, 2023, Shimla”.
✓ Around 15 real estate developers and some banks are participating in the expo.
✓ The expo is expected to attract potential buyers and investors from across the country.

Follow us: Official Site, Telegram, Facebook, Instagram, Instamojo 681


✓ Most of these developers are exhibiting their properties situated in Zirakpur, Mohali and other areas close
to Chandigarh.
✓ The expo is being organized by the Himachal Pradesh Real Estate Regulatory Authority (HPRERA) and
the Himachal Pradesh Chamber of Commerce and Industry (HPCCI).

Q. Who has become the first male footballer to play 200 international matches?
A) Lionel Messi B) Cristiano Ronaldo
C) Kylian Mbappe D) Karim Benzema
Answer : B
✓ Portugal's star footballer Cristiano Ronaldo has made another big record. Ronaldo has become the first
football player in history to play 200 international matches.
✓ The 38-year-old Ronaldo set this record as soon as he was named in the playing XI against Iceland in the
2024 Euro Cup qualifying match.
✓ The Guinness World Records team honored Ronaldo for this record before the match.
✓ In the match held in Iceland's capital Reykjavik, Ronaldo scored a goal in the 89th minute and gave the
team a 1–0 victory over Iceland.
✓ Kuwait's Bader Al-Mutawa is in second place in terms of playing most international matches, who has
played a total of 196 matches so far.
✓ Malaysia's Soh Chin Ain is third with 195 matches.
✓ Lionel Messi of Argentina is joint 11th with 175 matches.
✓ Team India captain Sunil Chhetri tops the list playing 137 international matches for India
✓ Cristiano Ronaldo
✓ Cristiano Ronaldo made his international debut for Portugal on 20 August 2003.
✓ Ronaldo also holds the record for most international goals with 123 goals.
Q. The Ministry of Defence has recently approved the deal to acquire the e Predator (MQ-9 Reaper) drones
from which country?
A) Israel B) France
C) Japan D) United States
Answer : D
✓ The Defence Ministry has approved the deal for acquiring the Predator (MQ-9 Reaper) drones from
America for which the final decision would be taken by the Cabinet Committee on Security (CCS).
✓ The deal for the Predator drones was approved by the Defence Acquisition Council meeting.
✓ The DAC is the highest body in the Defence Ministry to take decisions on acquisitions.
✓ The hunter-killer MQ-9 drones are capable of a variety of roles, including maritime surveillance, anti-
submarine warfare, over-the-horizon targeting, airborne early warning, electronic warfare.
✓ The SkyGuardian variant has a wingspan of 79 feet and can carry a payload of 2,155kg on nine
hardpoints.
✓ The SeaGuardian variant can fly over the horizon via satellite for more than 30 hours and has advanced
intelligence, surveillance and reconnaissance (ISR) capabilities

Q. Who is the first Indian swordsman to win a medal in the Asian Championships?
A) Anju Rani B) Bhavani Devi
C) Bhavna Kohli D) Babita Kumari
Answer : B
✓ Bhavani Devi creates history by winning India's first medal in Asian Fencing Championships.

Follow us: Official Site, Telegram, Facebook, Instagram, Instamojo 682


✓ Bhavani Devi won the bronze medal in the women's sabre event at the Asian Fencing Championships
held in Wuxi, China.
✓ Bhavani lost her semi-final match against Uzbekistan's Zainab Daybekova with a score of 14-15.
✓ In the quarterfinals, Bhavani defeated defending world champion Misaki Emura of Japan by a score of
15-10.
✓ Bhavani registered his first win against a Japanese swordsman.
✓ Bhavani's next challenge will be the World Championships to be held in Milan from July 22 to 30.

Q. International Day of Yoga is celebrated on which day?


A) 18 June B) 19 June
C) 20 June D) 21 June
Answer : D
✓ International Yoga Day is celebrated every year on 21 June.
✓ The day aims to promote the many benefits of practicing yoga.
✓ The date chosen coincides with the summer solstice, the longest day of the year in the Northern
Hemisphere.
✓ International Yoga Day encourages physical, mental and spiritual harmony.
✓ The theme chosen for International Day of Yoga 2023 is "Yoga for Vasudhaiva Kutumbakam".
✓ The theme represents the shared desire of "One Earth, One Family, One Future".
✓ Yoga promotes meditation, stress reduction, and overall health and vitality.
✓ In its 9th edition, this year (2023), Prime Minister Narendra Modi will be hosting the ceremony at the
United Nations (UN) headquarter in New York during his state visit to the United States between 20
June to 24th June 2023.
✓ This is the first time that a yoga session was organized by the Prime Minister at this place.
✓ History of International Yoga Day
✓ Prime Minister Narendra Modi proposed the idea of a dedicated Yoga Day in 2014 during his speech at
the 69th session of the United Nations General Assembly.
✓ On 11 December 2014, all 193 member states of the United Nations unanimously agreed to observe 21
June as the International Day of Yoga.
✓ The inaugural ceremony of International Day of Yoga took place on June 21, 2015.
✓ International Yoga Day has received global recognition and is celebrated in more than 190 countries.
Q. What is the theme of the 2023 International Yoga Day?
A) Yoga for Peace B) Yoga for well-being
C) Yoga for Vasudhaiva Kutumbakam D) Yoga for Climate Action
Answer : C
✓ The theme chosen for International Day of Yoga 2023 is "Yoga for Vasudhaiva Kutumbakam".
✓ The theme represents the shared desire of "One Earth, One Family, One Future".
Q. The World Music Day is observed on which day?
A) 19 June B) 20 June
C) 21 June D) 18 June
Answer : C
✓ World Music Day (Fête de la Musique, as called in French) takes place on 21 June every year to honor
the amateur and professional musicians, and to encourage them to showcase their work to the world.
✓ The first ever music day took place in 1982 in Paris

Follow us: Official Site, Telegram, Facebook, Instagram, Instamojo 683


Q. Who has been appointed as the first Muslim female federal judge in US history?
A) Zaynab bint Ali B) Shamila N. Chaudhary
C) Noor Al Hussein D) Nusrat Chowdhary
Answer : D
✓ Nusrat Chowdhary will become the first Muslim woman federal judge in US history.
✓ Nusrat Chowdhry, a civil rights lawyer, has been confirmed by the US Senate for appointment as the first
Muslim female federal judge in US history.
✓ She became the first Bangladeshi American and the first Muslim American woman to become a federal
judge.
✓ She will serve as a federal judge in Brooklyn federal court in New York after receiving a 50-49 vote along
party lines.
✓ Endorsed by the American Civil Liberties Union (ACLU), she currently holds the position of Legal
Director for the ACLU of Illinois.
✓ Chowdhary previously served in the ACLU National Office and was the Deputy Director of the ACLU
Racial Justice Program.
✓ Chowdhury has a notable academic background, having graduated from Columbia, Princeton and Yale
Law Schools.
✓ He has also served as clerk to chief justices in the US District Court and US Circuit Court of Appeals.
✓ Earlier, in 2019, Zahid Qureshi became the country's first federal Muslim judge when he was confirmed
to serve as a district court judge in New Jersey.
Q. Which company has started delivering ‘Armado,’ an Armoured Light Specialist Vehicle (ALSV), built
specifically for the Indian armed forces?
A) Mahindra Defence Systems B) BrahMos Aerospace Private Limited
C) Tata Advanced Systems D) Hindustan Aeronautics Limited
Answer : A
✓ Mahindra Defence Systems has started delivering ‘Armado,’ an Armoured Light Specialist Vehicle
(ALSV), built specifically for the Indian armed forces.
✓ The Mahindra ‘Armado’ Armored Light Specialist Vehicle (ALSV) is a light armoured specialist vehicle
built for use by military and defence forces.
✓ The modular type vehicle allows for efficient maintenance and can be upgraded or configured in the field
for a wide variety of operational roles. The ALSV offers ballistic protection up to B7, STANAG Level ll.
✓ The ALSV provides protected mobility at the front, side, and rear as per STANAG Level I Ballistics and
Blast for four crew members with battle load with ample stowage space for arms and ammunition inside
the crew compartment and additional 400 kgs cargo load-carrying capacity. It can also be upgradable up
to STANAG – II Ballistics.
✓ The Aramado has a maximum speed of 120 kmph and can accelerate from 0 to 60 kmph in 12 seconds.
It has a 30-degree gradability with parking brake holding capacity in full GVW, along with 50 km run-
flat system on all five wheels
Q. The Indian cabinet has given its approval to Micron Technology’s plan to invest $2.7 billion in setting up
a semiconductor testing and packaging unit in which state?
A) Bihar B) Uttar Pradesh
C) Kerala D) Gujarat
Answer : D

Follow us: Official Site, Telegram, Facebook, Instagram, Instamojo 684


✓ The Indian cabinet has given its approval to Micron Technology’s plan to invest $2.7 billion in setting up
a semiconductor testing and packaging unit in Gujarat.
✓ The government will provide production-linked incentives worth Rs 11,000 crore ($1.34 billion) for the
semiconductor plant.
✓ This investment is expected to boost India’s semiconductor industry and aligns with Prime Minister
Modi’s vision of making India a global hub for semiconductor production.
✓ The new facility is anticipated to create jobs and enhance India’s capabilities in the sector, contributing
to its digital ambitions and self-reliance in advanced technologies
Q. Which state will host India’s first MotoGP racing event on 22 and 23 September?
A) Tamil Nadu B) Telangana
C) Uttar Pradesh D) Maharashtra
Answer : C
✓ The Uttar Pradesh government announced that the Buddh International circuit in Greater Noida will
host India’s first MotoGP racing event on 22 and 23 September.
✓ The high-end racing machines in the adventurous championship will enhance the reputation of ‘Brand
UP’.
✓ The development marks the return of a big-ticket motorsport event in the country after Formula 1’s
departure in 2013.
✓ In September last year, top officials from MotoGP visited India to sign a seven-year agreement (MoU)
with Indian race promoters Fairstreet Sports (FSS), which has the same rights on behalf of MotoGP in
India.
Q. Which state's hockey team won the Hockey India Junior Men's National Championship 2023?
A) Uttar Pradesh B) Madhya Pradesh
C) Haryana D) Punjab
Answer : B
✓ Hockey Madhya Pradesh won the 13th Hockey India Junior Men National Championship 2023 by
defeating Hockey Chandigarh in the final at Birsa Munda Hockey Stadium.
Q. Which City has set a new Guinness World Record in the Yoga event 21st June 2023?
A) Pune B) Surat
C) Indore D) Bengaluru
Answer : B
✓ Surat sets Guinness World Record for largest gathering on Yoga Day.
✓ A new Guinness World Record was created with the participation of 1.53 lakh people in the Yoga Day
event in Surat, Gujarat.
✓ Largest gathering for a yoga session at one place, with 1.53 lakh people participating.
✓ The previous record was set in 2018 in Kota, Rajasthan with 1,00,984 participants.
✓ Event Description and Recognition:
✓ Gujarat Minister of State for Home Harsh Sanghaviannounced the new record.
✓ Gujarat Chief Minister Bhupendra Patel attended the state level 'International Yoga Day' celebrations in
Surat.
✓ The representatives of Guinness World Records presented the certificate to Chief Minister Patel.
✓ The participants were given wristbands with QR codes for accurate counting.
✓ The QR code data confirmed the participation of 1.53 lakh people surpassing the previous record.

Follow us: Official Site, Telegram, Facebook, Instagram, Instamojo 685


✓ Scale and Organization:
✓ 2.20 lakh people registered through the online link.
✓ The participants of the Yoga Day event in Surat were accommodated on two stretches of roads spread
over 10.5 km.
✓ A total of 135 blocks were built with LED screens and trainers, each accommodating around 1,000
people.
✓ International Day of Yoga was celebrated across Gujarat at 72,000 places with 1.25 crore participants.
✓ Significance and Schemes:
✓ Chief Minister Patel highlighted the efforts of Prime Minister Narendra Modi in popularizing Yoga
globally.
✓ Yoga and Pranayama played an important role during the COVID-19 pandemic.
✓ The state government is planning to open 21 "Yoga studios" to promote yoga.
✓ The State Yoga Board has trained 5,000 instructors.
Q. With whom did Hindustan Aeronautics Limited tie up for the production of Light Combat Aircraft for
the Indian Air Force?
A) GE Aerospace B) Airbus
C) Boeing D) Collins Aerospace
Answer : A
✓ As Prime Minister Narendra Modi is on his maiden US State visit, GE Aerospace said Thursday (22nd
June 2023) that it has signed a Memorandum of Understanding (MoU) with Hindustan Aeronautics
Limited (HAL) to produce fighter jet engines for the Indian Air Force (IAF).
✓ GE Aerospace and HAL have reached an agreement that includes potential joint production of GE
Aerospace's F414 engines in India. The collaboration is part of the Indian Air Force's Light Combat
Aircraft (LAC) Mk2 program.
✓ However, GE Aerospace is currently working with the US government to obtain the necessary export
authorization for this endeavour.
✓ The plans to develop complex jet engine technology in India by Ohio-based GE Aerospace, a subsidiary
of GE, has been under discussion for over a year now.
✓ The MoU between HAL and GE Aerospace represents a significant step forward for GE's involvement
in India.
✓ The agreement builds upon GE Aerospace's previous commitment to manufacturing 99 engines for the
IAF under the LCA Mk2 program. This new development positions GE Aerospace favourably to expand
its product offerings in India, including the F404 engine currently used in the LCA Mk1 and LCA Mk1A
aircraft.

Q. Who is the first Indian Prime Minister to address a joint session of the US Congress twice?
A) Narendra Modi B) H. D. Deve Gowda
C) Manmohan Singh D) Atal Bihari Vajpayee
Answer : A
✓ PM Modi has become the first Indian Prime Minister to address the joint session of the US Congress
twice. He is the second international leader after Israel's Prime Minister Benjamin Netanyahu to do so.
Earlier, PM Modi had addressed the joint session of the US Congress in the year 2016.

Q. Which country is included in NASA's Artemis program recently?


A) India B) China
C) South Africa D) Brazil

Follow us: Official Site, Telegram, Facebook, Instagram, Instamojo 686


Answer : A
✓ India and US sign Artemis Accord to take Space cooperation to new heights.
✓ During his state visit to the United States, Prime Minister Narendra Modi signed the Artemis agreement
with the US National Aeronautics and Space Administration (NASA), highlighting the growing
importance of India's role as a global space power.
✓ Joint Mission to the International Space Station (ISS)
✓ In addition to the Artemis agreement, NASA and the Indian Space Research Organization (ISRO) have
agreed to launch a joint mission to the International Space Station in 2024.
✓ This collaborative effort will enable both countries to advance their space research and exploration efforts.
✓ What is the Artemis Agreement?
✓ The Artemis Agreement outlines principles for international cooperation in space exploration,
particularly relating to the use of the Moon, Mars, comets and asteroids for peaceful purposes.
✓ Spain, Ecuador and now India have also joined it, taking the total number of participating countries to
28..
Q. Which of the following has partnered with Deendayal Antyodaya Yojana-National Urban Livelihoods
Mission (DAY-NULM) to empower women and provide them with better career opportunities in the
realm of entrepreneurship?
A) United Nations Development Programme (UNDP)
B) United Nations Population Fund (UNPF)
C) United Nations High Commissioner for Refugees (UNHCR)
D) International Renewable Energy Agency (IRENA)
Answer : A
✓ UNDP partners with DAY-NULM towards empowering women entrepreneurs.
✓ The United Nations Development Programme (UNDP) and the Deendayal Antyodaya Yojana-National
Urban Livelihoods Mission (DAY-NULM) recently have joined forces in a collaborative partnership.
✓ The objective of this partnership is to empower women to make career choices in the field of
entrepreneurship.
✓ The partnership aims to provide support to women who are interested in starting or expanding their own
enterprises.
✓ It focuses on sectors such as the care economy, digital economy, electric mobility, waste management,
food packaging, and more.
✓ Entrepreneurship Development and Enterprise Growth.
✓ The project, which spans three years and is extendable beyond 2025, is centered around fostering
entrepreneurship development and accelerating enterprise growth.
✓ In the initial phase, it will cover eight cities.
✓ On-Ground Mobilization and Business Development Services
✓ UNDP and DAY-NULM will work together to engage in on-ground mobilization activities.
✓ This includes identifying areas of urban poverty and potential entrepreneurs.
✓ Key facts about the DAY-NULM
✓ It is a flagship mission under the Union Ministry of Housing and Urban Affairs.
✓ It has the aim to uplift the urban poor by enhancing sustainable livelihood opportunities through skill
development.
✓ Funding: It will be shared between the Centre and the States in the ratio of 75:25. For North Eastern and
Special Categories – the ratio will be 90:10.
✓ United Nations Development Programme (UNDP)
✓ Formation : 22 November 1965
✓ Headquarters : New York City

Follow us: Official Site, Telegram, Facebook, Instagram, Instamojo 687


✓ Head : Achim Steiner
✓ Total members : 170
Q. IFFCO has recently tied up with which country for the export of Nano Liquid Urea?
A) USA B) France
C) Australia D) Germany
Answer : A
✓ Indian Farmers Fertiliser Cooperative Ltd (IFFCO) has signed an agreement with California-based
Kapoor Enterprises Inc for export of nano liquid urea to the US.
✓ The agreement comes at a time when Prime Minister Narendra Modi is on a state visit to the US.
✓ “IFFCO now started exporting world’s 1st Nano Urea invented & manufactured indigenously in India
by IFFCO to USA. An agreement in regard of exporting IFFCO Nano Urea to USA is signed between
IFFCO & Kapoor Enterprises Inc, California," .
✓ As of now, IFFCO exports more than 5 lakh bottles of nano liquid urea to more than 25 countries.
✓ In June 2021, IFFCO launched the world’s first nano urea fertiliser, while nano DAP in April this year
(2023)
✓ Indian Farmers Fertiliser Cooperative (IFFCO)
✓ Founded : 1967
✓ Headquarteres : New Delhi
✓ Chairmam : Dileep Sanghani
✓ MD & CEO : U.S Awasthi.
Q. Prime Minister Narendra Modi has been recently awarded the ‘Order of the Nile’. It is the highest honor
of which country?
A) Nepal B) Egypt
C) South Africa D) Suriname
Answer : B
✓ Prime Minister Narendra Modi was awarded the 'Order of the Nile,' Egypt's highest honor, by President
Abdel Fattah El-Sisi during a ceremony at the Presidential palace in Cairo on 25th June.
✓ The 'Order of the Nile' was established in 1915 and is bestowed upon heads of states, crown princes, and
vice presidents who have rendered invaluable services to Egypt or humanity.
✓ This is the 13th highest state honor conferred upon Prime Minister Modi.
✓ During the first bilateral visit by an Indian Prime Minister to Egypt in 26 years, India and Egypt elevated
their relationship to a 'Strategic Partnership' following extensive discussions between Modi and El-Sisi.
✓ Modi extended an invitation to El-Sisi to attend the G-20 Summit in New Delhi in September.
✓ About Egypt
✓ Egypt is a country located in North Africa.
✓ Egypt is home to famous monuments such as the Giza Pyramid Complex. One of the Seven Wonders of
the World, the Pyramids of Giza are located on the banks of the Nile River. The Great Pyramid of Giza
was built in 2560 BC.
✓ Capital - Cairo
✓ President – Abdel Fattah el-Sisi
✓ Currency - Egyptian pound

Q. Which is the world's best liveable city, according to the Economist Intelligence Unit?
A) Bern B) Vienna
C) Tokyo D) New York

Follow us: Official Site, Telegram, Facebook, Instagram, Instamojo 688


Answer : B
✓ According to the Global Liveability Index 2023 published by the Economic Intelligence Unit (EIU),
Vienna in Austria has topped the ranking of the most liveable cities.
✓ While Copenhagen in Denmark placed second on the list, followed by Melbourne and Sydney.
✓ Damascus in Syria and Tripoli in Libya remained at the bottom of the liveability index.
✓ The list contains the names of 173 cities around the world that have been ranked based on a number of
significant factors, including health care, education, stability, infrastructure and the environment.
✓ Five cities from India – Bengaluru, Ahmedabad, Chennai, New Delhi and Mumbai – are also in the list
based on their performance in various metrics.
✓ From India, New Delhi and Mumbai are at 141st position and Chennai at 144th. Ahmedabad and
Bengaluru are ranked 147 and 148, respectively.
✓ Rank Cities
✓ TOP : Vienna, Austria
✓ 2nd Copenhagen, Denmark
✓ 3rd Melbourne, Australia
✓ 4th Sydney, Australia
✓ 5th Vancouver, Canada
✓ 6th Zurich, Switzerland
✓ 7th Calgary, Canada
✓ 8th Geneva, Switzerland
✓ 9th Toronto, Canada
✓ 10 th
Osaka, Japan and Auckland, New Zealand
✓ 141st New Delhi & Mumbai, India
✓ 144th Chennai, India
✓ 147th Ahmedabad, India
✓ 148th Bengaluru, India
Q. India has signed a Mutual Recognition Arrangement with which country for Authorized Economic
Operators?
A) Saudi Arabia B) Japan
C) UAE D) Israel
Answer : C
✓ India and UAE signed a Mutual Recognition Arrangement for Authorized Economic Operators.
✓ India and the United Arab Emirates (UAE) on 23 June have signed a Mutual Recognition Arrangement
for Authorized Economic Operators.
✓ The agreement was formally signed during the World Customs Organization's (WCO) Customs Co-
operation Council meeting held in Brussels.
✓ Aim of Mutual Recognition Arrangement
✓ The Mutual Recognition Arrangement aims to promote trade facilitation and ease of doing business
between the two countries.
✓ It establishes a framework for recognizing Authorized Economic Operators (AEOs) from both India and
the UAE.
✓ India-UAE Comprehensive Economic Partnership Agreement (CEPA).
✓ The signing of this arrangement is in line with the broader objective of the India-UAE Comprehensive
Economic Partnership Agreement (CEPA) signed in February 2022.
✓ CEPA aims to significantly increase the total value of bilateral trade in goods and services within five
years.
✓ The Mutual Recognition Arrangement is expected to contribute to achieving the trade targets set under
the CEPA, fostering greater economic growth and cooperation between India and the UAE.

Follow us: Official Site, Telegram, Facebook, Instagram, Instamojo 689


Q. Who has received the Guinness World Record Award certificate to become the first men’s player to make
200 international appearances?
A) Cristiano Ronaldo B) Georgina Rodríguez
C) Lionel Messi D) Luka Modrić
Answer : A
✓ Portugal's star footballer Cristiano Ronaldo has made another big record. Ronaldo has become the first
football player in history to play 200 international matches.
✓ The Guinness World Records team honored Ronaldo for this record before the match.
✓ In the match held in Iceland's capital Reykjavik, Ronaldo scored a goal in the 89th minute and gave the
team a 1–0 victory over Iceland.
✓ Cristiano Ronaldo made his international debut for Portugal on 20 August 2003.
✓ Player with most international matches
✓ Kuwait's Bader Al-Mutawa is in second place in terms of playing most international matches, who has
played a total of 196 matches so far.
✓ Malaysia's Soh Chin Ain is third with 195 matches.
✓ Lionel Messi of Argentina is joint 11th with 175 matches.
✓ Team India captain Sunil Chhetri tops the list playing 137 international matches for India
Q. Who is the author of a new book titled “Finance Ministers of India: From Independence to the
Emergency (1947-1977)”
A) Simon Sebag Montefiore B) Nirmala James
C) Ashwini Shrivastava D) A.K. Bhattacharya
Answer : D
✓ Journalist A.K. Bhattacharya wrote a new book titled “Finance Ministers of India: From Independence
to the Emergency (1947-1977)”.
✓ The book is published by Penguin Business an imprint of Penguin Random House.
✓ The book is divided into three main parts:
1. Nehru and his Finance Minister
2. Finance Minister under Shastri and Indira Gandhi
3. Indira Gandhi as her own Finance Minister

Q. Who has inaugurated the Medha Rail Coach Factory in the Sangareddy district of Telangana?
A) Narendra Modi B) Draupadi Murmu
C) Amit Shah D) K. Chandrasekhar Rao
Answer : D
✓ Telangana Chief Minister K. Chandrasekhar Rao (KCR) inaugurated the Medha Rail Coach Factory in
Sangareddy district.
✓ Medha signed an MoU with the Government of Telangana in 2017.
✓ The objective of the MoU is to set up a coach factory in the state.
✓ The land was allotted by the Telangana State Industrial Infrastructure Corporation (TSIIC).
✓ TSIIC allotted 100 acres of land for Rail & Metro coach manufacturing.
✓ Medha Rail Coach Factory is located in Kondakal village.
✓ Medha and Swiss railway rolling stock manufacturer Stadler Rail signed a joint venture (JV) agreement.
✓ The joint venture Telangana unit is meant to cater to its tenders from the Indian market and exports from
customers in the Asia Pacific region.

Follow us: Official Site, Telegram, Facebook, Instagram, Instamojo 690


Q. Which state government to build 1st underwater tunnel at a cost of Rs 6,000 crore in the state?
A) Assam B) Kerala
C) Telangana D) Karnataka
Answer : A
✓ Assam's first underwater tunnel to be constructed between Numaligarh and Gohpur.
✓ The first underwater tunnel in Assam will be constructed between Numaligarh and Gohpur at an
estimated cost of Rs 6,000 crore.
✓ Assam Chief Minister Himanta Biswa Sarma announced that the tender for the preparation of Detailed
Project Report (DPR) will open in the next month.
✓ The idea of an underwater tunnel arose from the many bridges built over the Brahmaputra River to
facilitate the movement of both vehicles and railways.
✓ Discussions were held to determine the location and it was decided that the tunnel would be built between
Numaligarh and Gohpur.
✓ The tender for the DPR will open on July 4 and Sarma expressed hope that work on the tunnel could
begin during his tenure as chief minister.
✓ Prime Minister Narendra Modi has approved the project, and once the tunnel is completed, it will bring
the two banks, north and south, of the Brahmaputra river closer together.
Q. Which IT company has bagged a digital transformation deal from Denmark-based Danske Bank
estimated to be around USD 454 million for five years?
A) IBM B) HCL
C) TCS D) Infosys
Answer : D
✓ Infosys bags $454-million deal from Danske Bank.
✓ IT major Infosys has bagged a digital transformation deal from Denmark-based Danske Bank estimated
to be around USD 454 million for five years.
✓ As part of the strategic collaboration with Danske Bank, Infosys will acquire Danske Bank’s IT centre in
India, which employs 1400 digitally skilled professionals.
✓ This collaboration will help Danske Bank achieve its strategic priorities towards better customer
experiences, operational excellence, and a modernized technology landscape, powered by next-gen
solutions.
✓ INFOSYS
✓ Founded : 1981
✓ Headquarters : Bangalore, Karnataka
✓ Chairman : Nandan Nilekani
✓ MD & CEO : Salil Parekh

Q. The first hydrogen-powered trains in India are expected to start from which state by the end of 2023?
A) Assam B) Odisha
C) Haryana D) Madhya Pradesh
Answer : C
✓ India’s first hydrogen-powered train to run from Jind district, Haryana.
✓ The first hydrogen-powered trains in India are expected to start from the Jind district of Haryana by the
end of 2023.
✓ The officials claimed that hydrogen-powered trains are running only in Germany, and the whole world
is watching the project to see how India will start such trains.

Follow us: Official Site, Telegram, Facebook, Instagram, Instamojo 691


✓ India’s first hydrogen plant is being set up near the railway junction in Jind district. The development of
the plant has reached the final stage and hydrogen will be produced from water.
✓ The first prototype of the hydrogen train is expected to be introduced in the financial year 2023-2024
between Jind and Sonipat on the Northern Railway.
✓ The hydrogen fuel-based train of eight bogies will be environmentally friendly.
✓ Hydrogen trains use fuel cells to convert hydrogen and oxygen into electricity.
✓ They offer a clean and eco-friendly alternative to diesel trains.
✓ The initiative aims to reduce carbon emissions and tackle pollution.
✓ Hydrogen-powered trains produce zero direct emissions, with water vapour being the only byproduct.
Q. Who has won her second Ladies European Tour (LET) at the Tipsport Czech Ladies Open 2023?
A) Diksha Dagar B) Aditi Ashok
C) Anirban Lahiri D) Jyoti Randhawa
Answer : A
✓ Indian golf player, Diksha Dagar has secured her second Ladies European Tour (LET) at the Tipsport
Czech Ladies Open 2023.
✓ In the final, she defeated Trichat Cheenglab of Thailand.
✓ The 22-year-old southpaw Diksha won her first LET title back in 2019 in her rookie year and in 2021 she
was part of the winning team at the Aramco Team Series in London.
✓ After Aditi Ashok, Diksha Dagar is the only other Indian woman golfer to have won two or more LET
titles.
Q. Who has been named the prestigious (Kendra) Sahitya Akademi Bal Sahitya Puraskar 2023 for the novel
Perumazhayathe Kunjithalukal?
A) Basu Chatterjee B) Shubira Prasad
C) Meghan Markle D) Priya A S
Answer : D
✓ Indian writer, Priya A S has been named the prestigious (Kendra) Sahitya Akademi Bal Sahitya Puraskar
2023.
✓ She was awarded for children’s literature in Malayalam for her 2018 novel Perumazhayathe
Kunjithalukal.
✓ She also won the Kerala Sahitya Akademi Award for Children’s Literature for the same novel in 2020.
✓ The (Kendra) Sahitya Akademi Bal Sahitya Puraskar consists of Rs 50,000 and an engraved copper
plaque. Panels of three members each picked the winners in each language.
✓ In 2014, Priya won (Kendra) Sahitya Akademi Translation Prize for translating Arundhati Roy’s Booker
Prize-winning novel The God of Small Things into Malayalam

Q. When is the Micro, Small and Medium-sized Enterprises Day observed globally?
A) 27 June B) 25 June
C) 26 June D) 24 June
Answer : A
✓ The United Nations Micro, Small and Medium-sized Enterprises Day is held on June 27 since 2017 to
celebrate the work of MSME in local and global economies and their contribution to sustainable
development.
✓ The day also recognise the contribution of these industries in the implementation of the Sustainable
Development Goals (SDGs).

Follow us: Official Site, Telegram, Facebook, Instagram, Instamojo 692


✓ 2023 Theme is Future-ready MSMEs for India@100
Q. What is the theme of Micro, Small and Medium-sized Enterprises Day 2023?
A) MSME Day 2021: The Digital Dimension
B) COVID-19: The Great Lockdown and its impact on Small Business
C) Big Money For Small Business: Financing the SDGs
D) Future-ready MSMEs for India@100
Answer : D
Q. Who has flagged off the “Bharat in Paris” campaign with a short marathon at Jawaharlal Nehru Stadium
in New Delhi?
A) Narendra Modi B) Nitin Gadkari
C) Amit Shah D) Anurag Thakur
Answer : D
✓ Union Minister Anurag Thakur flagged off "Bharat in Paris" campaign.
✓ The Indian Olympic Association (IOA) has launched the "Bharat in Paris" campaign to create awareness
and promote sports across India ahead of the Paris Olympics games.
✓ The 2024 Olympics will be held in Paris, France, from July 26 to August 11.
✓ Target Olympic Podium Scheme (TOPS)
✓ It was created in July 2014 under the auspices of the National Sports Development Fund (NSDF) with
the objective of identifying and preparing potential medal prospects for the Olympic and Paralympic
Games.
✓ It is a flagship program of the Ministry of Youth Affairs and Sports.
✓ The idea of the scheme is to look to the future and fund a developmental group of athletes who have the
potential to bring medals to the Olympic Games in Paris in 2024 andLos Angeles in 2028.

Q. In which state the historic Kharchi Puja has been celebrated recently?
A) Manipur B) Tripura
C) Meghalaya D) Nagaland
Answer : B
✓ Historical Kharchi Puja commences in Tripura
✓ In a vibrant display of cultural heritage and religious fervour, the Historical ‘Kharchi Puja’ commenced
at the revered fourteen gods temple in Khayerpur, Tripura.
✓ The auspicious event marked the revival of ancient customs and traditions, drawing enthusiastic devotees
from far and wide.
✓ The Historical Kharchi Puja is a significant religious festival celebrated by the people of Tripura to honour
the fourteen deities, believed to be the guardians of the state.

Q. Who has launched NANDI (NOC Approvals for New Drugs & Inoculation System) portal?
A) G. Kishan Reddy B) Parshottam Rupala
C) Bhupender Yadav D) Mansukh L. Mandaviya
Answer : B
✓ Parshottam Rupala launched NANDI Portal.

Follow us: Official Site, Telegram, Facebook, Instagram, Instamojo 693


✓ Union Fisheries, Animal Husbandry and Dairying Minister Parshottam Rupala on 26th June at Krishi
Bhawan, New Delhi launched NANDI (NOC Approvals for New Drugs & Inoculation System).
✓ The development of the NANDI portal was carried out in collaboration with the Central Drugs Standard
Control Organization (CDSCO) and the Center for Development of Advanced Computing (CDAC).
✓ The portal aligns with the vision of Digital India and embodies the principles of Minimum Government
and Maximum Governance by leveraging IT systems.
✓ The primary objective of the portal is to expedite the assessment and examination of veterinary product
proposals, ensuring their safety, efficacy, and essentiality.
✓ The NANDI portal is expected to support the development of India as a global manufacturing hub for
veterinary products and encourage research and development efforts to enhance animal health

Q. In which city will the FIFA Club World Cup 2023 be organized?
A) Dubai B) New Delhi
C) Jeddah D) Tehran
Answer : C
✓ FIFA Club World Cup 2023 will be organized in Jeddah city of Saudi Arabia.
✓ The tournament will be played at King Abdullah Sport City Stadium and Prince Abdullah Al Faisal
Stadium.
✓ In February, FIFA selected Saudi Arabia for the Club World Cup 2023.
✓ Saudi Arabia will host the Asian Cup in the year 2027.
✓ FIFA WORLD CUP
✓ 2022 : Qatar (32 Teams)
✓ 2026 : Canada, Mexico,US (48 teams)
✓ FIFA U-20 Women's World Cup
✓ 2022 : Costa Rica
✓ FIFA U-17 World Cup
✓ 2022- Women's : India
✓ 2023 - Women's : Australia and New Zealand
✓ Most successful team(s) Men's : Brazil (5 titles)
✓ Fédération Internationale de Football Association (FIFA)
✓ Founded : 21 May 1904
✓ Founded : Paris, France
✓ Headquarters : Zürich, Switzerland
✓ Membership : 211
✓ Languages : English, French, German, Spanish & Italian
✓ First President : Robert Guérin
✓ President : Gianni Infantino (Switzerland)
✓ Secretary General : Fatma Samoura
Q. Who has launched a campaign called “Jeetenge Hum” to support the Indian cricket team in the 2023
Cricket World Cup?
A) Reliance Industries B) Wipro Group
C) TATA Group D) Adani Group
Answer : D
✓ Adani Group has launched a campaign called “Jeetenge Hum” to support the Indian cricket team in the
2023 Cricket World Cup.
✓ His ports-to-energy conglomerate on Saturday gathered 1983 World Cup Cricket legends including Kapil
Dev to launch the 2023 World Cup winning campaign.

Follow us: Official Site, Telegram, Facebook, Instagram, Instamojo 694


✓ The “Jeetenge Hum” campaign serves as a platform for national pride and support for the Indian cricket
team.
✓ The campaign encourages fans to believe in the team’s ability to emerge victorious at the World Cup.
✓ About “Jeetenge Hum” campaign.
✓ The campaign commemorates the 40th anniversary of India’s historic victory in 1983.
✓ The campaign encourages Indian cricket fans to unite and unite in support of Team India.
✓ The campaign draws inspiration from the Adani Group’s ethos of “Kar Ke Dikhaya, Kar Ke
Dikhayenge”.
✓ It epitomizes an indomitable spirit of achievement, both in cricket and business.
Q. Which airport has launched a new app called BLR Pulse to provide hassle-free experience to the
commuters at Kempegowda International Airport?
A) Mangaluru International Airport B) Shivamogga Airport
C) Kannur International Airport D) Bangalore International Airport
Answer : D
✓ Bangalore International Airport Limited (BIAL) has launched a new app called BLR Pulse to provide
hassle-free experience to the commuters at Kempegowda International Airport.
✓ The app is made available for both android and iPhone users.
✓ Through the BLR Pulse app, people can monitor real-time flight updates, book a table at restaurants
inside the airport, connect WiFi, gather information on check-in gates, and even track the flights which
are delayed.
✓ It is an omnichannel platform, developed in collaboration with GrayMatter Software Services.
Q. Who is the author of the book titled ‘The Yoga Sutra for Children’?
A) Amrita Mahale B) Meghna Pant
C) Roopa Pai D) Samhita Arni
Answer : C
✓ A book titled ‘The Yoga Sutra for Children’ by Roopa Pai.
✓ After her award-winning bestselling book ‘The Gita for Children’, author Roopa Pai’s upcoming
children’s book will unravel the secrets of Patanjali’s 2,000-year-old text on Yoga. ‘
✓ ‘The Yoga Sutras for Children’, published by Hachette India, aims to connect and relate the practice of
Yoga to children’s everyday lives and helps them use their mind power to bring out the best in themselves.
✓ Peppered with ‘Think’ and ‘Do’ sections for a wholesome and interactive experience, the book shares
Maharishi Patanjali’s techniques to “nurture energies and attitudes, enabling them to do small and big
things in a focused and balanced way”
Q. Union Minister Parshottam Rupala has recently launched ‘Report Fish Disease App’ to connect directly
to district fisheries officers and scientists. It is funded under which scheme?
A) Pradhan Mantri Matsya Sampada Yojana
B) Pradhan Mantri Vaya Vandana Yojana
C) Pradhan Mantri Jeevan Jyoti Bima Yojana
D) Pradhan Mantri Mudra Yojana
Answer : A
✓ Parshottam Rupala launched the ‘Report Fish Disease’ App.

Follow us: Official Site, Telegram, Facebook, Instagram, Instamojo 695


✓ Union Minister for Fisheries, Animal Husbandry and Dairying Parshottam Rupala launched the 'Report
Fish Disease' (RFD) app at Krishi Bhavan on 28 June to enhance the farmer-based disease reporting
system and improve the reporting of aquatic animal diseases in India.
✓ The RFD app has been developed by ICAR-NBFGR (Indian Council of Agricultural Research - National
Bureau of Fish Genetic Resources) under the National Surveillance Program for Aquatic Animal
Diseases (NSPAAD).
✓ NSPAAD is funded by the Department of Fisheries, Ministry of Fisheries, Animal Husbandry and
Dairying, Government of India, under the Pradhan Mantri Matsya Sampada Yojana.
✓ The primary objective of the RFD app is to assist farmers in improving their reporting methods, obtaining
scientific guidance, and minimizing losses caused by diseases.
✓ This, in turn, helps increase farmers' income and overall profitability.
✓ Benefits of RFD App
✓ The RFD app enables farmers to report disease incidents in finfish, shrimps, and mollusks on their farms,
allowing them to connect with field-level officers and fish health experts.
✓ The implementation of the Fish Disease Reporting App has a significant impact on fish disease
management
Q. How many products of Uttar Pradesh have been given the Geographical Indication (GI) tag from the
Geographical Indications Registry in Chennai recently?
A) 8 B) 7
C) 6 D) 9
Answer : B
✓ The seven products of Uttar Pradesh have been given the Geographical Indication (GI) tag from the
Geographical Indications Registry in Chennai.
✓ ‘Amroha Dholak’, ‘Mahoba Gaura Patthar Hastashlip’, ‘Mainpuri Tarkashi’, ‘Sambhal Horn Craft’,
‘Baghpat Home Furnishings’, ‘Barabanki Handloom Product’ and ‘Kalpi Handmade Paper’ are the
products that been given the Geographical Indication (GI) tag.
✓ The Amroha Dholak is a musical instrument made of natural wood. Mango, jackfruit and teakwood is
preferred for making the dholak’s.
✓ Baghpat and Meerut are famous for their exclusive handloom home furnishing product and running
fabrics in cotton yarn since generations, and only cotton yarn are used in the handloom weaving process.
✓ The Barabanki Handloom Product has also being bestowed with a GI tag.
✓ For Kalpi Handmade Paper, the application was filed by Hasht Nirmit Kagaj Samiti, Industrial Estate,
Kalpi, Uttar Pradesh. Kalpi has historically been a centre of handmade paper manufacturing.
✓ The Mahoba Gaura Patthar Hastashlip is a stone craft.
✓ Tarkashi, a popular art form from Mainpuri in Uttar Pradesh, is primarily brass wire inlay work on wood.
✓ In the Sambhal Horn Craft, the raw material is procured from dead animals. They are completely hand-
made.
Q. Which state’s police department has launched ‘Operation Conviction’ to combat criminals and mafias in
the state?
A) Uttar Pradesh B) Madhya Pradesh
C) Himachal Pradesh D) Kerala
Answer : A
✓ The Uttar Pradesh Police Department has launched ‘Operation Conviction’ to combat criminals and
mafias in the state.

Follow us: Official Site, Telegram, Facebook, Instagram, Instamojo 696


✓ The initiative will expedite the prosecution of cases registered under the Protection of Children from
Sexual Offences (POCSO) Act, as well as those involving cow slaughter, religious conversion, rape, and
murder.
✓ ‘Operation Conviction’ will ensure the immediate arrest of criminals, the collection of strong evidence
against them, quality investigation, and effective representation of cases in courts so that they are
punished in the shortest amount of time.
Q. The Ministry of Power has launched a new platform “UTPRERAK” to promote clean technologies and
enhance energy efficiency in the country. What is the full form of UTPRERAK?
A) Unnat Technical Pradarshan Kendra B) Unnat Technology Pradarshan Kendra
C) Unnat Technology Pariksha Kendra D) Unnat Takniki Pradarshan Kendra
Answer : D
✓ Govt launches UTPRERAK to adopt Energy Efficient Technologies.
✓ The Ministry of Power has established a new platform called Advanced Industrial Technology
Demonstration Center (AITDC) for industry stakeholders to demonstrate and exchange knowledge on
new energy-efficient technologies in the country.
✓ The new center, also called Unnat Takniki Pradarshan Kendra (UTPRERAK), is expected to promote
clean technologies and enhance energy efficiency in the country through knowledge sharing and
demonstrations of new technologies.
✓ The UTPRERAK center is expected to become a leading reference and resource institution for industrial
energy-efficient technologies.
✓ The center will offer comprehensive programs to enhance the skills and knowledge of industry personnel,
contributing to India’s goal of becoming a developed nation by 2047.

Q. Which University has topped the QS World Rankings 2024?


A) University of Oxford B) University of Cambridge
C) Stanford University D) Massachusetts Institute of Technology
Answer : D
✓ The Indian Institute of Technology (IIT), Bombay emerged as India’s highest-ranked educational
institute, according to QS World Rankings 2024.
✓ Globally, it was ranked 149th and was the only such Indian institute in the top 150. However, it is the
highest rank the institute has ever attained.
✓ IIT Bombay showed an improvement from last year’s QS world rankings when it took the 172nd spot.
✓ It was followed by IIT Delhi (ranked 197th globally), slipping from 174th last year. These were the only
two institutes in the top 200 on the list.
✓ At the third spot was the Indian Institute of Science, Bangalore, ranked 225th globally. It slipped sharply
from 155th place last year.
✓ While in the overall ranking, the Massachusetts Institute of Technology (MIT), United States topped the
rankings for the twelfth consecutive year.
✓ UK’s Cambridge University ranked second followed by Oxford University.
✓ QS World University Rankings 2024: Massachusetts Institute of Technology (MIT) Tops for 12th Year,
Indian Universities Make Gains.
✓ QS World University Rankings 2024: Top Performers
✓ Top : MIT, US
✓ 2nd : University of Cambridge, UK
✓ 3rd : University of Oxford, UK
✓ 4th : Harvard University, US
✓ 5th : Stanford University, US.

Follow us: Official Site, Telegram, Facebook, Instagram, Instamojo 697


Q. Which of the following IIT ranks 149th in the QS World University Rankings 2023?
A) IIT Bombay B) IIT Madras
C) IIT Kanpur D) IIT Kharagpur
Answer : A
✓ Indian Institute of Technology (IIT) Bombay has made it to the top 150 in the QS World University
Rankings 2023.
✓ IIT Bombay made a remarkable leap to climb 23 positions to secure the 149th rank globally in the 20th
edition of the rankings.
✓ Earlier, only the Indian Institute of Science (IISc) achieved the feat in 2016 with a rank of 147.
Q. India becomes the second-largest country with the biggest road network after beating which country?
A) China B) Japan
C) Russia D) United States
Answer : A
✓ India becomes the second-largest country with the biggest road network after beating China.
✓ While the United States remains in the number one position.
✓ The report says India successfully reserved the position by spreading 1.45-lakh km of road connectivity
over the last 8 years
✓ India had 91,287 km of road network only. During his tenure, the minister gripped on NHAI, which has
constructed, fixed and improved highways and expressways.
✓ NHAI has constructed more than 30,000 km of highways and expressways nationwide.
Q. Who has been appointed as the first Indian council member of the Institute of Chartered Accountants in
England and Wales (ICAEW) residing overseas?
A) Bhavish Aggarwal B) Shradha Sharma
C) Vishesh Chandiok D) Trishneet Arora
Answer : C
✓ Vishesh Chandiok appointed initiated into ICAEW.
✓ CEO of Grant Thornton Bharat, Vishesh C. Chandiok has been appointed as the first Indian council
member of the Institute of Chartered Accountants in England and Wales (ICAEW) residing overseas.
✓ This recognition honors Chandiok’s significant contributions to the global accounting profession.
✓ His appointment highlights the influence of Indian professionals in accounting and finance globally.
✓ Founded in 1880, ICAEW has played a pivotal role in shaping the accounting industry and works with
governments, regulators and business leaders globally
Q. Which organization has organized an Anusandhaan Chintan Shivir’ with an aim to encourage military
research and development?
A) HAL B) BDL
C) DRDO D) ISRO
Answer : C
✓ The Defence Research and Development Organisation (DRDO) has organized an Anusandhaan Chintan
Shivir in New Delhi aimed at promoting Defence Research and Development in the industry and
academia.

Follow us: Official Site, Telegram, Facebook, Instagram, Instamojo 698


✓ At the event, a list of 75 technology priority areas was released, encouraging indigenous innovation in
defence technologies and fostering self-reliance.
✓ The DRDO Technology Foresight 2023, listing current and future technology development tasks, was
also unveiled.
✓ Chief of Defence Staff General Anil Chauhan also attended the event as the Chief Guest with other
dignitaries.
DRDO IN NEWS 2023
▪ DRDO organises ‘Anusandhaan Chintan Shivir’ to encourage Defence R&D.
▪ Larsen & Toubro (L&T) and the Defence Research and Development Organisation (DRDO) have
formed a partnership to create an Indigenous Air Independent Propulsion (AIP) System for the
submarines in the Indian Navy.
▪ DRDO Industry Academia Centre of Excellence inaugurated at IIT Hyderabad.
▪ Indian Navy and Defence Research & Development Organisation (DRDO) collaborated to conduct the
first successful test trial of a locally-made Air Droppable Container called ‘ADC-150’ from the IL 38SD
aircraft off the coast of Goa on April 27, 2023.
▪ IIT Madras & DRDO tie-up for Advanced Defence Technologies.
▪ Defence Research and Development Organisation (DRDO) celebrated its 65th foundation day on 1st
January 2023
▪ Defence Research and Development Organisation (DRDO)
▪ Founded : 1 January 1958
▪ Headquarters : New Delhi
▪ Motto : "Strength's Origin is in Knowledge"
▪ Chairman : Sameer V Kamat
Q. Which Indian women’s cricketer has recently joined MCC World Cricket Committee (WCC)?
A) Mithali Raj B) Jhulan Goswami
C) Harmanpreet Kaur D) Poonam Yadav
Answer : B
✓ Indian woman cricketer Jhulan Goswami has joined England women’s team captain Heather Knight and
Eoin Morgan to become a member of the MCC World Cricket Committee (WCC).
✓ This committee is an independent body that consists of current and former international cricketers,
umpires, and officials from various countries.
✓ Jhulan Goswami retired from international cricket in 2022. Throughout her impressive two-decade-long
career, Goswami took over 300 wickets in white-ball cricket across 272 appearances and secured 44
wickets in 12 Test matches.

Q. The National Statistics Day is observed by the Government of India on which day?
A) 28 June B) 29 June
C) 27 June D) 30 June
Answer : B
✓ The Government of India celebrates Statistics Day on June 29 every year since 2007 to popularize the
use of Statistics in everyday life and sensitize the public as to how Statistics helps in shaping and framing
policies.
✓ The day marks the birth anniversary of Father of Indian Statistics, Prof. P C Mahalanobis, in recognition
of his invaluable contribution in establishing the National Statistical System.
✓ The theme of National Statistics Day, 2023 is “Alignment of State Indicator Framework with National
Indicator Framework for Monitoring Sustainable Development Goals”.

Follow us: Official Site, Telegram, Facebook, Instagram, Instamojo 699


Q. National Statistics Day in India is celebrated to commemorate which statistician's birth anniversary?
A) C.R. Rao B) Srinivasa, Ramanujan
C) V.S. Huzurbazar D) PC Mahalanobis
Answer : D
✓ National Statistics Day is marked on June 29 in India to celebrate the notable contribution in the fields
of economic planning and the statistical development made by Prof. Prasanta Chandra Mahalanobis.
Q. Which state government has signed an MoU with American chipmaker firm Micron Technology for a
semiconductor assembly and test facility in the state?
A) Gujarat B) Madhya Pradesh
C) Uttar Pradesh D) Tamil Nadu
Answer : A
✓ Gujarat government signs MoU with Micron Technology for semiconductor assembly and test facility.
✓ Gujarat government signs MoU with US chip maker firm Micron Technology for semiconductor
assembly and testing facility in Ahmedabad.
✓ The MoU was signed in the presence of Chief Minister Bhupendra Patel and Union Electronics and IT
Minister Ashwini Vaishnav.
✓ The semiconductor assembly and testing facility will be located at Sanand in Ahmedabad district.
✓ The Central Government will provide 50 per cent financial assistance towards the total project cost.
✓ The State Government of Gujarat will provide an incentive of 20 per cent of the total project cost.
✓ The Semiconductor Assembly and Test Plant will generate 5,000 direct employment opportunities.
✓ Micron Technology:
✓ Micron Technology, Inc. is an American manufacturer of computer memory and computer data storage.
✓ Their product portfolio includes dynamic random-access memory (DRAM), flash memory, and USB
flash drives.
✓ Micron markets its consumer products, such as memory modules, under the Crucial brand.
✓ Micron and Intel collaborated to form IM Flash Technologies, which focused on the production of
NAND flash memory.
✓ Micron owned Lexar between 2006 and 2017, further expanding its presence in the flash memory market.
✓ Establishment - October 5, 1978
✓ Founders - Ward Parkinson, Joe Parkinson, Dennis Wilson and Doug Pittman
✓ Headquarters - Boise Idaho.
Q. Who has unveiled India's first-ever report on Critical Minerals for India?
A) Pralhad Joshi B) Piyush Goyal
C) Narendra Modi D) Dharmendra Pradhan
Answer : A
✓ India's first-ever report on Critical Minerals for India unveils.
✓ The report identifies 30 critical minerals that are crucial for various sectors such as defense, agriculture,
energy, pharmaceuticals, and telecom.
✓ This comprehensive list represents a significant milestone in India's efforts to achieve self-reliance and
security in terms of mineral resources.
✓ The report aligns with India's vision of attaining 'Net Zero' targets, emphasizing the importance of
sustainable resource management.
✓ The purpose of the list is to provide guidance for policy formulation, strategic planning, and investment
decisions within the mining sector.

Follow us: Official Site, Telegram, Facebook, Instagram, Instamojo 700


✓ By identifying critical minerals, India aims to enhance its domestic production capabilities, reduce
dependence on imports, and strengthen its mineral security.
✓ The report's findings will play a vital role in shaping India's mineral resource policies and facilitating
long-term sustainability in key sectors of the economy.
✓ What are Critical Minerals?
✓ Critical minerals are a specific group of minerals that play a vital role in various sectors of the economy,
including defense, energy, telecommunications, agriculture, and pharmaceuticals.
✓ These minerals are essential for the production of high-tech products, renewable energy technologies,
advanced defense systems, and other critical applications.
✓ Critical minerals possess unique properties and characteristics that make them difficult to substitute or
replace with alternative materials.

Q. Which countries to upgrade defense partnership boost maritime security?


A) India and South Korea B) India and Singapore
C) India and Philippines D) India and Thailand
Answer : C
✓ India and the Philippines are embarking on an exciting journey of collaboration in the defence sector,
with a particular focus on the maritime domain.
✓ Through different initiatives including official-level interactions, resident Defense Attaché office in
Manila, and the consideration of concessional Line of Credit, both nations are keen on deepening their
defence ties.
✓ India and the Philippines had the 5th meeting of the Joint Commission on Bilateral Cooperation in Delhi

Follow us: Official Site, Telegram, Facebook, Instagram, Instamojo 701


BEST MCQ WITH FACTS
MAY 2023

Q. Who has been appointed as the first woman chairperson of the Competition Commission of India?
A) Rekha Sharma B) Ravneet Kaur
C) Gurmeet Singh Badal D) Kavya Kohli
Answer : B
✓ Ravneet Kaur has been appointed as the new chairperson of the Competition Commission of India (CCI)
by the Central Government.
✓ She has become the first woman officer to reach this position.
✓ Significantly, this post was vacant since the retirement of Ashok Kumar Gupta in October 2022.
✓ Sangeeta Verma is currently serving as the interim chairperson.
✓ Competition Commission of India is a statutory body, it was formed in 2003.
✓ Competition Commission of India (CCI)
✓ It is the chief national competition regulator in India.
✓ It was set up under the Competition Act, 2002.
✓ It comes under the Ministry of Corporate Affairs.
✓ It promotes healthy market competition and prevents activities that have an adverse effect on competition
in India.
✓ CCI also approves merger and acquisitions of companies in India so that the two merging entities do not
dominate the market unfairly.
✓ Headquarter - New Delhi
Q. Which rescue operation has been launched by India in Myanmar affected by Cyclone Mocha?
A) Operation Durga
B) Operation Shakti
C) Operation Madad
D) Operation Karuna
Answer : D
✓ India launched "Operation Karuna" to provide humanitarian assistance for people in Myanmar, which
is affected by Cyclone Mocha.
✓ Three ships from India carrying relief material reached Yangon on 18th May.
✓ Indian Naval Ships Shivalik, Kamorta and Savitri were the first Naval ships to arrive at Yangon with
relief material on 18th May.
✓ The ships are carrying emergency food supplies, tents, essential medicines, water pumps, portable
generators, clothes, hygiene items, etc.
✓ About Cyclone Mocha
✓ Cyclone Mocha which recently hit Myanmar has been classified as an extremely severe cyclonic storm
by the IMD.
✓ It has been classified as a 'Super Cyclone' by the global weather website Zoom Earth.
✓ The name of the storm 'Mocha' has been suggested by Yemen.
✓ Mocha originated in the Bay of Bengal.
✓ With a recorded wind speed of 277 kmph, Mocha has become the strongest all-season cyclone to form in
the Arabian Sea and Bay of Bengal since 1982.

Follow us: Official Site, Telegram, Facebook, Instagram, Instamojo 702


IMPORTANT OPERATION & CODE NAME IN NEWS 2023
OPERATION JERICHO
✓ The Mizo Rebellion of 1966, also known as Operation Jericho, witnessed the Indian Air Force’s strategic
involvement in aiding the Army’s efforts to quell the separatist movement led by the Mizo National
Front.
✓ OPERATION BROADER SWORD
India and the United States recently joined forces in Operation Broader Sword, a multi-agency operation
aimed at preventing the illegal shipment of pharmaceuticals, devices, and precursor chemicals through
the International Mail System (IMS).
✓ OPERATION KARUNA
India launched "Operation Karuna" to provide humanitarian assistance for people in Myanmar, which
is affected by Cyclone Mocha.
✓ OPERATION KAVERI
India launches Operation Kaveri to evacuate Indian citizens stranded in Sudan
✓ OPERATION DOST
Due to the recent earthquake in Turkey and Syria, India has launched a campaign named 'Operation
Dost' to help the affected people.
✓ OPERATION GARUDA
Central Bureau of Investigation has launched a multi-phase ‘Operation Garuda’ against illicit drug
trafficking network.
✓ OPERATION YATRI SURAKSHA
To give assurance of the safety to the passengers, the Indian railways with the coordination of Railway
Protection Force has launched a pan India drive under the code name Operation Yatri Suraksha.
✓ OPERATION “NARCOS”
Railway Protection Force (RPF) recently recovered narcotic products worth Rs 7.40 crore under
"Operation Narcos".
✓ OPERATION SANKALP
Indian Navy’s stealth Frigate, INS Talwar is presently deployed for Op-Sankalp commemorating the 3rd
continuous year of Indian Navy’s presence in the Gulf.
✓ OPERATION SATARK
Railway Protection Force (RPF) has recently started “Operation Satark” taking action against illicit
liquor, fake currency circulation, etc.
✓ OPERATION GANGA
India launches mission named Operation Ganga to evacuate Indian nationals from Ukraine
Q. Which IIT has launched 15 Centres of Excellence (CoE) as part of the 'Institute of Eminence (IoE)'
Scheme?
A) IIT Delhi B) IIT kharagpur
C) IIT Guwahati D) IIT Madras
Answer : D
✓ Indian Institute of Technology Madras (IIT Madras) launched 15 Centres of Excellence (CoE) as part of
the ‘Institute of Eminence’ (IoE) Scheme.
✓ The IoE Scheme was launched by the Government of India to empower Higher Educational Institutions
to become world-class teaching and research institutions.
✓ The financial support through IoE will enable IIT Madras faculty members to pursue ambitious research
goals, undertake cutting-edge projects and attract top talent to work on ground- breaking research topics.

Follow us: Official Site, Telegram, Facebook, Instagram, Instamojo 703


Q. In which state will Hindustan Petroleum Corporation Limited (HPCL) set up a new ethanol plant?
A) Himachal Pradesh B) Uttar Pradesh
C) Bihar D) Assam
Answer : A
✓ Himachal Pradesh Chief Minister Sukhwinder Singh Sukhu informed that Hindustan Petroleum
Corporation Limited (HPCL) will set up an ethanol plant at Jeetpur Baheri in Una district at a cost of Rs
500 crore.
✓ This ethanol plant will be set up on 30 acres.
✓ HPCL is a subsidiary of Oil and Natural Gas Corporation, functioning under the Ministry of Petroleum
and Natural Gas, Government of India.

Q. Who has been recently featured on the latest cover of TIME magazine?
A) Deepika Padukone B) Scarlett Johansson
C) Jennifer Lawrence D) Julia Roberts
Answer : A
✓ Bollywood star Deepika Padukone has been featured on the latest cover of TIME magazine.
✓ The iconic American magazine described Padukone as a ‘global star’ bringing ‘the world to Bollywood’.
✓ It had previously named Padukone among its list of 100 most influential people in the world in 2018.
✓ Deepika is called ‘the most popular actress in the world’s most populous country’.
Q. Who becomes the first city in India to adopt the localisation of the United Nations sustainable
development goals (SDG)?
A) Indore B) Bhopal
C) Surat D) Hyderabad
Answer : B
✓ Bhopal becomes the first city in India to adopt the localisation of the United Nations sustainable
development goals (SDG).
✓ India's Bhopal city has conducted India's first city-level Voluntary Local Review (VLR)of the Sustainable
Development Goals (SDGs).
✓ Bhopal has launched 'Agenda for Action: Sustainable Urban Transformation in Bhopal', which was
unveiled by Madhya Pradesh Chief MinisterShivraj Singh Chouhan.
✓ In 2015, all 193 UN member states adopted "Transforming our world: the 2030 Agenda for Sustainable
Development",which included 17 Sustainable Development Goals and 169 targets, as a plan of action
for people, planet and prosperity.
✓ Member countries report on their progress towards achieving the goals through Voluntary National
Reviews (VNRs) to the United Nations High-Level Political Forum (HLPF).

Q. Who has been conferred with the highest honors by Fiji and Papua New Guinea?
A) Amit Shah B) Yogi Adityanath
C) Narendra Modi D) Droupadi Murmu
Answer : C
✓ PM Modi conferred with highest civilian honours by Fiji, Papua New Guinea.
✓ PM Modi was honored with the island nation's highest civilian award during his visit to Papua New
Guinea to attend the 3rd India-Pacific Islands Cooperation (FIPIC) summit.

Follow us: Official Site, Telegram, Facebook, Instagram, Instamojo 704


✓ PM Modi was awarded "The Companion of the Order of Fiji" by his Fijian counterpart Sitweni Rabuka
for his global leadership, a rare honor for a non-Fijian.
✓ Papua New Guinea conferred its highest civilian honour, Grand Companion of the Order of Loghu
(GCL) on PM Modi for his efforts towards unity of the Pacific island nation.
✓ Republic of Fiji
✓ Capital: Suva
✓ Currency: Fijian Dollar
✓ Prime Minister: Sitiveni Rabuka
✓ Papua New Guinea
✓ Prime Minister: James Marape
✓ Capital: Port Moresby
✓ Currency: kina (K)
Q. India's First International Multimodal Logistics Park Coming at which state?
A) Odisha B) West Bengal
C) Assam D) Meghalaya
Answer : C
✓ Union Minister of Ports, Shipping & Waterways and Ayush, Sarbananda Sonowal reviewed progress of
India’s first International Multi Modal Logistics park in Jogighopa, Assam.
✓ About India’s First International Multimodal Logistics Park .
✓ The park is being developed under the ambitious Bharatmala Pariyojana.
✓ First such MMLP, it is being made by NHIDCL in Jogighopa of Assam.
✓ The park will be connected to road, rail, air and waterways.
✓ This is being developed in 317-acre land along the Brahmaputra.
Q. India's AI supercomputer 'AIRAWAT' has bagged which position in the prestigious Top 500 Global
Supercomputing list?
A) 70th B) 75th
C) 81st D) 86th
Answer : B
✓ The AI Supercomputer ‘AIRAWAT’, installed at C-DAC, Pune has been ranked 75th in the top 500
Global Supercomputing List.
✓ The announcement was made during the 61st edition of the International Supercomputing Conference
(ISC 2023) held in Germany.
✓ The establishment of 'AIRAWAT' at C-DAC Pune is part of the National Program on AI initiated by the
Government of India.
✓ The list has been topped by the United States (US) Frontier. This HPE Cray EX system is the first US
system with a performance exceeding one Exaflop/s (EXA Floating point Operations per Second).
✓ About supercomputer AIRAWAT
✓ AIRAWAT is the name for an in-house cloud platform for big data.
✓ The proposed AIRAWAT system is a 100+ petaflop AI-focused supercomputer.
✓ There are parallels drawn to Japan’s AI Bridging Cloud Interface supercomputer in the NITI Aayog
report.
✓ This system is a 130 petaflop computer focused on providing compute for AI, ML and big data tasks.
✓ What are supercomputers?
✓ A supercomputer can perform high-level processing at a faster rate than a normal computer.
✓ They work together to perform complex operations that are not possible with normal computing systems.
✓ Fast speed and fast memory are the characteristics of supercomputers.

Follow us: Official Site, Telegram, Facebook, Instagram, Instamojo 705


✓ National Supercomputing Mission
✓ The National Supercomputing Mission was launched In 2015.
✓ The aim of the mission was to enhance the research capacities in the country to form a Supercomputing
grid.
✓ It supports the government's vision of 'Digital India' and 'Make in India' initiatives.
✓ It is implemented by Center for Development of Advanced Computing (C-DAC), Pune and IISc,
Bengaluru.
✓ Some important facts about supercomputer
✓ China has the maximum number of supercomputers followed by the US, Japan, France, Germany,
Netherlands, Ireland and the United Kingdom..
✓ India’s first supercomputer - PARAM 8000
✓ first supercomputer assembled indigenously - PARAM Shivay, installed in IIT (BHU).
✓ PARAM Shakti, PARAM Brahma, PARAM Yukti, PARAM Sanganak are some name of india's
supercomputers.
✓ India's PARAM-Siddhi AI, has been ranked 63rd in the Top 500 list of most powerful supercomputers
in the world

Q. Who has been appointed as Confederation of Indian Industry (CII) President for 2023-2024?
A) R. Dinesh B) B. Govindarajan
C) Niranjan Gupta D) Puneet Chhatwal
Answer : A
✓ TVS Supply Chain Solutions Executive Vice President R. Dinesh was appointed as the President of the
Confederation of Indian Industry (CII) for 2023-24.
✓ Sanjiv Puri, Managing Director of ITC has been named the Chairman and the announcement was made
during the CII National Council meeting in New Delhi.
✓ Rajeev Memani, President, EY India Region, has been appointed as the Vice President of CII.
✓ Confederation of Indian Industry (CII) is a non-governmental trade association and advocacy group
headquartered in New Delhi, India, founded in 1895.
✓ President : R Dinesh
✓ Director General : Chandrajit Banerjee.
Q. Who has won the Best Male actor award in the International Indian Film Academy Awards (IIFA) 2023
held in Abu Dhabi?
A) Akshay Kumar B) Hrithik Roshan
C) Ranveer Singh D) Sidharth Malhotra
Answer : B
✓ Hrithik Roshan has won the award for Best Actor in a Leading Role at the International Indian Film
Academy Awards (IIFA) 2023 held in Abu Dhabi.
✓ Alia Bhatt also won the award for Best Actress in a Leading Role for her role in the movie Gangubai
Kathiawadi.
✓ Veteran actor Kamal Haasan was conferred the Outstanding Achievement in Indian Cinema award.
✓ Another veteran actor Anil Kapoor has presented the award for Performance In A Supporting Role
(male) for the movie Jug Jugg Jeeyo.

Q. Which film won the Best Film award at the IIFA Awards ceremony in 2023?
A) Shershaah B) The Kashmir Files
C) Adipurush D) Drishyam 2

Follow us: Official Site, Telegram, Facebook, Instagram, Instamojo 706


Answer : D
✓ Drishyam 2 is a 2022 Indian Hindi-language crime thriller film directed and co-produced by Abhishek
Pathak.
✓ List of all the award winners at IIFA 2023
✓ Best Film: Drishyam 2
✓ Best Director: R Madhavan for Rocketry: The Nambi Effect
✓ Best Actor in a Leading Role (Female): Alia Bhatt for Gangubai Kathiawadi
✓ Best Actor in a Leading Role (Male): Hrithik Roshan for Vikram Vedha
✓ Best Actor in a Supporting Role (Female): Mouni Roy for Brahmastra: Part One – Shiva
✓ Best Actor in a Supporting Role (Male): Anil Kapoor for Jugg Jugg Jeeyo.
✓ Outstanding Achievement for Fashion in Cinema: Manish Malhotra
✓ Outstanding Achievement in Indian Cinema: Kamal Haasan.
✓ Best Playback Singer (Female): Shreya Ghoshal for the song Rasiya from Brahmastra: Part One – Shiva
✓ Best Playback Singer (Male): Arijit Singh for the song Kesariya from Brahmastra: Part One – Shiva.
Q. Which team won the IPL 2023 title?
A) Gujarat Titans B) Chennai Super Kings
C) Mumbai Indians D) Rajasthan Royals
Answer : B
✓ Chennai Super Kings (CSK) has defeated defending champions Gujarat Titans by 5 wickets to win its
5th IPL title.
✓ The final match of IPL 2023 was held at Narendra Modi Stadium in Ahmedabad, Gujarat.
✓ Chennai Super Kings has equaled Mumbai Indians’ record for most IPL titles.
✓ Chennai Super Kings won the toss and invited Gujarat Titans to bat first, scoring 214 runs in 20 overs.
✓ After this, Chennai got the target of 171 runs in 15 overs due to Duckworth-Lewis rule due to rain.
✓ This was the 16th season of IPL
Q. Which player won the 'Emerging Player of the Season' award in IPL 2023?
A) Shubman Gill B) Yashasvi Jaiswal
C) Rinku Singh D) Akash Madhwal
Answer : B
✓ In this season of IPL 2023, Rajasthan Royals' young opener Yashasvi Jaiswal was awarded the 'Emerging
Player of the Season' award. Yashasvi batted brilliantly this season, scoring 625 runs at an average of
48.08 and a strike rate of 163.61.
✓ Jaiswal also scored the fastest half-century in IPL history, crossing the 50-run mark in just 13 balls.

Q. Which player won the 'Orange Cap' by scoring the most runs in IPL 2023?
A) Yashasvi Jaiswal B) David Warner
C) Virat Kohli D) Shubman Gill
Answer : D
✓ In IPL 2023, Gujarat Titans (GT) opener Shubman Gill won the 'Orange Cap' by scoring the most runs.
✓ Gill won this award by scoring 890 runs this season.
✓ This season, Gill scored maximum 890 runs in 17 matches at an average of 59.33. Gill also scored three
centuries in IPL 2023.
✓ 2023 Indian Premier League (also known as Tata IPL 2023)
✓ Title Sponsor : TATA (₹440 crores)

Follow us: Official Site, Telegram, Facebook, Instagram, Instamojo 707


✓ Season : 16th ( 2023)
✓ Opening Match : Narendra Modi Stadium, Ahmedabad ( CSK Vs GT)
✓ Final Match : Narendra Modi Stadium, Ahmedabad ( CSK Vs GT)
✓ Total teams : 10
✓ Champions : Chennai Super Kings (5th title) [ 2010, 2011, 2018, 2021, 2023]
✓ Runners-up : Gujarat Titans (1st title) [ 2022]
✓ Orange Cap award (most runs in a season) – Shubman Gill (Gujrat Titans) – 890 runs
✓ Purple Cap (highest wicket-taker)– Mohammad Shami (Gujrat Titans) – 28 wickets
✓ Most Valuable Player of the Season award – Shubman Gill (Gujrat Titans)
✓ Dream11 Gamechanger of the Season award – Shubman Gill (Gujrat Titans)
✓ Emerging Player of the Year award – Yashasvi Jaiswal (Rajasthan Royals).
✓ Sam Curran became the most expensive IPL player during the 2023 auction. He was signed by Punjab
Kings for ₹18.50 crore .
✓ Cameron Green was signed by Mumbai for Rs 17.50 crore at the IPL 2023 auction, becoming the second
most expensive player in the IPL.
✓ Chennai Super Kings (CSK).
✓ Captain : MS Dhoni
✓ Coach : Stephen Fleming
✓ Owner : N. Srinivasan (Indian Cements Limited)
✓ CSK became India's first unicorn sports enterprise.
✓ Super Kings have won the IPL title five times (in 2010, 2011, 2018, 2021, and 2023).
✓ Gujarat Titans (GT) .
✓ Captain : Hardik Pandya
✓ Coach : Ashish Nehra
✓ Owner : CVC Capital Partners
✓ The winning team will get Rs 20 crore, the runner-up will get Rs 13 crore
Q. Which bowler won the 'Purple Cap' by taking the most wickets in IPL 2023?
A) Rashid Khan B) Mohammed Shami
C) Hardik Pandya D) Mohit Sharma
Answer : B
✓ Purple Cap is given to the player who takes the most wickets in any season of IPL.
✓ In IPL 2023, Gujarat Titans (GT) fast bowler Mohammed Shami won the Purple Cap by taking 28
wickets.
✓ The Fairplay of the Season award was given to the Delhi Capitals team.
✓ This time the Most Valuable Player of the Season (MVP) award was given to Shubman Gill of Gujarat
Titans
Q. Who has been honoured with the 57th Jnanpith Award?
A) Nilmani Phookan B) Damodar Mauzo
C) Amitav Ghosh D) Krishna Sobti
Answer : B
✓ Goan writer Damodar Mauzo has been honoured with the 57th Jnanpith Award, India's highest literary
honour.
✓ He is a renowned short story writer, novelist, critic and screenwriter in Konkani language.
✓ Damodar Maujo is the second Goan to receive the Jnanpith Award after Ravindra Kelekar in 2008.
✓ He has written 25 books in Konkani and one in English, many of which have been translated into various
languages.

Follow us: Official Site, Telegram, Facebook, Instagram, Instamojo 708


✓ Mauzo’s famous novel ‘Karmelin’ received the Sahitya Akademi Award in 1983.
✓ The award ceremony took place at the Raj Bhavan near Panaji, the capital of Goa
✓ It is the oldest and highest Indian literary award presented annually by Bharatiya Jnanpith.
✓ JNANPITH AWARD
✓ Field : Literature
✓ Instituted : 1961
✓ First awarded : 1965
✓ First recipient : G. Sankara Kurup
✓ First women to receive : Ashapoorna Devi
✓ First Jnanpith in English : Amitav Ghosh (54th Jnanpith)
✓ First Jnanpith in Hindi : Kunwar Narayan in 2005
✓ Akkitham : Jnanpith Award (55th)
✓ Language : 22 languages mentioned in the Eighth
✓ Total awarded : 62
✓ First Jnanpith Awardee – G.Sankara Kurup (1965) Malayalam language
✓ First woman Jnanpith Awardee – Ashapurna Devi (1976) Bangla language
✓ First English author to receive Jnanpith Award: Amitav Ghosh (2018)
✓ First Hindi author to receive Jnanpith Award : Sumitranandan Pant (1968)
Q. India has operationalized the Sittwe Port with the departure of the inaugural shipment, the MV-ITT
LION (V-273), from Syall Prasad Mookerjee Port, Kolkata. Sittwe Port is located in which country?
A) Bangladesh B) Afghanistan
C) Maldives D) Myanmar
Answer : D
✓ India has operationalized the Sittwe Port in Myanmar with the departure of the inaugural shipment, the
MV-ITT LION (V-273), from Syall Prasad Mookerjee Port, Kolkata.
✓ Ports, Shipping and Waterways Minister Sarbananda Sonowal will receive the first Indian cargo ship at
Sittwe Port in Myanmar.
✓ Sittwe Port is a part of the Kaladan Multimodal Transit Transport Project and its development will enable
multi-modal transit connectivity with Southeast Asia.
✓ The port has been built under a framework agreement between India and Myanmar to set up a
multimodal transit transport facility on the Kaladan River, which connects Sittwe Port in Myanmar to
Mizoram in India.
✓ The Kaladan River route has the potential to open up economic opportunities for the entire Bay of Bengal
peninsula.
Q. The Indian Space Research Organization (ISRO) has announced a new introductory-level online training
programme called START. What is the full form of START?
A) Space Science and Technology Awareness Training
B) Scholarship of Science and Technology Awareness Training
C) Space Science and Technology Astrophysics Training
D) Space Science and Technology Academic Training
Answer : A
✓ Indian Space Research Organization (ISRO) has announced a new introductory-level online training
programme called Space Science and Technology Awareness Training (START).
✓ START is aimed at postgraduate and final-year undergraduate students of Physical Sciences and
Technology.

Follow us: Official Site, Telegram, Facebook, Instagram, Instamojo 709


✓ The programme will cover various domains of Space Science, including Astronomy and Astrophysics,
Heliophysics and Sun-Earth interaction, Instrumentation, and Aeronomy.
ISRO IN NEWS 2023
▪ ISRO has announced a new online training programme for post-graduate and final-year undergraduate
students of physical sciences and technology. The programme is called Space Science and Technology
Awareness Training (START) .
▪ ISRO will launch Singapore's TeLEOS-2 satellite using the Polar Satellite Launch Vehicle (PSLV) from
the Satish Dhawan Space Center in Sriharikota.
▪ ISRO has successfully conducted the Reusable Launch Vehicle Autonomous Landing Mission (RLV
LEX) at the Aeronautical Test Range (ATR), Chitradurga, Karnataka.
▪ ISRO has successfully launched India’s largest Launch Vehicle Mark-III (LVM3) rocket/OneWeb India-
2 Mission with 36 satellites onboard from Satish Dhawan Space Centre in Sriharikota, Andhra Pradesh.
▪ ISRO successfully launched a Polar Satellite Launch Vehicle -C52(PSLV-C52) on 14 February 2022 from
the Satish Dhawan Space Centre at Sriharikota in Andhra Pradesh.
▪ The PSLV-C52 rocket carried three satellites EOS-04 ,INSPIR Esat-1and INS-2TD).
▪ Indian Space Research Organisation (ISRO) will launch OneWeb India-2 mission from Satish Dhawan
Space Centre in Sriharikota.
▪ Under a commercial agreement with NewSpace India Ltd, will launch 72 satellites of UK-based Network
Access Associates Ltd and place them in low Earth orbit.
▪ Indian Space Research Organisation (ISRO) recently successfully carried out a controlled descent of its
12-year-old low earth orbit satellite Megha Tropiques-1 (MT-1) into the Pacific Ocean.
▪ Indian Space Research Organisation (ISRO) has recently released images of Earth captured by the EOS-
06 satellite.
▪ Earth Observation satellite (EOS-6) is the third-generation satellite in the Oceansat series.
▪ Indian Space Research Organization has launched its smallest rocket SSLV-D2 into space from Satish
Dhawan Space Center (Sriharikota).
▪ ISRO's SSLV-D2 flew into space carrying three satellites with it.
▪ These include ISRO's satellite EOS-07, Chennai-based space startup Spacekidz's satellite Azad SAT-2
and American company Antaris's satellite JANUS -1.
▪ All these three satellites will be placed in circular orbit 450 km apart.
▪ An Earth-observation satellite jointly developed by National Aeronautics and Space Administration
(NASA) and ISRO will be sent to India in late February 2023 for a possible launch in September.
▪ India's first ever private Vikram-S (Suborbital) rocket was launched on 18 November from the Satish
Dhawan Space Centre, Sriharikota.
▪ Vikram-S, designed by Skyroot Aerospace.
▪ ISRO has successfully launched its heaviest rocket LVM3-M2, from the Satish Dhawan Space Centre in
Sriharikota spaceport, Andhra Pradesh on 23 October 2022. The mission, which was named LVM3-
M2/OneWeb India-1, carried 36 broadband communication satellites of OneWeb Company.
▪ ISRO will launch 36 satellites of OneWeb, the UK's global communications network, on October 23,
2022.
▪ The name of the rocket is LVM3, it was earlier called GSLV Mk III.
▪ ISRO ‘Shukrayaan I’ mission to planet Venus reportedly shifted to 2031.
▪ ISRO is doing to boost the use of ‘NAVigation with the Indian Constellation’ (NavIC), India’s version
of GPS.
▪ ISRO has unveiled spectacular images of the Earth taken by its Earth Observation Satellite (EOS-06),
also known as Oceansat-3.
▪ CE-20 cryogenic engine, which will power the nation’s rocket for the third moon mission, Chandrayaan-
3, had a flight acceptance hot test that was successfully completed, according to the ISRO.
▪ Indian Space Research Organisation (ISRO)
▪ Founded : 15 August 1969
▪ HQ : Bengalore, Karnataka

Follow us: Official Site, Telegram, Facebook, Instagram, Instamojo 710


▪ Founder / 1st Chairman : Vikram Sarabhai
▪ 10th Chairman : S Somanath
▪ Aryabhata India's first satellites Launched on 19 April 1975.
▪ GSAT-1 India's first commercial satellite.
▪ SLV-3 was India's first experimental satellite launch vehicle.
▪ Ariane Passenger PayLoad Experiment, (APPLE) first communication satellite in India 19 June 1981.
▪ Udupi Ramachandra Rao : satellite Man of India

Q. Which is the first airport in India to have a 'Reading Lounge'?


A) Rajiv Gandhi International Airport
B) Lal Bahadur Shastri International (LBSI) Airport
C) Chennai International Airport
D) Chhatrapati Shivaji Maharaj International (CSMI) Airport
Answer : B
✓ The country's first reading lounge has been inaugurated at the Lal Bahadur Shastri International Airport
in Varanasi, the parliamentary constituency of PM Modi.

Q. Which player has been honored with the 2023 “Laureus Sportsman of the Year” Award?
A) Cristiano Ronaldo B) Lionel Messi
C) Lewis Hamilton D) Rafael Nadal
Answer : B
✓ Argentina footballer, Lionel Messi has been named the Sportsman of the Year at the 2023 Laureus World
Sports Awards.
✓ Messi also became the first athlete to scoop both the World Sportsman of the Year Award and the World
Team of the Year Award in the same year.
✓ With the win, the Argentine becomes the only footballer ever to win Laureus Awards twice. Messi
previously won the Sportsman of the Year award in 2020 in a ceremony that took place in Berlin.
✓ He was awarded for his remarkable achievements on the pitch in the past year, which included leading
Argentina to victory in the FIFA World Cup.
✓ Sportswoman of the Year: Shelly-Ann Fraser-Pryce (Jamaica) Athletics.
✓ World Breakthrough of the Year Award – Carlos Alcaraz (Spain)
✓ World Comeback of the Year Award – Christian Eriksen
✓ World Sportsperson of the Year with a Disability Award – Catherine Debrunner
✓ World Action Sportsperson of the Year Award – Eileen Gu
✓ Laureus Sport for Good Award -TeamUp
✓ World Sportsman of the Year Award – Lionel Messi
✓ World Sportswoman of the Year Award – Shelly-Ann Fraser-Pryce
✓ World Team of the Year Award – Argentina Men’s Football Team
Q. Name the female player who has been felicitated with the “Sportswoman of the Year” at the 2023
Laureus World Sports Awards
A) Simona Halep B) Anna Van Hooft
C) Ashleigh Barty D) Shelly-Ann Fraser-Pryce
Answer : D
✓ Sportswoman of the Year: Shelly-Ann Fraser-Pryce (Jamaica) Athletics

Follow us: Official Site, Telegram, Facebook, Instagram, Instamojo 711


Q. Who among the following has been awarded the World Breakthrough of the Year Award at the 2023
Laureus World Sports Awards?
A) Novak Djokovic B) Kylian Mbappe
C) Carlos Alcaraz D) Eliud Kipchoge
Answer : C
✓ World Breakthrough of the Year Award – Carlos Alcaraz.
✓ Carlos Alcaraz Garfia is a Spanish professional tennis player.
✓ Swiss tennis player Roger Federer holds the record for the most awards with six, five for Sportsman of
the Year and one for Comeback of the Year.
✓ Serena Williams holds the record for most awards held by a female with five, four for Sportswoman of
the Year and one for Comeback of the Year.

Q. Who was appointed as the new CBI director for a period of two years?
A) Ravi Gupta B) Sudhir Saxena
C) Praveen Sood D) Rajendranath Reddy
Answer : C
✓ Praveen Sood appointed as new CBI director.
✓ Praveen Sood, a 1986 batch Indian Police Service (IPS) officer of the Karnataka cadre, has been
appointed as the next director of the Central Bureau of Investigation (CBI).
✓ Praveen Sood will replace current CBI Director Subodh Jaiswal when his term ends on May 25.
✓ The Department of Personnel and Training approved the appointment of Sood as CBI Director for a
period of two years.
✓ Central Bureau of Investigation (CBI):
✓ CBI is the premier investigative agency of India.
✓ It was established in 1941 as Special Police Establishment and later renamed as Central Bureau of
Investigation in 1963.
✓ Its primary objective is to maintain the integrity and credibility of administration, ensure justice and
uphold the rule of law.
✓ It is responsible for the investigation of a wide range of cases including corruption, economic offences,
fraud, organised crime and other serious crimes of national and international importance.

Q. Who has become India’s 82nd Grandmaster?


A) Sayantan Das B) Vuppala Prraneeth
C) Pranesh M D) Rahul Srivastava
Answer : B
✓ Telangana’s Vuppala Prraneeth became India’s 82nd Grandmaster.
✓ A 15-year-old chess player from Telangana, V. Prraneeth has
✓ Achieved the title of Grandmaster, becoming the sixth from the state and the 82nd in India.
✓ He secured this milestone by defeating GM Hans Niemann from the US during the penultimate round
of the Baku Open 2023.

Q. Which country is hosting the G-7 Summit 2023?


A) Germany B) Italy
C) Canada D) Japan

Follow us: Official Site, Telegram, Facebook, Instagram, Instamojo 712


Answer : D
✓ The 49th G-7 summit will be held in Hiroshima, Japan from 19 to 21 May 2023.
✓ Japan has also invited eight countries including India as guest countries in this conference.
✓ Apart from India, Japanese Prime Minister Fumio Kishida has invited South Korea, Australia, Brazil,
Vietnam, Indonesia, Comoros and Cook Islands.
✓ PM Modi will participate in this summit from the Indian side.
✓ About the G7
✓ The G7 is a group of seven major industrialised countries, includingCanada, France, Germany, Italy,
Japan, the United Kingdom, and the United States.
✓ The G7 holds annual meetings to discuss various global issues, but these meetings are usually held in
different locations each year.
✓ SURPRISE FACTS G7 SUMMIT
1st - 1975 : France
46th - 2020 : United States
47th - 2021 : United Kingdom
48th - 2022 : Germany
49th - 2023 : Japan
50th - 2024 : Italy
✓ Trick to remember G7 Countries
✓ Trick - Drink JUICE with GF (girlfriend)
JUICE case :-
J for Japan
U for USA
I for Italy
C for Canada
E for England
✓ GF case :-
G for Germany
F for France

Q. Who has recently been conferred with the highest civilian honor of France?
A) Draupadi Murmu B) N. Chandrasekaran
C) Narendra Modi D) Sachin Tendulkar
Answer : B
✓ Tata Group Chairman N. Chandrasekaran has recently been awarded France's highest civilian honor
'Knight of the Legion of Honour'.
✓ French Foreign Minister Catherine Colonna gave him this honor.
✓ Chandrasekaran has been given this award for strengthening business relations between India and
France.

Q. Who is the author of the book titled "The Golden Years: The Many Joys of Living a Good Long Life"?
A) Chetan Bhagat B) Shashi Tharoor
C) Arvind Adiga D) Ruskin Bond
Answer : D
✓ Indian author Ruskin Bond wrote a book titled "The Golden Years: The Many Joys of Living a Good
Long Life".

Follow us: Official Site, Telegram, Facebook, Instagram, Instamojo 713


✓ The Golden Years book is published by HarperCollins India and released on 19 May 2023, Bond's 89th
birthday.
✓ "The Golden Years" focuses on Bond's experiences during the 60s, 70s and 80s.

Q. Which state has overtaken Maharashtra in sugar production in the current season?
A) Karnataka B) Andhra Pradesh
C) Uttar Pradesh D) Gujarat
Answer : C
✓ Uttar Pradesh surpasses Maharashtra in sugar production this season.
✓ Uttar Pradesh has overtaken Maharashtra in sugar production in the current season, with 118 sugar mills
operating in UP during the period as compared to 210 mills in the western state.
✓ The total sugar produced by Uttar Pradesh in the sugar season 2022-2023 is 107.29 lakh tonnes, while
the total sugar produced by Maharashtra is 105.30 lakh tonnes.
✓ The area under sugarcane cultivation in Uttar Pradesh is 28.53 lakh hectares (highest in India) as
compared to 14.87 lakh hectares in Maharashtra.
✓ The sugarcane crushing season runs from October to June.
✓ Total cane crushed by sugar mills in UP in the 2022-2023 season was 1,084.57 lakh tonnes, whereas it
was 1,053 lakh tonnes in Maharashtra.
✓ In Uttar Pradesh, 19.84 lakh tonnes of sugar was converted into ethanol, while in Maharashtra it was
15.70 lakh tonnes.
✓ The number of sugar mills established in UP is 157 as compared to 246 sugar mills established in
Maharashtra.
✓ There are 210 sugar mills operating in Maharashtra as compared to 118 in Uttar Pradesh.
✓ India has emerged as the world's largest producer and consumer of sugar.
✓ India has also become the second largest sugar exporter in the world after Brazil.
Q. Who has announced a new initiative titled ‘NEP SAARTHI to involve students in the implementation
of the National Education Policy (NEP) 2020?
A) UGC B) NCERT
C) CBSE D) UPSC
Answer : A
✓ UGC Launches ‘NEP SAARTHI’ to Involve Students in Implementing NEP 2020.
✓ University Grants Commission (UGC) has announced a new initiative titled ‘NEP SAARTHI (Student
Ambassador for Academic Reforms in Transforming Higher Education in India)’ to involve students in
the implementation of the National Education Policy (NEP) 2020.
✓ Objective: To create an environment where students could engage and participate to make effective use
of the provisions of NEP 2020.
UGC IN NEWS 2023.
▪ UGC Launches ‘NEP SAARTHI’ to Involve Students in Implementing NEP 2020.
▪ University Grants Commission (UGC) has launched two UGC websites, UTSAH (Undertaking
Transformative Strategies and Actions in Higher Education) and POP (Professor of Practice) portals.
▪ University Grants Commission (UGC) has launched a new portal named ‘CU-Chayan’ for faculty
appointments in central varsities..
▪ Prof. M. Jagadesh Kumar appointed as the Chairman of UGC.

▪ About University Grants Commission (UGC)

Follow us: Official Site, Telegram, Facebook, Instagram, Instamojo 714


▪ It came into existence in 1953.
▪ It became a statutory Organization of the Government of India by an Act of Parliament in 1956.
▪ Its functions are coordination, determination and maintenance of standards of teaching, examination
and research in university education

Q. India will host the Quad Leaders’ Summit in which year?


A) 2024 B) 2025
C) 2026 D) 2027
Answer : A
✓ India will host the Quad Leaders’ Summit in 2024.
✓ The Quadrilateral Security Dialogue (QSD), commonly known as the Quad, is a strategic security
dialogue between Australia, India, Japan, and the United States of America.
✓ It aims to help countries mitigate the effects of climate change and strengthen their climate resilience.
✓ Australia will host thethird in-person Quad summit in Sydney.
✓ The Quad summit is a meeting between the leaders of four countries: the United States, Japan, India,
and Australia.
✓ The Quad was first established as a strategic dialogue in 2007, but remained dormant for several years
until it was revived in 2017.
✓ The Quad Summit, to be held in Washington DC in September 2021, was the first in-person meeting of
the Quad leaders.
✓ The agenda of the Quad summit focused on four key areas: COVID-19 vaccine delivery, climate change,
critical and emerging technologies and regional security.
Q. Who has becomes world No. 1 in World Athletics men's javelin throw ranking?
A) Anderson Peters B) Neeraj Chopra
C) Jakub Vadlejch D) Julian Weber
Answer : B
✓ Neeraj Chopra makes history, becomes World No 1 in men's javelin throw.
✓ Tokyo Olympics gold medalist Neeraj Chopra on May 22 achieved the number one ranking in men's
javelin throw for the first time. This is a major milestone in his career.
✓ Neeraj Chopra is on top with 1455 points, 22 ahead of Anderson Peters of Granada.
✓ The Indian national record holder in men's javelin throw, took part in the season-opening Doha Diamond
League on May 5 and finished first with a throw of 88.67m.
✓ Anderson Peters finished third in Doha with a throw of 85.88m.
✓ Top 5 in Men's javelin throw
1. Neeraj Chopra (India) -1455 points
2. Anderson Peters (Granada) - 1433 points
3. Jakub Vadlejch (Czech Republic)- 1416 points
4. Julian Weber (Germany)- 1385 points
5. Arshad Nadeem (Pakistan) - 1306 points
✓ Switzerland Tourism has appointed Olympic Gold medalist Neeraj Chopra as the ‘Friendship
Ambassador’.
✓ Neeraj Chopra's Olympic gold medal-winning throw of 87.58m at Tokyo 2020 .
✓ The men's javelin must weigh at least 800g and be 2.6m-2.7m long while the women's javelin must weigh
600g and be 2.2m-2.3m long.
✓ Neeraj Chopra presently holds the rank of Subedar in the Indian Army.
✓ Neeraj Chopra is from Khandra village, Panipat district, Haryana.

Follow us: Official Site, Telegram, Facebook, Instagram, Instamojo 715


Q. A naval exercise Al Mohammad Al Hindi 2023 will be held between India and which country?
A) Qatar B) Iran
C) Saudi Arabia D) UAE
Answer : C
✓ AL-MOHED AL-HINDI 2023 Naval Exercise.
✓ Indian Navy ships INS TARKASH and INS SUBHADRA arrived at Port Al-Jubail in Saudi Arabia.
✓ The visit of these ships marks the beginning of the harbor phase of the second edition of the naval exercise
between the two countries, known as 'Al-Mohad Al-Hindi 2023'.
Q. Georgi Gospodinov has won International Booker Prize 2023 for ‘Time Shelter’. She is from which
country?
A) Norway B) Sweden
C) Bulgaria D) Ethiopia
Answer : C
✓ Bulgarian author Georgi Gospodinov won the 2023 International Booker Prize for his novel "Time
Shelter".
✓ The novel was translated into English by Angela Rodel and is the first Bulgarian novel to win the
International Booker Prize.
✓ "Time Shelter" is a captivating and remarkable literary work that explores themes of time, identity, and
the human experience.
✓ Georgi Gospodinov was born in Yambol, Bulgaria.
✓ Angela Rodel originally from Minnesota, the United States of America (USA) is a literary translator.
✓ The award carries a cash prize of 50,000 pounds, divided equally between the author and the translator.
In addition, the shortlisted authors and translators each receive £2,500.
✓ DON'T CONFUSED
✓ The International Booker Prize is an international literary prize awarded every year for a single book that
is translated into English and published in the UK or Ireland.
✓ Booker Prize for Fiction is a literary prize awarded each year for the best original novel written in the
English language and published in the United Kingdom
✓ Don't Confused First's Indian Booker Prize and International Booker Prize.
✓ Geetanjali Shree has become the first Indian author to win the prestigious International Booker Prize for
her novel 'Tomb of Sand'
✓ Arundhati Roy was the first Indian to win the Prestigious Booker prize in 1997 for her first novel The
God of Small Things
✓ DON'T CONFUSED
✓ Bulgarian author Georgi Gospodinov won the 2023 International Booker Prize for his novel "Time
Shelter".
✓ Shehan Karunatilaka, a Sri Lankan author wins the Booker prize 2022 for “The Seven Moons of Maali
Almeida”.

Q. Which of the following books was awarded the International Booker Prize 2023?
A) Klara and the sun B) Somebody's Daughter: A Memoir
C) Cloud Cuckoo Land D) Time Shelter
Answer : D
✓ Bulgarian author Georgi Gospodinov won the 2023 International Booker Prize for his novel "Time
Shelter".

Follow us: Official Site, Telegram, Facebook, Instagram, Instamojo 716


Q. Great prayer festival is celebrated in which state/UT?
A) Jammu and Kashmir B) Ladakh
C) Kerala D) Madhya Pradesh
Answer : B
✓ Ladakh’s Annual Great Prayer Festival ‘Ladakh Monlam Chenmo 2023’ held from May 11 to 16 2023.
✓ Ladak’s annual five day group prayer camp of Buddhist monks and nuns named ‘Ladakh Monlam
Chenmo 2023’ which is also called as Great Prayer Festival was held from May 11 to 16, 2023 in Leh
(Ladakh) for world peace and happiness
Q. Who is the author of the book "Made in India: 75 Years of Business and Enterprise" ?
A) Rajiv Dixit B) Amitabh Kant
C) Aravind Adiga D) B. V. R. Subrahmanyam
Answer : B
✓ The book "Made in India: 75 Years of Business and Enterprise" is written by Amitabh Kant and published
by Rupa Publications India.
✓ The book coincides with India becoming the 5th largest economy in the world, overtaking the United
Kingdom after 75 years of independence.
✓ This book presents a multidimensional survey of India's business heritage and culture to understand the
country's remarkably strong and resilient growth story of Indian business and enterprise.
✓ Through the book, Amitabh Kant aims to provide insight into the success of Indian businesses and
enterprises and how they can move forward in the future.
✓ Amitabh Kant is a retired Indian Administrative Service (IAS) officer who served as the Chief Executive
Officer (CEO) of the National Institution for Transforming India (NITI Aayog) from 2016 to 2022.
✓ He has also written other books like "Branding India-An Incredible Story," "Incredible India 2.0," and
edited "The Path Ahead - Transformative Ideas for India."
✓ India’s G-20 Sherpa Amitabh Kant.

Q. What is the rank of India in the 2023 World Press Freedom Index?
A) 172 B) 150
C) 161 D) 143
Answer : C
✓ India ranks 161 in 2023 World Press Freedom Index; Norway tops.
✓ India has slipped by 11 positions, to be ranked at 161st position among 180 countries in the 2023 World
Press Freedom Index.
✓ In 2022, India’s rank was at 150.
✓ The index is released annually by the global media watchdog Reporters Without Borders (RSF), to
evaluate the press freedom situation in 180 countries and territories.
✓ Norway has achieved the first position for the seventh consecutive year on the 2023 World Press Freedom
Index.
✓ It is followed by Ireland, Denmark, Sweden, Finland, Netherlands, Lithuania, Estonia, Portugal and
Timor-Leste among top 10 countries.
✓ The last three position has been occupied by Vietnam (178), China (179) and North Korea (180).
✓ India’s performance in the World Press Freedom Index is also poor compared to other countries in South
Asia.
✓ India is ranked 161 out of 180 countries. Press freedom in India has gone from “problematic” to “very
bad”, with the country slipping 11 ranks since the 2022 report.

Follow us: Official Site, Telegram, Facebook, Instagram, Instamojo 717


✓ Neighbours have a better rank than India -- Bhutan (90), Nepal (95), Sri Lanka (135), Pakistan (150),
Afghanistan (152).
✓ The ranking is based on five categories -- political, legislative, economic, sociocultural and safety of
journalists.
✓ Key Facts about Reporters Without Borders (RSF):
✓ It is an international NGO whose self-proclaimed aim is to defend and promote media freedom.
✓ Headquarters: Paris, France.
✓ It has consultative status with the United Nations.
✓ RSF defines press freedom as “the ability of journalists as individuals and collectives to select, produce,
and disseminate news in the public interest independent of political, economic, legal and social
interference and in the absence of threats to their physical and mental safety.”

Q. Which country has topped the 2023 World Press Freedom Index?
A) Netherlands B) Singapore
C) Norway D) Finland
Answer : C
✓ Norway has achieved the first position for the seventh consecutive year on the 2023 World Press Freedom
Index.
✓ It is followed by Ireland, Denmark, Sweden, Finland, Netherlands, Lithuania, Estonia, Portugal and
Timor-Leste among top 10 countries.
Q. The World Press Freedom Index is published by which institution?
A) World Economic Forum B) International Monetary Fund
C) Reporters Without Borders D) U.S. Chamber’s Global Intellectual Property Center
Answer : C
Q. Who has been awarded the prestigious Leipzig Book Prize for European Understanding in 2023?
A) Toni Morrison B) Maria Stepanova
C) Margaret Atwood D) Toni Morrison
Answer : B
✓ Maria Stepanova, a powerful contemporary voice of Russian literature now living in exile in Berlin, has
been awarded the prestigious Leipzig Book Prize for European Understanding in 2023.
✓ She was honored in Leipzig for her volume of poetry “Madchen ohne Kleider” (Girls Without Clothes).
✓ It is described as a sensitive and highly poetic study of often hidden violence against the female body and
the power imbalance that drives this oppression.
✓ The prize has been granted since 1994 with the aim of promoting reconciliation in Europe, and it was
previously won by another Russian exile, Masha Gessen, a journalist residing in the United States, for
her book The Future Is History: How Totalitarianism Reclaimed Russia in 2019.
✓ Prize money: 20,000 euros

Q. Who has become the first Indian man to summit Mt. Annapurna 1?
A) Pranav Rawat B) Kunal Joisher
C) Arjun Vajpai D) Prerna Dangi
Answer : C

Follow us: Official Site, Telegram, Facebook, Instagram, Instamojo 718


✓ Indian professional mountaineer and 2023 Fit India Champion, Arjun Vajpai, has become the first Indian
man to summit Mt. Annapurna 1.
✓ It is the 10th highest mountain in the world, located in Nepal, at 8,091 metres (26,545 ft) above sea level.
✓ Arjun is also the first Indian mountaineer to scale 7x8000m peaks, means seven mountain peaks which
are above 8000 metres in height. He now wants to become the first Indian to scale all the 14 8000m
summits in the world.
✓ He has already scaled Mt. Everest, Mt. Lhotse, Mt. Makalu, Mt. Kanchenjunga, Mt. Manaslu, and Cho-
Oyu. He even holds record of being the then youngest mountaineers to climb Mount Everest in 2010 and
Mt. Lhotse in May 2011
✓ ‘Ironman’ Krishna Prakash becomes first person to swim from Gateway of India to Elephanta Caves.

Q. Which state is set to declare itself India’s first “total e-governed state”?
A) Telangana B) Kerala
C) Karnataka D) Tamil Nadu
Answer : B
✓ Kerala is set to declare itself India’s first “total e-governed state”.
✓ The state has achieved this feat through policy initiatives aimed at transforming it into a digitally
empowered society.
✓ The government focused on setting up a knowledge-based economy and achieving 100% digital literacy.
✓ Important services across various sectors were digitized to ensure transparency, inclusiveness and
accessibility for all citizens.
Q. The Indian Railways handed over 20 Broad Gauge locomotives to which country as part of a grant
assistance program?
A) Bangladesh B) Sri Lanka
C) Maldives D) Nepal
Answer : A
✓ India hands over 20 broad gauge locomotives to Bangladesh
✓ Taking a significant step towards enhancing bilateral cooperation in the railway sector, India and
Bangladesh handed over 20 broad gauge locomotives to Bangladesh.
✓ The initiative is aimed at meeting the growing demand for passenger and freight traffic between the two
neighboring countries.
Q. Rayyanah Barnawi has becomes the first Arab woman astronaut to go to space. She is from which
country?
A) UAE B) Saudi Arabia
C) Kuwait D) Qatar
Answer : B
✓ Part of Axiom Space’s private mission to the International Space Station (ISS), Rayyanah Barnawi
became the first Arab woman astronaut to go into space.
✓ Barwani is part of a team which has travelled to the ISS in a SpaceX Falcon 9 rocket from the Kennedy
Space Center at Cape Canaveral in the southern state of Florida.
✓ The group — also including Peggy Whitson, a former NASA astronaut who will be making her fourth
flight to the ISS, and John Shoffner, a businessman from Tennessee — will stay aboard the space station
for around eight days to carry out a variety of experiments.
✓ What is Axiom Space’s private mission?

Follow us: Official Site, Telegram, Facebook, Instagram, Instamojo 719


✓ Called Axiom Mission 2, or Ax-2, this is the second private mission by Axiom Space, an American
privately funded space infrastructure manufacturing company that was co-founded by Michael
Suffredini, who served as NASA’s International Space Station Program Manager from 2005 to 2015

Q. Who has been appointed as the Tennis Ambassador for Sony Sports Network?
A) PV Sindhu B) Saina Nehwal
C) Sania Mirza D) Ankita Raina
Answer : C
✓ Sania Mirza appointed as the Tennis Ambassador for Sony Sports Network.
✓ Leading sports broadcaster Sony Sports Network has announced the exclusive onboarding Sania Mirza
as the Tennis Ambassador for Sony Sports Network.
✓ Sony Sports Network has also launched a campaign called Home of Tennis.
✓ The second film is Mirza presenting Roland Garros, Clay that Slays, to Indian audiences.
✓ International Tennis Career of Sania Mirza
✓ Sania has won the mixed doubles title of Australian Open in 2009, French Open in 2012, US Open in
2014.
✓ Apart from this, she has won the 2016 Australian Open, 2015 Wilmington, 2015 US Open title in
women's doubles.
✓ Sania Mirza has won many titles and honours in her career. She has been awarded the Arjuna Award in
2004, Padma Shri in 2006, Rajiv Gandhi Khel Ratna in 2015 and Padma Bhushan in 2016.
Q. Under which initiative of the government of India the Vande Bharat Express was designed and
manufactured by integral coach factory at perambur chennai?
A) Mitra Scheme B) NISHTHA
C) Make in India D) Niryat Rin Vikas
Answer : C
✓ Prime Minister Narendra Modi has virtually inaugurated the Delhi (Anand Vihar)-Dehradun Vande
Bharat Express.
✓ This is the first Vande Bharat to be introduced in Uttarakhand.
✓ The train will cover a distance of 302 km in 4 hours and 45 minutes.
✓ Earlier, PM Modi flagged off the Vande Bharat Express between Puri and Howrah.
✓ About Vande Bharat Express Train
✓ The first Vande Bharat Express was launched by Prime Minister Narendra Modi on 15 February 2019.
✓ These trains have a self-propelled engine which can save diesel and reduce electricity usage by up to 30%.
✓ The first Vande Bharat Express was manufactured by Integral Coach Factory (ICF), Chennai.
✓ It was manufactured under the ‘Make in India’ programme, at a cost of about Rs 100 crore.
✓ These trains can achieve a maximum speed of 160 kmph.
✓ In the Union Budget for 2022-2023 the government has proposed the development and manufacture of
400 new Vande Bharat trains in the next three years
Q. Which country will introduce mandatory health labelling of alcoholic drinks, warning of the links to
cancer, liver disease, and the risk of drinking while pregnant?
A) Ireland B) Norway
C) Denmark D) Sweden
Answer : A
✓ Ireland to become the first country in the world to introduce health warnings on alcohol.

Follow us: Official Site, Telegram, Facebook, Instagram, Instamojo 720


✓ Ireland's health minister, Stephen Donnelly, signed new regulations on May 22 that will carry warnings
about the health risks of drinking alcohol, as well as the product's calorie count
✓ This innovative step aims to raise awareness about the potential health risks associated with alcohol
consumption.
✓ The introduction of health warnings on alcohol bottles and cans will serve as a powerful tool for public
health promotion and education.
✓ Republic of Ireland
✓ The Republic of Ireland is a western European country which shares a boundary with Great Britain.
✓ The Parliament of Ireland is called the Oireachtas.
✓ President of Ireland: Michael D. Higgins
✓ Currency: Euro
✓ Capital: Dublin
Q. Which Country celebrates Bhoto Jatra?
A) Bhutan B) Mongolia
C) Nepal D) Sri Lanka
Answer : C
✓ Nepal celebrates Bhoto Jatra which is a public holiday in Kathmandu Valley.
✓ This celebration is a part of Rato Machhindranath Jatra which is a month-long chariot procession.
✓ In Bhoto Jatra, a "lost & found" bejeweled Nepali vest "bhoto" of Rato Machchindranath is displayed to
the public to find its "rightful owner"..
NEPAL IN NEWS 2023
▪ Nepal celebrates Bhoto Jatra which is a public holiday in Kathmandu Valley.
▪ Nepal designates 2025 as a ‘Special tourism year’.
▪ Nepali climber Kami Rita Sherpa reached the top of Mount Everest for the 27th time, reclaiming the
record for the most summits of the world’s highest mountain.
▪ Pasang Dawa Sherpa becomes 2nd person to scale Everest 26 times.
▪ Indian E-Wallets to be Accepted in Nepal for Digital Payments.
▪ Nepal becomes founding member of International Big Cats Alliance.
▪ Ram Chandra Paudel has been elected to serve as the new president of Nepal.
▪ Nepal have qualified for the Asia Cup 2023 after winning the ACC Men’s Premier Cup.
▪ Ram Sahaya Prasad Yadav becomes Nepal’s third Vice President.
▪ 16th Edition of Indo-Nepal joint training Exercise “SURYA KIRAN-XVI” between India and Nepal will
be conducted at Nepal Army Battle School, Saljhandi, Nepal.
▪ General Bipin Rawat, a bell has been placed at Nepal’s revered Shree Muktinath Temple. The bell named
“Bipin Bell,”.
▪ Aasif Sheikh, a wicketkeeper from Nepal, has been awarded the 2022 Christopher Martin-Jenkins Spirit
of Cricket Award.
▪ About Nepal
▪ The kingdom of Nepal was established by the Shah dynasty.
▪ It is a landlocked country in South Asia.
▪ Prime Minister - Pushpa Kamal Dahal
▪ President - Ram Chandra Paudel
▪ Capital - Kathmandu
▪ Currency - Nepalese Rupee

Follow us: Official Site, Telegram, Facebook, Instagram, Instamojo 721


Q. Who has been elected the chairman of the Internet and Mobile Association of India (IAMAI) for 2023-
25?
A) Bhavit Sheth B) Harsh Jain
C) Vani Kola D) Anand Jain
Answer : B
✓ Harsh Jain, CEO of online gaming platform 'Dream11', has been appointed as the new chairman of
'Internet and Mobile Association of India' (IAMAI) for two years (2023-2025).
✓ Harsh Jain has replaced Sanjay Gupta, Vice President and Country Manager, Google India at IAMAI.
✓ Along with this, MakeMyTrip co-founder and Group CEO Rajesh Mago has been appointed as IAMAI
Vice Chairman and Times Internet Vice Chairman Satyan Gajwani as Treasurer.
✓ Internet and Mobile Association of India (IAMAI)
✓ Establishment : 2004
✓ IAMAA is a non-profit industrial body registered under the Societies Act, 1896.
Q. Morarji Desai National Institute of Yoga (MDNIY) under the Union Ministry of AYUSH has organized
a mega ‘Yoga Festival’ in which city?
A) Bengaluru B) Hyderabad
C) New Delhi D) Mumbai
Answer : B
✓ Morarji Desai National Institute of Yoga (MDNIY) under the Union Ministry of AYUSH organized a
mega 'Yoga Festival' at NCC Parade Ground, Hyderabad.
✓ Telangana Governor Dr. Tamilisai Soundararajan was the chief guest at the Yoga Festival being
organized at NCC Parade Ground, Hyderabad on May 27.
✓ This event was organized 25 days before the Yoga Festival to be held in Jabalpur, Madhya Pradesh on
the occasion of 'International Yoga Day' on June 21, 2023.
HYDERABAD IN NEWS 2023
▪ Morarji Desai National Institute of Yoga (MDNIY) under the Union Ministry of AYUSH organized a
mega 'Yoga Festival' at NCC Parade Ground, Hyderabad.
▪ Telangana Chief Minister K Chandrasekhar Rao recently unveiled a 125-feet tall bronze statue of BR
Ambedkar in Hyderabad.
▪ Hyderabad to become India’s first city with 100% sewerage facilities by May 2023.
▪ The G20 Second Digital Economy Working Group meeting began on April 17 in Hyderabad.
▪ ‘Grand start-up Conclave’ on Animal Husbandry and Diary in Hyderabad.
▪ The two-day 'Food Conclave-2023' will be organized on April 28 and 29 in Hyderabad, Telangana.
▪ World Economic Forum (WEF) has chosen Hyderabad to set up its Center for the Fourth Industrial
Revolution focused on healthcare and life sciences.
▪ The seasoned Jean Eric Vergne of DS Penske automobiles won an action-packed Formula E Prix in
Hyderabad.
▪ Union Agriculture and Farmers Welfare Minister Narendra Singh Tomar officially inaugurated the
Integrated Biological Control Laboratory located in Hyderabad.
▪ two-day inception meeting of the Startup-20, the new engagement group of G-20 initiated by India, has
begun in Hyderabad.
▪ Hyderabad host Street Circuit Car Racing before the India’s first Formula E grand Prix to be held in
February 2023.

Follow us: Official Site, Telegram, Facebook, Instagram, Instamojo 722


Q. Which of the following state has recently achieved 100 percent household coverage of banking services
extending them to all sections of people under Jan Dhan Yojana?
A) Madhya Pradesh B) Gujarat
C) Haryana D) Telangana
Answer : D
✓ Telangana achieves 100% coverage of PMJDY.
✓ Since the launch of Prime Minister Jan Dhan Yojana, Telangana State has achieved 100 percent
household coverage of banking services extending them to all sections of people.
✓ As per the Social Security Administration survey, the state has achieved 100 percent coverage and the
benefits of 135 welfare schemes and programmes under 20 departments are credited to these zero-balance
bank accounts of the beneficiaries..
Q. Who has been selected to serve on the Advisory Committee to the President of the Conference of the
Parties (COP28) of the UNFCCC?
A) Mukesh Ambani B) Gautam Adani
C) Ratan Tata D) Kumar Mangalam Birla
Answer : A
✓ Indian business tycoon Mukesh Ambani has been nominated to the Consultative Committee for the 28th
session of the Conference of the Parties (COP28) to the United Nations Framework Convention on
Climate Change (UNFCCC).
✓ This committee will work to guide and advise the 28th session of the Conference of the Parties (COP28)
on Climate Change.
✓ Apart from Sunita Narayan, director general of the Center for Science and Environment, Ambani is the
only Indian to be included in the COP28 President's Advisory Committee.
✓ It is noteworthy that Mukesh Ambani's Reliance Industries Limited (RIL) is promoting renewable
energy.
✓ About COP28
✓ The United Nations Framework Convention on Climate Change convention is known as the Conference
of the Parties (COP).
✓ The COP28 conference will be held in Dubai from 30 November to 12 December 2023.
✓ The COP28 UAE Advisory Committee brings together climate experts from 6 continents, representing
policy, industry, energy, finance and civil society.
✓ There are 31 members in this committee
✓ About, UN Framework Convention on Climate Change (UNFCCC).
✓ Till date, it has been ratified by 197 countries.
✓ UNFCCC is headquartered in Bonn, Germany
✓ UNFCCC is the parent treaty of the 2015 Paris Agreement as well as 1997 Kyoto Protocol.
✓ The UNFCCC was established to work towards “stabilisation of greenhouse gas concentrations in the
atmosphere
✓ 26th session of the Conference of the Parties (COP 26) to the UNFCCC was originally scheduled to take
place from 9-19 November 2020, in Glasgow, UK.
✓ 27th session of the Conference of the Parties (COP 27) to the UNFCCC will take place in Sharm El-
Sheikh, Egypt.
✓ 28th session of the Conference of the Parties (COP 28) to the UNFCCC is scheduled to take place from
6-17 November 2023. It will take place in the United Arab Emirates (UAE).

Follow us: Official Site, Telegram, Facebook, Instagram, Instamojo 723


Q. Who has won the Men’s Singles title of Malaysia Masters 2023 at Kuala Lumpur?
A) HS Prannoy B) Srikanth Kidambi
C) Lakshya Sen D) B. Sai Praneeth
Answer : A
✓ India's star shuttler HS Prannoy won the gold medal in the Malaysia Masters Badminton Championship
2023 played in Kuala Lumpur, Malaysia.
✓ Prannoy won the final by defeating China's Wang Hongyang in three games 21-19, 13-21 and 21-18. The
match between the two lasted for 94 minutes.
✓ Prannoy became the third Indian after Saina Nehwal and PV Sindhu to win the Malaysia Masters
Championship.
✓ Also became the first Indian to win in the men's singles category.
✓ This is Pranay's first medal this season.
✓ With Prannoy, India has won the championship 4 times.
✓ Gold came in Men's Singles for the first time. At the same time, India has won gold three times in
women's singles.
✓ PV Sindhu twice and Saina Nehwal once won the championship.
Q. When is the World Hunger Day observed?
A) 25 May B) 28 May
C) 26 May D) 27 May
Answer : B
✓ Every year on May 28, The Hunger Project celebrates the World Hunger Day.
✓ The day was started in 2011 by The Hunger Project to raise awareness about the people living in chronic
hunger worldwide and celebrate sustainable solutions to hunger and poverty and to inspire everyone to
be a part of the solution!
✓ The Hunger Project is US based organization committed to the sustainable end of world hunger.

Q. Who has been appointed as the new Central Vigilance Commissioner (CVC)?
A) Rajiv Jain B) Praveen Sood
C) Praveen Kumar Srivastava D) Dineshwar Sharma
Answer : C
✓ Praveen Kumar Srivastava has been appointed as the new Central Vigilance Commissioner.
✓ President Draupadi Murmu (President Draupadi Murmu) administered the oath of office to Vigilance
Commissioner Praveen Kumar Srivastava as the Central Vigilance Commissioner (CVC).
✓ Srivastava was serving as the acting Central Vigilance Commissioner (CVC) since December last year
after the completion of Suresh N Patel's tenure as the Central Vigilance Commissioner.
✓ The Commission is headed by the CVC and can have a maximum of two Vigilance Commissioners..
✓ About Central Vigilance Commission
✓ CVC is an apex vigilance institution, which is free from any executive authority.
✓ It is an independent body whose accountability is only to the Parliament.
✓ It was established in February 1964 on the basis of the recommendations of the Anti-Corruption
Committee constituted under the chairmanship of K. Santhanam.
✓ The Central Vigilance Commission Act, 2003 (CVC Act) has been enacted by the Parliament conferring
statutory status on CVC.
✓ It submits its report to the President of India.
✓ Appointment of commissioners

Follow us: Official Site, Telegram, Facebook, Instagram, Instamojo 724


✓ The Central Vigilance Commissioner and Vigilance Commissioners are appointed by the President on
the recommendation of a committee consisting of the Prime Minister (Chairman), the Home Minister
(Member) and the Leader of the Opposition (Member) in the Lok Sabha.
✓ Tenure : Their tenure is for 4 years or 65 years (whichever is earlier).
Q. The Government of Nepal has approved India’s _ to develop the second hydroelectric project in the
country.
A) NTPC Limited B) JSW Energy
C) Adani Enterprises D) Sutlej Jal Vidyut Nigam Limited
Answer : D
✓ India gets approval to set up second hydroelectric project in Nepal.
✓ The Government of Nepal has approved India's Sutlej Jal Vidyut Nigam Limited (SJVN) to develop the
second hydroelectric project in the country.
✓ SJVN is currently involved in the development of the Arun-III Hydroelectric Project in eastern Nepal
with a capacity of 900 MW.
✓ The Investment Board Nepal (IBN), chaired by Prime Minister Pushpa Kamal Dahal, approved the draft
Project Development Agreement (PDA) between Nepal and SJVN.
✓ The PDA is for development of the Lower Arun Hydroelectric Project in eastern Nepal with a capacity
of 669 MW.
✓ The Lower Arun Hydroelectric Project is to be developed by SJVN.
Q. BPCL has developed a diesel-ethanol blend to reduce emissions and achieve its net zero targets by which
year?
A) 2025 B) 2030
C) 2035 D) 2040
Answer : D
✓ Public sector oil marketing company BPCL has developed an ethanol-diesel blend in a bid to reduce
emissions.
✓ The move is in line with its net zero target of 2040.
✓ The R&D centre of BPCL, according to the company has filed 164 patents and 87 patents have been
granted across multiple countries, 17 technologies or products commercialized so far said a company
statement.
✓ BPCL is focusing on petrochemicals and sustainable aviation fuel among other products to diversify its
operations.
✓ It has also developed two new technologies, namely the ‘K Model’ for crude compatibility and
‘BPMARRK’ for quick and accurate real-time crude assay

Q. World No Tobacco Day is observed on which day?


A) 31 May B) 28 May
C) 30 May D) 29 May
Answer : A
✓ World No Tobacco Day (WNTD) is observed every year on 31 May, globally to inform the public on
the dangers of using tobacco and to discourage the use of tobacco in any form.
✓ Theme of the 2023 WNTD is : "We need food, not tobacco”
✓ The Day was created by the member states of the World Health Organization (WHO) in 1987

Follow us: Official Site, Telegram, Facebook, Instagram, Instamojo 725


Q. Which state has been ranked the most innovative state by the National Manufacturing Innovation Survey
2021-22?
A) Karnataka B) Telangana
C) Tamil Nadu D) Maharashtra
Answer : A
✓ A survey conducted by the government’s Department of Science and Technology in collaboration with
the United Nations Industrial Development Organization (UNIDO) on the level of innovation in
manufacturing in India has found that the state of Karnataka is not only the most innovative in its
manufacturing sector but also has the highest number of companies in manufacturing that carry out
innovations.
✓ The National Manufacturing Innovation Survey (NMIS) 2021-22, released earlier this month, also found
that innovation in manufacturing is the lowest in the Northeastern states (excluding Assam), followed by
Bihar.
✓ DST ranked Karnataka, followed by Telangana, Tamil Nadu, Maharashtra, and Haryana as high
innovation states.
Q. Olympic champion javelin thrower Neeraj Chopra won the Diamond League 2023 title with a throw
___.
A) 85.55 meter B) 87.97 meter
C) 88.67 meter D) 86.88 meter
Answer : C
✓ Neeraj Chopra won the javelin throw event at the Doha Diamond League meet with a throw of 88.67m.
Q. Japan has decided to provide specialized training to how many Indian engineers ahead of starting work
on the high-speed rail track system for the Mumbai-Ahmedabad High Speed Rail Corridor (MAHSR)?
A) 800 B) 1000
C) 750 D) 1200
Answer : B
✓ Japan has decided to provide specialized training to 1,000 Indian engineers ahead of starting work on the
high-speed rail track system for the Mumbai-Ahmedabad High Speed Rail Corridor (MAHSR).
✓ National High Speed Rail Corporation Limited (NHSRCL) has signed an MoU with Japan Railway
Technical Service (JARTS) to operationalize bullet trains in the country.
✓ Work on the high-speed rail track system will be started only under this agreement.
✓ The bullet train being built between Mumbai and Ahmedabad will use a ballastless slab track system,
similar to that used in the Japanese Shinkansen high-speed railway. Because the Japanese Shinkansen
high speed railway will be used in the Indian bullet train.
✓ The ballastless slab track system is known as J slab track system.
✓ The Japanese track system is one of the best in the world and its construction requires a very high level
of skill from all the technicians.
✓ According to the National High Speed Rail Corporation Limited (NHSRCL) in this regard, the track is
the most important part of the HSR system and needs to be laid with a very high degree of precision.
✓ For this purpose, 20 Japanese experts will provide intensive training to Indian engineers, supervisors and
technicians and certify their skills.
✓ Mumbai-Ahmedabad High Speed Rail Corridor (MAHSR)
✓ The total length of the corridor between Ahmedabad and Mumbai is 508 km, of which 348 km falls in
Gujarat and 156 km in Maharashtra. The remaining four kilometers will fall in Dadra Nagar Haveli.

Follow us: Official Site, Telegram, Facebook, Instagram, Instamojo 726


✓ A total of 12 stations will be set up in the journey between Ahmedabad and Mumbai, of which eight
stations will be in Gujarat and four in Maharashtra.
Q. In which district, the Uttar Pradesh government has approved the setting up of the state’s first Pharma
Park?
A) Ghaziabad B) Meerut
C) Lalitpur D) Kushinagar
Answer : C
✓ The state government has given green signal to the project of setting up a state's first Pharma Park in
Lalitpur district of Bundelkhand.
✓ For this, the state government will transfer 1500 hectares of land in Lalitpur district to the Animal
Husbandry Department.
✓ The state government will spend Rs 1560 crore for development works and providing necessary facilities
to the investors in Lalitpur Pharma Park.

PARK IN NEWS 2023


▪ The government has given green signal to the project of setting Uttar Pradesh a state's first Pharma Park
in Lalitpur district of Bundelkhand.
▪ India’s First International Multimodal Logistics Park Coming at Jogighopa, Assam.
▪ Andhra Pradesh Chief Minister YS Jagan Mohan Reddy laid the foundation stone of Vizag Tech Park.
▪ Adani Group is setting up two data centers in the Rs 21,844-crore tech park, one of 200 MW and the
other of 100 MW.
▪ India has put forward a proposal to construct a G20 park in Delhi.
▪ Tiger spotted in Haryana’s Kalesar National Park after 10 years.
▪ Bandipur National Park, located in the southern Indian state of Karnataka, recently completed 50 years
as a Project Tiger Reserve.
▪ A rare orange-colored bat was spotted in the Banana plantation in Parali Bodal village of Kanger Valley
National Park in Bastar, Chhattisgarh.
▪ Union Minister for Road Transport and Highways Nitin Gadkari laid the Foundation Stone of the
world’s largest and unique Divyang Park – Anubhuti Inclusive Park in Nagpur, Maharashtra.
▪ Uttar Pradesh government is planning to develop a theme park ‘Ramaland’ modelled on Disneyland to
narrate the story of Lord Ram.
▪ Government, Ministry of Textiles, has issued a notification to set up 7 Mega Integrated Textile Region
and Apparel (PM MITRA) Parks with a total outlay of Rs. 4,445 crore.
▪ Seven sites were chosen out of 18 proposals for PM MITRA parks which were received from 13 States.
▪ Eravikulam National Park in Munnar, Kerala, which is home to the Nilgiri tahr, now features a new
attraction: a fernarium located within the park.
▪ World’s largest safari park to be developed in Gurugram.
▪ A Namibian Cheetah passed away at Kuno National Park in MP, India. The cheetah, named Sasha.
▪ Silent Valley National Park, located in Kerala, in the last month identified 141 species of birds of which
17 were new species of birds.
Q. India has operationalized the Sittwe Port with the departure of the inaugural shipment, the MV-ITT
LION (V-273), from Syall Prasad Mookerjee Port, Kolkata. Sittwe Port is located in which country?
A) Bangladesh B) Afghanistan
C) Maldives D) Myanmar
Answer : D

Follow us: Official Site, Telegram, Facebook, Instagram, Instamojo 727


✓ India has operationalized the Sittwe Port in Myanmar with the departure of the inaugural shipment, the
MV-ITT LION (V-273), from Syall Prasad Mookerjee Port, Kolkata.
✓ Ports, Shipping and Waterways Minister Sarbananda Sonowal will receive the first Indian cargo ship at
Sittwe Port in Myanmar.
✓ Sittwe Port is a part of the Kaladan Multimodal Transit Transport Project and its development will enable
multi-modal transit connectivity with Southeast Asia.
✓ The port has been built under a framework agreement between India and Myanmar to set up a
multimodal transit transport facility on the Kaladan River, which connects Sittwe Port in Myanmar to
Mizoram in India.
✓ The Kaladan River route has the potential to open up economic opportunities for the entire Bay of Bengal
peninsula.
Q. Which Indian Navy ship has been decommissioned recently after a distinguished service of 36 years?
A) INS Magar B) INS Karanj
C) INS Shardul D) INS Airavat
Answer : A
✓ INS Magar, the oldest landing ship of the Indian Navy has decommissioned after a prestigious service to
the country for 36 years.
✓ The ship, commanded by Cdr Hemant V Salunkhe, has decommissioned at a sunset ceremony held at
the Naval Base here.
✓ INS Magar was launched on November 16, 1984 by Meera Tahiliani and commissioned on July 18, 1987
at Garden Reach Shipyard and Engineers Limited, Kolkata by late Admiral R H Tahiliani.
✓ INS Magar – Indian Navy’s Landing Ship Decommissioned After 36 Years of Service at the Naval base
in Kochi.
Q. The Border Roads Organisation (BRO) observes its Raising Day every year on 7 May. In 2023, the BRO
is celebrating which edition of its raising day?
A) 64 B) 55
C) 72 D) 36
Answer : A
✓ Border Roads Organization (BRO) celebrated its 64th Raising Day on 7 May 2023 at all its detachments
across the country.
✓ The main function ‘Chief Engineers and Equipment Management Conference’ was organized at BRO
School & Centre in Pune.
✓ The Prime Minister has praised the initiative by Border Roads Organisation Project Dantak to
commemorate 64th Raising Day.
✓ Project Dantak is one of the oldest projects of the Border Roads Organization (BRO).
✓ It was established on 24th April1961.
✓ It was a result of the visionary leadership of the King of Bhutan and Jawahar Lal Nehru, then Prime
Minister of India.
✓ It was established under the provision of the Indo-Bhutan Treaty of Peace and Friendship, 1949.
✓ It is tasked with the construction and maintenance of roads suitable for motorized transport in Bhutan.

✓ About Border Roads Organisation (BRO)


✓ BRO, a pioneer in the field of infrastructural development in the border areas of the country.
✓ BRO is a major road construction agency under the Ministry of Defence with the primary objective of
providing road connectivity to the border areas.

Follow us: Official Site, Telegram, Facebook, Instagram, Instamojo 728


✓ BRO was formed on 7th May 1960.
✓ It develops and maintains the road network in India's border areas and friendly neighbouring countries.
✓ One of the biggest infrastructural achievements of BRO - longest highway tunnel built in Himachal
Pradesh, and its name is Atal Tunnel.
✓ Atal tunnel under Rohtang Pass is an 8.8-kilometer-long tunnel.
✓ This will reduce the distance between Manali and Leh by 46 kms and will save on transportation costs.
✓ Director General of BRO - Lieutenant General Rajeev Chaudhry
Q. The trailer of ‘Love in 90s’, the first film made in Tagin language has launched by Union Law and Justice
Minister Kiren Rijiju in New Delhi. Tagin is a language of which Indian state?
A) Sikkim B) Tripura
C) Nagaland D) Arunachal Pradesh
Answer : D
✓ The trailer of 'Love in 90s', the first film made in Tagin language of Arunachal Pradesh, was launched by
Union Law and Justice Minister Kiren Rijiju on 6th May in New Delhi.
✓ The film depicts the vibrant and colorful world of the 90s, based on the Tagin community in Upper
Subansiri district of Arunachal Pradesh and the first film made entirely in Tagin language.
✓ Directed by Tapen Natam, the film brings to the national platform the rich culture, tradition and local
filmmaking initiatives of Arunachal Pradesh.
✓ The film highlights the love story of two youths amidst the challenges faced by this community in
Arunachal Pradesh during the 1990s.
✓ The film not only celebrates local talent and filmmaking initiatives, but also offers a unique perspective
on the struggles and triumphs of the Tagin community.
✓ Through this film, viewers can witness the beauty and rich heritage of Tagine culture and traditions.
Q. The World Red Cross and Red Crescent Day is observed annually on which day?
A) 08 May B) 06 May
C) 07 May D) 05 May
Answer : A
✓ The World Red Cross and Red Crescent Day is observed every year on 8 May.
✓ The day aims to celebrate the principles of the International Red Cross and Red Crescent Movement, to
reduce the suffering of people and enabling them to lead a dignified life with independence, humanity,
impartiality, universality, unity and neutrality.
✓ Theme 2023 : "Everything we do comes #fromtheheart."
✓ The day also marks the anniversary of the birth of Henry Dunant (8 May 1828), who was the founder of
International Committee of the Red Cross (ICRC).
✓ He was the recipient of the first Nobel Peace Prize.
Q. Which ministry has launched the Learning Management Information System 'Saksham'?
A) Ministry of Health B) Ministry of Education
C) Ministry of Coal D) Ministry of Home Affairs
Answer : A
✓ Union Health Ministry launched the SAKSHAM Learning Management Information System and it was
launched by Health Secretary Rajesh Bhushan.
✓ SAKSHAM stands for Stimulating Advanced Knowledge for Sustainable Health Management.
✓ It aims to provide online training and medical education to all health professionals across the country.

Follow us: Official Site, Telegram, Facebook, Instagram, Instamojo 729


✓ This digital learning platform will facilitate comprehensive capacity building among health professionals.
✓ Currently, it offers over 200 public health and 100 clinical courses in an online format.
✓ Health professionals can register for these courses by visiting the website lmis.nihfw.ac.in.
✓ After completing the required training and meeting the evaluation criteria, participants will receive
certification.
✓ SAKSHAM was developed by the National Institute of Health and Family Welfare in New Delhi.
Q. The Petersberg Dialogue 2023 on Climate Change has been held in which city?
A) Berlin, Germany B) Paris, France
C) Vienna, Austria D) Geneva, Switzerland
Answer : A
✓ The Petersberg Dialogue on Climate Change was held in Berlin from May 2-3, 2023.
✓ The Petersberg Climate Dialogue (PCD) was hosted by Germany and the United Arab Emirates, which
is hosting the 28th Conference of the Parties (COP28) to the United Nations Framework Convention on
Climate Change.
✓ Ministers from 40 countries attended the conference to discuss the way forward towards COP28.
✓ The main topics of discussion at the event were renewable targets, climate finance and global stocktake.
Q. The 42nd Summit of the Association of Southeast Asian Nations (ASEAN) started in Indonesia. What
is the theme of ASEAN 2023?
A) We Care, We Prepare, We Prosper B) Stronger Together
C) ASEAN Together D) ASEAN Affairs: Epicenter of Growth
Answer : D
✓ The 42nd Summit of the Association of Southeast Asian Nations (ASEAN) started in Indonesia.
✓ Theme 2023 – ASEAN Affairs: Epicenter of Growth.
✓ The summit aims to demonstrate the bloc’s hopes and efforts to become the center and driving force
behind global development.
✓ President Joko Widodo, who holds the presidency of the bloc.
✓ Association of Southeast Asian Nations (ASEAN)
✓ Founded : 1967
✓ Headquarteres : Jakarta, Indonesia
✓ Secretary General : Lim Jock Hoi
✓ Member : 10 (Brunei, Cambodia, Indonesia, Laos, Malaysia, Myanmar, Philippines, Singapore,
Thailand & Vietnam)
✓ Trick to Remember Countries of ASEAN
✓ Trick : TV CLIP of MBBS
✓ T - Thailand
✓ V - Vietnam
✓ C - Cambodia
✓ L- Laos
✓ I - Indonesia
✓ P - Philippines of silent
✓ M - Malaysia
✓ B - Burma (Myanmar)
✓ B - Brunei
✓ S - Singapore.
✓ Association of Southeast Asian Nations (ASEAN) has agreed ‘in principle’ to admit East Timor as the
group’s 11th member.

Follow us: Official Site, Telegram, Facebook, Instagram, Instamojo 730


Q. Oil ministry panel has recommended the Indian government to ban the use of diesel-powered four-
wheeler vehicles by which year?
A) 2027 B) 2030
C) 2035 D) 2026
Answer : A
✓ Oil Ministry panel proposes ban on 4-wheeler diesel vehicles by 2027.
✓ An oil ministry panel has recommended the Indian government to ban the use of diesel-powered four-
wheeler vehicles by 2027 and switch to electric and gas-fuelled vehicles in cities with more than a million
people and polluted towns in order to cut emissions.
✓ By 2030, no city buses should be added which are not electric…diesel buses for city transport should not
be added from 2024 onwards.
✓ The Ministry of Petroleum and Natural Gas has set up an Energy Transition Advisory Committee,
headed by former oil secretary Tarun Kapoor.
✓ The country aims at producing 40% of its electricity from renewables to achieve its 2070 net zero goal.
Q. Which state has become first state to make digital health cards for children?
A) Andhra Pradesh B) Telangana
C) Kerala D) Uttar Pradesh
Answer : D
✓ Uttar Pradesh becomes first state to make digital health cards for children.
✓ The Uttar Pradesh’s Urban Development Department and Lucknow Smart City launched a “School
Health Program”.
✓ Under the pilot project, the School Health Program has been started by Lucknow Smart City in three
schools in Lucknow.
✓ A digital health report card for the overall physical and mental well-being of 1765 municipal school
students is also being prepared under the project.
✓ The benefit of health insurance worth Rs 25,000 is also being given to these children
Q. The National Technology Day is observed every year on which day?
A) 10 May B) 11 May
C) 09 May D) 08 May
Answer : B
✓ In India, the National Technology Day is observed every year on May 11 since 1999 to mark the
anniversary of 1998 Pokhran nuclear tests and India’s technological advancements in space.
✓ It was on this day in 1998 that India successfully fired an Operation Shakti missile at the Indian Army’s
Pokhran Test Range in Rajasthan. The Test was led by late aerospace engineer and President Dr. APJ
Abdul Kalam.
✓ It also highlight the achievements of our scientists and engineers in the field of science and technology
and encourage students to embrace Science as a career option.
Q. Which company has collaborated with the NASA to to convert satellite data into high resolution maps?
A) TCS B) IBM
C) Intel D) Oracle
Answer : B

Follow us: Official Site, Telegram, Facebook, Instagram, Instamojo 731


✓ International Business Machines (IBM) has collaborated with the National Aeronautics and Space
Administration (NASA) to launch a new geospatial foundation model.
✓ International Business Machines Corporation (IBM)
✓ Founded : 1911
✓ Founders : Charles Ranlett Flint
✓ Headquarters : Armonk, New York, U.S.
✓ Chairman & CEO : Arvind Krishna.

Q. Which country will host the 6th Indian Ocean Conference (IOC)?
A) India B) Bangladesh
C) Singapore D) Sri Lanka
Answer : B
✓ The 6th Indian Ocean Conference (IOC) is taking place in Dhaka, Bangladesh from May 12-13, 2023.
✓ The sixth edition of the conference is being organized by the India F oundation in collaboration with
the Ministry of Foreign Affairs of Bangladesh and the S Rajaratnam School of International Studies.
✓ The theme of this year's conference is 'Partnership for Peace, Prosp erity and a Resilient Future'.
✓ The conference was inaugurated by Bangladesh Prime Minister Sheikh Hasina.
✓ According to Bangladesh Minister of State for Foreign Affairs Shaha ryar Alam, around 150 foreign
guests including representatives from D8, SAARC and BIMSTEC will participate in the conference.
Q. Who has been appointed as Chairman and Managing Director of Infrastructure major Larsen & Toubro
(L&T)?
A) Vivek Sankaran B) Punit Renjen
C) Vasant Narasimhan D) S N Subrahmanyan
Answer : D
✓ S N Subrahmanyan has been appointed as Chairman and Managing Director of Infrastructure major
Larsen & Toubro, with effect from October 1, 2023.
✓ He will succeed A M Naik, who steps down as Non-Executive Chairman of the L&T Group at the end
of his tenure on 30 September 2023.
✓ Currently, S N Subrahmanyan is serving as the CEO and MD of L&T.
✓ Subrahmanyan has also worked as Deputy MD & President leading L&T’s infrastructure business to its
position as the country’s largest construction organization
Q. Who is the author of book titled “The Indian Metropolis: Deconstructing India’s Urban Spaces”?
A) Feroze Varun Gandhi B) Vikram Seth
C) Chetan Bhagat D) Salman Rushdie
Answer : A
✓ The book titled "The Indian Metropolis: Deconstructing India’s Urban Spaces" is authored by Feroze
Varun Gandhi.
✓ The book examines the challenges facing India’s urban spaces, including poverty, inequality, crime, and
environmental degradation.
✓ Gandhi argues that India’s cities need to be transformed in order to be more inclusive and sustainable.
✓ The book is based on Gandhi’s own experiences as a politician and urban planner.

Follow us: Official Site, Telegram, Facebook, Instagram, Instamojo 732


Q. Which actor has been included in the Indian team as an ambassador for the Special Olympics trip to
Berlin?
A) Akshay Kumar B) Ajay Devgn
C) Amitabh Bachchan D) Ayushmann Khurrana
Answer : D
✓ Ayushmann Khurrana has been chosen to inspire and support the Indian contingent participating in the
upcoming Special Olympics for individuals with intellectual disabilities, set to take place in Berlin from
June 16 to June 25, 2023.
✓ Bollywood actor Ayushmann Khurrana, has been appointed as the National Ambassador of Child Rights
for UNICEF India.
✓ Manufacturer of mattresses, Wakefit Innovations Private Limited, has signed actor Ayushmann
Khurrana as Wakefit.co’s brand ambassador.

Q. Which of the following entity has launched the Automated Return Scrutiny Module for GST returns?
A) Central Board of Indirect Taxes and Customs (CBIC)
B) Central Board of Direct Taxes (CBDT)
C) Reserve Bank Information Technology Private Limited (ReBIT)
D) Indian Financial Technology and Allied Services (IFTAS)
Answer : A
✓ The Central Board of Indirect Taxes and Customs (CBIC) has recently launched the Automated Return
Scrutiny Module for GST returns.
✓ This module will enable tax officers to scrutinize GST returns of centrally administered taxpayers based
on data analytics and risks identified by the system.
✓ The module identifies discrepancies in the returns and shows them to the tax officer, who can
communicate with the taxpayer through the GSTN Common Portal to resolve any issues.
✓ The Automated Return Scrutiny Module will help in streamlining the GST return scrutiny process and
ensure that it is more efficient, transparent and effective.

Q. When is the International Nurses Day celebrated?


A) May 10 B) May 11
C) May 12 D) May 09
Answer : C
✓ The International Nurses Day is celebrated on May 12, every year to mark the contributions and hard
work of nursing staff around the world.
✓ The theme of 2023 International Nurses Day is “Our Nurses, Our Future”.
✓ The day was created by the International Council of Nurses, and is celebrated globally since January
1974.
✓ This day also marks the birth anniversary of nursing pioneer Florence Nightingale, who was born on
May 12, 1820.
Q. The International Nurses Day marks the birth anniversary of which social reformer?
A) Clara Barton B) Florence Nightingale
C) Marie Curie D) Mother Teresa

Follow us: Official Site, Telegram, Facebook, Instagram, Instamojo 733


Answer : B
✓ The International Nurses Day marks the birth anniversary of nursing pioneer Florence Nightingale, who
was born on May 12, 1820.
Q. Which state has become the first state in the country to have a drone surveillance system in all police
districts in the state?
A) Uttar Pradesh B) Punjab
C) Kerala D) Tamil Nadu
Answer : C
✓ Kerala has become the first state in the country to have a drone surveillance system in all police districts
in the state.
✓ Chief Minister Pinarayi Vijayan distributed the drones to all police districts and the drone pilot licences
to the specially trained drone pilots.
✓ He also launched indigenously developed anti-drone software on the occasion.
✓ Kerala has been at the forefront of modernization of the police force and that since the use of drones has
increased in society, the development of an anti-drone system is also important.

Q. London Stock Exchange Group to set up Technology Centre of Excellence in Which City?
A) Hyderbad B) Chennai
C) Bengaluru D) Kolkata
Answer : A
✓ Telangana IT and Industries Minister "KT Rama Rao's" four-day tour of the United Kingdom (UK) to
attract investors to Telangana has got off to a good start.
✓ London Stock Exchange Group PLC (LSEG) announced that it will establish a Technology Centre of
Excellence in Hyderabad, which will generate employment for about 1000 people in a year.
✓ The establishment of the Technology Centre of Excellence by London Stock Exchange Group (LSEG)
in Hyderabad is a significant boost to the Banking, Financial Services, and Insurance (BFSI) sector in the
city.
Q. The first ministerial meeting of the India and the European Union Trade and Technology Council (TTC)
will be held in which City?
A) Paris B) London
C) Brussels D) Geneva
Answer : C
✓ The first ministerial meeting of the India and the European Union Trade and Technology Council (TTC)
will be held in Brussels on 16 May.
✓ The meeting will be co-chaired by the Ministers of External Affairs, Commerce and Industry and
Communications, Electronics and Information Technology from the Indian side.
✓ India is only the second country after the US with which the European Union (EU) has a TTC, and the
two sides agreed to launch the strategic mechanism during the visit of European Commission President
Ursula von der Leyen to India in April 2022.
✓ The TTC aims to deepen trade and investment ties between India and the European Union, as well as
foster greater cooperation on technology and innovation.
✓ About India-EU Trade and Technology Council
✓ The trade and technology council was launched by Prime Minister Narendra Modi and European
Commission President Ursula von der Leyen during Ms Leyen's visit to India in April 2022.

Follow us: Official Site, Telegram, Facebook, Instagram, Instamojo 734


✓ About European Union
✓ The European Union is a group of 27 European countries.
✓ The United Kingdom which was the founding member of the European Union has left the organisation.
✓ It was founded on 1 November 1993
✓ Headquarters: Brussels, Belgium

Q. Which Port has been awarded the prestigious Sagar Shreshta award 2023 ?
A) Jawaharlal Nehru Port B) Cochin Port
C) Tuticorin Port D) Paradip Port
Answer : B
✓ Cochin Port has been awarded the prestigious Sagar Shreshta award for the year 2023 by the Ministry of
Shipping in recognition of its outstanding performance in the maritime sector.
✓ The Sagar Shreshta award is given to Indian ports for achieving excellence in various categories,
including efficiency, productivity, and environmental sustainability.
✓ Cochin Port was selected for the award based on its impressive track record of achieving high standards
of operational efficiency, safety, and environmental sustainability, as well as its contribution to the
development of the maritime sector in India.
✓ Cochin Port's receipt of the Sagar Shrestha award reflects its leadership in the maritime sector and its
commitment to driving innovation and excellence in the industry
Q. Who has been appointed as the US Ambassador for Global Women's Issues?
A) Neera Tanden B) Vanita Gupta
C) Uzra Zeya D) Geeta Rao Gupta
Answer : D
✓ The US Senate has confirmed the appointment of Indian-American Geeta Rao Gupta as the 'Ambassador
at Large'for global issues related to women at the State Department.

Q. Karthik Balagopalan has been appointed as the new Managing Director of which company?
A) Puma B) Nike
C) Adidas D) Reebok
Answer : A
✓ Sportswear retailer Puma has appointed Karthik Balagopalan as the new Managing Director of PUMA
India.
✓ He will succeed Abhishek Ganguly, who worked for PUMA’s Indian business for 17 years and has been
the Managing Director of PUMA India since 2014.
Q. The Asian Kabaddi Championship 2023 is scheduled to be held in which country?
A) China B) Thailand
C) South Korea D) Singapore
Answer : C
✓ The Asian Kabaddi Championship 2023 is scheduled to be held in Busan, South Korea from June 27 to
30.

Follow us: Official Site, Telegram, Facebook, Instagram, Instamojo 735


Q. The 3rd Energy Transition Working Group Meeting (ETWG) under India's G20 chairmanship began in
which hich city?
A) New Delhi B) Mumbai
C) Chennai D) Hyderabad
Answer : B
✓ The 3rd Energy Transition Working Group Meeting (ETWG) under India's G20 chairmanship began in
Mumbai from May 15 to 17, 2023.
✓ The meeting was organized to discuss and address issues related to energy transition in the context of
global energy challenges and sustainable development.
Q. Indian government will soon launch a ‘Trusted Traveller Programme’ for faster immigration clearance
of pre-verified travellers. This program will be launched on pilot basis at how many airports this year?
A) three B) two
C) four D) six
Answer : B
✓ Indian govt to launch faster immigration clearance scheme for ‘trusted travellers’
✓ With an aim to reduce congestion at airport check points, Indian government will soon launch a ‘Trusted
Traveller Programme’ for faster immigration clearance of pre-verified travellers.
✓ The new scheme will be launched for travellers with Indian passports across international airports in
India.
✓ The program will be started on a pilot basis at Delhi and Mumbai airports this year.
✓ Later it will be expanded to 15 more international airports by 2027 and all airports by 2032.

Q. The technology firm Cisco will establish a manufacturing facility in which state of India?
A) Karnataka B) Telangana
C) Tamil Nadu D) Kerala
Answer : C
✓ The technology firm Cisco has announced its plans to commence manufacturing operations in India,
aiming to establish a more diverse and resilient global supply chain and support India’s vision of
becoming a global manufacturing hub.
✓ The company’s CEO, Chuck Robbins, stated that the manufacturing facility in Tamil Nadu would
generate 1,200 job opportunities.
✓ Cisco has set a target of $1 billion in combined domestic production and exports in the upcoming years.
✓ Cisco plans to establish core manufacturing capabilities in India, including testing, development,
logistics, and expanding in-house repair operations
Q. Who has been conferred the first Govind Swarup Lifetime Achievement Award instituted by the
Astronomical Society of India (ASI)?
A) Karan Jani B) Ramana Athreya
C) Shrinivas Kulkarni D) Jayant Naralikar
Answer : D
✓ Veteran astronomer Jayant Naralikar has been conferred the first Govind Swarup Lifetime Achievement
Award instituted by the Astronomical Society of India (ASI).

Follow us: Official Site, Telegram, Facebook, Instagram, Instamojo 736


✓ Naralikar, a past president of ASI, was the founding director of the Inter-University Centre for
Astronomy and Astrophysics (IUCAA) and amongst the first to initiate cosmology research in the
country.
✓ The award is named in honour of Prof. Govind Swarup (1929-2020), widely regarded to be the founder
of radio astronomy in the country.

Q. Who becomes 2nd person to scale Everest 26 times?


A) Pasang Dawa Sherpa B) Oliver Scoubes
C) Gurdial Singh D) Gombu Sherpa
Answer : A
✓ 46 year old Pasang Dawa Sherpa (From Nepal), has become the second person to climb Mount Everest
for the26th time.
✓ The Himalayan database confirms that Pasang Dawa Sherpa has previously climbed Everest 25 times,
including two in 2022.
✓ He has been climbing Everest almost every year since his first successful climb in 1998.
✓ Kami Rita, a climber who is a U.S. Leading the team, and is the first person to conquer Mount Everest
for the 26th time.
✓ Mount Everest:
✓ It is the highest mountain in the world, located in the Mahalangur Himal sub-range of the Himalayas.
✓ Its summit point is located on the China–Nepal border with a confirmed elevation of 8,848.86 m
(29,031.69 ft) in 2020.
Q. The annual ASEAN Tourism Forum will be hosted by Which Country in January 2024?
A) Laos B) Cambodia
C) Malaysia D) Philippines
Answer : A
✓ The annual ASEAN Tourism Forum will be hosted by Laos in January 2024.
✓ Theme : The theme of the forum will be "Quality and Responsible Tourism -- Sustaining ASEAN
Future".
✓ Previously, Laos hosted the ASEAN Tourism Forum in 2004 and 2013.
✓ The 42nd Summit of the Association of Southeast Asian Nations (ASEAN) started in Indonesia.
✓ Theme 2023 – ASEAN Affairs: Epicenter of Growth.

Q. Who has taken oath as the chairman of the Union Public Service Commission (UPSC)?
A) Vinay Mittal B) Arvind Saxena
C) Manoj Soni D) Bhim Sain Bassi
Answer : C
✓ Eminent educationist Manoj Soni has taken oath as the chairman of the Union Public Service
Commission (UPSC).
✓ Soni, who joined the Commission as a member on June 28, 2017, has been performing the duties of the
UPSC chairman since April 5, 2022.
✓ He was administered the oath of office and secrecy as chairman, UPSC by Smita Nagaraj, senior most
member in the Commission.
✓ Prior to his appointment in the UPSC, Soni has served three terms as Vice-Chancellor.
✓ Union Public Service Commission (UPSC) is the central recruiting agency in India. It is an independent
constitutional body.

Follow us: Official Site, Telegram, Facebook, Instagram, Instamojo 737


✓ Articles 315 to 323 in Part XIV of the constitution contain elaborate provisions regarding the composition
of the UPSC.
✓ The commission usually consists of 9 to11 members including the Chairman
✓ UPSC is consists of the Chairman and the members appointed by the President of India.
✓ The Chairman and the members of the UPSC hold the office for a term of 6 years or till the age of 65
years.
✓ Note – Article 315 of the Indian Constitution deals with Public Service Commissions.
✓ Article 316 deals with the appointment and term of office of members.
✓ Union Public Service Commission (UPSC)
✓ Formed : 1 October 1926
✓ Headquarters : New Delhi
✓ Chairman : Manoj Soni
Q. RuPay has introduced the CVV free payment experience for its debit, credit, and prepaid cardholders.
What is the full form of CVV?
A) Card Verified Value B) Card Value Verification
C) Credit Verification Value D) Card Verification Value
Answer : D
✓ RuPay has introduced the CVV (Card Verification Value) free payment experience for its debit, credit
and prepaid cardholders who have tokenized their cards on the merchant application or webpage.
✓ This new CVV less experience ensures that the cardholder will not have to reach out to their wallet or
remember any card details, if they have saved (tokenized) their card on the e-commerce merchant which
supports this feature.
✓ They will just have to enter the OTP or their device will auto-populate the OTP during the domestic e-
commerce transaction and the payment will be done.
✓ Tokenization is a simple technology to secure card transactions without sharing clear or real card details
with the merchants.

Q. Who has won La Liga for 1st time since 2019?


A) Manchester United B) Manchester City
C) Barcelona D) Liverpool
Answer : C
✓ Barcelona has won its first Spanish league title after the departure of Lionel Messi with a 4-2 victory
against Espanyol
✓ Robert Lewandowski scored twice to lead the Catalan club to its first league title since 2019.
✓ The title was secured with four rounds remaining and two years after Lionel Messi left amid the club’s
financial struggles.
✓ Alejandro Balde and Jules Koundé also scored for Barcelona, which now has 27 league titles, eight fewer
than Real Madrid
Q. Sumit Mukherjee won the gold medal at the Bridge Federation of Asia and Middle-East Championship
(BFAME). In which country is the BFAME Championship being organized?
A) Bangladesh B) Pakistan
C) Vietnam D) Thailand
Answer : B

Follow us: Official Site, Telegram, Facebook, Instagram, Instamojo 738


✓ Sumit Mukherjee, a metro employee from Kolkata, won four gold medals at the Bridge Federation of
Asia and Middle-East Championship (BFAME) held in Lahore, Pakistan.
✓ Sumit Mukherjee was the only railway staff member in the 24-member Indian contingent.
✓ The games were divided into four categories: Open, Women's, Mixed and Senior, and Sumit represented
India in the Open category.
✓ Sumit Mukherjee had earlier won a bronze medal at the 2018 Jakarta Asian Games and the 4th Asia Cup
Bridge Championships to be held in Jakarta in 2022.

Q. When is the International Museum Day observed globally?


A) 15 May B) 16 May
C) 17 May D) 18 May
Answer : D
✓ The International Museum Day is celebrated on 18 May, since 1977, to raise awareness about the fact
that “Museums are an important means of cultural exchange, enrichment of cultures and development
of mutual understanding, cooperation and peace among peoples.”
✓ Theme of International Museum Day 2023 : Museums, Sustainability and Well-being
✓ It is coordinated by the International Council of Museums (ICOM).
Q. University Grants Commission (UGC) has recently launched two UGC websites named, UTSAH and
POP portals. What is the meaning of S in UTSAH and second P in POP respectively?
A) Solution, Practice B) Strategies, Professor
C) Student, Professor D) Strategies, Practice
Answer : D
✓ UGC Chairman Prof. M Jagadesh Kumar redesigned the UGC website, UTSAH and POP portal.
✓ University Grants Commission (UGC) has launched two UGC websites, UTSAH (Undertaking
Transformative Strategies and Actions in Higher Education) and POP (Professor of Practice) portals.
✓ The redesign is part of the UGC's transformational strategies and actions for higher education.
✓ The updated website aims to provide a user-friendly interface for students, teachers and stakeholders.
✓ The UGC website, UTSAH and POP portals are now live and accessible.
Q. Who has been appointed as the new Law Minister of the country?
A) DK Shivakumar B) Arjun Ram Meghwal
C) KC Venugopal D) Kiren Rijiju
Answer : B
✓ Arjun Ram Meghwal was appointed as the new Law and Justice Minister and the previous Law and
Justice Minister Kiren Rijiju was assigned the Ministry of Earth Sciences.
✓ Meghwal will also continue in his current position as Minister of State for Parliamentary Affairs and
Culture.
✓ Arjun Ram Meghwal will replace Kiren Rijiju in the Ministry of Law and Justice
Q. Who was the chairman of Hinduja Group, who passed away recently?
A) S.P. Hinduja B) Ashok Hinduja
C) Ajay Hinduja D) Jai Hinduja
Answer : A

Follow us: Official Site, Telegram, Facebook, Instagram, Instamojo 739


✓ Recently, Chairman of Hinduja Group S.P. Hinduja died in London at the age of 87.
✓ After completing his education in 1952, he joined his father and Hinduja Group founder P.D. Entered
into business with Hinduja.
✓ The Hinduja Group was founded in 1914 by Parmanand Deepchand Hinduja.
Q. Which state government has approved the 'Mukhya Mantri Sikho Kamao Yojana' for unemployed
youth?
A) Madhya Pradesh B) Uttar Pradesh
C) Chhattisgarh D) Bihar
Answer : A
✓ Madhya Pradesh cabinet, chaired by Chief Minister Shivraj Singh Chouhan, on May 17 approved the
'Mukhya Mantri Sikho Kamao Yojana' for unemployed youth.
✓ It was decided in the meeting that the name of Mukhya Mantri Kaushal Kamai Yojana will be
'Mukhyamantri Sikho-Kamao Yojana' (Chief Minister Learn-Earn Scheme).

Q. What is the theme of World Food India 2023?


A) Recover Together, Recover Stronger B) Processing for Prosperity
C) People, Planet, Prosperity D) One Earth One Family One Future
Answer : B
✓ Secretary, Food Processing Industries (FPI) Anita Praveen, chaired the First Inter- Ministerial
Committee Meeting with senior representatives from Ministries/Departments/Boards on 16th May 2023
in New Delhi.
✓ The agenda of the committee meeting was to apprise the officials about the preparations made so far for
World Food India 2023 and to discuss the participation of concerned Ministries / Departments /
Organizations in the event.
✓ About World Food India
✓ World Food India (WFI) is a global event to facilitate partnerships between Indian and international
businesses and investors.
✓ It is a gateway to the Indian food economy and an opportunity to showcase, connect and collaborate.
✓ It is organized by the Ministry of Food Processing Industries, Government of India.
✓ World Food India 2023 theme is “Processing for Prosperity”.
✓ It aims to provide investment and business opportunities in the food processing sector for leading Indian
and international companies.
Q. Union Health Ministry announced the "75/25" initiative to provide standard care to 75 million people
with hypertension and diabetes by which year?
A) 2024 B) 2025
C) 2030 D) 2027
Answer : B
✓ Union Health Ministry announced the "75/25" initiative to provide standard care to 75 million people
with hypertension and diabetes by 2025 through Primary Health Centers (PHCs).
✓ 75/25 Initiative:
✓ 40,000 primary health care medical officers will be trained on standard treatment workflow for non-
communicable diseases (NCDs) through the SASHAKT portal.
✓ India has the largest coverage of NCDs globally, aiming to reach 75 million people with hypertension
under standard care in primary healthcare by 2025.

Follow us: Official Site, Telegram, Facebook, Instagram, Instamojo 740


✓ The initiative was launched during the G20 co-branded event "Accelerating Prevention and Management
of Hypertension and Diabetes".
✓ SASHAKT Portal:
✓ The National Health Systems and Resources Center (NHSRC) has developed a web portal named
SASHAKT (Systematic Assessment of Health care providers Knowledge and Training).
✓ The portal supports states in implementing and monitoring training for health workers
Q. Uttar Pradesh government will set up a pharmaceutical research and innovation institute in which city?
A) Lucknow B) Kanpur
C) Prayagraj D) Gaziabad
Answer : A
✓ Uttar Pradesh government will set up a pharmaceutical research and innovation institute in Lucknow.
✓ The institute will promote research and manufacturing in the pharma sector.
✓ UP is the sixth-largest state in the nation in terms of the number of pharmaceutical manufacturing
facilities.
✓ UP’s contribution to pharma manufacturing in the country is currently 2%, which needs to be increased
to 10-12%.

Q. The World Bee Day is observed on which day of the year?


A) 20 May B) 19 May
C) 18 May D) 17 May
Answer : A
✓ World Bee Day is held annually on 20 May to raise awareness and recognition of the important role that
bees and other pollinators play in our ecosystem.
✓ The theme for World Bee Day 2023 is “Engaging in Pollinator-Friendly Agricultural Production,”
promoting agricultural methods that prioritise the well-being of pollinators, especially bees.
✓ United Nations established World Bee Day in 2017 to raise awareness of the important role played by
bees and other pollinators in the ecosystem and food production.
Q. Who unveiled bust of Mahatma Gandhi at Hiroshima Peace Park in Japan?
A) Narendra Modi B) Amit Shah
C) J. P. Nadda D) S. Jaishankar
Answer : A
✓ Prime Minister Narendra Modi has unveiled bust of Mahatma Gandhi at Hiroshima Peace Park in Japan,
ahead of the second day of G-7 summit.
✓ The bust was gifted by India to Hiroshima as a symbol of friendship and goodwill between India and
Japan on the occasion of PM Modi’s visit to the city for the G7 summit.
✓ The 42-inch tall bronze bust has been sculpted by Padma Bhushan awardee Ram Vanji Sutar

JAPAN IN NEWS 2023


▪ Prime Minister of India, Shri Narendra Modi, was present at the unveiling ceremony of the bust of
Mahatma Gandhi in Hiroshima, Japan.
▪ Japan ‘seriously looking’ at joining India’s UPI payments system.
▪ India invited to attend G7 Hiroshima summit in Japan.
▪ United Kingdom has announced sanctions on the military-industrial complex and metals as well as
Russian diamonds in a new round of sanctions ahead of the G7 meeting in Japan.

Follow us: Official Site, Telegram, Facebook, Instagram, Instamojo 741


▪ Japan has decided to provide specialized training to 1,000 Indian engineers ahead of starting work on the
high-speed rail track system for the Mumbai-Ahmedabad High Speed Rail Corridor (MAHSR).
▪ The total length of the corridor between Ahmedabad and Mumbai is 508 km, of which 348 km falls in
Gujarat and 156 km in Maharashtra.
▪ Japan will provide an Official Development Assistance (ODA) loan of USD 1.24 billion to Bangladesh
for infrastructure and connectivity projects.
▪ Japan has decided to provide specialized training to 1,000 Indian engineers ahead of starting work on the
high-speed rail track system for the Mumbai-Ahmedabad High Speed Rail Corridor (MAHSR).
▪ Exercise ‘Veer Guardian 2023’ between the Indian Air Force and the Japanese Air Self-Defense Force
(JASDF) will be conducted from January 12 to 26 at Hyakuri air base and Iruma air base in Japan.
▪ The dialogue on April 05, 2023 was co-chaired by Mr. Giridhar Armane, Defense Secretary of India and
Mr. Oka Masami, Defense Minister of International Affairs of Japan.
▪ According to the latest Nikkei Asia industry data, India has overtaken Japan to become the third largest
auto market for the first time.
▪ Union Minister of Health and Family Welfare, Dr Mansukh Mandaviya addressed the G7 Health
Ministerial meeting on Global Health Architecture in Nagasaki, Japan.
▪ India and Japan will conduct their first Air exercise from 16 January to 26 January 2023 at the Hyakuri
Air Base and at the Iruma Air Base in Sayama Japan.
▪ The 4th edition of the joint military exercise between India and Japan, "X Dharma Guardians" will be
conducted from 17 February to 2 March 2023 at Camp Imazu, Shiga Prefecture, Japan.
▪ Bangladesh, India, and Japan are set to hold a connectivity event in Tripura, India on April 11-12.
▪ Japan to invest ₹7,200 crores in Uttar Pradesh, HMI Group is developing 30 hotels in the state.
▪ SJVN Ltd, a state-owned company, has secured Rs 915 crore in ‘GREEN’ financing from Japan Bank
for International Cooperation (JBIC) to support its 90 MW Omkareshwar floating solar project in
Madhya Pradesh and 100 MW Raghanesda solar project in Gujarat.
▪ Indian Air Force (IAF) participated in Exercise Shinyuu Maitri with the Japan Air Self-Defence Force
(JASDF) which was conducted from 13 February 2023 to 02 March 2023 at Komatsu, Japan.
▪ Japan, U.S., South Korea, Taiwan launch ‘Chip 4’ talks for supply chain.
▪ India has surpassed Japan in terms of auto sales last year to become the third-largest auto market globally
for the first time.
▪ JAPAN
▪ Capital : Tokyo
▪ Currency : Yen
▪ Emperor : Naruhito
▪ 100th Prime Minister : Fumio Kishida.
Q. In 2025 year to be observed as 'special tourism year' in Which country?
A) Nepal B) Sri Lanka
C) Maldives D) Vietnam
Answer : A
✓ The decade of the 2080s of Bikram Samvat will be marked as 'the Visit Nepal decade' and the year 2025
as the special year for tourism.
✓ This was announced by the President of Nepal Ram Chandra Paudel while presenting the policies and
programmes for the fiscal year, 2080/81 at the joint meeting of the federal parliament.

Q. Who took oath as the 24th Chief Minister of Karnataka?


A) D. K. Shivakumar B) B. S. Yediyurappa
C) H. D. Kumaraswamy D) Siddaramaiah
Answer : D

Follow us: Official Site, Telegram, Facebook, Instagram, Instamojo 742


✓ Congress Legislative Party (CLP) leader Siddaramaiah was sworn in as the 24th Chief Minister of
Karnataka for his second term and Karnataka Congress President DK Shivakumar was sworn in as the
Deputy Chief Minister.

KARNATAKA IN NEWS 2023


▪ Siddaramaiah was sworn in as the 24th Chief Minister of Karnataka for his second term .
▪ The National Manufacturing Innovation Survey (NMIS) 2021-22 Karnataka, overall, is the most
“innovative” State, followed by Telangana, and Tamil Nadu.
▪ According to the India Justice Report (IJR) 2022, which assesses the performance of states in terms of
justice delivery, the state of Karnataka has achieved the top rank .
▪ World Bank has granted a loan of USD 363 million to the state of Karnataka in India, which will help to
provide clean drinking water.
▪ Karnataka ended their 54-year wait to win the Santosh Trophy national football championship, beating
Meghalaya .
▪ The Karnataka Government will build the country’s first Marina or a boat basin offering dockage, at
Byndoor in Udupi district to promote coastal tourism in Karnataka.
▪ Ministry of Fisheries, Animal Husbandry and Dairying announced that the fourth phase of the Sagar
Parikrama program began in Karnataka.
▪ Karnataka bags National Award For Fasal Bima Yojana, implementing the Pradhan Mantri Fasal Bima
Yojana (PMFBY) scheme.
▪ Karnataka Bank awarded ‘Prathista Puraskar for achieving highest percentage in BHIM-UPI
transactions.
▪ Mysuru and Hampi are the two popular destinations in Karnataka to be promoted as a part of the new
Tourism policy ‘The Swadesh Darshan 2.0 Scheme, which focuses on revving up destination tourism.
▪ Union Home and Cooperation Minister Shri Amit Shah has laid the foundation stone of the Central
Detective Training Institute (CDTI) at Devanahalli in Karnataka.
▪ About Karnataka State
▪ Formation - 1 November 1956
▪ Capital- Bengaluru
▪ Chief minister : Siddaramaiah
▪ Governor- Thawar Chand Gehlot
▪ Legislative Assembly seats- 224
▪ Lok Sabha seats- 28
▪ Jog Falls (Garsoppa) located in Karnataka is the highest waterfall in India which is on Sharavathi River.
▪ Yakshagana is the main folk dance of Karnataka.
Q. Bandhavgarh Tiger Reserve, lies in which state?
A) Assam B) Karnakata
C) Madhya Pradesh D) Uttar Pradesh
Answer : C
✓ An official said a tiger cub was found dead with injuries in the core area of Bandhavgarh Tiger Reserve
(BTR) in Madhya Pradesh.
✓ The cub had injuries on the neck and back, which were caused by mauling.
✓ About Bandhavgarh Tiger Reserve :
✓ It is located in the Umaria district of Madhya Pradesh.
✓ It is spread over the Vindhya hills.
✓ Bandhavgarh was declared a national park in 1968 and then became Tiger Reserve in 1993.

Follow us: Official Site, Telegram, Facebook, Instagram, Instamojo 743


Q. Which state CM has inaugurated one of India’s longest Skywalk bridges, which spans 570 meters in
length and 4.2 meters in width?
A) Karnataka B) Madhya Pradesh
C) Uttar Pradesh D) Tamil Nadu
Answer : D
✓ Tamil Nadu Chief Minister MK Stalin has inaugurated a 570-meter long and 4.2-meter-wide skywalk
bridge connecting the Mambalam railway station and T Nagar bus terminus.
✓ The multi-modal project connects the railway station and bus stand. It will help pedestrians.
✓ The bridge was constructed by the Greater Chennai Corporation at a cost of Rs 28.45 crore from Smart
City funds.

Q. When is the United Nations recognised International Tea Day observed?


A) May 20 B) May 19
C) May 21 D) May 18
Answer : C
✓ The United Nations recognised International Tea Day is observed annually on May 21 by the UN Food
and Agriculture Organization (FAO) to raise awareness of the deep cultural and economic significance
of tea around the world and promote its importance in fighting hunger and poverty.
✓ The day was adopted by United Nations on December 21, 2019.
✓ DON'T CONFUSED
✓ It must be noted that the United Nations recognised International Tea Day is observed on 21 May.
✓ The International Tea Day is observed annually on December 15 since 2005 in tea producing countries
like India, Bangladesh, Sri Lanka, Nepal, Vietnam, Indonesia, Kenya, Malawi, Malaysia, Tanzania and
Uganda.
✓ Assam is the largest producer of tea in India.
✓ China is the largest producer of tea in the world.
✓ India is the second largest producer
Q. Amy Pope has been selected as the first female Director-General of International Organization for
Migration (IOM). She is from which country?
A) Denmark
B) Sweden
C) United States
D) United Kingdom
Answer : C
✓ Amy Pope of the United States has been selected as the first female Director-General of the United
Nations Migration Agency.
✓ She will head the Geneva-based International Organization for Migration (IOM).
✓ Pope is currently serving as deputy to Director General Antonio Vitorino.
✓ Her five-year term as director general will begin on October 1, replacing Vitorino
✓ International Organization for Migration (IOM)
✓ It is an agency of the United Nations and provides services and advice related to migration.
✓ Formation - 6 December 1951
✓ Headquarters - Geneva, Switzerland.
✓ Membership (2023) - 175 member states and 8 observer states.

Follow us: Official Site, Telegram, Facebook, Instagram, Instamojo 744


Q. Who has becomes the first state in the country to approve Good Governance Regulations?
A) Rajasthan B) Maharashtra
C) Gujarat D) Kerala
Answer : B
✓ Maharashtra Chief Minister Eknath Shinde approved the first Good Governance Regulations in the
country.
✓ The purpose of the rules is to enhance accountability, accessibility, dynamism and transparency in state
administration.
Q. Which country will host the 44th edition of the annual ISO COPOLCO Plenary ?
A) Japan B) Russia
C) India D) Germany
Answer : C
✓ India will host the 44th edition of the annual ISO COPOLCO Plenary from May 23-26, 2023, by the
Ministry of Consumer Affairs, Food and Public Distribution.
✓ The event to be held in New Delhi will be inaugurated by Commerce and Industry Minister Piyush
Goyal.
✓ COPOLCO President Sadie Denton, ISO Secretary General Sergio Mujica and other high ISO officials
will also attend the event.
✓ Organized by the Bureau of Indian Standards (BIS), the grand event will include interactive sessions and
workshops on relevant topics, apart from various activities aimed at enhancing consumer engagement in
the standardization process.
Q. Which state cabinet has approved a financial proposal of Rs 106.46 crore for developing Geographic
Information System (GIS) based 3D models of 4 cities?
A) Uttar Pradesh B) Uttarakhand
C) Rajasthan D) Madhya Pradesh
Answer : C
✓ The state cabinet of Rajasthan chaired by Chief Minister Ashok Gehlot approved a financial proposal of
Rs 106.46 crore for developing Geographic Information System (GIS) based 3D models of 4 cities in the
state namely Jodhpur, Udaipur, Kota and Ajmer.
✓ The state government will develop GIS-based 3D models of the four cities for better planning and
management of urban development.
✓ The financial approval has been made for the implementation of GIS-based 3D city and Rajdhara
Satellite Imagery Repository projects in these four cities.
Q. Who is the author of the book titled “Partitioned Freedom”?
A) Ram Madhav B) Umesh Lalit
C) Rajesh Verma D) Rajiv Dixit
Answer : A
✓ Gulab Chand Kataria, Governor of Assam, released a book titled “Partitioned Freedom”, which was
authored by Ram Madhav and published by Prabhat Prakashan Pvt. Ltd, in Guwahati, Assam.
✓ The book “Partitioned Freedom” offers profound insights into the events that unfolded during 1905 and
the pre-independence era.

Follow us: Official Site, Telegram, Facebook, Instagram, Instamojo 745


✓ Ram Madhav is an Indian Politician, Author, and thinker. His other books are “Uneasy Neighbours:
India and China After 50 Years of the War” and “Asahaj Padosi”.
Q. Which IIT has signed a Letter of Intent (LoI) with Israel to build an ‘India-Israel Center of Water
Technology’ (CoWT) in Water Resources Management and Water Technologies?
A) IIT Kanpur B) IIT Bombay
C) IIT Madras D) IIT Kharagpur
Answer : C
✓ Indian Institute of Technology (IIT) Madras has signed a Letter of Intent (LoI) with Israel to build an
‘India-Israel Center of Water Technology’ (CoWT) in Water Resources Management and Water
Technologies.
Q. Elena Rybakina has won the women’s single title at the Italian Open 2023 Tennis tournament. She
represents which country?
A) Russia B) Poland
C) Spain D) Kazakhstan
Answer : D
✓ Italian Open 2023: Daniil Medvedev wins Men’s Singles title; Elena Rybakina wins Women’s singles
title.
✓ Russian tennis player Daniil Medvedev won the Men’s Singles title at the 2023 Italian Open (also known
as the Rome Masters or the Internazionali BNL d’Italia) by defeating Holger Rune of Denmark in the
final. The 2023 Italian Open was from 2nd to 21st May 2023 at Foro Italico in Rome, Italy.
✓ This marks Daniil Medvedev’s first-ever Association of Tennis Professionals (ATP) tile on the clay court
of his career and his first-ever Italian Open title.
✓ With this win, Daniil Medvedev ascended to world number 2 in the ATP rankings.
✓ Women’s Singles:
✓ The 2022 Wimbledon champion Elena Rybakina won the Women’s singles title at the Italian Open 2023
by defeating Anhelina Kalinina of Ukraine. This marks her first major title on clay.
✓ Italian Open 2023:
✓ Italian Open 2023 is the 80th edition of the Internazionali BNL d’Italia
Q. Which player has won the men’s singles title at the Italian Open 2023 Tennis Tournament?
A) Daniil Medvedev B) Casper Ruud
C) Stefanos Tsitsipas D) Carlos Alcaraz
Answer : A
✓ Russian tennis player Daniil Medvedev won the Men’s Singles title at the 2023 Italian Open (also known
as the Rome Masters or the Internazionali BNL d’Italia) by defeating Holger Rune of Denmark in the
final

Q. PM Narendra Modi has addressed the 76th Session of the World Health Assembly in which country?
A) Switzerland B) Sweden
C) Norway D) Denmark
Answer : A

Follow us: Official Site, Telegram, Facebook, Instagram, Instamojo 746


✓ The Prime Minister, Narendra Modi has addressed the 76th Session of the World Health Assembly in
Geneva, Switzerland.
✓ India shipped almost 300 million doses of COVID-19 vaccines to over 100 countries including many
countries from the Global South.
✓ India’s commitment to international cooperation and informed that the country shipped almost 300
million doses of covid-19 vaccines to over 100 countries including many countries from the Global South.
✓ India’s vision for good health is ‘One Earth One Health’, which extends to the entir.

Q. Which Countries are signed agreement in the field of Green Hydrogen?


A) India and United Kingdom B) India and Switzerland
C) India and New Zealand D) India and Australia
Answer : D
✓ The Memorandum of Understanding (MoU) was exchanged after a bilateral meeting between Prime
Minister Narendra Modi and his Australian counterpart Anthony Albanese in Sydney.
✓ Prime Minister Modi invited Prime Minister Albanese and Australian cricket fans to India for the Cricket
World Cup.
✓ Prime Minister Albanese announced the establishment of a new Australian Consulate General in
Bengaluru to link Australian businesses to India's digital and innovation ecosystem.

Q. Which Company launched 'GAINS' 2023 Startup Challenge in Kolkata?


A) HSL B) GRSE
C) BEML D) MIDHANI
Answer : B
✓ Garden Reach Shipbuilders and Engineers (GRSE) Ltd launched GRSE Accelerated Innovation
Nurturing Scheme 2023 or ‘GAINS 2023’ in Kolkata, West Bengal (WB) to utilize the eco-system
that tackles current and future challenges in the ship design and construction sector.
✓ GAINS 2023 is a part of GRSE’s drive to identify and promote the development of innovative
solutions towards technological advances in shipbuilding by Startups.

Q. Which government has formulated the 'Green Hydrogen' policy to promote the use of green hydrogen?
A) Assam B) Himachal Pradesh
C) Meghalaya D) West Bengal
Answer : B
✓ Himachal Pradesh Chief Minister Sukhvinder Singh Sukhu has ‘Green Hydrogen’ policy would be
formulated to promote the use of green hydrogen and establish the state as a leading hub for its
production.
✓ The state’s abundant renewable energy resources, including ample sunlight, water and wind, make
it an ideal location for generating green hydrogen.
Q. Which country has won the 13th Sudirman Cup title defeating South Korea 3-0 at Suzhou?
A) China B) India
C) South Korea D) Denmark

Follow us: Official Site, Telegram, Facebook, Instagram, Instamojo 747


Answer : A
✓ China has claimed a record 13th Sudirman Cup title defeating South Korea 3-0 at Suzhou.
✓ Chen Yufei defeated World Number two An Se Young 21-16,22-20, in the women’s singles to seal
the victory.
✓ In the men’s singles, Korea’s Lee Yun Gyu was easily outmatched by Shi Yuqi.
✓ The tournament is one of badminton’s most important because performances will count towards
qualification for the 2024 Paris Olympics.

Q. Machilipatnam port is located in which state?


A) Kerala B) Gujarat
C) Maharashtra D) Andhra Pradesh
Answer : D
✓ Andhra Pradesh Chief Minister YS Jagan Mohan Reddy formally launched the Machilipatnam port
works worth Rs 5,156 crore recently.
✓ The port, with an initial cargo capacity of 35 million tonnes, will be completed in two years.
✓ The capacity of the 4-berth deep water port will be increased to 116 million tonnes as cargo traffic
gradually increases.
✓ The port will help develop Machilipatnam on the lines of developed port cities like Mumbai and
Chennai as it will be connected by National Highway 216 and Gudivada-Machilipatnam railway
line soon.
✓ About Machilipatnam Port
✓ It is a proposed deep sea port on the Bay of Bengal coast located at Machilipatnam, the district
headquarters of Krishna district in Andhra Pradesh.
✓ The project is being implemented by the Machilipatnam Port Development Corporation Limited
(MPDCL).
✓ The first phase of the port is expected to have a capacity of 35 million tonnes.
Q. Where will be the 3rd khelo India University Games?
A) Madhya Pradesh B) Karnataka
C) Haryana D) Uttar Pradesh
Answer : D
✓ The third Khelo India University Games will begin with a Kabaddi event on May 23 in Gautam
Budh Nagar district of Uttar Pradesh.
✓ 15 teams representing different universities are participating in the Kabaddi competition.
✓ The matches will be held at the Shaheed Vijay Singh Pathik Stadium in Gautam Budh Nagar district.
✓ The official opening ceremony of the Khelo India University Games will take place on May 25 in
Lucknow and will continue till June 3.
✓ The Khelo India University Games will be held in four cities: Gautam Buddha Nagar, Lucknow,
Gorakhpur and Varanasi and the closing ceremony of the Games will be held in Varanasi.
✓ Wrestling and Yogasana (yoga postures) events will be organised in Varanasi during the Khelo India
University Games.
Q. Who has become the 17th world chess champion after defeating Russia’s Ian Nepomniachtchi in a tie-
breaker?
A) Levon Aronian B) Magnus Carlsen
C) Anish Giri D) Ding Liren

Follow us: Official Site, Telegram, Facebook, Instagram, Instamojo 748


Answer : D
✓ Ding Liren became China's first world chess champion after defeating Russia's Ian Nepomniachtchi
in a tie-breaker.
✓ Ding Liren has become the 17th World Chess Champion.

Q. When is the World Press Freedom Day commemorated all over the world?
A) 01 May B) 02 May
C) 04 May D) 03 May
Answer : D
✓ World Press Freedom Day is observed annually on 3 May to increase understanding about the
importance of a free press.
✓ This year's theme is "Shaping a Future of Rights: Freedom of Expression as a Driver for All Other
Human Rights".
✓ World Press Freedom Day was proposed by UNESCO's General Conference in 1991 to celebrate
and promote press freedom.
✓ World Press Freedom Day also provides an opportunity to honour journalists who have lost their
lives in the line of duty.

Q. India's first undersea tunnels are being completed in which city, likely to open in November 2023?
A) Mumbai B) Kochi
C) Chennai D) Ahmedabad
Answer : A
✓ India’s First Undersea Tunnel in Mumbai Set to Open by November 2023.
✓ The tunnels begin close to Girgaon (before Marine Drive), travel under the Arabian Sea, Girgaon
Chowpatty, and Malabar Hill, and finally terminate at Breach Candy’s Priyadarshini Park.
✓ The high-speed coastal route aims to reduce the 45-minute peak-time trip from Girgaon to Worli to
only 10 minutes.
✓ The diameter of the tunnels is 12.19 meters which is 17-20 meters below sea level. About 1 kilometer
part is under the sea.
✓ The tunnels reach a depth of 72 meters in Malabar Hill.
✓ Construction of tunnels is a part of the Mumbai Coastal Road Project (MCRP) being built by the
Brihanmumbai Municipal Corporation (BMC).
Q. Who has become the first Indian to complete the prestigious Golden Globe Race 2022?
A) Armaan Ebrahim B) Akhil Rabindra
C) Abhilash Tomy D) Karun Chandhok
Answer : C
✓ Abhilash Tomy becomes the first Indian to complete Golden Globe Race 2022.
✓ He finished second in the race, a solo around-the-world sailing race.
✓ The race started on September 4, 2022, from Les Sables-d’Olonne in France, with 16 sailors from 11
countries.

Follow us: Official Site, Telegram, Facebook, Instagram, Instamojo 749


✓ Only two of them managed to complete the gruelling challenge while sailing non-stop. This tested
their physical and mental stamina to the limit.
✓ The only other finisher so far has been the sailor Simon Curwen who had been forced to make stops
to address problems with his boat, unlike Abhilash Tomy and Kirsten Neuschafer who completed
the race non-stop

Q. Who is the author of book titled ‘Collective Spirit, Concrete Action’?


A) Vikram Seth B) Anita Desai
C) Chetan Bhagat D) Shashi Shekhar Vempati
Answer : D
✓ A book called ‘Collective Spirit, Concrete Action’ written by Shashi Shekhar Vempati, former CEO
of Prasar Bharati (2017-2022) was launched.
✓ The book was released during a day-long National Conclave on Mann Ki Baat @100 held in the
New Delhi.
✓ According to the author, ‘Mann Ki Baat’ is a combination of the power of radio and the vision of a
nation’s leader.
Q. Three jailed women journalists Nilofar Hamidi, Elahe Mohammadi and Nargis Mohammadi were given
the UNESCO/Guillermo Cano World Press Freedom Prize 2023. These three women are from which
country?
A) Saudi Arabia B) Iran
C) Iraq D) United Arab Emirates
Answer : B
✓ Three imprisoned Iranian women journalists awarded 2023 UNESCO/Guillermo Cano World
Press Freedom Prize.
✓ Award winners include Niloufer Hamidi, Elaheh Mohammadi and Nargis Mohammadi.
✓ Neelofer had informed people about 22-year-old Mahsa Amini, who had died in September last year,
through a news item.
✓ Amini was arrested by the ethics police for not wearing a headscarf properly.
✓ Elaheh wrote about Amini's funeral.
✓ Amini's death triggered months of protests in Iran's cities.
✓ The third winner, Nargis Mohammadi, worked for many years as a journalist and is one of Iran's
most prominent activists.
✓ UNESCO World Press Freedom Prize.
✓ This award was started in 1997.
✓ The award is given to a person, organization or institution who, at the risk of life, has made an
outstanding contribution to the defense and, or promotion of freedom of the press, anywhere in the
world.
✓ The award was established at the initiative of the Executive Board of UNESCO.
✓ The award is formally presented by UNESCO on the occasion of World Press Freedom Day on 3
May.
✓ The award is named in honor of Guillermo Cano Isaza, a Colombian journalist who was
assassinated on 17 December 1986 in front of the office of his newspaper El Espectador in Bogotá,
Colombia.
Q. The World Press Freedom Prize is awarded annually by__
A) UNESCO B) World Bank

Follow us: Official Site, Telegram, Facebook, Instagram, Instamojo 750


C) International Monetary Fund D) UNICEF
Answer : A
Q. Finance Minister Nirmala Sitharaman attends the 56th Annual General Meeting of the Board of
Directors of the Asian Development Bank (ADB) in which country?
A) Saudi Arabia B) United Arab Emirates
C) South Korea D) Japan
Answer : C
✓ Finance Minister Nirmala Sitharaman attends the 56th Annual General Meeting of the Board of
Directors of the Asian Development Bank (ADB) in South Korea.
✓ The meetings are attended by official delegations of ADB members, observers, non-governmental
and civil society organizations, financial institutions and banks, and other private sector companies.
✓ During the four-day visit, Sitharaman will also participate in annual meeting focal events like the
Governors’ Business, and also as a panelist in the ADB Governors’ Seminar on Policies to Support
Asia’s Rebound.
✓ SOUTH KOREA
✓ Capital : Seoul
✓ Currency : Won
✓ President : Yoon Suk-yeol
✓ Prime Minister : Kim Boo-kyum

Q. Name the player who has won the 2023 World Snooker Championship?
A) Luca Brecel B) Ricky Walden
C) Mark Selby D) Robert Milkins
Answer : A
✓ Belgium’s Luca Brecel has become the first European player to win 2023 World Snooker
Championship. This was his first world title.
✓ He defeated Mark Selby of England with an 18-15 win.
✓ The tournament was held from 15 April to 1 May 2023 at the Crucible Theatre in Sheffield, England
Q. Name the recruitment portal launched by the University Grant Commission (UGC) for recruitment of
the faculties in Central Universities?
A) CU-Niyukt B) CU-Chayan
C) CU-Adhikar D) CU-Sweekar
Answer : B
✓ The University Grant Commission (UGC) Chairman M Jagadesh Kumar launched the Central
University Faculty recruitment portal CU-Chayan, which is completely user-friendly.
✓ CU-Chayan is a unified recruitment portal, designed and developed specifically for recruitment of
the faculties in Central Universities.
✓ CU-Chosen is an integrated recruitment portal, specially designed and developed for faculty
recruitment in Central Universities.
✓ The portal is completely user friendly and caters to the needs of all the stakeholders in the recruitment
process.

Follow us: Official Site, Telegram, Facebook, Instagram, Instamojo 751


✓ UGC has developed this portal to create an enabling environment for both universities and
applicants.
✓ The portal was launched on May 2, 2023 by Professor Mamidala Jagadesh Kumar,Chairman, UGC.
Q. The Ministry of Defence, in the Government of India, has recently handed over Fast Patrol Vessel and a
Landing Craft Assault ship, to which of the following countries?
A) Maldives B) Russia
C) Ukraine D) Bangladesh
Answer : A
✓ Defence Minister Shri Rajnath Singh handed over two made-in-India platforms, namely, Fast Patrol
Vessel and a Landing Craft Assault ship, to the Maldives National Defence Forces (MNDF) 0n May
02, 2023.
✓ Minister Rajnath Singh is on 3-day visit to the Maldives from May 01 to 03, 2023.
✓ The Fast Patrol Vessel was commissioned as MNDF Coast Guard ship Huravee. It is capable of
coastal and offshore surveillance at high speeds.
✓ The handing over of the two vessels is a symbol of the shared commitment of India and the Maldives
towards peace and security in the Indian Ocean Region (IOR).
✓ About Maldives
✓ It is also called Maldives Islands, an independent island country in the north-central Indian Ocean.
✓ Basis of economy -fisheries, tourism
✓ Industries - handicraft or cottage including the making of coir (coconut-husk fibre) and coir products,
fish canning, and boatbuilding.
✓ Capital -Male
✓ President -Ibrahim Mohamed Solih
✓ Currency -Rufiyaa
Q. What is the new deadline set by EPFO for employees to submit applications regarding higher pension?
A) June 26 B) May 03
C) July 01 D) May 31
Answer : A
✓ The retirement fund body Employees’ Provident Fund Organisation (EPFO) has extended the
deadline to apply for pension on higher wages till June 26, 2023.
✓ The decision has been taken to provide a larger window of opportunity and to enable all eligible
persons to file their applications.
✓ The initial deadline of March 3 was extended till May 3, 2023.
✓ The Central Board of Trustees (CBT) of the Employees’ Provident Fund Organisation (EPFO) has
raised the interest rate on the provident fund (PF) deposits marginally to 8.15 percent for FY23.
✓ Employees’ Provident Fund Organisation (EPFO) launched “Nidhi Aapke Nikat 2.0”- A District
Outreach Program for expanding its presence in more than 685 districts of the country
Q. The Ministry of Tourism participated in Arabian Travel Market (ATM) 2023, with the theme Incredible
India. The event was organised at which place?
A) Doha B) Cairo
C) Dubai D) Sharjah
Answer : C

Follow us: Official Site, Telegram, Facebook, Instagram, Instamojo 752


✓ The Ministry of Tourism, Government of India, is participating in the Arabian Travel Market
(ATM) 2023, being held in Dubai, UAE from May 01 to 04, 2023.
✓ The ATM 2023 is a leading global events for the travel and tourism industry, attracting visitors and
exhibitors from around the world.
✓ The Indian pavilion is participation in the ATM 2023 event with the theme “Incredible India”. It
shall showcase India’s rich cultural and natural heritage, including its UNESCO World Heritage
Sites, wildlife sanctuaries, and spiritual destinations.
✓ India has also launched Visit India Year 2023 campaign at the event.
✓ With this, India aims to present itself as a preferred travel destination among the Middle East and
North Africa (MENA) markets.
Q. Bharti Airtel has planned to merge its which overseas business operations with Dialog Axiata to become
the largest telecom provider in that country?
A) Bangladesh B) Maldives
C) Malaysia D) Sri Lanka
Answer : D
✓ The telecom major Bharti Airtel has announced its plan to merge its Sri Lanka operations with
Dialog Axiata, the largest telecom provider in Sri Lanka.
✓ The merged entity will become the largest telecom provider in Sri Lanka.
✓ Under the pact Bharti Airtel will be granted a stake in Dialog, representing fair value of Airtel Lanka.
Bharti Airtel will accordingly be issued new shares in Dialog upon completion of the deal.
✓ Bharti Airtel
✓ Founded : 1995
✓ Founder : Sunil Bharti Mittal
✓ Headquarters : New Delhi
✓ Chairman : Sunil Bharti Mittal
✓ MD & CEO : Gopal Vittal.

Q. Sharad Pawar has stepped down as the National President of which political party?
A) Bharatiya Janata Party (BJP) B) Nationalist Congress Party (NCP)
C) Indian National Congress (INC) D) Communist Party of India (Marxist)
Answer : B
✓ The Nationalist Congress Party (NCP) President Sharad Pawar has stepped down from his position
on May 02, 2023.
✓ The noted politician made the announcement of his retirement during the release of his
autobiography titled, ‘Lok Majhe Sangati – Political Autobiography’.
✓ He was the first and former president of the Nationalist Congress Party (NCP), which was founded
in 1999, after separating from the Indian National Congress.
✓ Apart from this, Pawar has served as the Chief Minister of Maharashtra for four terms.
✓ Pawar has also served as the Minister of Defence in the Cabinet of P.V Narsimha Rao and Minister
of Agriculture in the Cabinet of Manmohan Singh.

Q. Who is the author of the autobiography titled ‘Lok Majhe Sangati – Political Autobiography’?
A) Uddhav Thackeray B) Sharad Pawar
C) Devendra Fadnavis D) Eknath Shine
Answer : B

Follow us: Official Site, Telegram, Facebook, Instagram, Instamojo 753


✓ The noted politician made the announcement of his retirement during the release of his
autobiography titled, ‘Lok Majhe Sangati – Political Autobiography’.
AUTOBIOGRAPHY IN NEWS 2023
▪ Sharad Pawar autobiography ‘Lok Maze Sangati’ (Lok Maze Sangati) .
▪ Tennis player Jaideep Mukherjee launches his autobiography "Crosscourt".
▪ Ghulam Nabi Azad’s Autobiography ‘Azaad’ .
▪ K.K. Abdul Gaffar’s autobiography, ‘Njaan Sakshi’ (me as the witness).
Q. Finance Minister Nirmala Sitharaman recently launched a book titled ‘Reflections’. Who is the author
of this book?
A) Subramaniam Ramadorai B) Keki Dadiseth
C) Narayanan Vaghul D) Arundhati Bhattacharya
Answer : C
✓ Union Minister of Finance and Corporate Affairs Smt. Nirmala Sitharaman launched a book titled
‘Reflections’, in Mumbai recently.
✓ The book has been written by eminent banker Shri Narayanan Vaghul, who is widely considered as the
architect of modern banking in India..
✓ Author Shri Vaghul has complied his decades-long experiences in India’s financial landscape in this book.
✓ Narayanan Vaghul, Former Chairman ICICI Bank and Author of ‘Reflection’ about the Banking System
in India.
Q. The Indian Institute of Foreign Trade (IIFT), New Delhi celebrated its which Foundation Day in 2023?
A) 60th B) 90th
C) 50th D) 75th
Answer : A
✓ The Indian Institute of Foreign Trade (IIFT), New Delhi celebrated its Diamond Jubilee, means, 60th
Foundation Day on May 02, 2023.
✓ Established in 1963, IIFT is an autonomous body under the Ministry of Commerce and Industry.
✓ It is a leading business school in India with its main campus in New Delhi and additional campuses in
Kolkata and Kakinada.
✓ It also functions as a civil services training institute
✓ Late Shri HD Shourie was the founding Director General, who laid the foundation of IIFT with the
motto, ‘Excellence is a way of life’.
✓ Indian Institute of Foreign Trade (IIFT).
✓ The Indian Institute of Foreign Trade (IIFT) was established in 1963 as an autonomous body under the
Union Ministry of Commerce & Industry to contribute in the skill building for the external trade sector
of India.
✓ It is one of the top business schools of India with a special focus on Trade and Finance.
✓ The Institute was granted “Deemed to be University” status in 2002.
Q. Telangana state has introduced an insurance scheme for toddy tappers named ‘Geetha Karmikula
Bhima’. What amount of insurance cover will be provided under this scheme?
A) Rs 10 lakh B) Rs 3 lakh
C) Rs 2 lakh D) Rs 5 lakh
Answer : D

Follow us: Official Site, Telegram, Facebook, Instagram, Instamojo 754


✓ The state government of Telangana, led by Chief Minister K Chandrasekhar Rao has planned to
introduce a new insurance scheme for toddy tappers, under the name ‘Geetha Karmikula Bhima’.
✓ AS per the scheme, an insurance money of Rs 5 lakh would be deposited directly in the bank account of
the respective family members in case any toddy tapper dies accidentally while collecting toddy from
palm trees in the fields.
✓ The insurance amount would be disbursed within a week.
Q. India’s first Rafale lady pilot Shivangi Singh participating in Orion Military Exercise 2023 belongs to
which state?
A) Andhra Pradesh
B) Bihar
C) Kerala
D) Rajasthan
Answer : B
✓ Lieutenant Shivangi Singh is the first woman Rafale pilot to take part in Orion Exercise 2023.
✓ She hails from Fatehabad, Bihar and is considered a major fighter serving in the Indian Navy.
✓ The fourth and final phase of the Orion Military Exercise is being held between April 19 to May 05, 2023.
✓ This is a multilateral international exercise of the Indian Air Force..
Q. India has signed an MoU with which country on Industrial Research and Development Cooperation
with focus on several key technology areas
A) France B) South Korea
C) Israel D) Germany
Answer : C
✓ India and Israel signed a Memorandum of Understanding (MoU) to enhance industrial research and
development cooperation in high-tech areas such as AI, quantum, semiconductors, healthcare,
aerospace, sustainable energy and agriculture.
✓ The partnership aims to facilitate collaborative initiatives for industrial research and development in a
diverse range of high-tech domains, including artificial intelligence, quantum computing, semiconductor
technology, synthetic biology, healthcare, aerospace, sustainable energy and agriculture.
✓ The MoU will focus on specific projects and will be monitored by a Joint Steering Committee to ensure
effective implementation.
✓ Memorandum of Understanding (MOU):
✓ It is a type of agreement between two or more parties that expresses a shared intention to take a common
line of action.
✓ Memoranda of Understanding can be used in situations where the parties do not intend a legal
commitment or cannot form a legally enforceable agreement.
✓ About Israel
✓ Israel is a Middle Eastern country on the Mediterranean Sea.
✓ It is considered the Holy Land of the Bible by Jews, Christians and Muslims.
✓ Tel Aviv is the financial centre of the country and is known for its beaches and Bauhaus architecture.
✓ Capital and largest city - Jerusalem
✓ Official language - Hebrew
✓ Recognized Language - Arabic
✓ Government - Unitary parliamentary republic
✓ President - Isaac Herzog
✓ Prime Minister - Benjamin Netanyahu

Follow us: Official Site, Telegram, Facebook, Instagram, Instamojo 755


✓ India and Israel sign MoU for cooperation on industrial research and development.
✓ India and the United Kingdom have signed a Memorandum of Understanding (MOU) for collaboration
on science and innovation.
✓ United States and India will sign a memorandum of understanding on semiconductors.
✓ India-Armenia sign MoU on High Impact Community Development Projects.
✓ India and the United Arab Emirates (UAE) signed a Memorandum of Understanding (MoU) on Climate
Action.
✓ India, Mexico ink MoU on research, technology and innovation collaborations.
✓ India, Panama sign agreement on cooperation in training diplomats.
✓ India and Fiji sign MoU on visa exemption for holders of diplomatic and official passports.
✓ Cabinet Approves Signing of the MoU between the India and South Africa for cooperation in Disability
Sector.
✓ RBI, Central Bank of UAE sign MoU to promote innovation in financial products and services.
✓ India and Egypt signed an MOU to facilitate content exchange, capacity building, and Co-Productions
between Prasar Bharati and the National Media Authority of Egypt.
✓ India signs MoU with Indonesia-Malaysia-Thailand Growth Triangle Joint Business Council to promote
adoption of energy efficiency.
Q. Which country has named the cyclonic storm 'Cyclone Mocha' in the Bay of Bengal?
A) Yemen B) Sri Lanka
C) Indonesia D) Bangladesh
Answer : A
✓ The India Meteorological Department (IMD) has predicted a cyclonic storm 'Cyclone Mocha' in the Bay
of Bengal and adjoining areas which is expected to hit the east coast between May 7 and May 9.
✓ The cyclonic storm has been named Mocha which is given by Yemen.
✓ The name is inspired by the name of Mocha (or Mokha), a Yemeni city on the Red Sea coast

Q. India’s domestic coal production has risen by how much percent during the last five years?
A) 22% B) 25%
C) 16% D) 31%
Answer : A
✓ India’s domestic coal production rises by over 22 % during last 5 years.
✓ The country’s overall coal production has seen a quantum jump to 893.08 million tonnes in the Financial
Year 2022-23 as compared to 728.72 million tonnes in the Financial Year 2018-2019 with a growth of
about 22.6 percent.
✓ The priority of the Ministry is to enhance domestic coal production to reduce the dependence on
substitutable coal imports.
✓ The production of Coal India Limited (CIL) has increased by 703.21 million tonnes as compared to
606.89 million tonnes in the Financial Year 2018-2019 with a growth of 15.9 percent in the last five years

Q. Indian Army appointed five women officers to which division of military forces?
A) Armoured Regiment B) Artillery Regiment
C) Infantry Regiment D) Mechanised Regiment
Answer : B
✓ Indian Army inducts first batch of “Five Women Officers” into the Regiment of Artillery.

Follow us: Official Site, Telegram, Facebook, Instagram, Instamojo 756


✓ For the first time ever, five women cadets have been inducted into the Indian Army’s Regiment of
Artillery, on April 29, 2023, during their passing out parade at the Officers Training Academy (OTA),
Chennai.
✓ Lt Mehak Saini, Lt Sakshi Dubey, Lt Aditi Yadav, Lt Pious Mudgil and Lt Akanksha have been
commissioned into Army’s premier artillery units after successfully completing their training at the OTA,
Chennai.
✓ Out of the five women officers, three have been posted to units deployed along the borders with China
(Line of Actual Control (LAC)).
✓ The other two have been deployed in “challenging locations” near the frontier with Pakistan.
✓ These OTA, Chennai women cadets will receive the same opportunities and challenges as their male
counterparts.
Q. When is the International Workers’ Day commemorated?
A) 2 May B) 1 May
C) 29 April D) 30 April
Answer : B
✓ The International Workers’ Day is observed annually on May 01 to honour the contributions of the
labourers or working class people.
✓ This day is also known as Workers’ Day, Labour Day as well as May Day in some of the countries.
✓ India celebrated the first Labour day in the year 1923.
✓ In India, the day is referred to as Antarrashtriya Shramik Diwas or Kamgar Din
Q. Zafar Kizilkaya has been awarded the 2023 Goldman Environment Prize for the Asia region. Which
country is he from?
A) Turkey B) Thailand
C) Sweden D) Norway
Answer : A
✓ The 34th edition of the prestigious Goldman Environmental Prize was announced recently, to honor one
grassroot environmental activist from each of the world’s six geographic regions: Africa, Asia, Europe,
Islands and Island Nations, North America, and South and Central America.
✓ Region-wise winner of the Goldman Environmental Prize 2023.
1. Islands and Island Nations – Delima Silalahi from Indonesia,
2. Africa – Chilekwa Mumba from Zambia,
3. South & Central America – Alessandra Korap Munduruku from Brazil,
4. North America – Diane Wilson from United States,
5. Europe – Tero Mustonen from Finland,
6. Asia – Zafer Kizilkaya from Turkey
✓ The Goldman Environmental Prize is given annually by the Goldman Environmental Foundation
headquartered in San Francisco, California.
✓ The award is dubbed as the “Green Nobel Prize.
✓ Asia region winner Paul is the co-founder of the Karen Environmental and Social Action Network
(KESAN), which was established in 2001, which supported Karen indigenous communities in
establishing the Salween Peace Park in Myanmar and on the border of Thailand, to conserve the
biodiversity.
Q. Which Indian state organised the ‘Food Conclave 2023’ brainstorming event?
A) Karnataka B) Telangana

Follow us: Official Site, Telegram, Facebook, Instagram, Instamojo 757


C) Tamil Nadu D) Kerala
Answer : B
✓ The first edition of the Food Conclave-2023 was hosted by the Telangana government on April 28 and
29, 2023 at Hyderabad.
✓ The conclave was inaugurated by the state IT and Industries Minister KT Rama Rao. The state
government also announced that Food Conclave will become an annual event from now.
✓ As a part of this conclave, the Telangana Agriculture Minister S Niranjan Reddy launched a Grand
Challenge for Innovation to promote innovation in the food processing sector.
Q. Who has become the first Indian men’s doubles pair to win a gold medal at the Badminton Asia
Championships?
A) Lakshya Sen and Chirag Shetty
B) Srikanth Kidambi and Chirag Shetty
C) Satwik SaiRaj Rankireddy and Chirag Shetty
D) Srikanth Kidambi and Lakshya Sen
Answer : C
✓ Satwik SaiRaj Rankireddy and Chirag Shetty becomes first Indian Men’s Doubles pair to win Badminton
Asia Championships Title.
✓ In the Badminton Asia Championship 2023, the Indian pair of Chirag Shetty and Satwiksairaj
Rankireddy won the men’s doubles title, held in Dubai on April 30, 2023.
✓ They defeated eighth-seeded Malaysian pair Teo Ee Yi and Ong Yew Sin in the final, 16-21, 21-17, and
21-19.
✓ With this win, the duo became the first Indian men’s doubles pair to win the Badminton Asia
Championships title.
✓ Also, this was India’s second-ever Badminton Asia Championships title after Dinesh Khanna in 1965.
✓ Winner’s List
✓ Men’s singles – Anthony Sinisuka Ginting (Indonesia)
✓ Women’s singles – Tai Tzu-ying (Taiwan)
✓ Men’s doubles – Chirag Shetty and Satwiksairaj Rankireddy
✓ Women’s doubles – Yuki Fukushima and Sayaka Hirota (Japan)
✓ Mixed doubles – Jiang Zhenbang and Wei Yaxin (China)
Q. Tai Tzu-ying who won women’s single title at the 2023 Badminton Asia Championships belongs to which
country?
A) Japan B) China
C) Taiwan D) Indonesia
Answer : C
✓ Women’s singles – Tai Tzu-ying (Taiwan)
Q. Who has become first Indian to Finish on Podium in British F4 Championship?
A) Nikhil Bohra B) Jaden Pariat
C) Karun Chandhok D) Alisha Abdullah
Answer : B
✓ Jaden Pariat Becomes First Indian to Finish on Podium in British F4 Championship

Follow us: Official Site, Telegram, Facebook, Instagram, Instamojo 758


✓ Young Indian Racer Jaden Pariat, made the country proud by finishing on podium in the first round of
ROKit British F4 Championship at Donington Park in Leicestershire.
✓ The 16-year-old Meghalayan racer Pariat is hired for Argenti Motorsport.
✓ He is only the second Indian to get an international podium in a Tatuus F4 car after Kush Maini in 2017.
Q. India is going to host the 23rd edition of the Shanghai Cooperation Organization (SCO) summit in New
Delhi. What is the theme of this summit?
A) SECURE-SCO B) Cohesive and Responsive
C) Realizing Opportunities D) Shared Security and Innovative Growth
Answer : A
✓ India is set to host the 23rd edition of the Shanghai Cooperation Organisation (SCO) Summit on July 3-
4, 2023 in New Delhi.
✓ The theme of India’s Chairmanship of SCO in 2023 is ‘SECURE-SCO’.
✓ The 22nd edition of the SCO Summit was held in September 2022 in Samarkand, Uzbekistan.
✓ The SCO is an intergovernmental organization established in 2001. The members of the bloc includes
India, Kazakhstan, China, Kyrgyzstan, Pakistan, Russia, Tajikistan and Uzbekistan.
✓ Shanghai Cooperation Organisation (SCO)
Founded : 2001
HQ : Beijing, China
Secretary General : Zhang Ming (China)
Member Countries = 9 ( Iran ,China, India, Kazakhstan, Kyrgyzstan,Pakistan,Russia, Tajikistan &
Uzbekistan)
✓ SCO Summit 2021 : Dushanbe,Tajikistan.
✓ SCO Summit 2022 : Samarkand, Uzbekistan.
✓ SCO Summit to be hosted by the Indian side in 2023.
✓ The founding members (5) of Shanghai Cooperation Organisation (SCO) were China, Russia,
Kazakhstan, Kyrgyzstan,Tajikistan. India and Pakistan officially joined this organisation in 2017
✓ Trick to remember SCO Countries.
✓ TRICK :- PICK IN KURTA
✓ P - Pakistan
✓ I - India
✓ C- China
✓ K - Kazakhstan
✓ IN - Iran
✓ K - Kyrgyzstan
✓ U - Uzbekistan
✓ R - Russia
✓ TA - Tajikistan
Q. India will join the International Civil Aviation Organization (ICAO) Carbon Offsetting and Reduction
Scheme for International Aviation (CORSIA) and Long-Term Aspirational Goals (LTAG) from which
year?
A) 2025 B) 2027
C) 2026 D) 2030
Answer : B
✓ India to join international climate action in civil aviation from 2027.
✓ India will join the International Civil Aviation Organization (ICAO) Carbon Offsetting and Reduction
Scheme for International Aviation (CORSIA) and Long-Term Aspirational Goals (LTAG) from 2027.

Follow us: Official Site, Telegram, Facebook, Instagram, Instamojo 759


✓ ICAO has adopted several key ambitious targets, including a two percent annual fuel efficiency
improvement by 2050, carbon neutral development, and net zero by 2050, all covered under CORSIA
and LTAG.
✓ India's Ministry of Civil Aviation stated that joining these measures from 2027 would help airlines from
developing countries like India to achieve greater growth and avoid adverse financial consequences due
to CORSIA
✓ Headquarters -Montreal, Canada

Q. How many new wildlife conservation reserves have been declared in Rajasthan recently?
A) 5 B) 4
C) 3 D) 6
Answer : C
✓ The Forest Department of Rajasthan has announced the creation of three new conservation reserves in
the state to protect the rare and endangered wildlife of Rajasthan.
✓ These three new wildlife conservation reserves are
1. Sorsan in Baran,
2. Khichan in Jodhpur,
3. Hamirgarh in Bhilwara
✓ With the addition of these 3 new reserves, the total number of wildlife conservation reserves in Rajasthan
has gone up to 26.
Q. When is the World Tuna Day observed globally?
A) First Saturday of May B) May 1
C) May 2 D) First Sunday of May
Answer : C
✓ World Tuna Day is observed on May 2 every year to raise awareness about the Tuna fish and its
importance to humans and earth and also to promote more sustainable fishing practices.
✓ The United Nations (UN) officially recognised the World Tuna Day in 2017.
✓ The first World Tuna Day was observed in the year 2017.
✓ Tuna is a saltwater fish comprising of some 15 different species across five genera.

Q. India's exports to the UAE are expected to reach __ by 2026-27


A) $50 billion B) $75 billion
C) $100 billion D) $125 billion
Answer : A
✓ India's exports to the UAE are expected to reach $50 billion by 2026-27 from $31.3 billion at present on
the back of the free trade agreement between the countries.
✓ The India-UAE Comprehensive Economic Partnership Agreement (CEPA) which was signed between
the two nations on 18 February 2022.

UAE IN NEWS 2023


▪ UAE India's second largest export destination and third largest source of imports.
▪ The first conference of I2U2 Business Forum, a joint initiative of India, Israel, the US and the United
Arab Emirates, was held on 22 February 2023 in Abu Dhabi (UAE).

Follow us: Official Site, Telegram, Facebook, Instagram, Instamojo 760


▪ United Arab Emirates (UAE) will host the 13th World Trade Organisation (WTO) Ministerial meeting
in February 2024.
▪ UAE will host the 28th Conference of the Parties to the UN Framework Convention on Climate Change
(COP28) in November 2023.
▪ Asian Infrastructure Investment Bank (AIIB) signed an agreement to set up its first overseas office in Abu
Dhabi Global Market.
▪ UAE and India launched the UAE India Business Council - UAE Chapter (UIBCUC) to strengthen
economic ties and enhance bilateral trade and investment.
▪ EDGE, UAE’s leading defence group signed a Memorandum of Understanding with India’s aerospace
firm Hindustan Aeronautics (HAL) at the International Defence Exhibition and Conference (IDEX).
▪ India, France, UAE Establish Trilateral Cooperation Initiative, in fields including Energy, Defence &
Economy.
▪ India’s Unified Payments Interface (UPI) is shortly to be connected to comparable networks in Indonesia,
Mauritius, and the United Arab Emirates (UAE).
▪ RBI, Central Bank of UAE sign MoU to promote innovation in financial products and services.
▪ Sheikh Mohamed bin Zayed Al Nahyan appointed President of UAE.
▪ Emaar, a Dubai-based real estate developer, has become the first foreign company to invest in a mega-
mall spread over one million square feet in Srinagar.
▪ Indian Air Force contingent of 110 Air Warriors reached Al Dahfra airbase of United Arab Emirates
(UAE) on 25 February for participating in Exercise 'Desert Flag VIII.
▪ President of the United Arab Emirates (UAE), Sheikh Mohamed bin Zayed Al Nahyan, has appointed
Sheikh Mansour bin Zayed Al Nahyan, his brother, as the Vice President of the country.
▪ Agricultural and Process Food Export Development Authority (APEDA) is participating in the 28th
edition of Gulfood 2023 which will be held in UAE.
▪ The ‘Study in India Pavilion’ at the Global Education & Training Exhibition (GETEX) in Dubai, UAE.
▪ HDFC Bank and UAE-based financial services company Lulu Exchange, have partnered to strengthen
cross-border payments between India and Gulf Cooperation Council (GCC) region.
▪ About UAE
▪ Official Language : Arabic
▪ Capital : Abu Dhabi
▪ Currency : Emirati Dirham
▪ President of UAE : Sheikh Khalifa bin Zayed bin Sultan Al Nahyan
▪ Prime Minister: Sheikh Mohammed ibn Rashid Al Maktoum
Q. ASEAN Maritime Exercise 2023 begins today in Harbour Phase, the event is held at which military port?
A) Norfolk Naval Base B) Zapadnaya Litsa Naval Base
C) Washington Navy Yard D) Changi Naval Base
Answer : D
✓ The first ASEAN-India Maritime Exercise (AIME-2023) began on 2 May in Singapore.
✓ The maiden exercise is being conducted off the coast of Singapore.
✓ It aims to enhance interoperability and exchange of best practices among the participating navies.
✓ INS Satpura and INS Delhi, accompanied by RADM Gurcharan Singh, Flag Officer Commanding
Eastern Fleet, have reached Singapore to participate in the exercise.
✓ Indian Navy ships will also participate in the International Maritime Security Conference (IMSC) and
the International Maritime Defense Exhibition (IMDEX-23).
✓ Phase of the exercise
✓ Harbour Phase - to be conducted at Changi Naval Base.
✓ Sea Phase - to be conducted in the South China Sea.

Q. Who will be honored at the BAFTA (British Academy of Film and Television Arts) Television Awards?

Follow us: Official Site, Telegram, Facebook, Instagram, Instamojo 761


A) Nina Wadia B) Meera Syal
C) Mandy Takhar D) Rhona Mitra
Answer : B
✓ British-Indian Meera Syal to be awarded BAFTA TV fellowship.
✓ Meera Syal, who starred in the hit BBC comedy, Goodness Gracious Me and The Kumars at No 42, will
be honored at the BAFTA (British Academy of Film and Television Arts) Television Awards on May 14
at the Royal Festival Hall.
✓ It is the highest honor given to an individual for outstanding and extraordinary contribution in the field
of film, sports or television.
✓ Sanya is best known for her performances in 'Goodness Gracious Me' and 'The Kumars at No 42'.

Follow us: Official Site, Telegram, Facebook, Instagram, Instamojo 762


BEST MCQ WITH FACTS
APRIL 2023

Q. Which operation was launched to evacuate Indians from Sudan?


A) Operation Krishna B) Operation Ganga
C) Operation Godavari D) Operation Kaveri
Answer : D
✓ India launched Operation Kaveri to evacuate its nationals from unrest-hit Sudan and it is the latest
evacuation operation launched by India to rescue its citizens and those of friendly nations from war
zones.
✓ Official figures show that there are about 4,000 Indians in Sudan.
✓ Sudan is currently experiencing a violent power struggle between the army and the paramilitary Rapid
Support Force (RSF)across the country.
✓ About Sudan
✓ Capital - Khartoum
✓ Currency - Sudanese pound (SDG)
✓ Region -North Africa
✓ President - Abdel Fatah al-Burhan (Chairman of the Sovereignty Council)
✓ Official language - Arabic, English
Q. Which state launched the ‘State Wide Attention on Grievances by Application of Technology -
SWAGAT’ initiative?
A) Gujarat B) Haryana
C) Madhya Pradesh D) Rajasthan
Answer : A
✓ Prime Minister Narendra Modi virtually participated in a program to mark the completion of 20 years of
the StateWide Attention on Grievances by Application of Technology (SWAGAT) Initiative on April
27, 2023.
✓ The SWAGAT initiative was launched by the Gujarat government in 2003 when PM Modi was the Chief
Minister of the state.
✓ The state government is observing the SWAGAT Saptah (week) to mark the successful completion of 20
years of this problem-solving mechanism.
✓ SWAGAT is an effective grievance redressal mechanism of the Gujarat government, by using technology
to quickly and efficiently solve the grievances of the citizens, serving as a bridge between the government
and citizens.
Q. What is the rank of India globally in SIPRI’s report for military spending in 2022?
A) 3rd B) 4th
C) 6th D) 7th
Answer : B
✓ India was the fourth biggest military spender in the world in 2022 as per the report published by the
Stockholm International Peace Research Institute (Sipri) on April 24, 2023.
✓ The total military spending of India was of $81.4 billion in 2022. It was 6.0 percent more than in 2021,
and up by 47% from 2013.

Follow us: Official Site, Telegram, Facebook, Instagram, Instamojo 763


✓ AS per the report, the United States emerged as the largest military spender, reaching $877 billion in
2022.
✓ China (2nd)($292 billion), Russia (3rd) ($86.4 billion) and Saudi Arabia (5th) ($75.0 billion) are the top
5 biggest military spender in 2022.
✓ The total global military expenditure in 2022 increased by 3.7 percent in real terms to reach an all-time
high of $2240 billion. This was an increase for the eighth consecutive year.
✓ The three largest spenders of 2022, that means the United States, China and Russia,—accounted for 56
percent of the world total spending.
✓ About SIPRI
✓ The Stockholm International Peace Research Institute (SIPRI) is a global non-profit research institute.
✓ SIPRI focuses on resolving conflicts, controlling arms and promoting disarmament.
✓ The institute provides data, analysis and advice regarding armed conflicts, military expenditure and the
arms trade.
✓ Establishment - 6 May 1966
✓ Founders - Tage Erlander, Alva Myrdal
✓ Chairman - Stefan Lofven
✓ Director - Dan Smith
✓ Headquarter - Solna, Sweden.

Q. What was the total military spending across the globe in 2022, as per the report of SIPRI?
A) $2440 billion B) $2240 billion
C) $2220 billion D) $2420 billion
Answer : B
✓ The total global military expenditure in 2022 increased by 3.7 percent in real terms to reach an all-time
high of $2240 billion. This was an increase for the eighth consecutive year
Q. Which country was the biggest military spender in 2022, as per the report of SIPRI?
A) China B) Russia
C) United States D) France
Answer : C
✓ AS per the report, the United States emerged as the largest military spender, reaching $877 billion in
2022.
Q. India’s first “Water Metro” services has been inaugurated in which state?
A) Odisha B) West Bengal
C) Kerala D) Manipur
Answer : C
✓ India’s first “Water Metro” services will be inaugurated by the Prime Minister Narendra Modi on April
25, 2023 in Kochi, Kerala.
✓ This initiative will give boost to economic growth, tourism and transport connectivity, as well as
accelerate the growth and development of Kochi.
✓ The Water Metro service is a unique urban mass transit system, which will give similar experience and
ease of travel as that of conventional metro system.
✓ Overall project cost is of Rs 1,136.83 crore, which will be funded by Government of Kerala and German
bank KfW.

Follow us: Official Site, Telegram, Facebook, Instagram, Instamojo 764


Q. Who among these has been conferred with the Order of Australia (AO), the highest civil honour of
Australia in 2023?
A) Ratan Tata B) Bernard Arnault
C) Bill Gates D) Azim Premji
Answer : A
✓ Ratan Tata has been awarded with the highest civil honour of Australia, Order of Australia (AO), on
April 22, 2023.
✓ He has been awarded for his longstanding commitment to Indo-Australian ties specifically in the areas
of trade, investment, and philanthropy.
✓ Australia’s High Commissioner to India Barry O’Farrell conferred the honour on Ratan Tata, the titan
of biz, industry and philanthropy
Q. The Mana village has been designated as ‘India’s first village’ by BRO, as it is situated on the India-China
border. In which state is this village located?
A) Himachal Pradesh B) Uttarakhand
C) Sikkim D) Arunachal Pradesh
Answer : B
✓ Mana village on India-China border in UK’s Chamoli district declared as ‘India’s first village’ by BRO.
✓ The Border Road Organisation (BRO) of India has put up a signboard saying ‘India’s first village’ in
Mana, a village in Uttarakhand.
✓ The Mana village is on the India-China border in Chamoli District.
✓ This decision is based on the initiative by PM Modi to declare all border villages in the country as the
first villages and not the last as they are usually called.
✓ Mana village is situated near Himalayan temple, Badrinath.
✓ Devotees going to the temple usually go up to the village for sightseeing.
Q. Who has been sworn in as the 22nd President of Bangladesh?
A) Mohammad Shahabuddin Chuppu B) Sheikh Russell
C) Sheikh Hasina D) Sheikh Salahuddin Jewel
Answer : A
✓ Mohammad Shahabuddin Chuppu, a retired judge, has been sworn in as the new president of Bangladesh
for a five-year term, replacing Abdul Hamid.
✓ Chuppu, the nominee of the ruling Awami League (AL) party
✓ With 302 members, the AL party has the majority in the 350-seat national parliament, and no other party
had the numbers to nominate a presidential candidate
✓ People’s Republic of Bangladesh
✓ Capital: Dhaka
✓ Currency : Taka
✓ Prime Minister: Sheikh Hasina Wazed
✓ President: Mohammad Shahabuddin
Q. Dubai-based airline Emirates has unveiled the world’s first robotic check-in assistant. What is the name
of this robot?
A) Nadine B) Talos
C) Sara D) Sophia

Follow us: Official Site, Telegram, Facebook, Instagram, Instamojo 765


Answer : C
✓ Dubai-based airline Emirates has unveiled the world's first robotic check-in assistant named 'Sara'.
✓ The system is part of a new City Check-in and Travel Store that is set to launch in Dubai's Financial
District
✓ What is the world's first robotic check-in assistant 'Sara'?
✓ 'SARA' will match customers' faces with scanned passports, check in passengers and guide them to the
baggage drop area.
✓ This facility allows customers to drop off their luggage at the airport up to 24 hours in advance and up to
4 hours before their flight.
✓ Passengers cancheck-in via self-check-in kiosks, with Emirates agents at dedicated desks or with the help
of Sara.
Q. Prime Minister Narendra Modi has inaugurated NAMO Medical Education and Research Institute in
which city?
A) Port Blair B) Silvassa
C) Puducherry D) Kavaratti
Answer : B
✓ Prime Minister Narendra Modi has inaugurated NAMO Medical Education and Research Institute in
Silvassa, Dadra and Nagar Haveli and Daman and Diu.
✓ Notably, the foundation stone of NAMO Medical Education and Research Institute was also laid by the
Prime Minister himself in January 2019.
✓ Prime Minister also laid the foundation stone and inaugurated 96 projects worth more than Rs 4,850
crores to the nation at Sayli ground, Silvassa. People lit mobile flashlights to welcome PM Modi in
Silvassa.
Q. Who is the author of the “Smoke and Ashes: A Writer’s Journey Through Opium’s Hidden Histories”
A) Chetan Bhagat B) Shashi Tharoor
C) Amitav Ghosh D) Arundhati Roy
Answer : C
✓ Amitav Ghosh's new non-fiction book, "Smoke and Ashes: A Writer's Journey Through Opium's Hidden
Histories", will be published by HarperCollins Fourth Estate on July 15, 2023
✓ The book is a combination of memoir, travelogue, and cultural and economic history, which explores
the often overlooked effects of opium on society and history.
✓ About Amitav Ghosh
✓ He is a renowned writer who was born in Kolkata and brought up in India, Bangladesh and Sri Lanka.
✓ He has written many acclaimed works of fiction, including "The Shadow Lines," "The Glas Palace,"
"The Hungry Tide," "Gun Island," "The Nutmeg's Curse," "Jungle Nama," and "The Living Mountain."
.,
✓ His works have been translated into more than 30 languages, making him a globally recognized author.
✓ In 2019, Ghosh received the Jnanpith Award, India's highest literary honour, becoming the first English-
language writer to receive this recognition.

Q. The tallest Ambedkar statue in India has been unveiled in which state?
A) Uttar Pradesh B) Telangana
C) Bihar D) Rajasthan
Answer : B

Follow us: Official Site, Telegram, Facebook, Instagram, Instamojo 766


✓ The 132nd birth anniversary of Babasaheb Dr. Bhimrao Ambedkar, considered the architect of the Indian
Constitution, is being celebrated across the country.
✓ On the birthday of Dr. Bhimrao Ambedkar, Chief Minister of Telangana K Chandrasekhar Rao
inaugurated his 125 feet tall statue, which is believed to be the tallest statue of Ambedkar in India.
✓ This statue has been designed by renowned sculptor Ram Vanji Sutar and his son Anil Ram Sutar in
Noida, Uttar Pradesh.
✓ This statue has been built by KPC Projects Limited. Its total cost is Rs 146.50 crore.

Q. Which state has bagged the GI tag for its famed GOND Painting?
A) Bihar B) Madhya Pradesh
C) Maharashtra D) Uttar Pradesh
Answer : B
✓ The famed Gond painting of Madhya Pradesh has received the prestigious Geographical Indication (GI)
tag.
✓ GI Tag ensures that no one other than the registered authorised user is allowed to use the name of the
popular product.
✓ Apart from this, the famous Cumbum Panneer Thratchai from Tamil Nadu, commonly known as
Cumbum grapes has also receievd the GI tag.
✓ The Cumbum valley in Tamil Nadu’s Western Ghats, is regarded as the “Grapes City of South India”.
✓ The Authoor betel leaves, (locally known as vetrilai) of Tamil Nadu has also bagged GI tag.
✓ Tamil Nadu’s Malai Poondu, Srivilliputtur Palkova and Palani Panchamirtham have also been awarded
the GI tag
Q. The famous Cumbum grapes, which has bagged the GI tag, is from which state?
A) Tamil Nadu B) Kolkata
C) Arunachal Pradesh D) Karnataka
Answer : A
✓ The famous Cumbum Panneer Thratchai from Tamil Nadu, commonly known as Cumbum grapes has
also receievd the GI tag.
✓ The Cumbum valley also called Kambam Valley, located at the Western Ghats in Tamil Nadu, is known
as the 'Grapes city of South India'.
Q. Which among the following political party has been granted national party status?
A) Trinamool Congress B) AAP
C) CPI D) Nationalist Congress Party
Answer : B
✓ The Election Commission of India (ECI) has granted national party status to Arvind Kejriwal-led Aam
Aadmi Party (AAP).
✓ The AAP party has been named as a national party based on its electoral performance in four states
namely Delhi, Goa, Punjab and Gujarat.
✓ Currently, the Arvind Kejriwal-led party is in power in Delhi and Punjab.
✓ Aam Aadmi Party ( AAP) is a political party in India. It was founded in November 2012 by Arvind
Kejriwal and his then-companions,following the 2011 Indian anti-corruption movement.
✓ AAP is currently the governing party in the Indian state of Punjab and the union territory of Delhi.
✓ On 10 April 2023, AAP was officially granted the status of National party by ECI.
✓ The party's election symbol is a broom.

Follow us: Official Site, Telegram, Facebook, Instagram, Instamojo 767


✓ Apart from this, the commission has revoked the national party status of Trinamool Congress , CPI, and
Nationalist Congress Party (NCP).
✓ Now, the national political parties recognised in India will be the BJP, Congress, CPI(M), Bahujan Samaj
Party (BSP), National People’s Party (NPP) and the AAP.
✓ Right now there are only six national parties in the country:
✓ Indian National Congress (INC)
✓ Communist Party of India (Marxist) (CPI-M)
✓ Bharatiya Janata Party (BJP)
✓ Bahujan Samaj Party (BSP)
✓ National People's Party (NPP)
✓ Aam Aadmi Party (AAP)
Q. What is the name given to India’s first semi-high-speed regional rail services, launched by National
Capital Region Transport Corporation (NCRTC)?
A) TEMPO B) RAPIDX
C) AMRUTH D) ZOOM
Answer : B
✓ India’s first semi high-speed regional rail services in NCR named ‘RAPIDX’.
✓ The National Capital Region Transport Corporation (NCRTC), has named India’s first semi-high-speed
regional rail services as ‘RAPIDX’, which will run on the Regional Rapid Transit System (RRTS)
corridors.
✓ RAPIDX is being implemented by NCRTC to connect key urban nodes across the National Capital
Region (NCR).
✓ RAPIDX signifies speed and progress, while the letter X denotes next generation technology and the
new-age mobility solution. It also represents youth, optimism, and energy.
✓ The overall aim of launching this brand is decarbonisation, which means decongesting NCR by reducing
the number of vehicles on the road and by the use of green energy.
Q. The Kibithoo village, which is termed as India’s first village by Union Home Minister Amit Shah, is in
which state?
A) Tripura B) Arunachal Pradesh
C) Assam D) Meghalaya
Answer : B
✓ The Union Home Minister Amit Shah launched the “Vibrant Villages Program (VVP)” at the Kibithoo
town of Arunachal Pradesh.
✓ Home Minister Shah also urged people to term Kibithoo as India’s first village as the Sun’s rays fall first
in this village in the country.
✓ The VVP is a centrally sponsored scheme of the Modi government to link all the border villages and also
provide trans valley connectivity, in states sharing Line of Actual Control (LAC) with China.
✓ Under this VVP scheme, 2,967 villages in 46 blocks of 19 districts, in the states of Arunachal Pradesh,
Sikkim, Uttarakhand and Himachal Pradesh and UT of Ladakh have been identified for comprehensive
development.
✓ The programme will help in improving the quality of life of people living in identified border villages and
encourage people to stay in their native locations thereby reversing the out migration from these villages
and adding to security of the border.

Follow us: Official Site, Telegram, Facebook, Instagram, Instamojo 768


Q. Which metro company has become the first in India to launch under water service?
A) Hyderabad B) Mumbai
C) Delhi D) Kolkata
Answer : D
✓ The Kolkata Metro has become the first metro in India to operate an under water metro train. The trail
run for the same was successfully completed on April 13, 2023.
✓ The Kolkata Metro rake travelled through the tunnel under the Hooghly river, running from Howrah
Maidan to Esplanade, covering four stations along the way – Esplanade, Mahakaran, Howrah, and
Howrah Maidan.
✓ The new route will help to ease congestion, by providing a faster, more efficient means of transportation.
✓ During this trial, the metro rail was operated from a 520-meter tunnel under the Hooghly River

Q. Who has been crowned Femina Miss India World 2023?


A) Shreya Poonja B) Rubal Shekhawat
C) Thounaojam Strela Luwang D) Nandini Gupta
Answer : D
✓ 19-year-old Nandini Gupta, from Rajasthan, has been crowned as the Femina Miss India World 2023,
at the 59th edition of Femina Miss India beauty pageant, held on April 15, 2023 at Imphal, Manipur.
✓ She was crowned by the 2022 Femina Miss India Sini Shetty.
✓ Nandini hails from Kota in Rajasthan.
✓ She will now represent India at the 71st Miss World pageant, to be held in the United Arab Emirates
(UAE) in May 2023.
✓ Meanwhile, Delhi’s Shreya Poonja was declared the first runner-up, and Manipur’s Thounaojam Strela
Luwang as the 2nd runner-up.

Q. What is the name of the mission launched by the European Space Agency (ESA) to Jupiter’s moon?
A) JADE B) JUICE
C) JUPITER D) JUNG
Answer : B
✓ European Space Agency successfully launches its mission JUICE to reach Jupiter’s moons.
✓ The European Space Agency (ESA) successfully launched its mission to Jupiter’s moons on April 13,
2023.
✓ The Jupiter Icy Moons Explorer (JUICE) project of ESA was launched on the Ariane 5 rocket from the
spaceport in Kourou, French Guiana.
✓ The 1.6 billion Euros Project JUICE aims to send a satellite on an eight-year journey to reach the major
icy moons of the Jupiter, namely Ganymede, Callisto and Europa and find whether they have the
conditions to support simple life.
✓ The spacecraft is scheduled to finally reaching the gas giant in July 2031.
Q. The 125-ft tall bronze statue of Dr. BR Ambedkar, which is the highest such statue of him, has been built
in which city recently?
A) Lucknow B) Kolkata
C) Hyderabad D) Gandhinagar
Answer : C

Follow us: Official Site, Telegram, Facebook, Instagram, Instamojo 769


✓ In Telangana, the Chief Minister K Chandrasekhar Rao unveiled the 125-ft tall bronze statue of Dr. BR
Ambedkar in Hyderabad on April 14, 2023, on the occasion of the 132nd birth anniversary of the
architect of the Indian Constitution.
✓ The 125-ft tall bronze statue in Hyderabad is the tallest statue of Ambedkar in India.
✓ The statue costs ₹ 146.50 crore. It is built using 360 tonnes of stainless steel and 114 tonnes of bronze.

Q. Which team has won the Women's Premier League 2023 title?
A) Delhi Capitals B) UP Warriors
C) Mumbai Indians D) Royal Challengers Bangalore
Answer : C
✓ Mumbai Indians has defeated the Delhi Capitals by 7 wickets and with that they become the first-ever
Women’s Premier League champions.
✓ Mumbai Indians Captain : Harmanpreet Kaur
✓ Delhi Capitals Captain : Meg Lanning
✓ Player of the match : Nat Sciver-Brunt (Mumbai Indians)
✓ Purple cap : Hayley Matthews (Mumbai Indians)
✓ Orange cap : Meg Lanning (Delhi Capitals)
✓ Most valuable player : Hayley Matthews (Mumbai Indians).
✓ India’s star opener Smriti Mandhana has become the costliest player in the ongoing player auction for
the Women’s Premier League (WPL).
✓ Mandhana joined Royal Challengers Bangalore for Rs 3.4 crore.
✓ Viacom 18 has acquired the media rights of the Women’s Indian Premier League (WIPL).
✓ Viacom18 has bought the media rights of the women’s IPL for five years from 2023 to 2027 for Rs 951
crore.
✓ Tata Group has bagged the title sponsorship rights for the Women's Premier League (WPL) for five
seasons.
Q. The central government has sanctioned how much rupees under the Faster Adoption and Manufacturing
of Electric Vehicles (FAME) Scheme Phase-II for setting up 7432 public fast charging stations across the
country?
A) Rs 600 crore
B) Rs 800 crore
C) Rs 500 crore
D) Rs 700 crore
Answer : B
✓ Union Government has sanctioned 800 crore rupees under the Faster Adoption and Manufacturing of
Electric Vehicles (FAME) Scheme Phase -2 for setting up 7432 public fast charging stations across the
country.
✓ The government has formulated Faster Adoption and Manufacturing of Electric Vehicles (FAME) India
Scheme in 2015 to improve the infrastructure required for the large-scale use of electric vehicles.
✓ At present, there are about 6,586 charging stations across the country. The addition of the new 7,432
public charging stations will be a significant push to Electric Vehicles charging ecosystem.
✓ This move will boost the electric vehicle ecosystem in India and encourage more people to switch to
cleaner modes of transportation.

Follow us: Official Site, Telegram, Facebook, Instagram, Instamojo 770


Q. The government has recently launched a new centrally sponsored scheme named 'New India Literacy
Programme' with a financial outlay of over___ for implementation during the financial years 2022-23 to
2026-27.
A) Rs 500 crore B) Rs 800 crore
C) Rs 1000 crore D) Rs 700 crore
Answer : C
✓ The government has recently launched a new centrally sponsored scheme named 'New India Literacy
Programme' with a financial outlay of over Rs 1000 crore for implementation during the financial years
2022-23 to 2026-27.
✓ The scheme aims to cover five crore illiterates in the age group of 15 years and above.
✓ Beneficiaries under this scheme are identified through door to door survey on mobile app by the surveyors
in the States and Union Territories.
✓ About New India Literacy Program.
✓ The program has been launched for the period FY 2022-2027 to align with the National Education Policy
2020 and cover all aspects of adult education.
✓ The scheme has been approved with a financial outlay of Rs 1037.90 crore, which includes central share
of Rs 700.00 crore and state share of Rs 337.90 crore.
✓ The center and state share for all states is in the ratio of 60:40 while for North Eastern Region (NER) and
Himalayan states the share between center and state is in the ratio of 90:10.
✓ The scheme will cover non-literates in the age group of 15 years and above in all the States/UTs of the
country.
✓ It aims to cover 1 crore learners per year (total target of 5 crore) using "Online Teaching, Learning and
Assessment System (OTLAS)" in collaboration with National Informatics Centre, NCERT and NIOS
✓ Five components of the scheme
✓ Foundational Literacy and Numeracy.
✓ Critical Life Skills.
✓ Vocational Skills Development.
✓ Basic Education.
✓ Continuing Education
Q. Which state has received the protected geographical indication (PGI) tag from the European Commission
(EC) for the Kangra tea?
A) Himachal Pradesh B) Uttarakhand
C) Rajasthan D) Sikkim
Answer : A
✓ Kangra Tea of Himachal Pradesh grabs European geographical indication (GI) tag
✓ The European Commission (EC) has officially granted protected geographical indication (PGI) for the
Kangra tea, which is grown in Kangra district of Himachal Pradesh.
✓ The Kangra Tea received the PGI recognition on March 22, 2023 and will come into effect from April
11, 2023.
✓ The PGI tag will help tea farmers of Kangra to get better prices for their produce in the European
countries.
✓ Kangra tea, which is grown since the mid-19th Century, is known for its unique flavour and taste.
✓ It is available as black and green teas.

Follow us: Official Site, Telegram, Facebook, Instagram, Instamojo 771


Q. Finland has been officially inducted as the_ member of the NATO organisation.
A) 35th B) 29th
C) 31st D) 40th
Answer : C
✓ Finland has received the green light to join NATO, after Turkey unanimously voted to allow Finland to
become the 31st member of the alliance on March 31, 2023.
✓ Turkey was the only country among the 30-member Western military alliance to sign off for the
membership of Finland in the NATO alliance.
✓ It is necessary that all NATO members must vote unanimously to rope in a new country as the member
of NATO.
✓ With this, Finland will now be able to formally attend the NATO Summit 2023, which will be held in
July in Lithuania.
✓ North Atlantic Treaty Organization(NATO), is a military alliance between North American and
European countries formed in 1949 and headquartered in Brussels, Belgium.
✓ The founding members of NATO were the United States, Canada, and ten European countries, including
Belgium, Denmark, France, Iceland, Italy, Luxembourg, the Netherlands, Norway, Portugal, and the
United Kingdom.
✓ North Atlantic Treaty Organization (NATO)
✓ Formation : 4 April 1949
✓ Headquarters : Brussels, Belgium
✓ Secretary General : Jens Stoltenberg
✓ Total Member : 30 (North Macedonia)
✓ North Atlantic Treaty, also referred to as the Washington Treaty.
Q. The Anil Agarwal International Cricket Stadium is proposed to be developed at which place in India?
A) Amritsar B) Jaipur
C) Agra D) Ujjain
Answer : B
✓ India’s second largest and World’s third-largest cricket stadium to be developed in Jaipur.
✓ The world’s third-largest and India’s second largest cricket stadium is proposed to be set up in Jaipur
village, Chonp.
✓ For this, the Rajasthan Cricket Association (RCA) has signed an MoU with Vedanta’s Hindustan Zinc
Limited (HZL).
✓ The stadium would be named as Anil Agarwal International Cricket Stadium, Jaipur.
✓ It is the world’s third biggest Cricket stadiums with a seating capacity of 75,000.
✓ The Narendra Modi Stadium in Ahmedabad, (earlier known as Sardar Patel Stadium and Motera
Stadium) is the largest cricket stadium in India. It has seating capacity of 1.32 lakh spectators.
✓ The second largest stadium is Australia’s Melbourne Cricket Ground (MCG) which has a capacity of
1,00,024 spectators.

Q. MF Hydra is world’s first liquid hydrogen-powered ferry. It has been launched by which country?
A) Netherlands B) Germany
C) Finland D) Norway
Answer : D

Follow us: Official Site, Telegram, Facebook, Instagram, Instamojo 772


✓ The world’s first liquid hydrogen-powered ferry, MF Hydra, became operational on April 04, 2023 in
Norway after the Norwegian Maritime Administration granted MF Hydra the clearance to sail, following
rigorous development and testing.
✓ The MF Hydra has been launched by the Norwegian shipping company Norled.
✓ This 82.4-meter-long MF Hydra ferry will cover zero-emission journey on a triangular route in Norway
between Hjelmeland, Skipavik, and Nesvik.
✓ The hydrogen system was provided by Linde Engineering, the fuel cell by Ballard and the battery by
Corvus Energy. The ferry was completed by Westcon and systems integrator SEAM.
✓ About Norway
✓ Capital : Oslo
✓ Currency : Norwegian Krone
Q. Union Rural Ministry has launched Captive Employment scheme to provide employment opportunity
and livelihood to rural youths. How many Captive Employers have been hired under phase 1 of the
scheme to provide training to youths?
A) 15 B) 21
C) 19 D) 26
Answer : C
✓ The Union Ministry for Rural Development has launched a unique initiative named ‘CAPTIVE
EMPLOYMENT, under the Deen Dayal Upadhyaya Grameen Kaushalya Yojana (DDU-GKY).
✓ The Captive Employer scheme aims to bridge the gap between job seeker and job provider by fulfilling
job needs of rural poor and improve their standard of living. This program shall also contribute to
sustainable development goals.
✓ Union Minister Giriraj Singh also onboarded 19 Captive Employers, who will train Rural Poor Youth
and provide employment to trained youth in their company or subsidiary.
✓ The training to rural poor youth will be given in their respective industries, namely, hospitality, apparel
& textiles, manufacturing, IT/ITeS, telecom, retail, Power etc.
✓ Through this initiative, more than 31,000 jobs are expected to be provided to rural poor youth
Q. How many member of astronauts have been named for NASA’s Artemis II crew mission?
A) 4
B) 3
C) 7
D) 5
Answer : A
✓ American space agency, NASA and the Canadian Space Agency (CSA) announced the names of four
astronauts, who will venture around the Moon for science and exploration under Artemis II mission.
✓ The Artemis II mission is the first crewed voyage around the moon in more than 50 years. The previous
human flight to lunar surface was the Apollo program in 1972.
✓ The Artemis II crew members consists of three from NASA and one from CSA.
✓ They are Reid Wiseman, Victor Glover, and Christina Hammock Koch, from NASA and Jeremy Hansen
from CSA.
✓ Out of these four, Christina Hammock Koch is the first ever female, Victor Glover is the first ever African
American and Jeremy Hansen is the first ever Canadian and first non- American to be sent on a lunar
mission.
✓ The Artemis II will mark the debut crewed flight, but not the first lunar landing.

Follow us: Official Site, Telegram, Facebook, Instagram, Instamojo 773


Q. Kiran Nadar has recently been felicitated with the highest civilian honour award by which country?
A) Australia B) Sweden
C) France D) United States
Answer : C
✓ Philanthropist and art collector Kiran Nadar, (wife of Shiv Nadar, the founder of HCL Technologies),
has been conferred with the highest French civilian award “Chevalier de la Légion d’Honneur” (Knight
of the Legion of Honour).
✓ The award was conferred upon Kiran Nadar by French Ambassador to India Emmanuel Lenain, for her
outstanding contribution in the field of art, her commitment to providing greater access to culture both
nationally and internationally, and her leading role in fostering Indo-French cultural ties.
✓ Kiran Nadar is the chairperson of the Kiran Nadar Museum of Art (KNMA), a philanthropic initiative
in art, and a trustee of the Shiv Nadar Foundation.
Q. Dhawan-II is an advanced fully 3D-printed cryogenic rocket engine, which was successfully test fired
recently by which company?
A) Skyroot Aerospace B) ISRO
C) Pixxel D) Dhruva Space
Answer : A
✓ Hyderabad-based space tech startup Skyroot Aerospace successfully test fired an advanced fully 3D-
printed cryogenic rocket engine `Dhawan-II’ on April 04, 2023.
✓ Skyroot Aerospace is India’s first private rocket builder.
✓ The endurance test of ‘Dhawan-II’ was carried out at Solar Industries propulsion test facility in Nagpur,
Maharashtra. It lasted for a long duration of almost 200 seconds.
✓ The ‘Dhawan-II’ Cryogenic engine will power the upper stage of Vikram-2.
Q. According to the new joint report from the Institute for Energy Economics and Financial Analysis
(IEEFA) and JMK Research and Analytics, India could become the world’s second-largest solar
photovoltaic manufacturer by which year?
A) 2025 B) 2026
C) 2030 D) 2035
Answer : B
✓ According to the new joint report from the Institute for Energy Economics and Financial Analysis
(IEEFA) and JMK Research and Analytics, India could become the world’s second-largest solar
photovoltaic manufacturer by 2026, after China.
✓ India’s cumulative module manufacturing nameplate capacity more than doubled from 18GW in March
2022 to 38GW in March 2023.
✓ In terms of upcoming PV manufacturing installations, Gujarat is the leading state in India. It accounts
for nearly 57% of all the upcoming PV manufacturing capacity.

Q. Which state government has recently launched Cool Roof Policy 2023-28?
A) Kerala B) Gujarat
C) Telangana D) Tamil Nadu
Answer : C

Follow us: Official Site, Telegram, Facebook, Instagram, Instamojo 774


✓ Municipal Administration and Urban Development Minister K T Rama Rao launched the Telangana
Cool Roof Policy 2023-28 to make the state more thermally comfortable and heat-resilient by reducing
the impact of the urban heat island effect and heat stress and also to bring down energy consumption.
✓ A cool roof, which uses special paints or tile cover, is designed to reflect more sunlight than a
conventional roof, thus reducing heat retention and cooling indoor spaces.
✓ Telangana Cool Roof Policy 2023-28 suggests the adoption of a cool roof as a measure for
implementation to build resilience against extreme heat.
✓ Telangana became the first state to introduce a cool roof policy, having incorporated it in building
permission applications.
✓ Cool roofs are now mandatory for all government as well as commercial buildings irrespective of their
site area or built-up area
Q. Which country has successfully launched the “Ofek-13” spy satellite into space by the Shavit launch
vehicle, from Palmachim Airbase spaceport?
A) Japan B) China
C) Israel D) Russia
Answer : C
✓ Israel has successfully launched the “Ofek-13” spy satellite into space by the Shavit launch vehicle, from
Palmachim Airbase spaceport.
✓ This satellite was manufactured by state-owned Israel Aerospace Industries (IAI).
✓ The ‘Ofek 13’ is the most advanced of its kind with unique radar observation capabilities and will enable
intelligence collection in any weather and conditions of visibility.
✓ It is the latest in a series of domestically produced satellites that were first put into orbit in 1988.
✓ Israel joined the club of space powers in 1988 with the deployment of its first Ofek (Horizon in Hebrew)
satellite.
ISRAEL IN NEWS 2023.
▪ Israel successfully launched a new spy satellite, named Ofek-13, into orbit. The satellite, Ofek-13,
launched from the Palmachim Airbase in central Israel.
▪ Adani Group acquired the strategic Israeli port of Haifa for USD 1.2 billion.
▪ Bharat Electronics Ltd (BEL) to Manufacture Israel’s LORA Ballistic Missile for Indian Tri-Services.
▪ Benjamin Netanyahu Sworn in as the Prime minister of Israel for a Record 6th Time, Israel’s longest-
serving Prime Minister.
▪ India and Israel have started a series of events to mark the 30 years of establishment of diplomatic
relationship between the two countries.
▪ ISRAEL
▪ Capital : Jerusalem
▪ Currency : New Shekel
▪ President : Isaac Herzog
▪ Prime Minister : Benjamin Netanyahu
▪ Parliament : Knesset
Q. Which state has topped the world’s busiest airport list in 2022, by the Airports Council International
(ACI)?
A) Denver International Airport
B) Atlanta International Airport
C) Guangzhou Bai Yun International Airport
D) Orlando International Airport

Follow us: Official Site, Telegram, Facebook, Instagram, Instamojo 775


Answer : B
✓ The United States-based Atlanta airport has retained its position as the world’s busiest airport. It handled
9.37 crore passengers in 2022.
Q. Which airport in India is ranked among top 10 busiest airport in the world for passenger traffic, as per
Airports Council International (ACI) report?
A) Rajiv Gandhi International Airport, Hyderabad
B) Indira Gandhi International Airport, New Delhi
C) Kempegowda International Airport, Bengaluru
D) Chhatrapati Shivaji Maharaj International Airport, Mumbai
Answer : B
✓ The Indira Gandhi International Airport in New Delhi has been ranked among the world’s top 10 busiest
airports by passengers traffic, as per Airports Council International (ACI).
✓ It is the only airport from South and Southeast Asia to feature in ACI’s list of the 10 busiest airports for
2022.
✓ The global rank of the Delhi airport is at the ninth spot for 2022.
✓ It handled almost 5.95 crore passengers in 2022.
✓ In 2021, the airport was ranked at 13th and in 2019, at 17th.
✓ Top airport – The United States-based Atlanta airport has retained its position as the world’s busiest
airport. It handled 9.37 crore passengers in 2022.
✓ Top 10 busiest airport in the world list
1. Atlanta 2.Dallas
3. Denver 4. Chicago
5. Dubai 6. Los Angeles
7. Istanbul 8. London Heathrow
9. New Delhi 10. Paris
Q. Which state has topped the India Justice Report 2022?
A) Karnataka B) Uttar Pradesh
C) Maharashtra D) Tamil Nadu
Answer : A
✓ Karnataka has secured the top position in the India Justice Report-2022.
✓ This ranking has been given in the category of large and medium-sized states including 18 states.
✓ Tamil Nadu comes in 2nd place and Telangana is in 3rd place.
✓ While Uttar Pradesh acquires the 18th position in this list.
✓ Gujarat is 4th and Andhra Pradesh is 5th in this current index.
✓ It was prepared based on the comprehensive data collected on 4 pillars of justice delivery which are the
Police System, Judiciary, Prison System and Legal Aid.
✓ Among small states, Sikkim topped the list of seven small states with less than one crore population,
followed by Arunachal Pradesh. Goa is at the bottom, at seventh position.
✓ About India Justice Report
✓ Tata Trusts launched the India Justice Report (IJR) in 2019.
✓ This is the third edition.

Follow us: Official Site, Telegram, Facebook, Instagram, Instamojo 776


✓ Center for Social Justice, Common Cause, Commonwealth Human Rights Initiative, Daksh, TISS-
Prayas, Vidhi Institute for Law Policy, and How India Lives, data partner of IJR, are among its partners.
Q. Which state in India has been ranked at first position for having the most number of products recognised
with Geographical Indication (GI) tag ?
A) Bihar B) Uttarakhand
C) Kerala D) Himachal Pradesh
Answer : C
✓ As per the data shared by the GI Registry, Kerala has topped among Indian states for having most number
of products securing the Geographical Indication (GI) tag in financial year 2022-23.
✓ The data showed that between April 2022 and March 2023, 6 products from Kerala were selected for GI
recognition.
✓ These are Attappady Aattukombu Avara, Attappady Thuvara, Onattukara Ellu, Kanthaloor Vattavada
Veluthuli, Kodungallur Pottuvellari.
✓ Overall 12 products were selected for the recognition in FY23. Of them 2 were from overseas.
✓ Other Indian products to receive GI tag were Mithila Makhana (aquatic fox nut) from Bihar, Alibag
white onion from Maharashtra, Tandur Redgram (the local variety of pea) from Telangana, Ladakh
Raktsey Karpo Apricot from Ladakh and Gamosa handicrafts from Assam.
✓ The two overseas products include Brandy De Jerez from Spain, and Provolone Valpadana from Italy.
✓ For the period between April 2022 and March 2023, a total of 12 products were selected for GI
recognition, including two foreign ones - Brandy de Jerez from Spain and Provolone Valpadana from
Italy
Q. Kim Cotton is the first female umpire to officiate in a Men’s T20 International (T20I) cricket match
involving two ICC full-member nations. She is from which country?
A) South Africa B) England
C) Australia D) New Zealand
Answer : D
✓ Auckland, New Zealand-born Kim Cotton has become the first female umpire to officiate in a Men’s T20
International (T20I) cricket match involving two ICC full-member nations.
✓ She achieved the feat in the second T20I held between New Zealand and Sri Lanka in Dunedin on April
05, 2023, in which New Zealand won by nine wickets.
✓ Till date, Cotton has officiated in 16 women’s one-day international (ODI) games and 44 women’s T20Is.
✓ She was one of the on-field umpires for 50-over Women’s World Cup final in 2022 and the Women’s
T20 World Cup final in February 2023.
✓ New Zealand
✓ Capital: Wellington
✓ Currency : dollar
✓ Prime Minister : Chris Hipkins
Q. The Geographical Indications Registry has granted Geographical Indication (GI) tag to Nagari Dubraj,
fragrant rice to give a unique identity to the brand. Nagari Dubraj is the rice of which state?
A) Chhattisgarh B) Bihar
C) Assam D) Odisha
Answer : A

Follow us: Official Site, Telegram, Facebook, Instagram, Instamojo 777


✓ The Geographical Indications Registry has granted Geographical Indication (GI) tag to Nagari Dubraj,
a fragrant rice from Chhattisgarh, to give a unique identity to the brand.
✓ The Morena and Rewa Mango (Madhya Pradesh) have also been given the GI Tag.
✓ The origin of Dubraj is believed to be from Shringi Rishi Ashram area of Sihawa. Its reference is found
in Valmiki Ramayana. Sihawa area is considered to be the source of Dubraj in various research papers.
Q. Based on the report of India State of Forest Report (ISFR) 2021, the total forest cover in the country is
7,13,789 square kilometer. This is an increase of how much sqkm compared to ISFR 2019?
A) 1,540 B) 1,240
C) 1,340 D) 1,140
Answer : A
✓ As per latest India State of Forest Report (ISFR) 2021, the total forest cover in the country is 7,13,789
square kilometer.
✓ This is 21.71% of the geographical area of India.
✓ The ISFR report is released by the Forest Survey of India (FSI), Dehradun, biennially, since 1987 based
on forest cover.
✓ The last survey was done in 2019.
✓ As per the report, between ISFR 2019 and ISFR 2021, the forest cover has increased by 1,540 square
kilometer.
✓ FSI is an organisation under the Ministry of Environment, Forest and Climate Change, founded in June
1981.
✓ It conducts forest surveys, studies and researches to monitor the changing situations of land and forest
resources and present the data for national planning, conservation and sustainable management of
environmental protection as well for the implementation of social forestry projects.
Q. Name the high resolution space based air-quality monitoring instrument launched by NASA, onboard
SpaceX rocket recently to monitor air quality over North America?
A) TOUR B) TEMPO
C) TORPOR D) TURBO
Answer : B
✓ The US space agency National Aeronautics and Space Administration (NASA) launched an air-quality
monitor called TEMPO (Tropospheric Emissions Monitoring of Pollution instrument) on April 07, 2023,
in collaboration with Elon Musk’s SpaceX.
✓ TEMPO was launched from Cape Canaveral Space Force Station in Florida atop a SpaceX Falcon 9
rocket.
✓ It will be the first space-based instrument to measure air quality over North America hourly during the
daytime and at spatial regions of four square miles, far better than existing limits of about 100 square
miles in the U.S.
✓ TEMPO is merely the size of a dishwasher.
✓ TEMPO slogan is ‘It’s about time,’. It signifies at TEMPO’s ability to provide hourly air pollution data,
to help improve life on Earth by revolutionizing the way scientists observe air quality from space.
Q. ISRO conducted the successful launch of RLV Autonomous Landing Mission (RLV-LEX) recently from
Aeronautical Test Range (ATR), Chitradurga in Karnataka. What does RLV stands for?
A) Reusable Launch Vehicle B) Rotating Launch Vehicle
C) Radial Launch Vehicle D) Revolutionary Launch Vehicle
Answer : A

Follow us: Official Site, Telegram, Facebook, Instagram, Instamojo 778


✓ The Indian Space Research Organisation (ISRO) successfully conducted the Reusable Launch Vehicle
Autonomous Landing Mission (RLV LEX) on April 02, 2023.
✓ The test was conducted at the Aeronautical Test Range (ATR), Chitradurga, in Karnataka.
✓ Under this mission, a winged body was carried to an altitude of 4.5 km by a helicopter and released for
carrying out an autonomous landing of a space vehicle on a runway.
✓ It was the first-of-its-kind mission in the world.
Q. The Jatayu Conservation and Breeding Centre (JCBC) has been set up as the world’s first conservation
and breeding centre for Asian king vulture in which state of India?
A) Bihar B) Maharashtra
C) Uttar Pradesh D) Tamil Nadu
Answer : C
✓ World’s first Jatayu Conservation and Breeding Centre set up in Maharajganj district of Uttar Pradesh.
✓ The Jatayu Conservation and Breeding Centre (JCBC) has been set up as the world’s first conservation
and breeding centre built and designed exclusively for the conservation of Asian king vulture.
✓ The JCBC is built at Farenda in Maharajganj district, located in the Gorakhpur Forest Division of Uttar
Pradesh.
✓ The objective of this centre is to breed king vultures in captivity and release them in the wild to maintain
a sustainable population of the species. The Asian king vulture is critically endangered and protected
under the Wildlife Protection Act.
✓ The state-of-the-art facility is worth around Rs 15 crore and spread over 1.5 hectares area.
✓ The JCBC is a 15-year project, and aims to raise at least 40 vultures.
WORLD'S FIRST IN NEWS 2023
▪ World’s first Jatayu Conservation and Breeding Centre set up in Maharajganj district of Uttar Pradesh.
▪ World’s first’ bamboo crash barrier installed on Maharashtra highway.
▪ The world's first museum of palm leaf manuscripts established in Thiruvananthapuram, Kerala.
▪ Visva-Bharati University is expected to get the status of the world's first "Living Heritage University"
from UNESCO by April-May 2023.
▪ World's first vaccine for honeybees approved for use by United States.

Q. Who has been selected for the 2023 International Prize in Statistics?
A) David Cox B) CR Rao
C) Bradley Efron D) Nan Laird
Answer : B
✓ Indian origin American Mathematician CR Rao win 2023 International Prize in Statistics.
✓ Calyampudi Radhakrishna Rao, a prominent Indian-American mathematician and statistician, has been
named for the 2023 International Prize in Statistics, the highest honor in the field.
✓ The International Prize in Statistics is awarded biennially by a collaboration among five leading
international statistics organisations, to recognise a major achievement by an individual or team in the
statistics field, particularly an achievement of powerful and original ideas that have led to practical
applications and breakthroughs in other disciplines.

Q. AS per the 2022 Tiger Census, what is the estimated number of tigers in India?
A) 3,167 B) 3,758
C) 3,816 D) 3,286
Answer : A

Follow us: Official Site, Telegram, Facebook, Instagram, Instamojo 779


✓ Prime Minister Shri Narendra Modi released the 2022 India’s Tiger Census report on April 09, 2023.
✓ As per the latest census report, the numbers of tigers in India showed an increase in 2022, and stands at
3,167.
✓ There were 2,967 tigers in India, as per the 2018 Tiger Census, released in July 2019.
✓ The 2022 Tiger Census shows that tiger’s population has increased by 200 in the past four years.
✓ This is 6.7 percent higher than the number of tigers during the 2018 census.
✓ It must be noted that the Tiger Census is conducted every four years in India, since 2006 by the National
Tiger Conservation Authority.
✓ The 2022 Tiger Census is the 5th cycle of the tiger evaluation in the country.
✓ The estimate is based only on camera-caught snapshots of tigers and does not consider camera-free tiger
areas, which may have otherwise further increased the tiger’s number.
✓ The report recorded a substantial increase in the tiger population in the Shivalik Hills and Gangetic plains
where camera traps captured 804 unique tigers compared with 646 tigers in 2018.
✓ Similarly, in the Sunderbans, 100 unique tigers were photographed in the 2022 census, compared to 88
tigers in 2018.
✓ Estimates of Tiger in Different Cycles of Tiger Evaluation
✓ 2006 – 1,411
✓ 2010 – 1,706
✓ 2014 – 2,226
✓ 2018 – 2,967
✓ 2022 – 3,167
Q. Who has become the first woman Air Force officer to receive the 'Gallantry Award'?
A) Bhawana Kanth B) Deepika Mishra
C) Avni Chaturvedi D) Shalija Dhami
Answer : B
✓ Wing Commander Deepika Mishra has become the first woman Air Force officer to receive the Gallantry
Medal.
✓ The award was announced by the President on the occasion of Independence Day last year.
✓ Wing Commander Mishra, a Rajasthan-based helicopter pilot, has been awarded the Vayu Sena Medal
(Gallantry) for "extraordinary courage" displayed during a flood relief operation in Madhya Pradesh
Q. Which country has emerged as the most populous country in the world as per the data by UNFPA?
A) China B) United States
C) India D) Indonesia
Answer : C
✓ According to a report by the United Nations Population Fund (UNFPA), India is now the most populous
country in the world.
✓ The UN report titled “The State of World Population Report, 2023” with the subtitle ‘8 Billion Lives,
Infinite Possibilities: The Case for Rights and Choices’, was released on April 19, 2023.
✓ The report shows that India’s population is recorded to be 1,428.6 million, while that of China is 1,425.7
million, making a difference of 2.9 million. .
✓ It will be for the first time that India’s population has overtaken China since 1950 when the UN began to
collect and release population data.
✓ Around 25% of India’s population comprise of people within the age group of 0-14, 18% in the age group
of 10-19, 26% in the age group of 10-24, 68% in the age group of 15-64, and 7% above the age of 65.

Follow us: Official Site, Telegram, Facebook, Instagram, Instamojo 780


✓ Talking about cities, New Delhi is the most populated city in India. About 30 million inhabitants reside
in and around NCR. It is followed by Mumbai (20 million), Kolkata (15 million) and Bengaluru (12
million)

Q. The government is committed to build a drug-free India by which year?


A) 2035 B) 2047
C) 2030 D) 2045
Answer : B
✓ The Union Minister, Amit Shah (Union Minister of Home Affairs and Minister of Cooperation)
inaugurated and addressed the first National Conference of Heads of Anti-Narcotics Task Forces of
States and Union Territories in New Delhi on April 18, 2023.
✓ He informed that the government is committed to “build a drug-free India by 2047”, the year in which
India will celebrate 100th year of independence.

Q. Which city has been ranked as the wealthiest city in the world by the Henley & Partners?
A) New York B) London
C) Singapore D) Tokyo
Answer : A
✓ The investment migration consultancy Henley & Partners has released the World’s Wealthiest Cities
Report for 2023, on April 18, which ranks the world’s wealthiest and fastest-growing cities.
✓ As per the annual report, New York City has secured the top spot as the wealthiest in the world. The city
comprised of 340,000 high-net-worth individuals in 2022.
✓ Among Indian cities, Mumbai is India’s richest city with 60,000 high-net-worth individuals (HNIs).
Globally Mumbai is ranked at 21st spot.
✓ List of World’s Top 10 Wealthiest Cities
1. New York City (340,000) 2. Tokyo (290,300)
3. San Francisco Bay Area (285,000) 4. London (258,000)
5. Singapore (240,100) 6. Los Angeles (205,400)
7. Hong Kong (129,500) 8. Beijing (128,200)
9. Shanghai (127,200) 10. Sydney (126,900)

Q. Who has been selected for the Lata Deenanath Mangeshkar award for 2023?
A) Asha Bhosle B) Narendra Modi
C) KL Rahul D) Raghuram Rajan
Answer : A
✓ Bollywood’s Legendary singer Asha Bhosle has been named for the Lata Deenanath Mangeshkar award
for 2023.
✓ She will receive the award on April 24, 2023, which marks the memorial day of theatre-music veteran
Deenanath Mangeshkar.
✓ The Lata Deenanath Mangeshkar award has been instituted in the memory of Lata Mangeshkar, who
died on February 6, 2022.
✓ This award is given to an individual who has made a path-breaking contribution towards the nation, its
people and society.

Follow us: Official Site, Telegram, Facebook, Instagram, Instamojo 781


✓ Prime Minister Shri Narendra Modi was the first recipient of this award.
Q. Which city will host the 2023 Intercontinental Football Cup?
A) Chennai B) Bhubaneswar
C) Hyderabad D) Bengaluru
Answer : B
✓ The 2023 Intercontinental Football Cup will be held in Bhubaneswar, Odisha, from June 9 to 18, 2023.
✓ This will be the third edition of the Intercontinental Cup, organized by the AIFF.
✓ It will be a 4-nation football tournament, including India, Lebanon, Vanuatu and Mongolia.
✓ The first edition of the tournament was won by India in 2018, by defeating Kenya in the Final by 2–0.
Q. Which among these satellites were recently launched by SpaceX under its most recent Starlink mission?
A) Starlink “V1 mini” satellites B) Starlink “V2.5 mini” satellites
C) Starlink “V1.5 mini” satellites D) Starlink “V2 mini” satellites
Answer : D
✓ Elon Musk-led SpaceX has launched a new batch of 21 Starlink satellites into the orbit, from Florida’s
Cape Canaveral Space Force Station on April 19, 2023.
✓ The new Starlink “V2 mini” satellites were launched on a Falcon 9 rocket.
✓ This launched marked the 25th launch of SpaceX in 2023 so far.
✓ The “Starlink V2 Mini” satellites are launched under the mission known as Starlink 6-2.
✓ These satellites are fitted with improved phased array antennas with four times the communications
capacity of earlier generations of Starlink satellites, known as Version 1.5.
✓ SpaceX has already launched over 4,000 Starlink satellites in orbit.
Q. Who is the author of the book ‘The bee, the beetle, and the money bug- The bank bazaar guide to the
financial world’?
A) Arjun Shetty and IYPE ISAC B) Ramesh Shetty and Sriram V
C) Adhil Shetty and AR Hemant D) Rati Shetty and Pankaj Bansal
Answer : C
✓ A new book titled ‘The bee, the beetle, and the money bug- The bank bazaar guide to the financial world’
has been released.
✓ It has been authored by Adhil Shetty (CEO and Co-founder of Bankbazaar.com) and AR Hemant (former
journalist and now head of communications at Bankbazaar.com).
✓ The book emphasises on the advantage of starting investments at an early age.

Q. The first Global Buddhist Summit was recently held in which city?
A) New Delhi B) Hyderabad
C) Patna D) Gurugram
Answer : A
✓ Prime Minister Shri Narendra Modi inaugurated the first Global Buddhist Summit (GBS) in New Delhi
on April 20, 2023.
✓ The two-day Summit has been organised by the Ministry of Culture in collaboration with the
International Buddhist Confederation (IBC) .
✓ The theme of the Summit is “Responses to Contemporary Challenges: Philosophy to Praxis.”

Follow us: Official Site, Telegram, Facebook, Instagram, Instamojo 782


✓ The Summit will help to enhance the significance and importance of India in Buddhism, as Buddhism
was born in India.
Q. Name the winner of the Malcolm Adiseshiah Award for 2023?
A) Raja Jesudoss Chelliah B) Varadaraja Rao
C) Utsa Patnaik D) Jagdish Bhagwati
Answer : C
✓ Utsa Patnaik, a renowned Indian Marxian economist has been selected for the Malcolm Adiseshiah
Award 2023.
✓ The Malcolm Adiseshiah Award is given every year by the Malcolm & Elizabeth Adiseshiah Trust since
2000, for Distinguished Contributions to Development Studies.
✓ The award comes with a prize money of ₹2 lakh and a citation
Q. Who has launched the 'SATHI Portal and Mobile App'?
A) S. Jaishankar B) Parshottam Rupala
C) Nirmala Sitharaman D) Narendra Singh Tomar
Answer : D
✓ Agriculture and Farmers Welfare Minister Narendra Singh Tomar on 19th April launched the SATHI
(Seed Traceability, Authentication and Holistic Inventory) Portal and Mobile App to deal with the
challenges of seed production, quality seed identification, and seed certification.
✓ It has been created by NIC in collaboration with the Union Ministry of Agriculture and Farmers Welfare
on the theme of 'Best Seeds - Prosperous Farmer'.
✓ SATHI portal is also an important step in this direction. When its use starts till the bottom, it will prove
to be a revolutionary step in the field of agriculture.

Q. Who received Germany's highest Order of Merit from the German President Frank-Walter Steinmeier?
A) Joachim Sauer B) Angela Merkel
C) Irene Kasner D) Annalena Baerbock
Answer : B
✓ Former Chancellor "Angela Merkel" received Germany's highest Order of Merit.
✓ The Grand Cross award was handed to her by President Frank-Walter Steinmeier.
✓ The award has only been given twice before, to former chancellors Konrad Adenauer and Helmut Kohl.
✓ All three former leaders belong to the conservative Christian Democratic Union (CDU).
✓ Merkel led Germany from 2005 to 2021, totaling four terms in office. She did not seek a fifth term.
✓ She was the first woman to take the post of chancellor and the first German head of government to have
grown up in socialist East Germany..
✓ Former German chancellor Angela Merkel has been awarded the Felix Houphouet-Boigny UNESCO
Peace Prize in Yamoussoukro, Ivory Coast.
Q. Who has become the first Indian woman to win Wisden Almanack’s Leading Cricketer of the Year
Award 2022?
A) Smriti Mandhana B) Harmanpreet Kaur
C) Jemimah Rodrigues D) Shafali Verma
Answer : B

Follow us: Official Site, Telegram, Facebook, Instagram, Instamojo 783


✓ Wisden Cricketers Almanack has released the list of top-5 cricketers of the year 2022. It also includes
India's women's team captain Harmanpreet Kaur and Suryakumar Yadav of the men's team.
✓ Apart from Kaur, the list also includes the names of Tom Blundell (New Zealand), Ben Foakes
(England), Daryl Mitchell (New Zealand) and Matthew Potts (England).
✓ Suryakumar Yadav has been selected as the T20 Cricketer of the Year.
✓ Harmanpreet Kaur created history by becoming the first Indian woman to be named in Wisden
Cricketers of the Year.
✓ Apart from this, Ben Stokes of England has been selected as the best cricketer in the world for the year
2022-23 among male players.
✓ At the same time, Beth Mooney (Australia) has received this award among women cricketers.
Suryakumar Yadav has been selected as the T20 Cricketer of the Year.
Q. ISRO will launch the Earth Observation satellite TeLEOS-2 from the Satish Dhawan Space Centre in
Sriharikota . TeLEOS-2 is a satellite of which country?
A) Vietnam B) Singapore
C) Ethiopia D) Switzerland
Answer : B
✓ ISRO will launch Singapore's TeLEOS-2 satellite using the Polar Satellite Launch Vehicle (PSLV) from
the Satish Dhawan Space Center in Sriharikota.
✓ TeLEOS-1, Singapore's first commercial Earth observation satellite, was launched by ISRO in 2015 and
since then, ISRO has launched nine satellites for Singapore.
✓ TeLEOS-2 is an Earth observation satellite built by ST Engineering that will carry a Synthetic Aperture
Radar (SAR) capable of providing 1-metre resolution data. The satellite will have a 500 GB onboard
recorder and 800 Mbps downlink.
Q. IIT Madras has planned to set up its first International campus in which country?
A) Tanzania B) Kenya
C) Senegal D) Ethiopia
Answer : A
✓ The IIT Madras has planned to set up its first International campus in Zanzibar, Tanzania.
✓ This will be the first Indian Institute of Technology (IIT) to be set up in an African country.
✓ IIT Madras observed its 64th Institute Day on April 20, 2023.
✓ About Tanzania
✓ Tanzania, East African country situated just south of the Equator.
✓ President: Samia Suluhu Hassan
✓ Capital: Dodoma
✓ Currency: Tanzanian shilling
Q. Who is the author of the book titled “SACHIN@50: Celebrating a Maestro”?
A) Ravi Chaturvedi B) Jatin Sapru
C) Sushil Doshi D) Boria Majumdar
Answer : D
✓ Boria Majumdar, one of India’s most influential sports commentators and columnist has come out with
a new book on the legendary cricket star Sachin Ramesh Tendulkar.
✓ The book titled “SACHIN@50: Celebrating a Maestro” will be released on April 24, 2023, the day when
Sachin will mark his 50th birthday.

Follow us: Official Site, Telegram, Facebook, Instagram, Instamojo 784


✓ This book is a collection of the remarkable achievements of Tendulkar, the ‘God of cricket’ as a tribute.
✓ Playing It My Way is the autobiography of former Indian cricketer Sachin Tendulkar.
Q. The National Highways Authority of India (NHAI) is committed to develop around 10,000 kilometres
of Optic Fibre Cables (OFC) infrastructure across the country by which year?
A) 2025 B) 2024
C) 2023 D) 2026
Answer : A
✓ The National Highways Authority of India (NHAI) aims to develop around 10,000 kilometres of Optic
Fibre Cables (OFC) infrastructure across the country by 2024-2025.
✓ This initiative will help in providing internet connectivity to remote locations and expediting the roll-out
of new-age telecom technologies like 5G & 6G.
✓ The National Highways Logistics Management Limited (NHLML), NHAI’s fully-owned special
purpose vehicle, will implement the network of “digital highways” by developing integrated utility
corridors along the national highways to develop OFC infrastructure.
Q. INIOCHOS-23 Exercise will be conducted between India and which country from 24 Apr 2023 to 4 May
2023?
A) China B) Greece
C) Ethiopia D) Norway
Answer : B
✓ The Indian Air Force (IAF) will be participating in Exercise INIOCHOS-23, a multi-national air exercise
hosted by the Hellenic Air Force, the air force of Greece.
✓ The exercise will be conducted at the Andravida Air Base in Greece from 24 Apr 2023 to 4 May 2023.
✓ The Indian Air Force will be participating with four Su-30 MKI and two C-17 aircraft.
✓ The exercise will be conducted in a realistic combat scenario involving a wide variety of air and surface
assets.
✓ Objective of Exercise INIOCHOS-23.
✓ The exercise aims to enhance international cooperation, synergy and interoperability among the
participating air forces.

Q. Who launched the world's largest and most powerful rocket 'Starship' Key launching failure?
A) ISRO B) European Space Agency
C) NASA D) SpaceX
Answer : D
✓ SpaceX’s next-generation Starship, the world’s biggest rocket, exploded during its first test-flight to space
near Brownsville, Texas.
✓ SpaceX’s Starship spacecraft and Super Heavy rocket– collectively referred to as Starship.
✓ Super Heavy rocket is the first stage, or booster, of the Starship launch system.
✓ It is powered by 33 Raptor engines using sub-cooled liquid methane (CH4) and liquid oxygen (LOX).

SPACEX IN NEWS 2023


▪ Elon Musk sets low expectations before first SpaceX launch of Starship, most powerful rocket ever .
▪ SpaceX launched NASA’s Crew-6 mission to orbit en route to the International Space Station, with a
Russian cosmonaut and United Arab Emirates astronaut joining two NASA crewmates for the flight..
▪ SpaceX Sends First Saudi Arabian Astronauts Rayyanah Barnawi to the International Space Station.

Follow us: Official Site, Telegram, Facebook, Instagram, Instamojo 785


▪ SpaceX Launches 51 Starlink satellites from California.
▪ SpaceX Awarded shared NASA contract worth up to $100 million.
▪ Airtel-backed OneWeb successfully launched and deployed 40 satellites onboard a SpaceX launcher.

Q. International Mother Earth Day is observed on which day of the year?


A) 20 April B) 22 April
C) 21 April D) 19 April
Answer : B
✓ World Earth Day is an annual event celebrated on 22 April to show support for environmental protection.
✓ Its purpose is to create awareness among people about environmental protection.
✓ The first Earth Day was held on April 22, 1970, and is now coordinated globally by EARTHDAY.ORG
(formerly Earth Day Network).
✓ The event is celebrated by over 1 billion people in over 193 countries.
✓ The official theme for World Earth Day 2023 is "Invest in our Planet".
✓ The idea of Earth Day was proposed by peace activist John McConnell at a UNESCO conference in San
Francisco in 1969.
✓ The first Earth Day was celebrated on March 21, 1970, and later changed to April 22, 1970 by United
States Senator Gaylord Nelson
✓ World Earth Day was initially focused on the United States, but became an international event in 1990
with events held in 141 countries.
✓ The landmark Paris Agreement was signed by the United States, the United Kingdom, China and 120
other countries on Earth Day 2016.
Q. The book titled ‘Crosscourt’ is the autobiography of which sports person?
A) Zeeshan Ali B) Naresh Kumar
C) Ramesh Krishnan D) Jaidip Mukerjea
Answer : D
✓ Legendary Indian tennis player Jaidip Mukerjea has come out with his autobiography titled
“Crosscourt”.
✓ The book brings out the journey of Mukerjea, in which he shares his life experiences of tennis career,
which may act as motivation for future generations to take up the sport.
Q. What is the rank of India in the World Bank’s Logistic Performance Index 2023?
A) 38 B) 44
C) 25 D) 32
Answer : A
✓ India has been ranked at 38th position among 139 countries in the World Bank’s Logistic Performance
Index 2023, which is a measure of countries’ ability to move goods across borders with speed and
reliability.
✓ India has improved its ranking by 6 places, than its previous rank of 44 in 2018.
✓ The Logistics Performance Index by World Bank helps developing countries to identify where
improvements can be made to boost competitiveness.
✓ Singapore has topped the index. It is followed by Finland at second position

Q. Which country has topped the World Bank’s Logistic Performance Index 2023?

Follow us: Official Site, Telegram, Facebook, Instagram, Instamojo 786


A) Finland B) Singapore
C) Norway D) Switzerland
Answer : B
✓ Singapore has topped the index. It is followed by Finland at second position.
Q. The world’s largest cyber defence exercise named ‘Locked Shields-2023’ was conducted by which alliance
recently?
A) NATO B) SAARC
C) G-20 D) SCO
Answer : A
✓ NATO conducted the world’s largest cyber defence exercise named ‘Locked Shields-2023’ in Tallinn,
Estonia from April 18 to April 21, 2023.
✓ The four-day exercise saw participation of more than 3,000 IT-specialists from 38 nations.
✓ The event is organised every year since 2010, by NATO Cooperative Cyber Defense Center of Excellence
(CCDCOE).
✓ The purpose of this exercise is to boost the strategical and technical abilities of the participants in fighting
cyber attacks.
Q. Name the winner of World Press Photo of the Year award for 2023?
A) Anthony Suau B) Evgeniy Maloletka
C) Spencer Platt D) Tim Hetherington
Answer : B
✓ Ukrainian journalist and photographer, Evgeniy Maloletka, from the Associated Press has won the
World Press Photo of the Year award for 2023.
✓ He has won the image, which shows Iryna Kalinina, a 32-year-old wounded pregnant being carried on a
stretcher by emergency workers, through the shattered grounds of a maternity hospital in the Ukrainian
city of Mariupol, that was damaged during a Russian airstrike.
✓ The annual photo contest is awarded by World Press Photo Foundation which rewards photographers
for the best single exposure pictures contributing to the past year of visual journalism.

Q. Which state government has launched a ‘Salokha Yojana’ in the state?


A) Gujarat
B) Rajasthan
C) Maharashtra
D) Uttar Pradesh
Answer : C
✓ The Maharashtra government launched a ‘Salokha Yojana’ in the state.
✓ This scheme provides an opportunity for farmers to exchange their agricultural land in lieu of a nominal
registration fee of Rs 1,000 and a stamp duty of Rs 1,000.
✓ The aim of the scheme is to resolve disputes between farmers regarding land ownership and possession,
promote harmony in society and increase peace and harmony among them.

Q. Who has been selected as a finalist for the Aster Guardians Global Nursing Award 2023?

Follow us: Official Site, Telegram, Facebook, Instagram, Instamojo 787


A) Shanti Lakra B) Shanti Lal
C) Kajal Sharma D) Ramesh Kandula
Answer : A
✓ Shanti Lakra, a nurse from India, has been selected as a finalist for the Aster Guardians Global Nursing
Award 2023.
✓ The winner of the award will receive a cash prize of $25,000 and will be announced in May 2023.
✓ The award recognizes and honors exceptional nurses from around the world who have made significant
contributions to patient care.
✓ Lakra works as a nurse in a hospital in Port Blair.
Q. In India, which day is celebrated as the National Panchayati Raj Day?
A) 22 April B) 21 April
C) 23 April D) 24 April
Answer : D
✓ Panchayati Raj Day is celebrated every year on 24th April to commemorate the enactment of the 73rd
Amendment Act, 1992 of the Constitution in the year 1993.
✓ The day is celebrated to acknowledge national local self-governance and democratic decentralisation.
✓ The theme of National Panchayati Raj Day 2023 is 'Sustainable Panchayat: Building Healthy, Water
Sufficient, Clean & Green Villages.'
✓ The Ministry of Panchayati Raj is the nodal ministry responsible for empowering, enabling and ensuring
accountability of Panchayati Raj Institutions for achieving inclusive growth with social justice and
efficient service delivery.
✓ Prime Minister Narendra Modi attended the National Panchayati Raj Day celebrations in Madhya
Pradesh and laid the foundation stone and inaugurated projects worth over Rs 17,000 crore.
Q. Who has launched new features in eShram Portal ?
A) Ashwini Vaishnaw B) Bhupender Yadav
C) Kiren Rijiju D) Mansukh L. Mandaviya
Answer : B
✓ Union Minister for Labour & Employment and Environment, Forest and Climate Change, Bhupender
Yadav, launched new features in eShram Portal on 24th April
✓ The new features added in eShram portal will enhance the utility of the portal and facilitate ease of
registration for unorganised workers.
✓ The eShram registered workers can now connect with Employment opportunities, Skilling,
Apprenticeship, Pension Scheme, Digital Skilling and States’ schemes through eShram portal.
✓ The Union Minister also formally launched the Data Sharing Portal (DSP) for sharing of eShram data
with the State/ UT Governments.
✓ What are new features added?
✓ Facility to enter details of families of migrant workers has been added in the e-Shram Portal.
✓ This facility can help migrant workers migrating with family to get benefits of child education and women
centric schemes.
✓ In addition, a new feature has been added regarding sharing of data of construction workers registered
on Eshram with the concerned Building and Other Construction Workers (BOCW) Welfare Board.
✓ It will ensure registration of Ishram construction workers with the concerned BOCW board and ensure
access to schemes for them.
✓ About e-Shram portal

Follow us: Official Site, Telegram, Facebook, Instagram, Instamojo 788


✓ The Ministry of Labor and Employment has developed an eSHRAM portal to create a national database
of unorganised workers.
✓ It was launched in 2021.
✓ Its aim is to welfare of workers in the unorganised sector who are not members of the EPFO or ESIC.
✓ Registered members will be eligible for a range of benefits after signing up for Shramik Yojana and getting
the e-Shram Card.
✓ This will allow workers to receive Direct Benefit Transfer (DBT) through e-Shramik Unique ID Number.

Q. Which country refinery to be built by India by 2025?


A) Thailand B) Cambodia
C) Mongolia D) Philippines
Answer : C
✓ Mongolia will have its first oil refinery up and running by 2025.
✓ The first phase of the $1.2 billion Indian soft loan-funded Mongol Oil Refinery will be finished by the
end of the year.
✓ Given that Russia is the only source of energy imports for Mongolia, the development has strategic
ramifications.
✓ Mongolia refinery to be built by India by 2025 :
✓ The refinery will enable Mongolia to satisfy 70% of its domestic demand.
✓ According to the nation’s ambassador, Dambajav Ganbold, the first oil refinery in Mongolia, which is
being constructed on the outskirts of the capital city of Ulaanbaatar with funding from India, would be
finished by 2025
Q. Ministry of Health plans to launch how many food streets across country to promote hygiene and safety?
A) 75
B) 100
C) 150
D) 200
Answer : B
✓ The Health Ministry, in collaboration with the Ministry of Housing and Urban Affairs, has asked States
and Union Territories to develop 100 food streets in 100 districts across the country.
✓ Aim : The aim of this project is to encourage safe and healthy practices to reduce food-borne illnesses
and improve overall health outcomes, said the Health Ministry.
✓ The initiative is being taken up as a pilot project to create an example for best practices for food
businesses.
✓ The communication further said that safe food practices would not only boost the “eat right campaign”
and food safety but also improve the hygiene credibility of local food businesses.
✓ The initiative will be implemented through the National Health Mission (NHM) in convergence with the
Ministry of Housing and Urban Affairs and with technical support from the Food Safety and Standards
Authority of India (FSSAI).
✓ Financial assistance of ₹1 crore per food street/district will be given to States and Union Territories.
✓ The project envisages 100 such food streets in 100 districts across the country.
✓ Assistance will be provided under the NHM in the ratio of 60:40 or 90:10, with the condition that
standard branding of these food streets will be done as per FSSAI guidelines..

Q. Who took over as the Secretary of the Department for Promotion of Industry and Internal Trade (DPIIT)?
A) Sanjay Sinha B) Rajesh Kumar Singh

Follow us: Official Site, Telegram, Facebook, Instagram, Instamojo 789


C) Ashok Khemka D) Nripendra Mishra
Answer : B
✓ Senior IAS officer Rajesh Kumar Singh assumed the charge of Secretary in the Department for
Promotion of Industry and Internal Trade (DPIIT).
✓ He succeeds Anurag Jain, who was appointed Secretary of the Ministry of Road Transport and
Highways. Earlier, IAS Rajesh worked as a Secretary in the Department of Animal Husbandry and
Dairying.
✓ The Department for Promotion of Industry and Internal Trade functions under the Ministry of
Commerce and Industry.

Q. The iconic Sydney Cricket Ground unveiled a gate named after which player?
A) Kapil Dev B) Mahendra Singh Dhoni
C) Sachin Tendulkar D) Shane Warne
Answer : C
✓ A gate named after legendary cricketer Sachin Tendulkar was unveiled at Australia's iconic Sydney
Cricket Ground in his honour on his 50th birthday. Tendulkar turned 50 on April 24.
✓ Sachin played an unbeaten historical inning of 241 runs in the Test at the Sydney Cricket Ground in 2004
and he also scored 785 runs including 3 centuries in that series.
✓ This ground was one of Sachin's favourite cricket grounds
Q. Which State Gandhisagar Sanctuary to be second new home of Cheetahs?
A) Kerala B) Uttar Pradesh
C) Karnataka D) Madhya Pradesh
Answer : D
✓ Madhya Pradesh's Gandhisagar Sanctuary will be developed as a second home for cheetahs within six
months to retain the tag of cheetah state, Madhya Pradesh chief minister Shivraj Singh Chouhan.
✓ The move comes after experts suggested that Kuno National Park (KNP) does not have enough space
for all cheetahs.
Q. Which state in India has topped in terms of number of water bodies, as reported in the first-ever census
of water bodies across the nation conducted by the Union Ministry of Jal Shakti?
A) Sikkim B) Maharashtra
C) West Bengal D) Uttarakhand
Answer : C
✓ Union Ministry of Jal Shakti conducted the first-ever census of water bodies across the nation, in order
to have a comprehensive national database of all water bodies.
✓ The basic objective of this census was to provide a comprehensive inventory of India’s water resources,
including natural and man-made water bodies like ponds, tanks, lakes, and more, collect data on the
encroachment of water bodies, highlight disparities between rural and urban areas and varying levels of
encroachment, reveal crucial insights into the country’s water resources
✓ The key features of the Census
✓ 24,24,540 water bodies have been enumerated in the country. Of these, 97.1% (23,55,055) are in rural
areas and only 2.9% (69,485) are in urban areas.
✓ Top 05 States in terms of number of water bodies (constituting around 63% of the total water bodies)

Follow us: Official Site, Telegram, Facebook, Instagram, Instamojo 790


1. West Bengal, 2. Uttar Pradesh,
3. Andhra Pradesh, 4. Odisha
5. Assam
✓ Top 05 States in terms of number of water bodies in urban areas – West Bengal, Tamil Nadu, Kerala,
Uttar Pradesh and Tripura
✓ 59.5% of water bodies are ponds, followed by tanks (15.7%), reservoirs (12.1%), Water conservation
schemes/percolation tanks/check dams (9.3%), lakes (0.9%) and others (2.5%).
✓ Among public owned water bodies, maximum water bodies are owned by Panchayats, followed by State
Irrigation/State WRD.
✓ Among all private owned water bodies, maximum water bodies are owned by Individual owner/farmer
followed by group of individuals and other private bodies.
✓ Top 05 States which lead in the private owned water bodies – West Bengal, Assam, Andhra Pradesh,
Odisha and Jharkhand
✓ Among all ‘in use’ water bodies, mostly water bodies are used in pisciculture followed by Irrigation.
✓ Top 05 States with major use of water bodies in pisciculture – West Bengal, Assam, Odisha, Uttar
Pradesh and Andhra Pradesh;
✓ Top 05 States with major use of water bodies in irrigation – Jharkhand, Andhra Pradesh, Telangana,
West Bengal and Gujarat.
✓ 78% water bodies are man-made water bodies whereas 22% are natural water bodies.
✓ 1.6% (38,496) water bodies out of all the enumerated water bodies are reported to be encroached. Out of
them, 95.4% are in rural areas and remaining 4.6% in urban areas.
✓ Union Minister for Jal Shakti – Shri Gajendra Singh Shekhawat.
Q. Name the e-commerce web portal launched by Centre for artisans and weavers, so that they can sell their
handloom and handicraft products directly to customers without any middlemen
A) Indiacrafthouse.com B) Itokri.com
C) Indiahandmade.com D) HandicraftsInIndia.in
Answer : C
✓ The Central Government has launched an e-commerce portal named ‘Indiahandmade.com’, for artisans
and weavers, so that they can sell their handloom and handicraft products directly to customers without
any middlemen, and get fair remuneration.
✓ The portal will not charge any commission from sellers. It shall also provide free shipping with a return
option to customers along with multiple payment gateways to pay online.
✓ Products from more than 35 lakh handloom weavers and 27 lakh handicraft artisans will be displayed on
the portal from options like clothing, home decor, jewellery, accessories, and other products handmade
by skilled artisans
Q. The Indian Council of Agricultural Research – National Dairy Research Institute (ICAR-NDRI)
celebrated its 19th Convocation on April 24, 2023. The institute is located at which place?
A) Chennai B) Port Blair
C) Jodhpur D) Karnal
Answer : D
✓ The President of India, Smt. Droupadi Murmu addressed the 19th convocation of the Indian Council of
Agricultural Research – National Dairy Research Institute (ICAR-NDRI) at Karnal, Haryana on April
24, 2023.
✓ The dairy sector contributes about 5 percent to the country’s GDP.

Follow us: Official Site, Telegram, Facebook, Instagram, Instamojo 791


Q. India’s Jyothi Surekha Vennam has won gold medal for the country in individual event of which sports
recently?
A) Archery B) Badminton
C) Tennis D) Shooting
Answer : A
✓ The stage -I of the 2023 Archery World Cup was held from 18-23 April at Antalya, Turkey.
✓ India managed to win 4 medals which included 4 gold, 1 silver and 1 bronze.
✓ Jyothi Surekha Vennam won the gold medal in Women’s compound individual event
✓ The duo of Jyothi Surekha Vennam and Ojas Pravin Deotale won in mixed compound team event.
✓ The silver medal was won by the Indian men’s recurve team after losing to China in a shoot-off.
✓ The bronze medal came from Dhiraj Bommadevara, in men’s recurve individual

Q. Which of the following has been chosen as the World Book Capital for the year 2023 by UNESCO?
A) Accra B) Abuja
C) Dakar D) Cairo
Answer : A
✓ As a part of the Day, UNESCO selects the World Book Capital every year for a one-year period, effective
23 April each year. The World Book Capital for 2023 is Accra, Ghana.
✓ Director-General of UNESCO, Audrey Azoulay, has designated Strasbourg (France) as World Book
Capital for 2024..
Q. PM Modi has laid the foundation stone for India’s first Digital Science Park coming up at the Technocity
campus in which city?
A) Thiruvananthapuram B) Chennai
C) Bengaluru D) Hyderabad
Answer : A
✓ Prime Minister Narendra Modi has laid the foundation stone for India’s first Digital Science Park coming
up at the Technocity campus in Thiruvananthapuram, Kerala.
Q. The World Malaria Day (WMD) is observed globally on which day?
A) 24th April B) 26th April
C) 23rd April D) 25th April
Answer : D
✓ The World Malaria Day (WMD) is observed globally on 25 April each year to recognize the efforts of
the people worldwide to control malaria.
✓ The Day was established in May 2007 at the 60th session of the World Health Assembly, WHO’s
decision-making body.
✓ Theme of World Malaria Day 2023 is ‘Reaching the Zero Malaria Target’.

Q. Which state has launched the ‘One Panchayat, One Playground’ project to revive the sports culture in
the state?
A) Haryana B) Meghalaya

Follow us: Official Site, Telegram, Facebook, Instagram, Instamojo 792


C) Kerala D) Uttar Pradesh
Answer : C
✓ Kerala Government launches ‘One Panchayat, One Playground’ to revive sports culture.
✓ The Kerala Chief Minister Pinarayi Vijayan has launched a project dubbed as ‘One Panchayat, One
Playground’ for reviving the sports culture in the State, by opening quality playgrounds in every
panchayat of the southern state.

Q. The Khongjom Day is marked every year in which state?


A) Nagaland B) Manipur
C) Tripura D) Meghalaya
Answer : B
✓ In Manipur, the Khongjom Day was observed on April 23, 2023 at Khongjom in Thoubal district.
✓ The day remembers the battle fought in 1891 Anglo Manipuri War.
✓ The colonial British army, well equipped with modern weapons, started war with the people of the hills
and valley of Manipur.
Q. With which country did India announce making the 'Net Zero' Innovation Virtual Center?
A) UK B) USA
C) Japan D) Brazil
Answer : A
✓ India and the United Kingdom have agreed to create India-UK NET zero innovation virtual centre.
✓ This centre will provide a platform for stakeholders of both countries to work together in areas like the
decarbonization of manufacturing processes, transport systems, and green hydrogen as a renewable
source.
Q. The Mana village has been designated as ‘India’s first village’ by BRO, as it is situated on the India-China
border. In which state is this village located?
A) Himachal Pradesh B) Uttarakhand
C) Sikkim D) Arunachal Pradesh
Answer : B
✓ Mana village on India-China border in UK’s Chamoli district declared as ‘India’s first village’ by BRO.
✓ The Border Road Organisation (BRO) of India has put up a signboard saying ‘India’s first village’ in
Mana, a village in Uttarakhand.
✓ The Mana village is on the India-China border in Chamoli District.
✓ This decision is based on the initiative by PM Modi to declare all border villages in the country as the
first villages and not the last as they are usually called.
✓ Mana village is situated near Himalayan temple, Badrinath.
✓ Devotees going to the temple usually go up to the village for sightseeing.

Q. Manamadurai pottery has recently received the Geographical Indication (GI) Tag. Manamadurai is a
pottery of which state?
A) Kerala B) Tamil Nadu

Follow us: Official Site, Telegram, Facebook, Instagram, Instamojo 793


C) Karnataka D) Telangana
Answer : B
✓ Manamadurai pottery, a traditional form of pottery from Tamil Nadu, has received the Geographical
Indication (GI) tag.
✓ Manamadurai in Sivagangai district in Tamil Nadu is known for pottery making.
✓ The pottery is known for its distinctive style, which features intricate designs and vibrant colors.

Q. Name the winner of 2023 London Marathon in men’s event?


A) Mosinet Geremew B) Kenenisa Bekele
C) Birhanu Legese D) Kelvin Kiptum
Answer : D
✓ Kelvin Kiptum of Kenya has won the 2023 London Marathon in men’s category, with the second fastest
record time of 2 hours, 1 minute, 25 seconds.The marathon was held on April 23, 2023.
✓ The 23-year-old Kenyan runner finished the race just 16 seconds slow than the world record set by Eliud
Kipchoge in 2022 at Berlin Marathon at 2:01:09.
✓ In women’s category, Ethiopia-born Dutch distance-running champion Sifan Hassan won by finishing
the marathon in 2:18:33.
✓ TCS is beginning a six year deal as title sponsor of the London Marathon.
Q. Which country is helping Mongolia in building and financing its first greenfield oil refinery in the capital
Ulaanbaatar?
A) India B) Russia
C) Japan D) Saudi Arabia
Answer : A
✓ In Mongolia, India is helping to finance and construct country’s first greenfield oil refinery in the capital
Ulaanbaatar.
✓ The cost of the Mongol Oil Refinery project is estimated at $1.35 billion
✓ The project is proposed to be completed by 2025.
✓ The refinery would help Mongolia to meet 70% of its oil demand domestically.

Q. Who will be honoured with Billboard’s ‘Latin Women of the Year’ Award?
A) Taylor Swift B) Ariana Grande
C) Justin Bieber D) Shakira
Answer : D
✓ Colombian singer Shakira will be honoured with Billboard’s ‘Latin women of the year’ award at the first-
ever Mujeres Latinas en la Musica, or Latin Women in Music, gala.
✓ Shakira has sold over 70 million records worldwide and has won three Grammy Awards and 12 Latin
Grammy Awards.
✓ Shakira has earned the title of ‘Queen of Latin Music’ for her outstanding tracks over the course of three
decades in her career.
Q. Who has bagged Steel Ministry's "Ispat Rajbhasha Samman" for the year 2021-22?
A) RANL B) BINL
C) RINL D) RIML

Follow us: Official Site, Telegram, Facebook, Instagram, Instamojo 794


Answer : C
✓ Rashtriya Ispat Nigam (RINL) bagged Steel Ministry’s "Ispat Rajbhasha Samman" First Prize for the
year 2021-22.
Q. India will host the Shanghai Cooperation Organisation (SCO) Defence Ministers' Meeting on 28 April
2023 in Which City?
A) Mumbai B) New Delhi
C) Hyderabad D) Bengaluru
Answer : B
✓ India as the Chair of Shanghai Cooperation Organisation (SCO) in 2023 will host the SCO Defence
Ministers’ Meeting on 28 April 2023 in New Delh
✓ The theme of India’s Chairmanship of SCO in 2023 is ‘SECURE-SCO’. India attaches special
importance to SCO in promoting multilateral, political, security, economic and people-to-people
interactions in the region.
✓ The SCO was created in 2001 based on the “Shanghai five” formed after the signing by Kazakhstan,
Kyrgyzstan, China, Russia and Tajikistan.

Q. AJEYA WARRIOR is an annual joint military exercise of Indian Army with which country?
A) Singapore B) Malaysia
C) United States D) United Kingdom
Answer : D
✓ The 7th edition of joint military exercise between India and the United Kingdom, named “AJEYA
WARRIOR-23” is being conducted at Salisbury Plains, United Kingdom from 27 April to 11 May 2023.
✓ The biennial training exercise AJEYA WARRIOR is conducted alternatively in the United Kingdom
and India.
✓ The previous edition was held at Chaubatia, Uttarakhand in October 2021.
✓ The Indian Army soldiers from the BIHAR Regiment are participating in the exercise, while the UK will
be represented by the Soldiers of the 2 Royal Gorkha Rifles.
✓ Other Exercises between India and United Kingdom
✓ Exercise Konkan ( naval exercise )
✓ Exercise ‘Indradhanush’ (Air Force Exercise)
✓ Ajeya Warrior (Joint Military Exercise)
Q. The Union Cabinet has approved the establishment of how many new Nursing Colleges in the country?
A) 142 B) 157
C) 169 D) 178
Answer : B
✓ The Cabinet Committee on Economic Affairs, chaired by the Prime Minister Shri Narendra Modi, has
approved to set up 157 new nursing colleges in co-location with the existing medical colleges established
since 2014.
✓ The new nursing college add approximately 15,700 nursing graduates every year.
✓ The government has approved total financial allocation of Rs.1,570 crore for the project. The plan is to
complete the project within next two years.
✓ In the first place, this initiative will ensure quality, affordable, and equitable nursing education in India,
especially in underserved districts and States/Union Territories, and secondly, it will help in
strengthening nursing workforce in the country.

Follow us: Official Site, Telegram, Facebook, Instagram, Instamojo 795


✓ The highest number of colleges will be in Uttar Pradesh (27), Rajasthan (23), and Madhya Pradesh (14).
Q. How many medical device parks will be set up under the National Medical Device Policy?
A) 7 B) 6
C) 5 D) 4
Answer : D
✓ Cabinet approves National Medical Devices Policy, 2023 to increase global market share to 50 billion by
2030.
✓ The Union Cabinet has approved the National Medical Devices Policy, 2023, to facilitate growth in the
medical device sector and thereby meet the public health objectives of access, affordability, quality and
innovation.
✓ Cabinet has also initiated implementation of Production Linked Incentive (PLI) Scheme for Medical
Devices.
✓ Under the PLI scheme, a total of 26 projects have been approved with a committed investment of Rs
1,206 crore. Out of this, an investment of Rs 714 crore has already been achieved.
✓ Besides, the government has also proposed to set up 4 Medical devices Parks in the States of Himachal
Pradesh, Madhya Pradesh, Tamil Nadu and Uttar Pradesh.
✓ With this new policy, the government aims to achieve 10-12 percent share in the growing global market
over the next 25 years. Currently India’s share in the global medical devices market is just around 1.5
percent.
✓ The National Medical Devices Policy, 2023 is expected to help the market size of medical devices sector
to grow from $11 billion (Rs 90,000 crore) in 2020, to $50 billion (Rs 4.5 trillion) by 2030.
✓ Among other things, the policy aims at establishing “centres of excellence” in academic and research
institutions, innovation hubs, “plug and play” infrastructure and support to start-ups.
✓ The policy also envisages the creation of a dedicated Export Promotion Council for the sector.
✓ Union Minister for Health & Family Welfare Mansukh Mandaviya

Q. Who has been awarded Ramon Magsaysay Award 1959?


A) Shinzo Abe B) Dalai Lama
C) Angela Merkel D) Volodymyr Zelenskyy
Answer : B
✓ Dalai Lama personally received the 1959 Ramon Magsaysay Award from members of the Ramon
Magsaysay Award Foundation at his residence.
✓ President of the Ramon Magsaysay Award Foundation,Susanna B. Affan, along with Foundation
TrusteeEmily A. Abrera, presented the award to the Dalai Lama.
✓ The award was given in August 1959 by the Ramon Magsaysay Award Foundation in the Philippines.
✓ Ramon Magsaysay Award
✓ Established : 1957
✓ First award : 1958
✓ Vinoba Bhave was the 1st Magsaysay Award winner in 1958 (first Indian).
✓ This award was created to commemorate Ramon Magsaysay, the late president of Philippines.
✓ The Ramon Magsaysay Award is Asia’s premier prize and highest honour.
✓ It is considered as Asia’s equivalent of the Nobel Prize
✓ This award is given in 6 categories
✓ These categories are:
1. Government services (GS)
2. Public services (PS)
3. Community leadership(CL)

Follow us: Official Site, Telegram, Facebook, Instagram, Instamojo 796


4. Journalism, literature & creative communication arts (JLCCA)
5. Peace and International Understanding (PIU)
6. Emergent leadership (EL)

Q. The Ramon Magsaysay Award is an annual award given by which country?


A) United States B) New Zealand
C) Germany D) Philippines
Answer : D

Q. Which Ministry has launched 'City Beauty Competition' portal?


A) Ministry of Housing and Urban Affairs
B) Ministry of Health and Family Welfare
C) Ministry of Agriculture & Farmers' Welfare
D) Ministry of Environment Forest and Climate Change
Answer : A
✓ The Ministry of Housing and Urban Affairs (MoHUA) has launched a portal called ‘City Beauty
Competition’.
✓ The portal can be accessed at https://citybeautycompetition.in
✓ All the Urban Local Bodies (ULBs) across the country can participate in this competition through an
online process.
✓ The objective of the competition is to encourage cities and wards across the country to create beautiful,
innovative and inclusive public spaces and recognize the transformational efforts made by them.
✓ The competition at city level will judge wards and public spaces against five broad pillars viz., (i)
accessibility (ii) amenities (iii) activities (iv) aesthetics and (v) ecology.
✓ The top most beautiful wards would be felicitated at the city and State levels.
✓ The top most beautiful public spaces in the cities under four categories viz. Waterfronts, Green spaces,
tourist/ heritage spaces and market/commercial places would first be felicitated at the State level and the
shortlisted entries would thereafter be considered for National level awards.

Q. Which film has won the best film award at the 68th Filmfare Awards for the year 2023?
A) Gangubai Kathiawadi B) Drishyam 2
C) The Kashmir Files D) Brahmastra
Answer : A
✓ The 68th Filmfare Awards, (also known as Hyundai Filmfare Awards 2023 for promotional purpose)
was presented on 28 April 2023, to honour artistic and technical excellence in Hindi cinema for the year
2022.
✓ Gangubai Kathiawadi starring Alia Bhatt, won most number of awards, at 10
✓ Filmfare Awards is one of the oldest award dedicated to the Hindi film industry. The awards were first
introduced in 1954.
✓ The list of winners of the Filmfare Awards 2023 are given below.
1. Best Film: Gangubai Kathiawadi
2. 2..Best Film (Critics’) : Badhaai Do
3. Best Actor in a Leading Role (Male): Rajkummar Rao for Badhaai Do
4. Best Actor in a Leading Role (Female): Alia Bhatt for Gangubai Kathiawadi
5. Best Director: Sanjay Leela Bhansali for Gangubai Kathiawadi.
6. Best Actor (Critics’): Sanjay Mishra for Vadh

Follow us: Official Site, Telegram, Facebook, Instagram, Instamojo 797


7. Best Actress (Critics’): Bhumi Pednekar for Badhaai Do and Tabu for Bhool Bhulaiyaa 2.
8. Lifetime Achievement Award: Prem Chopra
9. Best Playback Singer (Male): Arijit Singh for Kesariya from Brahmastra: Part One – Shiva
10. Best Playback Singer (Female): Kavita Seth for Rangisari from Jug Jugg Jeeyo

Q. Who won the Best Actor Award at the 68th Filmfare Awards for the year 2023?
A) Ayushman Khurana B) Ranbir Kapoor
C) Rajkumar Rao D) Ranveer Singh
Answer : C
✓ Best Actor in a Leading Role (Male): Rajkummar Rao for Badhaai Do
Q. Who won the best director award at the 68th Filmfare Awards for the year 2023?
A) Kabir Khan B) Vishnuvardhan
C) Laxman Utekar D) Sanjay Leela Bhansali
Answer : D
✓ Best Director: Sanjay Leela Bhansali for Gangubai Kathiawadi

Q. Who won the best actress award in the Filmfare Awards 2023?
A) Alia Bhatt B) Kriti Sanon
C) Kiara Advani D) Ananya Panday
Answer : A
✓ Best Actor in a Leading Role (Female): Alia Bhatt for Gangubai Kathiawadi
Q. Who has launched the first of its kind ‘Millets Experience Centre (MEC)’ at Dilli Haat in New Delhi?
A) Narendra Modi B) Nirmala Sitharaman
C) Narendra Singh Tomar D) Draupadi Murmu
Answer : C
✓ Agriculture Minister Narendra Singh Tomar launched the first of its kind ‘Millets Experience Centre
(MEC)’ at Dilli Haat in New Delhi.
✓ India has an important role to play in the celebration of the International Year of Millets 2023.
✓ India is poised to become a 'Global Hub' for Millets and setting up of MEC is a major step in that
direction.
✓ The United Nations General Assembly has declared the year 2023 as the International Year of Millets
(IYM 2023).
✓ About Millets Experience Centre
✓ The National Agricultural Cooperative Marketing Federation of India Limited (NAFED) in association
with the Ministry of Agriculture has set up the Millets Experience Centre.
✓ It aimsto increase awareness about millet and encourage its adoption among the general public.
✓ Setting up a consumer-oriented 'Millets Experience Centre' will not only promote the dietary benefits of
millets but also popularize Shree Anna (Millets) as a nutritional powerhouse.
✓ International Year of Millet (IYOM)-2023
✓ The United Nations General Assembly (UNGA) adopted India's resolution and declared 2023 as the
International Year of millets in March 2021.
✓ India's proposal received the support of 72 countries.

Follow us: Official Site, Telegram, Facebook, Instagram, Instamojo 798


✓ In 2021, NITI Aayog signed a letter of intent with the United Nations World Food Program (WFP).
✓ Millets and India
✓ According to the Food and Agriculture Organisation (FAO), India is the largest producer of millet in the
world.
✓ The total production of millets in India in 2020-21 was 17.96 million tonnes which was nearly 41% of the
world production.
✓ Rajasthan was the largest millet producing state in India.
✓ Rajasthan, Karnataka, Maharashtra, Uttar Pradesh, Haryana, Gujarat, Madhya Pradesh, Tamil Nadu,
Andhra Pradesh, and Uttarakhand being the top 10 states in terms of millet production.
✓ In India, millet is primarily a kharif crop, requiring less water and agricultural inputs.
✓ India declared 2018 as the National Year for Millets to promote millets for nutrition security.

MILLET IN NEWS 2023


▪ India’s Agriculture Minister Narendra Singh Tomar recently launched the Millets Experience Centre
(MEC) at Dilli Haat in New Delhi.
▪ Prime Minister Narendra Modi inaugurated the Global Millets (Shree Anna) Conference in New Delhi
on 18th March, 2023.
▪ United Nations agency Food and Agriculture Organization (FAO) hosted an opening ceremony of the
International Year of Millets – 2023 (IYM2023) in Rome, Italy.
▪ The 4th International Trade Fair of Millets and Organics Products began on 20 January 2023 at
Tripuravasini, Bengaluru, Karnataka.
▪ The year 2023 is being observed all over the world as the International Year of the millets.
▪ The Government of India aims to make the International Year of the millets a ‘People’s Movement’ and
position India as the ‘Global Hub for Millets’.
▪ India is the largest producer of millets in the world.
▪ According to the Food and Agriculture Organisation, global millet production in 2020 was 30.464 million
metric tonnes (MMT), of which India accounted for 12.49 MMT or 41 per cent of the total millet
production.
▪ India is the largest producer of millets in the world followed by Niger and China and the second largest
exporter of these grains.
▪ The Government of India has set a target of production of 205 lakh tonnes of Nutri Cereals in the country
by 2022-23.
▪ To promote the export of millets, a one-day 'Millets-Smart Nutritive Food' conclave was organised by
the government on 5th December in New Delhi.
▪ Assam government launches millet mission to double the income of state farmers.
▪ Odisha Chief Minister Naveen Patnaik has announced to observe the first Thursday of the Margasira
month of the Hindu calendar on November 10 as 'Mandia Day' or 'Millet Day' in the state.
Q. Which country recently launched its first earth observation satellite, ‘Taifa-1 satellite,’ into orbit?
A) Egypt B) Ethiopia
C) Kenya D) Rwanda
Answer : C
✓ Kenya successfully launches its first satellite Taifa-1 onboard SpaceX Falcon-9 rocket
✓ On April 15, 2023, Kenya successfully launched its first operational earth observation satellite, ‘Taifa-1
satellite,’ into orbit.
✓ The launch took place on a SpaceX rocket Falcon-9 from the Vandenberg Base in California, United
States.
✓ Kenya is an East African country and its capital is Nairobi
✓ About Kenya
✓ Kenya is a country in East Africa.

Follow us: Official Site, Telegram, Facebook, Instagram, Instamojo 799


✓ President: William Ruto
✓ Capital: Nairobi
✓ Currency :Kenyan shilling

Q. Which Indian city is set to host the 2023 Asian Champions Trophy hockey tournament ?
A) Chennai B) Bhubaneshwar
C) Guwahati D) Kolkata
Answer : A
✓ Chennai will host the Asian Champions Trophy hockey tournament from August 3-12, 2023, as informed
by Bhola Nath Singh, Secretary-General of Hockey India.
✓ India will be hosting the event for the first time
✓ The tournament will feature the top six teams from India, Pakistan, South Korea, Malaysia, Japan and
China.
✓ This is the seventh edition of the top-level international hockey event.
✓ The game will serve as a precursor to the 2023 Asian Games in Hangzhou, China in September 2023.
Q. India has been placed under which category for international standards for aviation safety oversight as
per the US-based Federal Aviation Administration (FAA) in its International Aviation Safety Assessment
(IASA) program?
A) Category I B) Category II
C) Category III D) Category IV
Answer : A
✓ India has been retained in category-1 ranking for aviation safety oversight, by the Federal Aviation
Administration (FAA), the United States’ aviation watchdog.
✓ As per the ranking, the Indian aviation regulator, Directorate General of Civil Aviation’s (DGCA) met
all the international standards for aviation safety oversight under FAA’s International Aviation Safety
Assessment (IASA) program, to be placed under Category-I status.
✓ The IASA program determines whether a country, that operate, or seek to operate, into the U.S., or
codeshare with a U.S. air carrier, complies with safety standards established by the International Civil
Aviation Organization (ICAO).
✓ The FAA IASA was last assessed in July 2018.
✓ Category 1 means – Does Comply with ICAO Standards
✓ Category 2 means – Does Not Comply with ICAO Standards
Q. The 2023 Asian Games is scheduled to take place in which country?
A) India B) Greece
C) South Korea D) China
Answer : D
✓ 2023 Asian Games in Hangzhou, China in September 2023.
✓ The first edition of the Asian Games was held in New Delhi in March 1951.
✓ India has hosted the Asian Games twice, that is in 1951 and in 1982.
✓ LIST OF HOST COUNTRY ASIAN GAMES.
✓ 1st Asian games 1951 : New Delhi
✓ 9th edition 1982 : New Delhi
✓ 18th edition 2018 : Palembang, Indonesia, Jakarta
✓ 19th edition 2023 : Hangzhou, China

Follow us: Official Site, Telegram, Facebook, Instagram, Instamojo 800


✓ 20th edition 2026 : Nagoya, Japan
✓ 21st edition 2030 : Doha, Qatar
✓ 22nd edition 2034 : Riyadh, Saudi Arabia .
Q. Which country has become the world's foremost narco-state, as its economic survival now depends
heavily on the production and export of Captagon?
A) Iraq B) Iran
C) Israel D) Syria
Answer : D
✓ Syria has witnessed a troubling transformation into the world's foremost narco-state, as its economic
survival now depends heavily on the production and export of Captagon, also known as the "poor man's
coke".
✓ This highly addictive amphetamine has become the primary lifeline of the country's economy, generating
more than 90% of its foreign currency.

Q. Who has been awarded the prestigious Pravasi Bharatiya Samman?


A) Raj Subramaniam B) Pushpak P Bhattacharya
C) Umesh Bellur D) Sujoy Bhore
Answer : A
✓ The Indian-American CEO of global transportation giant FedEx, Raj Subramaniam has been presented
with the prestigious Pravasi Bharatiya Samman.
✓ It is the highest civilian award given by India to persons of Indian origin and Indian diaspora.
✓ Subramanian was presented with the award by India’s Ambassador to the US Taranjit Singh Sandhu at
a ceremony at the India House.
✓ Subramanian is the president and CEO of FedEx Corporation, one of the world’s largest transportation
companies.
Q. Which day is observed as the International Day for Monuments and Sites (World Heritage Day)
annually?
A) 15 April B) 16 April
C) 18 April D) 17 April
Answer : C
✓ 18 April is observed as World Heritage Day, also known as the International Day for Monuments and
Sites, to raise awareness of the importance of protecting cultural heritage around the world.
✓ The theme of World Heritage Day 2023 is "Heritage of Change", which addresses issues related to
traditional knowledge and knowledge systems in the context of climate action.

Q. Which state in India has become the first to adopt a Water Budget for proper management of the
resource?
A) Tamil Nadu B) Kerala
C) Odisha D) Karnataka
Answer : B
✓ Kerala becomes first Indian state to adopt a Water Budget

Follow us: Official Site, Telegram, Facebook, Instagram, Instamojo 801


✓ The state government of Kerala, under Chief Minister Pinarayi Vijayan has adopted a Water Budget, a
first-of-its type initiative in India.
✓ The Public Water Budget initiative would help in properly utilising the resource and preventing wastage
through proper Management.
✓ The first phase of the Water Budget will cover 94 grama panchayats in 15 block panchayats of the state.
Q. Which state becomes the first to prepare a DNA database for the identification of unidentified dead
bodies?
A) Uttar Pradesh B) Himachal Pradesh
C) Madhya Pradesh D) Maharashtra
Answer : B
✓ Himachal Pradesh has become the "first state" to create a DNA database of unidentified bodies.
Q. Who has launched a YUVA PORTAL for connecting and identifying potential young Start-Ups in the
country?
A) Nitin Gadkari B) Raj Kumar Singh
C) Jitendra Singh D) Mansukh L. Mandaviya
Answer : C
✓ Union Minister Dr. Jitendra Singh launches ‘YUVA PORTAL.
✓ Union Minister for Science and Technology Dr.Jitendra Singh launched the Youth Portal in New Delhi
.
✓ It aims to connect and identify potential youth start-ups.
✓ Science and Technology Minister Dr Jitendra Singh also launched the One Week - One Laboratory
programme.
✓ The event emphasises the need for broad-based participation from stakeholders in the industry for startups
to remain sustainable.
✓ Each of the 37 CSIR (Council of Scientific and Industrial Research) laboratories is dedicated to a different
specialised area of work.
✓ The One Week - One Lab program will provide an opportunity to CSIR laboratories to showcase their
work.

Q. The mobile application GS NIRNAY has been launched by which ministry?


A) Ministry of Rural Development & Panchayati Raj
B) Ministry of Commerce & Industry
C) Ministry of Road Transport and Highways
D) Ministry of Skill Development and Entrepreneurship
Answer : A
✓ The Ministry of Panchayati Raj is celebrating the National Panchayat Awards Week from 17th April to
21st April, 2023 to commemorate the Azadi Ka Amrit Mahotsav (AKAM) 2.0.
✓ It must be mentioned that the National Panchayati Raj Day is commemorated on 24th April every year.
✓ The theme for National Panchayat Awards Week in 2023 is “Panchayaton ke Sankalpon ki Siddhi ka
Utsav” (पंचायतों के संकल्पों की ससद्धि का उत्सव).
✓ A series of five National Conferences covering nine themes under Localization of Sustainable
Development Goals (LSDGs) through Panchayati Raj Institutions and Way Forward for 2047 have been
organized during the National Panchayat Awards Week Celebrations.

Follow us: Official Site, Telegram, Facebook, Instagram, Instamojo 802


✓ The Union Minister of Rural Development & Panchayati Raj, Shri Giriraj Singh launched a new mobile
application named GS NIRNAY, as a part of the occasion.
✓ GS NIRNAY stands for Gram Sabha National Initiative for Rural India to Navigate, InnovAte and
Resolve PanchaYat.
✓ The app Gram Sabha NIRNAY has been developed to further strengthen Gram Sabha for ensuring
participatory democracy and making empowered people and accountable Panchayats.
✓ The app will enable recording and uploading Gram Sabha proceedings open for public view.
✓ The National Panchayat Awards 2023 were awarded under four categories, namely
✓ Deen Dayal Upadhyay Panchayat Satat Vikas Puraskar (DDUPSVP)
✓ Nanaji Deshmukh Sarvottam Panchayat Satat Vikas Puraskar (NDSPSVP)
✓ Gram Urja Swaraj Vishesh Panchayat Puraskar
✓ Carbon Neutral Vishesh Panchayat Puraskar
Q. Which country has recently joined India’s initiative of International Big Cat Alliance (IBCA) as its
founding member?
A) Germany B) Russia
C) France D) Nepal
Answer : D
✓ Nepal has been designated as the founding member of the International Big Cat Alliance (IBCA), an
initiative by the Government of India.
✓ The IBCA was launched by Indian Prime Minister Shri Narendra Modi on 9th April 2023, at Mysuru,
Karnataka, to commemorate 50 years of Project Tiger.
✓ The objective of this alliance is conservation of seven big cats namely Tiger, Lion, Leopard, Snow
Leopard, Cheetah, Jaguar and Puma.
✓ The Indian Initiative of IBCA was launched in Nepal recently.
Q. The first ever census of which bird was conducted by Odisha forest officials at Bhitarkanika Wildlife
Sanctuary?
A) Blue-winged pitta B) Superb pitta
C) Hooded pitta D) Mangrove Pitta Bird
Answer : D
✓ Mangrove Pitta bird found in Odisha.
✓ Mangrove Pitta birds were counted in two coastal districts of Odisha, Kendrapara and Jagatsinghpur, in
which 179 birds were found.
✓ Mangrove pitta (Pitta Megharencha) birds are near threatened and are found in parts of eastern India.
✓ 179 different mangrove pitta birds were counted in the mangroves near the mouth of the Mahipura River
inside the Bhitarkanika National Park.

Q. The World Liver Day is marked every year on which day?


A) 18 April B) 19 April
C) 17 April D) 20 April
Answer : B
✓ World Liver Day is observed every year on 19 April to raise awareness about liver health and related
diseases.

Follow us: Official Site, Telegram, Facebook, Instagram, Instamojo 803


✓ World Liver Day aims to educate people about the importance of early detection, prevention and
treatment of liver diseases.
✓ The theme of World Liver Day 2023 is "Be alert, do regular liver check-ups, fatty liver can affect anyone,"
✓ Liver is the second largest and most important organ in the human body, responsible for functions such
as metabolism, digestion, immunity, toxin filtration, and nutrient storage.

Q. The 5th edition of HUN- a Thadou Cultural Festival 2023 begins in which state?
A) Nagaland B) Tripura
C) Manipur D) Meghalaya
Answer : C
✓ The 5th edition of HUN- a Thadou Cultural Festival 2023 kicked off at Thomas Ground in Kangpokpi
district of Manipur.
✓ HUN festival is an annual cultural festival and a civilizing act of the Thadous community of Manipur
and it is celebrated at the arrival of the new year according to the community.
✓ The festival celebrates the culture and traditions of the Hun Thadou tribe, one of the major ethnic groups
in the state.
✓ The festival features traditional dance performances, music, food, and handicrafts.
Q. Who has been awarded the prestigious Santokbaa Humanitarian Award for his contributions to society?
A) Sonam Wangchuk B) Parvatibai Athavale
C) Poonam Ahluwalia D) Arun Cherukavil
Answer : A
✓ Innovator Sonam Wangchuk conferred with prestigious Santokba Humanitarian Award.
✓ The prestigious Santokba Humanitarian Award was presented to Sonam Wangchuk, a notable engineer,
innovator, educationist, sustainable development reformist, and founder-director of the Students'
Educational and Cultural Movement of Ladakh (SECMOL).
✓ The award carries a cash prize of Rs 1 crore and has been instituted in the memory of Late Santokba
Dholakia, mother of SRK and SRKKF founder president Govind Dholakia.
✓ The award was presented to Wangchuk by the First Lady of Ladakh Mrs. Neelam Mishra and Mr. Rahul
Dholakia, Entrepreneur- SRK.
✓ The award ceremony was held at Hotel Zen in Ladakh on April 10, 2023, the death anniversary of
Santokba.
✓ Wangchuk is the inventor of the Ice Stupa technique which creates artificial glaciers used to store winter
water in the form of cone-shaped ice piles and solve the water crisis in Ladakh.
✓ The SECMOL campus, designed by Wangchuk, runs on solar power and does not use fossil fuels for
cooking, lighting or heating.
✓ Past award recipients include Ratan Tata, Kailash Satyarthi, A.S. Kiran Kumar, and Sudha Murthy.
Q. Which country to replace Indonesia as the Under-20 FIFA World Cup host?
A) Croatia B) Argentina
C) France D) China
Answer : B
✓ Argentina to replace Indonesia as under-20 FIFA World Cup host
Q. The Centre has approved the project of development of an Oil Jetty at Deendayal Port, Kandla in Gujarat
for how many crore rupees?

Follow us: Official Site, Telegram, Facebook, Instagram, Instamojo 804


A) Rs 110 crore B) Rs 123 crore
C) Rs 140 crore D) Rs 150 crore
Answer : B
✓ The Centre has approved the project of development of an Oil Jetty at Deendayal Port, Kandla in Gujarat
for over 123 crore rupees.
✓ The proposed project is designed to increase the cargo handling capacity of the port, which will attribute
to a reduction in the turn-around time of liquid vessels.
✓ This project will increase Deendayal Port’s income through the collection of Royalty from the
Concessionaire.
Q. According to Stanford University’s annual Al Index report 2023, India ranked fifth in terms of
investments received by startups offering artificial intelligence (Al)-based products. Which country has
topped this index?
A) United States B) United Kingdom
C) China D) Israel
Answer : A
✓ According to Stanford University’s annual Al Index report 2023, India ranked fifth in terms of
investments received by startups offering artificial intelligence (Al)-based products.
✓ Total investments in AI startups in India stood at $3.24 billion in 2022, placing it ahead of South Korea,
Germany, Canada and Australia, among others.
✓ Those ahead of India in the list are the US, China, the UK and Israel.
✓ The report also showed that AI startups in India received total funding of $7.73 billion during 2013-2022,
making it the sixth leading country with the most AI investments during the period. Nearly 40% of this
investment was made last year.
✓ Among Indian AI startups, Chennai-based conversational AI startup Uniphore raised $400 million in a
Series E funding round last year at a valuation of $2.5 billion.
Q. Which state has topped the State Energy Efficiency Index (SEEI) 2021-22 report in Front Runner
category?
A) Madhya Pradesh B) Haryana
C) Andhra Pradesh D) Karnataka
Answer : C
✓ Union Minister for Power and Renewable Energy R. K. Singh released the State Energy Efficiency Index
(SEEI) 2021-22 report.
✓ The SEEI was released during the RPM (Review, Planning and Monitoring) meeting of states and state
utility companies in New Delhi.
✓ Andhra Pradesh, Karnataka, Kerala, Rajasthan and Telangana are among the top ranked states in the
State Energy Efficiency Index-SEEI.
✓ Four states Assam, Haryana, Maharashtra and Punjab are in the achiever category with scores between
50 and 60.
✓ Besides, Karnataka, Andhra Pradesh, Assam and Chandigarh are the top performing states in their
respective state groups.
Q. External Affairs Minister S Jaishankar has launched the ‘Tulsi Ghat Restoration Project’ of Varanasi in
which city?
A) Ethiopia B) Uganda

Follow us: Official Site, Telegram, Facebook, Instagram, Instamojo 805


C) Nigeria D) Egypt
Answer : B
✓ External Affairs Minister S Jaishankar launched the ‘Tulsi Ghat Restoration Project’ of Varanasi, during
his visit to Uganda’s Kampala.
✓ UGANDA
✓ Capital : Kampala
✓ Currency : Shilling
Q. Which company has signed a perpetual license agreement with Switzerland-based Novartis Pharma AG
to manufacture and market Galvus and Galvus combination brands which are used in the treatment of
type 2 diabetes?
A) Cipla Pharmaceutical company B) Bharat Biotech
C) Dr. Reddy’s Laboratories D) Serum Institute of India
Answer : A
✓ Indian multinational pharmaceutical company Cipla entered into a perpetual licence agreement with
Switzerland-based pharmaceutical company Novartis Pharma AG.
✓ The agreement allows Cipla to manufacture and market diabetes drug Galvus and its combination brands
from January 1, 2026.
✓ Galvus is an oral anti-diabetic medicine that contains Vildagliptin as an active ingredient in it. It is used
for the treatment of type 2 diabetes.
✓ The perpetual licence agreement between Cipla and Novartis Pharma AG will enable Cipla to expand
its product offerings in the diabetes segment.
Q. Suhelwa Wildlife Sanctuary is a new area where photographic evidence of tigers has been recorded for
the first time, according to the latest report on tiger census in the country. Suhelwa Wildlife Sanctuary is
located in which state?
A) Rajasthan B) Uttar Pradesh
C) Karnataka D) Madhya Pradesh
Answer : B
✓ UP's Suhelwa sanctuary records first photographic proof of tigers.
✓ The recently released report on the census of tigers in the country states that Suhelwa Wildlife Sanctuary
is a new area where photographic evidence of tigers has been recorded for the first time.
✓ About Suhelwa Wildlife Sanctuary
✓ The sanctuary, located in Shravasti, Balrampur and Gonda districts of Uttar Pradesh, was declared a
wildlife sanctuary in 1988.
✓ It is spread over an area of 452 sq km.
✓ It is situated on the Indo-Nepal international border and is rich in natural resources.
✓ It is named after King Soheldev.
Q. Which state government has launched a new project, ‘Sanjeevani” to ensure that livestock is provided
quality treatment in a time-bound manner?
A) Uttar Pradesh B) Himachal Pradesh
C) Madhya Pradesh D) Tamil Nadu
Answer : B
✓ Sanjeevani Project launched by the Government of Himachal Pradesh.

Follow us: Official Site, Telegram, Facebook, Instagram, Instamojo 806


✓ To support small dairy farmers and cattle rearers, the state government launched a project called
Sanjeevani on April 9 to enhance their livelihood.
✓ The Sanjeevani project uses telemedicine and technology to provide convenient and high-quality
livestock care services to farmers' doorsteps, potentially preventing outbreaks and reducing turnaround
time for services.

Q. The International Day of Human Space Flight is marked annually by the United Nations on which day?
A) April 10 B) April 12
C) April 09 D) April 11
Answer : B
✓ The International Day of Human Space Flight is celebrated annually on April 12 to mark the anniversary
of the first human space flight by Yuri Gagarin on April 12, 1961.
✓ The day was proclaimed at the 65th session of the United Nations General Assembly on April 7, 2011, a
few days before the 50th anniversary of the flight.
✓ In Soviet Union, the day is observed as International Day of Aviation and Cosmonautics.
✓ Rakesh Sharma became the first Indian to orbit in space in the year 1984.
✓ Kalpana Chawla, the first Indian origin woman to go to space is related to Karnal district of Haryana.
✓ Yuri Gagarin was the first person to go into space.
✓ Neil Armstrong was the first person to land on the Moon.
Q. In which city was the first AIIMS of North-East India inaugurated?
A) Guwahati B) Itanagar
C) Imphal D) Kohima
Answer : A
✓ Prime Minister Narendra Modi dedicated the Guwahati AIIMS to the nation at a function held in
Changsari, Assam.
✓ PM Modi also inaugurated three medical colleges at Nalbari, Nagaon and Kokrajhar. This is the first
AIIMS in North-East India.
Q. In which country did Foreign Minister Dr. S. Jaishankar inaugurate the bridge built by India?
A) Ghana B) Senegal
C) Mozambique D) Sri Lanka
Answer : C
✓ External Affairs Minister Dr. S. Jaishankar inaugurated the Buzi Bridge built by India in Mozambique.
✓ The bridge has been built as part of the 132 km Tika-Buji-Nova-Sofala road project.
✓ External Affairs Minister Dr S Jaishankar began his visit with a meeting with the Speaker of the Assembly
Esperanca Bias in Maputo, Mozambique.
✓ Mozambique is a southern African nation with a long coastline along the Indian Ocean. Its capital is
Maputo and its currency is Mozambican metical
Q. Which player won India's first gold medal in the Asian Wrestling Championship 2023?
A) Deepak Dahiya B) Ravi Kumar
C) Bajrang Punia D) Aman Sehrawat
Answer : D

Follow us: Official Site, Telegram, Facebook, Instagram, Instamojo 807


✓ Aman Sehrawat has won India's first gold medal at the Asian Wrestling Championships 2023 in Astana,
Kazakhstan.
✓ Sehrawat won this medal in men's 57 kg freestyle category. He won the gold medal by defeating Almaz
Samanbekov of Kyrgyzstan 9-4.
✓ Last year, Sehrawat became the first Indian wrestler to win gold at the Under-23 World Championships
in Spain.
Q. According to the ADR report, who is the Chief Minister with the most assets in the country?
A) Mamata Banerjee B) Yogi Adityanath
C) Jagan Mohan Reddy D) Pema Khandu
Answer : C
✓ The Association for Democratic Reforms (ADR) recently released a report according to which 29 chief
ministers are crorepatis out of 30 chief ministers of 28 states and 2 union territories of India.
✓ Andhra Pradesh Chief Minister Jagan Mohan Reddy topped the list with Rs 510 crore.
✓ Arunachal Pradesh Chief Minister Pema Khandu is in second place on this list with Rs 163 crore.
✓ Also, West Bengal Chief Minister Mamata Banerjee has the least assets around Rs 15 lakh.

Q. Which state government has passed a bill to ban online gambling?


A) Uttar Pradesh B) Bihar
C) Tamil Nadu D) Goa
Answer : C
✓ Tamil Nadu Governor R. N. Ravi has approved the bill proposing to ban online gambling.
✓ The bill was passed by the Tamil Nadu Assembly for the second time in March.
✓ The bill was first passed in the assembly on October 19, 2022.
Q. Which country has recently launched an initiative to license crypto firms?
A) Morocco B) Cuba
C) El Salvador D) Argentina
Answer : C
✓ Central American country El Salvador has started an initiative to license crypto firms.
✓ Cryptocurrency exchange Bitfinex has become the first cryptocurrency company to receive an official
license in El Salvador.
✓ This license will allow Bitfinex to provide crypto-related services in the country as per the legal provisions
of the country.
✓ El Salvador became the first country in the world to accept Bitcoin as a payment option in 2021
✓ About El Salvador .
✓ El Salvador is the smallest and most densely populated country in Central America.
✓ Capital - San Salvador
✓ President- Nayib Bukele
✓ Currency - bitcoin
Q. PM Modi recently inaugurated the world’s first semi-high speed passenger train on high rise overhead
electric (OHE) territory in which state of India?
A) Rajasthan B) Uttar Pradesh
C) Karnataka D) Gujarat

Follow us: Official Site, Telegram, Facebook, Instagram, Instamojo 808


Answer : A
✓ PM Modi flags off India’s 15th and Rajasthan’s first Vande Bharat Express Train
✓ Prime Minister Narendra Modi flagged off the first Vande Bharat Express train of Rajasthan on April 04,
2023, via videoconferencing.
✓ This Vande Bharat Express train is world’s first semi-high speed passenger train on high rise overhead
electric (OHE) territory.
✓ It will operate between Ajmer and Delhi Cantt, with stops at Jaipur, Alwar and Gurgaon.
✓ This was India’s 15th Vande Bharat Express train
Q. Who among these is the oldest billionaire and the former Chairman of Mahindra and Mahindra Group,
who passed away recently?
A) Keshub Mahindra B) Haigreve Mahindra
C) Anish Mahindra D) Manoj Mahindra
Answer : A
✓ India’s oldest billionaire and the former Chairman of Mahindra and Mahindra Group, Keshub Mahindra
has passed away. He was 99.
✓ Keshub Mahindra served as the chairman of the Mumbai-based conglomerate from 1963 to 2012.
✓ With a net worth of $1.2 billion, he was among 169 Indian billionaires who made the coveted list in 2023.
✓ The Mahindra company was co-founded by his father, J C Mahindra, in 1945.
✓ Keshub joined the company in 1947 and assumed the role of Chairman in 1963.
Q. What is the rank of India in terms of rate of crime, as per the report by World of Statistics in 2023?
A) 54 B) 77
C) 65 D) 81
Answer : B
✓ India ranks 77th among 142 countries with highest crime rate; Venezuela tops: World of Statistics report.
✓ Based on the ranking of the world’s “most criminal countries” released by the World of Statistics,
Venezuela is the top-most country with highest criminal rate.
✓ It is followed by Papua New Guinea (2), Afghanistan (3), South Africa (4), Honduras (5), Trinidad (6),
Guyana (7), Syria (8), Somalia (9) and Jamaica (10), respectively.
✓ India is placed at 77 spots. The US and UK are at 55th and 65th rank, respectively.
✓ The least criminal countries in the list are Qatar (142), UAE (141), Taiwan (China) at 140
✓ Separately, according to the Population Review (WPR), the top most countries with highest crime rate
in 2023 are Venezuela, Papua New Guinea, South Africa and Afghanistan respectively.
Q. The World Population Review (WPR) report has ranked which country as the world’s top high crime
rate country in 2023?
A) South Africa B) Papua New Guinea
C) Venezuela D) Afghanistan
Answer : C
✓ According to the World Population Review (WPR), the top most countries with highest crime rate in
2023 are Venezuela, Papua New Guinea, South Africa and Afghanistan respectively.
Q. Which day of the year is marked as the Ambedkar Jayanti to commemorate the birth anniversary of
Babasaheb Dr. Bhim Rao Ambedkar?

Follow us: Official Site, Telegram, Facebook, Instagram, Instamojo 809


A) April 13 B) April 10
C) April 12 D) April 14
Answer : D
✓ Ambedkar Jayanti ( also known as Bhim Jayanti) is an annual festival observed on 14 April to
commemorate the birth anniversary of Babasaheb Dr. Bhim Rao Ambedkar.
✓ The Day has been observed as an official public holiday throughout India since 2015.
✓ In 2023, we are marking the 132nd birth anniversary of Babasaheb, who was born on 14 April 1891..
✓ Dr. Ambedkar is known as the Father (chief architect) of the Indian Constitution.
✓ He was the first Law and Justice Minister of the country after the independence.
✓ Dr. Bhim has been bestowed with the country’s highest civilian honor, Bharat Ratna in 1990
posthumously.
Q. The year 2023 is being celebrated as which birth anniversary of Babasaheb Dr. Bhim Rao Ambedkar?
A) 152 B) 153
C) 123 D) 132
Answer : D
✓ In 2023, we are marking the 132nd birth anniversary of Babasaheb, who was born on 14 April 1891.
Q. Which country has become the first country in the world to approve a new malaria vaccine from Oxford
University, with children under the age of three in line to benefit?
A) Ethiopia B) Zimbabwe
C) Ghana D) Nigeria
Answer : C
✓ Ghana has become the first country in the world to approve a new malaria vaccine from Oxford
University, with children under the age of three in line to benefit.
✓ The mosquito-borne disease kills more than 600,000 people each year, most of them children in Africa,
and scientists have been trying for years to develop vaccines.
✓ Childhood vaccines in Africa are typically paid for by international organizations such as Gavi and
UNICEF after they have been backed by the World Health Organization (WHO).
✓ This is the first time a major vaccine has been approved first in an African country before rich nations.
✓ About Ghana
✓ Capital : Accra
✓ Currency : Cedi
Q. Name the traditional folk dance which has recently entered the Guinness Book of World Records for the
largest traditional dance performed at a single venue?
A) Jhumar B) Naga dance
C) Bihu D) Kajri
Answer : C
✓ Assam’s Bihu creates Guinness World Record for largest traditional dance at single venue.
✓ The Northeastern state of India, Assam created history on April 13, 2023, when as many as 11,304
dancers and 2,548 drummers performed ‘Bihu’, the traditional folk dance of Assam, to mark the
beginning of the state’s spring festival.

Follow us: Official Site, Telegram, Facebook, Instagram, Instamojo 810


✓ In doing so, Assam marked its name in the Guinness Book of World Records for performing the largest
traditional Bihu dance at a single venue.
✓ The event was held at the Sarusajai Stadium in Guwahati.
✓ Two world records were made, one for the largest traditional dance, and the other record for the largest
musical instrument group.
Q. Fondly known as the Mother of Landsat by NASA, name the aerospace pioneer, who passed away
recently?
A) Lucy Salani B) Julie Peters
C) Sharon Acker D) Virginia Norwood
Answer : D
✓ Virginia Norwood, an American aerospace pioneer who developed technology to scan the surface of the
moon to map and study the earth from space, has passed away. She was 96.
✓ Norwood is best known for developing the Multispectral Scanner System that flew on the first Landsat
satellite.
✓ The scanner have been used for more than 50 years for scanning safe landing sites and map our planet
from space.
✓ For this invention, NASA has called her “the mother of Landsat.”
Q. Indian signed a MoU with the World Food Programme (WFP) for sending How many metric tonnes of
wheat to the people of Afghanistan?
A) 20,000 Metric Tonnes B) 15,000 Metric Tonnes
C) 10,000 Metric Tonnes D) 5,000 Metric Tonnes
Answer : C
✓ India signed a Memorandum of Understanding (MoU) with the World Food Programme (WFP) paving
the way for sending 10,000 metric tonnes of wheat for the people of Afghanistan.
✓ The fifth tranche [to be shipped through the Chahbahar port of Iran], builds upon assistance already
delivered to those who need it most by the WFP in Afghanistan.
✓ World Food Programme (WFP)
✓ Founded : 1961
✓ Headquarters : Rome, Italy
✓ Director General : David Beasley
✓ World Food Programme (WFP) wins Nobel Peace Prize 2020.

Q. National Highway Authority of India (NHAI) has signed an MoU with the Nagarjunasagar-Srisailam
Tiger Reserve to facilitate seamless and efficient entry process for vehicles entering into the forest area.
Nagarjunasagar-Srisailam Tiger Reserve is located in which state?
A) Andhra Pradesh B) Tamil Nadu
C) Kerala D) Karnataka
Answer : A
✓ National Highways Authority of India (NHAI) signed a Memorandum of Understanding (MoU) with
the Nagarjunasagar-Srisailam Tiger Reserve in Andhra Pradesh and Telangana.
✓ The objective of the MoU is to improve the entry process for vehicles entering the forest area by
facilitating a seamless and efficient system.

Follow us: Official Site, Telegram, Facebook, Instagram, Instamojo 811


✓ The initiative proposes a FASTag based payment system at forest entry points to avoid long queues and
delays.
✓ The benefit of collection of Ecosystem Management Coordination Fee through FASTag will be extended
to various entry points of the Tiger Reserve.
✓ About Nagarjunasagar-Srisailam Tiger Reserve
✓ The Nagarjunasagar-Srisailam Tiger Reserve, located in Andhra Pradesh and Telangana, is the largest
tiger reserve in India, covering a total area of 3,728 km (1,439 sq mi).
✓ It was established in 1983.
✓ The tiger reserve is spread over five districts, which are Nandyal, Prakasam, Palnadu, Nalgonda and
Mehboob Nagar.
✓ The tiger reserve is located inside the Nallamala forest area, which is known for its rich biodiversity and
diverse flora and fauna
Q. ISRO launched how many satellites of Network Access Associated Limited (OneWeb) simultaneously?
A) 25 B) 19
C) 31 D) 36
Answer : D
✓ Indian Space Research Organization (ISRO) successfully launched the OneWeb India-2 mission from
the Satish Dhawan Space Center in Sriharikota, Andhra Pradesh.
✓ Under this, 36 satellites were installed in the Lower Earth Orbit through India's largest Launch Vehicle
Mark-III (LVM3) rocket.
✓ These satellites belong to UK-based Network Access Associated Limited (OneWeb).
✓ OneWeb had entered into a contract with ISRO to place 72 satellites in Lower Earth Orbit.
✓ Last year in October 2022, ISRO launched 36 satellites of OneWeb.
Q. Who has been awarded the Wildlife Conservation Award 2023?
A) Kiran Ahuja B) Aliya Mir
C) Neera Tanden D) Garima Verma
Answer : B
✓ Wildlife conservationist Aliya Mir was awarded Wildlife Conservation Award by Jammu and Kashmir
Lieutenant Governor Manoj Sinha.
✓ The award makes her the first woman from the Union Territory to receive it in recognition of her
conservation efforts in the area.
✓ Alia Mir is also the first woman in Kashmir to work for Wildlife SOS, a part of the Wildlife Rescue
Team.

Q. Who has been conferred with a Special literary award?


A) Sheikh Mujibur Rahman B) William Nordhaus
C) Leonardo DiCaprio D) Greta Thunberg
Answer : A
✓ Bangabandhu Sheikh Mujibur Rahman honoured with literary award.
✓ The Foundation of SAARC Writers and Literature (FOSWAL) honoured Bangabandhu Sheikh Mujibur
Rahman with a special literary award for his trilogy - Unfinished Memoirs, The Prison Diaries and The
New China 1952.
✓ The award was presented at the ceremony to Bangladeshi writers and researchers Ramendu Majumdar
and Mofidul Haque by eminent Punjabi novelist and founder president of FOSWAL Ajit Kaur.

Follow us: Official Site, Telegram, Facebook, Instagram, Instamojo 812


✓ The award was given to Sheikh Mujibur Rahman for his outstanding literary excellence, as he is a
towering figure of national emancipation of the oppressed people of the world.
✓ A citation by the Foundation of SAARC Writers and Literature compares him with Mahatma Gandhi
and Martin Luther King, saying that no power on earth can erase him from history.
Q. The Adani port has announced the acquisition of Karaikal Port and now operates 14 ports in India. In
which state is this port located?
A) Chandigarh B) Daman and Diu
C) Goa D) Puducherry
Answer : D
✓ The Adani Ports and Special Economic Zone (APSEZ) has acquired the Karaikal Port.
✓ Karaikal Port is an all-weather deep-water port on the eastern coast of India in Karaikal District of
Puducherry. It was commissioned in 2009.
✓ The port is around 300 km south of Chennai Port and around 360 km north of Tuticorin Port.
✓ With this acquisition, APSEZ now operates 14 ports in the country.
✓ APSEZ is the subsidiary and port unit of Gautam Adani’s conglomerate Adani Group.
Q. What is the rank of India in “The Passport Index” 2023, released by financial advisory firm Arton
Capital?
A) 144 B) 138
C) 126 D) 112
Answer : A
✓ India’s Rank drop by 6 positions at 144 in The Passport Index 2023 by Arton Capital.
✓ India has been ranked at 144th spot in “The Passport Index” 2023.
✓ The mobility score of India in the 2023 index is 70.
✓ The Passport Index 2023 ranking is topped by the United Arab Emirates (UAE), having mobility score
of 181.
Q. Which country has topped the Passport Index 2023, released by financial advisory firm Arton Capital?
A) United Arab Emirates (UAE) B) Luxembourg
C) Netherlands D) Austria
Answer : A
✓ The Passport Index 2023 ranking is topped by the United Arab Emirates (UAE), having mobility score
of 181.
✓ UAE is followed by Sweden, Germany, Finland, Luxembourg, Spain, France, Italy, the Netherlands and
Austria, have joint mobility score of 174.

Q. Which US state has become the first American State to pass a resolution condemning Hinduphobia?
A) Florida B) Washington
C) Georgia D) Alaska
Answer : C
✓ In the United States, Georgia Assembly has passed a resolution condemning Hinduphobia and anti-
Hindu bigotry.
✓ With this Georgia has become the first American State to take such a legislative measure.

Follow us: Official Site, Telegram, Facebook, Instagram, Instamojo 813


✓ The resolution was introduced by Representatives Lauren McDonald and Todd Jones from Forsyth
County in the suburbs of Atlanta, which is the home to one of the largest Hindu and Indian-American
diaspora communities in Georgia.
Q. The 95-year-old Indian player Bhagwani Devi Dagar has recently won 3 gold medals for India in which
game?
A) Marathon B) Chess
C) Shooting D) Athletics
Answer : D
✓ India’s 95-year-old player Bhagwani Devi Dagar has won three gold medals for the country at the 9th
World Masters Athletics Indoor Championship at Torun in Poland. The championship was held from
25 to 31 March 2023.
✓ The veteran athlete from Haryana won the three gold medals in 60m running, shot put and discus throw
categories in the 95-99 age division.
✓ Bhagwani Devi Dagar is nicknamed as “Sprinter Dadi”. Her athletic career began at the age of 94
Q. Ministry of Defence has inked a Rs 1,700 crore contract with which company for procurement of next-
generation maritime mobile coastal batteries and BrahMos missiles?
A) Bharat Electronics Limited (BEL) B) Tata Advanced Systems
C) Hindustan Aeronautics Limited D) BrahMos Aerospace Private Limited
Answer : D
✓ Ministry of Defence has inked a Rs 1,700 crore contract with BrahMos Aerospace Private Limited
(BAPL) for procurement of next-generation maritime mobile coastal batteries and BrahMos missiles.
✓ These systems will be equipped with supersonic BrahMos missiles and will significantly enhance the
multi-directional maritime strike capability of the Indian Navy.
✓ BAPL is a joint venture between India and Russia making crucial contribution to augment the new
generation surface-to-surface missile with enhanced ranges.
Q. The Ministry of Electronics and Information Technology (MeitY) has constituted a nine-member task
force to make India a 'Product Developer and Manufacturing Nation'. who is chaired this committee?
A) Rajesh Verma B) Kaushal Pandey
C) Bhuvnesh Kumar D) Amaresh Goswami
Answer : C
✓ The Ministry of Electronics and Information Technology (MeitY) has constituted a nine-member task
force to make India a ‘product developer and manufacturing nation’.
✓ It will be chaired by the additional secretary (MeitY), Bhuvnesh Kumar with the joint secretary
(electronics), Amitesh Kumar Sinha as the member convenor.
✓ This is an effort to position itself as an alternative to the likes of China and Vietnam.
Q. How many railway stations will be modernized under Amrit Bharat station scheme?
A) 1,075 B) 1,275
C) 1,975 D) 1,575
Answer : B

Follow us: Official Site, Telegram, Facebook, Instagram, Instamojo 814


✓ The government has identified 1275 railway stations under the Amrit Bharat Station Scheme for the
development of railway stations in India.
✓ Amrit Bharat Station Scheme
✓ Amrit Bharat Station Scheme was introduced by the Ministry of Railways in December 2022 .
✓ The objective of the scheme is to prepare a master plan for railway stations and implement the master
plan in a phased manner to enhance facilities.
✓ Facilities Planned under this Scheme.
✓ Provision for creation of Roof Plaza in future.
✓ Free Wi-Fi, 5G Mobile Tower.
✓ Widening of roads, removal of unwanted structures, properly designed signage, dedicated walkways,
well planned parking areas, access to better lighting etc.
✓ High level platforms at all stations with a length of 600 meters.
✓ Special facilities for the disabled
Q. The Defence Ministry has signed how many contracts at a total cost of nearly 5400 crore rupees with
domestic producers to bolster the defence capabilities of the country?
A) Four B) Five
C) Three D) Six
Answer : C
✓ The Defense Ministry has signed three contracts with domestic manufacturers at a total cost of about Rs
5400 crore to enhance the country's defense capabilities.
✓ First Contract
✓ The first contract has been signed with Bharat Electronics Limited (BEL) which pertains to the
procurement of Project Akashtir, an automated air defense control and reporting system worth Rs 1982
crore for the Indian Army.
✓ Second Contract
✓ The second contract with BEL pertains to the acquisition of Sarang Electronic Support Measure System
for the Indian Navy at a total cost of Rs 412 crore.
✓ Sarang is an advanced electronic support measure system for helicopters of the Indian Navy.
✓ Both will empower the air defense units of the Indian Army to operate effectively in an integrated manner.
✓ Third Contract
✓ The third contract with New Space India Limited pertains to procurement of GSAT 7B, an advanced
communication satellite for the Indian Army at a total cost of Rs 2963 crore.
✓ The satellite will significantly enhance the communications capability of the Indian military by providing
mission-critical beyond-line-of-sight communications for troops and formations as well as weapon and
airborne platforms.
Q. The annual SLINEX is a bilateral maritime exercise of India and Sri Lanka. The 2023 SLINEX is which
edition of the annual event?
A) 11th B) 17th
C) 10th D) 12th
Answer : C
✓ The 10th edition of the annual India – Sri Lanka Bilateral Maritime exercise, SLINEX-2023, will be held
from April 03, 2023 in Colombo.
✓ The exercise SLINEX-2023 will be conducted in two phases – the Harbour Phase and Sea Phase – of
three days each.

Q. How many member of astronauts have been named for NASA’s Artemis II crew mission?

Follow us: Official Site, Telegram, Facebook, Instagram, Instamojo 815


A) 4 B) 3
C) 7 D) 5
Answer : A
✓ American space agency, NASA and the Canadian Space Agency (CSA) announced the names of four
astronauts, who will venture around the Moon for science and exploration under Artemis II mission.
✓ The Artemis II mission is the first crewed voyage around the moon in more than 50 years. The previous
human flight to lunar surface was the Apollo program in 1972.
✓ The Artemis II crew members consists of three from NASA and one from CSA.
✓ They are Reid Wiseman, Victor Glover, and Christina Hammock Koch, from NASA and Jeremy Hansen
from CSA.
✓ Out of these four, Christina Hammock Koch is the first ever female, Victor Glover is the first ever African
American and Jeremy Hansen is the first ever Canadian and first non- American to be sent on a lunar
mission.
✓ The Artemis II will mark the debut crewed flight, but not the first lunar landing.

Q. Which player has won the men’s singles title at the 2023 Miami Open Tennis Tournament?
A) Daniil Medvedev B) Santiago Gonzalez
C) Edouard Roger D) Jannik Sinner
Answer : A
✓ Russian Tennis star Daniil Medvedev beat Jannik Sinner (Italian) 7-5, 6-3, to claim his first title at the
Miami Open.
✓ This was Medvedev’s fourth title of the year, 5th ATP Masters 1000 title and 19th ATP title overall.
✓ Complete Overview of Result
✓ Men’s Single – Daniil Medvedev
✓ Women’s Single – Petra Kvitova (Czech Republic)
Q. Who has conquered Mount Kilimanjaro in Africa, Wearing Lunachari is traditional dress of Himachal
Pradesh?
A) Anjali Sud B) Amrapali Gan
C) Anjali Sharma D) Revathi Advaithi
Answer : C
✓ Anjali Sharma conquer Mount Kilimanjaro in Africa, wearing Luanchari.
✓ Dharamshala's daughter Anjali Sharma has brought glory to Himachal and the country by conquering
Africa's Mount Kilimanjaro peak wearing a Gaddi dress (Luanchadi).
✓ She has become the first girl from India to scale Mount Kilimanjaro in Gaddi costume.
✓ Anjali Sharma is working to promote Gaddi culture on the peaks of the mountains.
✓ Earlier, at the age of 15, Anjali had conquered the peak of 5289 meters in the first attempt.
✓ She has also conquered Hanuman Tibba and Pahar Deo with a height of 6001 meters.
✓ Lunachari is a traditional dress of Himachal Pradesh.
✓ Mount Kilimanjaro
✓ Mount Kilimanjaro is Africa’s tallest mountain and the world’s largest free-standing mountain.
✓ It is located in Tanzania, has an elevation of 5,895 meters (19,340 ft) and is the highest peak on the
African continent.
✓ The mountain is a snow-capped volcano.
✓ It is also called a stratovolcano (a term for a very large volcano made of ash, lava, and rock).
✓ It is made up of three cones: Kibo, Mawenzi and Shira.

Follow us: Official Site, Telegram, Facebook, Instagram, Instamojo 816


✓ It was included as a UNESCO World Heritage Site in 1987.
Q. Dhawan-II is an advanced fully 3D-printed cryogenic rocket engine, which was successfully test fired
recently by which company?
A) Skyroot Aerospace B) ISRO
C) Pixxel D) Dhruva Space
Answer : A
✓ Hyderabad-based space tech startup Skyroot Aerospace successfully test fired an advanced fully 3D-
printed cryogenic rocket engine `Dhawan-II’ on April 04, 2023.
✓ Skyroot Aerospace is India’s first private rocket builder.
✓ The endurance test of ‘Dhawan-II’ was carried out at Solar Industries propulsion test facility in Nagpur,
Maharashtra. It lasted for a long duration of almost 200 seconds.
✓ The ‘Dhawan-II’ Cryogenic engine will power the upper stage of Vikram-2.
Q. According to the new joint report from the Institute for Energy Economics and Financial Analysis
(IEEFA) and JMK Research and Analytics, India could become the world’s second-largest solar
photovoltaic manufacturer by which year?
A) 2025 B) 2026
C) 2030 D) 2035
Answer : B
✓ According to the new joint report from the Institute for Energy Economics and Financial Analysis
(IEEFA) and JMK Research and Analytics, India could become the world’s second-largest solar
photovoltaic manufacturer by 2026, after China.
✓ India’s cumulative module manufacturing nameplate capacity more than doubled from 18GW in March
2022 to 38GW in March 2023.
✓ In terms of upcoming PV manufacturing installations, Gujarat is the leading state in India. It accounts
for nearly 57% of all the upcoming PV manufacturing capacity.
Q. Which state government has recently launched Cool Roof Policy 2023-28?
A) Kerala B) Gujarat
C) Telangana D) Tamil Nadu
Answer : C
✓ Municipal Administration and Urban Development Minister K T Rama Rao launched the Telangana
Cool Roof Policy 2023-28 to make the state more thermally comfortable and heat-resilient by reducing
the impact of the urban heat island effect and heat stress and also to bring down energy consumption.
✓ A cool roof, which uses special paints or tile cover, is designed to reflect more sunlight than a
conventional roof, thus reducing heat retention and cooling indoor spaces.
✓ Telangana Cool Roof Policy 2023-28 suggests the adoption of a cool roof as a measure for
implementation to build resilience against extreme heat.
✓ Telangana became the first state to introduce a cool roof policy, having incorporated it in building
permission applications.
✓ Cool roofs are now mandatory for all government as well as commercial buildings irrespective of their
site area or built-up area
Q. Which company has signed a deal with Russia’s largest oil producer Rosneft to substantially increase oil
supplies and diversify oil grades delivered to India?

Follow us: Official Site, Telegram, Facebook, Instagram, Instamojo 817


A) Bharat Heavy Electricals Limited (BHEL)
B) Bharat Petroleum Corporation Limited (BPCL)
C) Indian Oil Corporation Limited (IOCL)
D) Hindustan Petroleum Corporation Limited (HPCL)
Answer : C
✓ Russia’s largest oil producer Rosneft and India’s top refiner Indian Oil Corp have signed a term
agreement to substantially increase oil supplies and diversify oil grades delivered to India.
✓ The parties also discussed ways of expanding cooperation between Rosneft Oil Company and Indian
companies in the entire value chain of the energy sector, including possibilities of making payments in
national currencies.
✓ India has been the biggest buyer of Russia’s benchmark Urals grade crude in March. Deliveries to India
are set to account for more than 50% of all seaborne Urals exports this month, with China in second
place.
✓ Russia for the first time has become one of the five largest trading partners of India as the volume of trade
between the countries reached $38.4 billion in 2022.
✓ Indian Oil Corporation Limited (IOCL)
✓ Founded : 1959
✓ Headquarters : New Delhi
✓ Chairman : Shrikant Madhav Vaidya
✓ Owner : Government of India (52.1%)
Q. How many Indians have been inducted in the new cohort of the honorary life membership by Marylebone
Cricket Club (MCC) in 2023?
A) 3 B) 4
C) 5 D) 7
Answer : C
✓ The Marylebone Cricket Club (MCC) has released the list of newly inducted cricket players with the
honorary life membership of the club.
✓ Overall 17 former players from all across the world have been listed in the latest cohort released on April
05, 2023.
✓ MCC, the owner of the famous Lord’s cricket stadium in England, gives honorary membership to the
finest men and women who have contributed to the game.
✓ Five Indian players are among 17 new members in the list.
✓ They are Mahendra Singh Dhoni, Yuvraj Singh, Suresh Raina, Mithali Raj and Jhulan Goswami.
Q. India has been elected to the UN Statistical Commission for a period of four-year from January 2024.
How many countries are elected as the member of the commission?
A) 35 B) 12
C) 20 D) 24
Answer : D
✓ India has been elected to the UN Statistical Commission for a period of four-year.
✓ India won the election by securing 46 out of 53 votes.
✓ India’s term will begin from January 01, 2024.

Follow us: Official Site, Telegram, Facebook, Instagram, Instamojo 818


✓ India has been elected as a member of the UN Statistical Commission, the Commission on Narcotic
Drugs and the Programme Coordinating Board of the Joint UN Programme on HIV/AIDS (UNAIDS)
by the Economic and Social Council (ECOSOC) of the United Nations.
✓ About UNSC
✓ The UN Statistical Commission (UNSC) is the highest statistical body of the United Nations.
✓ The commission has 24 member countries of the United Nations.
✓ The term of office of members is four years.

Q. Who has been appointed as the President of UEFA football association for 4 years till 2027?
A) Paolo Maldini B) Aleksander Ceferin
C) Francisco Gento D) Karim Benzema
Answer : B
✓ The Slovenian football administrator Aleksander Ceferin has been re-elected as the president of UEFA
(Union of European Football Associations).
✓ He was elected unopposed at the Ordinary Congress on European Soccer’s governing body in Lisbon on
April 05, 2023
✓ Ceferin has been re-elected for a four-year term, until 2027.
✓ He was first was elected as UEFA’s seventh president in 2016, He replaced Michel Platini

Q. FIFA has withdrawn which country from hosting 2023 Men’s FIFA Under-17 World Cup?
A) Peru B) Indonesia
C) UAE D) France
Answer : A
✓ The football’s governing body FIFA has withdrawn Peru from hosting the 2023 Men’s FIFA Under-17
World Cup.
✓ As per FIFA, Peru was not ready to stage the 24-team tournament, which is scheduled to be held from
November 10 to December 02, 2023.
✓ FIFA World Cup 2026 : Canada/Mexico/USA ( 48 team)
✓ About Peru
✓ Peru is the third largest country in South America, after Brazil and Argentina.
✓ President : Dina Boluarte
✓ Capital : Lima
✓ Currency : Sol
✓ Major Mountain Ranges.: Andes
✓ Major Rivers : Amazon, Ucayali, Madre de Dios

Q. Which product of Varanasi has recently been given GI tag?


A) Banarasi Paan B) Banarasi Langda Mango
C) Both (A) & (B) D) None of these
Answer : C
✓ Banarasi paan has been awarded the GI tag, which signifies its unique identity and origin. Three other
foods from the Varanasi region, including Banarasi Langda Mango, Ramnagar Bhanta (Brinjal), and
Adamchini rice, have also received the GI tag.
✓ Banarasi paan is a popular mouth-freshener made from betel leaves, betel nut, slaked lime and many
other ingredients.

Follow us: Official Site, Telegram, Facebook, Instagram, Instamojo 819


✓ Banarasi paan is an important part of the cultural heritage of the city of Varanasi, having its origins in
the Mughal era.
Q. The Geographical Indications Registry has granted Geographical Indication (GI) tag to Nagari Dubraj,
fragrant rice to give a unique identity to the brand. Nagari Dubraj is the rice of which state?
A) Chhattisgarh B) Bihar
C) Assam D) Odisha
Answer : A
✓ The Geographical Indications Registry has granted Geographical Indication (GI) tag to Nagari Dubraj,
a fragrant rice from Chhattisgarh, to give a unique identity to the brand.
✓ The Morena and Rewa Mango (Madhya Pradesh) have also been given the GI Tag.
✓ The origin of Dubraj is believed to be from Shringi Rishi Ashram area of Sihawa. Its reference is found
in Valmiki Ramayana. Sihawa area is considered to be the source of Dubraj in various research papers.

Q. Who is the author of the book titled "Gandhi: Siasat Aur Sampradaya" ?
A) Rajnish Srivastava B) Piyush Babele
C) Abhishek Singhvi D) Nagendra Yadav
Answer : B
✓ A new Hindi book titled Gandhi: Siasat Aur Communalism has been authored by journalist-turned-
author Piyush Babele, who currently heads the media department of the Madhya Pradesh Congress.
✓ The book quotes from Dr. Ambedkar's book Pakistan or the Partition of India and other sources to refer
to the events leading up to the partition of India in 1947.
✓ The book aims to break the myth spread by the Hindu right wing that Mahatma Gandhi was responsible
for the partition.
✓ The book is published by New Delhi based Genuine Publications & Media Private Limited.

Q. Which tiger reserve completed 50 years as Project Tiger Reserve on 1st April 2023?
A) Kanha Tiger Reserve B) Anaimalai Tiger Reserve
C) Bandipur Tiger Reserve D) Nagarahole Tiger Reserve
Answer : C
✓ Bandipur completed 50 years as a Project Tiger Reserve 1 April 2023.
✓ About Bandipur Tiger Reserve
✓ This reserve is located in the state of Karnataka and is spread over an area of 912.04 sq km.
✓ It is located in two districts of Karnataka, Mysore and Chamarajanagar.
✓ It is considered one of the world's premier tiger habitats and is an important component of the country's
first biosphere reserve - the Nilgiri Biosphere Reserve.
✓ It was launched by the then Prime Minister Indira Gandhi in 1973 with the aim of arresting the decline
in the tiger population.
✓ Bandipur was one of the first nine sanctuaries to be brought under the flagship program of Project Tiger
in 1973.
✓ Bandipur Tiger Reserve is surrounded by-
✓ Nagarahole Tiger Reserve (Tamil Nadu) in the North West (Kabini Reservoir separates the two).
✓ Mudumalai Tiger Reserve (Tamil Nadu) in the South.
✓ Wayanad Wildlife Sanctuary (Kerala) in the South West.
✓ Project Tiger

Follow us: Official Site, Telegram, Facebook, Instagram, Instamojo 820


✓ Project Tiger was started by the Union Ministry of Environment, Forest and Climate Change in the year
1973.
✓ Under this program, central assistance is provided to the states having tiger populations for conservation
of tigers.
✓ At the time of the launch of Project Tiger in the year 1973, there were only 9 Tiger Reserves in the
country, at present the total number of Tiger Reserves in the country has increased to 54.
✓ Other Tiger Reserves in Karnataka
✓ Bhadra Tiger Reserve
✓ Nagarahole Tiger Reserve
✓ Dandeli-Anshi Tiger Reserve

Q. Nevado del Ruiz Volcano, which was recently seen in the news, is located in which country?
A) Colombia B) Tanzania
C) Switzerland D) Netherlands
Answer : A
✓ Recently, People living on the upper slopes of the Nevado del Ruiz volcano in Colombia are being
evacuated after an increase in seismic activity.
✓ Nevado del Ruiz is an ice-clad stratovolcano located in the Andes Mountains of Colombia, South
America.
✓ It is part of the former Ruiz-Tolima volcanic complex as well as the dangerous Pacific Ring of fire.

Q. The Indian government has announceda plan to add_ of renewable energy capacity in the next five years.
A) 250 GW B) 300 GW
C) 350 GW D) 400 GW
Answer : A
✓ The government announced a plan to add 250 GW of renewable energy capacity in the next five years
to achieve its target of 500 GW of clean energy by 2030.
✓ The government has decided to invite bids for 50 GW of renewable energy capacity annually for the next
five years, i.e., from fiscal 2023–24 to fiscal 2027–28.
Q. Which of the following has successfully conducted field firing trials of its latest third generation man-
portable Anti Tank Guided Missile (ATGM), Amogha-III?
A) Bharat Electronics Ltd (BEL) B) Hindustan Aeronautics Ltd (HAL)
C) Bharat Dynamics Ltd (BDL) D) Antrix Corporation Ltd (ACL)
Answer : C
✓ Bharat Dynamics (BDL) has successfully conducted a field firing test of its latest 3rd generation man-
portable Anti Tank Guided Missile (ATGM), Amogha-III.
✓ The Amogha-III Missile has been developed indigenously under Integrated Guided Missile Development
Programme (IGMDP).
✓ It is a third generation fire-and-forget Anti-Tank Guided Missile.
✓ Developed by the Research and Development Division of BDL, the missile also has a dual-mode IIR
seeker with a range of 200 to 2500 metres.
✓ About Anti-tank Guided Missile (ATGM)
✓ An anti-tank guided missile is a guided missile designed primarily to hit and destroy heavily armored
military vehicles.
✓ These missiles can be carried by a single soldier as a large tripod-mounted weapon.

Follow us: Official Site, Telegram, Facebook, Instagram, Instamojo 821


Q. Who is the author of the book titled "War and Women"?
A) M A Hasan B) Dilip Singh
C) Om prakash Iyer D) Anshuman Dubey
Answer : A
✓ Dr. M A Hasan's book, "War and Women," was launched during the 52nd session of the United Nations
Human Rights Council (UNHRC) in Geneva, Switzerland.
✓ The event was organised by the Bangladesh Freedom Fighters Parliament in Europe.
✓ The book was released at the Serpentine Cafeteria inside the United Nations Building.
✓ The book throws light on the suffering of Bengali women who were victims of sexual violence during the
1971 war.
Q. Which city in India has been ranked among the world’s 19 best cities for public transport, as per the report
by Time Out?
A) Mumbai B) Chennai
C) Bengaluru D) Hyderabad
Answer : A
✓ India’s financial capital Mumbai is the only Indian city among the list of the world’s 19 best cities for
public transport, as per the report by Time Out.
✓ Mumbai claimed the last spot, that means 19th spot on the list for great public transit, due to its wide-
reaching suburban rail network.
✓ More than 20,000 people in 50 cities across the world were surveyed for the rankings.
✓ The Time Out’s world’s best city for public transport went to Berlin in Germany.
✓ Second position went to Prague in Czech Republic while Tokyo in Japan ranked third
Q. Name the city which has been ranked as the world’s best city for public transport by Time Out?
A) London B) Oslo
C) Paris D) Berlin
Answer : D
✓ List of world’s best 19 cities for public transport
1. Berlin, Germany 2. Prague, Czech Republic
3. Tokyo, Japan 4. Copenhagen, Denmark
5. Stockholm, Sweden 6. Singapore
7. Hong Kong 8. Taipei, Taiwan
9. Shanghai, China 10. Amsterdam, Netherlands
11. London, UK 12. Madrid, Spain
13. Edinburgh, UK 14. Paris, France
15. New York City, US 16. Montreal, Canada
17. Chicago, US 18. Beijing, China
19. Mumbai, India

Follow us: Official Site, Telegram, Facebook, Instagram, Instamojo 822


Q. Which among the given statements are true regarding the Indian Space Policy 2023 approved by the
Cabinet?
I. Focus of the policy is to increase private sector participation in the space sector
II. ISRO shall focus on research and development of advanced space technologies and not do any
operational and production work
III. NSIL to carry out strategic activities related to the space sector
A) Only I & II B) Only II & III
C) Only I & III D) All I, II & III are correct
Answer : D
✓ The Cabinet Committee on Security chaired by Prime Minister Narendra Modi has approved the Indian
Space Policy 2023 on April 06, 2023.
✓ The policy seeks to institutionalise the private sector participation in the space sector, and channelise the
focus of ISRO on research and development of advanced space technologies.
✓ The private sector will be able to take part in end-to-end space activities including building satellites,
rockets and launch vehicles, data collection and dissemination.
✓ The roles and responsibilities of ISRO, space sector PSU NewSpace India Limited (NSIL) and Indian
National Space Promotion and Authorization Center (IN-SPACe) have also been specified in the new
policy.
✓ AS per the policy, ISRO will not do any operational and production work for the space sector. It shall
focus on developing new technologies, new systems and research and development.
✓ The strategic activities related to the space sector will be carried out by NSIL.
✓ The INSPACe will be the interface between ISRO and non-governmental entities.
✓ India's progress in the field of space exploration.
✓ India has been making significant progress in the field of space exploration in recent years, with the
successful launch of the Chandrayaan-2 mission and the development of the Gaganyaan mission, which
aims to send astronauts into space.
✓ The country is also working on developing its own satellite navigation system, the Indian Regional
Navigation Satellite System (IRNSS).
Q. Which state’s Mircha rice has recently received the Geographical Indication (GI) Tag?
A) Assam B) Odisha
C) Bihar D) Jharkhand
Answer : C
✓ ‘Mircha’ rice of Bihar’s West Champaran has been awarded the GI tag.
✓ The size and shape of the grain appear like that of black pepper, hence it is known as Mircha or Marcha
Rice.
✓ The grains and flakes of this rice have a unique aroma that makes it different.
✓ This rice is famous for its aroma, palatability and its aromatic chura (rice flakes) making qualities.
✓ An application for GI tag was submitted on behalf of Marcha Dhan Utpadak Pragatisheel Samuh, a
registered organization of paddy cultivators.
✓ There are some important GI tags in Bihar.
✓ Madhubani Paintings ( 2007)
✓ Shahi Litchi ( 2018)
✓ Magahi Paan (2018)
✓ Silao Khaja (2018)
✓ Katarni Rice (2018)

Follow us: Official Site, Telegram, Facebook, Instagram, Instamojo 823


✓ Mithila Makhana ( 2022)
✓ Mircha rice (2023).
✓ GEOGRAPHICAL INDICATIONS (GI) :-
✓ GI tags are issued as per the Geographical Indications of Goods (Registration and Protection) Act,1999.
This tag is issued by the Geographical Indication Registry under the Department of Industry Promotion
and Internal Trade, Ministry of Commerce and Industry.
✓ Darjeeling Tea was the first Indian product to get the geographical indication tag in 2004.
✓ The second GI tag in India was given to Aranmula Kannadi (Handicraft) which is a mirror and made in
Kerala.
✓ A GI is registered for an initial period of ten years, which must be renewed after 10 years.
✓ Kerala has topped among Indian states for having most number of products securing the Geographical
Indication (GI) tag in financial year 2022-23.
✓ What is GI tag?
✓ A GI is a sign used on products that have a specific geographical origin and possess qualities or a
reputation that are due to that origin.
Q. Ukraine’s president Volodymyr Zelensky has been awarded the Order of the White Eagle for his services
to security, resilience and the defense of human rights. Order of the White Eagle is the highest honour of
which country?
A) Sweden B) Poland
C) Finland D) Norway
Answer : B
✓ Ukraine's President Volodymyr Zelensky was awarded the Order of the White Eagle, Poland's highest
award.
✓ The award was given to him for his exceptional services to protect security, resilience and human rights.
✓ The Order of the White Eagle is a prestigious and highly regarded decoration of the Republic of Poland,
awarded for outstanding civil and military contributions to the country.
✓ It is considered the oldest and highest decoration in Poland, whose history dates back to the 14th century.
✓ Poland is also a member of several international organisations including the United Nations, NATO,
and the World Trade Organization.

VOLODYMYR ZELENSKY IN NEWS 2023


▪ Ukraine's President Volodymyr Zelensky was awarded the Order of the White Eagle, Poland's highest
award.
▪ Time Magazine has named Ukrainian president Volodymyr Zelensky and "the spirit of Ukraine" as its
2022 Person of the Year.
▪ The Liberty Medal 2022 has been awarded to the President of Ukraine, Volodymyr Zelensky.
▪ UKRAINE .
▪ Ukraine, a country located in Eastern Europe, is the second largest country in the continen after Russia.
▪ Its capital is Kiev, located on the Dnieper River in north-central Ukraine.
▪ Prime Minister: Denys Shmyhal
▪ President: Volodymyr Zelensky
▪ Capital: Kyiv
▪ Currency : Rryvnycontine

Q. Who has unveiled a 54-feet-tall statue of Lord Hanuman at a temple in Gujarat, India?
A) Amit Shah B) Yogi Adityanath
C) J. P. Nadda D) Droupadi Murmu

Follow us: Official Site, Telegram, Facebook, Instagram, Instamojo 824


Answer : A
✓ Union Home Minister Amit Shah unveiled a 54 feet tall statue of Lord Hanuman at Sarangpur temple in
Botad district of Gujarat.
✓ The statue was unveiled on the occasion of Hanuman Jayanti, which is celebrated to mark the birth of
Lord Hanuman.
✓ The idol is located in the Sarangpur Temple, a popular pilgrimage site for devotees of Lord Hanuman.
✓ The idol is 54 feet tall, making it one of the tallest idols of Lord Hanuman in the country.
✓ Apart from unveiling the statue, Amit Shah also inaugurated the temple's new community kitchen on
April 6, 2023.

Q. India has strongly criticized the Organisation of Islamic Cooperation (OIC) for its "__" agenda.
A) anti-terror B) anti-drug
C) Anti-Doping D) anti-India
Answer : D
✓ India on 4 April slammed the Organisation of Islamic Cooperation (OIC) for its “communal mindset”
and “anti-India” agenda.
✓ India's strong reaction came after the OIC secretariat issued a statement alleging the targeting of the
Muslim community during Ram Navami processions in several Indian states.
✓ Earlier, India criticized the OIC for having a "communal mindset" amid the Karnataka hijab controversy.
✓ About Organisation of Islamic Cooperation (OIC)
✓ It is the second largest multilateral body in the world after the United Nations.
✓ It was established by the First Islamic Summit held in Morocco in September 1969.
✓ Its objective is to protect and safeguard the interests of the Muslim world in the spirit of promoting
international peace and harmony.
✓ Currently it has 57 member countries, all of them are Islamic countries or Muslim majority members.
✓ Headquarter: Jeddah, Saudi Arabia
✓ India, which has the third largest muslim population in the world after Indonesia and Pakistan, is not a
member of OIC.
✓ The 49th session of the Council of Foreign Ministers was held on 16 - 17 March 2023 in Nouakchott,
Islamic Republic of Mauritania
Q. Aleksandar Ceferin was re-elected as the President of the Union of European Football Associations
(UEFA) till 2027. He is in which country?
A) Slovenian B) Norway
C) Sweden D) Portugal
Answer : A
✓ Aleksandar Ceferin was re-elected as the President of the Union of European Football Associations
(UEFA).
✓ It was re-elected during UEFA's Ordinary Congress held in Lisbon, Portugal, and Ceferin was elected
unopposed, meaning that there were no other candidates for the position.
✓ He was first time elected as the seventh president of UEFA in 2016, succeeding Michel Platini.
✓ Ceferin's new term will last for another four years (2027).
✓ The Union of European Football Associations (UEFA) is responsible for governing association football,
futsal and beach soccer in Europe.

✓ UEFA.

Follow us: Official Site, Telegram, Facebook, Instagram, Instamojo 825


✓ Establishment - 15 June 1954
✓ Headquarters - Nyon, Switzerland
✓ Speaker - Aleksander Ceferin
✓ Parent organisation - FIFA
Q. President Droupadi Murmu has inaugurated a two-day 'Gaj Utsav-2023' on April 7 at the Kaziranga
National Park in Assam. Kaziranga National Park is famous for ____
A) One-horned Rhinos B) Tigers
C) Swamp Dears (Barasingha) D) Elephants
Answer : A
✓ President Droupadi Murmu inaugurated a two-day 'Gaj Utsav-2023' on April 7 at the Kaziranga National
Park High School’s playground field in Assam.
✓ Meanwhile, President Murmu also enjoyed cultural performances including Bhortal, Jhumur and Bihu
dance forms by Assamese artistes at Kohora.
✓ The President also flag off the ‘Mount Kanchenjunga Expedition-2023’.
✓ President Murmu attended the Platinum Jubilee celebrations of the Gauhati High Court as the chief guest
and also launched a mobile app "Bhoroxa" designed for the safety of women and the elderly
✓ About Gaj Utsav
✓ Kaziranga Gaja Utsav is an annual festival held in the national park to promote the conservation and
protection of elephants.
✓ It is jointly organized by the Forest and Tourism Departments to draw attention and find a solution to
the growing human-elephant onflict in the state.
✓ This is being organized to mark the completion of 30 years of Project Elephant.
✓ The Government of India started Project Elephant in the year 1992 to protect elephants, their migration
routes and natural habitats.
✓ Assam has the second largest population of elephants in the country after Karnataka.
✓ Kaziranga National Park being a UNESCO World Heritage Site is an ideal place for Gaja Utsav.
✓ Kaziranga National Park is located at Assam, India.
✓ Designated with National Park status in 1968.
✓ Declared a UNESCO World Heritage Site in 1985.
✓ Kaziranga National Park hosts two-thirds of the world's great one-horned rhinoceroses.

Q. The joint military exercise KAVACH was conducted by which united of Indian Armed Force?
A) Southern Naval Command B) Andaman and Nicobar Command
C) Western Naval Command D) Eastern Naval Command
Answer : B
✓ The Andaman and Nicobar Command (ANC) conducted a large-scale joint military exercise 'X Kavach'
involving assets of the Army, Navy, Air Force and Coast Guard
✓ Commencing on 23 February 2023, the exercise concluded on 7 April 2023.
✓ The exercise was aimed at fine-tuning joint combat capabilities and Standard Operating Procedures
(SOPs) and enhancing interoperability and operational synergy between the armies.
✓ Army's 'Shatrujit Brigade', Armed Forces Special Operations Division (AFSOD), Navy's Special Forces
and ANC's amphibious troops carried out an exercise at a remote island in the Andaman and Nicobar
Islands.
✓ The exercise featured multi-domain exercises involving amphibious landings, airborne operations,
heliborne operations and rapid entry of special forces off the coast.
✓ 'X Kavach' showcased the capabilities and preparedness of the Armed Forces to safeguard India's
maritime interests and ensure the security of the Andaman and Nicobar Islands.

Follow us: Official Site, Telegram, Facebook, Instagram, Instamojo 826


Q. Who has been appointed as the new Vice Chief of Naval Staff wef April 2023?
A) Kailash K. Kohli B) Nirmal Kumar Verma
C) Robin K. Dhowan D) Sanjay Jasjit Singh
Answer : D
✓ Vice Admiral Sanjay Jasjit Singh took over as the new Vice Chief of Naval Staff on April 02, 2023.
✓ He succeeds Vice Admiral Satishkumar Namdeo Ghormade who superannuated on April 01, 2023 after
more than 39 years of service to the Indian Navy.
✓ Prior to this, Vice Admiral Sanjay Jasjit Singh held charge of Deputy Chief of Integrated Defence Staff
✓ Chiefs of Indian Services
✓ Chief of Defense Staff: Lt. Gen. Anil Chauhan (Retd.)
✓ Army: General Manoj Pandey
✓ Navy: Admiral Radhakrishnan Hari Kumar
✓ Air Force: Air Marshal VR Chowdhary
✓ Air Marshal Amanpreet Singh has been appointed as the new Vice Chief of the Air Staff
✓ Lt. Gen. M.V. Suchindra Kumar took over as Vice Chief of the Army Staff.
✓ Vice Admiral Sanjay Jasjit Singh took over as the new Vice Chief of Naval Staff on April 02, 2023
✓ CDS is a four-star General

Q. Which country has assumed the Presidency of the UN Security Council (UNSC) in April 2023?
A) United Kingdom
B) Russia
C) United States
D) France
Answer : B
✓ Russia has taken over the Presidency of the UN Security Council (UNSC) on April 01, 2023. It will be
presided by Russian President Vladimir Putin.
✓ It must be informed that each of the 15 members of the UNSC takes up the presidency for a month, on a
rotating basis.
✓ Previously, Russia took over the Presidency in February 2022. During that time, Russia launched a full-
scale invasion of Ukraine.
✓ The United Nations Security Council (UNSC) is one of the six principal organs of the United Nations
(UN) and is responsible for maintaining international peace and security.
✓ UNSC has 15 member countries. Among them, 5 are permanent while other 10 are non-permanent
members.
✓ The five permanent members of UNSC are China, France, Russia, United Kingdom and United States.
✓ The non-permanent members are Albania, Brazil, Ecuador, Gabon, Ghana, Japan, Malta, Mozambique,
Switzerland, and United Arab Emirates.
✓ United Nations Security Council (UNSC)
✓ Formation : 24 October 1945
✓ Headquarteres : New York, US
✓ President for August, 2021
✓ Ruchira Kamboj, India's Permanent Representative to the UN.
✓ UNSC has 15 members of which 5 are permanent and 10 are non-permanent members.
✓ Non-permanent members elected for two-year terms by the UN General Assembly.
✓ Five permanent members are- US, UK, France, Russia, and China.

Follow us: Official Site, Telegram, Facebook, Instagram, Instamojo 827


Q. The Government of India has set the vision to become a one trillion dollar tourism economy by which
year?
A) 2023 B) 2047
C) 2030 D) 2045
Answer : B
✓ India aims to become one trillion dollar tourism economy by 2047.
✓ The Government of India has set the vision to make India a one trillion dollar tourism economy by the
year 2047, which marks India’s 100th year of Independence.
✓ In line with this, India is observing ‘Visit India Year 2023′ in order to give a boost to the uniqueness of
Indian tourism and make India a seamless , modern, and smart tourism destination.
✓ The ‘Visit India Year 2023’ is a unique, collective movement that invites the world to explore India in
2023, the historical year of India’s G20 presidency.
✓ The year 2023 also marks the grand celebrations of 75 years of Indian Independence – India@75 Azadi
ka Amrit Mahotsav
Q. India has become the first country to develop a country-level mathematical model to estimate the
prevalence of cases of which disease?
A) Alzheimer’s disease B) Hypertension
C) Tuberculosis D) Cirrhosis
Answer : C
✓ India has become the first country in the world to develop a country-level mathematical model to estimate
the prevalence of tuberculosis (TB) cases in India.
✓ Using this model, the data regarding TB incidence and mortality estimate for India will be available by
March every year.
✓ The new model has been created based on the natural history of the disease, individual cases of infection,
healthcare-seeking behaviour, missed or correct diagnosis, treatment coverage, and outcome.
✓ Government Initiatives to Combat TB.
✓ National Strategic Plan (NSP) for Tuberculosis (TB) Elimination (2017-2025)
✓ Nikshay Poshan Yojana (NPY- financial support)
✓ TB Harega Desh Jeetega Campaign.
✓ Universal Immunisation Programme.
✓ The government of India aims to have a TB-free India by 2025.
✓ Revised National TB Control Program under National Health Mission.
✓ Ending the TB epidemic by 2030 under United Nations SDG target 3.3
Q. World Health Day is observed annually on which day?
A) April 04 B) April 05
C) April 06 D) April 07
Answer : D
✓ The World Health Day is celebrated every year on April 7 since 1950.
✓ The day commemorates the establishment of World Health Organisation (WHO) on 7 April 1948.
✓ World Health Day is a global health awareness day to draw worldwide attention to a specific subject of
major importance to global health.
✓ The Theme of 2023 World Health Day is ‘Health For All.’
✓ On 7 April 2023 World Health Day the World Health Organization will observe its 75th birthday.

Follow us: Official Site, Telegram, Facebook, Instagram, Instamojo 828


Q. What is the theme for the 2023 World Health Day?
A) Working together for health B) Health for all
C) Move for health D) Healthy Cities for better life
Answer : B
✓ The Theme of 2023 World Health Day is ‘Health For All.’
Q. The International Big Cats Alliance (IBCA) has been launched by the Government of India for the
conservation of how many species of big cats?
A) 5 B) 7
C) 3 D) 9
Answer : B
✓ Prime Minister Narendra Modi inaugurated the program ‘Commemoration of 50 years of Project Tiger’
on April 09, 2023 at Mysuru University in, Karnataka.
✓ The year 2023 marks 50 years of the Project Tiger.
✓ The Project Tiger was launched in November 1973 as tiger conservation programme by the then Prime
Minister Indira Gandhi.
✓ International Big Cats Alliance (IBCA)
✓ PM Modi launched the International Big Cats Alliance (IBCA) in Karnataka.
✓ IBCA aims to focus on the protection and conservation of seven major big cats of the world, in
collaboration with countries harbouring these species.
✓ These include Tiger, Lion, Leopard, Snow Leopard, Puma, Jaguar and Cheetah.
✓ The IBCA will provide assured support over five years with guaranteed funding of over Rs 800 crore.
✓ The alliance membership will be open to 97 “range” countries, which contain the natural habitat of these
big cats
✓ Other launches that took place at the event includes:
✓ Release of the publications – ‘Amrit Kaal Ka Vision For Tiger Conservation’, a summary report of the
5th cycle of Management Effectiveness Evaluation of Tiger Reserves.
✓ Release of the summary report of All India Tiger Estimation (5th cycle).
✓ Release of a commemorative coin on the completion of 50 years of Project Tiger.
✓ Other facts
✓ India is home to 75% of the world’s tiger population in the 75th year of Indian independence.
✓ The tiger reserves in India cover 75,000 square kilometers of land
✓ In the past 10-12 years, the tiger population in the country has increased by 75 percent
Q. How much amount has been contributed by India to the United Nations Trust Fund in support of the
African Union Transition Mission in Somalia (ATMIS)?
A) US$ 3 million B) US$ 4 million
C) US$ 5 million D) US$ 2 million
Answer : D
✓ India has contributed USD 2 million to the United Nations Trust Fund in support of the African Union
Transition Mission in Somalia (ATMIS).
✓ The cheque for the same was handed over by India’s Permanent Representative to the United Nations
Ruchira Kamboj.
✓ ATMIS was established on 1 April 2022, to reduce the threat posed by the terrorist group Al-Shabaad
and to support the implementation of the Somali Transitional Plan towards a full handover of security
responsibility to the Somali security forces and institutions by the end 2024.

Follow us: Official Site, Telegram, Facebook, Instagram, Instamojo 829


✓ Earlier, India had contributed USD 4 million to African Union Mission in Somalia (AMISOM).
Q. Chinese startup company Space Pioneer recently launched first ever liquid fueled rocket Sky Dragon-2
into orbit. What is the native name of this rocket?
A) Chang Zheng-2 B) Wenchan-2
C) Dong Feng-2 D) Tianlong-2
Answer : D
✓ The Chinese space startup ‘Space Pioneer’ successfully launched ‘Tianlong-2 rocket” into orbit from
China’s Jiuquan Satellite Launch Center located in the Gobi Desert, Inner Mongolia.
✓ This for the first time that a liquid fueled rocket was launched into orbit by a Chinese aerospace company.
✓ Also, it was the first time that a startup company (Space Pioneer- also known as Beijing Tianbing
Technology Co.) successfully reached orbit on its first attempt.
✓ The Tianlong-2 rocket is also known as “Sky Dragon-2.
Q. Name the Indian badminton player who has recently won the men’s single title at the 2023 Orleans
Masters in France?
A) Priyanshu Rajawat B) Satwiksairaj Rankireddy
C) Manu Attri D) Subhankar Dey
Answer : A
✓ India’s emerging badminton star Priyanshu Rajawat defeated Magnus Johannesen of Denmark, 21-15,
19-21, 21-16, to win men’s single title at the 2023 Orleans Masters in France with a win over in the
decider.
✓ It was his maiden BWF World Tour Super 300 title as well as his first appearance in the final.
✓ The 21-year-old Priyanshu Rajawat from Madhya Pradesh was a part of the Indian team that registered
an epic win at the 2022 Thomas Cup.
✓ Winners of 2023 Orléans Masters
✓ Men’s Single – Priyanshu Rajawat (India)
✓ Women’s single – Carolina Marín (Spain)
Q. The Central Reserve Police Force (CRPF) Valour Day (Shaurya Diwas) is celebrated every year on April
09. In 2023 which edition of CRPF Valour Day was celebrated?
A) 55th B) 61st
C) 58th D) 68th
Answer : C
✓ The Central Reserve Police Force (CRPF) Valour Day (Shaurya Diwas) is observed on 9th April every
year, as a tribute to the brave men of the Force.
✓ 2023 marks 58th CRPF Valour Day.
✓ It was on this day in 1965, a small contingent of CRPF created history by defeating an invading Pakistani
army, several time larger, at the Sardar Post located in the Rann of Kutch, Gujarat.

Follow us: Official Site, Telegram, Facebook, Instagram, Instamojo 830

You might also like